Neurocritical Care Board Review

ZZakaria_87574_PTR_CH00_10-06-13_i-xx.inddakaria_87574_PTR_CH00_10-06-13_i-xx.indd i 66/19/2013/19/2013 8:43:228:43:22 PMPM

66485457-66485438 www.ketabpezeshki.com

ZZakaria_87574_PTR_CH00_10-06-13_i-xx.inddakaria_87574_PTR_CH00_10-06-13_i-xx.indd iiii 66/19/2013/19/2013 8:43:228:43:22 PMPM Neurocritical Care Board Review Questions and Answers

Editor

Asma Zakaria, MD Assistant Professor Departments of Neurology and Fellowship Director, Neurocritical Care Division of Neurocritical Care University of Texas, Health Science Center at Houston

New York

66485457-66485438 www.ketabpezeshki.com

ZZakaria_87574_PTR_CH00_10-06-13_i-xx.inddakaria_87574_PTR_CH00_10-06-13_i-xx.indd iiiiii 66/19/2013/19/2013 8:43:228:43:22 PMPM Visit our website at www.demosmedpub.com

ISBN: 978-1-936287-57-4 eISBN: 978-1-61705-033-6

Acquisitions Editor: Beth Barry Compositor: Newgen Imaging Systems, Ltd.

© 2014 Demos Medical Publishing, LLC. All rights reserved. This book is protected by copyright. No part of it may be reproduced, stored in a retrieval system, or transmitted in any form or by any means, electronic, mechanical, pho- tocopying, recording, or otherwise, without the prior written permission of the publisher.

Medicine is an ever-changing science. Research and clinical experience are continually expanding our knowledge, in particular our understanding of proper treatment and drug therapy. The authors, editors, and publisher have made every effort to ensure that all information in this book is in accordance with the state of knowledge at the time of production of the book. Nevertheless, the authors, editors, and publisher are not responsible for errors or omissions or for any consequences from application of the information in this book and make no warranty, express or implied, with respect to the contents of the publication. Every reader should examine carefully the package inserts accom- panying each drug and should carefully check whether the dosage schedules mentioned therein or the contraindi- cations stated by the manufacturer differ from the statements made in this book. Such examination is particularly important with drugs that are either rarely used or have been newly released on the market.

Library of Congress Cataloging-in-Publication Data Neurocritical care board review / [edited by] Asma Zakaria. p. ; cm. Includes bibliographical references and index. ISBN 978-1-936287-57-4 — ISBN 978-1-61705-033-6 (ebook) I. Zakaria, Asma, editor of compilation. [DNLM: 1. Nervous System Diseases—therapy—Examination Questions. 2. Critical Care—Examination Questions. WL 18.2] 616.02’8—dc23 2013001190

Special discounts on bulk quantities of Demos Medical Publishing books are available to corporations, professional associations, pharmaceutical companies, health care organizations, and other qualifying groups. For details, please contact: Special Sales Department Demos Medical Publishing 11 West 42nd Street, 15th Floor New York, NY 10036 Phone: 800-532-8663 or 212-683-0072 Fax: 212-941-7842 Email: [email protected]

Printed in the United States of America by Bradford and Bigelow. 13 14 15 16 17 / 5 4 3 2 1

66485457-66485438 www.ketabpezeshki.com

ZZakaria_87574_PTR_CH00_10-06-13_i-xx.inddakaria_87574_PTR_CH00_10-06-13_i-xx.indd iviv 66/19/2013/19/2013 8:43:228:43:22 PMPM To My Parents– For their support and patience

66485457-66485438 www.ketabpezeshki.com

ZZakaria_87574_PTR_CH00_10-06-13_i-xx.inddakaria_87574_PTR_CH00_10-06-13_i-xx.indd v 66/19/2013/19/2013 8:43:228:43:22 PMPM 66485457-66485438 www.ketabpezeshki.com

ZZakaria_87574_PTR_CH00_10-06-13_i-xx.inddakaria_87574_PTR_CH00_10-06-13_i-xx.indd vivi 66/19/2013/19/2013 8:43:228:43:22 PMPM Contents

Contributors xi Preface xvii

PART I: NEUROLOGIC DISEASE STATES: PATHOLOGY, PATHOPHYSIOLOGY, AND THERAPY

Section I: Cerebrovascular Diseases 1. Infarction, Ischemia, and Hemorrhage 3 John J. Volpi 2. Subarachnoid Hemorrhage and Vascular Malformations 19 Bülent Yapicilar and Asma Zakaria

Section II: Neurotrauma 3. Neurotrauma 35 Scott R. Shepard

Section III: Disorders, Diseases, Seizures, and Epilepsy 4. Seizures and Epilepsy 51 Grant M. Warmouth 5. Neuromuscular Medicine 71 Suur Biliciler, Justin Kwan, and Cecile L. Phan 6. CNS Infections 79 Doris Kung

vii

66485457-66485438 www.ketabpezeshki.com

ZZakaria_87574_PTR_CH00_10-06-13_i-xx.inddakaria_87574_PTR_CH00_10-06-13_i-xx.indd viivii 66/19/2013/19/2013 8:43:228:43:22 PMPM CONTENTS

7. Toxic–Metabolic Disorders 95 Elissa K. Fory 8. Infl ammatory and Demyelinating Diseases 107 Flavia Nelson 9. Neuroendocrine Disorders 121 Matthew Flaherty and Howard J. Fan 10. Neuro-Oncology 131 Yoshua Esquenazi and Nitin Tandon 11. Encephalopathies 141 Corey E. Goldsmith 12. Clinical Syndromes 159 Howard J. Fan and Asma Zakaria 13. Perioperative Neurosurgical Care 175 Yoshua Esquenazi and Nitin Tandon 14. Pharmacology and Practical Use of Medications in Neurocritical Care 183 Teresa A. Allison and Sophie Samuel

PART II: GENERAL CRITICAL CARE: PATHOLOGY, PATHOPHYSIOLOGY, AND THERAPY 15. Cardiovascular Physiology 205 David J. Powner 16. Cardiovascular Diseases 217 Jean Onwuchekwa Ekwenibe, Francisco Fuentes, Siddharth Mukerji, Husnu Evren Kaynak, Nicoleta Daraban, Charles Hebenstreit, and Ketan Koranne 17. Pulmonary Physiology and Fundamentals of Mechanical Ventilation 239 David J. Powner 18. Respiratory Diseases 267 Imoigele P. Aisiku 19. Renal Diseases 285 Ala Abudayyeh and Maen Abdelrahim 20. Electrolyte and Endocrine Disorders 299 David J. Powner 21. Infectious Diseases 319 Safdar A. Ansari

viii

66485457-66485438 www.ketabpezeshki.com

ZZakaria_87574_PTR_CH00_10-06-13_i-xx.inddakaria_87574_PTR_CH00_10-06-13_i-xx.indd viiiviii 66/19/2013/19/2013 8:43:228:43:22 PMPM CONTENTS

22. Acute Hematologic Disorders 333 Jitesh Kar and Hanh T. Truong 23. Acute Gastrointestinal and Genitourinary Disorders 349 Luis J. Garcia and Asad Latif 24. Diagnosis of Death 361 David J. Powner 25. General Trauma and Burns 371 Sasha D. Adams and Amy R. Alger 26. Ethical and Legal Aspects of Critical Care Medicine 383 Nasiya Ahmed 27. Principles of Research in Critical Care 391 Suur Biliciler and Justin Kwan 28. Procedural Skills and Monitoring 397 George W. Williams 29. Clinical Cases 415 Asma Zakaria and Bülent Yapicilar

Index 433

ix

66485457-66485438 www.ketabpezeshki.com

ZZakaria_87574_PTR_CH00_10-06-13_i-xx.inddakaria_87574_PTR_CH00_10-06-13_i-xx.indd ixix 66/19/2013/19/2013 8:43:228:43:22 PMPM 66485457-66485438 www.ketabpezeshki.com

ZZakaria_87574_PTR_CH00_10-06-13_i-xx.inddakaria_87574_PTR_CH00_10-06-13_i-xx.indd x 66/19/2013/19/2013 8:43:228:43:22 PMPM Contributors

Maen Abdelrahim, MD, PhD Resident Department of Internal Medicine Baylor College of Medicine Houston, Texas

Ala Abudayyeh, MD Assistant Professor Department of General Internal Medicine Division of Nephrology University of Texas, MD Anderson Cancer Center Houston, Texas

Sasha D. Adams, MD Assistant Professor Department of Surgery Division of Trauma and Critical Care University of North Carolina, School of Medicine Chapel Hill, North Carolina

Nasiya Ahmed, MD Assistant Professor Department of Internal Medicine Division of Geriatric and Palliative Medicine University of Texas, Health Science Center at Houston Houston, Texas xi

66485457-66485438 www.ketabpezeshki.com

ZZakaria_87574_PTR_CH00_10-06-13_i-xx.inddakaria_87574_PTR_CH00_10-06-13_i-xx.indd xixi 66/19/2013/19/2013 8:43:228:43:22 PMPM CONTRIBUTORS

Imoigele P. Aisiku, MD Associate Professor Department of Neurosurgery Division of Neurocritical Care University of Texas, Health Science Center at Houston Houston, Texas

Amy R. Alger, MD, FACS Assistant Professor Department of Surgery Division of Trauma and Critical Care University of North Carolina, School of Medicine Chapel Hill, North Carolina

Teresa A. Allison, PharmD, BCPS Neurosciences Clinical Pharmacy Specialist Memorial Hermann–Texas Medical Center Houston, Texas

Safdar A. Ansari, MD Assistant Professor Department of Neurosurgery Division of Neurocritical Care University of Utah Salt Lake City, Utah

Suur Biliciler, MD Assistant Professor Department of Neurology Neuromuscular Diseases Section University of Texas, Health Science Center at Houston Houston, Texas

Nicoleta Daraban, MD Chief Cardiovascular Fellow Department of Internal Medicine University of Texas, Health Science Center at Houston Houston, Texas

Jean Onwuchekwa Ekwenibe, MD Cardiovascular Fellow Department of Internal Medicine University of Texas, Health Science Center at Houston Houston, Texas

xii

66485457-66485438 www.ketabpezeshki.com

ZZakaria_87574_PTR_CH00_10-06-13_i-xx.inddakaria_87574_PTR_CH00_10-06-13_i-xx.indd xiixii 66/19/2013/19/2013 8:43:228:43:22 PMPM CONTRIBUTORS

Yoshua Esquenazi, MD Chief Resident Department of Neurosurgery University of Texas, Health Science Center at Houston Houston, Texas

Howard J. Fan, MD Neurocritical Care Fellow Department of Neurosurgery Ronald Reagan UCLA Medical Center Los Angeles, California

Matthew Flaherty, MD Neurocritical Care Fellow Department of Neurosurgery University of Texas, Health Science Center at Houston Houston, Texas

Elissa K. Fory, MD Department of Neurosciences Winthrop University Hospital Mineola, New York

Francisco Fuentes, MD Professor Department of Internal Medicine Division of Cardiology University of Texas, Health Science Center at Houston Houston, Texas

Luis J. Garcia, MD Department of Surgery Division of Acute Care Surgery Johns Hopkins University School of Medicine Baltimore, Maryland

Corey E. Goldsmith, MD Assistant Professor Department of Neurology Baylor College of Medicine Houston, Texas

xiii

66485457-66485438 www.ketabpezeshki.com

ZZakaria_87574_PTR_CH00_10-06-13_i-xx.inddakaria_87574_PTR_CH00_10-06-13_i-xx.indd xiiixiii 66/19/2013/19/2013 8:43:238:43:23 PMPM CONTRIBUTORS

Charles Hebenstreit, MD Chief Resident Department of Internal Medicine University of Texas, Health Science Center at Houston Houston, Texas

Jitesh K. Kar, MD, MPH Chief Resident Department of Neurology University of Texas, Health Science Center at Houston Houston, Texas

Husnu Evren Kaynak, MD Cardiovascular Fellow Department of Internal Medicine University of Texas, Health Science Center at Houston Houston, Texas

Ketan Koranne, MD Resident Department of Internal Medicine University of Texas, Health Science Center at Houston Houston, Texas

Doris H. Kung, DO Assistant Professor Department of Neurology Baylor College of Medicine Houston, Texas

Justin Kwan, MD Assistant Professor Department of Neurology University of Maryland School of Medicine Baltimore, Maryland

Asad Latif, MD, MPH Assistant Professor Department of Anesthesiology and Critical Care Medicine Division of Adult Critical Care Medicine Johns Hopkins University School of Medicine Baltimore, Maryland

xiv

66485457-66485438 www.ketabpezeshki.com

ZZakaria_87574_PTR_CH00_10-06-13_i-xx.inddakaria_87574_PTR_CH00_10-06-13_i-xx.indd xivxiv 66/19/2013/19/2013 8:43:238:43:23 PMPM CONTRIBUTORS

Siddharth Mukerji, MD Cardiovascular Fellow Department of Internal Medicine University of Texas, Health Science Center at Houston Houston, Texas

Flavia Nelson, MD Associate Professor Department of Neurology Multiple Sclerosis Research Group University of Texas, Health Science Center at Houston Houston, Texas

Cecile L. Phan, MD, FRCPC Assistant Professor Department of Neurology Neuromuscular and EMG Sections Baylor College of Medicine Houston, Texas

David J. Powner, MD, FCCP, FCCM Professor Departments of Neurosurgery and Internal Medicine Division of Neurocritical Care University of Texas, Health Science Center at Houston Houston, Texas

Sophie Samuel, PharmD, BCPS Neuroscience Clinical Pharmacy Specialist Memorial Hermann–Texas Medical Center Houston, Texas

Scott R. Shepard Assistant Professor Department of Neurosurgery University of Texas, Health Science Center at Houston Houston, Texas

Nitin Tandon, MD Associate Professor Departments of Neurosurgery and Pediatric Surgery Director Epilepsy Surgery Program University of Texas, Health Science Center at Houston Houston, Texas

xv

66485457-66485438 www.ketabpezeshki.com

ZZakaria_87574_PTR_CH00_10-06-13_i-xx.inddakaria_87574_PTR_CH00_10-06-13_i-xx.indd xvxv 66/19/2013/19/2013 8:43:238:43:23 PMPM CONTRIBUTORS

Hanh T. Truong, MD Assistant Professor Departments of Neurosurgery and Emergency Medicine Division of Neurocritical Care University of Texas, Health Science Center at Houston Houston, Texas

John J. Volpi, MD Assistant Professor Department of Neurology The Methodist Hospital Weill Cornell Medical College Houston, Texas

Grant M. Warmouth, MD Staff Neurophysiologist St. Louis, Missouri

George W. Williams, MD Assistant Professor Departments of Anesthesiology and Neurosurgery Division of Neurocritical Care University of Texas, Health Science Center at Houston Houston, Texas

Bülent Yapicilar, MD Assistant Professor Department of Neurosurgery MetroHealth Medical Center Case Western Reserve University Cleveland, Ohio

Asma Zakaria, MD Assistant Professor Departments of Neurology and Neurosurgery Division of Neurocritical Care University of Texas, Health Science Center at Houston Houston, Texas

xvi

66485457-66485438 www.ketabpezeshki.com

ZZakaria_87574_PTR_CH00_10-06-13_i-xx.inddakaria_87574_PTR_CH00_10-06-13_i-xx.indd xvixvi 66/19/2013/19/2013 8:43:238:43:23 PMPM Preface

In the last decade, Neurocritical Care has emerged from under the shadow of Stroke and Critical Care Medicine into an entity on its own. In 2005 the United Council of Neurologic Subspecialties (UCNS)* offi cially recognized neurocritical care as a subspecialty. Exhaustive efforts by the Neurocritical Care Society resulted in recognition of the sub-specialty by Leapfrog, and it has been established as standard of care in neuroscience centers across the United States. This has led to a need for consistency in training and knowledge base among practicing and future neuro-intensivists. The fi rst neurocritical care board examination was administered by UCNS in 2007. The grandfathering period for the practice tract will end in December 2013, after which a two-year neurocritical care fellowship will be mandatory to qualify for the boards. The test consists of 200 multiple-choice questions of which 50% will address neurologic disease states and the remaining will assess competency in general critical care. Neurocritical Care Board Review: Questions and Answers is intended to be a comprehensive study guide for candidates sitting for the UCNS neurocritical care specialty board examina- tion. It is a learning and self-assessment tool for practitioners and trainees who treat neuro- logic patients in an emergent or critical care capacity, and for those who are preparing for the UCNS board, or any other specialty examination that requires knowledge of neurologic or general critical care. In keeping with the original intent of this book, I approached physicians from various medical and surgical specialties to author chapters based on the UCNS curriculum, frequently encountered challenges or consults, and what they believed to be an appropriate level of understanding that an intensivist should have on a given topic. Together, we compiled this anthology of over 500 multiple-choice questions with answers, detailed explanations, and ref- erences for further self study. The chapters are named and arranged in a similar format to the UCNS curriculum to allow for easy review and organization when studying for the boards. I encourage all candidates to visit www.ucns.org to peruse eligibility criteria in the grandfa- thering period as well as information regarding examination format and content.

* The United Council for Neurologic Subspecialties did not participate in or contribute materials or advice in the development of this book nor does UCNS endorse or recommend this book or any other texts or other teaching aids for examination preparation purposes.

xvii

66485457-66485438 www.ketabpezeshki.com

ZZakaria_87574_PTR_CH00_10-06-13_i-xx.inddakaria_87574_PTR_CH00_10-06-13_i-xx.indd xviixvii 66/19/2013/19/2013 8:43:238:43:23 PMPM PREFACE

Neurocritical Care is a ubiquitous subspecialty and is of clinical relevance to all inten- sivists and emergency physicians in addition to general and vascular neurologists, cardiol- ogists, nephrologists, internists, as well as vascular and trauma surgeons among others. By no means an exhaustive reference, Neurocritical Care Board Review: Questions and Answers, addresses many of the day-to-day clinical conundrums pertaining to neurologic and general critical care patients encountered in all ICUs and ERs. The compilation includes chapters on neurovascular catastrophes, neurotrauma, encephalopathies, epilepsy, and neuromuscular emergencies as well as general trauma, respiratory, cardiovascular, renal, hematologic, and infectious diseases. This broad range of topics makes it a handy tool for medical students, residents, fellows, advanced care practitioners, and physicians working in any acute care setting. Assembling this book has been a labor of love for me. All credit goes to my students, residents, and fellows for their exhaustive questions on rounds, to my mentors, (especially Dr. Powner) for urging me to embark on this journey, and most of all, the friends and col- leagues who participated as authors. This effort was meant to be an extension of my passion for teaching, and as so often happens when dealing with bright minds, the teacher became the student! I hope this text is as educational and enjoyable to its readers as it has been for me. For those of you taking the Neurocritical Care Boards, I hope it serves its purpose of being a quick and easy self-assessment tool, reference, and guide. Good Luck!

Asma Zakaria, MD

xviii

66485457-66485438 www.ketabpezeshki.com

ZZakaria_87574_PTR_CH00_10-06-13_i-xx.inddakaria_87574_PTR_CH00_10-06-13_i-xx.indd xviiixviii 66/19/2013/19/2013 8:43:238:43:23 PMPM Neurocritical Care Board Review

66485457-66485438 www.ketabpezeshki.com

ZZakaria_87574_PTR_CH00_10-06-13_i-xx.inddakaria_87574_PTR_CH00_10-06-13_i-xx.indd xixxix 66/19/2013/19/2013 8:43:238:43:23 PMPM I Neurologic Disease States: Pathology, Pathophysiology, and Therapy

66485457-66485438 www.ketabpezeshki.com

ZZakaria_87574_PTR_CH01_10-06-13_01-18.inddakaria_87574_PTR_CH01_10-06-13_01-18.indd 1 66/19/2013/19/2013 8:44:178:44:17 PMPM Section I: Cerebrovascular Diseases 1 Infarction, Ischemia, and Hemorrhage John J. Volpi QUESTIONS

1. A 58-year-old man with long-standing hypertension has acute onset of right hemiplegia and aphasia 6 hours prior to his arrival in the ED. He is obtunded, with a dense left gaze deviation, and the left pupil is 6 mm and poorly reactive to light, while the right is 4 mm and reacts to light. The patient vomits and is subsequently intubated and then taken to CT scan, where he is found to have a large left hemisphere hypodensity with mass effect and uncal herniation. Of the following steps, which is most proven to lead to a good outcome? A. Osmotherapy with 23.4% saline B. Increase respiratory rate to 24 C. Hemicraniectomy D. Administer intra-arterial thrombolysis E. Elevate head of bed to 30°

ANSWERS TO THIS SECTION CAN BE FOUND ON PAGE 10 3

66485457-66485438 www.ketabpezeshki.com

ZZakaria_87574_PTR_CH01_10-06-13_01-18.inddakaria_87574_PTR_CH01_10-06-13_01-18.indd 3 66/19/2013/19/2013 8:44:178:44:17 PMPM CEREBROVASCULAR DISEASES

2. A 32-year-old woman with a history of complex partial seizures is well controlled on car- bamazepine for many years. She is found unresponsive in her bedroom. EMS transports her to the ED where her blood pressure is 175/80, heart rate 110, respiratory rate 12, tem- perature 37.0°C. She has mid-gaze pinpoint pupils that cannot be overcome with oculo- cephalic maneuvers, and frequent extensor posturing. CT scan does not show any obvious hypodensity. The best evaluation to carry out next is: A. B. Urine toxicology C. EEG D. Review CT for hyperdense vessel

3. A 78-year-old man with hypertension, tobacco use, and hyperlipidemia presents with an episode of transient aphasia lasting 30 minutes and diffi culty with right-arm coordination. He is examined 1 hour later and found with no defi cits. An MRI shows no acute stroke. Carotid duplex suggests 70% to 90% stenosis of the proximal left internal carotid artery at the bulb. He has no history of coronary artery disease, and his EKG is normal. The best option for stroke prevention in this patient is: A. High-dose statin therapy B. Aspirin C. Carotid stenting D. Carotid endarterectomy

4. A 54-year-old woman with hypertension and hyperlipidemia experiences acute right hemiparesis for 45 minutes. She has a blood pressure of 185/107, recovers, and under- goes a workup that reveals a left middle cerebral artery stenosis of 75%, but no stroke or other vessel disease. She is placed on aspirin, atorvastatin 80 mg, and hydrochlorothiazide 25 mg. She does well for 2 days, is discharged, and then experiences a similar episode last- ing 1 hour. An initial blood pressure is 165/70, which later, when she is asymptomatic, is 145/72. Her repeat workup has no other fi ndings different than prior evaluation. The next best step proven to reduce her risk of subsequent stroke is: A. Stop aspirin, start anticoagulation B. Endovascular stenting of the left-middle cerebral artery C. Stop hydrochlorothiazide D. Start lisinopril 10 mg

5. A 21-year-old woman presents with brief right facial sensory loss and mild right upper extremity incoordination. Her symptoms resolve without any residual defi cit. The MRI shows no infarct, but the magnetic resonance angiography suggests high-grade right- middle cerebral artery stenosis. An angiogram confi rms the stenosis and reveals exten- sive hypertrophy and collateralization in the lenticulostriate vessels. Her blood pressure is 135/65, and her lipid profi le reveals a total cholesterol of 236 and a low-density lipoprotein of 112. The best option for subsequent management of this patient is: A. High-dose statin therapy B. Aspirin C. Endovascular stenting of the right-middle cerebral artery D. Surgical bypass of the right-middle cerebral artery

4

66485457-66485438 www.ketabpezeshki.com

ZZakaria_87574_PTR_CH01_10-06-13_01-18.inddakaria_87574_PTR_CH01_10-06-13_01-18.indd 4 66/19/2013/19/2013 8:44:178:44:17 PMPM INFARCTION, ISCHEMIA, AND HEMORRHAGE: Questions

6. An 11-year-old boy is admitted with a left putamen hemorrhage. His peripheral blood smear reveals sickle cell disease. He undergoes appropriate acute management and reha- bilitation and has no long-term defi cits. His discharge blood pressure is 125/66. To prevent future strokes, the best strategy is: A. Start blood pressure–lowering medications B. Encourage the patient to drink plenty of fl uids C. Monitor routine (TCD) D. Perform weekly complete blood counts (CBCs) for the fi rst month

7. An 85-year-old woman with mild dementia but no signifi cant vascular disease presents to the ED within 1 hour of abrupt onset of nausea and altered mental status. Her blood pres- sure is 135/76, heart rate is 104, respiratory rate 16, and temperature 36.8°C. Her exam is notable for dense left hemineglect. Her motor exam is limited by her neglect but appears normal. A CT scan of the brain shows a 4-cm right parietal intracerebral hemorrhage (ICH). Which of the following treatments could signifi cantly worsen her outcome? A. Placement of an intraventricular catheter to measure intracranial pressure (ICP) B. Surgical evacuation of the hematoma within 4 hours C. Prophylactic seizure therapy with levetiracetam D. Early nutrition support with tube feeds

8. A 45-year-old man with poorly controlled hypertension presents with new onset ataxia and dysarthria with a systolic blood pressure of 225/115. A noncontrast head CT shows a 4-cm right cerebellar hemisphere hemorrhage with near compression of the fourth ven- tricle and dilation of the lateral and third ventricles. The next best step in management is: A. Target mean arterial pressure (MAP) of less than 100 mmHg with labetalol push and nicardipine drip B. Insert intraventricular drain and normalize intracranial pressure (ICP) C. Surgical evacuation of the cerebellar hematoma D. Perform a cerebral angiogram to evaluate for aneurysm

9. A 35-year-old man with poorly controlled hypertension presents to the ED with acute dys- arthria. His initial blood pressure is 175/110. A noncontrast head CT shows a 1-cm pontine hemorrhage. He is placed on a nicardipine drip, but before his blood pressure responds, he becomes obtunded and a follow-up CT shows signifi cant hematoma expansion to involve 3 cm of the mid pons. He is intubated and moved to the ICU with a (GCS) score of 6. Upon arrival to the ICU, the nursing staff comments that this patient has a poor prognosis and asks you to discuss do-not-resuscitate (DNR) status with the family. The most appropriate next step is to: A. Explain to the family that the patient will likely not survive to a functional status and they should consider no CPR if he worsens overnight B. Call an ethics consult to evaluate elements of the case for futility of care C. Notify the family of the severity of the injury but strongly urge them to allow CPR and full code status for the next 24 hours D. Notify the case manager that the patient will likely need long-term care, and plan for early tracheotomy and a gastrostomy tube

5

66485457-66485438 www.ketabpezeshki.com

ZZakaria_87574_PTR_CH01_10-06-13_01-18.inddakaria_87574_PTR_CH01_10-06-13_01-18.indd 5 66/19/2013/19/2013 8:44:178:44:17 PMPM CEREBROVASCULAR DISEASES

10. A 60-year-old woman has a severe headache followed by left hemiplegia. She is brought to the ED and a noncontrast head CT shows a right thalamic hemorrhage with intra- ventricular extension. Her blood pressure is 187/100, heart rate is 20, respiratory rate is 20, and temperature is 37.2°C. She is awake and cooperative but is slowly becoming more lethargic. Her blood pressure is cautiously lowered with labetalol to a mean arterial pressure (MAP) of 120 mmHg. She has an intraventricular drain placed that allows the

intracranial pressure (ICP) to be lowered from 25 to 10 cm H2O. She continues to slowly decline and a follow-up CT shows stability of the bleed but progression of the surround- ing edema. In addition to continuing ICP drainage, which of the following therapies has been shown in preliminary trials to reduce perihematoma edema? A. Induced hypothermia B. High-dose methylprednisolone C. Recombinant factor VII D. Intraventricular tissue plasminogen activator (tPA)

11. A 23-year-old woman is postpartum day 2 from a normal vaginal delivery for which she received an epidural anesthetic, which was a “wet tap” with spinal leak. She is scheduled to be discharged home but has a lingering headache and mild nausea, which has been attributed to a spinal headache. She then has a generalized seizure and becomes diffi cult to arouse. Her sclera are injected. She is intubated and taken for noncontrast CT, which shows diffuse cerebral edema with multiple dilated vessels in the vertex. There are two small cortical hemorrhages, one 1 cm in the left frontal region, and the other 6 mm in the right parietal region. What is the likely diagnosis? A. Ruptured arteriovenous malformation B. Ruptured left-middle cerebral artery aneurysm C. Cerebral venous thrombosis D. Brainstem infarction

12. For the same patient, what is the best initial management? A. Heparin drip B. Interventional thrombolysis C. Mannitol D. Hemicraniectomy

13. A 45-year-old woman is an unrestrained passenger in a motor vehicle accident where she sustains a blow to the right side of her head. Other than a brief loss of consciousness and head pain, she has no defi cits. She has a negative noncontrast head CT in an ED after the event and is discharged home. Over the next several weeks, she notices occasional diplo- pia and eye redness, prompting further evaluation. Upon seeing her in your offi ce, the most useful diagnostic test is: A. Brain MRI with contrast B. Cerebral angiogram C. Optic nerve sheath ultrasound D. Formal visual fi eld assessment

6

66485457-66485438 www.ketabpezeshki.com

ZZakaria_87574_PTR_CH01_10-06-13_01-18.inddakaria_87574_PTR_CH01_10-06-13_01-18.indd 6 66/19/2013/19/2013 8:44:188:44:18 PMPM INFARCTION, ISCHEMIA, AND HEMORRHAGE: Questions

14. A 13-year-old girl has a fall while horseback riding. She has no loss of consciousness, but complains of left-sided posterior neck pain. She is referred to the ED, where an MRI of the cervical spine is unremarkable. While she has no motor or sensory defi cits, the ED physi- cian detects a left lid lag and pupil asymmetry and calls for a neurological evaluation. The pupil asymmetry will be most noticeable: A. In the dark B. In direct light C. During the swinging fl ash light test D. When examined with a red lens

15. A 26-year-old nurse has frequent nausea and vomiting during pregnancy. After an epi- sode of vomiting, she becomes aphasic and has diffi culty moving her right arm. She is brought to the ED, where her symptoms begin to improve, but on exam, she remains impaired in language fl uency and has a right-arm drift. Initial CT is normal, but subse- quent MRI shows an infarction in the left insula and pre-central gyrus. She undergoes MR angiogram of the neck, which shows a 6-cm dissection of the carotid artery originating from the carotid bulb. The lumen is reduced in diameter to approximately 70%. She has been improving with no further symptoms. The best treatment at this point is: A. Endovascular stenting B. Aspirin C. Warfarin D. Endarterectomy

16. Which of the following statements regarding blood pressure management in stroke patients is true? A. There is signifi cant class-specifi c evidence for the superiority of calcium channel block- ers in stroke patients over other blood pressure–reducing medications B. Stopping blood pressure medications in hospitalized acute stroke patients leads to worse outcomes C. Careful blood pressure lowering in the hospital was demonstrated to lead to better outcomes in the SCAST trial D. Two-drug therapy is indicated as an initial strategy in patients with an observed blood pressure of 160/100

17. A 75-year-old man presents with a right facial droop that is apparent at rest and does not improve with grimace. He is able to elevate his forehead symmetrically. His speech is slurred but understandable, and he has mild sensory loss on the right with a right-arm drift, but no other defi cits. What would his National Institutes of Health (NIH) Stroke Scale be? A. 4 B. 5 C. 6 D. 7

7

66485457-66485438 www.ketabpezeshki.com

ZZakaria_87574_PTR_CH01_10-06-13_01-18.inddakaria_87574_PTR_CH01_10-06-13_01-18.indd 7 66/19/2013/19/2013 8:44:188:44:18 PMPM CEREBROVASCULAR DISEASES

18. A 60-year-old woman with hypertension but no prior stroke or other disease presents to the hospital via EMS after being found down by her son. Her son had dinner with her the evening before and she said goodnight to him around 11 p.m. He saw her walking normally from her bedroom at 8 a.m., but they did not talk. At 10 a.m., she called him at work and was diffi cult to understand so he came home and found her on the fl oor. It is now 11:30 a.m. Her blood pressure is 175/100, heart rate is 74 and regular, and tempera- ture is 36.9°C. The patient has a right gaze with dense left hemiplegia. CT of the brain shows blurring of the gray–white junction in the right-middle cerebral artery territory, but no defi nite hypodensity. Her NIH Stroke Scale is calculated to be 22. The right-middle cerebral artery is hyperdense. Is this patient a candidate for thrombolysis? A. No, her last-known normal was the prior evening, which is outside the tissue plasmi- nogen activator (tPA) window B. No, her last-known normal was 8 a.m., which is outside the approved 3-hour window for tPA C. Yes, her time of onset was 10 a.m., which is within the approved 3-hour window for tPA D. Yes, her last-known normal was 8 a.m., which is within the 4.5-hour window for tPA

19. A 45-year-old man is on hospital day 2 after a large left-middle cerebral artery infarc- tion. He develops a temperature of 39.2°C and has a pulmonary infi ltrate in the right- middle lobe of the lung. Which of the following strategies is most effective in achieving euthermia? A. Early initiation of broad-spectrum antibiotics B. Scheduled oral acetaminophen C. Scheduled IV diclofenac infusions D. Surface cooling device

20. An 89-year-old woman who lives alone has acute onset of left hemiparesis and dysarthria. She is brought to the ED, receives IV tissue plasminogen activator (tPA), and improves to her baseline despite evidence of a 3-cm infarction in the right insula on MRI. The MRI otherwise shows age-appropriate atrophy without a signifi cant amount of white matter disease. During her evaluation, she is found to be in atrial fi brillation. What regimen should be recommended for secondary stroke prevention? A. Aspirin B. Aspirin plus clopidogrel C. Warfarin D. Amiodarone without anticoagulation

21. In which scenario is anticonvulsant therapy recommended? A. Prophylaxis in ischemic stroke patients with large cortical infarcts B. Prophylaxis in hemorrhagic lobar stroke patients with signifi cant edema C. Prophylaxis in ischemic stroke patients, post hemicraniectomy D. Ischemic stroke patients who have an isolated, brief seizure 2 weeks after initial stroke

8

66485457-66485438 www.ketabpezeshki.com

ZZakaria_87574_PTR_CH01_10-06-13_01-18.inddakaria_87574_PTR_CH01_10-06-13_01-18.indd 8 66/19/2013/19/2013 8:44:188:44:18 PMPM INFARCTION, ISCHEMIA, AND HEMORRHAGE: Questions

22. A 31-year-old woman with no signifi cant medical problems experienced a severe headache while jogging. She went to the ED and was alert but had a severe, throbbing headache and no focal neurologic symptoms. Her blood pressure was 173/105, and noncontrast head CT was normal. She went home from the ED, and the following day, she experienced aphasia while speaking to a friend on the phone. She returned to the ED, where her symptoms resolved, but a repeat CT scan showed a small, distal, right parietal convexity subarachnoid hemorrhage. Lumbar puncture (LP) was normal with no infl ammation. Cerebral angiog- raphy showed no aneurysm or arteriovenous malformation (AVM), but multiple areas of vasoconstriction. She was treated with fl uids, and an extensive autoimmune workup was negative. On follow-up imaging 3 months later, one would expect to fi nd: A. Complete resolution of the vasoconstriction B. Mycotic aneurysms C. Hypertrophy of the lenticulostriates D. Diffuse white matter disease

23. A 49-year-old woman with no signifi cant past medical history is staying at a hotel on a business trip abroad when she has acute onset of dysarthria and left-side weakness with sensory loss. She is found to have a moderate-sized right-middle cerebral artery infarc- tion on MRI, but no sign of large vessel disease. Her hypercoaguable workup is negative, EKG and telemetry are normal, and transthoracic echocardiogram (TTE) is normal, but transesophageal echocardiogram (TEE) shows a right-to-left shunt across a patent fora- men ovale (PFO) during the Valsalva maneuver. The patient is very interested in “fi xing the problem” and would like to have the PFO closed. The most appropriate next step in management is: A. Refer the patient to a cardiologist for PFO closure B. Start aspirin and counsel the patient that her future stroke risk is negligible C. Start anticoagulation for life D. Refer the patient for a clinical trial in PFO closure

24. A 73-year-old woman with hyperlipidemia and hypertension experiences a 30-minute episode of dysarthria and right-side weakness. She comes to the ED and is fully recov- ered. Her initial blood pressure is 163/102. CT is normal, and EKG shows normal sinus rhythm. What is her ABCD2 score, and should she be admitted to the hospital? A. 3, no admission required B. 4, no admission required C. 4, yes admission required D. 5, yes admission required

25. A 69-year-old man has chest pain and is found to have an aortic dissection requiring emergent OR repair. Postoperatively, he is noted to be paraplegic. Which of the following strategies is worthwhile in this patient? A. Reduction of mean arterial pressure to target less than 100 mmHg B. Placement of a lumbar drain C. Avoid bypass methods during surgery for distal reperfusion D. Prolong ICU sedation to avoid oxygen consumption

9

66485457-66485438 www.ketabpezeshki.com

ZZakaria_87574_PTR_CH01_10-06-13_01-18.inddakaria_87574_PTR_CH01_10-06-13_01-18.indd 9 66/19/2013/19/2013 8:44:188:44:18 PMPM 1

ANSWERS

1. The answer is C. Hemicraniectomy is a life-saving remedy in the setting of massive hemispheric infarct. The results of the HAMLET–DESTINY–DECIMAL pooled analysis of hemicraniectomy versus medical management provide strong evidence for this therapy. A total of 93 patients were included in the pooled analysis. More patients in the decompres- sive-surgery group than in the control group had a modifi ed Rankin Score (mRS) less than or equal to 4 (75% vs. 24%; pooled absolute risk reduction 51%), an mRS ≤ 3 (43% vs. 21%; 23%), and survived (78% vs. 29%; 50%); with numbers needed to treat of two for survival with mRS ≤ 4, four for survival with mRS ≤ 3, and two for survival irrespective of functional outcome (1). In this scenario, the patient is an ideal candidate for surgery based on his age and the early nature of the edema. Osmotherapy is an important temporizing measure to reduce edema for 2 to 6 hours, but it is not the best answer as it is not a therapy that has been subjected to a large, randomized trial to show better outcomes in the absence of defi ni- tive management. Similarly, raising the head of the bed or increasing the respiratory rate will produce a decrease in intracranial pressure (ICP) but will not be expected to provide suffi cient benefi t to lead to a better outcome. Intra-arterial thrombolysis would be poorly tolerated in a patient with a large hypodensity on CT and signs of herniation. It would most likely worsen the patient’s outcome based on analysis of the PROACT II trial (2).

2. The answer is D. This is an abrupt coma with a reportedly normal CT. In this case, the dif- ferential diagnosis would include a post-ictal state, meningitis, intoxication, subarachnoid hemorrhage, or brainstem stroke. All are realistic possibilities in this patient, although the absence of fever makes meningitis the least likely. In terms of the proper steps in carry- ing out the evaluation for these conditions, basilar thrombosis presents the most morbid possibility and one that requires prompt recognition in order to consider thrombolysis. A hyperdense basilar artery is often the only sign of basilar thrombosis on initial CT because

10

66485457-66485438 www.ketabpezeshki.com

ZZakaria_87574_PTR_CH01_10-06-13_01-18.inddakaria_87574_PTR_CH01_10-06-13_01-18.indd 1010 66/19/2013/19/2013 8:44:188:44:18 PMPM INFARCTION, ISCHEMIA, AND HEMORRHAGE: Answers

hypodensity may not yet be present, and it is diffi cult to discern early ischemia in the pos- terior fossa. A hyperdense vessel is not always noted in a radiology report and should be independently evaluated by the clinician. This review takes only a moment and can dra- matically alter the subsequent evaluation and treatment options. The remaining choices are reasonable but require too much time to complete in a patient who may be a candidate for thrombolysis.

3. The answer is D. This patient has suffered a transient ischemia attack (TIA) of the left hemisphere. The high-grade (>70%) stenosis found in the ipsilateral carotid artery places this patient at approximately 26% risk of stroke in 2 years, according to the North American Symptomatic Carotid Endarterectomy Trial (NASCET). The addition of aspi- rin or statin therapy is appropriate, but they are not suffi cient to exclude revascular- ization, which remains the best option and offers the most relative risk reduction of the choices provided (65% for endarterectomy vs. approximately 20% for aspirin and approximately 30%–35% for statins). Carotid stenting and endarterectomy are compa- rable methods to achieve revascularization, and choosing the right option depends on the patient’s comorbidities. In a patient with a high risk of perioperative myocardial infarction, stenting would be preferable. This patient has no such risk from the infor- mation given. Furthermore, in the CREST trial, patients over the age of 70 had less risk of stroke with endarterectomy than stenting. In this case, endarterectomy is the best option. Considerations such as location of the plaque, risk of cranial nerve palsies, and cosmetic scarring should be kept in mind, but in this case, the lesion is easily accessible with surgery (3).

4. The answer is D. Crescendo TIAs represent a challenging clinical scenario for which there are few options. The WASID trial evaluated the use of anticoagulation in large- vessel intracranial stenosis, and there was a 4.3% rate of death in the aspirin group and a 9.7% rate in the warfarin group (P = .02). This increased risk of death makes warfarin an unacceptable choice in this patient (4). The SAMMPRIS trial evaluated endovascular stenting versus aggressive medical management and found a higher 30-day stroke risk in patients undergoing stent than those on medical management (14.7% vs. 5.8%) (5). Controversy remains regarding the future of endovascular therapies to address patients who do not respond to medical management, but currently there are no clinical trials of stenting to support this strategy. The question of blood pressure management in this population is perplexing. Permissive hypertension remains a commonplace practice with relatively little prospective data to validate its effi cacy. If hemodynamic insuffi ciency alone were thought to be the cause of the symptoms, this patient should not be normal with the reportedly lower blood pressure. Furthermore, the WASID subgroup analysis of patients with recurrent stroke showed a lower risk of recurrent stroke with blood pres- sure improvement (6). Based on the currently available data, stopping blood pressure medications would likely worsen the patient’s outcome, and the best strategy is, in fact, to improve blood pressure control with the addition of another drug, such as lisinopril.

5. The answer is D. This patient presents with classic angiographic fi ndings of Moyamoya disease. Statin therapy has no proven role in Moyamoya disease and based on this patient’s age, it is highly unlikely that the angiographic results represent atherosclerosis. Aspirin

11

66485457-66485438 www.ketabpezeshki.com

ZZakaria_87574_PTR_CH01_10-06-13_01-18.inddakaria_87574_PTR_CH01_10-06-13_01-18.indd 1111 66/19/2013/19/2013 8:44:188:44:18 PMPM CEREBROVASCULAR DISEASES

therapy is a reasonable consideration but should not be carried out in the long term with- out surgical management because of the risk of the hypertrophied vessels rupturing and causing a basal ganglia hemorrhage. Endovascular stenting is contraindicated and results in rapid restenosis. Surgical management with superfi cial temporal artery bypass to the middle cerebral artery (also called EC-IC bypass) is effective and leads to less risk of isch- emia and hemorrhage.

6. The answer is C. The STOP trial evaluated the role of transfusion exchange in patients with sickle cell disease and found a signifi cant benefi t to this therapy (7). The tool used to monitor patients and assess hyperviscosity requiring transfusion exchange is transcranial doppler (TCD). Pediatric patients with sickle cell disease should be referred for TCD on a routine basis and undergo transfusion exchange if the middle cerebral artery velocity exceeds 200 cm/sec. Blood pressure lowering has limited utility in a patient with nearly normal blood pressure, and while the patients should remain well hydrated to avoid sickle crisis, it is not a suffi cient strategy for stroke prevention. Weekly CBCs have no benefi t.

7. The answer is B. The STICH trial showed no benefi t to surgical evacuation of ICH and sub- sequent trials of early evacuation showed worse outcomes (8,9). In particular, this patient is likely to have cerebral amyloid angiopathy, and surgery carries a signifi cant risk of adja- cent tissue hemorrhage. Although surgery remains controversial in most cases, isolated craniectomy without hematoma evacuation would be worthwhile to consider in a patient with impending herniation from supratentorial ICH. There is no evidence that prophy- lactic seizure therapy improves outcomes, although in the case of levetiracetam, there is also no evidence of worsening outcomes (10). Early nutritional support has been shown to improve outcomes. An intraventricular catheter for monitoring intracranial pressure (ICP) is indicated if the GCS score is less than 8 and has not been observed to worsen outcomes.

8. The answer is C. Unlike supratentorial hemorrhages, infratentorial bleeds require urgent surgical evacuation to prevent herniation, tissue destruction from compression, acute hydrocephalus, and impaired cerebral blood fl ow. Although blood pressure lowering is an important part of intracerebral hemorrhage (ICH) management, it must be done in a rational way to avoid hypoperfusion. In this case, it is important to recall the formula for cerebral perfusion pressure (cerebral perfusion pressure [CPP] = mean arterial pressure [MAP] – intracranial pressure [ICP]). Obstruction of the fourth ventricle will cause the ICP to rise signifi cantly. If the MAP is lowered abruptly as listed in the fi rst answer, the patient is at risk of cerebral hypoperfusion. Adequate cerebral perfusion pressure is 70 to

90 mmHg. If the patient developed acute hydrocephalus with an ICP of 40 cm H2O, this roughly equals 30 mmHg. Targeting an MAP of less than 100 mmHg means that CPP = 100 – 30 = 70, which is at the threshold for hypoperfusion. The other answer of inserting a would certainly improve ICP and CPP, but it would do so at the risk of causing upward herniation of the brainstem and is not advisable in a patient at risk of herniation unless it is done in combination with suboccipital craniectomy. Cerebral angio- gram is a worthwhile diagnostic test, but it should be performed after the craniectomy.

9. The answer is C. This patient has a potentially devastating hemorrhage in a region of the brain that could lead to permanent disability. Nonetheless, based on his intracerebral hemorrhage (ICH) score, his 30-day mortality risk is only 26%. The use of DNR orders in

12

66485457-66485438 www.ketabpezeshki.com

ZZakaria_87574_PTR_CH01_10-06-13_01-18.inddakaria_87574_PTR_CH01_10-06-13_01-18.indd 1212 66/19/2013/19/2013 8:44:188:44:18 PMPM INFARCTION, ISCHEMIA, AND HEMORRHAGE: Answers

the ICU varies signifi cantly by institution. In a meta-analysis of ICU prognosis, factors such as gender, GCS score, ICH volume, intraventricular hemorrhage (IVH), age, mid- line shift, uncal herniation, cisternal effacement, location of the hemorrhage, and glucose level were all considered as prognostic factors, but DNR status was the only variable that signifi cantly predicted mortality. Furthermore, in this study DNR orders were imple- mented and care withdrawn on average at 2 days. This suggests that DNR orders worsen patient outcome, especially when implemented early (11). It is important to involve fam- ily members in the decision-making process, but unless a patient has a pre-existing DNR order or long-standing, well-known wishes to not receive aggressive care, DNR orders should be postponed for at least 24 hours.

10. The answer is A. There are few well-proven therapies for intraventricular hemorrhages. Many therapies are being considered and investigated that may yield new therapeutic options. Induced hypothermia in a single center trial showed that it could halt perihema- toma edema (12). There is no evidence for high-dose steroids in ICH. The FAST trial of recombinant Factor VII showed that it reduced hematoma expansion by reducing the risk of rebleeding, but there was no evidence for an effect on the edema around the hematoma. Intraventricular tPA is a promising therapy for reducing the burden of intraventricular clot and is the subject of recent trials, but its effect is also hypothesized to be directly reducing the clot burden and not the surrounding edema.

11. The answer is C. Cerebral venous thrombosis is a diffi cult diagnosis to make based on noncontrast head CT and requires a high degree of pretest suspicion. A contrast-enhanced CT will often reveal the thrombosis more readily, and if suspected, this is a rare indi- cation for contrast-enhanced CT in acute stroke. The CT fi ndings include dilated corti- cal veins, cortical subarachnoid blood, and dense-appearing cerebral sinuses. With con- trast, the dense sinus sign becomes the so-called empty delta sign and is present in about one-third of cases. CT remains normal in many cases. A number of patients who lack typical vascular risk factors are at risk for venous thrombosis, such as pregnant women; patients with hematological, oncological, and autoimmune diseases; and patients with head trauma or recent intrathecal or spinal procedures. The fi ndings of scleral injection make venous insuffi ciency very likely. Ruptured arteriovenous malformation (AVM) or aneurysm would be expected to produce more obvious signs of subarachnoid blood in a patient who deteriorates rapidly and is preceded with a much more severe headache. A brainstem infarction should not produce cortical hemorrhages.

12. The answer is A. In most patients with cerebral venous thrombosis, there is an excel- lent response to heparin infusion. This treatment should be continued even in the presence of small cortical bleeds. Bleeding in cerebral venous thrombosis is caused by high venous pressure, and thus adequate anticoagulation is necessary to improve the underlying problem. The other treatments listed are reasonable considerations in a patient who does not respond to intravenous heparin. Osmotherapy can cause dehydration, venous constriction and worsen the thrombotic situation, but it has a role when attempts at reducing intracranial pressure (ICP) have failed or are not available, including lumbar drainage, acetazolamide, and optic nerve fenestration. Hemicraniectomy should be considered when all other options at relieving ICP have failed and herniation is a concern.

13

66485457-66485438 www.ketabpezeshki.com

ZZakaria_87574_PTR_CH01_10-06-13_01-18.inddakaria_87574_PTR_CH01_10-06-13_01-18.indd 1313 66/19/2013/19/2013 8:44:188:44:18 PMPM CEREBROVASCULAR DISEASES

13. The answer is B. The presentation is typical of a carotid-cavernous fi stula. This abnor- mal communication between the carotid artery and the cavernous sinus often arises from trauma, but can occur spontaneously, and should be suspected in any patient with unex- plained chemosis, especially if it involves any degree of ophthalmoplegia. The contents of the cavernous sinus include cranial nerves III and IV, which are compressed by the enlarg- ing fi stula and cause ophthalmoplegia. Treatment is typically endovascular occlusion of the fi stula.

14. The answer is A. The patient has a vertebral artery dissection. This can produce a Wallenberg syndrome from loss of fl ow in the ipsilateral posterior inferior cerebellar artery (PICA), although it is typically an incomplete presentation because of collateral blood fl ow. In this case, the patient has signs of ptosis and miosis, suggesting a Horner’s syndrome. Unlike ptosis from third nerve palsy, ptosis from sympathetic disruption is often rather subtle with only a lid lag when noted and not frank eye closure. The sympa- thetic innervation of the pupil causes dilation via the dilator papillae, the radially oriented smooth muscles of the iris. Sympathetic tone increases in the dark to allow more light to enter the eye; in this setting, the affected eye will appear miotic. Conversely, the constrict- ing sphincter papillae muscles are also smooth muscles of the iris, but they are innervated by parasympathetic fi bers and are most active in light. In this patient, the parasympathet- ics would be spared and the pupils would appear symmetrical in full light. The swinging fl ash light test is helpful for detecting an afferent papillary defect, but in this scenario, one would not expect anisochoria. The red lens test is useful for isolating slight diplopia and does not help identify pupillary abnormalities.

15. The answer is B. Although carotid dissection remains a disease with scant trial evi- dence, there are a number of single center series and a 2008 Cochrane meta-analysis of these series that show anticoagulation and antiplatelet treatments to be equally effec- tive. In this pregnant woman, warfarin would not be desirable because of teratogenic- ity. Surgical options for management of dissection exist, but are quite rare and typically reserved for ligation when endovascular and medical management are not possible. Endovascular repair has become more frequent, and while prospective data for its effi - cacy and safety are lacking, it is commonly used when a patient does not respond to initial medical management or has intracranial extension of the dissection, neither of which are present in this scenario.

16. The answer is D. Two-drug therapy is indicated as an initial strategy in patients with an observed blood pressure of 160/100. According to the JNC 7 guidelines, patients with blood pressure greater than 160 systolic or greater than 100 diastolic are considered stage 2 hypertension, and initial therapy with two drug classes is recommended. Blood pres- sure goals and strategies remain controversial, and increasing evidence suggests there is little to be gained by either permissive hypertension or modest lowering of blood pres- sure acutely. In the Scandinavian Candesartan Acute Stroke Trial (SCAST) trial, careful blood pressure lowering had no effect on outcome, and in the Continue or Stop post- Stroke Antihypertensives Collaborative Study (COSSACS) trial, neither continuing nor stopping blood pressure medications in hospitalized stroke patients had an effect on out- comes (13,14). There is good evidence for secondary stroke prevention with a number of classes of antihypertensive medications. While calcium channel blockers are a reasonable

14

66485457-66485438 www.ketabpezeshki.com

ZZakaria_87574_PTR_CH01_10-06-13_01-18.inddakaria_87574_PTR_CH01_10-06-13_01-18.indd 1414 66/19/2013/19/2013 8:44:188:44:18 PMPM INFARCTION, ISCHEMIA, AND HEMORRHAGE: Answers

strategy, there is no reason to choose this class over thiazide diuretics, angiotensin convert- ing enzyme (ACE) inhibitors, angiotensin receptor blockers, or beta-blockers. Thiazide diuretics, remain the fi rst line of recommendation in JNC 7.

17. The answer is B. The NIH Stroke Scale is an effi cient and reliable means of communicat- ing stroke severity, and it is used in most clinical trials of stroke. This patient receives two points for a central VII facial palsy, one point for dysarthria, one point for sensory loss, and one point for right-arm drift.

18. The answer is D. Determining eligibility for thrombolysis depends on establishing a “last known well” time in most cases, unless the stroke onset is directly observed. The 10 a.m. phone call cannot be used as a last known normal since the patient was not observed between 8 a.m. and 10 a.m. to be well. Nonetheless, the clinician should use all available informa- tion to make this determination and consider each scenario in the context of both his- tory and imaging. The CT suggests an acute stroke, so it is reasonable to assume that the stroke is only a few hours old. Furthermore, the son’s report of his mother walking at 8 a.m. is suffi cient to establish that she was well at 8 a.m. even though she did not speak. If a patient has isolated aphasia or dysarthria, this might not be enough information, but in a patient with an obvious middle cerebral artery infarction, it is safe to assume that she would not have been able to walk normally if this stroke was in evolution at 8 a.m. Finally, the window for thrombolysis has been expanded to 4.5 hours based on the ECASS III trial (15). Patients over 80, those with NIH Stroke Scale score greater than 25, and those with diabetes and prior stroke were excluded in this trial. This patient has none of these exclusions, so she qualifi es for the extended time window.

19. The answer is D. While broad-spectrum antibiotics are necessary in many ICU settings, it may take days for a fever curve to resolve after an infection. Studies of oral acetamino- phen and IV diclofenac have shown them to be ineffective at fever control and to carry the risk of worsening hepatic and renal function. Surface cooling is highly effi cient and much more effective at controlling temperature.

20. The answer is C. Although the risk of major intracranial hemorrhage increases with advanced age, the risk of ischemic stroke from atrial fi brillation increases as well and remains signifi cantly greater than the risk of intracranial hemorrhage. A prospective trial of 973 patients over the age of 75 assigned to warfarin versus aspirin for atrial fi brilla- tion found that ischemic strokes were more than twice as common in the aspirin group (44 events vs. 21 in warfarin group), but that major bleeds were equivalent (3 in the warfa- rin group and 4 in the aspirin group). Warfarin in this population produced a 52% relative risk reduction in ischemic stroke with no signifi cant increase in risk of major hemorrhage (16). The combination of aspirin plus clopidogrel for stroke prevention produced a mod- est improvement in ischemic stroke in the ACTIVE-A trial, but a similar increase in bleed- ing risk equating to no net benefi t (17).

21. The answer is D. There is no data to support the use of prophylactic anticonvulsants in stroke patients. Furthermore, even in patients who are at an elevated risk of poststroke seizure, such as the scenarios listed in choices A to C, there is no evidence that preventing seizure improves outcomes. There is some data to suggest worsening of ischemia with

15

66485457-66485438 www.ketabpezeshki.com

ZZakaria_87574_PTR_CH01_10-06-13_01-18.inddakaria_87574_PTR_CH01_10-06-13_01-18.indd 1515 66/19/2013/19/2013 8:44:188:44:18 PMPM CEREBROVASCULAR DISEASES

phenytoin, and possibly anticonvulsants in general. In ischemic stroke patients, current guidelines support the use of anticonvulsants only if a patient has a clinically defi nite seizure. It is reasonable to use anticonvulsants for a period of time and consider gradual discontinuation once the patient has become seizure-free.

22. The answer is A. The patient most likely has Call–Fleming syndrome or reversible cere- bral vasoconstriction syndrome (RCVS). This is an underdiagnosed entity that causes thunderclap headache and can easily be mistaken for primary (CNS) angiitis and delayed cerebral ischemia from subarachnoid hemorrhage. It is also known as “pseudovasculitis” because the fi ndings on angiography so closely mimic vasculitis. RCVS is associated with a number of drugs, as well as exertion, pregnancy, and other headache types. A number of treatments have been used, but none are well established by trial data. The disease is self-limited but can be complicated by seizure and cerebral ischemia in up to 20% of patients. The clinical picture is not consistent with endocarditis, so mycotic aneurysms would be unlikely. The hypertrophy of lenticulostri- ates is a result of Moyamoya disease, which affects only the proximal vessels, whereas this patient had diffuse disease on initial imaging. While white matter infarctions are possible, they are rare, and often limited only to a single region, not diffuse.

23. The answer is D. This patient has a cryptogenic stroke. PFO is more prevalent in patients with cryptogenic stroke than in the general population, but based on the results of the CLOSURE study, there is no benefi t to PFO closure with the device used in that trial (18). Patients with cryptogenic stroke in general have a 2-year risk of stroke of 2% to 5%, which is considerably higher than age-matched cohorts who have never had a stroke. In the absence of a hypercoagulable state, there is no proven benefi t of warfarin over aspirin in patients with cryptogenic stroke. While aspirin remains the mainstay of treatment, tri- als investigating other devices, which may be safer than the CLOSURE trial device, are ongoing, and interested patients should be referred to trial centers. Off-label PFO closure, using devices outside of clinical trials, is not recommended.

24. The answer is D. The ABCD2 system is a method for determining 2-, 7-, 30-, and 90-day stroke risk. Hospitals, EDs, and insurers are using it as a guide for decisions on admission versus outpatient evaluation. “A” stands for age: a patient gets one point for age greater than or equal to 60. “B” stands for blood pressure: a patient gets one point if either the systolic blood pressure is greater than or equal to 140 mmHg, or the diastolic blood pres- sure is greater than or equal to 90 mmHg. “C” stands for clinical criteria: a patient gets one point for isolated speech impairment without weakness, and two points for unilateral weakness. “D” stands for diabetes: a patient gets one point if he or she is diabetic. The sec- ond “D” stands for duration: a patient gets one point for an episode of 10 to 59 minutes, and two points for an episode greater than or equal to 60 minutes. This patient scores a fi ve, based on the description. Although different organizations use various cutoffs for decision making, a score of four or fi ve results in a 2-day stroke risk of 4.1%, which justi- fi es hospital admission (19).

25. The answer is B. infarction remains a poorly studied condition, but among the available strategies, two are worthwhile based on case series. First, the placement of a lumbar drain reduces intrathecal pressures and allows for increased cord perfusion.

16

66485457-66485438 www.ketabpezeshki.com

ZZakaria_87574_PTR_CH01_10-06-13_01-18.inddakaria_87574_PTR_CH01_10-06-13_01-18.indd 1616 66/19/2013/19/2013 8:44:188:44:18 PMPM INFARCTION, ISCHEMIA, AND HEMORRHAGE: Answers

Mean arterial pressure (MAP) should not be lowered for the same reason. Second, the use of distal bypass during surgery to restore cord perfusion in segments disrupted by grafting seems to improve outcomes. This strategy is likely best accomplished by monitoring somatosensory evoked potentials (SSEPs) during aortic repair and opting for bypass when the SSEPs show poor signal transmission. Prolonging ICU sedation may lead to other complications such as critical illness polyneuropathy or myonecrosis and is not supported by any data.

References

1. Vahedi K, Hofmeijer J, Juettler E, et al. Early decompressive surgery in malignant infarc- tion of the middle cerebral artery: a pooled analysis of three randomized control trials. Lancet Neurol. 2007;6(3):215–222. 2. Wechsler LR, Roberts R, Furlan AJ et al. Factors infl uencing outcome and treatment effect in PROACT II. Stroke. 2003;34(5):1224–1229. 3. Brott TG, Hobson RW 2nd, Howard G, et al. Stenting versus endarterectomy for treat- ment of carotid-artery stenosis. N Engl J Med. 2010;363(1):11–23. 4. Chimowitz MI, Lynn MJ, Howlett-Smith H, et al. Comparison of warfarin and aspirin for symptomatic intracranial arterial stenosis. N Engl J Med. 2005;352(13):1305–1316. 5. Chimowitz MI, Lynn MJ, Derdeyn CP, et al. Stenting versus aggressive medical therapy for intracranial arterial stenosis. N Engl J Med. 2011;365(11):993–1003. 6. Turan TN, Cotsonis G, Lynn MJ, Chaturvedi S, Chimowitz M. Relationship between blood pressure and stroke recurrence in patients with intracranial arterial stenosis. Circulation. 2007;115(23):2969–2975. 7. Adams RJ, McKie VC, Hsu L, et al. Prevention of a fi rst stroke by transfusions in children with sickle cell anemia and abnormal results on transcranial doppler ultrasonography. N Engl J Med. 1998;339(1):5–11. 8. Mendelow AD, Gregson BA, Fernandes HM, et al. Early surgery versus initial conser- vative treatment in patients with spontaneous supratentorial intracerebral haematomas in the International Surgical Trial in Intracerebral Haemorrhage (STICH): a randomised trial. Lancet. 2005;365(9457):387–397. 9. Morgenstern LB, Demchuk AM, Kim DH, Frankowski RF, Grotta JC. Rebleeding leads to poor outcome in ultra-early for intracerebral hemorrhage. Neurology. 2001;56(10):1294–1299. 10. Naidech AM, Garg RK, Liebling S, et al. Anticonvulsant use and outcomes after intrace- rebral hemorrhage. Stroke. 2009;40(12):3810–3815. 11. Becker KJ, Baxter AB, Cohen WA, et al. Withdrawal of support in intracerebral hemor- rhage may lead to self-fulfi lling prophecies. Neurology. 2001;56(6):766–772. 12. Kollmar R, Staykov D, Dörfl er A, Schellinger PD, Schwab S, Bardutzky J. Hypothermia reduces perihemorrhagic edema after intracerebral hemorrhage. Stroke. 2010;41(8):1684–1689. 13. Sandset EC, Bath PM, Boysen G, et al. The angiotensin-receptor blocker candesartan for treatment of acute stroke (SCAST): a randomised, placebo-controlled, double-blind trial. Lancet. 2011;377(9767):741–750.

17

66485457-66485438 www.ketabpezeshki.com

ZZakaria_87574_PTR_CH01_10-06-13_01-18.inddakaria_87574_PTR_CH01_10-06-13_01-18.indd 1717 66/19/2013/19/2013 8:44:188:44:18 PMPM CEREBROVASCULAR DISEASES

14. Robinson TG, Potter JF, Ford GA, et al.. Effects of antihypertensive treatment after acute stroke in the Continue or Stop Post-Stroke Antihypertensives Collaborative Study (COSSACS): a prospective, randomised, open, blinded-endpoint trial. Lancet Neurol. 2010;9(8):767–775. 15. Hacke W, Kaste M, Bluhmki E, et al. Thrombolysis with alteplase 3 to 4.5 hours after acute ischemic stroke. N Engl J Med. 2008;359(13):1317–1329. 16. Mant J, Hobbs FD, Fletcher K, et al. Warfarin versus aspirin for stroke prevention in an elderly community population with atrial fi brillation (the Birmingham Atrial Fibrillation Treatment of the Aged Study, BAFTA): a randomised controlled trial. Lancet. 2007;370(9586):493–503. 17. Connolly SJ, Pogue J, Hart RG, et al. Effect of clopidogrel added to aspirin in patients with atrial fi brillation. N Engl J Med. 2009;360(20):2066–2078. 18. Furlan AJ, Reisman M, Massaro J, et al. Closure or medical therapy for cryptogenic stroke with patent foramen ovale. N Engl J Med. 2012;366(11):991–999. 19. Johnston SC, Rothwell PM, Nguyen-Huynh MN, et al. Validation and refi nement of scores to predict very early stroke risk after transient ischaemic attack. Lancet. 2007;369(9558):283–292.

18

66485457-66485438 www.ketabpezeshki.com

ZZakaria_87574_PTR_CH01_10-06-13_01-18.inddakaria_87574_PTR_CH01_10-06-13_01-18.indd 1818 66/19/2013/19/2013 8:44:188:44:18 PMPM 2 Subarachnoid Hemorrhage and Vascular Malformations Bülent Yapicilar and Asma Zakaria QUESTIONS

1. A 40-year-old female presents to the ED complain- ing of severe headache and new vision changes in her right eye. She is anxious because her mother died of a brain hemorrhage. Her past medical his- tory is signifi cant for nephrectomy 2 years ago, lupus, and poorly controlled hypertension. She has been smoking one pack of cigarettes per day for the past 20 years and is a heavy drinker. Her exam is unremarkable except her right eye is blind with a dilated pupil and ptosis. What is the next step in the management of this patient after reviewing this initial CT scan? A. Get a CT with contrast B. Get a cerebral angiogram C. Spinal tap D. Ophthalmology consult E. Magnetic resonance venogram (MRV)

ANSWERS TO THIS SECTION CAN BE FOUND ON PAGE 24 19

66485457-66485438 www.ketabpezeshki.com

ZZakaria_87574_PTR_CH02_10-06-13_19-34.inddakaria_87574_PTR_CH02_10-06-13_19-34.indd 1919 66/19/2013/19/2013 8:44:428:44:42 PMPM CEREBROVASCULAR DISEASES

2. The spinal tap is done, and the cerebrospinal fl uid (CSF) shows xanthochromia. The patient undergoes a cerebral angiogram and a right posterior communicating aneurysm is clipped. Two days after surgery, the patient becomes less responsive but without focal defi cits. What do you suspect? A. Seizure B. Stroke C. Hydrocephalus D. Vasospasm E. Subdural hematoma (SDH)

3. The patient is more awake after an external ventric- ular drain (EVD) is placed; however, a few hours later she becomes lethargic with anisocoria and right hemiparesis. What is the cause of the neuro- logical change? A. Seizure B. Vasospasm C. Subdural hematoma D. Stroke E. Hyponatremia

4. What is not a risk factor for having a subarachnoid hemorrhage in this patient? A. Smoking B. Family history C. Hypertension D. Systemic Lupus Erythematosus (SLE) E. Alcohol

5. A 65-year-old male presents to the ED with headache and lethargy. His wife states that he has had a bad headache for the past 5 days. A CT scan shows acute subarachnoid hem- orrhage (SAH). An external ventricular drain is placed, and an anterior communicating aneurysm is coiled. Two days after the procedure, the patient’s sodium drops to 126 and he has a left pronator drift. What is the cause for his neurologic change? A. Seizure B. Vasospasm C. Rebleeding D. Stroke E. Hyponatremia

20

66485457-66485438 www.ketabpezeshki.com

ZZakaria_87574_PTR_CH02_10-06-13_19-34.inddakaria_87574_PTR_CH02_10-06-13_19-34.indd 2020 66/19/2013/19/2013 8:44:428:44:42 PMPM SUBARACHNOID HEMORRHAGE AND VASCULAR MALFORMATIONS: Questions

6. A cerebral angiogram with angioplasty is done and the patient improves. Over the next couple of days, the patient should be closely monitored for all of the following signs of vasospasm, except: A. Abulia B. Motor defi cit C. Aphasia D. Headaches E. Hyporefl exia

7. The risk for developing vasospasm is greatest with: A. Intracerebral hematoma B. Intraventricular hematoma and posterior fossa subarachnoid blood C. Subarachnoid blood less than 1 mm thickness D. Subarachnoid blood more than 1 mm thickness E. Peri-mesencephalic subarachnoid blood

8. The gold standard for diagnosing cerebral vasospasm is: A. Transcranial duplex (TCD) B. CT perfusion scan (CTP) C. CT angiogram (CTA) D. Cerebral angiogram E. MRI diffusion

9. Which of the following cardiac fi ndings is not associated with subarachnoid hemorrhage? A. Elevated troponin B. Short QT interval C. Elevated B-type natriuretic peptide (BNP) D. Sinus bradycardia E. Left ventricular dysfunction

10. Risk factors for rebleeding of an aneurysm include all of the following, except: A. Size of aneurysm B. Hunt and Hess grade at admission C. Fisher grade at admission D. Seizure at ictus E. SBP >160 mmHg

11. Which of the following drugs is preferred for controlling hypertension in subarachnoid hemorrhage (SAH)? A. Nicardipine B. Esmolol C. Labetalol D. Sodium nitroprusside E. Enalapril

21

66485457-66485438 www.ketabpezeshki.com

ZZakaria_87574_PTR_CH02_10-06-13_19-34.inddakaria_87574_PTR_CH02_10-06-13_19-34.indd 2121 66/19/2013/19/2013 8:44:438:44:43 PMPM CEREBROVASCULAR DISEASES

12. Cerebral aneurysm can typically present with all of the following, except: A. Seizure B. Hemiparesis C. Exophthalmus D. Horner’s syndrome E. Third nerve palsy with pupillary sparing F. Inferior nasal quadrantanopsia 13. According to the International Subarachnoid Aneurysm Trial (ISAT), all of the following are true, except: A. Anterior circulation aneurysms less than 7 mm have a 0% risk of rupture B. The risk for epilepsy was lower in the endovascular group C. Higher rebleeding rate was seen in the endovascular group D. Higher aneurysm occlusion rate was seen in the surgical group E. There was a reduction in risk of death or dependency in the endovascular group 14. All of the following are true regarding neurogenic pulmonary edema, except: A. Increases the incidence of cerebral vasospasm B. Can mimic congestive heart failure

C. Results in PaO2/FiO2 < 200 D. Can be seen within minutes of initial insult E. Should be treated with an α-agonist

15. All of the following are true regarding hyponatremia in subarachnoid hemorrhage, except: A. Hyponatremia is the most common electrolyte imbalance in subarachnoid hemorrhage (SAH) B. Hyponatremia is associated with a higher risk of vasospasm and poor prognosis C. Urine osmolality (UOsm) and urine sodium (UNa) levels can be used to distinguish cerebral salt wasting (CSW) and Syndrome of Inappropriate Antidiuretic Hormone (SIADH) D. Hyponatremia in SAH is hypotonic hyponatremia E. It usually occurs between day 3 and day 14 after rupture 16. The correct transducer position in monitoring cerebral perfusion pressure with an ele- vated head position is: A. Arterial line and intracranial pressure (ICP) monitor transducers at the level of the heart B. Arterial line and ICP monitor transducers at the level of the foramen of Monroe C. Arterial line transducer at the level of the heart and ICP monitor transducer at the level of the foramen of Monroe D. No difference as long as the arterial line and ICP monitor transducers are at the same level E. ICP monitor transducer should always be above the arterial line transducer 17. Which of the following vasopressors is used as a fi rst-line agent to augment blood pres- sure in cerebral vasospasm? A. Phenylephrine B. Epinephrine C. Dopamine D. Dobutamine E. Vasopressin

22

66485457-66485438 www.ketabpezeshki.com

ZZakaria_87574_PTR_CH02_10-06-13_19-34.inddakaria_87574_PTR_CH02_10-06-13_19-34.indd 2222 66/19/2013/19/2013 8:44:438:44:43 PMPM SUBARACHNOID HEMORRHAGE AND VASCULAR MALFORMATIONS: Questions

18. What is the strongest prognostic indicator for poor outcome in subarachnoid hemorrhage? A. Initial clinical grade B. Aneurysm rerupture C. Older age D. Global cerebral edema on initial CT scan E. Fever 19. All of the following are true regarding the use of nimodipine in the management of sub- arachnoid hemorrhage, except: A. Nimodipine is an L-type calcium channel blocker B. It has been shown to improve neurological outcome C. It can cause hypotension D. It improves cerebral vasospasm E. There is no difference in clinical effi cacy between intravenous and enteral nimodipine use 20. All of the following regarding continuous (cEEG) monitoring in subarachnoid hemorrhage (SAH) are true, except: A. cEEG changes can be seen up to 24 hours prior to onset of vasospasm B. Intracortical recording has less artifact compared to scalp recording C. Alpha–delta ratio (ADR) correlates better than mean amplitude for predicting vasospasm D. EEG can detect ischemia in the absence of infarction E. cEEG monitoring is particularly useful in poor grade patients 21. A patient presents with the CT scan shown. Which is the correct answer regarding this condition? A. Cerebral angiogram should always be done B. Will Never rebleed C. Does not cause vasospasm D. Needs long-term follow-up E. Has a rehemorrhage rate of 4% in the fi rst 24 hours

22. Which of the following is true regarding transcranial doppler (TCD) ultrasound in cere- bral vasospasm? A. Lindegaard ratio greater than 3 correlates with severe spasm B. TCD has good sensitivity in identifying patients with vasospasm C. It is a screening tool to assess peripheral vasculature D. It is most accurate for detecting spasm in the middle cerebral artery (MCA) E. It can detect vasospasm only in the anterior circulation 23

66485457-66485438 www.ketabpezeshki.com

ZZakaria_87574_PTR_CH02_10-06-13_19-34.inddakaria_87574_PTR_CH02_10-06-13_19-34.indd 2323 66/19/2013/19/2013 8:44:438:44:43 PMPM 2

ANSWERS

1. The answer is C. CT can detect greater than 95% of subarachnoid hemorrhages (SAHs) within 24 hours and 98% within 12 hours after rupture. Blood appears as high-density signal in the cisterns. A CT can look falsely negative, as in this case, if the volume of blood is very small, if the bleeding occurred several days prior, or if the hematocrit is extremely low. The next step in managing this patient is to perform a lumbar puncture to rule out blood in the cerebrospinal fl uid (CSF) space. The presence of xanthochromia is helpful in distinguishing true SAH from a traumatic tap. According to the guidelines (1,2), spectros- copy is the recommended method of analysis and should be done on the fi nal bottle. An increased bilirubin and oxyhemoglobin level is suggestive of SAH while a negative value practically rules it out. Xanthochromia can be detected if the patient presents greater than 4 hours after ictus. If less than 4 hours, the less reliable method of counting red blood cells (RBCs) in the CSF and comparing the clearance between bottles 1 and 4 may be employed. In a comparative study, the non-SAH group had a clearance rate of 70% and an RBC count in bottle 4 of less than 500, while patients with SAH cleared only 30% of their red cells with an RBC count anywhere between 900 and 2 million in bottle 4. The patient has ptosis from a third nerve palsy, suggesting a posterior communicating artery (PComm) aneu- rysm, in addition to blindness secondary to a concurrent vitreous hemorrhage (Terson’s syndrome). A magnetic resonance venography (MRV) is indicated if there is concern for a venous sinus thrombosis (3,4).

2. The answer is C. The patient is likely developing early hydrocephalus, which occurs in 20% to 30% of cases within the fi rst 48 hours. Delayed hydrocephalus (up to several weeks) (3) develops in one-fourth of surviving patients and is associated with older age, female gender, and intraventricular blood. There is no difference between patients treated by clipping or coiling. Poor clinical grade is the best predictor for the occurrence of hydro- cephalus. It is not typical for vasospasm to develop within 2 days. Vasopasm can be seen

24

66485457-66485438 www.ketabpezeshki.com

ZZakaria_87574_PTR_CH02_10-06-13_19-34.inddakaria_87574_PTR_CH02_10-06-13_19-34.indd 2424 66/19/2013/19/2013 8:44:438:44:43 PMPM SUBARACHNOID HEMORRHAGE AND VASCULAR MALFORMATIONS: Answers

as early as day 3, reaching a maximum incidence on days 6 to 8, and rarely as late as day 15 and beyond. A SDH should present earlier and usually with a focal defi cit like hemiparesis or dilated pupil. Stroke and seizure are unlikely, given the patient’s clinical presentation (5).

3. The answer is C. The patient has a rapid clinical change after placement of a ventricular drain. The clinical presentation is highly suggestive of an acute hemorrhage caused by the drain placement (6).

4. The answer is D (7).

5. The answer is B. The patient has signs and symptoms consistent with cerebral vasos- pasm as apparent on angiography below.

Cerebral Vasospasm on Angiography

Cerebral vasospasm usually occurs between days 3 to 15 post ictus. In some patients, it may persist beyond day 15. Clinical vasospasm must be differentiated from radio- graphic vasospasm. Although we observe angiographic vasospasm in up to 70% of patients, only 30% of these patients become clinically symptomatic, supporting the idea that additional factors play a role in delayed cerebral ischemia (DCI). DCI is a clinical diagnosis of a new neurologic defi cit in the setting of SAH. Cerebral vasospasm can cause DCI; however, not every patient with DCI has vasospasm and vice versa. Nimodipine is the only drug available to reduce long-term poor outcome (level 1 evi- dence). The CONSCIOUS 1 and 2 studies showed a reduction in cerebral vasospasm with the use of endothelin 1 antagonist; however, this did not translate into improved outcomes. The pathophysiology of DCI seems to be multifactorial at this time. Patients with SAH are at risk for DCI at any time within the fi rst 21 days. Triple H therapy has been widely used for the treatment of cerebral vasospasm; however, recent studies question the benefi t of hemodilution and hypervolemia. In patients refractory to medi- cal management, endovascular therapy has been utilized. The problem is the transient

25

66485457-66485438 www.ketabpezeshki.com

ZZakaria_87574_PTR_CH02_10-06-13_19-34.inddakaria_87574_PTR_CH02_10-06-13_19-34.indd 2525 66/19/2013/19/2013 8:44:438:44:43 PMPM CEREBROVASCULAR DISEASES

effect of the intervention. A randomized controlled trial comparing endovascular and conventional treatment of cerebral vasospasm is near completion (8). Statins have also been used to treat cerebral vasospasm with possible benefi cial effects, and a phase-3 trial is currently underway. A meta-analysis of existing data showed that statins reduced DCI and mortality (9). Statins decrease the glutamate-mediated excitotoxicity and moderate the infl ammatory response by upregulating cytokines and increasing nitric oxide (NO) levels. DCI has been the target of vigorous research, and many promising approaches like lumbar cerebrospinal fl uid (CSF) drainage, endothelin 1 antagonists (Conscious 1–3), IV Magnesium, Nicardipine pellets placed intraoperatively, and intrathecal thrombolytics and calcium channel blockers are being investigated (10–14).

6. The answer is E. All of the other options have been observed in patients with cerebral vasospasm. Abulia and lower extremity motor defi cits are seen in vasospasm involving the anterior cerebral artery. Aphasia and motor defi cit involving upper and lower extrem- ity is seen with middle cerebral artery (MCA) spasm. An increased tone is rather sugges- tive of vasospasm. Hyporefl exia is not usually a fi nding in cerebral vasospasm (15).

7. The answer is D. The relationship between CT imaging of subarachnoid blood and the risk of vasospasm was fi rst published by Miller Fisher in 1980 and is since used as the Fisher classifi cation (16). Twenty-three of 24 patients with localized clot or blood greater than 1 mm in thickness developed severe spasm and one patient, moderate vasospasm. Intraventricular or intracerebral hematoma had no effect on vasospasm. Of the fi ve patients in this group, three had no vasospasm and two had mild spasm. One patient had subarachnoid blood in the posterior fossa. Subarachnoid blood can be seen in a grade 4 hemorrhage as described by Fisher; however, it cannot be a signifi cant amount (16).

8. The answer is D. is the gold standard for diagnosing cerebral vasospasm. It is the best imaging study to evaluate large and small vessel disease and to initiate treatment at the same time. It is invasive and has a 1% complication rate. Therefore, alternative imaging tools are used more often. Transcranial duplex is a good noninvasive monitoring tool with high specifi city, particularly for the middle cerebral artery (MCA). However, it has low sensitivity and negative predictive value and is very operator depen- dent. CTA is a very promising tool if beam artifact and processing limitations can be over- come. CTA tends to overestimate the degree of spasm compared to conventional angio- gram and is less accurate for distal vasospasm. A recent meta-analysis using data from 10 studies showed 80% sensitivity and 93% specifi city for cerebral vasospasm. Computer tomographic perfusion (CTP) is of particular interest since we understand that angio- graphic vasospasm is only one of multiple factors contributing to delayed cerebral isch- emia. Measuring cerebral blood fl ow might provide more comprehensive information. Early CTP showed a signifi cant difference in cerebral blood fl ow, mean transit time, and cerebral blood volume in patients who developed symptomatic vasospasm. Late CTP had a specifi city of 91% and sensitivity of 95% compared to conventional angiogram. Early CTP may help in predicting vasospasm and late CTP fi ndings correlate highly with the development of cerebral vasospasm and delayed cerebral ischemia (12,17,18).

9. The answer is B. Subarachnoid hemorrhage is associated with many systemic complica- tions, among them cardiac dysfunction and the so-called stunned neurogenic myocardium. It is believed that increased intracranial pressure causes hypothalamic injury and release

26

66485457-66485438 www.ketabpezeshki.com

ZZakaria_87574_PTR_CH02_10-06-13_19-34.inddakaria_87574_PTR_CH02_10-06-13_19-34.indd 2626 66/19/2013/19/2013 8:44:448:44:44 PMPM SUBARACHNOID HEMORRHAGE AND VASCULAR MALFORMATIONS: Answers

of catecholamines. Classic changes after elevated levels of catecholamines are myocardial contraction band necrosis causing left ventricular dysfunction and a rise in troponin and BNP levels. Troponin is 100% sensitive in detecting LV dysfunction after subarachnoid hemorrhage (SAH) compared to MB isoenzyme of creatine kinase (CKMB) and reaches peak levels at day 1 to decay thereafter. Peak troponin and BNP levels have predictive value and are associated with higher mortality after SAH. Higher admission Hunt and Hess grade, intraventricular hemorrhage, loss of consciousness at ictus, and abnormal admission EKG changes are predictive of a higher troponin level. In 50% to 100% of cases, a variety of EKG changes along with arrhythmias can be found after SAH. These changes are transient and include ST elevation and depression, QT prolongation but not shortening, peaked or inverted T waves, large U waves, peaked P, and pathological Q waves. Clinically important cardiac arrhythmias, most often atrial fi brillation and fl utter but also sinus bra- dycardia, are associated with poor outcomes and can be seen in 4% of cases. An interest- ing phenomenon encountered in SAH is the apical ballooning syndrome, also known as Takotsubo cardiomyopathy. Regional wall motion abnormalities (RWMA) involving the apex with sparing of the base produces an LV dysfunction with a ventriculogram resem- bling a Japanese octopus-trapping pot. The motion defect can cause an apical LV thrombus with fatal outcome (19–23).

10. The answer is E. Rebleeding of an aneurysm has devastating consequences and is asso- ciated with higher mortality. Most studies show that the rerupture risk is highest in the fi rst 24 hours and particularly within the fi rst 6 hours after the ictus. The traditional rebleeding rate based on the International Cooperative Aneurysm study is 4% (24), but the actual rebleeding rate is likely much higher since the 4% refers to the population that reaches a tertiary care center and excludes patients in the fi eld or in community EDs. Ohkuma et al. reported a 13% prehospitalization rate of rebleeding (25). Aneurysm size greater than 1 cm along with poor initial neurological presentation and seizure at onset were risk factors for rebleeding. Controversy still exists regarding hypertension as a risk factor or the result of rebleeding. The 2012 AHA guidelines recommend an SBP < 160 preoperatively to balance the risk of rebleeding with maintaining cerebral perfusion pressure (level 2A). Along with early intervention (open or endovascular), antifi brin- olytic therapy has been used to prevent rebleeding (AHA level 2B evidence; 7,26–31).

11. The answer is A. The AHA has recommended in its guidelines to control hypertension using esmolol, labetalol, or nicardipine due to their short-acting nature and safety profi le. In a retrospective study, nicardipine was superior in controlling hypertension compared to labetolol. In addition, the blood pressure responded more rapidly to nicardipine. Hypotension also occurred more often in the labetolol group: 9% versus 3%. Nicardipine, a pure vasodilator (L-type calcium channel) should also be used for patients with COPD, congestive heart failure, or a history of bronchospasm. Esmolol and labetolol should be used if tachycardia needs to be treated along with hypertension. Nitroprusside should be avoided because of its ability to dilate intracerebral venous and arterial vasculature and raise intracranial pressure (ICP). It has an unreliable dose–response profi le, tends to cause rebound hypertension, and can cause cyanide toxicity. Enalapril is not suitable for emergency treatment because of its long duration (6–12 hours) and delayed onset (15–30 minutes) of action (32,33).

27

66485457-66485438 www.ketabpezeshki.com

ZZakaria_87574_PTR_CH02_10-06-13_19-34.inddakaria_87574_PTR_CH02_10-06-13_19-34.indd 2727 66/19/2013/19/2013 8:44:448:44:44 PMPM CEREBROVASCULAR DISEASES

12. The answer is E. Third nerve palsy can occur with a posterior communicating artery (PComm) aneurysm but usually involves the pupil. Pupillary involvement is almost pathognomonic for aneurysms, particularly if they are greater than or equal to 5 mm. Cases of pupillary sparing have been reported; however, this is usually the exception. The third nerve lies in the subarachnoid space close to the PComm, and the pupillary fi bers lie on the dorsomedial surface, making them more vulnerable to compression from aneurysms. In contrast, ischemic causes for third nerve palsies affect the central fi bers, sparing the pupil. Middle cerebral artery (MCA) aneurysms can present with temporal lobe seizures or hemiparesis, even if they are unruptured, due to mass effect. Carotid artery aneurysms, particularly of the ophthalmic segment but also anterior communicat- ing and cavernous sinus aneurysms, can present with various visual fi eld defi cits such as bitemporal hemianopsia, homonymous hemianopsia, and quadrantanopsia. Ophthalmic aneurysms present initially with an inferior nasal fi eld cut because of pressure from the overlying falciform ligament. Horner’s syndrome can occur with a cavernous segment aneurysm of the carotid artery compressing the postganglionic sympathetic pathway, while exophthalmus may be a symptom of a carotid cavernous fi stula after aneurysm rup- ture into the cavernous sinus. Posterior circulation aneurysms can present with Weber’s syndrome and other brainstem compression syndromes as well as third, sixth, and lower cranial nerve palsies. Unruptured anterior communicating artery aneurysms can present with dementia, abulia, or pituitary dysfunction from mass effect (34–37).

13. The answer is A. The rupture risk of an aneurysm according to size and location was published in the International Study of Unruptured Intracranial Aneurysms (ISUIA). The study enrolled and found that smaller aneurysms located in the anterior circulation had a lower rupture risk compared to larger aneurysms and aneurysms located in the posterior circulation. All the other answers are correct (38,39).

14. The answer is E. Neurogenic pulmonary edema is a fairly common pulmonary complica- tion and can be seen with any type of brain injury. The pathogenesis involves overacti- vation of the sympathetic nervous system. Two theories exist, and both probably play a role in the pathogenesis: the blast theory and the permeability theory. The sympathetic surge causes capillary leakage regardless of the systemic blood pressure. Edema is caused by damage to capillary endothelium, which contains both α- and β-adrenergic receptors. According to the blast theory, the systemic blood pressure surge shifts blood from the systemic circulation to the low-pressure pulmonary circulation, causing barotrauma to the capillary endothelium, damaging the alveolar membrane, and resulting in pulmo- nary edema. This can be seen within minutes, and symptoms often resolve after 24 to 48 hours unless there are persistently high intracranial pressures (ICPs) and ongoing CNS injury. The radiographic fi ndings are bilateral infi ltrates, more centrally distributed, with frothy blood-tinged sputum. The therapy is mainly supportive using mechani- cal ventilation as well as alpha-antagonists like phentolamine or beta-stimulating cate- cholamines to treat adrenergic-induced systemic vascular and pulmonary hypertension. Alternatively, some reports suggest dobutamine or dopamine to improve cardiac contrac- tility. Use of high positive end-expiratory pressure (PEEP) is controversial in the setting of elevated ICP. Several studies have shown that the incidence of cerebral vasospasm is increased with neurogenic pulmonary edema, and some studies have shown a higher mortality rate. These results are probably due to a higher initial Hunt and Hess grade,

28

66485457-66485438 www.ketabpezeshki.com

ZZakaria_87574_PTR_CH02_10-06-13_19-34.inddakaria_87574_PTR_CH02_10-06-13_19-34.indd 2828 66/19/2013/19/2013 8:44:448:44:44 PMPM SUBARACHNOID HEMORRHAGE AND VASCULAR MALFORMATIONS: Answers

less aggressive hemodynamic resuscitation and hypertensive treatment of these patients because of the cardiac and pulmonary dysfunction, and the possibility of worsening car- diac failure with hypervolemic or euvolemic therapy (40–43).

15. The answer is C. Hyponatremia is the most common electrolyte imbalance seen in SAH, occurring in 30% to 50% of SAH patients, and usually presents 3 to 14 days after aneu- rysm rupture. It can be seen more frequently with blood in the third ventricle, supra- sellar cistern and with anterior communication artery (AComm) aneurysm rupture. It can present as headache, nausea, vomiting, anorexia, lethargy, and if untreated, these patients may rapidly develop cerebral edema, experience seizures, and brain herniation. The differential diagnosis is broad and includes hormonal disorders (e.g., thyroid, car- diac, medications, and volume-related problems). It may be caused by either cerebral salt wasting (CSW) or SIADH in subarachnoid hemorrhage patients; however, prospective data showing decreased plasma volume and increased natriuretic peptide in SAH with hyponatremia favors CSW as the underlying cause. The pathogenesis of the former is still unknown, but it is believed to be caused by elevated natriuretic peptides mediated by the sympathetic system. Both cause hypotonic hyponatremia with elevated urine osmolal- ity greater than 200 mOsm/kg and urine Na greater than 25 mOsm/kg. The fl uid bal- ance in cerebral salt wasting is negative, and resuscitation is necessary. SIADH is associ- ated with expanded intravascular volume and fl uid restriction is the treatment. In the setting of SAH and vasospasm this is dangerous and this management is discouraged in practice. The use of mineralocorticoids has been shown to ameliorate hyponatremia. Fludrocortisone, because of its greater mineralocorticoid profi le, is the preferred form. The recommended rate of correction varies between 8 and 12 mmol/L in 24 hours. There is no conclusive data that shows that hyponatremia infl uences prognosis in SAH patients. In a prospective study with 298 patients, hypernatremia was independently associated with poor outcome at 3 months. This study, contrary to older literature, did not show an association of hyponatremia with vasospasm or poor outcome. The likely explanation is that in the past, patients were treated with fl uid restriction (44–47).

16. The answer is B. Incorrect placement and calibration of the arterial line transducer may overestimate cerebral perfusion pressure (CPP) by greater than 25%. According to a study, the arterial line transducer was at the ear level in only 10% of ICUs. When the patient’s head is elevated by 30°, both transducers must be placed at the foramen of Monroe for most accurate monitoring (48).

17. The answer is A. Phenylephrine is the drug preferred to elevate blood pressure in cere- bral vasospasm, although it should be used with caution in patients with coronary artery disease, glaucoma, and thyroid disease. It is an alpha-agonist elevating the blood pressure by vasoconstriction and may cause some degree of refl ex bradycardia. Phenylephrine has essentially no β-adrenergic receptor activity at clinically used intravenous doses. It has an immediate onset of action and the effect lasts for 20 to 40 minutes. Epinephrine is an α-1 and β-1 agonist, causes tachycardia, and should be used for patients with bra- dycardia from Cushing’s refl ex or where blood pressure and cardiac output needs to be augmented. Dopamine and dobutamine can also activate β-1 receptors, causing tachycar- dia. Dopamine in high doses is used in similar situations as norepinephrine. Dobutamine is preferred when the blood pressure is already elevated and cardiac output is low. It

29

66485457-66485438 www.ketabpezeshki.com

ZZakaria_87574_PTR_CH02_10-06-13_19-34.inddakaria_87574_PTR_CH02_10-06-13_19-34.indd 2929 66/19/2013/19/2013 8:44:448:44:44 PMPM CEREBROVASCULAR DISEASES

is not a strong vasopressor, because its β-2 action causes vasodilation counteracting its β-1 action, but it does have positive inotropic and chronotropic effects. Vasopressin causes vasoconstriction through AVPR1 receptor activity and has been shown to be effective in the management of refractory hypertensive therapy in conjunction with other vasopres- sors. It should be used with caution given its association with vasospasm and cerebral edema in animal models (33,49–52).

18. The answer is A. According to the ASA/AHA guidelines for the management of aneurys- mal subarachnoid hemorrhage, the severity of clinical presentation is the strongest prog- nostic indicator for subarachnoid hemorrhage (Class 1 evidence). Additional predictors for poor outcome are rebleeding, older age, pre-existing severe illness, hyperglycemia, fever, cerebral edema, symptomatic vasospasm, intraventricular or intracerebral hema- toma, and cerebral infarction (7).

19. The answer is D. Nimodipine is an L-type calcium channel blocker and is the only med- ication that has been shown to improve outcome in subarachnoid hemorrhage (SAH). Although the exact mechanism of this observation is not known, it does not appear to be through alleviation of cerebral vasospasm. Proposed hypotheses include blocking calcium-dependent excitotoxicity, antiplatelet aggregation activity, and dilation of lepto- meningeal or small dural collaterals not seen on angiogram. Adverse effects have also been described like hypotension, increased intracranial pressure (ICP), and decreased brain tissue oxygenation. The use of IV nimodipine did not show any benefi t compared to the enteral route (53–55).

20. The answer is A. The use of EEG to detect reversible ischemic changes was fi rst applied in the 1970s in carotid surgeries. Different parameters have since been investigated that correlate best with ischemic changes. An α–δ ratio is practical and correlates with delayed cerebral ischemia (DCI) and vasospasm. Changes can be seen up to 3 days prior, which allows for timely therapeutic interventions and is particularly benefi cial in poor grade patients with no clinical examination to follow. EEG monitoring in these patients is also benefi cial to detect nonconvulsive seizures (NCS), which can be seen in nearly 20% of patients after SAH. Intracortical recording seems to be more accurate in predicting vasos- pasm, is less prone to artifact, has better signal-to-noise ratio, and can be used for auto- mated detection. Loss of faster frequencies can be seen when cerebral blood fl ow (CBF) level decreases to 25 to 35 mL/g/min. As the CBF decreases further toward infarction threshold (10–12 mL/g/min), the EEG becomes silent and the damage becomes irrevers- ible (56–58).

21. The answer is A. Peri-mesencephalic subarachnoid hemorrhage accounts for approxi- mately 10% of patients with subarachnoid hemorrhage (SAH). Despite extensive workup, the bleeding source remains unknown in these patients. Hypothesized causes include venous anomaly, ruptured perforating artery, and capillary telangiectasia. By defi nition the bleeding is anterior to the midbrain with or without extension to the ambient cistern or basal part of the Sylvian cistern with incomplete fi lling of the interhemispheric fi ssure. Recognizing the correct bleeding pattern is important because non–peri-mesencephalic SAH with a negative angiogram has a more signifi cant clinical course. Patients with peri- mesencephalic SAH usually have a good clinical grade at presentation. They can develop

30

66485457-66485438 www.ketabpezeshki.com

ZZakaria_87574_PTR_CH02_10-06-13_19-34.inddakaria_87574_PTR_CH02_10-06-13_19-34.indd 3030 66/19/2013/19/2013 8:44:448:44:44 PMPM SUBARACHNOID HEMORRHAGE AND VASCULAR MALFORMATIONS: Answers

vasospasm with studies showing up to a 16% incidence. These patients therefore need admission to the ICU and a workup like any other subarachnoid hemorrhage. The clini- cal course, however, is much more benign and the ICU stay much shorter. Only one case report exists describing a patient with a recurrent hemorrhage. All patients with peri- mesencephalic SAH should undergo at least one 6-vessel cerebral angiogram at admis- sion. CT angiogram (CTA) or MR angiogram (MRA) is not recommended as an initial study. Controversy exists regarding a follow-up angiogram because of the low yield, and some authors do not recommend it. The patient does not need to be followed long term as good data exists suggesting an asymptomatic course (59,60).

22. The answer is D. The MCA is the most reliable vessel for accurately detecting vasospasm with TCD. Posterior circulation vasculature can also be measured, but less accurately than the anterior circulation. Lindegaard ratio is the ratio between MCA and extracranial inter- nal carotid artery (ICA) velocities, and a value greater than 6 correlates with severe cere- bral vasospasm. Transcranial duplex studies have poor sensitivity. Nearly 40% of patients who develop delayed cerebral ischemia failed to reach the velocity threshold of 120 cm/s. It is not useful for screening peripheral vasculature (61).

References

1. Gorchynski J, Oman J, Newton T. Interpretation of traumatic lumbar punctures in the setting of possible subarachnoid hemorrhage: who can be safely discharged? Cal J Emerg Med. 2007;8(1):3–7. 2. Tormey W, O’Shea P, Brennan P. National guidelines for analysis of cerebrospinal fl uid for bilirubin in suspected subarachnoid haemorrhage. Ann Clin Biochem. 2012;49 (Pt 1):102–103. 3. Diringer MN. Management of aneurysmal subarachnoid hemorrhage. Crit Care Med. 2009;37(2):432–440. 4. Beetham R. Recommendations for CSF analysis in subarachnoid haemorrhage. J Neurol Neurosurg Psychiatr. 2004;75(4):528. 5. Germanwala AV, Huang J, Tamargo RJ. Hydrocephalus after aneurysmal subarachnoid hemorrhage. Neurosurg Clin N Am. 2010;21(2):263–270. 6. Maniker AH, Vaynman AY, Karimi RJ, Sabit AO, Holland B. Hemorrhagic complications of external ventricular drainage. Neurosurgery. 2006;59(4 Suppl 2):ONS419–24; discussion ONS424. 7. Connolly ES Jr, Rabinstein AA, Carhuapoma JR, et al. Guidelines for the management of aneurysmal subarachnoid hemorrhage: a guideline for healthcare professionals from the American Heart Association/American Stroke Association. Stroke. 2012;43(6): 1711–1737. 8. Muroi C, Seule M, Mishima K, Keller E. Novel treatments for vasospasm after subarach- noid hemorrhage. Curr Opin Crit Care. 2012;18(2):119–126. 9. Tseng MY. Summary of evidence on immediate statins therapy following aneurysmal subarachnoid hemorrhage. Neurocrit Care. 2011;15(2):298–301. 10. Kramer AH, Fletcher JJ. Locally-administered intrathecal thrombolytics following aneu- rysmal subarachnoid hemorrhage: a systematic review and meta-analysis. Neurocrit Care. 2011;14(3):489–499.

31

66485457-66485438 www.ketabpezeshki.com

ZZakaria_87574_PTR_CH02_10-06-13_19-34.inddakaria_87574_PTR_CH02_10-06-13_19-34.indd 3131 66/19/2013/19/2013 8:44:448:44:44 PMPM CEREBROVASCULAR DISEASES

11. Weir B, Grace M, Hansen J, Rothberg C. Time course of vasospasm in man. J Neurosurg. 1978;48(2):173–178. 12. Washington CW, Zipfel GJ. Detection and monitoring of vasospasm and delayed cerebral ischemia: a review and assessment of the literature. Neurocrit Care. 2011;15(2):312–317. 13. Rabinstein AA, Lanzino G, Wijdicks EF. Multidisciplinary management and emerg- ing therapeutic strategies in aneurysmal subarachnoid haemorrhage. Lancet Neurol. 2010;9(5):504–519. 14. Schubert GA, Seiz M, Hegewald AA, et al. Hypoperfusion in the acute phase of subarach- noid hemorrhage. Acta Neurochir. 2011;110(1):35–39 15. Keyrouz SG, Diringer MN. Clinical review: Prevention and therapy of vasospasm in sub- arachnoid hemorrhage. Crit Care. 2007;11(4):220. 16. Fisher CM, Kistler JP, Davis JM. Relation of cerebral vasospasm to subarachnoid hemor- rhage visualized by computerized tomographic scanning. Neurosurgery. 1980;6(1):1–9. 17. Wintermark M, Dillon WP, Smith WS, et al. Visual grading system for vasospasm based on perfusion CT imaging: comparisons with conventional angiography and quantitative perfusion CT. Cerebrovasc Dis. 2008;26(2):163–170. 18. Greenberg ED, Gold R, Reichman M, et al. Diagnostic accuracy of CT angiography and CT perfusion for cerebral vasospasm: a meta-analysis. AJNR Am J Neuroradiol. 2010;31(10):1853–1860. 19. Naidech AM, Kreiter KT, Janjua N, et al. Cardiac troponin elevation, cardiovascular morbidity, and outcome after subarachnoid hemorrhage. Circulation. 2005;112(18): 2851–2856. 20. Tung PP, Olmsted E, Kopelnik A, et al. Plasma B-type natriuretic peptide levels are associated with early cardiac dysfunction after subarachnoid hemorrhage. Stroke. 2005;36(7):1567–1569. 21. Mayer SA, LiMandri G, Sherman D, et al. Electrocardiographic markers of abnor- mal left ventricular wall motion in acute subarachnoid hemorrhage. J Neurosurg. 1995;83(5):889–896. 22. Frontera JA, Parra A, Shimbo D, et al. Cardiac arrhythmias after subarachnoid hemor- rhage: risk factors and impact on outcome. Cerebrovasc Dis. 2008;26(1):71–78. 23. Lee VH, Oh JK, Mulvagh SL, Wijdicks EF. Mechanisms in neurogenic stress cardiomyop- athy after aneurysmal subarachnoid hemorrhage. Neurocrit Care. 2006;5(3):243–249. 24. Kassell NF, Torner JC. Aneurysmal rebleeding: a preliminary report from the Cooperative Aneurysm Study. Neurosurgery. 1983;13(5):479–481. 25. Ohkuma H, Tsurutani H, Suzuki S. Incidence and signifi cance of early aneurysmal rebleed- ing before neurosurgical or neurological management. Stroke. 2001;32(5):1176–1180. 26. Steiger HJ, Medele R, Brückmann H, Schroth G, Reulen HJ. Interdisciplinary manage- ment results in 100 patients with ruptured and unruptured posterior circulation aneu- rysms. Acta Neurochir (Wien). 1999;141(4):359–66; discussion 366. 27. Hillman J, Fridriksson S, Nilsson O, Yu Z, Saveland H, Jakobsson KE. Immediate administration of tranexamic acid and reduced incidence of early rebleeding after aneurysmal subarachnoid hemorrhage: a prospective randomized study. J Neurosurg. 2002;97(4):771–778. 28. Leipzig TJ, Redelman K, Horner TG. Reducing the risk of rebleeding before early aneurysm surgery: a possible role for antifi brinolytic therapy. J Neurosurg. 1997;86(2): 220–225.

32

66485457-66485438 www.ketabpezeshki.com

ZZakaria_87574_PTR_CH02_10-06-13_19-34.inddakaria_87574_PTR_CH02_10-06-13_19-34.indd 3232 66/19/2013/19/2013 8:44:448:44:44 PMPM SUBARACHNOID HEMORRHAGE AND VASCULAR MALFORMATIONS: Answers

29. Lord AS, Fernandez L, Schmidt JM, et al. Effect of rebleeding on the course and incidence of vasospasm after subarachnoid hemorrhage. Neurology. 2012;78(1):31–37. 30. Naidech AM, Janjua N, Kreiter KT, et al. Predictors and impact of aneurysm rebleeding after subarachnoid hemorrhage. Arch Neurol. 2005;62(3):410–416. 31. Guo LM, Zhou HY, Xu JW, Wang Y, Qiu YM, Jiang JY. Risk factors related to aneurysmal rebleeding. World Neurosurg. 2011;76(3–4):292–8; discussion 253. 32. Woloszyn AV, McAllen KJ, Figueroa BE, DeShane RS, Barletta JF. Retrospective evalua- tion of nicardipine versus labetalol for blood pressure control in aneurysmal subarach- noid hemorrhage. Neurocrit Care. 2012;16(3):376–380. 33. Rose JC, Mayer SA. Optimizing blood pressure in neurological emergencies. Neurocrit Care. 2004;1(3):287–299. 34. Rucker CW. The causes of paralysis of the third, fourth and sixth cranial nerves. Am J Ophthalmol. 1966;61(5 Pt 2):1293–1298. 35. Arle JE, Abrahams JM, Zager EL, Taylor C, Galetta SL. Pupil-sparing third nerve palsy with preoperative improvement from a posterior communicating artery aneurysm. Surg Neurol. 2002;57(6):423–6; discussion 426. 36. Batjer HH, Cerebrovascular Disease, Lippincott Williams & Wilkins, 1997. 37. Trobe JD, Glaser JS, Post JD. Meningiomas and aneurysms of the cavernous sinus: Neuro-ophthalmologic features. Arch Ophthalmol. 1978;96(3):457–467. 38. Wiebers DO, Whisnant JP, Huston J 3rd, et al. Unruptured intracranial aneurysms: nat- ural history, clinical outcome, and risks of surgical and endovascular treatment. Lancet. 2003;362(9378):103–110. 39. Molyneux AJ, Kerr RS, Yu LM, et al. International subarachnoid aneurysm trial (ISAT) of neurosurgical clipping versus endovascular coiling in 2143 patients with ruptured intra- cranial aneurysms: a randomised comparison of effects on survival, dependency, sei- zures, rebleeding, subgroups, and aneurysm occlusion. Lancet. 2005;366(9488):809–817. 40. Lakkireddigari SK, Durga P, Nayak M, Ramchandran G. Preoperative neurogenic pulmonary edema: A dilemma for decision making. J Anaesthesiol Clin Pharmacol. 2012;28(2):232–234. 41. Davison DL, Terek M, Chawla LS. Neurogenic pulmonary edema. Crit Care. 2012; 16(2):212. 42. Muroi C, Keller M, Pangalu A, Fortunati M, Yonekawa Y, Keller E. Neurogenic pulmo- nary edema in patients with subarachnoid hemorrhage. J Neurosurg Anesthesiol. 2008; 20(3):188–192. 43. Friedman JA, Pichelmann MA, Piepgras DG, et al. Pulmonary complications of aneurys- mal subarachnoid hemorrhage. Neurosurgery. 2003;52(5):1025–31; discussion 1031. 44. Hasan D, Wijdicks EF, Vermeulen M. Hyponatremia is associated with cerebral isch- emia in patients with aneurysmal subarachnoid hemorrhage. Ann Neurol. 1990;27(1): 106–108. 45. Qureshi AI, Suri MF, Sung GY, et al. Prognostic signifi cance of hypernatremia and hyponatremia among patients with aneurysmal subarachnoid hemorrhage. Neurosurgery. 2002;50(4):749–55; discussion 755. 46. Diringer MN, Bleck TP, Claude Hemphill J 3rd, et al. Critical care management of patients following aneurysmal subarachnoid hemorrhage: recommendations from the Neurocritical Care Society’s Multidisciplinary Consensus Conference. Neurocrit Care. 2011;15(2):211–240.

33

66485457-66485438 www.ketabpezeshki.com

ZZakaria_87574_PTR_CH02_10-06-13_19-34.inddakaria_87574_PTR_CH02_10-06-13_19-34.indd 3333 66/19/2013/19/2013 8:44:448:44:44 PMPM CEREBROVASCULAR DISEASES

47. Rabinstein AA, Wijdicks EF. Hyponatremia in critically ill neurological patients. Neurologist. 2003;9(6):290–300. 48. Nates JL, Niggemeyer LE, Anderson MB, Tuxen DV. Cerebral perfusion pressure moni- toring alert! Crit Care Med. 1997;25(5):895–896. 49. Miller JA, Dacey RG Jr, Diringer MN. Safety of hypertensive hypervolemic therapy with phenylephrine in the treatment of delayed ischemic defi cits after subarachnoid hemor- rhage. Stroke. 1995;26(12):2260–2266. 50. Dóczi T, László FA, Szerdahelyi P, Joó F. Involvement of vasopressin in brain edema for- mation: further evidence obtained from the Brattleboro diabetes insipidus rat with exper- imental subarachnoid hemorrhage. Neurosurgery. 1984;14(4):436–441. 51. Trandafi r CC, Nishihashi T, Wang A, Murakami S, Ji X, Kurahashi K. Participation of vasopressin in the development of cerebral vasospasm in a rat model of subarachnoid haemorrhage. Clin Exp Pharmacol Physiol. 2004;31(4):261–266. 52. Muehlschlegel S, Dunser MW, Gabrielli A, Wenzel V, Layon AJ. Arginine vasopressin as a supplementary vasopressor in refractory hypertensive, hypervolemic, hemodilutional therapy in subarachnoid hemorrhage. Neurocrit Care. 2007;6(1):3–10. 53. Stiefel MF, Heuer GG, Abrahams JM, et al. The effect of nimodipine on cerebral oxy- genation in patients with poor-grade subarachnoid hemorrhage. J Neurosurg. 2004; 101(4):594–599. 54. Soppi V, Karamanakos PN, Koivisto T, et al. A randomized outcome study of enteral ver- sus intravenous nimodipine in 171 patients after acute aneurysmal subarachnoid hemor- rhage. World Neurosurg. 2012;78(1–2):101–109. 55. Dumont AS, Tjoumakaris SI, Jabbour PM, Gonzalez LF, Rosenwasser RH. Intravenous versus enteral nimodipine in aneurysmal subarachnoid hemorrhage: is there an advan- tage? World Neurosurg. 2012;78(1–2):48–49. 56. Claassen J, Mayer SA, Kowalski RG, Emerson RG, Hirsch LJ. Detection of electro- graphic seizures with continuous EEG monitoring in critically ill patients. Neurology. 2004;62(10):1743–1748. 57. Stuart RM, Waziri A, Weintraub D, et al. Intracortical EEG for the detection of vasospasm in patients with poor-grade subarachnoid hemorrhage. Neurocrit Care. 2010;13(3):355–358. 58. Foreman B, Claassen J. Quantitative EEG for the detection of brain ischemia. Crit Care. 2012;16(2):216. 59. Kim YW, Lawson MF, Hoh BL. Nonaneurysmal subarachnoid hemorrhage: an update. Curr Atheroscler Rep. 2012;14(4):328–334. 60. Gross BA, Lin N, Frerichs KU, Du R. Vasospasm after spontaneous angiographically neg- ative subarachnoid hemorrhage. Acta Neurochir (Wien). 2012;154(7):1127–1133. 61. Carrera E, Schmidt JM, Oddo M, et al. Transcranial Doppler for predicting delayed cere- bral ischemia after subarachnoid hemorrhage. Neurosurgery. 2009;65(2):316–23; discus- sion 323.

34

66485457-66485438 www.ketabpezeshki.com

ZZakaria_87574_PTR_CH02_10-06-13_19-34.inddakaria_87574_PTR_CH02_10-06-13_19-34.indd 3434 66/19/2013/19/2013 8:44:448:44:44 PMPM SECTION II: NEUROTRAUMA 3 Neurotrauma Scott R. Shepard QUESTIONS

1. Indications for craniotomy for acute epidural hematoma (EDH) include all of the follow- ing except: A. Pupillary anisocoria B. Midline shift greater than 5 mm C. Hematoma volume greater than 30 cm3 D. Lack of a lucid interval E. Clot thickness greater than 15 mm

2. The lesion seen on the scan at right is most consistent with: A. Cerebellar contusion B. Cerebellar intracerebellar hematoma C. Posterior fossa subdural hematoma (SDH) D. Posterior fossa epidural hematoma (EDH)

ANSWERS TO THIS SECTION CAN BE FOUND ON PAGE 41 35

66485457-66485438 www.ketabpezeshki.com

ZZakaria_87574_PTR_CH03_10-06-13_35-50.inddakaria_87574_PTR_CH03_10-06-13_35-50.indd 3535 66/19/2013/19/2013 8:44:578:44:57 PMPM NEUROTRAUMA

3. The primary effect of mannitol in the treatment of elevated intracranial pressure (ICP) is: A. Osmotic fl uid removal from the brain B. Rheologic effects to change the shape of red blood cells and decrease blood viscosity C. Free-radical sequestration D. Decrease in the cerebral metabolic rate

4. Of the surgical techniques listed, which one is not indicated for the treatment of acute subdural hematomas (SDHs)? A. Burr hole evacuation B. Craniotomy with dural grafting and bone fl ap replacement C. Decompressive hemicraniectomy D. Osteoplastic craniotomy

5. All of the following statements regarding the older patient and acute subdural hematomas (SDHs) are true except: A. It is possible to predict death and low Glasgow Coma Scale (GCS) scores in older patients undergoing surgery for acute SDH on the basis of the admission eye opening score of the GCS B. Patients age 18 to 30 years with acute SDH undergoing surgery have a 25% overall mortality, whereas patients older than 50 years with acute SDH undergoing surgery have a 75% mortality rate C. Several multivariate analyses have failed to demonstrate age as an independent pre- dictor of outcome in older patients undergoing craniotomy for acute SDH D. In one study, 17 of 23 comatose patients with acute SDH older than 65 died, while the rest remained in a persistent vegetative state

6. All of the following are criteria to repair compound (open) depressed skull fractures except: A. Gross contamination B. Dural penetration C. Fragment depression greater than 3 mm D. Frontal sinus involvement E. Wound infection

7. Which of these statements regarding basilar skull fractures is not true? A. Basilar skull fractures occur in 7% to 16% of all patients with skull fractures B. These fractures are often indicated by the presence of pneumocephalus on head CT in the absence of open cranial vault fractures C. Prophylactic antibiotics have been demonstrated to reduce the risk of meningitis D. These skull fractures are frequently accompanied by periorbital ecchymoses (raccoon’s eyes) or postauricular ecchymoses (Battle’s sign) E. These fractures rarely require surgical intervention

36

66485457-66485438 www.ketabpezeshki.com

ZZakaria_87574_PTR_CH03_10-06-13_35-50.inddakaria_87574_PTR_CH03_10-06-13_35-50.indd 3636 66/19/2013/19/2013 8:44:578:44:57 PMPM NEUROTRAUMA: Questions

8. Which of the following statements regarding posttraumatic cerebrospinal fl uid (CSF) fi s- tulae is true? A. CSF otorrhea is more likely to resolve spontaneously than CSF rhinorrhea B. Posttraumatic CSF fi stulae occur in 50% of all patients with basilar skull fractures C. Less than one-third resolve spontaneously D. Meningitis occurs in less than 3% E. Most posttraumatic CSF fi stulae are not evident for 3 to 6 days after injury

9. Hypertonic saline has been demonstrated to: A. Increase cerebral blood fl ow more than mannitol on an equimolar basis B. Decrease intracranial pressure (ICP) more than mannitol on an equimolar basis C. Have a shorter duration of action than mannitol D. Have a greater risk of nephrotoxicity than mannitol

10. Which of the following is not a risk factor for blunt cerebrovascular injury? A. Displaced midface fracture (LeFort II or LeFort III) B. Basilar skull fracture with carotid canal involvement C. C7-T1 jumped facets and subluxation with spinal cord transection D. Closed head injury with diffuse axonal injury (DAI) and GCS < 6 E. Near hanging with anoxia

11. Use of corticosteroids in the treatment of acute spinal cord injury in adults has been dem- onstrated to: A. Dramatically increase motor function in patients 6 months after the injury B. Increase the risk of systemic complications, including wound infections, sepsis, and pneumonia C. Decrease the risk of systemic complications following acute spinal cord injury D. Demonstrate benefi t to patients if administration is initiated between 24 and 72 hours following acute spinal cord injury

12. A 21-year-old man is involved in a high-speed motor vehicle accident in which he strikes a telephone pole, resulting in an extended loss of consciousness. On arrival at the ICU, he is noted to have a GCS of 5T, with a motor score of 4 and intact brainstem refl exes. He is intubated with the vital signs of temperature of 98.3°F, HR 68 beats/minute, regular,

MAP 90 mmHg, and oxygen saturation of 99% with fraction of inspired oxygen (FiO2) of 40%. Urine drug screen is negative, and there is no detectable alcohol in his serum sample. Referring to his CT scan, the most likely diagnosis is: A. Traumatic encephalopathy B. Acute subdural hematoma (SDH) C. Brainstem contusion D. Diffuse axonal injury (DAI) E. Nonconvulsive status epilepticus

37

66485457-66485438 www.ketabpezeshki.com

ZZakaria_87574_PTR_CH03_10-06-13_35-50.inddakaria_87574_PTR_CH03_10-06-13_35-50.indd 3737 66/19/2013/19/2013 8:44:578:44:57 PMPM NEUROTRAUMA

13. Which of the following radiologic studies is the most sensitive in detecting diffuse axonal injury (DAI)? A. PET scanning B. Contrast-enhanced CT scan C. MRI T1-weighted sequences D. Xenon CT scan E. MRI diffusion-weighted imaging (DWI) sequences

14. Risk factors for increase in the size of traumatic cerebral contusions/hemorrhages include: A. Male sex B. Age older than 60 years C. Elevation of partial thromboplastin time (PTT) D. Deterioration of GCS after resuscitation E. All of the above

15. Which of these statements regarding prophylactic hypothermia in traumatic brain injury (TBI) is true? A. Prophylactic hypothermia has been clearly demonstrated to benefi t outcome from acute TBI in children B. Prophylactic hypothermia has a complication rate similar to other brain injury treat- ments in normothermic patients C. Prophylactic hypothermia decreases mortality, but does not improve outcome in adults with acute TBI D. Prophylactic hypothermia does not improve mortality or outcome in the treatment of adults and children with TBI

16. Prophylactic anticonvulsants in the setting of traumatic brain injury (TBI) have been demonstrated to: A. Decrease the incidence of early posttraumatic seizures B. Decrease the incidence of both early and late posttraumatic seizures C. Not alter the incidence of posttraumatic seizures but reduce the severity of post- traumatic seizures D. Decrease the incidence of late posttraumatic seizures

17. Which of the following statements is true regarding the role of decompressive hemi- craniectomy in the treatment of the patient with traumatic brain injury (TBI)? A. Randomized prospective trials have shown that decompressive hemicraniectomy decreases mortality and improves outcomes B. Decompressive hemicraniectomy may be effective in treating refractory intracranial pressure (ICP) C. Decompressive hemicraniectomy has no role in the treatment of increased ICP in the patient with TBI and should only be used to treat the malignant middle cerebral artery (MCA) syndrome D. Decompressive hemicraniectomy should never be performed in patients younger than 18 years

38

66485457-66485438 www.ketabpezeshki.com

ZZakaria_87574_PTR_CH03_10-06-13_35-50.inddakaria_87574_PTR_CH03_10-06-13_35-50.indd 3838 66/19/2013/19/2013 8:44:578:44:57 PMPM NEUROTRAUMA: Questions

18. Risk factors for the development of posttraumatic seizures include all of the following except: A. Depressed skull fracture B. Penetrating wound C. Intracranial hemorrhage (intracerebral, subdural, or epidural) D. GCS < 10 E. History of a lucid interval

19. Indications for intracranial pressure (ICP) monitoring include all of the following except: A. Severe alcohol withdrawal in the traumatic brain injury (TBI) patient B. GCS score 3 to 8 with an abnormal CT scan C. Normal head CT scan with any two of these features (age older than 40 years, unilat- eral or bilateral motor posturing, SBP < 90) D. Patients with abnormal head CT scans with GCS > 8 who are undergoing prolonged general anesthesia or pharmacologic muscle relaxants and will not have a neurologic examination that may be assessed

20. Treatments designed to decrease intracranial pressure (ICP) should begin when: A. ICP increases 10 mmHg from the pressure when the ICP monitor was placed B. There is a presence of any midline shift on the head CT C. ICP reaches a threshold of between 15 and 20 mmHg D. ICP reaches a threshold of between 20 and 25 mmHg

21. Ventriculostomy infections have been demonstrated to: A. Increase with the duration of use, and infection rates increase substantially after 7 days B. Occur in less than 3% of patients C. Occur less commonly in patients with intraventricular hemorrhage (IVH) D. Signifi cantly decrease with routine catheter exchange every 5 days

22. Which of the following is the least common complication after decompressive craniectomy (DC)? A. Blossoming of contusions/expansion of a contralateral mass lesion B. Infection C. Hydrocephalus D. Subdural hygroma

39

66485457-66485438 www.ketabpezeshki.com

ZZakaria_87574_PTR_CH03_10-06-13_35-50.inddakaria_87574_PTR_CH03_10-06-13_35-50.indd 3939 66/19/2013/19/2013 8:44:578:44:57 PMPM NEUROTRAUMA

23. A 35-year-old man is 3-week status post- decompressive hemicraniectomy, when he develops a new contralateral left upper-extremity monoparesis over sev- eral days. He is afebrile with a normal white count and denies any new trauma. Extensive laboratory evaluation is within normal limits. Electroencephalogram demonstrates expected slowing over the affected hemisphere and does not reveal any evidence of seizure. There has not been any clinical evidence of seizure. Referring to this repeat CT scan, the most likely diagnosis is: A. Postictal paralysis B. Hydrocephalus C. Conversion disorder D. Syndrome of the trephined

24. Which of the following statements regarding barbiturate-induced coma is true? A. Has been demonstrated to be benefi cial in improving patient outcome from severe traumatic injury when used in a prophylactic fashion B. Is less effective than mannitol in lowering increased intracranial pressure (ICP) C. Rarely induces systemic hypotension when infused at target rates D. Causes an unacceptable increase in core body temperature

25. Which of the following is not true regarding the complications associated with ventricu- lostomy placement? A. The risk of all hemorrhage after ventriculostomy placement is 7% B. The risk of clinically signifi cant hemorrhage after ventriculostomy placement is approximately 1% C. The risk of hemorrhage increases signifi cantly in patients whose INR is between 1.4 and 1.6 compared with patients whose INR is between 1.2 and 1.4 D. In children, ventriculostomy insertion is associated with a 3-fold higher risk of hemor- rhage compared with intraparenchymal monitor placement

40

66485457-66485438 www.ketabpezeshki.com

ZZakaria_87574_PTR_CH03_10-06-13_35-50.inddakaria_87574_PTR_CH03_10-06-13_35-50.indd 4040 66/19/2013/19/2013 8:44:578:44:57 PMPM 3

ANSWERS

1. The answer is D. The indications for surgery for an acute EDH include the following: hematoma size greater than 30 cm3, clot thickness greater than 15 mm, midline shift greater than 5 mm, and pupillary anisocoria in the presence of an EDH not due to other factors such as ocular trauma. The lucid interval occurs in less than 20% of patients with EDH and is not a factor in the decision for surgery in patients with EDH. Patients with EDH in whom the hematoma is less than 30 cm3, the clot is less than 15 mm in diameter, there is less than 5 mm of midline shift, and whose GCS is greater than 8 can be managed conservatively; however, they must be monitored very closely and undergo early repeat CT scan (1).

2. The answer is C. This lesion is a posterior fossa SDH. Hematomas in the posterior fossa in traumatic brain injury (TBI) are uncommon and occur in less than 3% of TBI patients. The most common posterior fossa hematoma seen in the setting of trauma is an epidural hematoma (EDH), accounting for less than 10% of all EDHs. Posterior fossa SDHs are even less common and account for only 0.5% to 2.5% of all SDHs, whereas posterior fossa intraparenchymal hematomas account for only 1.7% of all traumatic intraparenchymal hematomas. Although trauma-related posterior fossa hemorrhages are uncommon, they pose a particular hazard for patients because of the limited size of the posterior fossa and the tendency for these patients to rapidly deteriorate (2).

3. The answer is B. The primary mechanism of mannitol’s effect on ICP is through its ability to decrease blood viscosity by altering the shape of the red blood cells (rheologic effect) and is not through its osmotic effects. This change in viscosity improves cerebral blood fl ow, especially at the level of the microcirculation, resulting in a decrease in ICP. The rheologic effects of mannitol occur within minutes of mannitol administration, whereas the osmotic effects of mannitol are not apparent for 15 to 30 minutes after administration.

41

66485457-66485438 www.ketabpezeshki.com

ZZakaria_87574_PTR_CH03_10-06-13_35-50.inddakaria_87574_PTR_CH03_10-06-13_35-50.indd 4141 66/19/2013/19/2013 8:44:588:44:58 PMPM NEUROTRAUMA

Mannitol is also a free-radical scavenger. Mannitol does not directly affect the cerebral metabolic rate (3).

4. The answer is A. Acute SDHs cannot be effectively evacuated through burr holes. All of the other techniques involve removal of a large bone fl ap to allow wide access to the subdural space. If there is signifi cant intraoperative cerebral edema during craniotomy to remove a SDH, then the bone fl ap can be left off and the dura left open or augmented with a wide duroplasty to allow for cerebral expansion. Decompressive hemicraniec- tomy involves removing a very large craniotomy fl ap to greatly expand the volume of the intracranial space and improve intracranial compliance. Decompressive hemicraniec- tomy is used for the treatment of refractory intracranial hypertension in different condi- tions. The most common indications are traumatic brain injury (TBI), malignant middle cerebral artery (MCA) syndrome, and hemispheric stroke. Osteoplastic craniotomy is a form of craniotomy in which the muscle attachments to the bone fl ap are preserved dur- ing removal of the bone fl ap to maintain its blood supply.

5. The answer is A. Although there is a relationship between poor outcome and age as well as poor outcome and low GCS, it is not possible to predict death based on advanced age and poor presentation GCS. In a study by Kotwica and Brzezinski, a statistically signifi - cant difference in outcome at 3 months was demonstrated between younger patients and older patients (4). Patients aged 18 to 30 had a mortality rate of 25%, and patients older than 50 years with an acute SDH had a mortality rate of 75%. Although many studies have demonstrated a relationship between age and outcome in patients with acute SDH, several multivariate analyses have not demonstrated such a relationship (5,6).

6. The answer is C. Open depressed skull fractures require surgery for wound cleansing and repair of the osseous injury and scalp laceration. Frequently, there is gross contami- nation by foreign objects such as glass fragments or dirt. Any dural laceration should be repaired to minimize the risk of subsequent meningitis. Devitalized bone is often grossly contaminated and may need to be discarded. The degree of bone fragment displacement is not itself a criterion in the surgical management of open depressed skull fractures. The criteria to repair closed depressed skull fractures include fragment displacement greater than the width of the skull or cosmetic deformity.

7. The answer is C. Although the use of prophylactic antibiotics in basilar skull fractures is a common practice, it has not been demonstrated to decrease the risk of meningitis in these patients. Five randomized controlled trials failed to demonstrate that there was a reduction in the frequency of meningitis, mortality from all causes, and meningitis- related mortality in patients receiving prophylactic antibiotics for skull base fractures compared with those who did not receive prophylactic antibiotics. Current data do not support the use of prophylactic antibiotics in patients with closed basilar skull fractures. This includes patients with evidence of cerebrospinal fl uid (CSF) leakage. Basilar skull fractures occur in up to one-sixth of patients with skull fractures and rarely require sur- gical intervention. Pneumocephalus on CT scan is often an indicator of a basilar skull fracture, especially in the absence of a cranial vault fracture in proximity to the region of pneumocephalus. Periorbital ecchymoses (raccoon’s eyes) or postauricular ecchymoses (Battle’s sign) are common indicators of skull base fractures (7).

42

66485457-66485438 www.ketabpezeshki.com

ZZakaria_87574_PTR_CH03_10-06-13_35-50.inddakaria_87574_PTR_CH03_10-06-13_35-50.indd 4242 66/19/2013/19/2013 8:44:588:44:58 PMPM NEUROTRAUMA: Answers

8. The answer is A. Posttraumatic CSF leakage is a common problem in the traumatic brain injury (TBI) patient and occurs in 12% to 30% of patients with basilar skull frac- tures. The fractures most commonly associated with CSF leakage are frontal or ethmoid sinus fractures and longitudinal temporal bone fractures. Sixty percent of traumatic CSF leaks present within 48 hours of injury, and approximately 70% spontaneously resolve within 48 hours. CSF otorrhea has a higher spontaneous resolution rate than CSF rhin- orrhea. Some patients develop posttraumatic CSF leakage months to years after a TBI. Meningitis occurs in between 7% and 30% of all patients who develop a posttraumatic CSF leak. Streptococcus pneumoniae is the most frequent pathogen in patients developing meningitis after a posttraumatic CSF leak (8).

9. The answer is B. Hypertonic saline and mannitol are both effective in lowering ICP. In the study by Battison et al., nine patients received two treatments each of 200 mL of 20% mannitol and 100 mL of 7.5% saline, with 6% dextran-70 solution (HSD) in a random order. Median ICP reductions were greater using hypertonic saline solution versus man- nitol infusion, with the mean ICP reduction of 13 mmHg for hypertonic saline versus 7.5 mmHg reduction with mannitol (9). In another study by Vialet et al., 20 patients were assigned treatment with either 20% mannitol or 7.5% hypertonic saline solution in a ran- dom order, each given at a dose of 2 mL/kg. The frequency and duration of recurrent elevated ICP episodes were higher in patients treated with mannitol than in those treated with hypertonic saline (10,11).

10. The answer is C. There are two widely used guidelines for screening patients with blunt trauma for cerebrovascular injury: the Denver criteria and the Memphis criteria. According to the Denver criteria, the following are risk factors for blunt cerebrovascu- lar injury: LeForte II or III fracture pattern, cervical spine fracture or subluxation, basilar skull fracture with involvement of the carotid canal, DAI with GCS < 6, and near hanging with anoxic brain injury. The Memphis criteria list the following criteria as risk factors for blunt cerebrovascular injury: cervical spine fracture, LeFort II or LeForte III facial fracture, basilar skull fracture with involvement of the carotid canal, Horner’s syndrome, neuro- logic defi cit not explained by imaging studies, and neck soft-tissue injury (seatbelt sign, hematoma, or hanging). A fracture/dislocation at C7-T1 is generally not considered a risk factor for vertebral artery injury because the vertebral arteries enter the foramen trans- versarium at the level of C6 in 90% of patients and enter at the C7 level in the remaining 10%. Therefore a subluxation at C7-T1 is unlikely to result in a vertebral artery injury in the absence of other risk factors for cerebrovascular injury (12).

11. The answer is B. The use of ultra high-dose corticosteroids following acute, nonpene- trating spinal cord injury has not been demonstrated to signifi cantly improve outcomes in spinal cord injury patients in three separate randomized controlled clinical trials, the NASCIS. The most recent of these studies, NASCIS 3, examined megadose methylpred- nisolone (initial bolus 30 mg/kg, then 5.4 mg/kg infusion) for 24 or 48 hours in acute nonpenetrating spinal cord injury (13). There was no statistically signifi cant improvement in outcome or motor scores in the original analysis. In NASCIS 3, mortality from respira- tory morbidity was 6 times higher in the 48-hour group. There was a 2-fold increase in the incidence of severe pneumonia and a 4-fold increase in the incidence of severe sepsis in the 48-hour group compared with the 24-hour group. This was not statistically signifi cant,

43

66485457-66485438 www.ketabpezeshki.com

ZZakaria_87574_PTR_CH03_10-06-13_35-50.inddakaria_87574_PTR_CH03_10-06-13_35-50.indd 4343 66/19/2013/19/2013 8:44:588:44:58 PMPM NEUROTRAUMA

however, as the NASCIS 3 study was underpowered for this analysis. In NASCIS 2, there was a 2-fold increase in wound infections in the steroid group, and in NASCIS 1, there was a 4-fold increase in wound infection rates in the steroid group (14). The NASCIS stud- ies are part of a signifi cant controversy and have undergone extensive post hoc analyses that point to some benefi t in specifi c subgroups of patients. This post hoc analysis was used to justify treatment of spinal cord–injured patients with high-dose steroids. Nearly all of the major spine and trauma organizations now have position statements stating that the use of steroids in acute spinal cord injury is not indicated (15).

12. The answer is D. The most likely diagnosis is DAI. The most common cause of depressed consciousness in the traumatic brain injury (TBI) patient without metabolic or pharma- cologic alterations and without elevated intracranial pressure (ICP) or large structural brain injury is DAI. DAI is also known as shear injury and occurs most frequently when the brain accelerates or decelerates at a high speed with arc-type vectors through the axons. This leads to axonal disruption and neuronal dysfunction and often results in severe neurologic impairment with minimal evidence of injury on CT scan. Recovery from DAI is very variable, and at this time, there is no algorithm that can accurately predict outcomes.

13. The answer is E. DWI sequences of the MRI are the most sensitive commonly available MRI sequences for the detection of DAI-type injury; DWI sequences are more sensitive than FLAIR (fl uid attenuated inversion recovery) or T2-weighted images in identifying DAI lesions. Diffusor tensor imaging (DTI) of white matter tracts is highly sensitive to axonal injury and is able to detect DAI in the hyperacute phase. It also has a high negative predictive value in the diagnosis of axonal injury. Unfortunately, it is a study that requires signifi cant postprocessing and is not widely available in the clinical setting at this time. PET scanning and xenon CT scanning are metabolic studies and do not provide a reliable means of identifying DAI. CT scan has minimal sensitivity for DAI and demonstrates only a very small percentage of injured axons (16).

14. The answer is E. All are risk factors for the progression of traumatic cerebral con- tusions in acute head injury. Early progressive hemorrhage occurs in almost half of acutely head-injured patients who undergo a CT scan within 2 hours of injury. It is most common in cerebral contusions, and it is associated with intracranial pressure (ICP) elevations. The factors listed—male sex, age older than 60 years, early deteriora- tion of GCS after initial resuscitation, and elevated PTT—appear to be key determinants of hemorrhage progression. Although it is not possible to predict which patients will demonstrate clinically signifi cant progression of their cerebral contusions and there is no defi nitive guideline for the timing of follow-up CT scans in patients with cerebral contusions, it is important to understand who is at the highest risk for such progression and to obtain early repeat CT scans (17,18).

15. The answer is D. Induced hypothermia is one of the most investigated treatments in traumatic brain injury (TBI). Although some smaller studies have demonstrated some benefi t to prophylactic hypothermia in the treatment of TBI, several large randomized studies have failed to demonstrate any benefi t of prophylactic hypothermia on mortality in children or adults with TBI. In pooled data of some randomized controlled trials of

44

66485457-66485438 www.ketabpezeshki.com

ZZakaria_87574_PTR_CH03_10-06-13_35-50.inddakaria_87574_PTR_CH03_10-06-13_35-50.indd 4444 66/19/2013/19/2013 8:44:588:44:58 PMPM NEUROTRAUMA: Answers

hypothermia in adults, there is a tendency toward better Glasgow Outcome Scale scores in the hypothermia group. Individual randomized trials have failed to demonstrate this benefi t, and thus there is not a defi nitive improvement in outcome in TBI patients treated with prophylactic hypothermia. Several meta-analyses by the Cochrane Injuries Group have failed to demonstrate that hypothermia is a benefi cial treatment for TBI (19). The guidelines for the management of severe traumatic brain injury state that prophylactic hypothermia is an option for the treatment of TBI; however, it is not a Class I or Class II intervention (3). Hypothermia remains an option for the treatment of patients with refrac- tory intracranial hypertension following maximal medical therapy (20,21).

16. The answer is A. Posttraumatic seizures are a signifi cant problem in the TBI patient, occur- ring in between 4% and 25% of adults and in as many as one-third of children with TBI. Posttraumatic seizures may be divided into three groups: immediate seizures (occurring less than 24 hours after injury), early seizures (occurring less than a week after injury), and late seizures (occurring more than 7 days after injury). Prophylactic anticonvulsants in the setting of TBI have been demonstrated to decrease the risk of early posttraumatic seizures. In the landmark study by Temkin et al. (22), phenytoin was demonstrated in a randomized double-blind study to decrease the risk of early posttraumatic seizures from 14.2% to 3.6%. Unfortunately, treatment with phenytoin beyond a week did not decrease the rate of late posttraumatic seizures (23).

17. The answer is B. The routine use of decompressive hemicraniectomy to improve outcome in adults with severe TBI and refractory ICP elevation is not supported by any randomized clinical trials. For the pediatric population, DC reduces the risk of death and unfavorable outcome in one small pilot study in which there were limiting issues (24). This treatment may be justifi ed in patients younger than 18 years when medical treatment has failed to control ICP. In the adult population, the results of nonrandomized trials and controlled trials with historical controls suggest that DC is a useful option when sustained and maxi- mal medical treatment has failed to control ICP. In the recently published DECRA study from Australia, the following conclusion was published: “In adults with severe diffuse traumatic brain injury and refractory intracranial hypertension, early bifrontotemporopa- rietal decompressive craniectomy decreased intracranial pressure and the length of stay in the ICU but was associated with more unfavorable outcomes” (25). This study provides data that the use of up-front, early decompressive hemicraniectomy does not improve outcome in TBI. However, this study does not address the utility of DC in treating patients with elevated ICP refractory to maximal medical therapy, and DC remains a useful option in the treatment of these patients.

18. The answer is E. In the review by Temkin (26), the risk factors for development of seizures after the fi rst week following TBI included patients undergoing evacuation of a SDH or an intracerebral hematoma, GCS in the severe range of 3 to 8, early seizures, especially delayed early seizures, depressed skull fractures that were not surgically elevated, dural penetration by injury, at least one nonreactive pupil, and parietal lesions on CT scan. In a review by Frey, signifi cant risk factors for the development of seizures more than 7 days after TBI included seizures within the fi rst week, acute intracerebral hematoma (espe- cially SDH), brain contusion, increased injury severity, and age older than 65 years at the time of injury (27).

45

66485457-66485438 www.ketabpezeshki.com

ZZakaria_87574_PTR_CH03_10-06-13_35-50.inddakaria_87574_PTR_CH03_10-06-13_35-50.indd 4545 66/19/2013/19/2013 8:44:588:44:58 PMPM NEUROTRAUMA

19. The answer is A. Alcohol withdrawal itself is not a reason for placement of an ICP monitor. The Guidelines for the Management of TBI, published by the Brain Trauma Foundation, review this subject extensively and provide a consensus statement that there is strong level II evidence to support the use of ICP monitoring in patients with severe TBI (28). There is also level III evidence that any patients who have any two out of three features (age older than 40 years, unilateral or bilateral motor posturing, SBP < 90) should have an ICP monitor placed. ICP monitor placement should also be considered for any trauma patient with an abnormal head CT who will not have a neurologic examination that may be assessed on a frequent basis because of prolonged general anesthesia or extended use of skeletal muscle relaxants. These guidelines are not restrictive, and ICP monitoring can be used for any patient about whom there is a concern for elevated ICP due to radiographic or clinical features.

20. The answer is D. This is one of the most debated topics in neurosurgery and neurocriti- cal care. There is a study by Eisenberg et al., which is the only prospective, double-blind, placebo-controlled study, demonstrating improvements in outcome for lowering ICP (29). In this study, a threshold of 25 was used. The consensus position in the guidelines for the management of severe traumatic brain injury is that ICP-lowering treatments should be initiated at an ICP threshold of 20 to 25 mmHg (28). This is considered a level II recom- mendation on the basis of available evidence. It should be noted that occasional patients may herniate at ICPs less than 25 mmHg, especially when mass lesions greater than 20 mL are present in the cerebellum or temporal lobe. In addition, the ICP at which patients begin to experience deleterious effects is different for each patient, and some patients will not experience deleterious effects until their ICP is greater than 25 mmHg, especially if their cerebral perfusion is maintained in an acceptable range.

21. The answer is A. Infection rates after ventriculostomy placement are one of the most heav- ily studied and controversial issues in neurocritical care and neurosurgery. The published infection rates range from less than 1% to 45%. Most studies report between a 5% and 23% infection rate, depending on the criteria used to defi ne infections and the statistical methodology used. The key issue is that ventriculostomy infections are a signifi cant prob- lem and increase patient risk and cost signifi cantly. Several factors are well established with respect to ventriculostomy infections. They increase signifi cantly after a catheter has been in place for more than 7 days, and they are signifi cantly increased in patients with IVH. Routine catheter exchange after 5 to 7 days, a common practice, does not appear to decrease the infection rate (30–33).

22. The answer is B. In a comprehensive review by Stiver (34), infection after DC occurs in between 2% and 6% of patients. The most common complication after DC is the blossom- ing of a cerebral contusion, which can occur in up to 58% of patients. Subdural hygromas after DC occur in between 16% and 50% of patients, and hydrocephalus occurs in up to 29% of patients after DC. As many as one in four patients will have expansion of a contra- lateral mass lesion after DC. These are signifi cant complication rates and refl ect the severe degree of initial injury as well as the extensive procedure performed in this patient group.

23. The answer is D. This patient is experiencing the “syndrome of the trephined.” This is an uncommon and poorly understood phenomenon of the delayed onset of a neurological defi cit after decompressive craniectomy (DC). It can range from a mild neurologic defi cit

46

66485457-66485438 www.ketabpezeshki.com

ZZakaria_87574_PTR_CH03_10-06-13_35-50.inddakaria_87574_PTR_CH03_10-06-13_35-50.indd 4646 66/19/2013/19/2013 8:44:588:44:58 PMPM NEUROTRAUMA: Answers

to severe hemispheric dysfunction and alteration in consciousness. There are many theo- ries as to its etiology; however, there is no proven mechanism. It is frequently reversed by , although there often is a delay in improvement of symptoms. Given the lack of clinical or radiographic evidence of seizure, it is unlikely that this patient has a postictal paralysis. Although hydrocephalus is a common sequela of TBI, this patient does not have hydrocephalus. Conversion disorder is an unlikely diagnosis in a patient with a serious head injury (35,36).

24. The answer is B. Mannitol is more effective in lowering ICP than pentobarbital. In the study by Schwartz et al., 59 patients were treated in a prospective, randomized compari- son of pentobarbital and mannitol for the treatment of elevated ICP following TBI (37). In both patients with and without signifi cant intracranial hematomas, mannitol was more effective than pentobarbital in decreasing and controlling ICP. In a prospective random- ized study by Ward et al., prophylactic use of barbiturate coma failed to provide any improvement in outcome compared with the control group (38). Furthermore, barbitu- rate use caused clinically signifi cant hypotensive episodes in half of the patients. In the Cochrane review of barbiturate coma, approximately one in four patients developed clini- cally signifi cant hypotensive episodes (39). Hypotension is a major risk factor for poor outcome in TBI patients. Barbiturates cause a decrease in core body temperature, not an increase in temperature (40).

25. The answer is C. In a recent study, Bauer et al. demonstrated that there is no difference in the risk of hemorrhage after ventriculostomy insertion in patients with an INR between 1.2 and 1.6 compared with patients whose INR was less than 1.2 (41). The risk of hemor- rhage after ventriculostomy placement in a large meta-analysis by Bauer was reported as 7.0% from pooled data of studies with more than 25 patients, with most of the studies reporting a rate between 4.5% and 9.4% (42). The same study reported a clinically signifi - cant hemorrhage rate of 0.8%, with most studies reporting a rate between 0.2% and 1.4%. In a study by Anderson et al., the rate of any type of hemorrhage after ventriculostomy insertion in children was 17.6% compared with a 6.5% risk of hemorrhage after intra- parenchymal monitor placement (43).

References

1. Bullock MR, Chesnut R, Ghajar J, et al. Surgical management of acute epidural hemato- mas. Neurosurgery. 2006;58(3 Suppl):S7–S15; discussion Si. 2. Bullock MR, Chesnut R, Ghajar J, et al. Surgical management of posterior fossa mass lesions. Neurosurgery. 2006;58(3 Suppl):S47–S55; discussion Si. 3. Bratton SL, Chestnut RM, Ghajar J, et al. Guidelines for the management of severe traumatic brain injury. II. Hyperosmolar therapy. J Neurotrauma. 2007;24(Suppl 1): S14–S20. 4. Kotwica Z, Brzezinski J. Acute subdural haematoma in adults: an analysis of outcome in comatose patients. Acta Neurochir (Wien). 1993;121(3–4):95–99. 5. Bullock MR, Chesnut R, Ghajar J, et al. Surgical management of acute subdural hemato- mas. Neurosurgery. 2006;58(3 Suppl):S16–S24; discussion Si.

47

66485457-66485438 www.ketabpezeshki.com

ZZakaria_87574_PTR_CH03_10-06-13_35-50.inddakaria_87574_PTR_CH03_10-06-13_35-50.indd 4747 66/19/2013/19/2013 8:44:588:44:58 PMPM NEUROTRAUMA

6. Howard MA 3rd, Gross AS, Dacey RG Jr, Winn HR. Acute subdural hematomas: an age- dependent clinical entity. J Neurosurg. 1989;71(6):858–863. 7. Ratilal BO, Costa J, Sampaio C, Pappamikail L. Antibiotic prophylaxis for prevent- ing meningitis in patients with basilar skull fractures. Cochrane Database Syst Rev. 2011;8:CD004884. 8. Friedman JA, Ebersold MJ, Quast LM. Persistent posttraumatic cerebrospinal fl uid leak- age. Neurosurg Focus. 2000;9(1):e1. 9. Battison C, Andrews PJ, Graham C, Petty T. Randomized, controlled trial on the effect of a 20% mannitol solution and a 7.5% saline/6% dextran solution on increased intracranial pressure after brain injury. Crit Care Med. 2005;33(1):196–202; discussion 257. 10. Vialet R, Albanèse J, Thomachot L, et al. Isovolume hypertonic solutes (sodium chlo- ride or mannitol) in the treatment of refractory posttraumatic intracranial hyperten- sion: 2 mL/kg 7.5% saline is more effective than 2 mL/kg 20% mannitol. Crit Care Med. 2003;31(6):1683–1687. 11. Munar F, Ferrer AM, de Nadal M, et al. Cerebral hemodynamic effects of 7.2% hyper- tonic saline in patients with head injury and raised intracranial pressure. J Neurotrauma. 2000;17(1):41–51. 12. Arthurs ZM, Starnes BW. Blunt carotid and vertebral artery injuries. Injury. 2008;39(11): 1232–1241. 13. Bracken MB, Shepard MJ, Holford TR, et al. Administration of methylprednisolone for 24 or 48 hours or tirilazad mesylate for 48 hours in the treatment of acute spinal cord injury. Results of the Third National Acute Spinal Cord Injury Randomized Controlled Trial. National Acute Spinal Cord Injury Study. JAMA. 1997;277(20):1597–1604. 14. Bracken MB, Shepard MJ, Collins WF, et al. A randomized, controlled trial of meth- ylprednisolone or naloxone in the treatment of acute spinal-cord injury. Results of the Second National Acute Spinal Cord Injury Study. N Engl J Med. 1990;322(20): 1405–1411. 15. TRAUMA.ORG. Position Statement on Use of Steroids in Acute Spinal Cord Injury, 2005. http://www.trauma.org/archive/spine/steroids.html 16. Huisman TA, Sorensen AG, Hergan K, Gonzalez RG, Schaefer PW. Diffusion-weighted imaging for the evaluation of diffuse axonal injury in closed head injury. J Comput Assist Tomogr. 2003;27(1):5–11. 17. Chang EF, Meeker M, Holland MC. Acute traumatic intraparenchymal hemorrhage: risk factors for progression in the early post-injury period. Neurosurgery. 2006;58(4):647–56; discussion 647. 18. Oertel M, Kelly DF, McArthur D, et al. Progressive hemorrhage after head trauma: pre- dictors and consequences of the evolving injury. J Neurosurg. 2002;96(1):109–116. 19. Alderson P, Gadkary C, Signorini DF. Therapeutic hypothermia for head injury. Cochrane Database Syst Rev. 2004;4:CD001048. 20. Clifton GL, Miller ER, Choi SC, et al. Lack of effect of induction of hypothermia after acute brain injury. N Engl J Med. 2001;344(8):556–563. 21. Hutchison JS, Ward RE, Lacroix J, et al. Hypothermia therapy after traumatic brain injury in children. N Engl J Med. 2008;358(23):2447–2456. 22. Temkin NR, Dikmen SS, Wilensky AJ, Keihm J, Chabal S, Winn HR. A randomized, dou- ble-blind study of phenytoin for the prevention of post-traumatic seizures. N Engl J Med. 1990;323(8):497–502.

48

66485457-66485438 www.ketabpezeshki.com

ZZakaria_87574_PTR_CH03_10-06-13_35-50.inddakaria_87574_PTR_CH03_10-06-13_35-50.indd 4848 66/19/2013/19/2013 8:44:588:44:58 PMPM NEUROTRAUMA: Answers

23. Khan AA, Banerjee A. The role of prophylactic anticonvulsants in moderate to severe head injury. Int J Emerg Med. 2010;3(3):187–191. 24. Taylor A, Butt W, Rosenfeld J, et al. A randomized trial of very early decompressive craniectomy in children with traumatic brain injury and sustained intracranial hyperten- sion. Childs Nerv Syst. 2001;17(3):154–162. 25. Cooper DJ, Rosenfeld JV, Murray L, et al. Decompressive craniectomy in diffuse trau- matic brain injury. N Engl J Med. 2011;364(16):1493–1502. 26. Temkin NR. Risk factors for posttraumatic seizures in adults. Epilepsia. 2003;44(Suppl 10):18–20. 27. Frey LC. Epidemiology of posttraumatic epilepsy: a critical review. Epilepsia. 2003;44(Suppl 10):11–17. 28. Bratton SL, Chestnut RM, Ghajar J, et al. Guidelines for the management of severe trau- matic brain injury. XV. Steroids. J Neurotrauma. 2007;24(Suppl 1):S91–S95. 29. Eisenberg HM, Frankowski RF, Contant CF, Marshall LF, Walker MD. High-dose bar- biturate control of elevated intracranial pressure in patients with severe head injury. J Neurosurg. 1988;69(1):15–23. 30. Holloway KL, Barnes T, Choi S, et al. Ventriculostomy infections: the effect of monitoring duration and catheter exchange in 584 patients. J Neurosurg. 1996;85(3):419–424. 31. Beer R, Lackner P, Pfausler B, Schmutzhard E. Nosocomial ventriculitis and meningitis in neurocritical care patients. J Neurol. 2008;255(11):1617–1624. 32. Hoefnagel D, Dammers R, Ter Laak-Poort MP, Avezaat CJ. Risk factors for infections related to external ventricular drainage. Acta Neurochir (Wien). 2008;150(3):209–14; dis- cussion 214. 33. Harrop JS, Sharan AD, Ratliff J, et al. Impact of a standardized protocol and antibiotic- impregnated catheters on ventriculostomy infection rates in cerebrovascular patients. Neurosurgery. 2010;67(1):187–91; discussion 191. 34. Stiver SI. Complications of decompressive craniectomy for traumatic brain injury. Neurosurg Focus. 2009;26(6):E7. 35. Stiver SI, Wintermark M, Manley GT. Reversible monoparesis following decompressive hemicraniectomy for traumatic brain injury. J Neurosurg. 2008;109(2):245–254. 36. Akins PT, Guppy KH. Sinking skin fl aps, paradoxical herniation, and external brain tamponade: a review of decompressive craniectomy management. Neurocrit Care. 2008;9(2):269–276. 37. Schwartz ML, Tator CH, Rowed DW, Reid SR, Meguro K, Andrews DF. The University of Toronto head injury treatment study: a prospective, randomized comparison of pento- barbital and mannitol. Can J Neurol Sci. 1984;11(4):434–440. 38. Ward JD, Becker DP, Miller JD, et al. Failure of prophylactic barbiturate coma in the treat- ment of severe head injury. J Neurosurg. 1985;62(3):383–388. 39. Roberts I. Barbiturates for acute traumatic brain injury. Cochrane Database Syst Rev. 2000;2:CD000033. 40. Lee MW, Deppe SA, Sipperly ME, Barrette RR, Thompson DR. The effi cacy of barbiturate coma in the management of uncontrolled intracranial hypertension following neurosur- gical trauma. J Neurotrauma. 1994;11(3):325–331. 41. Bauer DF, McGwin G Jr, Melton SM, George RL, Markert JM. The relationship between INR and development of hemorrhage with placement of ventriculostomy. J Trauma. 2011;70(5):1112–1117.

49

66485457-66485438 www.ketabpezeshki.com

ZZakaria_87574_PTR_CH03_10-06-13_35-50.inddakaria_87574_PTR_CH03_10-06-13_35-50.indd 4949 66/19/2013/19/2013 8:44:588:44:58 PMPM NEUROTRAUMA

42. Bauer DF, Razdan SN, Bartolucci AA, Markert JM. Meta-analysis of hemorrhagic complications from ventriculostomy placement by neurosurgeons. Neurosurgery. 2011;69(2):255–260. 43. Anderson RC, Kan P, Klimo P, Brockmeyer DL, Walker ML, Kestle JR. Complications of intracranial pressure monitoring in children with head trauma. J Neurosurg. 2004;101(1 Suppl):53–58.

50

66485457-66485438 www.ketabpezeshki.com

ZZakaria_87574_PTR_CH03_10-06-13_35-50.inddakaria_87574_PTR_CH03_10-06-13_35-50.indd 5050 66/19/2013/19/2013 8:44:588:44:58 PMPM SECTION III: DISORDERS, DISEASES, SEIZURES, AND EPILEPSY 4 Seizures and Epilepsy Grant M. Warmouth QUESTIONS

1. A 63-year-old right-handed man is brought to the ED with acute onset of right-sided facial droop, right arm weakness (graded 2/5), and “halting speech.” According to his wife, the patient complained of a headache. He was cursing at the football game on television, when he started to stutter, make guttural noises, and then seemingly stared blankly and did not respond to her. She noticed that his right arm was shaking rhythmically. Maybe 5 minutes later, the shaking stopped and the patient started to make semipurposeful move- ments (reaching for objects with his left hand) but still appeared “dazed.” The patient has never had a prior stroke or head trauma, has never had any surgeries, has had no medical procedures done this year, and has had no recent complaints of melena or discolored urine. He takes no blood thinners, and his prothrombin time (PT)/partial thromboplastin time (PTT)/International Normalized Ratio (INR) is normal. His serum glucose is 130, his plate- lets are 220 (normal), and his blood BP is 160/89 currently (with similar readings obtained by EMS in the fi eld). The initial head CT scan is unremarkable, and in particular, no intrac- erebral or subarachnoid hemorrhage is seen. During your interview in the ED, you observe the patient halt purposeful movements and stare blankly, and you witness subtle shaking of the right arm for about 20 seconds. His nurse tells you that he has seemed “spaced out” and sleepy during his brief period in the ED. The next most appropriate step is: A. Give the patient tissue plasminogen activator (tPA) per standard protocol and arrange for him to be admitted to the Neuro-ICU for monitoring B. Do a lumbar puncture and send cerebrospinal fl uid (CSF) for WBC count, RBC count, glucose, protein, herpes simplex virus-1 (HSV-1) poymerase chain reaction (PCR), HSV-2 PCR, and Gram stain C. Load the patient with phenytoin and arrange for continuous EEG monitoring in the Neuro-ICU D. Do an urgent MRI to evaluate for acute cerebral infarction

ANSWERS TO THIS SECTION CAN BE FOUND ON PAGE 63 51

66485457-66485438 www.ketabpezeshki.com

ZZakaria_87574_PTR_CH04_10-06-13_51-70.inddakaria_87574_PTR_CH04_10-06-13_51-70.indd 5151 66/19/2013/19/2013 8:45:258:45:25 PMPM DISORDERS, DISEASES, SEIZURES, AND EPILEPSY

2. A patient has just been admitted to your ICU who presented to the ED with status epilep- ticus (SE) and no further history. The seizure semiology was reported as generalized tonic– clonic seizures per EMS and ED staff. The patient’s temperature is 38.0°C, HR 99, BP 140/77,

RR 14, and SpO2 97% on 2 L NC. There are two peripheral IVs in place. Lab tests show no electrolyte abnormalities, normal blood urea nitrogen and creatinine, normal liver function tests, and glucose of 135, and the urine toxicology screen results are pending. The patient received a total of 8 mg of lorazepam IV in the ED and was loaded with fosphenytoin at 20 mg/kg IV (at 150 mg/minute). Owing to continued brief seizure activity, a subsequent dose of fosphenytoin 10 mg/kg was administered; this fosphenytoin fi nished infusing about 3 minutes ago. The patient now begins to arch his back and assume tonic posturing and then proceeds over the next 20 seconds to generalized clonic activity. You have ordered loraze- pam 2 mg IV × 1 STAT. You plan to proceed with which of the following medications? A. Fosphenytoin 20 mg/kg IV B. Thiamine 100 mg IV C. Midazolam 0.4 mg/kg IM D. Intubate the patient and start an IV propofol infusion

3. A 23-year-old male patient is known to have juvenile absence epilepsy. He takes lamotrig- ine as an outpatient. He is admitted to the Neuro-ICU with subacute onset of confusion. He is suspected to be in nonconvulsive status epilepticus (NCSE) and hooked up for con- tinuous EEG (cEEG) monitoring. The EEG shows irregular generalized spike and wave activity at 3.0 Hz to 3.5 Hz waxing and waning throughout the entirety of the fi rst 20 min- utes the patient is monitored. At this point, the patient is correctly diagnosed as being in absence status epilepticus (SE). Which of the following medications should be avoided? A. Lorazepam B. Diazepam C. Phenytoin D. Keppra E. Valproic acid 4. An adult patient is unresponsive, has absent brainstem refl exes, and demonstrates apnea on a standardized apnea test. In the United States, these clinical criteria establish the diag- nosis of brain death contingent upon which of the following? A. MRI or CT of the brain shows an acute central nervous system (CNS) catastrophe con- sistent with brain death B. Patient’s temperature is above 32°C (89.6°F) C. Evidence of drug intoxication (illicit or iatrogenic) and poisoning has been sought and ruled out D. There are no metabolic derangements or endocrine abnormalities to explain the clinical setting E. An EEG done per brain-death protocol shows electrocerebral silence F. Tibial nerve somatosensory-evoked potentials (SEPs) show absent cortical responses G. Answers A, B, C, and D are correct H. Answers A, B, C, D, and E are correct I. None of the above answers are correct. Brain death can be established based solely on the clinical fi ndings of coma, absent brainstem refl exes, and demonstrated apnea on a standardized apnea test

52

66485457-66485438 www.ketabpezeshki.com

ZZakaria_87574_PTR_CH04_10-06-13_51-70.inddakaria_87574_PTR_CH04_10-06-13_51-70.indd 5252 66/19/2013/19/2013 8:45:258:45:25 PMPM SEIZURES AND EPILEPSY: Questions

5. An adult patient is unresponsive and has absent brainstem refl exes. A standardized apnea test was not able to be successively completed because of patient decompensation. The patient had an episode of ventricular fi brillation (following a cervical spine ) with a prolonged 40-minute course of CPR. A CT scan of the head done 18 hours later showed diffuse loss of the gray–white matter junction. It is now 48 hours after the initial cardiac abnormality. A routine bedside EEG is ordered, and a sample is shown below. The entire 20-minute EEG appeared consistent with this sample. This EEG: A. Is not consistent with the clinical diagnosis of brain death because of preserved EEG activity B. Is consistent with the clinical diagnosis of brain death (but it is not suffi cient as a confi r- matory test for brain death, as the duration was only 20 minutes and less than 16 EEG channels were used) C. Shows generalized approximately 1-Hz spikes, consistent with seizure activity D. Is a fi nding typically seen with anesthetic doses of propofol

Source: Courtesy of Giridhar Kalamangalam, MD, Department of Neurology, University of Texas, Houston.

6. Relevant special requirements for a brain death EEG protocol include: A. The EEG gain should be recorded and interpreted at a gain of no less than 2 µV/mm B. The EEG should show at least 30 minutes of interpretable data C. During the EEG, the technician should test EEG reactivity to external stimuli, espe- cially noxious excitations D. Following resuscitation from circulatory arrest, at least 8 hours should pass from the onset of coma until the EEG examination E. All of the above

53

66485457-66485438 www.ketabpezeshki.com

ZZakaria_87574_PTR_CH04_10-06-13_51-70.inddakaria_87574_PTR_CH04_10-06-13_51-70.indd 5353 66/19/2013/19/2013 8:45:258:45:25 PMPM DISORDERS, DISEASES, SEIZURES, AND EPILEPSY

7. An adult patient meets all the clinical diagnostic criteria for brain death. An EEG is done as per brain death standard protocol, which shows electrocerebral inactivity (ECI). Approximately what percentage of clinically brain-dead patients show this EEG pattern? A. 100% B. 90% C. 80% D. 50% E. 25% F. Less than 10%

8. A 29-year-old woman with Takayasu’s arteritis presented to the ED 6 hours after the onset of lethargy and somnolence. In the ED, she was unresponsive and had sluggish pupil- lary responses, bilateral oculomotor palsies, quadriparesis, and weak fl exor responses to noxious stimuli in all four limbs. An MRI revealed bilateral midbrain infarctions. She was admitted to the Neuro-ICU, and an EEG within the fi rst 24 hours is shown below. During the EEG, the patient was not responsive to noxious stimuli in all four limbs or to loud auditory stimuli. What type of pattern does this EEG show, and what are the implications for recovery? A. Spindle coma, generally a favorable prognosis for meaningful neurologic recovery B. Spindle coma, generally a very poor prognosis for meaningful neurologic recovery C. Stage II sleep, excellent neurologic recovery D. Alpha coma, excellent neurologic recovery

Source: Courtesy of Hirsch LJ, Brenner RP. Atlas of EEG in Critical Care. West Sussex, UK: Wiley-Blackwell, 2010: Figure 2.29, p. 78.

54

66485457-66485438 www.ketabpezeshki.com

ZZakaria_87574_PTR_CH04_10-06-13_51-70.inddakaria_87574_PTR_CH04_10-06-13_51-70.indd 5454 66/19/2013/19/2013 8:45:258:45:25 PMPM SEIZURES AND EPILEPSY: Questions

9. A 62-year-old man with hypertension and diabetes is admitted to the ICU after cardiac arrest. The time period from cardiac arrest to initiation of CPR is unknown. CPR was performed for 35 minutes before a sustained sinus rhythm was acquired. Approximately 24 hours after admission, the patient remains unresponsive to all noxious stimuli and is on no paralytic or sedative medications. During the EEG, the patient is not responsive to noxious stimuli in all four limbs or to loud auditory stimuli. The EEG shows the following pattern. What type of pattern does this EEG show, and what are its implications? A. Beta coma, good prognosis for meaningful neurologic recovery B. Delta–theta coma, poor prognosis for meaningful neurologic recovery C. Spindle coma, good prognosis for meaningful neurologic recovery D. Alpha coma, poor prognosis for meaningful neurologic recovery

Source: Courtesy of Giridhar Kalamangalam, MD, Department of Neurology, University of Texas, Houston.

10. The causes of a diffuse encephalopathy are multiple and diverse. In general, the EEG fi ndings seen in a diffuse encephalopathy are typically nonspecifi c for etiology, but there is a typi- cal progression of nonspecifi c EEG fi ndings that correlate with worsening encephalopathy. Which of the following EEG patterns is most consistent with a mild diffuse encephalopathy? A. A generalized diffuse burst-suppression (BS) pattern B. Frontal intermittent rhythmic delta activity (FIRDA) seen upon a background of dif- fuse delta–theta activity C. On a 48-hour recording, no variability in EEG pattern. No normal sleep activity or state changes are seen. D. Slowing of the occipital dominant rhythm (“alpha-rhythm”) to a poorly sustained, reactive 7 Hz. There are fragmentary runs of generalized 4 to 5 Hz theta activity.

55

66485457-66485438 www.ketabpezeshki.com

ZZakaria_87574_PTR_CH04_10-06-13_51-70.inddakaria_87574_PTR_CH04_10-06-13_51-70.indd 5555 66/19/2013/19/2013 8:45:258:45:25 PMPM DISORDERS, DISEASES, SEIZURES, AND EPILEPSY

11. A 53-year-old woman presented with a 5-day history of fevers (102°F), headache, con- fusion, and bizarre behavior. Examination is remarkable for slowing of mentation, and the patient appears hypomanic. An EEG done upon admission is shown below. What is the abnormal pattern shown on this EEG, and is it related to the patient’s current presentation? A. Frequent left temporal spikes; not related to current presentation. Patient likely has a long history of localization-related epilepsy. B. Periodic lateralized epileptiform discharges (PLEDs) in the left temporal lobe; not related to current presentation. PLEDs are often seen in chronic disorders, and the patient has an acute illness. C. Frequent left temporal spikes; likely related to current presentation. Patient likely has an acute illness, leading to exacerbation of an underlying epileptic focus in the left temporal lobe. D. PLEDs in the left temporal lobe; due to current presentation. PLEDs are a manifesta- tion of acute cerebral injury.

Source: Courtesy of Giridhar Kalamangalam, MD, Department of Neurology, University of Texas, Houston.

56

66485457-66485438 www.ketabpezeshki.com

ZZakaria_87574_PTR_CH04_10-06-13_51-70.inddakaria_87574_PTR_CH04_10-06-13_51-70.indd 5656 66/19/2013/19/2013 8:45:268:45:26 PMPM SEIZURES AND EPILEPSY: Questions

12. The following EEG could be seen in which of these clinical settings? A. After appropriate treatment for convulsive status epilepticus (SE) B. Lithium toxicity C. Creutzfeldt–Jakob disease D. All of the above E. None of the above

Source: Courtesy of Giridhar Kalamangalam, MD, Department of Neurology, University of Texas, Houston.

57

66485457-66485438 www.ketabpezeshki.com

ZZakaria_87574_PTR_CH04_10-06-13_51-70.inddakaria_87574_PTR_CH04_10-06-13_51-70.indd 5757 66/19/2013/19/2013 8:45:278:45:27 PMPM DISORDERS, DISEASES, SEIZURES, AND EPILEPSY

13. A 55-year-old man with cirrhosis is admitted to the Neuro-ICU with a 3-day history of fl uctuating mental status. Per history, there have been periods of alert, confused inter- activity alternating with periods of somnolence. Complex partial seizures with postictal confusion, nonconvulsive status epilepticus (NCSE), and metabolic encephalopathy are on the differential. An EEG is done (shown below). What are these EEG complexes? A. Periodic lateralized epileptiform discharges (PLEDs) B. Frontal intermittent rhythmic delta activity (FIRDA) C. Spike-slow wave complexes D. Triphasic waves

Source: Courtesy of Giridhar Kalamangalam, MD, Department of Neurology, University of Texas, Houston.

58

66485457-66485438 www.ketabpezeshki.com

ZZakaria_87574_PTR_CH04_10-06-13_51-70.inddakaria_87574_PTR_CH04_10-06-13_51-70.indd 5858 66/19/2013/19/2013 8:45:278:45:27 PMPM SEIZURES AND EPILEPSY: Questions

14. A 25-year-old man sustained right frontal head injury in an automobile accident. The patient has been in the Neuro-ICU for 24 hours, and he is being monitored with continuous EEG (cEEG). The following fi nding was seen on a routine EEG. No clinical changes were seen in the patient during the EEG. What does this EEG show? A. Nonconvulsive focal right frontal-temporal seizure B. Right frontal-temporal breach rhythm C. Mu rhythm (benign normal variant) D. Electrode artifact at F4

Source: Courtesy of Giridhar Kalamangalam, MD, Department of Neurology, University of Texas, Houston.

15. A 45-year-old man suffered cardiac arrest and remains comatose 3 days after resuscita- tion. He is normothermic, is on no sedative or paralytic medications, and shows absent brainstem refl exes. Defi nitive clinical determination of brain death has not been possible because the patient deteriorates prior to completion of apnea testing. The patient has not had myoclonic status epilepticus (SE). Median nerve somatosensory-evoked potential (SEP) testing is done. Which of the following fi ndings on SEP testing 3 days after cardiac arrest is predictive of a poor neurologic outcome? A. Bilaterally absent N9 potentials B. Bilaterally absent N13 responses C. Bilaterally absent N20 responses D. SEPs have not been established as useful for predicting neurologic outcome in this clinical setting

59

66485457-66485438 www.ketabpezeshki.com

ZZakaria_87574_PTR_CH04_10-06-13_51-70.inddakaria_87574_PTR_CH04_10-06-13_51-70.indd 5959 66/19/2013/19/2013 8:45:288:45:28 PMPM DISORDERS, DISEASES, SEIZURES, AND EPILEPSY

16. The following EEG pattern is commonly seen in comatose patients. This pattern is associ- ated with which of these clinical settings? A. Hypothermia B. Hyperthermia C. Untreated convulsive status-epilepticus D. Untreated nonconvulsive status-epilepticus E. Creutzfeldt–Jakob disease

Source: Courtesy of Giridhar Kalamangalam, MD, Department of Neurology, University of Texas, Houston.

60

66485457-66485438 www.ketabpezeshki.com

ZZakaria_87574_PTR_CH04_10-06-13_51-70.inddakaria_87574_PTR_CH04_10-06-13_51-70.indd 6060 66/19/2013/19/2013 8:45:288:45:28 PMPM SEIZURES AND EPILEPSY: Questions

17. A 61-year-old woman is undergoing left carotid endarterectomy (CEA). EEG is monitored continuously during the procedure. The baseline EEG shows symmetric hemispheric amplitudes and EEG frequencies. Three minutes after cross clamping of the left internal carotid artery, the EEG appears as below. Based on these fi ndings, the surgeon should be advised of which of the following? A. The patient has experienced a new left middle cerebral artery (MCA) distribution infarct; proceed with the surgery without further intervention B. The surgery is progressing as expected; there are no surprising fi ndings on EEG, as left hemispheric amplitude decrement is anticipated after left carotid cross clamping C. The patient is having a seizure; lorazepam should be administered D. There is marked and signifi cant new amplitude decrement over the left hemisphere, and arterial shunting should be done to prevent irreversible ischemia to the left cere- bral hemisphere

Source: Courtesy of Giridhar Kalamangalam, MD, Department of Neurology, University of Texas, Houston.

61

66485457-66485438 www.ketabpezeshki.com

ZZakaria_87574_PTR_CH04_10-06-13_51-70.inddakaria_87574_PTR_CH04_10-06-13_51-70.indd 6161 66/19/2013/19/2013 8:45:298:45:29 PMPM DISORDERS, DISEASES, SEIZURES, AND EPILEPSY

18. Nonconvulsive status epilepticus (NCSE) in the ICU setting is: A. Associated with no signifi cant morbidity or mortality beyond that typically seen with a generalized tonic–clonic seizure lasting 2 minutes B. Associated with high mortality (>50%) regardless of NCSE duration C. Associated with low mortality (<10%) regardless of NCSE duration D. Associated with higher morbidity and mortality with longer NCSE duration 19. A patient with a right frontal subdural hemorrhage might be expected to show which of the following patterns on EEG? A. Focal slowing, attenuation of faster frequencies, and lower amplitudes in the right frontal region B. Intermittent right frontal sharp waves C. Right frontal periodic lateralized epileptiform discharges (PLEDs) D. Seizures originating from the right frontal region E. All of the above 20. Which of the following clinical scenarios would be appropriate for continuous EEG (cEEG) monitoring in the ICU? A. A patient with fl uctuating mental status and concern for nonconvulsive status epilepticus (NCSE) B. A patient presenting with convulsive status epilepticus (SE), now somnolent after appropriate treatment in the ED C. A patient with acute subarachnoid hemorrhage D. A patient with a large acute right hemisphere intracerebral hemorrhage E. All of the above

62

66485457-66485438 www.ketabpezeshki.com

ZZakaria_87574_PTR_CH04_10-06-13_51-70.inddakaria_87574_PTR_CH04_10-06-13_51-70.indd 6262 66/19/2013/19/2013 8:45:298:45:29 PMPM 4

ANSWERS

1. The answer is C. This patient presents with a possible acute left hemispheric cerebral infarction. Even though he is within the 3-hour window for tissue plasminogen activa- tor (TPA) administration, TPA is contraindicated if the patient has a seizure with stroke onset as it seems this patient did per history. In addition, this patient appears to be in status epilepticus (SE). Traditionally, SE was considered persistent or repetitive seizure activity lasting at least 30 minutes, without recovery of consciousness between attacks (1). This patient appears to be having repetitive complex partial seizures (dazed, star- ing, right-hand movements) over the past 50 minutes. Although he is not unconscious or comatose, he has not recovered to his baseline level of consciousness. For practical purposes, a patient should be considered to be in SE if a seizure persists for 5 minutes or longer (1), and the treatment protocol for SE should be initiated. This patient’s SE should be treated emergently and should be the fi rst priority of the possible answers. Thus, this patient should be loaded with fosphenytoin, be admitted to the Neuro-ICU, and have cEEG monitoring for nonconvulsive and clinically silent ongoing seizure activity. The differential for an acute confusional state (and seizures) does include cerebral infections such as encephalitis. However, the clinical history here is suggestive of an acute left middle cerebral artery (MCA) distribution cerebral infarction with complex partial sei- zures. An argument could also be made for simply new onset of complex partial SE with Todd’s paralysis. An MRI should be obtained to evaluate for acute infarction. The history is rather abrupt for encephalitis, but HSV encephalitis should be considered if the MRI were to show typical T2 and FLAIR (fl uid-attenuated inversion recovery) hyperintensi- ties in the temporal lobes (or if the MRI did not show an acute infarction as suspected). The most common cause of SE is a prior history of epilepsy, but approximately half of SE occurs in patients without prior history of seizures. In these patients, the most common cause is stroke, accounting for about 20% (1).

63

66485457-66485438 www.ketabpezeshki.com

ZZakaria_87574_PTR_CH04_10-06-13_51-70.inddakaria_87574_PTR_CH04_10-06-13_51-70.indd 6363 66/19/2013/19/2013 8:45:298:45:29 PMPM DISORDERS, DISEASES, SEIZURES, AND EPILEPSY

2. The answer is D. The management of SE is rather complicated. The treating physician has two concurrent goals in the management of SE: one is to stop all seizure activity as quickly as possible, and the other is to determine an etiology for the SE. There are differ- ent published treatment protocols for SE, which tend to only vary on minor details. All protocols begin with basic life support, sending labs to look for an etiology for the SE, and then the administration of thiamine 100 mg IV and 50 mL of 50% dextrose IV unless an adequate glucose level is known. (Note that this patient’s glucose was normal, and in clinical practice, a glucose level can be established rapidly with a fi ngerstick.) The follow- ing recommendations are per the Columbia University Comprehensive Epilepsy Center Status Epilepticus Adult Treatment Protocol (1): Start treatment with lorazepam 4 mg IV × 1, and a second identical dose after 5 minutes if seizures persist.* (Diazepam 20 mg per rectum or midazolam 10 mg intranasally, buccally, or IM can be substituted for loraze- pam if there is no IV access yet.) The patient should then be loaded with fosphenytoin at 20 mg/kg IV at 150 mg/minute with concurrent blood pressure (BP) and EKG monitoring as hypotension and cardiac arrhythmias can occur. If seizures persist, there are four con- ventional treatment options, all of which require intubation except for valproate: i. Midazolam continuous IV (CIV) infusion, load at 0.2 mg/kg (repeat 0.2–0.4 mg/kg boluses every 5 minutes until seizures stop up to maximum loading dose of 2.9 mg/kg); CIV dose range 0.05 to 2.9 mg/kg/hour ii. Propofol CIV infusion; load at 1 to 2 mg/kg (repeat 1–2 mg/kg boluses every 5 min- utes until seizures stop up to maximum loading dose of 10 mg/kg); CIV dose range 1 to 15 mg/kg/hour iii. Valproate IV 40 mg/kg over 10 minutes, with subsequent 20 mg/kg dose over 5 min- utes if seizures persist iv. Phenobarbital 20 mg/kg IV (run at 50–100 mg/minute) *Other protocols recommend lorazepam 2 mg IV × 1, followed by an identical dose in 2 minutes with continued seizure activity. Additional 2 mg doses are given until the seizure activity stops, respiratory suppression is seen, or 8 to 10 mg have been administered.

3. The answer is C. The EEG shows irregular generalized spike and wave, or polyspike and wave activity at approximately 3.0 to 3.5 Hz (consistent with absence SE). There are several medications known to exacerbate absence seizures. These include phenytoin, car- bamazepine, oxcarbazepine, and tiagabine (1). Therefore, the patient should not be given phenytoin. This is a caveat to the treatment of SE. If a patient is known to have childhood or juvenile absence epilepsy, then the treatment regimen protocol should exclude pheny- toin. The most appropriate initial drugs in the setting of absence SE are benzodiazepines such as lorazepam or diazepam to break the seizure. It would then be most appropriate to load valproic acid to reach a rapid serum therapeutic range. In the clinical scenario outlined in this question, it is also appropriate to check serum lamotrigine levels to see whether the patient is subtherapeutic on this outpatient medication, as medication non- compliance could be the cause of the absence SE.

4. The answer is G. The defi nition of brain death is deceptively simple: “the irreversible loss of function of the brain, including the brain stem.” The diffi culty lies in the confi rmation of the term irreversible. Thus, it is the physician’s duty to exclude all reversible causes that can mimic the clinical fi ndings of brain death (unresponsive, absent brainstem refl exes,

64

66485457-66485438 www.ketabpezeshki.com

ZZakaria_87574_PTR_CH04_10-06-13_51-70.inddakaria_87574_PTR_CH04_10-06-13_51-70.indd 6464 66/19/2013/19/2013 8:45:298:45:29 PMPM SEIZURES AND EPILEPSY: Answers

and demonstrated apnea on a standardized apnea test). Defi nite demonstration that the criteria listed in choices A, B, C, and D have been met is needed for the establishment of brain death in the United States (2). There is no requirement for EEG or somatosensory- evoked potential (SEP) testing if the clinical fi ndings of brain death are present and revers- ible mimics are eliminated.

5. The answer is B. The EEG shows electrocerebral inactivity (ECI) and diffuse EKG artifact. The EEG is fl at except for the prominent EKG artifact. Note that all the apparent cerebral activity is always time linked with EKG QRS complex, and this is how EKG artifact is con- fi rmed in the EEG. The electric fi eld produced by the heart can be recorded from the scalp, and it is commonly seen on the EEG of ECI recordings. Note that two of the established criteria for a brain death protocol EEG are duration of at least 30 minutes and the use of a minimum of 16 EEG channels (2).

6. The answer is E. All of the answers are correct. These are the relevant special consider- ations that must be followed for a brain death protocol EEG. In particular, note that the EEG must be interpreted at a gain of 2 µV/mm. (Typically, adult EEGs are interpreted at gains of 7–10 µV/mm.) With lower gain settings, the EEG waveforms appear larger. Thus, this setting of 2 µV/mm will essentially magnify any present electrocerebral activ- ity, and this is necessary before concluding that an EEG shows complete electrocerebral inactivity (ECI). Also note that typical routine hospital and outpatient EEGs are 20 min- utes in duration. For a brain death protocol, a minimum of 30 minutes of interpretable recording should be obtained (2).

7. The answer is C. When thinking about EEG patterns seen in the setting of clinical brain death, the most common pattern seen is ECI. However, it is important to note that this fi nding is not diagnostic of brain death (which is a clinical defi nition). The ECI pattern is in fact seen in about 80% of patients fulfi lling clinical criteria for brain death. (Thus 20% of the time, clinically brain-dead patients have some preserved EEG activity!) (3)

8. The answer is A. The fi gure shows a background of mixed 4 to 7 Hz theta activity with prominent 12 to 16 Hz spindle activity, consistent with spindle coma. Spindles are typi- cally one of the hallmarks of stage II sleep. However, spindle coma and stage II sleep are distinguished by clinical context; patients are arousable from stage II sleep with gentle to noxious stimuli or with auditory stimuli, whereas those in spindle coma remain unre- sponsive. Most commonly, spindle coma is seen in patients with high mesencephalic (midbrain) lesions, as in this case. However, spindle coma can also be seen with posttrau- matic and postencephalitic encephalopathies. This EEG pattern is in general a harbinger of a favorable prognosis for some meaningful neurologic recovery (but defi nitely not nec- essarily an indicator of complete recovery or a speedy recovery) (4).

9. The answer is D. The EEG shows prominent diffuse alpha activity, and we were informed that this EEG was not reactive to noxious or auditory stimuli. Thus, the EEG represents alpha coma. Typically, the clinical setting of alpha coma is in patients with diffuse brain insults after cardiopulmonary arrest, patients with brainstem lesions at or just caudal to the pontomesencephalic junction, and patients with various toxic or metabolic abnormali- ties, including barbiturate overdose and hyperglycemic, hyperosmolar coma (5). Most

65

66485457-66485438 www.ketabpezeshki.com

ZZakaria_87574_PTR_CH04_10-06-13_51-70.inddakaria_87574_PTR_CH04_10-06-13_51-70.indd 6565 66/19/2013/19/2013 8:45:298:45:29 PMPM DISORDERS, DISEASES, SEIZURES, AND EPILEPSY

patients with alpha coma after cardiopulmonary arrest die or have severe neurologic defi cits upon recovery (5), and therefore, the appearance of an alpha coma pattern in this clinical context carries a particularly grim prognosis.

10. The answer is D. A discussion of the myriad of causes of diffuse encephalopathies is beyond the scope of this chapter. However, regardless of underlying etiology, the EEG often follows a typical progression as an encephalopathy progresses from “mild” to “severe.” The earliest changes are often a mild slowing of the occipital dominant rhythm (alpha rhythm) in wakefulness into the theta range; diffuse fragmentary or more sus- tained theta and delta activity can also be seen. As encephalopathy worsens, the alpha rhythm and normal faster activities (alpha and frontal beta) will be lost, and diffuse theta and delta activity become more prominent and sustained. Normal sleep architecture can also be lost (K-complexes and spindles). With worsening, periods of diffuse amplitude attenuation and FIRDA can be seen; the normal EEG variability and state transitions are lost. The EEG becomes unreactive to external stimuli (pain). As encephalopathy worsens still, the EEG will reach a diffuse BS pattern and ultimately will reach electrocerebral inactivity (ECI) (6).

11. The answer is D. The EEG clearly shows PLEDs in the left temporal lobe. These are not just frequent left temporal spikes, given the rhythmicity and periodic nature of the dis- charges. PLEDs typically recur every 1 to 2 seconds (as seen in this EEG) and consist of a spike or sharp wave that may be followed by a slow wave. Acute stroke is the most common cause of PLEDs, but any acute cerebral injury that could result in seizures can show PLEDs (and PLEDs are commonly seen in association with seizure activity). Herpes simplex virus (HSV) encephalitis is commonly considered when PLEDs are seen in the appropriate clinical context (as in this case), since most patients with HSV encephalitis will develop PLEDs. In summary, PLEDs should be considered a marker of acute cerebral injury, and these discharges are a transient fi nding, diminishing over days to weeks (7).

12. The answer is D. The EEG shows generalized periodic epileptiform discharges (GPEDs). GPEDs are a nonspecifi c fi nding that is commonly seen in the ICU in a variety of clini- cal settings. These include postanoxic coma after convulsive SE, metabolic disorders, Creutzfeldt–Jakob disease, and Hashimoto encephalopathy. Medication toxicity can also result in this EEG pattern, most notably with lithium toxicity (but can also be seen with baclofen and cefepime). GPEDs (unlike periodic lateralized epileptiform discharges [PLEDs]) are less commonly associated with seizures (7).

13. The answer is D. The EEG shows triphasic waves; note the initial negative (upward) com- ponent of the complex, with subsequent larger amplitude positive (downward) defl ection, and with a fi nal negative component. Note the frontal predominance, as this is commonly seen with triphasic waves. Triphasic waves are a type of generalized periodic discharge, classically seen in hepatic encephalopathy. However, triphasic waves can be seen in any toxic/metabolic encephalopathy, and they should not be considered diagnostic of hepatic encephalopathy. Typically, triphasic waves are repetitive every 1 to 2 seconds and may wax and wane in morphology and persistence throughout the recording (7). Triphasics can be seen in renal failure, hyperosmolar state, hypoglycemia, hyponatremia, hypercal- cemia, and hyperthyroidism (8).

66

66485457-66485438 www.ketabpezeshki.com

ZZakaria_87574_PTR_CH04_10-06-13_51-70.inddakaria_87574_PTR_CH04_10-06-13_51-70.indd 6666 66/19/2013/19/2013 8:45:298:45:29 PMPM SEIZURES AND EPILEPSY: Answers

14. The answer is A. This EEG shows a typical nonconvulsive seizure as seen in the ICU set- ting. The electrographic criteria for seizure activity include clear evolution in frequency, morphology, or location of an ongoing EEG pattern (9). This particular EEG shows the evolution of approximately 2-Hz right frontal-temporal spikes into 1-Hz spikes on the sec- ond half of the EEG. The following 10 to 30 seconds of acquired EEG would be expected to show resolution of the spikes, with possible right frontal–temporal delta–theta slowing. Thus, this fi nding is an electrographic seizure (labeled as nonconvulsive based on the clin- ical history provided). A breach rhythm is from a skull defect (such as craniotomy or burr hole), which results in an increase in voltage and more sharp morphology in the underly- ing EEG. Faster frequencies are more accentuated with breach than slower frequencies (9). Mu rhythm is a benign, normal variant typically seen in healthy individuals in the ambulatory setting, consisting of 7- to 11-Hz arciform waveforms over the central head regions. This normal variant is attenuated with movement or thought of movement of the contralateral hand (10). The EEG abnormalities seen are not limited to the F4 electrode.

15. The answer is C. According to the practice parameters regarding the prediction of out- come of comatose survivors after CPR published by the American Academy of Neurology in 2006, the bilateral absence of the N20 component of the median nerve SEP 3 days or later after CPR accurately predicts a poor outcome (11). The N9 (Erb’s point) waveform refl ects peripheral nerve activity propagating through the brachial plexus and would be expected to remain present. The N13 waveform refl ects activity in the dorsal horns of the spinal cord, and this would also be expected to remain present. The N20 waveform refl ects activity in the thalamocortical radiations/sensory cortex (some debate remains to the exact generator) (12).

16. The answer is A. This EEG pattern is known as a burst-suppression (BS) pattern, and it is seen only in the clinical context of coma. BS is an alternating pattern. It consists of bursts of bilaterally synchronous, widespread mixed-frequency activity, which may have intermixed spikes or sharp waves. These bursts alternate with periods of globally absent EEG activity (or low-voltage delta–theta activity) (13). BS is commonly seen after a severe insult to the brain, most commonly hypoxic or anoxic insult (in which case it carries a poor prognosis). BS can also be seen with reversible causes of coma, such as hypothermia; drug intoxication (benzodiazepines, barbiturates, high doses of narcotics); and after adminis- tration of anesthetics (propofol, barbiturates, sevofl urane, isofl urane, etc.) (14). During the treatment of convulsive status epilepticus (SE), the goal is often to achieve a global BS pattern with the use of anesthetics. Note that Creutzfeldt–Jakob disease results in the appearance of generalized periodic epileptiform discharges (GPEDs) and not BS.

17. The answer is D. During CEA surgery, it is common to monitor EEG and median nerve and posterior tibial nerve somatosensory-evoked potential (SEP). The EEG is sensitive for detecting new cortical ischemia and assesses large areas of the cerebral cortex. New cortical ischemia is seen as ipsilateral slowing or amplitude attenuation/decrement, or both. The normal mean cerebral blood fl ow (CBF) is approximately 50 mL/100 g/minute. Mild hypoperfusion down to approximately 22 mL/100 g/minute typically is well toler- ated without EEG changes. When CBF falls below this 22 mL/100 g/minute threshold, the EEG will show slowing or amplitude attenuation, or both. A further decrease to 7 to 15 mL/100 g/minute will result in suppression of EEG activity. Perfusion below 12 to

67

66485457-66485438 www.ketabpezeshki.com

ZZakaria_87574_PTR_CH04_10-06-13_51-70.inddakaria_87574_PTR_CH04_10-06-13_51-70.indd 6767 66/19/2013/19/2013 8:45:298:45:29 PMPM DISORDERS, DISEASES, SEIZURES, AND EPILEPSY

15 mL/100 g/minute will result in neural damage. In most patients, there will be adequate collateral fl ow (via the circle of Willis) to prevent cerebral ischemia when one internal carotid artery is clamped. However, in about 20% of patients, carotid clamping results in signifi cant cerebral ischemia with an associated high risk of acute cerebral infarction (15). The changes seen in this EEG are indicative of new left cerebral hemispheric hypoperfu- sion and ischemia. The surgeon should be alerted to this and to the fact that carotid artery bypass shunting is needed.

18. The answer is D. In NCSE, seizure duration tends to be the best predictor of mortality. A study of NCSE in the ICU setting demonstrated that if seizure duration was less than 10 hours, 60% of patients returned home and 10% died. However, if NCSE duration was greater than 20 hours, none returned home and 85% died (16). Thus, rapid recognition and treatment of NCSE in the ICU has important implications for clinical outcome.

19. The answer is E. Subdural (and epidural) hematomas can be associated with all of the EEG patterns mentioned. The focal slowing and amplitude attenuation are caused by the underlying cortical malfunction and the increased distance between the brain and the recording electrodes. Blood is irritating to the cerebral cortex, so focal sharp waves or spikes, PLEDs, and seizures may be seen with any area of cerebral hemorrhage affecting the cortex (17).

20. The answer is E. The rationale for cEEG monitoring in the ICU is founded on the follow- ing attributes: (a) EEG is tightly linked to cerebral metabolism; (b) EEG is sensitive to ischemia and hypoxia; (c) EEG detects neuronal dysfunction at a reversible stage; (d) EEG detects neuronal damage or recovery, while clinical examination may not; (e) EEG is the best method to detect epileptiform activity, including NCSE; (f) cEEG provides dynamic information that routine bedside EEG cannot; and (g) EEG provides useful information about localization. Thus, cEEG has utility in many clinical ICU settings, including acute cerebral ischemia, intracranial hemorrhage, uncontrolled seizures (including SE and con- cern for NCSE), coma, intracranial infections, and head trauma. Beyond its obvious role for assessing epileptiform activity and seizures, cEEG is a valuable tool for assessing post- subarachnoid hemorrhage vasospasm, focal ischemia, and early detection of increased intracranial pressures. cEEG is also useful for monitoring anesthetic and drug effects (commonly described as the depth of anesthesia) and for monitoring for periodic patterns that may be of prognostic value or indicate a higher risk for seizures (i.e., periodic later- alized epileptiform discharges [PLEDs] or generalized periodic epileptiform discharges [GPEDs]) (18).

References

1. Hirsch LJ, Arif H. Status epilepticus. Continuum, lifelong learning in neurology, epilepsy. AAN. 2007;13(4):121–151. 2. Chatrian GE, Turella GS. Electrophysiological evaluation of coma, other states of dimin- ished responsiveness, and brain death. In: Ebersole JS, Pedley TA, eds. Current Practice of Clinical Electroencephalography. 3rd ed. Philadelphia, PA: Lippincott Williams & Wilkins, 2003:441–443.

68

66485457-66485438 www.ketabpezeshki.com

ZZakaria_87574_PTR_CH04_10-06-13_51-70.inddakaria_87574_PTR_CH04_10-06-13_51-70.indd 6868 66/19/2013/19/2013 8:45:298:45:29 PMPM SEIZURES AND EPILEPSY: Answers

3. Grigg MM, Kelly MA, Celesia GG, Ghobrial MW, Ross ER. Electroencephalographic activity after brain death. Arch Neurol. 1987;44(9):948–954. 4. Ikeda AI, Klem GH, Luders HO. Metabolic, infectious, and hereditary encephalopathies. In: Ebersole JS, Pedley TA, eds. Current Practice of Clinical Electroencephalography. 3rd ed. Philadelphia, PA: Lippincott Williams & Wilkins, 2003:356. 5. Austin EJ, Wilkus RJ, Longstreth WT Jr. Etiology and prognosis of alpha coma. Neurology. 1988;38(5):773–777. 6. Hirsch LJ, Brenner RP. Atlas of EEG in Critical Care. West Sussex: Wiley-Blackwell, 2010:39. 7. Hirsch LJ, Brenner RP. Atlas of EEG in Critical Care. West Sussex: Wiley-Blackwell, 2010:129. 8. Ikeda AI, Klem GH, Luders HO. Metabolic, infectious, and hereditary encephalopathies. In: Ebersole JS, Pedley TA, eds. Current Practice of Clinical Electroencephalography. 3rd ed. Philadelphia, PA: Lippincott Williams & Wilkins, 2003:359. 9. Hirsch LJ, Brenner RP. Atlas of EEG in Critical Care. West Sussex: Wiley-Blackwell, 2010: 89, 189. 10. Westmoreland BF. Electroencephalography: adult, normal, and benign variants. In: Daube JR, Rubin DI, eds. Clinical Neurophysiology. 3rd ed. Oxford: Oxford University Press, 2009:126. 11. Wijdicks EF, Hijdra A, Young GB, Bassetti CL, Wiebe S. Practice parameter: prediction of outcome in comatose survivors after cardiopulmonary resuscitation (an evidence-based review): report of the Quality Standards Subcommittee of the American Academy of Neurology. Neurology. 2006;67(2):203–210. 12. Carter JL, Stevens JC. Somatosensory evoked potentials. In: Daube JR, Rubin DI, eds. Clinical Neurophysiology. 3rd ed. Oxford: Oxford University Press, 2009:265. 13. Chatrian GE, Turella GS. Electrophysiological evaluation of coma, other states of dimin- ished responsiveness, and brain death. In: Ebersole JS, Pedley TA, eds. Current Practice of Clinical Electroencephalography. 3rd ed. Philadelphia, PA: Lippincott Williams & Wilkins, 2003:408. 14. Westmoreland BF. Electroencephalography: adult EEG: abnormal nonepileptiform activ- ity. In: Daube JR, Rubin DI, eds. Clinical Neurophysiology. 3rd ed. Oxford: Oxford University Press, 2009:163. 15. Sepkuty JP, Gutierrez S. Carotid surgery. In: Husain AM, ed. A Practical Approach to Neurophysiologic Intraoperative Monitoring. New York, NY: Demos, 2008:269–270. 16. Young GB, Jordan KG, Doig GS. An assessment of nonconvulsive seizures in the intensive care unit using continuous EEG monitoring: an investigation of variables associated with mortality. Neurology. 1996;47(1):83–89. 17. Hirsch LJ, Brenner RP. Atlas of EEG in Critical Care. West Sussex: Wiley-Blackwell, 2010:161. 18. Jordan KG, Bleck TP. Continuous EEG monitoring in the intensive care unit. In: Ebersole JS, Pedley TA, eds. Current Practice of Clinical Electroencephalography. 3rd ed. Philadelphia, PA: Lippincott Williams & Wilkins, 2003:761–799.

69

66485457-66485438 www.ketabpezeshki.com

ZZakaria_87574_PTR_CH04_10-06-13_51-70.inddakaria_87574_PTR_CH04_10-06-13_51-70.indd 6969 66/19/2013/19/2013 8:45:298:45:29 PMPM 66485457-66485438 www.ketabpezeshki.com

ZZakaria_87574_PTR_CH04_10-06-13_51-70.inddakaria_87574_PTR_CH04_10-06-13_51-70.indd 7070 66/19/2013/19/2013 8:45:298:45:29 PMPM 5 Neuromuscular Medicine Suur Biliciler, Justin Kwan, and Cecile L. Phan QUESTIONS

1. A 48-year-old woman presented with a 25-lb weight loss, increasingly slurred speech, swallowing diffi culty, and arm and leg muscle weakness over the past 14 months. On examination, she had fasciculations on her tongue, proximal and distal muscle weakness, atrophy, fasciculations, and hyperrefl exia. Her sensory examination was normal. Which of the following is not an effective intervention to prolong survival in a patient with the diagnosis of amyotrophic lateral sclerosis (ALS)? A. Noninvasive ventilation B. Nutritional support via percutaneous endoscopic gastrostomy (PEG) C. Lithium D. Riluzole

2. Which of the following descriptions best describes cognitive status in amyotrophic lateral sclerosis (ALS)? A. Cognitive and behavior impairment is never present B. Alzheimer-type dementia is common and expected C. Hallucination and delusion are early features of impaired cognition D. Executive dysfunction, impulsivity, and impaired judgment are clinical features of cog- nitive impairment

ANSWERS TO THIS SECTION CAN BE FOUND ON PAGE 75 71

66485457-66485438 www.ketabpezeshki.com

ZZakaria_87574_PTR_CH05_10-06-13_71-78.inddakaria_87574_PTR_CH05_10-06-13_71-78.indd 7171 66/19/2013/19/2013 4:43:514:43:51 PMPM DISORDERS, DISEASES, SEIZURES, AND EPILEPSY

3. A 24-year-old woman with a history of an eating disorder consisting of binge eating fol- lowed by self-induced vomiting presented with a 6-month history of generalized weak- ness and exercise intolerance. The examination revealed generalized weakness affecting proximal more than distal muscles, muscle atrophy, and normal sensation. The serum cre- atine phosphokinase was 620 units/L. Electrocardiography showed sinus tachycardia and inverted T waves. What is the most likely cause of this patient’s muscular symptoms? A. Polymyositis B. Emery–Dreifuss muscular dystrophy C. Complication of bulimia D. Ipecac-induced myopathy 4. Which of the following is an important intervention in the management of rhabdomyoly- sis to prevent acute renal injury? A. Aggressive fl uid repletion B. Antioxidant therapy C. Peritoneal dialysis D. Rapid infusion of hypertonic saline 5. Which of the following conditions is/are susceptible to succinylcholine-induced hyperkalemia? A. Muscular dystrophy B. Infl ammatory myopathy C. Critical illness neuropathy D. All of the above 6. A 35-year-old African American woman presented to the ED with respiratory failure, dif- fi culty swallowing, and slurred speech. These symptoms had been present for the past several weeks, but recently worsened after she contracted a viral upper respiratory tract infection. On examination, the patient was in respiratory distress with increased work of breathing, signifi cant facial weakness and atrophy, neck fl exor weakness, and severe fl ac- cid dysarthria. Extraocular movements were intact, and her limbs were strong with normal tendon refl exes. She had seen an outside neurologist who diagnosed her with seronegative myasthenia gravis (MG). The next step of her evaluation is: A. Evaluate for motor neuron disease B. Send anti-muscle-specifi c-kinase (anti-MUSK) antibody C. Obtain electromyography (EMG)/nerve conduction study with repetitive nerve stimulation D. Obtain a muscle biopsy to evaluate for mitochondrial disorder 7. An 80-year-old man recently diagnosed with myasthenia gravis (MG) had worsen- ing weakness, slurred speech, excessive drooling, as well as severe muscle cramps and diarrhea. He was taking regular pyridostigmine 180 mg every 4 hours while awake and extended release pyridostigmine 180 mg at bedtime. In addition, he was recently started on prednisone 20 mg daily. His worsening symptoms are caused by: A. Steroid-induced weakness in MG B. Current gastrointestinal infection with diarrhea C. Excessive cholinergic activity from overdosing of acetylcholine esterase inhibitors D. Something other than MG; he does not have MG, because cramps and diarrhea are not clinical features of MG

72

66485457-66485438 www.ketabpezeshki.com

ZZakaria_87574_PTR_CH05_10-06-13_71-78.inddakaria_87574_PTR_CH05_10-06-13_71-78.indd 7272 66/19/2013/19/2013 4:43:514:43:51 PMPM NEUROMUSCULAR MEDICINE: Questions

8. A 20-year-old woman with seropositive myasthenia gravis (MG) was admitted to the ICU with respiratory failure, severe dysphagia, and weakness. The best treatment option for this patient is: A. Plasma exchange because it works faster than intravenous immunoglobulin (IVIg) and is better tolerated than IVIg B. IVIg since it is easily available and does not require the placement of a central line C. Either plasma exchange or IVIg D. IV pyridostigmine 2 mg IV every 4 hours

9. A patient who was previously diagnosed with myasthenia gravis (MG) is in the ED with respiratory insuffi ciency. His bedside spirometry values are as follows: forced vital capac-

ity (FVC) 1.6 L (he weighs 83 kg), negative inspiratory force (NIF) −18 cmH2O, and peak

expiratory fl ow (PEF) −35 cmH2O. What should be done next? A. Intubate the patient immediately for airway protection B. Obtain a stat arterial blood gas (ABG), as bedside spirometry is not as accurate at pre- dicting respiratory failure C. Start patient on bilevel positive airway pressure (BiPAP) D. Observe the patient closely with q2hr FVC and NIF, and intubate once FVC drops below 15 mL/kg

10. For patients with myasthenic crisis in the ICU on mechanical ventilation, which of the following statements is true regarding initiation of steroid therapy? A. High-dose steroid should be avoided because it can cause transient worsening of the myasthenia B. High-dose prednisone 60 to 80 mg PO daily should be initiated in conjunction with plasma exchange or intravenous immunoglobulin (IVIg) C. Steroid should be started at the lowest dose possible and slowly titrate up to alternate- day dosing of 60 to 80 mg q.o.d D. IV Solu Medrol is the preferred drug since prednisone tablets cannot be given via per- cutaneous endoscopic gastrostomy (PEG) tube

Please read the clinical presentation paragraph below and answer questions 11–15. A 66-year-old man with no signifi cant medical history presented to the ED complaining of progressive numbness and weakness in his legs for 5 days after an episode of diarrhea. He denied recent foreign travel or outdoor activities. He was not taking any medica- tions. On his general examination, he was afebrile and had a heart rate of 120 beats/min- ute, blood pressure of 100/70 mmHg, and a respiratory rate of 13 breaths/minute. His neurological examination showed normal mental status, intact cranial nerves except for mild facial weakness, proximal and distal weakness in both upper and lower extremities, decreased light touch and pinprick in lower extremities and hands, diffuse loss of muscle stretch refl exes, no sensory level, and fl exor plantar responses. His chest x-ray, complete blood count, thyroid function tests, liver function studies, cardiac enzymes, serum and urine toxicity screens, and comprehensive metabolic panel, including sodium, calcium, potassium, serum protein, blood urea nitrogen, and creatinine, were normal. His EKG showed sustained sinus tachycardia. During his medical evaluation, the patient became increasingly short of breath and was admitted to the neurologic ICU after intubation.

73

66485457-66485438 www.ketabpezeshki.com

ZZakaria_87574_PTR_CH05_10-06-13_71-78.inddakaria_87574_PTR_CH05_10-06-13_71-78.indd 7373 66/19/2013/19/2013 4:43:514:43:51 PMPM DISORDERS, DISEASES, SEIZURES, AND EPILEPSY

11. What is the most likely diagnosis? A. Guillain–Barré syndrome (GBS) B. Chronic infl ammatory demyelinating polyradiculoneuropathy (CIDP) C. Spinal cord injury D. Multiple sclerosis

12. What is the next diagnostic study that needs to be ordered? A. Cranial MRI B. Transthoracic echocardiogram C. Lumbar puncture D. Evoked potentials

13. What is the best treatment for this patient? A. Intravenous immunoglobulin (IVIg) B. Plasmapheresis and oral steroids C. IVIg followed by plasma exchange D. IV steroids

14. The day after admission to the neurologic ICU, the patient’s serum sodium level was 128 mEq/L. What is/are possible explanation(s) for the hyponatremia? A. Syndrome of inappropriate secretion of antidiuretic hormone (SIADH) B. Pseudohyponatremia secondary to the best treatment option (per question 13) patient received for his disease C. Both A and B D. None of the above

15. What is/are the common cardiovascular complication(s) of this disorder? A. Sustained sinus tachycardia B. Hypotension C. Atrial and ventricular arrhythmias D. All of the above

74

66485457-66485438 www.ketabpezeshki.com

ZZakaria_87574_PTR_CH05_10-06-13_71-78.inddakaria_87574_PTR_CH05_10-06-13_71-78.indd 7474 66/19/2013/19/2013 4:43:514:43:51 PMPM 5

ANSWERS

1. The answer is C. Lithium carbonate was shown to be ineffective in delaying disease pro- gression in amyotrophic lateral sclerosis (ALS) in a randomized, placebo-controlled study. Noninvasive ventilation, nutritional support using PEG, and riluzole have all been shown to delay disease progression in ALS (1,2).

2. The answer is D. Frontotemporal dementia (FTD) occurs in approximately 15% of ALS patients. Many ALS patients who do not fulfi ll the diagnostic criteria for FTD will have cog- nitive impairment manifesting as executing dysfunction and/or behavior impairment (3).

3. The answer is D. Ipecac syrup can be used as an emetic in large doses, especially in acci- dental poisoning. Ipecac-induced myopathy should be considered in an individual who has progressive myopathy and a history of eating disorder. Patients may have myopathy and cardiomyopathy due to excessive ipecac use. Discontinuation of ipecac can result in resolution of the symptoms (4).

4. The answer is A. Early and aggressive fl uid repletion can be an effective intervention to prevent acute renal injury in rhabdomyolysis. Water retention by damaged muscle can deplete intravascular fl uid in the patients who have severe muscle injury. The quantity of fl uids needed depends on the severity of the muscle injury, and the optimal composition of the fl uid to be used remains uncertain. Antioxidant therapy and hypertonic saline are not used to prevent renal failure in rhabdomyolysis. Dialysis may be necessary in patients who have electrolyte disturbance, fl uid overload, or acidosis in the setting of acute renal failure (5).

5. The answer is D. Muscular dystrophy, muscle infl ammation, disuse atrophy, denervation, thermal trauma, and severe infection have all been identifi ed as potential risk factors for succinylcholine-induced hyperkalemia. Spread of the acetylcholine receptors (AChRs)

75

66485457-66485438 www.ketabpezeshki.com

ZZakaria_87574_PTR_CH05_10-06-13_71-78.inddakaria_87574_PTR_CH05_10-06-13_71-78.indd 7575 66/19/2013/19/2013 4:43:514:43:51 PMPM DISORDERS, DISEASES, SEIZURES, AND EPILEPSY

throughout the muscle membrane and upregulation of AChRs with resultant effl ux of potassium during activation of the receptors are proposed to be the mechanisms of hyper- kalemia. Fatal arrhythmia may be the consequence of this under-recognized condition (6).

6. The answer is B. This patient has MG associated with anti-MUSK antibody. Forty percent of “seronegative MG” patients who test negative for AChR antibodies have MUSK anti- body. Myasthenic patients with anti-MUSK antibody are predominantly women and have several distinctive features, including prominent facial weakness and atrophy, signifi - cant pharyngeal and respiratory involvement with frequent crises, and good response to plasma exchange for acute crises and rituximab for long-term immunosuppression (7).

7. The answer is C. In patients with MG who have worsening symptoms, it is important to distinguish the disease itself from cholinergic crisis or paradoxical worsening due to high- dose steroid use. In this case, the patient has other signs of cholinergic overdose (excessive cramps and diarrhea), and the steroid dosage is likely not high enough to cause worsen- ing of symptoms (8).

8. The answer is C. A recent class I randomized controlled trial showed that IVIg has com- parable effi cacy to plasma exchange in moderate to severe MG. Both are well tolerated, and the duration of effects is comparable (9).

9. The answer is A. Indications for mechanical ventilation in MG patients include an FVC ≤ ≥ ≤ ≥ ≤ 15 mL/kg (normal 60 mL/kg), a NIF 20 cmH2O (normal 70 cmH2O), or PEF 40 ≥ cmH2O (normal 100 cmH2O). One should not wait for the ABG to become abnormal before securing the airway, because this occurs late in the course of MG crisis after the patient has already decompensated (10).

10. The answer is B. A high-dose initiation therapy is recommended for patients in myasthenic crisis who are already in the controlled setting of an ICU and in need of rapid improve- ment. This should be done in parallel with IVIg or plasma exchange because high-dose corticosteroid can cause transient worsening of the myasthenia symptoms (11).

11. The answer is A. In spinal cord injury, a sensory level is often present on the neurological examination. Patients usually have extensor plantar responses, suggesting corticospinal tract dysfunction. In addition, the presentation is sudden rather than progressive. Similar fi ndings, including long-tract signs, are observed in multiple sclerosis. Loss of refl exes is more suggestive of a peripheral nervous system disorder. CIDP presents with similar fi nd- ings, but the clinical course is more insidious. The diagnosis of CIDP requires that symp- toms are present for 8 weeks or more in a progressive fashion. In this case, the most likely diagnosis is GBS. GBS usually presents as an acute ascending paralysis, and it is an acute immune-mediated disorder of the peripheral nervous system. GBS is the most common cause of postinfectious neuromuscular paralysis worldwide, with an incidence between 0.6 and 4 cases per 100,000. The most commonly identifi ed precipitant is Campylobacter jejuni. Epstein-Barr virus, cytomegalovirus, HIV, and Mycoplasma pneumoniae have all been associated with GBS. The weakness reaches nadir at 2 to 4 weeks followed by recovery, which may occur over the course of several months or be incomplete. Respiratory weak- ness and dysautonomia are potentially life-threatening complications. Variants of GBS are acute infl ammatory demyelinating polyradiculoneuropathy, which is the most frequent

76

66485457-66485438 www.ketabpezeshki.com

ZZakaria_87574_PTR_CH05_10-06-13_71-78.inddakaria_87574_PTR_CH05_10-06-13_71-78.indd 7676 66/19/2013/19/2013 4:43:514:43:51 PMPM NEUROMUSCULAR MEDICINE: Answers

type; acute motor axonal neuropathy; acute motor and sensory axonal neuropathy; acute pandysautonomia; and Miller Fisher syndrome. The Miller Fisher syndrome classically presents with ophthalmoplegia, ataxia, and hyporefl exia in a descending fashion rather than ascending paralysis (12,13).

12. The answer is C. Lumbar puncture is an important investigation tool in diagnosing Guillain–Barré syndrome (GBS). Elevated cerebrospinal fl uid (CSF) protein with a nor- mal CSF cell count (albuminocytologic dissociation) is seen. Even though cell counts up to 50 cells/mm3 are reported, an abnormal CSF cell count warrants further CSF studies to exclude underlying infections. In such cases, the patient’s HIV status must be assessed. Of note, CSF studies can be normal within the fi rst week of the disease. Other important studies that are necessary to evaluate this patient’s symptoms include electromyogra- phy (EMG) and nerve conduction velocity (NCV) studies. EMG is performed to assess axonal injury and is more meaningful in the later phases of the disease. NCV studies can show prolongation of H-refl ex latencies, prolonged F-wave latencies, or absent F-wave responses. These fi ndings may be the only abnormalities in the initial study, particularly early in the disease course. Prolongation of latencies and slowing of conduction veloci- ties can be seen either initially or later on as the disease progresses. If the NCV studies are normal, the study should be repeated in 1 to 2 weeks. The clinical presentation will dictate an infectious disease evaluation that may include stool cultures and serology for Campylobacter jejuni, HIV serology, Epstein-Barr virus, mycoplasma pneumonia, or cytomegalovirus (13).

13. The answer is A. On the basis of Class 1 studies, IVIg and plasmapheresis are both equally effi cacious in treating Guillain–Barré syndrome (GBS). Trials combining IVIg and plasmapheresis have failed to reveal additional clinical benefi t. The most common side effects encountered with IVIg are headache, nausea, chills, fever, and hypertension. Hypercoagulability and renal failure are more serious adverse effects related to IVIg treat- ment. The most common side effects encountered with plasmapheresis are hypotension, bleeding, and allergic reaction. Each patient should be evaluated individually regarding adverse effect profi le before choosing IVIg versus plasmapheresis. Corticosteroids do not modify the long-term outcome and are found to be ineffective in most studies. Therefore, corticosteroids are not recommended for the treatment of GBS (14,15).

14. The answer is C. SIADH is a well-known and usually overlooked electrolyte disorder associated with Guillain–Barré syndrome (GBS). Many patients are asymptomatic. The pathophysiology of SIADH in GBS is unknown; however, several hypotheses have been offered, including downward osmotic resetting and enhanced renal tubular sensitivity to antidiuretic hormone. Some studies have suggested that GBS is more severe in patients who developed SIADH. Pseudohyponatremia can also occur as a laboratory artifact in GBS patients who have been treated with intravenous immunoglobulin (IVIg) (16).

15. The answer is D. Cardiovascular abnormalities seen in Guillain–Barré syndrome (GBS) are attributed to autonomic neuropathy. Common cardiovascular complications are rhythm abnormalities (bradyarrhythmias, sustained sinus tachycardia, atrial and ventric- ular arrhythmias); blood pressure variability (hypotension or hypertension); myocardial involvement (myocarditis, neurogenic stunned myocardium, heart failure); acute coronary

77

66485457-66485438 www.ketabpezeshki.com

ZZakaria_87574_PTR_CH05_10-06-13_71-78.inddakaria_87574_PTR_CH05_10-06-13_71-78.indd 7777 66/19/2013/19/2013 4:43:524:43:52 PMPM DISORDERS, DISEASES, SEIZURES, AND EPILEPSY

syndromes (ST elevation myocardial infarction); and EKG changes (giant T waves, pro- longed QT intervals, ST-T changes, U waves, atrioventricular block, bradycardia, and tachycardia) (17).

References

1. Aggarwal SP, Zinman L, Simpson E, et al. Safety and effi cacy of lithium in combination with riluzole for treatment of amyotrophic lateral sclerosis: a randomised, double-blind, placebo-controlled trial. Lancet Neurol. 2010;9(5):481–488. 2. Miller RG, Jackson CE, Kasarskis EJ, et al. Practice parameter update: the care of the patient with amyotrophic lateral sclerosis: drug, nutritional, and respiratory therapies (an evidence-based review): report of the Quality Standards Subcommittee of the American Academy of Neurology. Neurology. 2009;73(15):1218–1226. 3. Hardiman O, van den Berg LH, Kiernan MC. Clinical diagnosis and management of amyotrophic lateral sclerosis. Nat Rev Neurol. 2011;7(11):639–649. 4. Mastaglia FL. Iatrogenic myopathies. Curr Opin Neurol. 2010;23(5):445–449. 5. Bosch X, Poch E, Grau JM. Rhabdomyolysis and acute kidney injury. N Engl J Med. 2009;361(1):62–72. 6. Martyn JA, Richtsfeld M. Succinylcholine-induced hyperkalemia in acquired pathologic states: etiologic factors and molecular mechanisms. Anesthesiology. 2006;104(1):158–169. 7. Guptill JT, Sanders DB. Update on muscle-specifi c tyrosine kinase antibody positive myasthenia gravis. Curr Opin Neurol. 2010;23(5):530–535. 8. Hetherington KA, Losek JD. Myasthenia gravis: myasthenia vs. cholinergic crisis. Pediatr Emerg Care. 2005;21(8):546–8; quiz 549. 9. Barth D, Nabavi Nouri M, Ng E, Nwe P, Bril V. Comparison of IVIg and PLEX in patients with myasthenia gravis. Neurology. 2011;76(23):2017–2023. 10. Bershad EM, Feen ES, Suarez JI. Myasthenia gravis crisis. South Med J. 2008;101(1):63–69. 11. Jani-Acsadi A, Lisak RP. Myasthenic crisis: guidelines for prevention and treatment. J Neurol Sci. 2007;261(1–2):127–133. 12. Uncini A, Kuwabara S. Electrodiagnostic criteria for Guillain-Barré syndrome: a critical revision and the need for an update. Clin Neurophysiol. 2012;123(8):1487–1495. 13. Vucic S, Kiernan MC, Cornblath DR. Guillain-Barré syndrome: an update. J Clin Neurosci. 2009;16(6):733–741. 14. Hughes RA, Swan AV, van Doorn PA. Intravenous immunoglobulin for Guillain-Barré syndrome. Cochrane Database Syst Rev. 2010;6:CD002063. 15. Patwa HS, Chaudhry V, Katzberg H, Rae-Grant AD, So YT. Evidence-based guideline: intravenous immunoglobulin in the treatment of neuromuscular disorders: report of the Therapeutics and Technology Assessment Subcommittee of the American Academy of Neurology. Neurology. 2012;78(13):1009–1015. 16. Saifudheen K, Jose J, Gafoor VA, Musthafa M. Guillain-Barré syndrome and SIADH. Neurology. 2011;76(8):701–704. 17. Mukerji S, Aloka F, Farooq MU, Kassab MY, Abela GS. Cardiovascular complications of the Guillain-Barré syndrome. Am J Cardiol. 2009;104(10):1452–1455.

78

66485457-66485438 www.ketabpezeshki.com

ZZakaria_87574_PTR_CH05_10-06-13_71-78.inddakaria_87574_PTR_CH05_10-06-13_71-78.indd 7878 66/19/2013/19/2013 4:43:524:43:52 PMPM 6 CNS Infections Doris Kung QUESTIONS

1. A 24-year-old woman undergoes a lumbar puncture (LP). The following day she develops a headache that is worsened upon sitting upright and relieved with lying down. What fac- tor would not have predisposed her to this type of headache? A. The number of attempts made by the physician B. The size of the spinal needle C. The position of the bevel of the spinal needle as it was inserted D. The volume of cerebrospinal fl uid (CSF) removed E. The reinsertion of the stylet before removal of the spinal needle

2. A 45-year-old man presents with 2 to 3 days of altered behavior, fever, seizures, and head- aches. His EEG reveals left temporal periodic lateralized epileptiform discharges and slowing in that region. Cerebrospinal fl uid (CSF) studies are obtained. What would be the best test to confi rm his diagnosis? A. CSF Gram stain B. CSF polymerase chain reaction (PCR) study C. CSF viral load D. Elevated CSF red blood cells (RBCs) E. CSF lymphocytic predominance

ANSWERS TO THIS SECTION CAN BE FOUND ON PAGE 84 79

66485457-66485438 www.ketabpezeshki.com

ZZakaria_87574_PTR_CH06_10-06-13_79-94.inddakaria_87574_PTR_CH06_10-06-13_79-94.indd 7979 66/19/2013/19/2013 4:44:004:44:00 PMPM DISORDERS, DISEASES, SEIZURES, AND EPILEPSY

3. West Nile virus (WNV) has been shown to affect all of these areas of the nervous system except: A. B. Basal ganglia C. Cerebellum D. Anterior horn cell E. Cerebral vasculature

4. Which central nervous system (CNS) complications can occur with infective endocarditis (IE)? Put in order from most to least common. A. Meningitis, embolic stroke, hemorrhagic stroke B. Embolic stroke, hemorrhagic stroke, meningitis C. Embolic stroke, intracranial mycotic aneurysm, hemorrhagic stroke D. Intracranial mycotic aneurysm, hemorrhagic stroke, embolic stroke E. Hemorrhagic stroke, embolic stroke, intracranial mycotic aneurysm

5. A 3-month-old infant is brought in with decreased alertness, lethargy, and fever. Which is the least likely cause of infection in this patient? A. Neisseria meningitidis B. Streptococcus agalactiae C. Listeria monocytogenes D. Haemophilus infl uenzae E. Escherichia coli

6. A 40-year-old man returned from Ghana and admitted to forgetting to take his mefl oquine. He complained of headache, fever, and abdominal pain, and these quickly progressed to a comatose state. What stain is used to confi rm the infection? A. Hematoxylin and eosin (H&E) B. India ink C. Gram stain D. Congo red E. Giemsa

7. What will be the fi rst line of treatment for the patient in Question 6? A. Oral quinine with doxycycline B. IV quinine or IV artesunate C. Oral mefl oquine D. Steroids E. Atovaquone

80

66485457-66485438 www.ketabpezeshki.com

ZZakaria_87574_PTR_CH06_10-06-13_79-94.inddakaria_87574_PTR_CH06_10-06-13_79-94.indd 8080 66/19/2013/19/2013 4:44:004:44:00 PMPM CNS INFECTIONS: Questions

8. A 47-year-old HIV patient presents with headache, stiff neck, and altered mental sta-

tus (AMS). A lumbar puncture (LP) is done showing opening pressure of 38 cmH2O. Cerebrospinal fl uid (CSF) studies reveal a positive India ink stain, white cell count, pro- tein, and glucose are normal. MRI brain shows enhancing lesions with little mass effect. What is the best treatment for this patient? A. Amphotericin B × 2 weeks B. Amphotericin B + fl ucytosine × 1 week C. Amphotericin B + fl ucytosine × 2 weeks D. Amphotericin B + fl ucytosine × 2 weeks + fl uconazole for 10 weeks E. Amphotericin B + fl ucytosine × 10 weeks + fl uconazole for 10 weeks

9. The patient in the preceding question has increased intracranial pressures based on the elevated cerebrospinal fl uid (CSF) opening pressure. Which factor(s) would not increase the risk of morbidity or mortality?

A. High baseline CSF opening pressures greater than or equal to 250 mmH2O B. Worsening or persistent signs of fungal meningitis symptoms after 2 weeks of antifun- gal treatment C. Repeat a lumbar puncture (LP) showing an increased opening pressure of greater than

10 mmH2O from the baseline opening pressure D. Positive CSF culture for Cryptococcus after 2 weeks of antifungal treatment E. All of the above would increase the risk of morbidity and mortality

10. A 32-year-old man from Mexico presents with generalized tonic–clonic seizures. CT scan shows characteristic diffuse lesions in the parenchyma. On examination, he also has focal weakness, parkinsonian symptoms, and papilledema, and complains of decreased vision. Which symptom(s) is/are not typical of neurocysticercosis? A. Focal weakness B. Decreased vision C. Seizures D. Parkinsonian symptoms E. None of the above

11. A 15-year-old girl with myasthenia gravis presents with diffi culty breathing. Her family reports that she had a urinary tract infection and was on trimethoprim–sulfamethoxazole. She is admitted in crisis and intubated. While in the ICU, she receives 5 days of intravenous immunoglobulin (IVIg). Her weakness and shortness of breath improve, but she develops a mild headache, which is treated with ibuprofen. Before discharge, she is administered a pneumococcal vaccine. Two days later, she presents with altered mental status (AMS), fever, and neck stiffness. Cerebrospinal fl uid (CSF) shows a mildly elevated protein of 53 (normal range = 12–45 mg/dL), normal glucose of 60 (normal range = 40–80 mg/dL), normal RBC count, elevated WBC count of 50 (normal < 10 per µL) with lymphocytic pre- dominance and a negative Gram stain. What is the most likely cause of her symptoms? A. IVIg B. Trimethoprim–sulfamethoxazole C. Vaccine D. Ibuprofen E. Any of the above

81

66485457-66485438 www.ketabpezeshki.com

ZZakaria_87574_PTR_CH06_10-06-13_79-94.inddakaria_87574_PTR_CH06_10-06-13_79-94.indd 8181 66/19/2013/19/2013 4:44:004:44:00 PMPM DISORDERS, DISEASES, SEIZURES, AND EPILEPSY

12. A 67-year-old man presents with neck stiffness, headache, right leg weakness, and fever. His symptoms progress rapidly to fl accid paralysis and arefl exia. Cerebrospinal fl uid (CSF) studies show elevated WBC count with lymphocytic predominance, normal RBC count, elevated protein (85 mg/dL), and normal glucose. CSF Gram stain and bacterial cultures are negative. What is a possible viral agent responsible for this man’s meningitis? A. Flavivirus family B. Adenovirus family C. Enterovirus family D. Herpes virus family E. Hanta virus family

13. A 45-year-old man with HIV, on antiretroviral therapy inconsistently, presents with 1 month of worsening mental status, left visual fi eld defi cit, and right-sided weakness. His MRI shows two nonenhancing subcortical white matter lesions in the left parietal and right occipital areas without surrounding edema. On T2-weighted imaging, the lesions are hyperintense, and on T1-weighted imaging, the lesions are hypointense. Which fac- tors may increase the risk of worsening the disease? A. Low CD4 B. Hepatitis C C. IV drug use D. Highly active antiretroviral therapy (HAART) treatment E. Steroid treatment

14. A 30-year-old woman presents with back pain and fever. On examination, she has some mild lower extremity weakness, but the examination is limited by pain. She admits to IV drug use in the past month. What is the most likely causative organism? A. Mycobacterium tuberculosis B. Pseudomonas aeruginosa C. Staphylococcus aureus D. Cryptococcus E. Haemophilus infl uenza

15. A 47-year-old woman is brought into the hospital for chest pain. Having been admitted, she is noted to have altered behavior the next morning. Her neurological examination is nonfocal except that when she is shown a pen, she says it is a button; when asked what color shirt the doctor is wearing, she states an incorrect color; when she walks to the bathroom, she runs into the walls; when asked how many fi ngers are held up in front of her face, she is always incorrect in all quadrants. Her CT scan shows bilateral occipi- tal strokes, and transthoracic echocardiogram shows a mobile mass on the mitral valve. Blood cultures confi rm bacteremia. Which factor is the least likely to increase her risk for embolism? A. Vegetation of 15 mm B. Mitral valve vegetation C. S. agalactiae infection D. Staphylococcus aureus infection E. Tricuspid valve vegetation

82

66485457-66485438 www.ketabpezeshki.com

ZZakaria_87574_PTR_CH06_10-06-13_79-94.inddakaria_87574_PTR_CH06_10-06-13_79-94.indd 8282 66/19/2013/19/2013 4:44:004:44:00 PMPM CNS INFECTIONS: Questions

16. In the preceding case, which factor may delay surgery for valve replacement? A. Uncontrolled infection B. Ischemic stroke C. Intracranial hemorrhage D. Heart failure E. Prosthetic valve endocarditis 17. A 65-year-old woman with Alzheimer’s dementia presents with fever, fl ank pain, pyuria, seizures, and altered mental status (AMS). She is found to have a single ring-enhancing lesion with mass effect in the right frontal region on brain imaging. She is also noted to have a facial fracture possibly resulting from a fall after a seizure. What would be your initial choice in treatment? A. First- or second-generation cephalosporin, carbapenem, and stereotactic aspiration B. Third- or fourth-generation cephalosporin, metronidazole, Difl ucan, craniotomy, and anticonvulsant C. Third- or fourth-generation cephalosporin, metronidazole, carbapenem, craniotomy, and anticonvulsant D. Third- or fourth-generation cephalosporin, metronidazole, vancomycin, stereotactic aspiration, and anticonvulsant E. None of the above 18. A patient with AIDS presents with headaches and altered mental status (AMS) for the past 2 days. The MRI of the brain shows a single ring-enhancing lesion in the parenchyma with some mass effect. What percentage of patients with toxoplasmosis in the central ner- vous system (CNS) have multiple ring-enhancing lesions on presentation? A. 40% B. 50% C. 60% D. 70% E. 80% 19. A 12-year-old child who recently emigrated from India presents with headache, neck stiff- ness, fever, and multiple cranial neuropathies. Two weeks before, the patient had been complaining of malaise, anorexia, cough, and weight loss. A chest x-ray shows hilar ade- nopathy. The patient’s MRI with contrast reveals hydrocephalus, basal cistern enhancement, and several ring-enhancing lesions. What is the best treatment of choice in this patient? A. Rifampin and isoniazid B. No treatment; wait for biopsy C. Rifampin, isoniazid, pyrazinamide, ethambutol, and dexamethasone D. Rifampin, isoniazid, pyrazinamide, and ethambutol E. Dexamethasone alone 20. A CT scan of the brain needs to be obtained prior to a lumbar puncture (LP) in the follow- ing situations except: A. Immunocompromise B. Focal neurological signs C. Seizure activity D. Papilledema E. History of cancer

83

66485457-66485438 www.ketabpezeshki.com

ZZakaria_87574_PTR_CH06_10-06-13_79-94.inddakaria_87574_PTR_CH06_10-06-13_79-94.indd 8383 66/19/2013/19/2013 4:44:004:44:00 PMPM 6

ANSWERS

1. The answer is D. A postdural headache occurs in more than 30% of patients following an LP. According to the International Headache Society, postdural (post-LP) headaches occur within 5 days of an LP and resolve spontaneously within 1 week or within 48 hours after treatment of the CSF leak, usually by blood patch. Headaches characteristically worsen within 15 minutes of upright position and are relieved within 15 minutes of supine posi- tion. This is accompanied by one of fi ve associated symptoms (nausea, photophobia, neck stiffness, tinnitus, or hypacusia). There are several factors that increase the risk of devel- oping headaches after an LP. The size of the needle increases the risk. If the size of the needle is between 16 and 19 gauge, the incidence of headache is 70%. If the size of the needle is between 20 and 22 gauge, the incidence of headache drops to 40%. If the needle is between 24 and 27 gauge, the incidence is 12% but the rate of fl ow from these gauge needles may be impractical for most purposes. The direction of the bevel is another risk factor. The bevel of the needle needs to be inserted parallel to the dural fi bers that run in a longitudinal direction down the vertical axis of the spine. This means that if the patient is placed in a lateral decubitus position, the bevel of the needle needs to be facing up at the ceiling or down to the fl oor. The needle design is also a risk factor for development of headaches. Using atraumatic needles may reduce the risk of headache. Replacement of the stylet is also an important simple method to decrease the risk of LP headaches. The theory is that if the stylet is not replaced, a vacuum effect can be created, thereby allowing a strand of arachnoid tissue to enter the needle with the CSF and establish a dural defect with prolonged CSF leakage. Finally, the number of attempts can also potentially increase the risk by creating a larger dural defect. Factors that have not been shown to defi nitively increase the risk of headache include volume of CSF removed, duration of bed rest follow- ing LP, hydration, the opening pressure of the CSF, or the position of the patient during the LP (1).

84

66485457-66485438 www.ketabpezeshki.com

ZZakaria_87574_PTR_CH06_10-06-13_79-94.inddakaria_87574_PTR_CH06_10-06-13_79-94.indd 8484 66/19/2013/19/2013 4:44:004:44:00 PMPM CNS INFECTIONS: Answers

2. The answer is B. Herpes simplex encephalitis (HSE) is rare; however, it carries a high rate of death in up to 70% of patients. Almost all cases of HSE are caused by herpes sim- plex virus-1 (HSV-1). The virus, for unknown reasons, has a predilection for the temporal lobes. It can affect the temporal lobes and adjacent limbic areas usually asymmetrically in adults. In neonates, there can be more diffuse involvement. Diagnosis can be made on the basis of a combination of , CSF studies, and EEG. Initially, MRI and CT scans can show asymmetric involvement of one temporal lobe with bilateral involvement within a period of 7 to 10 days. HSE causes infl ammation, and even hemorrhage can be seen on imaging. EEG evaluation shows characteristic periodic lateralized epileptiform discharges from the temporal lobe. The sensitivity and specifi city of EEG are 84% and 32.5%, respectively. CSF analysis can show WBC count elevated to around 100 cells/µL and protein usually about 100 mg/dL. CSF HSV PCR study is the best choice for diagno- sis and has a sensitivity and specifi city of 94% and 98%, respectively. Treatment of HSE requires early treatment with antiviral drugs, usually acyclovir (2).

3. The answer is E. WNV is an arthropod-borne fl avivirus. It is transmitted from infected mosquito species to susceptible avian species, which amplify replication of the virus. The life cycle of the virus is then continued through “bridging” mosquitoes that infect other mammals, including humans. In humans, the majority (80%–90%) of infected patients develop West Nile fever, which is self-limited. Ten percent to 20% of patients can develop neuroinvasive disease, including West Nile meningitis, West Nile encephalitis, or West Nile poliomyelitis. The virus tends to affect the neurons if it invades the central nervous system (CNS). The areas in the CNS most commonly involved are the brainstem, basal ganglia, , cerebellum, and motor nuclei of the lower cranial nerves. WNV has not been seen to cause cerebral vasculitis. The areas of the peripheral nervous system that can be infected by WNV include the anterior horn motor neurons and nerve roots. Diagnosis of WNV is considered defi nitive in a patient with clinical symptoms of infec- tion and detection of WN virus–specifi c IgM antibodies in the cerebrospinal fl uid (CSF). WNV IgM antibodies can remain positive in the serum for more than a year, and thus a positive diagnosis of WNV infection based on serum studies must show a 4-fold rise in antibody titers to confi rm acute infection (3).

4. The answer is B. IE can cause multiple organ complications. Neurologic complications from IE portend a poor prognosis and occur in about 20% to 55% of IE cases. Left-sided valve IE with Staphylococcus aureus infection is a major risk factor for neurologic com- plications. Embolic strokes occur in 20% to 50% of cases, hemorrhagic strokes occur in 12% to 30% of cases, meningitis occurs in 2% to 20% of cases, and intracranial mycotic aneurysms occur in less than 10% of cases. Embolic strokes can be clinically silent or the presenting symptom of IE. The risk of embolic events is 20% to 50% but decreases sig- nifi cantly to 6% to 21% after antibiotic treatment is started. However, there is an increase in embolic events in the fi rst few days after initiation of antibiotics, and thus surgery is often recommended within the fi rst few days of presentation. Intracranial hemorrhage may result from hemorrhagic transformation of an embolic stroke, ruptured mycotic aneurysms, or even erosion of the vessel walls from septic emboli. Microbleeds have also been observed. Cerebral hemorrhage management is discussed in a later question. In meningitis cases associated with IE, the cerebrospinal fl uid (CSF) is only transiently

85

66485457-66485438 www.ketabpezeshki.com

ZZakaria_87574_PTR_CH06_10-06-13_79-94.inddakaria_87574_PTR_CH06_10-06-13_79-94.indd 8585 66/19/2013/19/2013 4:44:014:44:01 PMPM DISORDERS, DISEASES, SEIZURES, AND EPILEPSY

positive for the infective agent. Patients often present with fever, heart murmur, pete- chiae, and meningeal signs. Mycotic aneurysms are thought to result from arterial emboli that travel into the intraluminal space or vasa vasorum, or possibly from the spread of the infection hematogenously through the intimal vessels. Vessel imaging with CT or MR angiography is necessary. Four-vessel angiography should be performed if noninvasive imaging is negative and suspicion is high. Some unruptured mycotic aneurysms will resolve with antibiotic treatment and can be clinically silent. Ruptured, large, or enlarg- ing aneurysms need to have neurosurgical or endovascular intervention (4).

5. The answer is C. When a patient comes in with symptoms of meningitis, it is important that it is recognized and treated appropriately. A lumbar puncture (LP) and blood cultures should be obtained immediately. The LP should not be delayed unless there is suspicion of possible herniation. If there is a possible risk of mass lesions or increased intracranial pressure, then empiric antibiotic treatment is recommended. Empiric treatment should be chosen on the basis of the age of the patient. Listeria monocytogenes is more common in patients younger than 1 month or older than 50 years. Ampicillin is recommended plus cefotaxime or aminoglycoside for coverage of S. agalactiae, E. coli, and Klebsiella species in patients younger than 1 month. In patients age 1 to 23 months, the recommendation is to use vancomycin plus a third-generation cephalosporin to cover Streptococcus pneumoniae, N. meningitidis, S. agalactiae, H. infl uenzae, and E. coli. In patients age 2 to 50 years, the recommendation is again vancomycin and a third-generation cephalosporin to cover N. meningitidis and S. pneumoniae. Finally, vancomycin, ampicillin, and a third-generation cephalosporin are recommended in patients older than 50 years to cover S. pneumoniae, N. meningitidis, L. monocytogenes, and aerobic Gram-negative bacilli. In children, the use of dexamethasone is still debated, but may be recommended as adjunctive treatment in infants and children with H. infl uenzae B infection (5).

6. The answer is E. Malaria is transmitted to humans via a female Anopheles mosquito that carries the Plasmodium organism. Four different species of the genus Plasmodium are known to cause malaria: Plasmodium falciparum, Plasmodium ovale, Plasmodium vivax, and Plasmodium malariae. Clinically, the symptoms of malaria can be characterized by cyclic fevers. The most severe infection is caused by P. falciparum. Complications of malaria can involve the nervous system, causing seizures, altered mentation, and coma. Diagnosis of malaria can be suspected in patients with anemia and elevated levels of bilirubin, lactate dehydrogenase, and reticulocyte counts. Malaria is confi rmed by Giemsa stains on thick and thin blood smears and can also be detected with Wright’s and Field’s stains. Rapid diagnostic tests (RDTs) are also available, which detect plasmodial antigens or enzymes. Currently, most RDTs target P. falciparum–specifi c proteins (6,7).

7. The answer is B. This patient is most likely infected with P. falciparum malaria causing central nervous system (CNS) complications or “cerebral malaria.” IV artesunate (used in specialized centers) has now become the recommended treatment for cerebral malaria or severe malaria according to World Health Organization (WHO) recommendations. IV quinine (or quinidine in the United States) is still used as an alternative in countries that do not have IV artesunate readily available. Once the patient can swallow, the IV qui- nine is switched to oral quinine for 5 to 7 days with a 7-day course of doxycycline. Some serious side effects of quinine include arrhythmias, hypoglycemia, vision loss, hearing

86

66485457-66485438 www.ketabpezeshki.com

ZZakaria_87574_PTR_CH06_10-06-13_79-94.inddakaria_87574_PTR_CH06_10-06-13_79-94.indd 8686 66/19/2013/19/2013 4:44:014:44:01 PMPM CNS INFECTIONS: Answers

loss, hypotension, psychosis, and liver damage. Symptomatic treatment of CNS complica- tions such as cerebral edema is balanced with cerebral perfusion pressure optimization. Seizures are treated with phenobarbital or phenytoin and benzodiazepines, but down regulation of γ-amino butyric acid receptors from malaria infection can make treatment more diffi cult (8,9).

8. The answer is D. Diagnosis of cryptococcal central nervous system (CNS) infection in HIV patients is dependent on a high index of suspicion. CNS infection can involve the brain parenchyma and cause single or multiple focal mass lesions called cryptococcomas. CSF evaluation is essential for the diagnosis. The CSF pressure, cryptococcal antigen titer, CSF India ink positivity for Cryptococcus, and fungal culture can be helpful. Induction and consolidation treatment of patients with CNS cryptococcal disease is amphotericin B, 0.7 to 1 mg/kg/day plus fl ucytosine 100 mg/kg/day for 2 weeks, followed by fl ucon- azole 400 mg/day for at least 10 weeks. Maintenance treatment is required lifelong in patients with a history of cryptococcal CNS infection, and use of fl uconazole 200 to 400 mg orally daily is recommended, as is highly active antiretroviral therapy (HAART) (10).

9. The answer is D. Treatment of increased intracranial pressures in cryptococcal meningitis may be very diffi cult and not as responsive to standard treatment. A retrospective observa- tional study of 381 AIDS patients with cryptococcal meningitis showed that patients with

high baseline CSF opening pressures greater than or equal to 250 mmH2O were at higher risk for death in the fi rst 2 weeks of diagnosis. A clinically poor outcome (worsening or persistent signs of fungal meningitis) after 2 weeks of antifungal treatment was associated

with CSF pressures that had increased from baseline of greater than 10 mmH2O on the follow-up LP. Positive CSF culture for Cryptococcus after 2 weeks of antifungal treatment was seen more commonly in patients with higher CSF opening pressures greater than or

equal to 250 mmH2O, but this did not ultimately correlate with a poor prognosis at the 10-week assessment of the study (11).

10. The answer is E. Neurocysticercosis remains a very serious health problem in the world, especially in developing countries. In a majority of patients (70%) with neurocysticerco- sis, the primary manifestation is seizures, making it the leading cause of acquired epi- lepsy in the developing world. However, manifestations of neurocysticercosis are varied and numerous, depending on the number and location of central nervous system (CNS) lesions. Cysts can be located in the parenchyma, subarachnoid space, eyes, ventricles, and so on. Massive infection can even affect the muscles of the body. In the acute phase, cysts can cause surrounding edema and infl ammation, resulting in increased intracranial pressure usually due to hydrocephalus. If there are numerous cysticerci, there can be a severe immune response from the host resulting in an encephalitis picture and intrac- ranial hypertension. All of these symptoms (seizures, focal weakness, decreased vision, parkinsonian symptoms, etc.) can be manifestations of neurocysticercosis (12).

11. The answer is E. Ruling out bacterial meningitis is the obvious fi rst step in the approach to this patient. The Bacterial Meningitis Score developed in 2002 for pediatric patients can be used to risk stratify a patient. The variables that are considered are (a) positive CSF Gram stain, (b) CSF absolute neutrophil count ≥ 1,000 cells/µL, (c) CSF protein ≥ 80 mg/dL, (d) peripheral blood absolute neutrophil count ≥ 10,000 cells/µL, and (e) history of seizure

87

66485457-66485438 www.ketabpezeshki.com

ZZakaria_87574_PTR_CH06_10-06-13_79-94.inddakaria_87574_PTR_CH06_10-06-13_79-94.indd 8787 66/19/2013/19/2013 4:44:014:44:01 PMPM DISORDERS, DISEASES, SEIZURES, AND EPILEPSY

before or at the time of presentation. If the patient does not present with any of these vari- ables, the risk of bacterial meningitis is very low, with a negative predictive value of 100% and a 95% confi dence interval (97%–100%). Any of these medications, including IVIg and the pneumococcal vaccine, that the patient received could have caused her to have asep- tic meningitis. Once bacterial meningitis is ruled out, drug-induced aseptic meningitis is very likely, given the number of possible causative medications she took (13,14).

12. The answer is A. The man most likely has West Nile virus (WNV) infection causing poliomyelitis and meningitis. WNV belongs to the fl avivirus family and is transmitted to humans by infected mosquitoes. Poliomyelitis is the most common neuromuscular manifestation of WNV infection. It is often accompanied by meningitis, encephalitis, or respiratory distress, but can present as an acute fl accid paralysis without other signs of neuroinvasive disease. Patients with West Nile poliomyelitis may present with fever myal- gias, encephalopathy, and asymmetric weakness, which can progress quickly to quadri- plegia. CSF studies may show pleocytosis and elevated protein. Electrodiagnostic studies will show acute denervation in affected muscles with low-amplitude compound muscle action potentials and normal sensory nerve action potentials, which represent axonal loss or motor neuron involvement. Limbs with no motor response or no voluntary muscle activity on needle examination have a poor prognosis for recovery (15,16).

13. The answer is D. This patient has progressive multifocal leukoencephalopathy (PML), which is caused by a reactivation of the John Cunningham (JC) virus in the central ner- vous system (CNS). PML is a serious disease that unfortunately has no effective treat- ment. HAART treatment can lead to immune reconstitution infl ammatory syndrome (IRIS) and in some cases can lead to reactivation or enhanced immune response to an inactive or active infection, respectively. IRIS is defi ned as a paradoxical decline in the clinical status of a patient following immune restoration with antiretroviral therapy and has been commonly associated with M. tuberculosis, cryptococcal, and cytomegalovirus infections. Several factors have been shown to increase the risk of developing IRIS, includ- ing fi rst introduction of antiretroviral drugs, active or subclinical opportunistic infection at the time of initiation of combined antiretroviral therapy, low CD4 count (less than 50 cell/mm3), and rapid immune recovery based on a rapid decrease in HIV plasma viral load. Steroid treatment has been tried but shown to be defi nitely effective in only one study for the treatment of TB associated with IRIS. Although it may improve outcomes in other reactivated infections, adequate treatment trials have not been done to determine this (17–19).

14. The answer is C. The most common cause of spinal epidural abscess is still Staphylococcus aureus (two-thirds of patients). However, Pseudomonas and M. tuberculosis have been increas- ingly associated with IV drug use. Presentation of a spinal epidural abscess may start with back pain at the level of infection (Stage 1), and cause radicular-type pain in the same dermatomal level (Stage 2), progressing to weakness, sensory symptoms, or bladder and bowel dysfunction (Stage 3) and fi nally paralysis (Stage 4). The classic clinical triad is back pain, fever, and neurological signs, but these symptoms may not present in every patient. On laboratory testing, many patients will have leukocytosis and positive blood cultures for the offending bacteria. Cerebrospinal fl uid (CSF) studies are not routinely recommended because of the risk of meningitis or subdural infection. MRI with IV contrast of the spine

88

66485457-66485438 www.ketabpezeshki.com

ZZakaria_87574_PTR_CH06_10-06-13_79-94.inddakaria_87574_PTR_CH06_10-06-13_79-94.indd 8888 66/19/2013/19/2013 4:44:014:44:01 PMPM CNS INFECTIONS: Answers

is the diagnostic evaluation of choice and can help differentiate the mass from other types of tumors of the spine. Treatment includes emergency decompressive laminectomy if there are no contraindications and concurrent antibiotic (intravenous route usually) treatment for 6 weeks. Antimicrobial treatment is started with empiric antibiotics for staphylococcal infection (vancomycin for possible methicillin-resistant Staphylococcus aureus) and Gram- negative bacilli, usually third- or fourth-generation cephalosporin (20,21).

15. The answer is E. This patient suffers from Anton’s syndrome or cortical blindness, which is a stroke that affects both occipital cortices from occlusion of bilateral posterior cerebral arteries. Patients with this syndrome will often deny blindness or are not aware of their defi cit. Stroke or transient ischemic attacks (TIAs) comprise 40% to 50% of cases of neurologic complica- tions due to infective endocarditis (IE). Several factors that predispose patients to cerebral emboli include vegetation measuring greater than 10 to 15 mm in size, mobile vegetation, S. aureus infection, S. agalactiae or fungal infection (due to the size of the vegetation with these latter two infections), and/or mitral valve involvement. Right-sided IE is less likely to cause embolic strokes unless there is a right-to-left shunt. Appropriate treatment with antibiotics early after the recognition of IE can decrease the incidence of stroke (22,23).

16. The answer is C. The timing of surgery after an embolic event has been debated, and some have argued that waiting 2 weeks is necessary, whereas other studies have suggested that earlier surgery (median of 4–10 days) can be performed safely as well. Recommendations for management of infective endocarditis (IE) complications have been set forth by The Task Force on the Prevention, Diagnosis, and Treatment of Infective Endocarditis of the European Society of Cardiology: i. Surgery is recommended without delay (within a few days) after a silent cerebral embolism or transient ischemic attacks (TIAs). ii. After a stroke, surgery is indicated for heart failure, uncontrolled infection, abscess, or persisting high embolic risk and should not be delayed (within a few days, unless there is refractory pulmonary edema or cardiogenic shock, which then requires emer- gent surgery within 24 hours). This does not apply to comatose patients. iii. After intracranial hemorrhage, surgery should be postponed for at least 1 month. Treatment with antithrombotic medicines has been debated in the past, but the recom- mendations from the European Society of Cardiology argue that there is no indication for antithrombotic treatment unless the patient has been on antiplatelet or anticoagulant treatment for other appropriate reasons in the absence of cerebral hemorrhage. If patients were on antiplatelets, then this can be continued if there is no evidence of major hemor- rhage. In ischemic stroke patients without intracranial hemorrhage, oral anticoagulants need to be replaced with unfractionated heparin for 2 weeks with close monitoring of activated partial thromboplastin time. In the presence of intracranial hemorrhage, discon- tinuation of all anticoagulation is recommended. In patients with intracranial hemorrhage and a mechanical valve, unfractionated heparin is recommended as soon as possible with close monitoring of partial thromboplastin time (24).

17. The answer is D. Cerebral abscesses are fortunately rare. Patients can present with head- aches, altered mentation, focal neurological symptoms, and fever. Abscesses can be compli- cations of penetrating trauma of the cranium or neurosurgical operations. Hematogenous

89

66485457-66485438 www.ketabpezeshki.com

ZZakaria_87574_PTR_CH06_10-06-13_79-94.inddakaria_87574_PTR_CH06_10-06-13_79-94.indd 8989 66/19/2013/19/2013 4:44:014:44:01 PMPM DISORDERS, DISEASES, SEIZURES, AND EPILEPSY

or contiguous spread from infections of the oropharynx, paranasal sinuses, and the middle ear are also possible sources of abscesses. Infections can be bacterial, opportunistic, fungal, protozoal, or helminthic. Bacterial infection usually depends on the origin of the infection and the patient’s comorbidities. For example, streptococcal infection is usually due to infection from the nasopharynx or oropharynx. Staphylococci can be seen in cases asso- ciated with infective endocarditis (IE), trauma, or iatrogenic procedures. Opportunistic and fungal infections are more commonly encountered in immunocompromised patients. Initial antimicrobials are chosen empirically to treat polymicrobial etiologies, since this can be a common cause of abscesses. Choices are usually a third- or fourth-generation cephalosporin, metronidazole, and vancomycin to cover Gram-positive, Gram-negative, and anaerobic organisms. Open craniotomy versus closed stereotactic drainage has been controversial in the past, but recent reviews have found that stereotactic aspiration in the age of CT imaging has dramatically improved mortality rates compared with excision (6.6% vs. 12.7% mean mortality rates, respectively). Anticonvulsants are recommended early in the treatment, with or without a history of seizures, since they can occur in up to a quarter of patients with cerebral abscesses (25,26).

18. The answer is D. CNS toxoplasmosis infection is a common neurologic complica- tion of AIDS. Reactivation of latent toxoplasmosis parasite causes cerebral toxo- plasmosis and occurs usually in AIDS patients with CD4 cell counts below 200/µL. Patients often present with fever, headaches, seizures, and focal neurological defi cits. Cerebral toxoplasmosis presents with multiple ring-enhancing lesions in about 70% of patients, typically involving the parenchymal white matter or the subcortical gray matter, especially the thalamus and basal ganglia. Unfortunately, distinguishing cere- bral toxoplasmosis from lymphoma can often be diffi cult, and although lesions of pri- mary CNS lymphoma are often multifocal and periventricular, more often standard neuroimaging is insuffi cient to distinguish the two diseases. Ultimately, antitoxoplas- mosis treatment with pyrimethamine and sulfadiazine remains the most revealing method of distinguishing the two. Response to treatment for toxoplasmosis can be seen in 5 days with signifi cant improvement in more than 90% of patients in 14 days (27–29).

19. The answer is C. Tuberculous meningitis is a serious, devastating manifestation of extra- pulmonary TB infection. The exact incidence of extrapulmonary TB is not well known, but is expected to be high because of high rates of pulmonary TB in developing countries. One study in Germany found that out of 26,302 patients, 21.6% had extrapulmonary TB. In this study, risk of extrapulmonary TB was highest in females, children younger than 15 years, and patients immigrating from Asia or Africa. In patients with HIV, the rate of progression to extrapulmonary TB is fi ve times higher than in non-HIV patients. TB meningitis is thought to result from hematogenous spread from pulmonary TB result- ing in small granulomas in the brain parenchyma and the meninges. Patients with TB meningitis often present with advanced disease, with fever, headache, altered mental sta- tus (AMS), and meningismus. Cranial neuropathies, focal neurological symptoms, and raised intracranial pressure are seen in more severe cases. The gold standard of diagnosis for tuberculous meningitis is a culture positive for M. tuberculosis from the spinal fl uid; however, time and low sensitivity make this diffi cult. Polymerase chain reaction (PCR)

90

66485457-66485438 www.ketabpezeshki.com

ZZakaria_87574_PTR_CH06_10-06-13_79-94.inddakaria_87574_PTR_CH06_10-06-13_79-94.indd 9090 66/19/2013/19/2013 4:44:014:44:01 PMPM CNS INFECTIONS: Answers

for the organism has moderate sensitivity and excellent specifi city, 58% and 98%, respec- tively. Neuroimaging of TB meningitis often shows hydrocephalus, parenchymal and/or basilar cistern enhancement, infarctions, edema, or tuberculomas in decreasing frequency. Treatment of tuberculous meningitis relies on early recognition and antituberculous drugs, including isoniazid, rifampicin, streptomycin, ethambutol, and pyrazinamide, for 2 months. Isoniazid and rifampin are then continued for 7 or 10 months; some recommend longer, up to 12 months. A 2008 Cochrane Database review of the treatment of tuberculous meningitis shows that the use of steroids in the setting of tuberculous meningitis is recom- mended to decrease death and disability, but this was found to be signifi cant in only non- HIV patients. Studies in HIV patients were small and not powered to detect a signifi cant result, so a recommendation could not be made for this patient population (30,31).

20. The answer is E. Determining when a CT scan of the brain is necessary prior to LP in a suspected case of bacterial meningitis can be diffi cult. Concerns about delays in appropri- ate treatment and the fear of brain herniation are important considerations. In one study of 418 patients, the incidence of brain herniation after LP with papilledema was less than 1.2%, but in a group of 129 patients, 12.2% of patients without papilledema developed unfavorable outcomes. In another study of patients with bacterial meningitis, age older than 60 years, history of central nervous system (CNS) tumor, stroke, CNS infection, sei- zure less than 1 week prior to presentation, immunocompromise, and neurological defi - cits predicted an abnormal CT of the brain. Based on this study, Tunkel et al. in their paper made recommendations on when to do a CT scan prior to LP: A CT scan of the brain is necessary when patients present with new onset seizures, focal neurological signs, abnor- mal level of consciousness, papilledema, or history of CNS disease, including tumors, strokes, or focal infections (5).

References

1. Ahmed SV, Jayawarna C, Jude E. Post lumbar puncture headache: diagnosis and manage- ment. Postgrad Med J. 2006;82(973):713–716. 2. Whitley RJ. Herpes simplex encephalitis: adolescents and adults. Antiviral Res. 2006;71(2–3):141–148. 3. Sejvar JJ, Marfi n AA. Manifestations of West Nile neuroinvasive disease. Rev Med Virol. 2006;16(4):209–224. 4. Sonneville R, Mourvillier B, Bouadma L, Wolff M. Management of neurological complica- tions of infective endocarditis in ICU patients. Ann Intensive Care. 2011;1(1):10. 5. Tunkel AR, Hartman BJ, Kaplan SL, et al. Practice guidelines for the management of bacterial meningitis. Clin Infect Dis. 2004;39(9):1267–1284. 6. Tangpukdee N, Duangdee C, Wilairatana P, Krudsood S. Malaria diagnosis: a brief review. Korean J Parasitol. 2009;47(2):93–102. 7. Wilson W, Sande M. Current Diagnosis and Treatment in Infectious Diseases. New York, NY: Lange Medical Books/McGraw-Hill; 2001:888–889. 8. Mishra SK, Newton CR. Diagnosis and management of the neurological complications of falciparum malaria. Nat Rev Neurol. 2009;5(4):189–198. 9. Laloo DG, Shingadia D, Pasvol G, et al. UK malaria treatment guidelines. J Infect. 2007;54(2):111–121.

91

66485457-66485438 www.ketabpezeshki.com

ZZakaria_87574_PTR_CH06_10-06-13_79-94.inddakaria_87574_PTR_CH06_10-06-13_79-94.indd 9191 66/19/2013/19/2013 4:44:014:44:01 PMPM DISORDERS, DISEASES, SEIZURES, AND EPILEPSY

10. Saag M, Graybill RJ, Larsen RA, et al. Practice guideline for the management of cryptococ- cal disease. Infectious Diseases Society of America. Clinical Infect Dis. 2000;30(4):710–718. 11. Graybill JR, Sobel J, Saag M, et al. Diagnosis and management of increased intracranial pressure in patients with AIDS and cryptococcal meningitis. The NIAID Mycoses Study Group and AIDS Cooperative Treatment Groups. Clin Infect Dis. 2000;30(1):47–54. 12. Del Brutto OH. Neurocysticercosis: a review. Scientifi c World Journal. 2011;2012:1–8. 13. Nigrovic LE, Kuppermann N, Macias CG, et al. for identifying children with cerebrospinal fl uid pleocytosis at very low risk of bacterial meningitis. JAMA. 2007;297(1):52–60. 14. Jolles S, Sewell WA, Leighton C. Drug-induced aseptic meningitis: diagnosis and man- agement. Drug Saf. 2000;22(3):215–226. 15. Leis AA, Stokic DS. Neuromuscular manifestations of West Nile virus infection. Front Neurol. 2012;37(3):1–10. 16. Preston D, Shapiro B. Electromyography and Neuromuscular Disorders. Philadelphia, PA: Elsevier; 2005. 17. Tan K, Roda R, Ostrow L, McArthur J, Nath A. PML-IRIS in patients with HIV infection: clinical manifestations and treatment with steroids. Neurology. 2009;72(17):1458–1464. 18. Müller M, Wandel S, Colebunders R, Attia S, Furrer H, Egger M. Immune reconstitution infl ammatory syndrome in patients starting antiretroviral therapy for HIV infection: a systematic review and meta-analysis. Lancet Infect Dis. 2010;10(4):251–261. 19. Meintjes G, Wilkinson RJ, Morroni C, et al. Randomized placebo-controlled trial of pred- nisone for paradoxical tuberculosis-associated immune reconstitution infl ammatory syn- drome. AIDS. 2010;24(15):2381–2390. 20. Chuo CY, Fu YC, Lu YM, et al. Spinal infection in intravenous drug abusers. J Spinal Disord Tech. 2007;20(4):324–328. 21. Darouiche RO. Spinal epidural abscess. N Engl J Med. 2006;355(19):2012–2020. 22. Sonneville R, Mourvillier B, Bouadma L, Wolff M. Management of neurological complica- tions of infective endocarditis in ICU patients. Ann Intensive Care. 2011;1(1):10. 23. Brazis PW, Masdeu JC, Biller J. Localization in Clinical Neurology. 5th ed. Philadelphia, PA: Lippincott Williams & Wilkins; 2007. 24. Habib G, Hoen B, Tornos P, et al. Guidelines on the prevention, diagnosis, and treatment of infective endocarditis (new version 2009): the Task Force on the Prevention, Diagnosis, and Treatment of Infective Endocarditis of the European Society of Cardiology (ESC). Endorsed by the European Society of Clinical Microbiology and Infectious Diseases (ESCMID) and the International Society of Chemotherapy (ISC) for Infection and Cancer. Eur Heart J. 2009;30(19):2369–2413. 25. Honda H, Warren DK. Central nervous system infections: meningitis and brain abscess. Infect Dis Clin North Am. 2009;23(3):609–623. 26. Ratnaike TE, Das S, Gregson BA, Mendelow AD. A review of brain abscess surgical treatment—78 years: aspiration versus excision. World Neurosurg. 2011;76(5):431–436. 27. Kiderlen TR, Liesenfeld O, Schürmann D, Schneider T. Toxoplasmic encephalitis in AIDS- patients before and after the introduction of highly active antiretroviral therapy (HAART). Eur J Clin Microbiol Infect Dis. 2011;30(12):1521–1525. 28. Berger JR. Mass lesions of the brain in AIDS: the dilemmas of distinguishing toxoplasmo- sis from primary CNS lymphoma. AJNR Am J Neuroradiol. 2003;24(4):554–555.

92

66485457-66485438 www.ketabpezeshki.com

ZZakaria_87574_PTR_CH06_10-06-13_79-94.inddakaria_87574_PTR_CH06_10-06-13_79-94.indd 9292 66/19/2013/19/2013 4:44:014:44:01 PMPM CNS INFECTIONS: Answers

29. Chang L, Cornford ME, Chiang FL, Ernst TM, Sun NC, Miller BL. Radiologic-pathologic correlation. Cerebral toxoplasmosis and lymphoma in AIDS. AJNR Am J Neuroradiol. 1995;16(8):1653–1663. 30. Prasad K, Singh B. Corticosteroids for managing tuberculous meningitis. Cochrane Database Syst Rev. 2008;1:CD002244. 31. Garg RK. Tuberculous meningitis. Acta Neurol Scand. 2010;122(2):75–90. 32. Tunkel AR, Hartman BJ, Kaplan SL, et al. Practice guidelines for the management of bac- terial meningitis. Clin Infect Dis. 2004;39(9):1267–1284.

93

66485457-66485438 www.ketabpezeshki.com

ZZakaria_87574_PTR_CH06_10-06-13_79-94.inddakaria_87574_PTR_CH06_10-06-13_79-94.indd 9393 66/19/2013/19/2013 4:44:014:44:01 PMPM 66485457-66485438 www.ketabpezeshki.com

ZZakaria_87574_PTR_CH06_10-06-13_79-94.inddakaria_87574_PTR_CH06_10-06-13_79-94.indd 9494 66/19/2013/19/2013 4:44:014:44:01 PMPM 7 Toxic–Metabolic Disorders Elissa K. Fory QUESTIONS

1. A 68-year-old woman with bipolar disorder, depression, and hypertension was admitted to the hospital after a fall. Her home medications included lithium and fl uoxetine. She had never consumed alcohol because of religious beliefs. On initial examination, she had expressive aphasia but no weakness. A CT scan of the head revealed an acute left fronto- temporo-parietal subdural hematoma with mass effect and midline shift. The subdural was evacuated by craniotomy, and her aphasia improved. Three days after evacuation, she became confused and agitated. A CT scan of the head showed expected postoperative changes. A 24-hour electroencephalogram had diffuse slowing, with occasional sharp waves seen from the left temporal lobe. She was given small intermittent doses of haloperidol and loraze- pam for agitation. On postoperative day 4, vital signs were T 100.8°F, P 120 beats/minute, RR 18 breaths/minute, and BP fl uctuated from 90/50 mmHg to 150/80 mmHg. She was agitated, more confused, and diaphoretic. Her pupils were 6 mm and reactive in ambient light. She held both arms up antigravity without drift but was tremulous. Muscle tone was increased in bilateral lower extremities, and she had clonus on both patellar and ankle refl ex testing. Liver function tests and creatine phosphokinase (CPK) were normal. A lithium level was 0.7 mmol/L. What is the most likely diagnosis? A. Ethanol withdrawal B. Lithium toxicity C. Serotonin syndrome D. Neuroleptic malignant syndrome (NMS) E. Malignant hyperthermia (MH)

ANSWERS TO THIS SECTION CAN BE FOUND ON PAGE 100 95

66485457-66485438 www.ketabpezeshki.com

ZZakaria_87574_PTR_CH07_10-06-13_95-106.inddakaria_87574_PTR_CH07_10-06-13_95-106.indd 9595 66/19/2013/19/2013 8:45:488:45:48 PMPM DISORDERS, DISEASES, SEIZURES, AND EPILEPSY

2. What is the most common cause of acute liver failure in the United States? A. Acute hepatitis B infection B. Acute hepatitis C infection C. Idiosyncratic drug reactions to drugs such as phenytoin, valproic acid, ribavirin, and nucleoside reverse transcriptase inhibitors D. Autoimmune hepatitis E. Acetaminophen overdose

3. A 30-year-old man was admitted to an inpatient psychiatric hospital for depression with psychotic features. He was on escitalopram and quetiapine as an outpatient, and these medications were continued. Due to acute psychosis, he received multiple doses of IV haloperidol and the quetiapine was up titrated. One week after his hospitalization, he had gradual onset of slowed movements and decreased motor activity. On neurologic exami- nation 10 days after admission, he is awake but nonconversant. His pupils were 4 mm and briskly reactive. There was rigidity in all four extremities without cogwheeling, decreased deep tendon refl exes, and no clonus. His vitals were T 103°F, P 105 beats/minute, and BP 165/100 mmHg. Laboratory studies included a white blood cell (WBC) count of 12 × 103/µL and a creatine phosphokinase (CPK) of 10,000 units/L. All the following are reasonable options in the treatment of this patient’s condition except: A. Start cyproheptadine 4 mg PO q8h B. Start dantrolene 1 mg/kg IV push C. Start bromocriptine 2.5 mg PO q8h D. Stop haloperidol and quetiapine E. Start lorazepam 1 mg IV or intramuscular (IM) q6h

4. Which of the following choices best describes the inheritance pattern and the associated molecular abnormality in malignant hyperthermia (MH)? A. Autosomal dominant, myosin kinase B. Autosomal dominant, peripheral myelin protein 22 (PMP 22) C. X-linked, dystrophin protein D. Autosomal recessive, α-sarcoglycan protein E. Autosomal dominant, ryanodine receptor

5. A 45-year-old man is brought to the ED after being found unresponsive on a city side- walk. The EKG shown at the top of the following page is obtained. What is the most likely diagnosis? A. Benzodiazepine overdose B. Hypothermia C. Tricyclic antidepressant (TCA) overdose D. Subarachnoid hemorrhage E. Hypercalcemia

96

66485457-66485438 www.ketabpezeshki.com

ZZakaria_87574_PTR_CH07_10-06-13_95-106.inddakaria_87574_PTR_CH07_10-06-13_95-106.indd 9696 66/19/2013/19/2013 8:45:488:45:48 PMPM TOXIC–METABOLIC DISORDERS: Questions

Source: Reproduced with permission of the publisher. Copyright 2008 Society of Critical Care Medicine (31).

6. A 35-year-old woman with a medical history only of depression is admitted with lethargy and acute respiratory failure after taking sixty 100 mg tablets of amitriptyline. She had last been seen 4 hours prior to being discovered with altered mental status. Her initial vital signs were T 38°C, P 120 beats/minute, RR 20 breaths/minute, and BP 120/60 mmHg. In the ED, a large-bore nasogastric tube was placed after intubation, but no pill fragments were recovered after water lavage. Normal saline was started at 75 mL/hour. An EKG showed sinus tachycardia with PR interval 0.18 seconds (s), QRS interval 0.12 seconds, and QTc 0.60 seconds. Which of the following treatments is classically initiated in this scenario? A. β-blockers such as esmolol or metoprolol B. Amiodarone C. Calcium gluconate D. Sodium bicarbonate E. Magnesium sulfate

7. Which of the following is not found in the clinical presentation of serotonin syndrome? A. Miosis B. Clonus C. Diarrhea D. Fever E. Hypertension

97

66485457-66485438 www.ketabpezeshki.com

ZZakaria_87574_PTR_CH07_10-06-13_95-106.inddakaria_87574_PTR_CH07_10-06-13_95-106.indd 9797 66/19/2013/19/2013 8:45:488:45:48 PMPM DISORDERS, DISEASES, SEIZURES, AND EPILEPSY

8. Because of the short half-life of naloxone, patients with opiate overdose may be placed on continuous intravenous infusions of naloxone. What is the most accurate statement about the half-life of intravenous naloxone and its duration of action when given as a one-time dose? A. 5 to 7 minutes; 5 to 7 minutes B. 5 to 7 minutes; 20 to 30 minutes C. 60 to 90 minutes; 10 to 90 minutes D. 60 to 90 minutes; 60 to 90 minutes E. 60 to 90 minutes; 240 to 360 minutes

9. The risk for clinically signifi cant arrhythmias such as atrial fi brillation or ventricular fi brillation increases when the body temperature falls below: A. 36°C B. 34°C C. 32°C D. 30°C E. 28°C

10. A 30-year-old woman with no medical or surgical history undergoes an emergent appen- dectomy. Thirty minutes after extubation, she is noted to be hot to touch and to have mus- cle rigidity in her jaw and extremities. Her vitals signs are T 41°C, P 140 beats/minute, RR 30 breaths/minute, and BP 100/50 mmHg. A portable chest x-ray was clear. Brown urine is noted in the catheter bag. What is the best treatment for her underlying condition? A. Succinylcholine B. Dantrolene C. Broad-spectrum antibiotics D. Rocuronium E. Lorazepam

11. A 45-year-old physician from Seattle travels to Houston in July for a medical conference. His regular exercise schedule is running approximately 3 miles, two to three times per week. He wants to take advantage of his extra time away from work and goes on a 5-mile run at noon. He carries a 1-L bottle of water with him. After the run, he has generalized weakness and feels nauseated. He is walking back to his hotel room when he vomits and then has syncope. EMS is called, and he is taken to the ED. There, he is awake and fully oriented, but feels generally weak. His vital signs are T 38.5°C, P 120 beats/minute, RR 12 breaths/minute, BP 90/55 mmHg. His neurologic examination is normal. Laboratory data include sodium of 130 mmol/L, creatinine of 1.0 mg/dL, CPK of 400 units/L, WBC 8 × 103/µL, and a normal urinalysis. Which of the following best describes this patient’s condition? A. Heat syncope B. Heat cramp C. Heat exhaustion D. Exertional heat stroke E. Nonexertional heat stroke

98

66485457-66485438 www.ketabpezeshki.com

ZZakaria_87574_PTR_CH07_10-06-13_95-106.inddakaria_87574_PTR_CH07_10-06-13_95-106.indd 9898 66/19/2013/19/2013 8:45:498:45:49 PMPM TOXIC–METABOLIC DISORDERS: Questions

12. A young adult is brought to the ED for altered mental status and combative behavior. The police were called after he was wandering drunkenly on the street, shouting paranoid phrases about the government and his parents. When the offi cers tried to take him into custody, he fl ew into a rage and it took fi ve men to restrain him and place him in handcuffs. The police offi cers called EMS because they were concerned about his altered mentation. Initial vital signs in the ED were T 37°C, P 130 beats/minute, RR 14 breaths/ minute, and BP 180/100 mmHg. The patient would alternate between being quiet and staring with eyes open to being extremely agitated and requiring leather four-point restraints. His skin was fl ushed, he had copious saliva manifested by drooling, and there were mild expira- tory wheezes. His pupils were 2 mm and reactive, and there was spontaneous rotatory nystagmus noted and vertical nystagmus on upward gaze. Face was symmetrical, and strength was full and symmetrical. Which of the following is most likely to be positive on urine toxicology screening? A. Cocaine B. Opiates C. Methylenedioxymethamphetamine (MDMA; “Ecstasy”) D. Phencyclidine (“PCP”) E. Amphetamines

13. Several unrelated persons in a large metropolitan city are brought into a local ED because of illness after they exited the subway at the same stop. All patients had marked miosis. Other symptoms and signs varied among the patients and included headache, rhinor- rhea, salivation, nausea, vomiting, diarrhea, cough, shortness of breath, blurry vision, and muscle twitching. One patient developed seizures and respiratory failure requiring mechanical ventilation. What is the mechanism of action of the toxin responsible for this clinical scenario? A. Anticholinergic activity B. Excessive cholinergic activity C. Antidopaminergic activity D. Excessive dopaminergic activity E. Excessive histaminergic activity

14. In the clinical scenario described in Question 13, what is the treatment of choice? A. Pralidoxime B. Flumazenil C. Naltrexone D. Physostigmine E. Methylene blue

15. Which of the following is not a side effect of barbiturate overdose, either accidental or therapeutic (such as thiopental or pentobarbital coma for status epilepticus)? A. Hypotension B. Immunosuppression C. Gastroparesis D. Hypokalemia E. Ventricular tachycardia

99

66485457-66485438 www.ketabpezeshki.com

ZZakaria_87574_PTR_CH07_10-06-13_95-106.inddakaria_87574_PTR_CH07_10-06-13_95-106.indd 9999 66/19/2013/19/2013 8:45:498:45:49 PMPM 7

ANSWERS

1. The answer is C. Serotonin syndrome is a clinical diagnosis and requires that the patient had been recently exposed to a serotonergic agent. Common symptoms of serotonin syndrome include agitation or confusion, fever, hypertension, tachycardia, diaphoresis, mydriasis, tremor, akathisia, clonus, and muscle rigidity. Many drugs have been impli- cated in the development of serotonin syndrome, including selective serotonin reuptake inhibitors, monoamine oxidase inhibitors, other antidepressants, opiates such as meper- idine and tramadol, lithium, and triptans. Although some of the clinical features of sero- tonin syndrome overlap with NMS, NMS is associated with normal pupil size, rigidity in all limbs, hyporefl exia, and a state of quiet delirium or mutism (1). Therefore, NMS would not be the best answer for this patient. Alcohol withdrawal can indeed have symptoms of delirium and autonomic instability. However, hyperrefl exia does not develop over time, and muscle tone and pupils should be normal. Lithium toxicity is not the correct answer, as the lithium level was within therapeutic range. Clinical signs and symptoms of acute lithium toxicity include dysarthria, ataxia, nystagmus, nausea, vomiting, diarrhea, and nephrogenic diabetes insipidus (2). MH is not the correct answer, as it develops soon after general anesthesia and is associated with very high fever, pipe-like rigidity, and hypore- fl exia (1).

2. The answer is E. Viral hepatitides are the most common cause of acute liver failure world- wide (3), but in the United States acetaminophen overdose is the culprit in 39% to 51% of cases. Idiosyncratic drug reactions are the second most common reason for acute liver failure, accounting for 13% of the cases (4,5).

3. The answer is A. This patient has neuroleptic malignant syndrome (NMS). Cypro- heptadine is a histamine and serotonin antagonist, and can be used in the treatment of serotonin syndrome, but does not have action on dopamine receptors. Choices

100

66485457-66485438 www.ketabpezeshki.com

ZZakaria_87574_PTR_CH07_10-06-13_95-106.inddakaria_87574_PTR_CH07_10-06-13_95-106.indd 100100 66/19/2013/19/2013 8:45:498:45:49 PMPM TOXIC–METABOLIC DISORDERS: Answers

B through E are reasonable options in the treatment of NMS. Dantrolene is a skeletal muscle relaxant that works by decreasing the release of intracellular calcium from the sarcoplasmic reticulum. It is used in the treatment of malignant hyperthermia (MH) and moderate to severe NMS. In cases of signifi cant rigidity, dantrolene at doses of 1 to 2.5 mg/kg IV push can be given. Bromocriptine is a dopamine receptor agonist and also has indications for treatment in Parkinsonism and in prolactinomas. Certainly, all dop- amine antagonists should be discontinued in the setting of NMS. Lorazepam is consid- ered a fi rst-line agent for sedation and to promote muscle relaxation. Finally, isotonic IV fl uids should be given to all patients due to insensible losses from fever and to prevent acute renal failure from rhabdomyolysis (6).

4. The answer is E. Autosomal dominant is the inheritance pattern, and ryanodine receptor is the associated molecular abnormality. Choice A describes the inheritance of myotonic dystrophy. Choice B describes the inheritance of Charcot–Marie–Tooth disease (CMT-1a) or hereditary neuropathy with liability to pressure palsies when there is a gene duplica- tion or deletion, respectively. Choice C describes inheritance of Duchenne muscular dys- trophy. Choice D is the defect in one of the limb girdle muscular dystrophies (7).

5. The answer is B. The EKG shows Osborn waves, a characteristic J-point elevation of at least 1-mm elevation at the end of the QRS complex found in patients with hypo- thermia. In one study, Osborn waves were found in every patient whose temperature was less than or equal to 30.5°C. The height of the Osborn wave is inversely related to body temperature (8). Benzodiazepine overdose is infrequently associated with reversible atrioventricular block (9). TCA overdose does not cause Osborn waves, but rather leads commonly to QT prolongation and prolongation of the QRS interval (10). Common EKG changes seen after subarachnoid hemorrhage include ST-segment eleva- tion or depression, inverted or peaked T waves, Q waves, or prolonged QTc interval (11). Severe hypercalcemia can cause lethargy and even coma; the EKG shows a short- ened QTc interval (12).

6. The answer is D. The evidence to use sodium bicarbonate in tricyclic antidepressant (TCA) poisoning is from case series and animal studies; there have been no randomized clinical trials testing this treatment. One series showed that a majority of patients treated had normalization or near-normalization of the QRS complex after being treated with sodium bicarbonate. Bicarbonate should be given for wide complex tachyarrhythmias after TCA overdose and may be considered to a pH goal of 7.45 to 7.55 in patients with a wide QRS or prolonged QTc interval (13,14).

7. The answer is A. Miosis is not a part of serotonin syndrome; rather, pupils tend to be dilated and reactive (mydriasis). Clonus may be present, especially in the lower extremi- ties. Bowel function tends to be increased in serotonin syndrome, with diarrhea and hyperactive bowel sounds (1).

8. The answer is C. Naloxone has a half-life of about 60 to 90 minutes (15). In some stud- ies, the duration of action of naloxone has been shown to be as long as its half-life (16). However, in other studies and in clinical practice, the duration of effect seems to be much shorter than the pharmacokinetic elimination, as the drug may be redistributed out of the brain more rapidly than opiates (17).

101

66485457-66485438 www.ketabpezeshki.com

ZZakaria_87574_PTR_CH07_10-06-13_95-106.inddakaria_87574_PTR_CH07_10-06-13_95-106.indd 101101 66/19/2013/19/2013 8:45:498:45:49 PMPM DISORDERS, DISEASES, SEIZURES, AND EPILEPSY

9. The answer is D. Clinically signifi cant arrhythmias are rare with body temperatures greater than 30°C. Atrial fi brillation is more commonly seen than ventricular fi brillation or ventricular tachycardia (8,18).

10. The answer is B. This patient has malignant hyperthermia (MH). MH is a syndrome of muscle hypermetabolism that follows the administration of succinylcholine or inhaled anesthetic agents. Various muscle protein abnormalities predispose persons to MH, with the most common being a ryanodine receptor mutation. The clinical syndrome can begin during induction of anesthesia, intraoperatively, or in the several hours after sur- gery. Clinical signs and symptoms include marked hyperpyrexia, tachycardia, cyanosis,

masseter muscle spasm, muscle rigidity, a rise in end-tidal CO2, and rhabdomyoly- sis. Dantrolene is the mainstay of therapy, which decreases the excitation–contraction coupling in skeletal muscle by reducing the amount of calcium released from the sar- coplasmic reticulum, with no effect on the neuromuscular junction. With aggressive recognition and treatment, mortality from MH has fallen from 70% to less than 10% (19). Succinylcholine would be the wrong answer, as it is associated with MH and could worsen the syndrome (20). Broad-spectrum antibiotics would be used in the setting of sepsis. Although sepsis could cause fever, tachycardia, tachypnea, and hypotension, it does not cause muscle rigidity and rhabdomyolysis. Therefore, antibiotics would not treat the underlying condition, making them an incorrect response. In a real-life sce- nario, it is likely that antibiotics would be given (in addition to dantrolene) while cul- tures were pending. Nondepolarizing muscle blockers such as rocuronium are generally considered safe in MH (20), but would not treat the MH itself. Finally, benzodiazepines can be used to promote muscle relaxation and are the fi rst-line treatment in the rigidity associated with neuroleptic malignant syndrome (NMS) (6). However, benzodiazepines alone would not suffi ciently treat the severe muscle rigidity of MH.

11. The answer is C. Heat exhaustion describes a syndrome of generalized weakness, nau- sea, vomiting, and syncope without neurologic dysfunction or tissue damage. The core temperature may be somewhat elevated, but is not extremely elevated. In heat exhaus- tion, salt and/or water losses from sweating lead to hypovolemia and syncope. Heat syncope describes syncope due to peripheral vasodilation from elevated environmental temperature. Heat cramp refers to exertional cramping in the heat due to dehydration. It would not be the right answer here, as the patient had no cramps. Heat stroke is a loss of thermoregulation, which leads to an extremely elevated core body temperature (gener- ally defi ned as greater than 40.6°C); neurologic dysfunction (such as delirium, lassitude, seizures, and coma); and anhidrosis. There may be signs or symptoms of other end-organ or tissue damage such as acute renal failure, elevated liver function tests, cardiac conduc- tion defects, disseminated intravascular coagulopathy, or rhabdomyolysis. Heat stroke is important to recognize, as early treatment—decreasing the core temperature to below 38.9°C within 30 minutes—may decrease mortality. Heat stroke may be exertional or non- exertional. Exertional heat stroke is usually seen in previously healthy persons exercising or working in a hot and humid environment. Nonexertional heat stroke is generally seen in debilitated or elderly persons with a prolonged exposure to high temperatures and humidity (21,22). The two most reasonable answers for this patient’s condition are heat exhaustion or exertional heat stroke. Since his core temperature was well below 40.6°C

102

66485457-66485438 www.ketabpezeshki.com

ZZakaria_87574_PTR_CH07_10-06-13_95-106.inddakaria_87574_PTR_CH07_10-06-13_95-106.indd 102102 66/19/2013/19/2013 8:45:498:45:49 PMPM TOXIC–METABOLIC DISORDERS: Answers

and he had no overt neurologic dysfunction other than syncope, heat exhaustion is the better answer.

12. The answer is D. PCP is a hallucinogen, which has an affi nity for multiple types of neu- rotransmitter receptors. It antagonizes N-methyl-d-aspartate receptors, inhibits mono- amine reuptake, and has cholinergic effects via both nicotinic and muscarinic receptors. Persons intoxicated with PCP classically have periods of super-human strength and seem anesthetized to painful stimuli. They often have ataxia and vertical or rotatory nystagmus. Owing to monoamine reuptake inhibition, PCP has a sympathomimetic effect with hyper- tension and tachycardia. Cholinergic activity can lead to miosis, salivation, bronchospasm, and diaphoresis (23). Although cocaine or amphetamines can cause sympathomimetic effects and paranoia, they do not cause ataxia or bizarre nystagmus. Opiate intoxication would cause sedation, depressed RR, miosis, constipation, and analgesia. MDMA is both a stimulant and a hallucinogen; it increases the release of serotonin, dopamine, and nor- epinephrine and then decreases their metabolism via monoamine oxidase inhibition. Classically, MDMA intoxication is associated with severe hyperthermia. Like cocaine and PCP, MDMA can cause sympathomimetic effects, paranoia, and hallucinations (24). The lack of ataxia, nystagmus, and cholinergic signs helps to distinguish MDMA intoxication from PCP intoxication.

13. The answer is B. This scenario describes a hypothetical sarin nerve gas attack, as occurred in Japan in the mid-1990s. Sarin is an organophosphate and acts via acetyl-cholinesterase inhibition, leading to excessive acetylcholine action and cholinergic crisis. Sarin acts on both the peripheral and the central nervous system and on both the nicotinic and the muscarinic acetylcholine receptors. Symptoms include neuromuscular weakness, loss of deep tendon refl exes, muscle fasciculations, seizures, altered mental status, miosis, blurred vision, ocular pain, tachycardia or bradycardia, and the SLUDGE symptoms of salivation, lacrimation, urination, diarrhea, gastrointestinal upset (nausea), and emesis. Serum cholinesterase or erythrocyte acetyl-cholinesterase activity may be measured, but treatment should not be delayed for the results (25). Anticholinergic syndrome would be manifested peripherally with decreased bowel sounds, dry and warm skin, dry mouth, mydriasis, urinary retention, and tachycardia. Central signs of anticholinergic syndrome include either a depressed level of consciousness (sedation or coma) or agitation, anxiety, and disorientation (26). Excessive antidopaminergic activity could result in parkinsonism, tardive dyskinesia, and/or hyperprolactinemia. Excessive dopaminergic activity, such as in overdoses of dopamine agonists like pramipexole or bromocriptine, can lead to psy- chosis, orthostasis, nausea, hyperkinetic movement disorders (dyskinesias), and patho- logical addictive behaviors. Finally, excessive histamine release can lead to bronchocon- striction, vasodilation, nasal congestion and rhinorrhea, hives, erythema, and gastric acid secretion.

14. The answer is A. Pralidoxime is the antidote to organophosphates (27) and was effective in reversing the signs of cholinergic poisoning (miosis and respiratory depression) in the patients in Japan when given up to 6 hours after exposure to sarin (25). One should note that the most current Cochrane review does not support giving oximes for organophos- phate poisoning because of lack of suffi cient evidence (28). Flumazenil is the antidote to

103

66485457-66485438 www.ketabpezeshki.com

ZZakaria_87574_PTR_CH07_10-06-13_95-106.inddakaria_87574_PTR_CH07_10-06-13_95-106.indd 103103 66/19/2013/19/2013 8:45:498:45:49 PMPM DISORDERS, DISEASES, SEIZURES, AND EPILEPSY

benzodiazepines. Naltrexone is an oral opiate antagonist. Physostigmine is an anticholin- esterase inhibitor and can be used to treat overdoses of anticholinergic agents, but would worsen these patients’ current symptoms. Finally, methylene blue is given for the treat- ment of methemoglobinemia (27).

15. The answer is E. High-dose barbiturate therapy is known to be associated with gastropa- resis, immunosuppression, respiratory failure, hypotension, and poikilothermia. Infusions of thiopental have also been associated with life-threatening hypokalemia and rebound hyperkalemia on infusion discontinuation (29). Ventricular tachycardia is not associated with barbiturate therapy, and in fact, barbiturates have been associated with decreased incidence of torsades de points in animal models with prolonged QT intervals (30).

References

1. Boyer EW, Shannon M. The serotonin syndrome. N Engl J Med. 2005;352(11):1112–1120. 2. Grandjean EM, Aubry JM. Lithium: updated human knowledge using an evidence-based approach: part III: clinical safety. CNS Drugs. 2009;23(5):397–418. 3. Khashab M, Tector AJ, Kwo PY. Epidemiology of acute liver failure. Curr Gastroenterol Rep. 2007;9(1):66–73. 4. Ostapowicz G, Fontana RJ, Schiødt FV, et al. Results of a prospective study of acute liver fail- ure at 17 tertiary care centers in the United States. Ann Intern Med. 2002;137(12):947–954. 5. Larson AM, Polson J, Fontana RJ, et al. Acetaminophen-induced acute liver failure: results of a United States multicenter, prospective study. Hepatology. 2005;42(6):1364–1372. 6. Strawn JR, Keck PE Jr, Caroff SN. Neuroleptic malignant syndrome. Am J Psychiatry. 2007;164(6):870–876. 7. Bird TD, Tapscott SJ. Clinical neurogenetics. In: Bradley WG, Daroff RB, Fenichel GM, Jankovic J, eds. Neurology in Clinical Practice. Philadelphia, PA: Butterworth-Heinemann; 2004:803–807. 8. Vassallo SU, Delaney KA, Hoffman RS, Slater W, Goldfrank LR. A prospective evalu- ation of the electrocardiographic manifestations of hypothermia. Acad Emerg Med. 1999;6(11):1121–1126. 9. Arroyo Plasencia AM, Ballentine LM, Mowry JB, Kao LW. Benzodiazepine-associated atrioventricular block. Am J Ther. 2012;19(1):e48–e52. 10. Harrigan RA, Brady WJ. ECG abnormalities in tricyclic antidepressant ingestion. Am J Emerg Med. 1999;17(4):387–393. 11. Sommargren CE. Electrocardiographic abnormalities in patients with subarachnoid hem- orrhage. Am J Crit Care. 2002;11(1):48–56. 12. Diercks DB, Shumaik GM, Harrigan RA, Brady WJ, Chan TC. Electrocardiographic mani- festations: electrolyte abnormalities. J Emerg Med. 2004;27(2):153–160. 13. Kerr GW, McGuffi e AC, Wilkie S. Tricyclic antidepressant overdose: a review. Emerg Med J. 2001;18(4):236–241. 14. Body R, Bartram T, Azam F, Mackway-Jones K. Guidelines in Emergency Medicine Network (GEMNet): guideline for the management of tricyclic antidepressant overdose. Emerg Med J. 2011;28(4):347–368. 15. Glass PS, Jhaveri RM, Smith LR. Comparison of potency and duration of action of nalme- fene and naloxone. Anesth Analg. 1994;78(3):536–541.

104

66485457-66485438 www.ketabpezeshki.com

ZZakaria_87574_PTR_CH07_10-06-13_95-106.inddakaria_87574_PTR_CH07_10-06-13_95-106.indd 104104 66/19/2013/19/2013 8:45:498:45:49 PMPM TOXIC–METABOLIC DISORDERS: Answers

16. Kaufman RD, Gabathuler ML, Bellville JW. Potency, duration of action and pA2 in man of intravenous naloxone measured by reversal of morphine-depressed respiration. J Pharmacol Exp Ther. 1981;219(1):156–162. 17. Clarke SF, Dargan PI, Jones AL. Naloxone in opioid poisoning: walking the tightrope. Emerg Med J. 2005;22(9):612–616. 18. Polderman KH, Herold I. Therapeutic hypothermia and controlled normothermia in the intensive care unit: practical considerations, side effects, and cooling methods. Crit Care Med. 2009;37(3):1101–1120. 19. Krause T, Gerbershagen MU, Fiege M, Weisshorn R, Wappler F. Dantrolene—a review of its pharmacology, therapeutic use and new developments. Anaesthesia. 2004;59(4):364–373. 20. Hopkins PM. Malignant hyperthermia: advances in clinical management and diagnosis. Br J Anaesth. 2000;85(1):118–128. 21. Grogan H, Hopkins PM. Heat stroke: implications for critical care and anaesthesia. Br J Anaesth. 2002;88(5):700–707. 22. Dematte JE, O’Mara K, Buescher J, et al. Near-fatal heat stroke during the 1995 heat wave in Chicago. Ann Intern Med. 1998;129(3):173–181. 23. Bey T, Patel A. Phencyclidine intoxication and adverse effects: a clinical and pharmaco- logical review of an illicit drug. Cal J Emerg Med. 2007;8(1):9–14. 24. Smith KM, Larive LL, Romanelli F. Club drugs: methylenedioxymethamphetamine, fl unitrazepam, ketamine hydrochloride, and gamma-hydroxybutyrate. Am J Health Syst Pharm. 2002;59(11):1067–1076. 25. Yanagisawa N, Morita H, Nakajima T. Sarin experiences in Japan: acute toxicity and long- term effects. J Neurol Sci. 2006;249(1):76–85. 26. Torline RL. Extreme hyperpyrexia associated with central anticholinergic syndrome. Anesthesiology. 1992;76(3):470–471. 27. Dart RC, Goldfrank LR, Chyka PA, et al. Combined evidence-based literature analysis and consensus guidelines for stocking of emergency antidotes in the United States. Ann Emerg Med. 2000;36(2):126–132. 28. Buckley NA, Eddleston M, Li Y, Bevan M, Robertson J. Oximes for acute organophos- phate pesticide poisoning. Cochrane Database Syst Rev. 2011;2:CD005085. 29. Neil MJ, Dale MC. Hypokalaemia with severe rebound hyperkalaemia after therapeutic barbiturate coma. Anesth Analg. 2009;108(6):1867–1868. 30. Weissenburger J, Nesterenko VV, Antzelevitch C. Transmural heterogeneity of ventric- ular repolarization under baseline and long QT conditions in the canine heart in vivo: torsades de pointes develops with halothane but not pentobarbital anesthesia. J Cardiovasc Electrophysiol. 2000;11(3):290–304. 31. Stahmer SA. Arrhythmia. In: Rehm CGR, Fuhrman TMF, eds. Adult Problem-Based Learning Discussions. Mt. Prospect, IL: Society of Critical Care Medicine; 2008:8.

105

66485457-66485438 www.ketabpezeshki.com

ZZakaria_87574_PTR_CH07_10-06-13_95-106.inddakaria_87574_PTR_CH07_10-06-13_95-106.indd 105105 66/19/2013/19/2013 8:45:498:45:49 PMPM 66485457-66485438 www.ketabpezeshki.com

ZZakaria_87574_PTR_CH07_10-06-13_95-106.inddakaria_87574_PTR_CH07_10-06-13_95-106.indd 106106 66/19/2013/19/2013 8:45:498:45:49 PMPM 8 Infl ammatory and Demyelinating Diseases Flavia Nelson QUESTIONS

1. What is the mechanism of action related to a reduction in the total lymphocyte count of the FDA-approved oral drug for multiple sclerosis (MS)? A. Attaches to VLA-1 and blocks interaction with its ligand on central nervous system (CNS) endothelium VCAM-1 B. Induces TH2 lymphocytes C. Blocks capacity of lymphocytes to egress from lymph nodes by binding to sphingosine- 1-phosphate receptors D. Suppresses T-cell activation and inhibits binding of T-cell receptor complexes E. Unknown

ANSWERS TO THIS SECTION CAN BE FOUND ON PAGE 114 107

66485457-66485438 www.ketabpezeshki.com

ZZakaria_87574_PTR_CH08_10-06-13_107-120.inddakaria_87574_PTR_CH08_10-06-13_107-120.indd 107107 66/19/2013/19/2013 8:46:118:46:11 PMPM DISORDERS, DISEASES, SEIZURES, AND EPILEPSY

2. A 20-year-old woman presented with headache, feverish sensation, and anxiety, rapidly followed by homicidal ideation, aggressive agitation, seizures, oral dyskinesias, hypoven- tilation, hyperthermia, and prominent autonomic instability requiring intubation and sedation. She developed episodes of hypotension and bradycardia with periods of asystole lasting up to 15 seconds. Upon weaning off sedation, her eyes opened but she was unre- sponsive to stimuli. There was muscle rigidity, frequent facial grimacing, lip smacking, rhythmic abdominal contractions, kicking motions of the legs, and intermittent dystonic postures of the right arm. Cerebrospinal fl uid (CSF) showed a WBC count of 28 with lym- phocytic pleocytosis, an elevated protein of 30, normal glucose and normal IgG index, no oligoclonal bands (OCBs), negative herpes simplex virus (HSV) polymerase chain reaction (PCR), and negative cytology. Blood cultures were also negative. Electroencephalogram showed diffuse slowing without focal temporal lobe abnormalities. Cranial MRI showed no abnormalities. Conventional angiography was normal. was inconclusive. An ovarian teratoma was found on further workup. What is the diagnosis? A. Limbic encephalitis B. Rabies C. Acute disseminated encephalomyelitis (ADEM) D. N-methyl-d-aspartate (NMDA) receptor encephalitis E. Central Nervous System (CNS) vasculitis

3. A 35-year-old Hispanic woman is in the medical ICU for plasma exchange (PE) for the treatment of vision loss refractory to IV steroids. She originally presented to the ED with a 6-day history of painful vision loss in her right eye. A 5-day course of IV methylpredniso- lone was given without clinical improvement. Two years ago, she was evaluated because of severe refractory nausea, hiccups, and diffi culty walking from weakness in her left leg. These symptoms lasted several weeks. Her weakness never fully recovered. Her brain MRI at that time was normal. Which would be the next step in managing this patient? A. Spinal tap B. Initiate treatment with PE and order brain MRI w/contrast C. Initiate treatment with PE and order brain and spine MRI w/contrast D. Initiate treatment with a disease-modifying agent E. Treat the patient with PE and observe

4. The patient from Question 3 was treated with plasma exchange (PE) with signifi cant improve- ment. Brain MRI was normal. Spinal MRI was performed, which showed a hyperintense T2 lesion extending from C2 to C6. The gold standard for diagnosis of this entity is: A. Presence of oligoclonal bands (OCBs) B. Positive IgG for aquaporin 4 antibody C. Low glucose levels with increased WBC count D. Protein 14–3-3 E. Increased IgG index and synthesis rate

108

66485457-66485438 www.ketabpezeshki.com

ZZakaria_87574_PTR_CH08_10-06-13_107-120.inddakaria_87574_PTR_CH08_10-06-13_107-120.indd 108108 66/19/2013/19/2013 8:46:118:46:11 PMPM INFLAMMATORY AND DEMYELINATING DISEASES: Questions

5. Which of the following medications has shown to be benefi cial in the treatment of neuro- myelitis optica (NMO)? A. Chronic use of oral prednisone B. Glatiramer acetate C. Natalizumab D. Rituximab E. Interferon-β1a

6. A 30-year-old woman with a history of recurrent aphthous mouth ulcerations, uveitis, and skin ulcers presents with headache and ataxia. In the ED, she has a generalized tonic– clonic seizure and is intubated. MRI of the brain shows lesions predominantly in the brain- stem. Cerebrospinal fl uid (CSF) analysis shows pleocytosis, elevated protein, and normal glucose. This disease has been associated to which of the following? A. HLADR15 B. HLA-B5 and HLA-B1 C. HLA-DQB1–0602 D. Cytoplasmic antineutrophil cytoplasmic antibodies (C-ANCA) E. HLA B27

7. A 40-year-old woman with a 1 year medical history of worsening symptoms manifested by skin abnormalities, diffi culty breathing, and hearing loss presents to the ED with cognitive decline and seizures. She has been diagnosed with a condition associated to antibodies Scl-70. Biopsy shows thickening of the skin as well as fi brosis. The patient is most likely suffering from: A. Progressive systemic sclerosis B. Sjögren’s syndrome C. Systemic lupus erythematosus (SLE) D. Rheumatoid arthritis E. Thromboangiitis obliterans

8. Which of the following medications has been associated with aseptic meningitis? A. Amphotericin B B. Nalidixic acid C. Ethambutol D. Vancomycin E. Penicillins

9. A 50-year-old man on treatment with tacrolimus for an autoimmune condition develops mental changes, seizures, and blindness. This medication has been associated to which of the following diseases? A. Acute demyelinating encephalomyelitis (ADEM) B. Posterior reversible encephalopathy syndrome (PRES) C. Multiple sclerosis (MS) D. Benign intracranial hypertension E. Vestibular damage

109

66485457-66485438 www.ketabpezeshki.com

ZZakaria_87574_PTR_CH08_10-06-13_107-120.inddakaria_87574_PTR_CH08_10-06-13_107-120.indd 109109 66/19/2013/19/2013 8:46:118:46:11 PMPM DISORDERS, DISEASES, SEIZURES, AND EPILEPSY

10. A 70-year-old woman presents with a 2-week history of jaw claudication, neck pain, and vertigo. Brain imaging studies (CT and MRI) were reported as normal. Patient was treated symptomatically with poor response. Which is the most likely diagnosis? A. Cerebrovascular disease B. Polymyalgia rheumatica C. Takayasu’s arteritis D. Temporal arteritis E. Wegener’s granulomatosis

11. A 45-year-old right-handed man with a history of relapsing remitting multiple sclerosis (RRMS) being treated with natalizumab (Tysabri) presents to the ED with a 4-week his- tory of cognitive decline, ataxia, and recent right hemiparesis and vision disturbances. He deteriorates over the next couple of days and is transferred to the ICU after intubation for airway protection. Initial head CT scan showed multiple foci of hypodensities. Brain MRI showed new large T2 hyperintense white matter lesions beginning at gray–white mat- ter junctions and coalescing to form confl uent lesions. No enhancements were noticed. Cerebrospinal fl uid (CSF) examination would most likely show which of the following? A. Protein kinase inhibitor (protein 14–3-3) B. Depressed glucose levels C. Positive capsular polysaccharide antigen titer D. John Cunningham (JC) virus by polymerase chain reaction (PCR) E. Presence of oligoclonal bands (OCBs) and increased IgG index

12. Despite appropriate management, the patient in Question 11 dies a few months later. Which fi nding would you expect to fi nd by brain pathology? A. Leptomeningeal infl ammatory infi ltrates with enlarged cystic spaces B. Reactive astrocytosis with bizarre appearance of giant astrocytes C. Caseating granulomas with multinucleated giant cells and activated macrophages D. Intracellular vacuolation of neuropil primarily affecting gray matter

13. From the options below please choose the one that best fi ts the following description: Aggressive form of demyelinating disease that presents with extensive white matter lesions in the brain and spine. On MR imaging the demyelinating lesions tend to show homogeneous enhancement with gadolinium. This entity has an acute onset, rapid pro- gression and may cause death. A. Devic’s disease neuromyelitis optica (NMO) B. Acute disseminated encephalomyelitis (ADEM) C. Marburg’s variant D. Tumefactive multiple sclerosis (MS) E. Posterior reversible encephalopathy syndrome (PRES)

110

66485457-66485438 www.ketabpezeshki.com

ZZakaria_87574_PTR_CH08_10-06-13_107-120.inddakaria_87574_PTR_CH08_10-06-13_107-120.indd 110110 66/19/2013/19/2013 8:46:118:46:11 PMPM INFLAMMATORY AND DEMYELINATING DISEASES: Questions

14. A 23-year-old woman with a history of migraines presents to the ED with concerns for a stroke. She reports right-arm numbness that began 3 days ago and right-arm weakness since this morning, both becoming progressively worse. She is only on oral contracep- tives. She also describes a tingling sensation traveling down her right arm when bend- ing her neck forward. Hypercoagulable and cardiovascular workup are normal. Brain MRI shows six nonenhancing white matter lesions in both cerebral hemispheres—fi ve perpendicular to the ventricles and one juxtacortical—all of which are seen on T2 FLAIR (fl uid attenuated inversion recovery), with no enhancements on T1W post contrast series. Cervical MRI shows a small white matter lesion on the left side of the spinal cord at the level of C6 with gadolinium enhancement. What would be the best next step? A. She meets the 2010 McDonald diagnostic criteria for multiple sclerosis (MS) and needs treatment with IV methylprednisolone B. She needs to have cerebrospinal fl uid (CSF) analysis to confi rm diagnosis of MS and evoked potentials prior to the administration of IV methylprednisolone C. She needs repeated hypercoagulable workup for possible stroke as she is on oral contraceptives D. Start aspirin E. She has acute disseminated encephalomyelitis (ADEM) and needs treatment with IV methylprednisolone

15. A 35-year-old woman with aggressive multiple sclerosis (MS) is admitted to the ICU to receive her fi rst rituximab infusion. Shortly after beginning the infusion, she becomes tachycardic and febrile. She is immediately taken off the drug and is given a histamine blocker. She improves within 30 minutes and becomes stable again. The best next step is: A. To not give her rituximab ever again, as she had an anaphylactic reaction to it B. To wait 30 to 60 minutes, give IV corticosteroids, and start the rituximab infusion at half of the rate it was given and titrated to tolerance C. To give her dopamine to prevent further blood pressure drop D. To try rituximab infusion at half the dose and over a longer infusion time E. To premedicate with IV corticosteroids and an H2 blocker and try rituximab infusion at half the dose and over a longer infusion time

16. A 27-year-old woman presents with a 2-week history of progressive right hemiparesis. MRI of the brain demonstrates a 4.5-cm enhancing mass in the left posterior frontal white matter, with several smaller lesions in the white matter of the contralateral hemisphere. A stereotactic biopsy shows no evidence of neoplasia, but reveals demyelinated neurons with axonal sparing and an infi ltrate of foamy macrophages and perivascular lympho- cytes. What would be the most appropriate diagnosis? A. Marburg’s variant B. Primary progressive multiple sclerosis (MS) C. Tumefactive MS D. False negative for malignancy E. Balo’s concentric sclerosis

111

66485457-66485438 www.ketabpezeshki.com

ZZakaria_87574_PTR_CH08_10-06-13_107-120.inddakaria_87574_PTR_CH08_10-06-13_107-120.indd 111111 66/19/2013/19/2013 8:46:128:46:12 PMPM DISORDERS, DISEASES, SEIZURES, AND EPILEPSY

17. A 30-year-old man with an unremarkable medical history wakes up with right upper extremity weakness. He is also experiencing tingling sensation along his spine when he moves his neck forward. He has never experienced these symptoms before. He recalls that for the past month, he has been feeling more tired than usual and that a year ago, he had tingling sensation on his left leg that lasted for a week and resolved. A cervical MRI shows a 4.5-mm lesion in the right anterior horn at the level of C6–C8 with mild enhance- ment with gadolinium. Thoracic MRI shows multiple patch-like lesions at the levels of T2–T3, T5–T6, and T9–T11, none of them enhancing. Brain MRI shows two small white matter lesions perpendicular to the left ventricle. What is the most likely diagnosis? A. Recurrent transverse myelitis B. Relapsing remitting multiple sclerosis (RRMS) C. Devic’s disease

D. Vitamin B12 defi ciency E. Idiopathic transverse myelitis 18. A 7-year-old boy with no signifi cant medical history is taken to the ED by his parents for altered mental status and right hemiparesis. One week ago, he had new onset of persistent headaches with personality changes 3 days ago, and last night, his mother noted mild weak- ness on the right side. Three weeks ago, he missed 2 days of school because of a cold that he caught from his little sister. Brain MRI shows large, confl uent white matter lesions in both cerebral hemispheres that enhance with gadolinium. What is the most likely diagnosis? A. Guillain–Barré syndrome B. Multiple sclerosis (MS) C. Clinically isolated syndrome D. Acute disseminated encephalomyelitis (ADEM) E. Viral encephalitis 19. Infection with which pathogen carries the highest risk for acute disseminated encephalo- myelitis (ADEM)? A. Measles B. Epstein-Barr syndrome (EBS) C. Varicella-Zoster Virus (VZV) D. Haemophilus infl uenzae E. Cytomegalovirus (CMV) 20. The following symptoms are typical of acute disseminated encephalomyelitis (ADEM) except: A. Sensory defi cit B. Encephalopathy C. Seizures D. Ataxia E. Ascending paralysis 21. The most typical presentation of acute disseminated encephalomyelitis (ADEM) is: A. Sudden loss of consciousness, fever, headache, malaise, after the development of neu- rological symptoms B. Ascending paralysis, fever, diarrhea, and personality changes that develop over hours C. Headache, low-grade fever, myalgias, and malaise prior to the onset of neurological symptoms

112

66485457-66485438 www.ketabpezeshki.com

ZZakaria_87574_PTR_CH08_10-06-13_107-120.inddakaria_87574_PTR_CH08_10-06-13_107-120.indd 112112 66/19/2013/19/2013 8:46:128:46:12 PMPM INFLAMMATORY AND DEMYELINATING DISEASES: Questions

22. The following are all in the differential diagnosis of acute disseminated encephalomyeli- tis (ADEM) except: A. Multiple sclerosis (MS) B. Neuromyelitis optica (NMO) C. Guillain–Barré syndrome D. Balo’s concentric sclerosis E. Posterior reversible encephalopathy syndrome (PRES)

23. A 40-year-old African American man with a medical history of hypertension was seen by his ophthalmologist for subacute loss of vision in his left eye. During the eye examination, he was found to have papilledema and visual fi eld defects. He was sent to a neurologist for evaluation and started on IV steroids for 5 days. His vision improved promptly. A brain MRI showed abnormal hyperintense T2 signal in the left optic nerve. A month later, he again presented with subacute vision loss in his left eye and mild changes in his right eye vision. A second round of steroids was given, with improvement in both eyes. A new MRI of the brain shows faint enhancement of both optic nerves and meningeal enhance- ment. Cerebrospinal fl uid (CSF) analysis is done and is positive for oligoclonal bands (OCBs), elevated protein, elevated WBC count, and elevated angiotensin-converting enzyme (ACE) levels in CSF and blood. What is the most likely diagnosis? A. Neuromyelitis optica (NMO) B. Multiple sclerosis (MS) C. Neurosarcoidosis D. Recurrent viral optic neuritis (ON) E. Ischemic ON

24. Which of the following is/are the most common central nervous system (CNS) manifestation(s) of systemic lupus erythematosus (SLE)? A. Stroke (arterial) B. Acute confusional state, psychosis C. Ataxia D. Venous thrombosis E. Chorea, parkinsonism

25. A 64-year-old woman admitted to the ICU for severe hyponatremia secondary to diuretic use develops worsening confusion, dysarthria, and dysconjugate gaze over 24 hours. She has a history of an upper respiratory tract infection 3 weeks earlier, at which time the serum sodium was normal. MRI of the brain is most likely to show: A. Brainstem ischemic infarct B. Abnormal T2 signal in the pons C. Cerebral edema D. Multiple T2 hyperintense lesions in the white matter E. Cerebellar tonsil herniation

113

66485457-66485438 www.ketabpezeshki.com

ZZakaria_87574_PTR_CH08_10-06-13_107-120.inddakaria_87574_PTR_CH08_10-06-13_107-120.indd 113113 66/19/2013/19/2013 8:46:128:46:12 PMPM 8

ANSWERS

1. The answer is C. Currently, two oral therapies are approved for relapsing remitting mul- tiple sclerosis (RRMS) (fi ngolimod—United States/Russia and cladribine—Australia/ Russia). Three are in Phase 3 trials (BG-12, laquinimod, and terifl unomide). FTY720 (fi n- golimod), the only one approved in the United States, itself is not bioactive, but when phos- phorylated by sphingokinase 2, it becomes active through modulation of four of the fi ve known G protein–coupled sphingosine-1-phosphate (S1P) receptors and alters lympho- cyte traffi cking by trapping them in the peripheral lymph nodes. Circulating lymphocyte counts reduce rapidly when therapy is started, remain stable during chronic treatment, and on average return to the normal range within 6 weeks after therapy is stopped (1).

2. The answer is D. NMDA receptor encephalitis typically begins as a fulminant enceph- alopathy, with prominent neuropsychiatric manifestations, seizures, dyskinesias, hypoventilation, and autonomic instability in young people. After this often-dramatic presentation, one to three relapses may occur. Most patients either die or recover from the disease. It is associated with antibodies against NR1–NR2 heteromers of the NMDA receptor. It can be associated with tumors, especially ovarian teratoma. Tumor removal and immunotherapy can reverse the symptoms (2).

3, 4, and 5. The answers are C, B, and D, respectively. The classical presenting clinical fea- tures of NMO are optic neuritis (ON) and acute transverse myelitis. These events can occur simultaneously or in close succession. ON can be severe and steroid resistant. Spinal cord attacks are often but not always clinically severe, and accompanied by a lesion that spans three or more vertebral segments on MRI. Bilateral limb paresis, spinal cord sensory syn- dromes, and bowel and bladder dysfunction are hallmarks of individual events. Events that involve the brainstem can be manifested by refractory nausea, vomiting, or hiccups owing to lesions in the pericanal region of the medulla, likely affecting the area postrema

114

66485457-66485438 www.ketabpezeshki.com

ZZakaria_87574_PTR_CH08_10-06-13_107-120.inddakaria_87574_PTR_CH08_10-06-13_107-120.indd 114114 66/19/2013/19/2013 8:46:128:46:12 PMPM INFLAMMATORY AND DEMYELINATING DISEASES: Answers

and medial lateral portions of the nucleus tractus solitarius. Diagnostic criteria for NMO have recently been revised to incorporate the high specifi city of NMO IgG (aquaporin 4) antibody testing. Two of the elements of the NMO diagnostic criteria are based on neuroim- aging assessments. Most patients with NMO have normal brain MRI or only nonspecifi c white matter lesions that fail to meet multiple sclerosis (MS) imaging criteria, especially early in the disease. Second and more specifi c is the fi nding of a longitudinally extensive spinal cord lesion in association with acute transverse myelitis. Such lesions are defi ned by a contiguous T2 hyperintensity extending over three or more vertebral segments of the spi- nal cord and often have a core of T1 hypointensity. Treatment of the acute clinical relapses include IV corticosteroids, such as methylprednisolone. Oral azathioprine and chimeric anti-CD20 monoclonal protein rituximab are now commonly used (3).

6. The answer is B. Behcet’s disease is a relapsing infl ammatory disorder without a defi ned cause. It may affect the central nervous system (CNS), and it has a preference for the brainstem and diencephalon. Vasculitis is a characteristic feature (a third of the cases) with narrowing, occlusion, and aneurysm formation (88% venous). The brain- stem is vulnerable because of poor venous collaterals. It may be a viral or autoimmune disorder, although familial cases and an association with HLA-B5 and HLA-B1 support a genetic role. HLADR15 has been associated to multiple sclerosis (MS). HLA-DQB1– 0602 has been associated to narcolepsy, and cytoplasmic antineutrophil cytoplasmic antibodies (C-ANCA) to Wegener’s granulomatosis. HLA-B27 is associated to ankylos- ing spondylitis (4,5).

7. The answer is A. Progressive systemic sclerosis is a condition associated with thickening of the skin and subcutaneous tissues as well as smooth muscle atrophy and fi brosis of internal organs such as the gastrointestinal tract, lungs, heart, and kidneys. Antinuclear antibodies are typically found, usually in a nucleolar pattern; antibodies to Scl-70 are spe- cifi c. Central nervous system (CNS) disease has been reported. It may present as global cognitive decline or as a focal lesion. Encephalopathy, migraine, psychiatric changes, sei- zures, and focal neurological defi cits have also been reported (6).

8. The answer is E. Penicillins and cephalosporins have been associated with aseptic men- ingitis. Amphotericin B and nalidixic acid have been associated with benign intracranial hypertension. Ethambutol has been associated with cerebellar ataxia and optic neuritis (ON). Vancomycin may cause cochlear and vestibular damage (7).

9. The answer is B. Neurotoxicity is a signifi cant complication of the use of immunosup- pressive medications, especially tacrolimus and cyclosporine, both of which have been associated with PRES. This syndrome is characterized clinically by mental status changes, seizures, and cortical blindness that are usually reversible with discontinuation of the offending medication (8).

10. The answer is D. Giant cell arteritis (temporal arteritis, GCA) is a large vessel, T-cell mediated vasculitis. CD4 cells aggregate with a response centered on the internal elastic lamina. Pain is commonly experienced by patients with GCA. Headache, sometimes with scalp tenderness (most common symptom), jaw claudication, and neck pain have also been associated to the disease. Erythrocyte sedimentation rate (ESR) is usually more than 30 mm/hour. Symptoms usually last for at least 2 weeks. Diagnosis can be confi rmed by biopsy (9).

115

66485457-66485438 www.ketabpezeshki.com

ZZakaria_87574_PTR_CH08_10-06-13_107-120.inddakaria_87574_PTR_CH08_10-06-13_107-120.indd 115115 66/19/2013/19/2013 8:46:128:46:12 PMPM DISORDERS, DISEASES, SEIZURES, AND EPILEPSY

11. The answer is D. Progressive multifocal leukoencephalopathy (PML) is an opportunis- tic infection caused by the JC virus. It usually occurs in the late stages of HIV infection and has recently been associated with the use of natalizumab in patients with mul- tiple sclerosis (MS). Clinical manifestations are variable, depending on the localization and extent of lesions. Common features include cognitive defi cits, focal paralysis or generalized weakness, visual disturbances, and gait abnormalities. The most common manifestation in AIDS patients is hemiparesis. Brain MRI shows hyperintense T2 sub- cortical white matter multifocal lesions beginning at the gray–white matter junction and coalescing to form confl uent lesions. Lesions appear hypointense in T1 images. Typically, there is little or no enhancement unlike active MS lesions. The prognosis of natalizumab-associated PML is better than in AIDS patients. Diagnosis can be made by CSF analysis, which shows the presence of JC virus by PCR. This test has a sensitivity of more than 80% and specifi city of more than 90%. Once the test is positive, plasma exchange (PE) can be done to remove natalizumab from the system. A negative test does not exclude diagnosis (10).

12. The answer is B. In progressive multifocal leukoencephalopathy (PML) pathology, mac- roscopic appearance shows confl uent demyelination predominantly at juxtacortical white matter or near the deep gray matter (in contrast to multiple sclerosis (MS) lesions, which have predilection for periventricular white matter). Microscopically, there are enlarged oligodendrocytes fi lled with virions and reactive astrocytosis with bizarre appearing giant astrocytes (10).

13. The answer is C. Marburg’s variant is a rapidly progressive demyelinating process, with extensive lesions that can be multifocal, located in the cerebral hemispheres. It can also involve brainstem and optic nerves. It can present as a pseudotumor variant with head- ache, vomiting, altered mental status, and/or focal neurological defi cits depending on the sites of the lesions. It differentiates from the tumefactive form by the contrast uptake, which is homogeneous in the Marburg’s variant. Pathological studies will show extensive necrosis, massive macrophage infi ltration in the acute lesions, and a severe and extensive demyelination. Cerebrospinal fl uid (CSF) analysis may show mononuclear pleocytosis. Oligoclonal bands (OCBs) are usually absent (11).

14. The answer is A. The patient meets the criteria of dissemination in place and time (pre- vious possible left optic neuritis [ON]) and new episode of partial transverse myelitis at the level of C6. In addition, she has at least six other white matter lesions in the typical location for diagnosis of multiple sclerosis (MS). CSF is not necessary to confi rm diagnosis in her case. IV methylprednisolone treatment is suggested. The possibility of a stroke is reduced given the imaging, which is diagnostic for MS (12).

15. The answer is B. This patient most likely experienced an infusion reaction due to cytokine release, which is common in patients who take rituximab for the very fi rst time. The appropriate treatment is to wait 30 to 60 minutes and give corticosteroids before starting the rituximab infusion at half of the previous rate and titrate to tolerance. Repeating the medication challenge depends on the severity of the reaction. It is absolutely contraindi- cated if the patient experienced a severe or true anaphylactic reaction (13).

116

66485457-66485438 www.ketabpezeshki.com

ZZakaria_87574_PTR_CH08_10-06-13_107-120.inddakaria_87574_PTR_CH08_10-06-13_107-120.indd 116116 66/19/2013/19/2013 8:46:128:46:12 PMPM INFLAMMATORY AND DEMYELINATING DISEASES: Answers

16. The answer is C. Tumefactive MS may mimic the clinical and MRI characteristics of glioma or a cerebral abscess. Usually more than 50% of the lesions are gadolinium enhancing and tend to respond to IV steroids. The gold standard for their diagnosis and proper treatment is a biopsy, which will show typical histopathology of a demyelinating disease (14).

17. The answer is B. The patient is experiencing an episode of acute partial (incomplete) transverse myelitis (APTM). Patients with APTM have greater chances of converting to multiple sclerosis (MS). Earlier studies have clearly shown that patients with asymmetric and patchy spinal cord abnormal signal had converted to clinically defi nite MS (CDMS) within 3 years. Thirteen out of 15 patients (87%) who converted had abnormal brain MRI at onset of disease. The length of the lesions is typically shorter than two vertebral seg- ments (15).

18. The answer is D. ADEM is often a monophasic demyelinating disorder that usually begins within 6 weeks after a viral infection or immunization. It causes rapid develop- ment of multifocal or focal neurological symptoms. Brain MRI shows enlarged and con- fl uent white matter edematous lesions which tend to enhance simultaneously or nearly simultaneously (16).

19. The answer is A. ADEM has been reported to follow a number of immunizations, usu- ally within 6 weeks, including those for pertussis, diphtheria, measles, mumps, rubella, infl uenza, tetanus, and yellow fever. Parainfectious ADEM usually follows onset of the infectious illness, often during the recovery phase, but because of the latency between pathogen exposure and illness, it may precede clinical symptoms of infection or the two may occur simultaneously. Measles carries the highest risk for ADEM of any infection, occurring in 1 per 400 to 1,000 cases (16).

20. The answer is E. From all of the symptoms listed, ascending paralysis is not typical of ADEM. Ascending paralysis is seen more commonly in Guillain–Barré syndrome (16).

21. The answer is C. Typical clinical features of ADEM are prodrome of headache, low- grade fever, myalgia, and malaise, often preceding the onset of ADEM by a few days. The most frequent clinical signs are motor defi cits followed by sensory defi cits, brain- stem signs, and cerebellar signs. Neurological symptoms develop rapidly in the acute phase and are commonly associated with encephalopathy, stupor, coma, meningismus, and seizures. Peak severity occurs within several days, and recovery may begin soon afterward (16).

22. The answer is C. Guillain–Barré syndrome is a peripheral nervous system demyelinat- ing disease. ADEM is usually distinguished from multiple sclerosis (MS) by a history of antecedent vaccination or infection. ADEM usually has a rapid onset and involvement of the gray and white matter structures as well as the cerebellum is often seen on MRI. It can also commonly present with seizures and altered consciousness. ADEM, MS, and NMO are Central Nervous System (CNS) demyelinating diseases (17).

117

66485457-66485438 www.ketabpezeshki.com

ZZakaria_87574_PTR_CH08_10-06-13_107-120.inddakaria_87574_PTR_CH08_10-06-13_107-120.indd 117117 66/19/2013/19/2013 8:46:128:46:12 PMPM DISORDERS, DISEASES, SEIZURES, AND EPILEPSY

23. The answer is C. Recurrent steroid-dependent ON is a presenting form of neurosarcoido- sis. The presence of elevated ACE levels in the CSF and serum is suggestive of the disease. Further tests such as gallium 67 scan and biopsy must be done to confi rm the diagnosis and establish a defi nite treatment (18).

24. The answer is B. SLE is an autoimmune disease manifested by malar rash, photosensi- tivity, arthritis, sclerosis, mucosal ulcers, and many other systemic symptoms. Central nervous system (CNS) involvement is common. Fifty percent of the patients can have a neuropsychiatric presentation. The most common symptom is acute confusional state, psychosis, and dementia. Other CNS symptoms are stroke (most important manifesta- tion), venous thrombosis, ataxia, and movement disorders (19).

25. The answer is B. Central pontine myelinolysis is in most cases associated with rapid correction of hyponatremia or hyperosmolar states. It typically presents in a devastat- ing fashion as quadriplegia and pseudobulbar palsy, partial forms present as confusion, dysarthria, and/or disturbances of conjugate gaze without quadriplegia. MRI is useful in establishing the diagnosis, showing a symmetric area of abnormal high signal inten- sity in the pons on T2-weighted images; occasional cases present with lesions outside the brainstem. Therapeutic guidelines for restoration of severe hyponatremia should aim for gradual correction, that is, by 10 mmol/L (10 mEq/L) within 24 hours and 20 mmol/L (20 mEq/L) within 48 hours (20).

References

1. Brinkmann V, Billich A, Baumruker T, et al. Fingolimod (FTY720): discovery and develop- ment of an oral drug to treat multiple sclerosis. Nat Rev Drug Discov. 2010;9(11):883–897. 2. Dalmau J, Gleichman AJ, Hughes EG, et al. Anti-NMDA-receptor encephalitis: case series and analysis of the effects of antibodies. Lancet Neurol. 2008;7(12):1091–1098. 3. Wingerchuk DM. Neuromyelitis optica spectrum disorders. Continuum (Minneap Minn). 2010;16(5 Multiple Sclerosis):105–121. 4. Bartt R, Shannon K. Autoimmune and infl ammatory disorders. In: Goetz CG, ed. Textbook of Clinical Neurology. 2nd ed. Philadelphia, PA: Elsevier Science; 2003:1124–1125. 5. Grossman RI, Yousem DM. Neuroradiology: The Requisites. 2nd ed. Philadelphia, PA: Mosby; 2003:201. 6. Bartt R, Shannon K. Autoimmune and infl ammatory disorders. In: Goetz CG, ed. Textbook of Clinical Neurology. 2nd ed. Philadelphia, PA: Elsevier Science; 2003:1116–1117. 7. Kompoliti K, Horn S. Drug-induced and iatrogenic neurological disorders. In: Goetz CG, ed. Textbook of Clinical Neurology. 2nd ed. Philadelphia, PA: Elsevier Science; 2003:1233. 8. Lessig S, Corey-Bloom J. Systemic diseases. In: Corey J, David R, eds. Clinical Adult Neurology. 3rd ed. New York, NY: Demos Medical Publishing; 2009:429. 9. Slevin J, Ryan M. Headaches. In: Corey J, David R, eds. Clinical Adult Neurology. 3rd ed. New York, NY: Demos Medical Publishing; 2009:143. 10. Tracy J, Mowzoon N. Neurology of infectious diseases. In: Mowzoon N, Flemming K, eds. Neurology Board Review: An Illustrated Study Guide. Mayo Foundation for Medical Education and Research. Mayo Clinic Scientifi c Press. Rochester, MN. 2007:613–614.

118

66485457-66485438 www.ketabpezeshki.com

ZZakaria_87574_PTR_CH08_10-06-13_107-120.inddakaria_87574_PTR_CH08_10-06-13_107-120.indd 118118 66/19/2013/19/2013 8:46:128:46:12 PMPM INFLAMMATORY AND DEMYELINATING DISEASES: Answers

11. Barahona-Strauch J. Encefalomielitis diseminada aguda y sclerosis multiple. In: Arriagada CR, Nogales-Gaete J, eds. Esclerosis Multiple Una Mirada Ibero-Panamericana. New York, NY: Demos Medical; 2008:555–563. 12. Polman CH, Reingold SC, Banwell B, et al. Diagnostic criteria for multiple sclerosis: 2010 revisions to the McDonald criteria. Ann Neurol. 2011;69(2):292–302. 13. Vogel WH. Infusion reactions: diagnosis, assessment, and management. Clin J Oncol Nurs. 2010;14(2):E10–E21. 14. Dagher AP, Smirniotopoulos J. Tumefactive demyelinating lesions. Neuroradiology. 1996; 38(6):560–565. 15. Pandit L. Transverse myelitis spectrum disorders. Neurol India. 2009;57(2):126–133. 16. Pirko I, Noseworthy JH. Demyelinating disorders of the central nervous system. In: Goetz CG, ed. Textbook of Clinical Neurology. Philadelphia, PA: Elsevier Science; 2003:1059–1082. 17. Cree BA. Diagnosis and differential diagnosis of multiple sclerosis. Continuum (Minneap Minn). 2010;16(5 Multiple Sclerosis):19–36. 18. Zajicek JP, Scolding NJ, Foster O, et al. Central nervous system sarcoidosis—diagnosis and management. QJM. 1999;92(2):103–117. 19. Kanttarci OH. Infl ammatory and demyelinating disorders of the central nervous system. In: Mowzoon N, Flemming K, eds. Neurology Board Review: An Illustrated Study Guide. Mayo Foundation for Medical Education and Research. Mayo Clinic Scientifi c Press. Rochester, MN. 2007:576–578. 20. Hauser SL, Josephson SA. Harrison’s Neurology in Clinical Medicine. 2nd ed. New York, NY: McGraw-Hill; 2010:290.

119

66485457-66485438 www.ketabpezeshki.com

ZZakaria_87574_PTR_CH08_10-06-13_107-120.inddakaria_87574_PTR_CH08_10-06-13_107-120.indd 119119 66/19/2013/19/2013 8:46:128:46:12 PMPM 66485457-66485438 www.ketabpezeshki.com

ZZakaria_87574_PTR_CH08_10-06-13_107-120.inddakaria_87574_PTR_CH08_10-06-13_107-120.indd 120120 66/19/2013/19/2013 8:46:128:46:12 PMPM 9 Neuroendocrine Disorders Matthew Flaherty and Howard J. Fan QUESTIONS

1. An 18-year-old African American woman, with no medical history, presented with a temperature of 101.2°F, photophobia, and nuchal rigidity. Cerebrospinal fl uid was col- lected with an elevated opening pressure and the following chemistries: white blood cells 234 mm3, glucose 23 mg/dL, protein 300 mg/dL. She was diagnosed with bacte- rial meningitis and started on vancomycin and ceftriaxone. On hospital day 2, she developed polyuria. Serum sodium was 158 mEq/L (135–145 mEq/L), urine osmolality

240 mOsmol/kgH2O (50–1,200 mOsmol/kgH2O), and urine sodium 20 mEq/L (40–200 mEq/L/day). What is the most likely cause of her polyuria? A. Syndrome of inappropriate antidiuretic hormone (SIADH) B. Cerebral salt wasting (CSW) C. Diabetes insipidus (DI) D. Psychogenic polydipsia E. None of the above 2. What is a typical response to urine osmolality after the administration of desmopressin (DDAVP) in a patient with central diabetes insipidus (DI)? A. 50% increase B. 50% decrease C. 5% increase D. 5% decrease E. No change

ANSWERS TO THIS SECTION CAN BE FOUND ON PAGE 126 121

66485457-66485438 www.ketabpezeshki.com

ZZakaria_87574_PTR_CH09_10-06-13_121-130.inddakaria_87574_PTR_CH09_10-06-13_121-130.indd 121121 66/19/2013/19/2013 4:44:384:44:38 PMPM DISORDERS, DISEASES, SEIZURES, AND EPILEPSY

3. A 35-year-old man with recent diagnosis of von Hippel–Lindau disease presents to the ICU with respiratory failure secondary to pulmonary edema and blood pressure of 193/110. Prior to admission, he had been complaining of a 3-day history of episodic headaches and palpitations. What is the most likely diagnosis? A. Hyperaldosteronism B. Pheochromocytoma C. Renal artery stenosis D. Fibromuscular dysplasia E. Medication noncompliance

4. What is the most appropriate test to establish a diagnosis for the patient in Question 3? A. Renal ultrasound with doppler B. Ratio of plasma aldosterone to plasma renin activity C. Plasma metanephrines D. Restart home antihypertensives E. None of the above

5. A 40-year-old man has been complaining of increasing fatigue and loss of libido. He denies symptoms of headache, diplopia, and changes in urination or thirst. The patient under- went an elective transsphenoidal surgery 6 months before to remove a nonfunctioning pituitary adenoma; results of postoperative pituitary functioning were normal. Three months after the surgery, he had stereotactic radiation to treat the residual tumor. What is the most likely diagnosis? A. Regrowth of the adenoma B. Diabetes insipidus (DI) C. Hydrocephalus D. Hypopituitarism

6. A 24-year-old woman with a history of bipolar disorder, on lithium, presented to the ICU intubated and in a coma. Two days before admission, she had been suffering from “the fl u.” Her vitals are BP 101/68 mmHg, HR 65 beats/minute, and temperature 96°F. Physical examination revealed no scars, dry skin, no eye opening to noxious stimuli, localizing throughout, and absent deep tendon refl exes. Pertinent laboratory fi ndings are negative for infl uenza A/B and West Nile antigen, lithium level 1 mEq/L (0.8–1.2 mEq/L), thyrotro- pin (TSH) 10 µU/mL (0.4–4.0 µU/mL), and free thryoxine 0.09 ng/dL (0.7–2.0 ng/dL). What is the most likely cause of her presentation? A. Hyperthyroidism B. Hypothyroidism C. Lithium toxicity D. West Nile infection E. None of the above

122

66485457-66485438 www.ketabpezeshki.com

ZZakaria_87574_PTR_CH09_10-06-13_121-130.inddakaria_87574_PTR_CH09_10-06-13_121-130.indd 122122 66/19/2013/19/2013 4:44:384:44:38 PMPM NEUROENDOCRINE DISORDERS: Questions

7. A 74-year-old African American man with a history of atrial fi brillation is transferred to the ICU from an outside hospital after experiencing acute symptoms of right mid- dle cerebral artery (MCA) syndrome and receiving intravenous tPA. For a month before admission, he had been complaining of heat intolerance, diarrhea, and weight loss. He also stated that he felt like his neck was enlarging. Upon arriving to the ICU, he is found to have a rapid ventricular rate (RVR) and is subsequently started on an amiodarone drip with good rate control. The next morning he has a generalized motor seizure lasting 20 minutes before being controlled, is back in RVR, and has a temperature of 104°F. What is the likely diagnosis for his acute decompensation? A. Cerebral infection B. Reinfarction C. Hypothyroidism D. Thyroid storm

8. A 72-year-old Asian man with a history of hypertension, hyperlipidemia, non–insulin- dependent diabetes, and gout presents to the emergency department with signs of left middle cerebral artery (MCA) syndrome. On brain CT, there appears to be a hyperdense left MCA sign. Serum glucose is found to be 980 mg/dL with a mild ketoacidosis. On arte- rial blood gas (ABG), there is a base defi cit of –11. What is the most likely diagnosis? A. Hyperosmolar nonketotic (HONK) dehydration syndrome B. Diabetic ketoacidosis C. Cushing syndrome D. Addison’s disease

9. What is/are the appropriate therapy(ies) for a patient with newly diagnosed hyperthy- roidism presenting to the ICU for thyroid storm? A. Propylthiouracil B. Levothyroxine C. Lugol’s solution D. Propranolol E. A, C, and D F. A only G. B, C, and D

10. The most common electrolyte abnormality seen in subarachnoid hemorrhage (SAH) is: A. Hypocalcemia B. Hyperkalemia C. Hyponatremia D. Hypophosphatemia E. Hypomagnesemia

123

66485457-66485438 www.ketabpezeshki.com

ZZakaria_87574_PTR_CH09_10-06-13_121-130.inddakaria_87574_PTR_CH09_10-06-13_121-130.indd 123123 66/19/2013/19/2013 4:44:394:44:39 PMPM DISORDERS, DISEASES, SEIZURES, AND EPILEPSY

11. An elderly male patient presents with Uosm > 100 mOsm/L (reference 50–1,200) and a

UNa > 30 mmol/L (reference 15–250), consistent with which of the following etiologies of hyponatremia? A. Syndrome of inappropriate secretion of antidiuretic hormone (SIADH), hypothyroid- ism, adrenal insuffi ciency B. Polydipsia, inappropriate water administration to children C. Vomiting, diarrhea, and third spacing D. Congestive heart failure (CHF), cirrhosis, renal failure E. Diuretic use, renal tubular dysfunction

12. The primary feature distinguishing cerebral salt wasting (CSW) syndrome from syn- drome of inappropriate secretion of antidiuretic hormone (SIADH) is: A. Hypovolemia and negative sodium balance B. Hypernatremia and positive fl uid balance C. Hypotonic plasma D. High urine output E. Intracellular volume depletion

13. The treatment of cerebral salt wasting (CSW) typically includes: A. Hypotonic volume replacement B. Restoration of positive sodium balance with hypertonic saline C. Fludrocortisone D. Treatment of underlying etiology

14. A 40-year-old man with renal insuffi ciency in the ICU is found to have symptomatic hyponatremia of 119 mmol/L on his second day of admission. He is given hyperosmolar saline, and his sodium rises to 128 mmol/L the next day. After tube feeding is started, his sodium falls to 122 mmol/L. What is the next best step in normalizing his sodium over the long term? A. High-protein diet B. Demeclocycline C. Conivaptan D. Tolvaptan E. Hypertonic saline

15. Which of the following comorbidities can predispose to the development of osmotic demyelination syndrome (ODS)? A. Chronic alcoholism B. Malnutrition C. Renal insuffi ciency D. Liver cirrhosis E. All of the above

124

66485457-66485438 www.ketabpezeshki.com

ZZakaria_87574_PTR_CH09_10-06-13_121-130.inddakaria_87574_PTR_CH09_10-06-13_121-130.indd 124124 66/19/2013/19/2013 4:44:394:44:39 PMPM NEUROENDOCRINE DISORDERS: Questions

16. The most important risk factors for the development of osmotic demyelination syndrome (ODS) include: A. Duration of hyponatremia B. Serum sodium at presentation C. Rate of serum sodium correction D. All of the above

17. An elderly unconscious immigrant female presents with low serum, unbound thyrox- ine (fT4) and triiodothyronine (fT3), with associated sinus bradycardia and hypothermia. Which of the following therapies should be considered? A. Rewarming B. Corticosteroids C. IV fl uids and vasopressors D. Mechanical ventilation E. All of the above

125

66485457-66485438 www.ketabpezeshki.com

ZZakaria_87574_PTR_CH09_10-06-13_121-130.inddakaria_87574_PTR_CH09_10-06-13_121-130.indd 125125 66/19/2013/19/2013 4:44:394:44:39 PMPM 9

ANSWERS

1. The answer is C. Although not a common etiology, meningitis can cause central DI. Other etiologies of DI include cranial trauma, brain surgery (trans-sphenoidal), sellar and supra- sellar tumors, pituitary apoplexy, Sheehan’s syndrome, and sarcoidosis. Typically, labo- ratory values seen in DI are hypernatremia, low urine osmolality, normal urine sodium, and low urine specifi c gravity. With SIADH and CSW, there is hyponatremia, high urine osmolality, and high urine sodium. Psychogenic polydipsia will show hyponatremia, low to normal urine osmolality, and low to normal urine sodium (1).

2. The answer is A. Central DI occurs due to an impaired production of antidiuretic hormone (ADH), resulting in decreased water reabsorption in the collecting ducts of the kidneys. Administering desmopressin (DDAVP), an analog of ADH, will cause a 50% increase in urine osmolality in patients with DI. Patients without DI will show a normal response of a 5% increase in urine osmolality. No change is seen in urine osmolality with nephrogenic DI (1,2).

3. The answer is B. Von Hippel–Lindau syndrome is an autosomal dominant disease con- sisting of cerebellar and spinal cord hemangioblastomas, retinal angiomas, renal cell car- cinoma, and pheochromocytoma. Given the patient’s presentation and history, the most likely diagnosis is pheochromocytoma. Fibromuscular dysplasia is most commonly seen in females (1,3).

4. The answer is C. Plasma metanephrines are the most useful test if the pretest probability is high, as in this case. When normal, it effectively excludes the diagnosis of pheochromo- cytoma, and mild elevations can be false positives. Renal ultrasound with doppler helps in the diagnosis of both renal artery stenosis and fi bromuscular dysplasia. Serum aldos- terone to renin ratio will diagnose hyperaldosteronism (1,3).

126

66485457-66485438 www.ketabpezeshki.com

ZZakaria_87574_PTR_CH09_10-06-13_121-130.inddakaria_87574_PTR_CH09_10-06-13_121-130.indd 126126 66/19/2013/19/2013 4:44:394:44:39 PMPM NEUROENDOCRINE DISORDERS: Answers

5. The answer is D. Radiation therapy to the can cause defi ciencies of anterior pituitary hormones. Cortisol defi ciency, hypothyroidism, and hypogonadism could be possible causes of his symptoms. Regrowth of the adenoma is unlikely, as radiation therapy is effective in “curing” nonfunctioning adenomas. DI usually mani- fests 24 to 48 hours after surgery to the pituitary. Radiation to the pituitary generally does not damage the posterior pituitary. Neither transsphenoidal surgery nor radiation causes hydrocephalus (4).

6. The answer is B. One of lithium’s many side effects is hypothyroidism. Myxedema coma is a life-threatening manifestation of hypothyroidism. This patient presenting in a coma after 2 days of being ill likely is in myxedema coma manifested by the side effect of lithium and the added stress of illness. Her high TSH and low free T4 help to validate her etiol- ogy. Hyperthyroidism is unlikely, as TSH is low and free T4 is high in this disease. Given the normal lithium level, her coma is unlikely to be a manifestation of lithium toxicity. A negative West Nile antigen can help rule out West Nile infection as an etiology (1).

7. The answer is D. The most likely diagnosis is thyroid storm characterized by seizure, RVR, and hyperthermia. For a month, the patient had been experiencing symptoms of thyro- toxicosis, which could be secondary to Graves’ disease given his neck mass. Amiodarone has been associated with thyrotoxicosis with two major forms: Type 1 is associated with underlying thyroid abnormality, and type 2 is associated with no intrinsic thyroid abnor- mality. Given the patient’s likelihood of Graves’ disease, the amiodarone drip that was started to control his rate precipitated a thyroid storm. Cerebral infection such as men- ingitis can cause seizure and hyperthermia; given the history and presentation, thyroid storm is more likely. Reinfarction may cause a seizure and cardiac arrhythmias, but is unlikely to cause extreme hyperthermia. Symptoms of hypothyroidism are cold intoler- ance, constipation, and bradyarrhythmia, and these are not present in this patient (1,5).

8. The answer is A. This patient has presented with a stroke that has precipitated HONK. This syndrome is typically seen in elderly patients with precipitating factors such as stroke and myocardial infarction. With HONK, the patient may or may not have a history of non–insulin-dependent diabetes, extreme hyperglycemia greater than 1,000 mg/dL, and decreased sensorium/coma secondary to osmotically mediated cellular dehydration. Diabetic ketoacidosis is typically manifested in younger insulin-dependant diabetics with hyperglycemia in the range of 400 to 800 mg/dL, marked ketoacidosis, and hyperkalemia. Cushing syndrome is manifested after prolonged exposure to high doses of corticoster- oids, which is not the case in this patient. Addison’s disease is a disorder of the adrenal gland not producing suffi cient glucocorticoids or mineralocorticoids. Hypoglycemia is a symptom of Addison’s disease (1,6).

9. The answer is E. Propylthiouracil inhibits the production of thyroid hormones by inhibiting the conversion of T4 to T3. Lugol’s solution (sodium ipodate or potassium iodide) inhibits the release of thyroid hormones and is typically started after an hour following the fi rst dose of antithyroidal drug to avoid stimulating thyroid hormone synthesis. Propranolol, a β-adrenergic blocker, is used to treat the hypertension and tachycardia associated with thyroid storm. In larger doses, greater than 240 mg, it too can inhibit the conversion of T4 to T3. Levothyroxine is a synthetic T4 used to treat hypothyroidism (1,5).

127

66485457-66485438 www.ketabpezeshki.com

ZZakaria_87574_PTR_CH09_10-06-13_121-130.inddakaria_87574_PTR_CH09_10-06-13_121-130.indd 127127 66/19/2013/19/2013 4:44:394:44:39 PMPM DISORDERS, DISEASES, SEIZURES, AND EPILEPSY

10. The answer is C. Avoid hyponatremia, which is commonly encountered. Its presentation is associated with SAH, intracranial hemorrhage, infl ammatory and demyelinating dis- eases, brain tumors, stroke, traumatic brain injury (TBI), and spinal cord injury, as well as drugs such as carbamazepine, serotonin reuptake inhibitors, opioids, and lamotrigine (7). Hyponatremia in SAH patients typically refl ects cerebral salt wasting (CSW) rather than syndrome of inappropriate secretion of antidiuretic hormone (SIADH). Fluid restriction, typical for treatment of SIADH, should not be used for CSW. Both etiologies of hypona- tremia have abnormally high urine osmolalities, and this cannot be used to distinguish between CSW and SIADH (8). Hyponatremia should be corrected slowly, as central pon- tine myelinolysis can occur from aggressive sodium correction. Slower sodium correction should be used in patients with suspected chronic hyponatremia.

11. The answer is A. Euvolemic hyponatremia, typically associated with SIADH, hypothy-

roidism, and adrenal insuffi ciency, is characterized by a Uosm > 100 mOsm/L (usually

>300) and a UNa > 30 mmol/L (9). Other causes of euvolemic hyponatremia include poly-

dipsia and inappropriate water administration, which have Uosm < 100 Osm/L and a UNa > 30 mmol/L. Vomiting, diarrhea, and third spacing can cause hypovolemic hyponatremia

(Uosm > 300 mOsm/L, UNa < 20 mmol/L, and FeNa < 1%). CHF and cirrhosis present with

hypervolemic hyponatremia (UOsm > 300 mOsm/L, UNa < 20 mmol/L, and FeNa < 1%).

Excessive diuretic use typically presents with an elevated UOsm > 300 mOsm/L, UNa > 20 mmol/L, and elevated FeNa > 1%.

12. The answer is A. CSW syndrome is defi ned by the development of extracellular volume depletion due to a renal sodium transport abnormality in patients with intracranial dis- ease and normal adrenal and thyroid function. Pathologies are similar to SIADH in that CSW is associated with intracranial disorders, including tuberculous meningitis, meta- static adenocarcinoma of lung, TBI, and classically subarachnoid hemorrhage (SAH) (8). CSW has been linked with elevations of brain natriuretic peptide. It is treated by fl uid replacement and maintenance of positive salt balance, with consideration not to correct serum sodium greater than 0.5 mEq/L/hour for danger of central pontine myelinolysis (10). It may be diffi cult to distinguish from SIADH, which develops under similar circum- stances and also presents with hyponatremia. The main clinical difference between these two conditions is the total fl uid status of the patient: CSW patients present with hypov- olemia and a negative sodium balance, whereas SIADH is associated with euvolemia or hypervolemia. Another useful tool in differentiating CSW from SIADH is a laboratory fi nding: Random urine sodium concentrations tend to be greater than 100 mEq/L in CSW. SIADH rarely leads to a random urine sodium of greater than 100 mEq/L.

13. The answer is B. The treatment of CSW typically includes assessment of fl uid bal- ance and restoration of serum sodium levels with hypertonic saline. CSW refractory to hypertonic saline may require the use of fl udrocortisone, a synthetic mineralocor- ticoid, indicated for replacement in primary adrenal insuffi ciency and in refractory CSW (11).

14. The answer is D. High-protein diet or oral urea administration induces an osmotic diure- sis, increases free water excretion, and thus ameliorates hyponatremia in syndrome of inappropriate secretion of antidiuretic hormone (SIADH). Demeclocycline, which blocks

128

66485457-66485438 www.ketabpezeshki.com

ZZakaria_87574_PTR_CH09_10-06-13_121-130.inddakaria_87574_PTR_CH09_10-06-13_121-130.indd 128128 66/19/2013/19/2013 4:44:394:44:39 PMPM NEUROENDOCRINE DISORDERS: Answers

antidiuretic hormone (ADH) action through inhibition of cyclic adenosine monophos- phate (cAMP) generation and thus increases free water excretion, can be an alternative for refractory hyponatremia (12,13). Demeclocycline is contraindicated in patients with renal disease, hepatic cirrhosis, or congestive heart failure (CHF). Conivaptan is for short-term use only and therefore not useful for long-term management. Tolvaptan, a selective V2 receptor blocker, is available for long-term oral use.

15. The answer is E. ODS results from too rapid correction of serum sodium, causing demyelination of susceptible neurons, classically in the pons. Symptoms include spas- tic paralysis, pseudobulbar palsy, dysarthria, horizontal gaze paralysis, and decreased consciousness, which progresses over hours to days. Increased risk of ODS has been associated with chronic alcoholism, malnutrition, renal insuffi ciency, and liver cirrhosis.

16. The answer is D. The majority of ODS cases occur in patients whose sodium concentra- tions at presentation are less than or equal to 105 mEq/L, and nearly all reported cases arise in patients who present with a serum sodium of 120 mEq/L or less. Studies in experimental animals showed that brain damage does not occur when hyponatremia of less than a day’s duration is rapidly corrected. However, if hyponatremia persists for 2 to 3 days or more, the same treatment can result in severe demyelination (7,10,14). The recommended maximum rate of correction, based on numerous animal studies, is 0.5 mmol/L/hour, 10 to 12 mmol/L in the fi rst 24 hours, and no more than 18 mmol/L in the fi rst 48 hours (12).

17. The answer is E. Myxedema coma is the most extreme form of hypothyroidism, typi- cally presenting as coma or lethargy with bradycardia, hypothermia, hyponatremia, and respiratory failure precipitated by infection, heart failure, or amiodarone exposure. It is a medical emergency associated with a high mortality rate, although rare in presentation because of widespread availability of thyrotropin (TSH) assays. Treatment involves ven- tilator support, cautious rewarming, glucocorticoid therapy 50 to 100 mg hydrocortisone q6hrs, and hemodynamic support. Sodium levels less than 120 mEq/L should be treated. The optimal mode of thyroid hormone therapy in patients with myxedema coma is con- troversial, largely because the condition is so rare that there are no clinical trials compar- ing the effi cacy of different treatment regimens. Whether patients with myxedema coma should be treated with T4, T3, or both is controversial, but IV therapy remains a common option.

References

1. James K. An endocrinology consult. In: Torbey M, ed. Neurocritical Care. New York, NY: Cambridge University Press; 2010:397–409. 2. Singer I, Oster JR, Fishman LM. The management of diabetes insipidus in adults. Arch Intern Med. 1997;157(12):1293–1301. 3. Pacak K, Linehan WM, Eisenhofer G, Walther MM, Goldstein DS. Recent advances in genetics, diagnosis, localization, and treatment of pheochromocytoma. Ann Intern Med. 2001;134(4):315–329.

129

66485457-66485438 www.ketabpezeshki.com

ZZakaria_87574_PTR_CH09_10-06-13_121-130.inddakaria_87574_PTR_CH09_10-06-13_121-130.indd 129129 66/19/2013/19/2013 4:44:394:44:39 PMPM DISORDERS, DISEASES, SEIZURES, AND EPILEPSY

4. Corenblum B. Hypopituitarism (panhypopituitarism). In: Griffi ng GT, ed. Medscape. 2011. http://emedicine.medscape.com/article/122287-overview. 5. Kannan CR, Seshadri KG. Thyrotoxicosis. Dis Mon. 1997;43(9):601–677. 6. Lorber D. Nonketotic hypertonicity in diabetes mellitus. Med Clin North Am. 1995; 79(1):39–52. 7. Adrogué HJ, Madias NE. Hyponatremia. N Engl J Med. 2000;342(21):1581–1589. 8. Harrigan MR. Cerebral salt wasting syndrome. Crit Care Clin. 2001;17(1):125–138. 9. Mclean L, Zimmerman JL. Management of life-threatening electrolyte and metabolic dis- turbances. In: Fundamental Critical Care Support. 4th ed. Mount Prospect, IL: Society of Critical Care Medicine; 2007:12–16. 10. Laureno R, Karp BI. Myelinolysis after correction of hyponatremia. Ann Intern Med. 1997;126(1):57–62. 11. Woo MH, Kale-Pradhan PB. Fludrocortisone in the treatment of subarachnoid hemor- rhage-induced hyponatremia. Ann Pharmacother. 1997;31(5):637–639. 12. Zietse R, van der Lubbe N, Hoorn EJ. Current and future treatment options in SIADH. NDT Plus. 2009;2(Suppl 3):iii12–iii19. 13. Oster JR, Epstein M. Demeclocycline-induced renal failure. Lancet. 1977;1(8001):52. 14. Kleinsschmidt-Demasters BK, Norenburg MD. Rapid correction of hyponatremia causes demyelination: relation to central pontine myelinolysis. Science. 1981;211(4486): 1068–1070.

130

66485457-66485438 www.ketabpezeshki.com

ZZakaria_87574_PTR_CH09_10-06-13_121-130.inddakaria_87574_PTR_CH09_10-06-13_121-130.indd 130130 66/19/2013/19/2013 4:44:394:44:39 PMPM 10 Neuro-Oncology Yoshua Esquenazi and Nitin Tandon QUESTIONS

1. A 54-year-old right-handed man with a medical history of hypertension and diabe- tes presented to the ED after an episode of word-fi nding diffi culty that lasted for about 15 minutes. On further questioning, the patient admitted to word-fi nding diffi culty for about 3 weeks and a 2-week history of bifrontal headaches. He had been started on antibi- otics by his primary care physician for a presumed frontal sinusitis, without improvement. On physical examination, he was found to have an expressive aphasia. A noncontrast CT scan done in the ED showed evidence of a well-circumscribed left frontal hypodense lesion. The MRI of the brain shown at the top of the following page was obtained. What is the appropriate next step for the defi nitive management of this patient? A. Lumbar puncture and broad-spectrum antibiotics B. Fosphenytoin load and 24-hour video electroencephalogram (vEEG) C. IV dexamethasone and levetiracetam and plans for surgical resection as part of an awake craniotomy D. Functional MRI E. Aspirin and CT angiogram of the brain

ANSWERS TO THIS SECTION CAN BE FOUND ON PAGE 135 131

66485457-66485438 www.ketabpezeshki.com

ZZakaria_87574_PTR_CH10_10-06-13_131-140.inddakaria_87574_PTR_CH10_10-06-13_131-140.indd 131131 66/19/2013/19/2013 4:44:484:44:48 PMPM DISORDERS, DISEASES, SEIZURES, AND EPILEPSY

2. What is the median survival rate for patients with a World Health Organization (WHO) grade IV glioblastoma (GBM) with the current standard of care, including adjuvant chemotherapy? A. Greater than 5 years B. 12 to 15 months C. Less than 6 months D. About 2 to 2.5 years E. Less than 9 months

3. Which of the following is the most common intracranial neoplasm in adults? A. Glioblastoma (GBM) B. Meningioma C. Pituitary tumors D. Medulloblastoma E. Metastases

4. A 42-year-old woman with no signifi cant medical history presents to the ED 7 hours after the sudden onset of headache associated with nausea and vomiting. On physical examina- tion, she is found to have evidence of meningismus, nuchal rigidity, and bilateral cranial nerve III palsy, as well as progressive blurry vision in the right eye. The accompanying MRI of the brain was obtained. What is the best next step in the management of this patient? A. External ventricular placement and emergent diagnostic cerebral angiogram B. Broad-spectrum antibiotics and lum- bar puncture C. Corticosteroids and immediate surgi- cal intervention D. Emergent anticoagulation with IV heparin E. Diagnostic cerebral angiogram and intra-arterial tissue plasminogen activator (tPA)

132

66485457-66485438 www.ketabpezeshki.com

ZZakaria_87574_PTR_CH10_10-06-13_131-140.inddakaria_87574_PTR_CH10_10-06-13_131-140.indd 132132 66/19/2013/19/2013 4:44:484:44:48 PMPM NEURO-ONCOLOGY: Questions

5. What is the most likely diagnosis in Question 4? A. Subarachnoid hemorrhage from a ruptured posterior communicating aneurysm B. Cavernous sinus thrombosis C. Bacterial meningitis D. Benedikt syndrome E. Pituitary apoplexy

6. Which of the following is best characterized as an intradural extramedullary spinal cord tumor? A. Meningioma B. Ependymoma C. Astrocytoma D. Spinal metastasis E. Hemangioblastoma

7. Which of the following is/are the most common symptom of metastatic epidural spinal cord compression (MESCC)? A. Weakness B. Sensory defi cits C. Back pain D. Autonomic symptoms E. Ataxia and gait disturbances

8. A 58-year-old woman with a medical history of hypertension presents to the ED with a 3-week history of worsening back pain and 4-day history of weakness in her lower extremities. On physical examination, strength in her lower extremities is 4/5 throughout, her refl exes are normal, and there is no evidence of clonus or abnormal plantar response. The CT scan and MRI of the thoracic spine shown here are obtained, revealing a lesion at T9 level. Which of the following is the best management option for this patient? A. Corticosteroids and radiation therapy B. Kyphoplasty C. Corticosteroids, thoracic corpec- tomy, and instrumentation plus radiation therapy D. Thoracic laminectomy and instrumentation E. Spinal stereotactic radiosurgery (SRS) and kyphoplasty

133

66485457-66485438 www.ketabpezeshki.com

ZZakaria_87574_PTR_CH10_10-06-13_131-140.inddakaria_87574_PTR_CH10_10-06-13_131-140.indd 133133 66/19/2013/19/2013 4:44:494:44:49 PMPM DISORDERS, DISEASES, SEIZURES, AND EPILEPSY

9. Which of the following is the most common presenting symptom in carcinomatous men- ingitis (CM)? A. Photophobia B. Mental status changes C. Trigeminal neuralgia D. Nuchal rigidity E. Diplopia

10. Which of the following is the most useful laboratory test in the diagnosis of carcinoma- tous meningitis (CM)? A. Leptomeningeal biopsy B. Cerebrospinal fl uid (CSF) examination C. Serology for tumor markers D. Flow cytometry and DNA single-cell cytometry E. Fluorescence in situ hybridization

11. Which of the following paraneoplastic syndromes (PNSs) is associated with involuntary, arrhythmic, chaotic multidirectional saccades with horizontal, vertical, and torsional components, accompanied by myoclonic jerks in the limbs and trunk? A. Anti-Hu limbic encephalitis B. Opsoclonus-myoclonus C. Paraneoplastic cerebellar degeneration D. Lambert–Eaton syndrome E. Polymyositis

134

66485457-66485438 www.ketabpezeshki.com

ZZakaria_87574_PTR_CH10_10-06-13_131-140.inddakaria_87574_PTR_CH10_10-06-13_131-140.indd 134134 66/19/2013/19/2013 4:44:494:44:49 PMPM 10

ANSWERS

1. The answer is C. MRI of the brain shows a left inferior frontal gyrus area of low signal intensity on T1, with increased signal intensity on T2, without evidence of susceptibility weighted imaging or contrast enhancement, consistent with a low-grade glioma. Low- grade gliomas typically arise in and around eloquent cortex. Patients present with either seizures; symptoms of raised intracranial pressure (headache, nausea, vomiting, leth- argy); or focal neurological defi cits (weakness, sensory abnormalities, neglect, visual or speech diffi culty), depending on tumor size and location. In patients with symptoms of local mass effect, increased intracranial pressure, or intractable seizures, the role of surgi- cal resection is well established. Greater extent of resection correlates with improved sur- vival time and reduces the risk of malignant transformation. An awake craniotomy with intraoperative brain mapping has shown promising results in the resection of lesions in eloquent cortex. Functional MRI can help localize language function in patients with brain tumors, but lacks both specifi city and sensitivity and cannot be used alone for lan- guage localization. There is no evidence of acute ischemia on diffusion-weighted MRI, where a CT angiogram or aspirin administration may be indicated. The patient presented with a simple partial seizure, and even though fosphenytoin should be administered for seizure prophylaxis, a 24-hour EEG is not indicated. A lumbar puncture followed by broad- spectrum antibiotics is not indicated here because bacterial meningitis is not the etiology (1,2).

2. The answer is B. Median survival rates for GBM, a WHO grade IV malignant glioma, range from 12 to 15 months. The current standard of care has evolved from resection fol- lowed by adjuvant radiotherapy to resection, concurrent chemotherapy (temozolomide) and radiation, and additional adjuvant chemotherapy. The expression of specifi c molecu- lar biomarkers, especially O-6-methylguanine methyltransferase (MGMT) and isocitrate

135

66485457-66485438 www.ketabpezeshki.com

ZZakaria_87574_PTR_CH10_10-06-13_131-140.inddakaria_87574_PTR_CH10_10-06-13_131-140.indd 135135 66/19/2013/19/2013 4:44:494:44:49 PMPM DISORDERS, DISEASES, SEIZURES, AND EPILEPSY

dehydrogenase mutation, may determine the response of the tumor to treatment and helps in identifying the magnitude of benefi t from this regimen. Bevacizumab therapy has also had a defi nitive impact in slowing the progression of recurrent GBMs (3–5).

3. The answer is E. Metastatic brain tumors are the most common intracranial neoplasm in adults (about 30% of all tumors), arising in 10% to 15% of all patients with systemic malig- nancies. They are most often located at the gray–white matter junction of the cerebral and cerebellar hemispheres, but they may affect any part of the central nervous system (CNS), including the leptomeninges. The most common primary sites of origin are the lung, breast, melanoma, kidney, and gastrointestinal tract. GBM is the most common and lethal adult primary brain tumor. Meningiomas account for an estimated 13% to 26% of primary intracranial neoplasms, occurring most frequently in women in the middle decades of life. Pituitary tumors account for 10% to 15% of all primary brain tumors. Medulloblastomas account for 17% of all brain tumors in the pediatric population, but they only account for less than 1% of all adult CNS tumors (6).

4 and 5. The answers are C and E, respectively. The patient has signs and symptoms of oph- thalmoplegia and visual compromise from pituitary apoplexy, and a noncontrast coronal MRI of the brain shows evidence of a pituitary mass with suprasellar extension and acute intratumoral hemorrhage. Pituitary tumor apoplexy is an uncommon event characterized by the abrupt onset of severe headache, restriction of visual fi elds, deterioration of visual acuity, and weakness of ocular motility, and associated frequently with clinical signs of pituitary endocrine dysfunction. Hemorrhage into or necrosis of a preexisting sellar mass, most commonly a pituitary macroadenoma, produces expansion and compression of the sellar components and surrounding structures with displacement of the optic nerves and chiasm, and impingement of the cranial nerves along the cavernous sinus (III, IV, VI). Damage and destruction of the anterior pituitary leads to multiple acute and/or chronic hormone defi ciencies in many patients. Medical management may be indicated in rare cases where the signs and symptoms are mild, restricted to meningismus or ophthal- moplegia, and deemed to be stable. In patients with visual or oculomotor compromise or altered level of consciousness, emergent surgical decompression, most commonly through a transsphenoidal approach, should be performed. There is no evidence of subarachnoid hemorrhage, and a ruptured posterior communicating aneurysm that projects laterally into the oculomotor triangle would produce unilateral cranial nerve palsy. The extension of blood into the suprasellar cistern may provoke the development of a chemical menin- gitis, but there is no evidence of other symptoms suggestive of bacterial meningitis. The constellation of signs and symptoms and the radiographic evidence of an expansive sellar mass with intratumoral hemorrhage rule out the possibility of cavernous sinus thrombo- sis or a midbrain infarction (7).

6. The answer is A. Primary tumors of the spinal cord are 10 to 15 times less common than primary intracranial tumors and represent 2% to 4% of all primary tumors of the central nervous system (CNS). Primary spinal cord tumors are divided into three categories on the basis of anatomic location: i. Intramedullary ii. Intradural extramedullary iii. Extradural

136

66485457-66485438 www.ketabpezeshki.com

ZZakaria_87574_PTR_CH10_10-06-13_131-140.inddakaria_87574_PTR_CH10_10-06-13_131-140.indd 136136 66/19/2013/19/2013 4:44:504:44:50 PMPM NEURO-ONCOLOGY: Answers

Extradural tumors primarily consist of systemic cancer metastases that result in epidural spinal cord compression. Intramedullary spinal cord tumors (IMSCTs) constitute 8% to 10% of all primary spinal cord tumors, with the majority comprising gliomas (80%–90%), of which 60% to 70% are ependymomas and 30% to 40% are astrocytomas. Fifteen percent of all primary intradural spinal cord tumors are ependymal in origin and include one of three histopathologic subtypes: ependymoma, subependymoma, and myxopapillary ependymoma. The third most common IMSCT is hemangioblastoma, representing 3% to 8% of all IMSCTs. Intradural extramedullary spinal cord tumors are predominantly either meningiomas (50%) or peripheral nerve sheath tumors (50%). The clinical presentation of primary spinal cord tumors is determined in part by the location of the tumor, and in nearly all clinical instances, pain is the predominant presenting symptom (8).

7. The answer is C. MESCC is a devastating complication of systemic cancer that affects about 5% of all patients. Prostate, breast, and lung cancer each accounts for 15% to 20% of all cases; non-Hodgkin’s lymphoma, renal cell cancer, and multiple myeloma account for 5% to 10%, and the remainder of cases are due to colorectal cancers, sarcomas, and tumors from unknown primaries. MESCC usually occurs in patients who have a preex- isting diagnosis of cancer, although MESCC is the fi rst manifestation of cancer in about 20% of patients. Metastatic tumors reach the epidural space and compress the spinal cord via the growth of a paravertebral tumor in about 15% of cases, a process more commonly associated with lymphomas and neuroblastomas. In the majority (85% of patients), the tumor reaches the spine via hematogenous metastasis to the vertebral body, grows into the bone, and then spreads into the epidural space, causing secondary compression of the spinal cord. The compression can be gradual, although acute compression secondary to bone destruction with vertebral body collapse and displacement of bone fragments into the epidural space can also occur. Back pain is the earliest and most common symp- tom of MESCC and is present in more than 95% of patients at diagnosis. Radicular pain due to compression or invasion of the nerve roots is commonly present in patients who develop MESCC. The pain is frequently unilateral, with cervical or lumbosacral spine involvement, or bilateral, with thoracic spine involvement. Weakness is the second most common symptom and is present in 35% to 75% of patients. Sensory defi cits rarely occur before motor defi cits or pain and are present in about 50% to 70% of patients. Occasionally, patients may present with ataxia of gait owing to involvement of the spinocerebellar tracts without pain, motor, or sensory fi ndings. Autonomic symptoms occur late in the progres- sion, and isolated bowel or bladder dysfunction are rarely the presenting symptoms. MRI is the diagnostic imaging modality of choice for the diagnosis of MESCC. First-line ther- apy is corticosteroids, and once the diagnosis is suspected, patients with neurologic defi - cits should be started on high-dose corticosteroids. Defi nitive treatment almost always includes radiotherapy, and for selected patients, the combination of radical surgery and radiation is superior to radiation only and is preferred in patients with life expectancy of more than 6 months. Early diagnosis and appropriate treatment will prevent paraplegia in most patients (9,10).

8. The answer is C. Therapeutic options for patients with metastatic epidural spinal cord compression (MESCC) include corticosteroids, surgery, and radiation therapy. Corticosteroids decrease spinal cord edema and might have an oncolytic effect on certain

137

66485457-66485438 www.ketabpezeshki.com

ZZakaria_87574_PTR_CH10_10-06-13_131-140.inddakaria_87574_PTR_CH10_10-06-13_131-140.indd 137137 66/19/2013/19/2013 4:44:504:44:50 PMPM DISORDERS, DISEASES, SEIZURES, AND EPILEPSY

tumors, including lymphoma and breast cancer. They are effective in preventing neuro- logical deterioration in the short term. Earlier studies looking at the role of laminectomy in MESCC have suggested that posterior decompression alone may be associated with a poor outcome. Decompressive surgery addressing the anterior compressive disease from either a transpedicular or a thoracotomy approach followed by an instrumented fusion may result in better neurological outcomes compared with patients who were treated with radiation alone. New methods of delivering more focused radiation with the poten- tial delivery of higher radiation doses to the tumor with less exposure to the spinal cord and healthy tissue are in development. Spinal SRS is an emerging therapeutic option for the treatment of spinal metastasis, and percutaneous injection of polymethylmethacrylate into a pathologically collapsed vertebral body represents a successful option for the pal- liative treatment of intractable spinal pain (11).

9. The answer is E. Clinical features of CM are caused by the obstruction of normal cerebro- spinal fl uid (CSF) fl ow or by direct tumor infi ltration. The clinical signs and symptoms are associated with increased intracranial pressure and by infi ltration of the nerves producing local neurological defi cit. The most common presenting clinical features are cranial nerve palsies, followed by headaches, cerebral disturbances, spinal nerve involvement, mental status changes, and limb weakness. Diplopia is the most common symptom of cranial nerve dysfunction, with cranial nerve VI being the most frequently affected, followed by cranial nerves III and IV. Trigeminal involvement, cochlear dysfunction, and optic neu- ropathy are also common fi ndings. Spinal signs and symptoms include weakness, derma- tomal or segmental sensory loss, and pain in the neck, back, or a radicular pattern. Nuchal rigidity is present in only 15% of cases (12).

10. The answer is B. The most useful laboratory test in the diagnosis of CM is the CSF

examination. Abnormalities include increased opening pressure (>200 mmH2O), increased leukocytes (>4/mm3), elevated protein (>50 mg/dL), and decreased glucose (<60 mg/dL), which, though suggestive of CM, are not diagnostic. The demonstration of malignant cells in the CSF is the cornerstone of the fi nal diagnosis. A positive CSF cytology is found on the initial lumbar puncture in 50% to 70% and in nearly all cases after three attempts. False-positive cytologies are associated with infectious or infl am- matory diseases demonstrating reactive lymphocytes. Increased CSF opening pressure is found in 50% to 70% of patients and depends on the extent of the leptomeningeal involvement. Elevated protein in the CSF and low glucose are seen in approximately 75% and 40% of the cases, respectively. Biochemical markers have also been evalu- ated, but their use has been limited by poor sensitivity and specifi city. Among these substances are carcinoembryonic antigen (CEA), lactate dehydrogenase (LDH), alka- line phosphatase, β-human chorionic gonadotropin (β-HCG), and others. Cytogenetic studies, including fl ow cytometry, DNA single-cell cytometry, and fl uorescence in situ hybridization can give additional diagnostic information, are especially useful in liquid tumors (leukemia and lymphoma), and appear more sensitive than CSF cytology. In cases where there is no manifestation of systemic cancer and CSF examinations remain inconclusive, a meningeal biopsy may be diagnostic, and the yield of this test may be higher if the biopsy is taken from an enhancing region seen on the MRI. Nevertheless, the gold standard of CM diagnosis remains the combination of the full clinical picture and positive cytology (13).

138

66485457-66485438 www.ketabpezeshki.com

ZZakaria_87574_PTR_CH10_10-06-13_131-140.inddakaria_87574_PTR_CH10_10-06-13_131-140.indd 138138 66/19/2013/19/2013 4:44:504:44:50 PMPM NEURO-ONCOLOGY: Answers

11. The answer is B. PNSs affecting the nervous system are rare neurologic syndromes caused by cancer but not ascribable to metastases. Any portion of the nervous system may be involved. The pathogenesis of these disorders appears to be an immune reac- tion against antigens shared by the cancer and the nervous system. Two clinical features shared by most PNSs affecting the Central Nervous System (CNS) are the rapid develop- ment of symptoms and signs of infl ammation in the cerebrospinal fl uid (CSF), including moderated lymphocytic pleocytosis, increased protein concentration, high IgG index, and CSF-specifi c oligoclonal bands. In about 70% of patients with paraneoplastic neurologic disorders (PND), neurological symptoms are the fi rst manifestation of a tumor. Of these patients, 70% to 80% will have a positive screening for cancer on initial assessment. Most tumors are identifi ed with imaging of the chest, abdomen, and pelvis using CT, fl uo- rodeoxyglucose-PET, or both. Opsoclonus comprises involuntary, arrhythmic, and cha- otic multidirectional saccades with horizontal, vertical, and torsional components, and is commonly accompanied by myoclonic jerks in the limbs and trunk, cerebellar ataxia, tremor, and encephalopathy. Recent fi ndings suggest that disinhibition of the fastigial nucleus of the cerebellum may be involved in the pathophysiological mechanism of opso- clonus. Opsoclonus-myoclonus can occur with infections, toxic–metabolic disorders, and paraneoplastic mechanisms. In children, the disorder is related to the presence of a neu- roblastoma in about 50% of cases. In adults, the tumors most commonly involved include small cell lung cancer and cancer of the breast and ovary. Cerebellar dysfunction is one of the most common paraneoplastic presentations of cancer. The hallmark of paraneoplastic cerebellar degeneration is an extensive loss of Purkinje cells that might be associated with infl ammatory infi ltrates in the cerebellar cortex, deep cerebellar nuclei, and inferior oli- vary nuclei. Neurological defi cits are sometimes preceded by prodromal symptoms, such as viral-like illness, dizziness, nausea, or vomiting. These symptoms are followed by gait unsteadiness that rapidly develops into ataxia, diplopia, dysarthria, and dysphagia. Some patients may have blurry vision, oscillopsia, and transient opsoclonus. Limbic encephali- tis is an infl ammatory process confi ned to the limbic system. Patients may develop mood and sleep disturbances, seizures, hallucinations, and short-term memory loss that may progress to dementia. EEG usually reveals foci of epileptic activity in one or both tem- poral lobes or focal or generalized slow activity. In 70% to 80% of the cases, MRI FLAIR (fl uid-attenuated inversion recovery) or T2 sequences show hyperintense signals in the medial portion of one or both temporal lobes. The clinical picture combined with fi ndings of EEG, MRI, and CSF infl ammatory changes suggest the diagnosis. The tumors more frequently involved are small cell lung cancer, testicular germ-cell neoplasms, thymoma, Hodgkin’s lymphoma, or teratoma. Lambert–Eaton myasthenic syndrome is a rare pre- synaptic disorder of neuromuscular transmission; unlike myasthenia gravis, the muscles of the trunk, shoulder girdle, pelvic girdle, and lower extremities are the ones that are most frequently involved (14,15).

References

1. Lubrano V, Draper L, Roux FE. What makes surgical tumor resection feasible in Broca’s area? Insights into intraoperative brain mapping. Neurosurgery. 2010;66(5):868–875; discussion 875.

139

66485457-66485438 www.ketabpezeshki.com

ZZakaria_87574_PTR_CH10_10-06-13_131-140.inddakaria_87574_PTR_CH10_10-06-13_131-140.indd 139139 66/19/2013/19/2013 4:44:504:44:50 PMPM DISORDERS, DISEASES, SEIZURES, AND EPILEPSY

2. Sanai N, Berger MS. Operative techniques for gliomas and the value of extent of resection. Neurotherapeutics. 2009;6(3):478–486. 3. Quick A, Patel D, Hadziahmetovic M, Chakravarti A, Mehta M. Current therapeutic par- adigms in glioblastoma. Rev Recent Clin Trials. 2010;5(1):14–27. 4. Walbert T, Mikkelsen T. Recurrent high-grade glioma: a diagnostic and therapeutic chal- lenge. Expert Rev Neurother. 2011;11(4):509–518. 5. Stupp R, Mason WP, van den Bent MJ, et al. Radiotherapy plus concomitant and adjuvant temozolomide for glioblastoma. N Engl J Med. 2005;352(10):987–996. 6. Al-Shamy G, Sawaya R. Management of brain metastases: the indispensable role of sur- gery. J Neurooncol. 2009;92(3):275–282. 7. Verrees M, Arafah BM, Selman WR. Pituitary tumor apoplexy: characteristics, treatment, and outcomes. Neurosurg Focus. 2004;16(4):E6. 8. Chamberlain MC, Tredway TL. Adult primary intradural spinal cord tumors: a review. Curr Neurol Neurosci Rep. 2011;11(3):320–328. 9. Cole JS, Patchell RA. Metastatic epidural spinal cord compression. Lancet Neurol. 2008; 7(5):459–466. 10. Ribas ES, Schiff D. Spinal cord compression. Curr Treat Options Neurol. 2012; 14(4):391–401. 11. Patchell RA, Tibbs PA, Regine WF, et al. Direct decompressive surgical resection in the treatment of spinal cord compression caused by metastatic cancer: a randomised trial. Lancet. 2005;366(9486):643–648. 12. Chamberlain MC. Neoplastic meningitis. Oncologist. 2008;13(9):967–977. 13. Pavlidis N. The diagnostic and therapeutic management of leptomeningeal carcinomato- sis. Ann Oncol. 2004;15(Suppl 4):iv285–iv291. 14. Dalmau J, Rosenfeld MR. Paraneoplastic syndromes of the CNS. Lancet Neurol. 2008;7(4):327–340. 15. Posner JB, Dalmau JO. Paraneoplastic syndromes affecting the central nervous system. Annu Rev Med. 1997;48:157–166.

140

66485457-66485438 www.ketabpezeshki.com

ZZakaria_87574_PTR_CH10_10-06-13_131-140.inddakaria_87574_PTR_CH10_10-06-13_131-140.indd 140140 66/19/2013/19/2013 4:44:504:44:50 PMPM 11 Encephalopathies Corey E. Goldsmith QUESTIONS

1. Which of the following statements is true about eclampsia? A. Eclampsia can develop up to 48 hours postpartum B. Sudden severe throbbing headache is the most common herald of a seizure C. The BP is elevated for several hours prior to convulsions D. EEG and cerebrospinal fl uid (CSF) are normal E. All of the answers are true

2. Ammonia levels above ___ µmol/L are more associated with cerebral herniation. A. 50 B. 100 C. 150 D. 200 E. 250

ANSWERS TO THIS SECTION CAN BE FOUND ON PAGE 149 141

66485457-66485438 www.ketabpezeshki.com

ZZakaria_87574_PTR_CH11_10-06-13_141-158.inddakaria_87574_PTR_CH11_10-06-13_141-158.indd 141141 66/19/2013/19/2013 4:45:024:45:02 PMPM DISORDERS, DISEASES, SEIZURES, AND EPILEPSY

3. A 70-year-old man with a history of chronic renal insuffi ciency is admitted to the ICU after a motor vehicle accident with multiple fractures. He receives morphine but no other form of sedatives. He develops a hyperactive delirium and is also noted to have diffuse myoclo- nus, asterixis, and tremors, but cranial nerves are intact with antigravity movement and withdrawal to pain in all four extremities. CT scan without contrast of his head is negative for acute pathology. Which management option is likely to be most effective? A. Naloxone B. Mild hypothermia C. Anticoagulation D. Acute hemodialysis E. Fosphenytoin load

4. The most common neurologic fi nding in preeclampsia is: A. Vision problems B. Headache C. Clonus D. Epigastric pain E. Seizures

5. Which of the following patients is/are at risk for developing posterior reversible enceph- alopathy syndrome (PRES)? A. A 26-year-old woman with lupus being treated with high-dose steroids B. A 62-year-old man with chronic renal failure but well-controlled blood pressure (BP) being given erythropoietin C. A 33-year-old man with HIV on combined antiretroviral therapy (cART) presenting with hypercalcemia and found to have hyperparathyroidism D. An 18-year-old girl receiving cyclosporine E. All of the above

6. Hypothermia has been shown to complicate prediction of prognosis after cardiac arrest. Which of the following statements is false? A. Hypothermia causes the somatosensory evoked potential (SSEP) responses to be falsely absent B. Hypothermia elevates serum neuron-specifi c enolase, making the cutoff value inaccurate C. Hypothermia can delay the return of pupillary and motor function, making the previ- ous timing of these examinations inaccurate D. Metabolism and pharmacokinetics of the sedatives/paralytics are unknown during hypothermia

142

66485457-66485438 www.ketabpezeshki.com

ZZakaria_87574_PTR_CH11_10-06-13_141-158.inddakaria_87574_PTR_CH11_10-06-13_141-158.indd 142142 66/19/2013/19/2013 4:45:024:45:02 PMPM ENCEPHALOPATHIES: Questions

7. Management guidelines of blood pressure (BP) in hypertensive encephalopathy include: A. BP should be aggressively dropped until neurologic symptoms resolve B. Initial treatment should lower the diastolic BP to about 100 mmHg or no more than a 25% drop C. Antihypertensive medicines should be reduced or stopped if there is an increase in creatinine after initial BP lowering D. Nitroprusside can be given at its maximal dose of 10 mcg/kg/minute for up to 30 minutes E. Nitroglycerin is a good fi rst-line choice for hypertensive management in hypertensive encephalopathy

8. MRI fi ndings in eclampsia show: A. Normal brain B. Periventricular white matter T2 FLAIR (fl uid attenuated inversion recovery) lesions C. Multifocal parieto-occipital subcortical T2 FLAIR lesions D. Cortical infarctions E. Mainly pontine lesions

9. Imaging fi ndings suggestive of mitochondrial encephalopathy with lactic acidosis and stroke like episodes (MELAS) include: A. Equal involvement of deep white matter and cortex B. Relative sparing of the deep white matter compared with the cortex C. Specifi c involvement of the anterior temporal lobes D. Mainly frontal involvement E. MR spectroscopy (MRS) fi ndings that show high N-acetyl aspartate (NAA) peaks

10. Uremic encephalopathy: A. Occurs usually when glomerular fi ltration rate (GFR) is less than 10% B. Occurs always in association with other electrolyte disturbances C. Occurs in chronic renal failure more than acute renal failure D. Can be easily distinguished from status epilepticus by examination alone E. All of the above

11. Which of the following medicines should be avoided when treating hypertension in pregnancy? A. Enalapril B. Methyldopa C. Labetalol D. Nicardipine E. Hydralazine

143

66485457-66485438 www.ketabpezeshki.com

ZZakaria_87574_PTR_CH11_10-06-13_141-158.inddakaria_87574_PTR_CH11_10-06-13_141-158.indd 143143 66/19/2013/19/2013 4:45:024:45:02 PMPM DISORDERS, DISEASES, SEIZURES, AND EPILEPSY

Please read the clinical presentation paragraph below and answer questions 12–17. A 26-year-old woman with no signifi cant medical history is admitted to the medical ICU with altered mental status. Her family reports that she has been complaining of fatigue, muscle aches, yellow eyes, low-grade fevers with poor appetite, nausea, and vomiting for the past week. In the past 2 days, she has become progressively confused. Her family reports no alcohol use, although she may have been taking some acetaminophen in the past couple of weeks. On examination, she is arousable with vigorous stimuli, confused, and disoriented, but follows some simple commands. She appears euvolemic. Her pupils are icteric, and she has some right upper quadrant (RUQ) tenderness to palpation, but no other signs of chronic liver disease. Her pupils are reactive, she has full extraocular muscle movements with nystagmus on end-gaze, and she is dysarthric. She is unable to ambulate because of ataxia and is noted to have asterixis. CT head scan is normal. She has

elevated bilirubins (TBili 20 mg/dL) and transaminases. Her PT/INR is 112 seconds/6.7, fi brinogen is 415, platelet count is 100 K/µL, blood urea nitrogen (BUN) is 70 mg/dL, and creatinine (Cr) is 3.6 mg/dL. Acetaminophen level is 0, and hepatitis A IgM is positive.

12. This patient’s condition: A. Usually develops in patients with chronic cirrhosis B. Is associated with ascites and cachexia C. Is most commonly caused by viral hepatitis D. Has a high mortality rate often secondary to cerebral edema E. Has a mortality rate of 15% to 20% without liver transplantation

13. This patient’s presentation is consistent with Grade ___ hepatic encephalopathy (HE). A. 0 B. 1 C. 2 D. 3 E. 4

14. The feature in the patient’s presentation most convincing of the need for transplant is: A. Etiology B. Creatinine value C. Bilirubin value D. Prothrombin time (PT) value E. Severity of encephalopathy

15. All of the following treatments would be recommended for this patient who was deemed to be a transplant candidate except: A. Intracranial pressure (ICP) monitoring B. Hyperosmotic agents such as mannitol or hypertonic saline if ICP is elevated C. Neomycin D. Prophylactic antibiotics E. N-acetylcysteine

144

66485457-66485438 www.ketabpezeshki.com

ZZakaria_87574_PTR_CH11_10-06-13_141-158.inddakaria_87574_PTR_CH11_10-06-13_141-158.indd 144144 66/19/2013/19/2013 4:45:024:45:02 PMPM ENCEPHALOPATHIES: Questions

16. Prior to placement of intracranial pressure (ICP) monitoring, the following should be done: A. Vitamin K 10 mg IV once B. Fresh-frozen plasma (FFP) C. Desmopressin (DDAVP) 0.3 mcg/kg once D. Two of the above E. All of the above

17. In regard to the patient’s renal failure: A. Renal failure is an unusual accompaniment to fulminant liver failure B. Fluid resuscitation will improve her renal failure, and no other intervention is warranted C. Intermittent hemodialysis is the best option for this patient D. Continuous veno-venous hemofi ltration (CVVH) is the best option for this patient E. The renal failure will self-correct with improvement in her hepatic failure

18. Which of these physical examination fi ndings accurately predicts universally poor out- come post-CPR without hypothermia? A. Continued comatose status at 72 hours B. Hyperthermia C. Absent pupillary response at 8 hours D. Absent pupillary response at 24 hours E. Absent motor response at 24 hours

19. A 55-year-old man with normally well-compensated hepatitis C cirrhosis presents with acute confusion and disorientation after being given a new medicine. Which of the fol- lowing would not be a possible culprit? A. Lorazepam B. Haloperidol C. Furosemide D. Hydrocodone/acetaminophen E. Topiramate

20. A 30-year-old woman is receiving magnesium sulfate for severe preeclampsia at 36 weeks gestation. Each of the following is a maternal effect of this treatment except: A. Sedation B. Hypocalcemia C. Sensitization to nondepolarizing muscle relaxants D. Cardiac dysfunction prior to respiratory depression E. Loss of deep tendon refl exes (DTRs) prior to signifi cant cardiac dysfunction

21. Drugs that abolish all brainstem function and may mimic brain death include: A. Valproic acid overdose B. Tricyclic antidepressant (TCA) overdose C. Barbituate coma D. Two of the above E. All of the above

145

66485457-66485438 www.ketabpezeshki.com

ZZakaria_87574_PTR_CH11_10-06-13_141-158.inddakaria_87574_PTR_CH11_10-06-13_141-158.indd 145145 66/19/2013/19/2013 4:45:024:45:02 PMPM DISORDERS, DISEASES, SEIZURES, AND EPILEPSY

22. Which of these ancillary study fi ndings is associated with the worst prognosis post- cardiac arrest without hypothermia? A. Burst-suppression pattern on EEG B. Bilateral periodic lateralized epileptiform discharges (BiPLEDs) or generalized peri- odic lateralized epileptiform discharges (GPEDs) on EEG C. Theta coma on EEG D. Absent N20 responses on somatosensory evoked potentials (SSEP) at 72 hours E. Absent brainstem auditory evoked potentials at 72 hours

23. In the development of cerebral anoxia, the most resistant neurologic function is: A. Pupillary light response B. Memory C. Control of spontaneous respirations D. Motor movement E. Eye movements

24. A 45-year-old man without known medical history is found down in his home. He is somnolent though arousable to vigorous stimulation. On examination, he is noted to have nuchal rigidity, normal cranial nerve function, and diffuse arrhythmic twitching of his muscles. Labs show normal complete blood count, blood urea nitrogen (BUN) 90 mg/dL, and Cr 4.4 mg/dL. He is intubated for airway protection, and a lumbar puncture (LP) is done after a normal CT head scan without contrast. Which of the following, would lead you away from the diagnosis of uremic encephalopathy as the primary etiology? A. Nuchal rigidity B. Myoclonus C. Hemiparesis D. Cerebrospinal fl uid (CSF) protein greater than 60 mg/dL E. All could be seen in uremic encephalopathy

25. A 36-year-old G2P1 woman at 35 weeks 3 days presents with nausea, vomiting, and mus- cle aches for the past 2 days. She had sudden onset of severe right upper quadrant (RUQ) pain 2 hours ago and presented to the doctor. Urine dipstick shows no proteinuria. She did have a headache. The most common maternal complication of this syndrome is: A. Seizures B. Disseminated intravascular coagulation (DIC) C. Liver rupture D. Cerebral hemorrhage E. Pulmonary edema

26. When diagnosing hypertensive encephalopathy, it is important to keep in mind that: A. The cerebrospinal fl uid (CSF) is normal, including IgG studies B. Patients with hypertensive encephalopathy always show features of posterior revers- ible encephalopathy syndrome (PRES) on MRI C. Seizures are a more common presenting symptom than headache or visual loss in PRES D. Cord lesions on MRI eliminate the possibility of PRES E. EEG generally shows periodic lateralized epileptiform discharges (PLEDs) in the pari- etal and occipital areas

146

66485457-66485438 www.ketabpezeshki.com

ZZakaria_87574_PTR_CH11_10-06-13_141-158.inddakaria_87574_PTR_CH11_10-06-13_141-158.indd 146146 66/19/2013/19/2013 4:45:024:45:02 PMPM ENCEPHALOPATHIES: Questions

27. The brain structures most sensitive to hypoxia include: A. Cortical layer 4 and 5 more than other cortical layers B. Caudate/putamen more than thalami C. Frontotemporal areas more than occipitoparietal areas D. White matter more than gray matter E. regions 3 and 4 more than other hippocampal areas

28. The nurse calls you with the laboratory results of your patient currently on a hypothermia protocol status—post cardiac arrest. Which of the following would not be an expected laboratory abnormality from the hypothermia alone? A. Hypokalemia B. Hypomagnesemia C. Hyperglycemia D. Thrombocytopenia E. Leukocytosis

29. A 65-year-old man on chronic hemodialysis is found down in his home. Vitals show normothermia, BP 110/70 mmHg, pulse 100 beats/minute. He is lethargic and confused, although he does follow very simple commands and is moving all four extremities. He is noted to have bilateral abducens nerve palsies but an otherwise nonfocal neurologic examination. CT head scan is normal. Labs show Cr 3.1 mg/dL (baseline), blood urea nitrogen (BUN) 40 mg/dL, normal liver function tests, and ammonia 30 µmol/L. Which intervention should be done fi rst? A. Lactulose B. Hemodialysis C. Thiamine IV D. EEG E. Empiric antibiotics

30. Which of the following scenarios could not be consistent with hypertensive encephalopathy? A. A 48-year-old man with end-stage renal disease and hypertension who presents with 3 to 4 days of acute dizziness, nausea, and unsteady gait but clear sensorium and BP 240/130 mmHg. CT head scan shows diffuse brainstem hypodensity B. A 60-year-old woman with sudden onset vision loss in the setting of rapidly ascend- ing paralysis of all four extremities over the past 3 days and bilateral facial palsies C. A 65-year-old man with uncontrolled hypertension presenting with BP 190/100 mmHg and a headache for the past month. He has a normal neurologic examination. D. A 55-year-old man with untreated hypertension who presented with drowsiness, left gaze deviation, and BP 240/140 mmHg. CT head scan shows noncommunicating hydrocephalus with cerebellar and brainstem hypodensity. E. All of the above

147

66485457-66485438 www.ketabpezeshki.com

ZZakaria_87574_PTR_CH11_10-06-13_141-158.inddakaria_87574_PTR_CH11_10-06-13_141-158.indd 147147 66/19/2013/19/2013 4:45:024:45:02 PMPM DISORDERS, DISEASES, SEIZURES, AND EPILEPSY

31. A 26-year-old G2P12 woman, who delivered a healthy baby girl 8 hours ago without complications, starts to complain of a severe headache. While the nurse is getting pain medication, she suddenly becomes unresponsive and has a generalized tonic–clonic sei- zure that lasts 3 minutes. She does not fully return to baseline, has another seizure, and has now returned to baseline. She had not previously been hypertensive, although BP now is 200/110 mmHg. CT head scan without contrast is negative. In regard to her treat- ment, you should: A. Load with fosphenytoin B. Start labetalol to lower blood pressure (BP) C. Start high-dose magnesium sulfate D. Do nothing, since the patient has already delivered and will likely not seize again E. Two of the above

32. A 28-year-old woman with diabetes mellitus (DM) for 3 years presents 6 hours after acute onset of hemisensory loss and loss of vision on one side. Her mother also has DM, and her brother is deaf, but there is no history of early cerebrovascular disease in the family. Which of the following is true about her most likely diagnosis? A. Headache, memory problems, hearing loss, short stature, and diabetes are uncommon comorbidities B. The most common presenting symptom in the second and third decade is stroke C. The most common identifi ed mutation is a point mutation at mitochondrial nucle- otide pair 3,243 (a tRNA-Leu gene) D. Medications to be avoided include metformin and l-arginine E. The syndrome is fully penetrant

148

66485457-66485438 www.ketabpezeshki.com

ZZakaria_87574_PTR_CH11_10-06-13_141-158.inddakaria_87574_PTR_CH11_10-06-13_141-158.indd 148148 66/19/2013/19/2013 4:45:034:45:03 PMPM 11

ANSWERS

1. The answer is B. Sudden severe throbbing headache is the most common herald of a seizure. Eclampsia historically has been thought to develop up to 48 hours postpartum, but is now well established to occur up to 4 weeks postpartum. The BP is often elevated for only a short time before the convulsion and in 20% of women is never elevated above 140 mmHg systolic BP. EEG in eclamptic patients shows diffuse or focal slowing that gradually returns to normal over several weeks, and CSF usually shows elevated protein and may show mild pleocytosis (1,2).

2. The answer is D. Ammonia levels above 200 µmol/L are more often associated with cerebral herniation, although there is no direct relationship with the ammonia levels (3). High intracranial pressure (ICP) can be seen at much lower ammonia levels, so depen- dence on monitoring ammonia levels alone is not recommended. Uncal herniation can be seen at much lower values.

3. The answer is D. Uremic encephalopathy is the most likely etiology in a patient with known chronic renal insuffi ciency and acute development of a hyperactive delirium associated with myoclonus and asterixis but preserved cranial nerve function (4). Anoxic injury or status epilepticus are also possibilities and need to be ruled out, but are less likely in this scenario. Morphine overdose would not cause the myoclonus seen and would show pupillary changes.

4. The answer is B. Almost all of these features are seen in high prevalence in preeclamp- sia, but headache is the most common, seen in 83% of patients. Clonus is seen in 46%, vision problems in 45%, and epigastric pain in 20% (1). Seizures are the defi ning feature of eclampsia, but are not seen in preeclampsia.

149

66485457-66485438 www.ketabpezeshki.com

ZZakaria_87574_PTR_CH11_10-06-13_141-158.inddakaria_87574_PTR_CH11_10-06-13_141-158.indd 149149 66/19/2013/19/2013 4:45:034:45:03 PMPM DISORDERS, DISEASES, SEIZURES, AND EPILEPSY

5. The answer is E. PRES is a clinico-radiological syndrome associated with multiple clini- cal conditions, including hypertensive encephalopathy, acute or chronic renal diseases, thrombotic thrombocytopenic purpura/hemolytic uremic syndrome (TTP/HUS), high- dose steroid therapy, liver failure and/or transplantation, lupus, endocrine dysfunctions (including primary aldosteronism, pheochromocytoma, hypercalcemia/hyperpara- thyroidism), bone marrow transplantation, massive blood transfusion/erythropoietin therapy, and porphyria, and with multiple drugs (cyclosporine, cis-platinum, some mono- clonal antibodies, antiretroviral therapy, intravenous immunoglobulins, contrast media, scorpion poison, and abuse of stimulants, etc.) (5).

6. The answer is A. The ability to accurately predict poor prognosis in anoxic brain injury has changed in the hypothermia era. Hypothermia does not cause absent N20 responses on SSEP, so this prognostic marker seems to remain valid (6). However, all of the other statements are true. Several series have shown good recovery in postcardiac arrest patients treated with therapeutic hypothermia who had high serum neuron-specifi c enolase values or had absent pupillary function at 24 hours or absent motor function at 72 hours (6–8). Hypothermia slows hepatic metabolism of sedatives and may require high doses of sedatives to treat the shivering associated with it, further complicating the picture.

7. The answer is B. When treating hypertensive encephalopathy, initial treatment usu- ally with IV medications should aim to drop the diastolic BP to about 100 mmHg or about a 25% drop (whichever is less) within 2 to 6 hours. Faster or further drops in BP may reduce the BP below the autoregulatory range and worsen neurologic symptoms. The goal is not necessarily to eliminate all neurologic symptoms. Sometimes there is an elevation in the creatinine associated with this initial blood-pressure lowering; how- ever, antihypertensive medicines should be continued. Nitroprusside is an effi cacious and quick BP-lowering agent; however, its use is limited at its maximal dose of 10 mcg/ kg/minute for at most 10 minutes, especially in renal insuffi ciency (9). Nitroglycerin is an effective antihypertensive agent in hypertensive emergency, but its use has been reported in multiple cases to worsen posterior reversible encephalopathy syndrome (PRES). Therefore, the use of nitroglycerin should be avoided in any case of hyperten- sive encephalopathy (10).

8. The answer is C. Multifocal parieto-occipital subcortical FLAIR lesions are seen in a major- ity of patients with eclampsia if MRI is obtained soon after convulsions. The pattern is sim- ilar to that seen in posterior reversible encephalopathy syndrome (PRES) and, similarly, is usually transient and reversible. Up to 25% of women may have persistent lesions (2,11).

9. The answer is B. MRI fi ndings in MELAS are usually asymmetric in nonvascular dis- tributions and characteristically involve the cortex and spare the deeper white matter. Usually the parietal/temporal/occipital lobes are more involved than frontal lobes on imaging, although neuropsychiatric testing of MELAS patients primarily shows a frontal pattern, suggesting that there is involvement at this level. MRS can be very useful, usually showing a low NAA peak and high lactate peak in affected areas—a very sensitive marker of metabolic disease. There is a strong correlation between high ventricular lactate as mea- sured by MRS and the degree of neuropsychological and neurologic impairment (12).

150

66485457-66485438 www.ketabpezeshki.com

ZZakaria_87574_PTR_CH11_10-06-13_141-158.inddakaria_87574_PTR_CH11_10-06-13_141-158.indd 150150 66/19/2013/19/2013 4:45:034:45:03 PMPM ENCEPHALOPATHIES: Answers

Specifi c involvement of the anterior temporal lobes is seen in cerebral autosomal dominant arteriopathy with subcortical infarctions and leukoencephalopathy (CADASIL) but not MELAS.

10. The answer is A. GFR is usually less than 10% when uremic encephalopathy occurs. Although uremic encephalopathy is often associated with electrolyte disturbances such as hyponatremia, hyperkalemia, metabolic acidosis, and hyperthyroidism, it can occur in isolation. The acuity of renal failure is a risk factor for the development of uremic enceph- alopathy more than the degree of azotemia. Uremic encephalopathy can present with myoclonus and multifocal myoclonus in the setting of altered mental status, which can be indistinguishable from status epilepticus without EEG (13).

11. The answer is A. Angiotensin converting enzyme (ACE) inhibitors and angiotensin- receptor blockers should be avoided in all stages of pregnancy (increased fetal renal abnormalities associated with latter half of pregnancy exposure and increased fetal cardiac abnormalities associated with fi rst trimester exposure). Methyldopa is usually considered the drug of choice for long-term management of hypertension during pregnancy, whereas labetalol, nicardipine, and/or hydralazine are often used for acute treatment (1).

12. The answer is D. Acute hepatic failure is defi ned as patients with previously normal liver function developing encephalopathy within 8 weeks of onset of their liver disease (14). The most common cause is drug exposure, most commonly acetaminophen, and the cause is viral hepatitis only about 10% of the time. Clinical presentation is usually acute onset of enceph- alopathy associated with nausea, vomiting, and abdominal pain with evidence of elevated transaminases and coagulopathy on labs. Cerebral edema occurs in 80% of comatose patients with acute hepatic failure and is a leading cause of death among fulminant patients (15). Mortality is 30% to 80% in patients who do not receive a liver transplant, although the out- come is very dependent on etiology and aggressive management strategies. Acute liver failure from acetaminophen overdose, pregnancy, or hepatitis A has better transplant-free survival rates than other drug toxicities (14–16).

13. The answer is D. Grade 3 HE demonstrates semistupor with gross confusion, bizarre or inap- propriate behavior with fi ts of paranoia or rage, associated with ataxia, nystagmus, and dys- arthria, with progression to coma and posturing or signs of herniation in Grade 4. Grade 2 HE entails disorientation, inappropriate behavior, personality changes, and impaired cognition with some incoordination and dysarthria. Grade 1 HE involves only a trivial lack of awareness, short attention span, and possibly euphoria or anxiety. Asterixis can occur at any grade (17).

14. The answer is D. The King’s College criteria are most commonly used to assess prognosis and therefore are needed for transplant in acute liver failure (ALF) patients. For acetamin- ophen-induced ALF, arterial pH < 7.3 irrespective of coma grade or PT > 100 seconds (INR 6.5) and serum creatinine 300 mol/L (3.4 mg/dL) in patients with grade III/IV coma are associated with a very poor outcome without transplant. In non–acetaminophen-induced ALF, the criteria used to predict poor prognosis without transplant are a PT > 100 seconds irrespective of coma grade or any three of the following irrespective of coma grade: drug toxicity; indeterminate cause of ALF; age younger than 10 years or older than 40 years; jaundice to coma interval greater than 7 days; PT > 50 seconds (INR > 3.5); serum bilirubin greater than 300 µmol/L (17.5 mg/dL) (16,18).

151

66485457-66485438 www.ketabpezeshki.com

ZZakaria_87574_PTR_CH11_10-06-13_141-158.inddakaria_87574_PTR_CH11_10-06-13_141-158.indd 151151 66/19/2013/19/2013 4:45:034:45:03 PMPM DISORDERS, DISEASES, SEIZURES, AND EPILEPSY

15. The answer is C. Retrospective trials have shown no benefi t of lactulose or neomycin on morbidity or mortality in acute hepatic failure and can cause gaseous abdominal disten- tion, which could interfere with liver transplant. Neomycin is specifi cally contraindicated because of its propensity for nephrotoxicity. ICP monitoring has been shown to be ben- efi cial and is recommended in all Grade III/IV hepatic encephalopathy (HE)—cerebral edema occurs in 86% to 95% of Grade III/IV HE. CT head scan is an insensitive indi- cator of elevated ICP. Intracranial hypertension should be treated as in other etiologies with conservative measures and then with hyperosmolar therapy. Prophylactic antibiotics are recommended in patients if there is advanced stage HE, refractory hypotension, and presence of systemic infl ammatory response syndrome, or who are candidates for organ transplant. N-acetylcysteine is known to be benefi cial in acute hepatic failure secondary to acetaminophen toxicity, but is now recognized to also be helpful in non–acetaminophen- induced liver failure, especially if given early (16,18–20).

16. The answer is E. Mild-to-moderate coagulopathies are expected in acute liver failure and should be corrected if possible prior to ICP monitor placement, which is known to have bleeding complications in 10% to 20% of hepatic encephalopathy (HE) patients; although, as reported in the literature, these are usually mild. Vitamin K is recommended in all patients with acute liver failure. FFP would be recommended to help correct the INR more quickly if a procedure is warranted, although giving it prophylactically is more controversial. DDAVP should be given to help correct uremia-induced platelet dysfunction (16,18,19).

17. The answer is D. CVVH is the preferred modality for dialysis in patients with fulminant liver failure because of its ability to smooth fl uid shifts, continuously address electro- lyte disturbances, and allow adjustments in osmotic therapy. Intermittent hemodialysis is often poorly tolerated because of hemodynamic instability and fl uid shifts—it in fact can increase intracranial pressure (ICP). Acute renal failure complicates up to 50% of patients with acute liver failure. Renal failure results in increased urea, which is acted upon by colonic bacteria similarly to protein, resulting in increased ammonia and leading to hepa- torenal syndrome. It is unlikely that fl uid resuscitation will correct the renal failure, and intervention is required (16,18,19).

18. The answer is D. According to the American Academy of Neurology (AAN) Practice Parameters, the following clinical fi ndings accurately predict universally poor outcome: myoclonic status epilepticus within the fi rst 24 hours in patients with primary circulatory arrest, absence of pupillary responses within 1 to 3 days after CPR, absent corneal refl exes within 1 to 3 days after CPR, and absent or extensor motor responses after 3 days (21). Up to 90% of survivors of anoxic injury have awoken by 72 hours, so a continued comatose state would be a discouraging sign, but it is not an accurate predictor of universally poor outcome (22). Earlier pupillary or motor examination fi ndings have not been found to be as accurate in prediction. Hyperthermia is associated with worse outcome, but is not predictive.

19. The answer is B. Benzodiazepines, diuretics, and narcotics are all associated with wors- ening or decompensation of hepatic encephalopathy (HE). Topiramate can cause hyper- ammonemia as well. Haloperidol is generally considered safer in patients at risk for or with HE (17).

152

66485457-66485438 www.ketabpezeshki.com

ZZakaria_87574_PTR_CH11_10-06-13_141-158.inddakaria_87574_PTR_CH11_10-06-13_141-158.indd 152152 66/19/2013/19/2013 4:45:034:45:03 PMPM ENCEPHALOPATHIES: Answers

20. The answer is D. Magnesium sulfate administration is associated with predictable maternal effects, including loss of DTRs (>4–6 mEq/L), then somnolence and respira- tory depression (>8 mEq/L), then cardiotoxicity (>15 mEq/L). Magnesium sulfate infu- sion also causes fl ushing (likely secondary to its vasodilatory effect) and hypocalcemia. Magnesium sulfate can sensitize patients to nondepolarizing muscle relaxants (and to a lesser extent depolarizing muscle relaxants), so smaller doses are recommended (1).

21. The answer is E. Many drug overdoses are well known to present with coma, with pupil- lary function often the distinguishing feature from an examination mimicking brain death. However, some drugs in overdose, including TCAs, barbiturates, and valproic acid, have been associated with complete abolition of brainstem function, including pupillary responses (21,23).

22. The answer is D. Although most of these fi ndings are associated with poor outcome, only absent N20 responses within days 1 to 3 are associated with an invariably poor prognosis. Burst-suppression, BiPLEDs or GPEDs, and alpha or theta coma on EEG are associated with poor outcome but not universally. Brainstem auditory evoked potentials have not been shown to have prognostic value (21).

23. The answer is C. The cortex is more sensitive to anoxia than the brainstem, so memory is one of the fi rst systems affected. This explains the common residual memory problems after an episode of cardiac arrest. Of the brainstem functions, control of spontaneous res- pirations is usually the last affected (22).

24. The answer is E. Signs and symptoms of uremic encephalopathy include delirium, visual hallucination, agitation, and torpor evolving to coma. Seizures (partial or gener- alized) are seen in one-third of uremic encephalopathy patients, as are signs of menin- geal irritation. Multifocal myoclonus and asterixis are very common in later stages of uremic encephalopathy that could progress to total body multifocal myoclonus—the described “uremic twitch syndrome.” Hemiparesis occurs in almost half of the patients, which may alternate sides during the illness. Papilledema and dysarthria can also be seen. Cerebrospinal fl uid (CSF) fi ndings include elevated protein and pleocytosis, mim- icking aseptic meningitis (13,24).

25. The answer is B. This vignette describes the presentation of a patient with HELLP syn- drome, which is defi ned as hemolysis with a microangiopathic blood smear, elevated liver enzymes, and a low platelet count. It usually occurs in the third trimester, although it can occur earlier or postpartum. It can be associated with severe preeclampsia and eclamp- sia. Maternal complications include DIC most frequently, but also liver rupture, cerebral hemorrhage, and pulmonary edema. Fatal pontine hemorrhage and cerebellar infarctions have been described (25,26). Neonatal complications include placental abruption, intra- uterine growth retardation (IUGR), preterm labor, and perinatal death. If the syndrome develops after 34 weeks, delivery is usually recommended (27).

26. The answer is C. Hypertensive encephalopathy often shows MRI FLAIR (fl uid attenu- ated inversion recovery) lesions consistent with PRES; however, the MRI can be normal. When PRES is present radiographically, however, encephalopathy is noted in more than

153

66485457-66485438 www.ketabpezeshki.com

ZZakaria_87574_PTR_CH11_10-06-13_141-158.inddakaria_87574_PTR_CH11_10-06-13_141-158.indd 153153 66/19/2013/19/2013 4:45:034:45:03 PMPM DISORDERS, DISEASES, SEIZURES, AND EPILEPSY

90% of patients, seizures in more than 85%, headache in 53%, and visual symptoms in only 39% (28). The MRI can show cord lesions. The EEG most commonly shows general- ized slowing, not PLEDS. Although lumbar puncture (LP) is not usually recommended or required in hypertensive encephalopathy due to cerebral edema, the CSF usually reveals normal protein and cell count with elevated IgG studies (29).

27. The answer is B. Different brain structures are variably sensitive to hypoxia and therefore react differently. The gray matter is more sensitive than the white matter, and the occip- itoparietal regions are more sensitive than the frontotemporal regions. Of the six cortical layers, layer 3 (the external pyramidal layer) is the most sensitive to hypoxia, explaining the laminar necrosis sometimes seen as sequelae of hypoxic injury. Hippocampal region 2 is the most vulnerable of the hippocampal regions. The caudate and putamen are more sensitive to hypoxia than the thalami (22).

28. The answer is E. Hypothermia is associated with a “cold diuresis” resulting in hypovolemia and resultant hypokalemia and hypomagnesemia directly and from intracellular shifts. Hypothermia also decreases insulin secretion and increases insulin resistance, resulting in hyperglycemia that may be resistant to treatment. A mild bleeding diathesis is well described in hypothermia secondary to usually mild thrombocytopenia as well as impair- ment of qualitative platelet function and the clotting cascade. Leukopenia and increased susceptibility to infections are well known in hypothermia, but leukocytosis is not an expected side effect of the hypothermia itself (30).

29. The answer is C. Differential diagnosis includes intracranial cerebral hemorrhage (ICH) or subarachnoid hemorrhage (SAH) resulting in elevated intracranial pressure (ICP), basi- lar meningitis (especially Listeria given known immunosuppressed status), Wernicke’s encephalopathy, nonconvulsive status epilepticus, alcohol withdrawal, and uremic and/ or hepatic encephalopathy (HE), among others. A brain CT scan should be done in all chronic dialysis patients presenting with focal defi cits due to the high risk of subdural hemorrhage secondary to platelet dysfunction and brain atrophy, and is normal in this case. Stroke is also a possibility, but more focal fi ndings would be expected. Basilar men- ingitis, especially Listeria given the patient’s risk factors, should be considered, but the lack of other suggestive features makes this less likely. Nonconvulsive status epilepticus is always a possibility, and seizures occur in up to one-third of uremic patients; however, this would not explain the lateral rectus palsies. Uremic and HE may be contributing, but do not explain the entire picture. This patient has two of the triad for Wernicke’s enceph- alopathy, and this should be considered early and be treated as in this case. Wernicke’s encephalopathy is rare, but patients on dialysis are at higher risk because of low thiamine intake and hemodialysis causes accelerated loss of thiamine (31). Because it can mimic uremic encephalopathy and often remains unrecognized, Wernicke’s encephalopathy needs to be treated early.

30. The answer is C. Hypertensive encephalopathy is defi ned as the presence of signs of cere- bral edema caused by breakthrough hyperperfusion from severe and sudden rises in BP and classically characterized by the relatively rapid onset of headache, nausea, and vomit- ing, followed by nonlocalizing neurologic symptoms such as restlessness, confusion, and, if the hypertension is not treated, seizures and coma. However, atypical presentations

154

66485457-66485438 www.ketabpezeshki.com

ZZakaria_87574_PTR_CH11_10-06-13_141-158.inddakaria_87574_PTR_CH11_10-06-13_141-158.indd 154154 66/19/2013/19/2013 4:45:034:45:03 PMPM ENCEPHALOPATHIES: Answers

include focal neurologic signs, or brainstem rather than parieto-occipital radiologic involve- ment (32). While imaging can be normal in hypertensive encephalopathy, there is usually much more extensive radiologic involvement than is clinically apparent—the “clinical– r adiological dissociation.” Development of frank noncommunicating hydrocepha- lus necessitating external ventricular drain (EVD) has been reported—symptoms that improved with hypertensive management (33). The autonomic instability associated with GBS and other autonomic neuropathies can also precipitate hypertensive encephalopathy (34). The absolute level of BP may not be as important as the rate of increase, and patients with long-standing hypertension can tolerate higher mean arterial pressures (MAPs) without development of hypertensive encephalopathy—the patient described in C does not have progressive symptoms and does not have hypertensive encephalopathy (9).

31. The answer is E. Eclampsia can occur up to 4 weeks postpartum. Magnesium sulfate should be started and continued for 24 hours to prevent further seizures. Multiple trials comparing magnesium sulfate to benzodiazepines and phenytoin have shown superior- ity of magnesium sulfate in preventing further seizures in eclampsia as well as preventing the development of seizures in severe preeclampsia (35). Labetalol or other antihyperten- sive should also be started to control this patient’s BP.

32. The answer is C. This patient most likely has mitochondrial encephalopathy with lac- tic acidosis and stroke-like episodes (MELAS), given her family history and presenta- tion. Based on a study of the natural history of MELAS, hearing loss rather than strokes is the most common presenting symptom of MELAS in the second and third decade. Headache, exercise intolerance, memory problems, hearing loss, short stature, and dia- betes are common comorbidities. The point mutation at mitochondrial nucleotide pair 3,243 (a tRNA-Leu gene) has been identifi ed as likely responsible for 80% of patients with MELAS. This mutation has also been identifi ed as a cause of diabetes and cardiomyo- pathy in patients who never manifest the entire MELAS syndrome. It is estimated that this mutation accounts for 2% of all type 2 DM cases. Given its propensity to cause lactic acidosis, it is recommended that metformin be avoided in patients with DM and MELAS, but l-arginine is thought to be helpful, perhaps even capable, in reversing or preventing the stroke like episodes in MELAS (12,36).

References

1. Kaplan PW. Eclampsia. In: Kaplan PW, ed. Neurologic Disease in Women. New York, NY: Demos Medical Publishing; 2006:235–246. 2. Shah AK, Rajamani K, Whitty JE. Eclampsia: a neurological perspective. J Neurol Sci. 2008;271(1–2):158–167. 3. Clemmesen JO, Larsen FS, Kondrup J, Hansen BA, Ott P. Cerebral herniation in patients with acute liver failure is correlated with arterial ammonia concentration. Hepatology. 1999;29(3):648–653. 4. Brouns R, De Deyn PP. Neurological complications in renal failure: a review. Clin Neurol Neurosurg. 2004;107(1):1–16. 5. Servillo G, Bifulco F, De Robertis E, et al. Posterior reversible encephalopathy syndrome in intensive care medicine. Intensive Care Med. 2007;33(2):230–236.

155

66485457-66485438 www.ketabpezeshki.com

ZZakaria_87574_PTR_CH11_10-06-13_141-158.inddakaria_87574_PTR_CH11_10-06-13_141-158.indd 155155 66/19/2013/19/2013 4:45:034:45:03 PMPM DISORDERS, DISEASES, SEIZURES, AND EPILEPSY

6. Rossetti AO, Oddo M, Logroscino G, Kaplan PW. Prognostication after cardiac arrest and hypothermia: a prospective study. Ann Neurol. 2010;67(3):301–307. 7. Al Thenayan E, Savard M, Sharpe M, Norton L, Young B. Predictors of poor neuro- logic outcome after induced mild hypothermia following cardiac arrest. Neurology. 2008;71(19):1535–1537. 8. Fugate JE, Wijdicks EF, Mandrekar J, et al. Predictors of neurologic outcome in hypo- thermia after cardiac arrest. Ann Neurol. 2010;68(6):907–914. 9. Marik PE, Rivera R. Hypertensive emergencies: an update. Curr Opin Crit Care. 2011; 17(6):569–580. 10. Sheta MA, Paladugu M, Mendelson J, Holland NR. When should nitroglycerine be avoided in hypertensive encephalopathy? Hypertension. 2011;58(5):e187–e188. 11. Zeeman GG. Neurologic complications of pre-eclampsia. Semin Perinatol. 2009;33(3):166–172. 12. Sproule DM, Kaufmann P. Mitochondrial encephalopathy, lactic acidosis, and strokelike episodes: basic concepts, clinical phenotype, and therapeutic management of MELAS syndrome. Ann N Y Acad Sci. 2008;1142:133–158. 13. Raskin NH. Neurologic complications of renal failure. In: Aminoff MJ, ed. Neurology and General Medicine. Philadelphia, PA: Churchill Livingstone; 2001:293–296. 14. Lockwood AH. Hepatic encephalopathy. In: Aminoff MJ, ed. Neurology and General Medicine. Philadelphia, PA: Churchill Livingstone; 2001:233–244. 15. Ostapowicz G, Fontana RJ, Schiødt FV, et al. Results of a prospective study of acute liver failure at 17 tertiary care centers in the United States. Ann Intern Med. 2002;137(12):947–954. 16. Frontera JA, Kalb T. Neurological management of fulminant hepatic failure. Neurocrit Care. 2011;14(2):318–327. 17. Bismuth M, Funakoshi N, Cadranel JF, Blanc P. Hepatic encephalopathy: from pathophys- iology to therapeutic management. Eur J Gastroenterol Hepatol. 2011;23(1):8–22. 18. Polson J, Lee WM. AASLD position paper: the management of acute liver failure. Hepatology. 2005;41(5):1179–1197. 19. Stravitz RT, Kramer AH, Davern T, et al. Intensive care of patients with acute liver failure: recommendations of the U.S. Acute Liver Failure Study Group. Crit Care Med. 2007;35(11):2498–2508. 20. Wright G, Chattree A, Jalan R. Management of hepatic encephalopathy. Int J Hepatol. 2011;2011:841407. 21. Wijdicks EF, Hijdra A, Young GB, Bassetti CL, Wiebe S. Practice parameter: prediction of outcome in comatose survivors after cardiopulmonary resuscitation (an evidence-based review): report of the Quality Standards Subcommittee of the American Academy of Neurology. Neurology. 2006;67(2):203–210. 22. Longstreth Jr WT. Neurologic complications of cardiac arrest. In: Aminoff MJ, ed. Neurology and General Medicine. Philadelphia, PA: Churchill Livingstone; 2001:151–170. 23. Auinger K, Müller V, Rudiger A, Maggiorini M. Valproic acid intoxication imitating brain death. Am J Emerg Med. 2009;27(9):1177.e5–1177.e6. 24. Burn DJ, Bates D. Neurology and the kidney. J Neurol Neurosurg Psychiatr. 1998; 65(6):810–821.

156

66485457-66485438 www.ketabpezeshki.com

ZZakaria_87574_PTR_CH11_10-06-13_141-158.inddakaria_87574_PTR_CH11_10-06-13_141-158.indd 156156 66/19/2013/19/2013 4:45:034:45:03 PMPM ENCEPHALOPATHIES: Answers

25. Zeidman LA, Videnovic A, Bernstein LP, Pellar CA. Lethal pontine hemorrhage in post- partum syndrome of hemolysis, elevated liver enzyme levels, and low platelet count. Arch Neurol. 2005;62(7):1150–1153. 26. Altamura C, Vasapollo B, Tibuzzi F, et al. Postpartum cerebellar infarction and haemolysis, elevated liver enzymes, low platelet (HELLP) syndrome. Neurol Sci. 2005;26(1):40–42. 27. Haram K, Svendsen E, Abildgaard U. The HELLP syndrome: clinical issues and manage- ment. A review. BMC Pregnancy Childbirth. 2009;9:8. 28. Lee VH, Wijdicks EF, Manno EM, Rabinstein AA. Clinical spectrum of reversible poste- rior leukoencephalopathy syndrome. Arch Neurol. 2008;65(2):205–210. 29. Takano T, Koyanagi A, Osawa Y, Taga T, Fujino H. Cerebrospinal fl uid interleukin-6 lev- els in hypertensive encephalopathy: a possible marker of disease activity. Ann Neurol. 2001;49(5):685. 30. Polderman KH, Herold I. Therapeutic hypothermia and controlled normothermia in the intensive care unit: practical considerations, side effects, and cooling methods. Crit Care Med. 2009;37(3):1101–1120. 31. Ihara M, Ito T, Yanagihara C, Nishimura Y. Wernicke’s encephalopathy associated with hemodialysis: report of two cases and review of the literature. Clin Neurol Neurosurg. 1999;101(2):118–121. 32. Kitaguchi H, Tomimoto H, Miki Y, et al. A brainstem variant of reversible posterior leuko- encephalopathy syndrome. Neuroradiology. 2005;47(9):652–656. 33. Kumar A, Keyrouz SG, Willie JT, Dhar R. Reversible obstructive hydrocephalus from hypertensive encephalopathy. Neurocrit Care. 2012;16(3):433–439. 34. Bavikatte G, Gaber T, Eshiett MU. Posterior reversible encephalopathy syndrome as a complication of Guillain-Barré syndrome. J Clin Neurosci. 2010;17(7):924–926. 35. Sibai BM. Magnesium sulfate prophylaxis in preeclampsia: Lessons learned from recent trials. Am J Obstet Gynecol. 2004;190(6):1520–1526. 36. Kaufmann P, Engelstad K, Wei Y, et al. Natural history of MELAS associated with mito- chondrial DNA m.3243A>G genotype. Neurology. 2011;77(22):1965–1971.

157

66485457-66485438 www.ketabpezeshki.com

ZZakaria_87574_PTR_CH11_10-06-13_141-158.inddakaria_87574_PTR_CH11_10-06-13_141-158.indd 157157 66/19/2013/19/2013 4:45:034:45:03 PMPM 66485457-66485438 www.ketabpezeshki.com

ZZakaria_87574_PTR_CH11_10-06-13_141-158.inddakaria_87574_PTR_CH11_10-06-13_141-158.indd 158158 66/19/2013/19/2013 4:45:034:45:03 PMPM 12 Clinical Syndromes Howard J. Fan and Asma Zakaria QUESTIONS

1. A 50-year-old man with no vascular risk factors presents to the ED with a low Glasgow Coma Scale (GCS) score and noted systolic BP 220 mmHg, pulse 50 beats/minute, and abnormal respirations. Before neuroimaging, the nurse reports episodic extensor postur- ing, poor mentation, and cardiac dysrhythmias but no lateralizing motor signs or pupil- lary changes. She is concerned that the patient is herniating. What can be expected on CT scan? A. Transtentorial (uncal) herniation B. Central herniation C. Upward herniation D. Cerebellar herniation E. External herniation

2. An overnight resident concerned with intracranial pressure (ICP) waveforms on an ICU patient wakes you with a picture of the waveform. It shows elevation of P2 wave greater than P1. What would you tell the resident? A. P1 pressure wave is produced from venous pulsations B. P1 pressure wave is typically lower than P2 C. P2 pressure wave results from restriction of ventricular expansion D. Small increases in intracranial volume may cause large increases in ICP E. Both C and D

ANSWERS TO THIS SECTION CAN BE FOUND ON PAGE 167 159

66485457-66485438 www.ketabpezeshki.com

ZZakaria_87574_PTR_CH12_10-06-13_159-174.inddakaria_87574_PTR_CH12_10-06-13_159-174.indd 159159 66/19/2013/19/2013 8:46:308:46:30 PMPM DISORDERS, DISEASES, SEIZURES, AND EPILEPSY

3. A male patient in the ICU presents with a right hemispheric mass with minimal edema and a poor Glasgow Coma Scale (GCS) score on initial evaluation. The cause(s) of his poor mental status can be attributed to: A. Small brainstem lesion B. Metabolic encephalopathy C. Drug overdose D. Dominant hemispheric infarct E. All of the above

4. All of the following clinical or radiographic features are more common in oral anticoagulant–associated intracranial hemorrhage (ICH) than in spontaneous hyperten- sive intracerebral hemorrhage except: A. Presence of a fl uid level B. Focal neurologic defi cit C. Multiple discrete areas of hemorrhage D. Papilledema and headache E. Rapid expansion of the hematoma

5. An intracranial pressure (ICP) monitor is indicated in which of the following scenarios? A. Obstructive hydrocephalus B. Communicating hydrocephalus with signs of elevated ICP C. Subarachnoid hemorrhage with altered mental status D. TBI with Glasgow Coma Scale (GCS) score less than 8 E. Intracranial hemorrhage (ICH) with intraventricular involvement of the third and fourth ventricles F. All of the above

6. A 40-year-old patient presents with a focal parietal lesion with increased signal intensity on T2, diffusion-weighted imaging, apparent diffusion coeffi cient, and primarily white matter edema with maintenance of gray–white matter interface on CT and MRI imaging. Which of the following is/are true? A. 10 mg dexamethasone IV every 6 hours B. 0.5 to 1 g/kg mannitol IV every 6 hours to maintain serum Na 145 to 155 mEq/L C. Fever, if present, should be treated with acetaminophen 650 mg PO q4hrs or external cooling devices if required D. Both B and C E. Both A and C

7. A 20-year-old man with traumatic brain injury (TBI) and uncontrolled intracranial hyper-

tension undergoes prolonged hyperventilation for 8 hours overnight. At what PaCO2 would you begin to worry about provoking or worsening cerebral ischemia?

A. PaCO2 35

B. PaCO2 30

C. PaCO2 28

D. PaCO2 25

160

66485457-66485438 www.ketabpezeshki.com

ZZakaria_87574_PTR_CH12_10-06-13_159-174.inddakaria_87574_PTR_CH12_10-06-13_159-174.indd 160160 66/19/2013/19/2013 8:46:308:46:30 PMPM CLINICAL SYNDROMES: Questions

8. The respiratory rate for the patient in Question 7 is lowered, and the intracranial pressure (ICP) starts spiking soon thereafter. You are now concerned for: A. Refractory intracranial hypertension B. Inadequate sedation C. Rebound intracranial hypertension D. Refl ex cerebral vasoconstriction

9. The intracranial pressure (ICP) monitor placed in a young traumatic brain injury (TBI) patient is reviewed the next morning by the attending on service. Sustained, high-ampli- tude elevation of ICP at 55 mmHg is seen for 30 minutes. Which of the following should there be concern about? A. Lundberg A wave B. Lundberg B wave C. Lundberg C wave D. Plateau wave E. Both A and D

10. A 62-year-old man with unknown medical history presents to the ED with headache and emesis. A CT scan of the head shows dilated lateral ventricles and bowing of the third ventricle with absence of fourth ventricular dilation. MRI of the brain shows increased T2 signal in the periventricular area. Compression of the quadrigeminal plate will cause which of the following focal neurological defi cits? A. Parinaud’s syndrome B. Abducens nerve palsy C. Oculomotor palsy D. Visual fi eld defi cit

11. A 37-year-old man with a history of recent IV drug abuse presents with progressive obtun- dation leading to coma. He is febrile with a grade 3 systolic ejection murmur and Janeway lesions. His head CT scan demonstrates multifocal abnormalities consistent with embolic infarctions in multiple vascular territories of the brain. Injury to which of the following areas would be suffi cient to cause coma? A. Basis pontis B. Bilateral occipital lobes C. Bilateral parietal lobes D. Right frontal lobe and left occipital lobe E. Bilateral thalami

12. In which of the following conditions has therapeutic hypothermia been documented to improve clinical outcomes? A. Acute cerebral infarction B. Acute spinal cord trauma C. Cardiac arrest D. Subarachnoid hemorrhage E. Traumatic brain injury (TBI)

161

66485457-66485438 www.ketabpezeshki.com

ZZakaria_87574_PTR_CH12_10-06-13_159-174.inddakaria_87574_PTR_CH12_10-06-13_159-174.indd 161161 66/19/2013/19/2013 8:46:308:46:30 PMPM DISORDERS, DISEASES, SEIZURES, AND EPILEPSY

13. Specifi c consideration during the induction phase of therapeutic hypothermia includes: A. Shivering B. Core temperature maintenance C. Electrolyte disturbance D. Induction time

14. A 50-year-old woman suffers a massive embolic stroke during cardiac valve surgery. She is treated with hypothermia to a temperature of 32°C. After 6 hours, she is rewarmed. Which of the following is a risk specifi c to the rewarming phase of hypothermia treatment? A. Asystole B. Coagulopathy C. Hypotension D. Thrombocytopenia E. Increased intracranial pressure (ICP)

15. A 73-year-old woman presents with sudden onset of left hemiparesis and right gaze pref- erence. Her family wishes to know more when informed of the prognosis for early death from her condition. Which of the following is a known risk factor for fatal brain edema as a complication of acute ischemic stroke? A. Blood glucose of 120 mg/dL B. High NIH stroke scale score C. Diastolic BP greater than 110 mmHg D. Diffusion lesion volume greater than 40 mL in 24 hours E. Elevated creatinine

16. A 61-year-old man with vascular risk factors for hypertension, coronary artery disease, and hyperlipidemia was witnessed collapsing at a gas station. He was found to be in ven- tricular fi brillation by emergency services. After 24 hours postarrest, he remains comatose with pupils reactive to light. Which of the following tests might provide predictive dis- crimination of outcome? A. MRI brain B. Duration of cardiopulmonary resuscitation at scene C. EEG D. CT scan E. Neuron-specifi c enolase level

17. Undergoing therapeutic hypothermia portends which of the following complications? A. Autoimmune renal injury B. Coagulopathy C. Hepatic failure D. Hypertension E. Seizures

162

66485457-66485438 www.ketabpezeshki.com

ZZakaria_87574_PTR_CH12_10-06-13_159-174.inddakaria_87574_PTR_CH12_10-06-13_159-174.indd 162162 66/19/2013/19/2013 8:46:318:46:31 PMPM CLINICAL SYNDROMES: Questions

18. A 56-year-old man presents with sudden onset of severe headache and sleepiness. His med- ical history is remarkable for alcohol and tobacco abuse. On triage, his BP is 150/90 mmHg, Glasgow Coma Scale (GCS) score is 13, and the pupils are both 3 mm and reactive. A CT scan of the head reveals a 12 cm3 left basal ganglia hemorrhage with extension into the left lateral and third ventricles. Two hours later, he is poorly responsive, and his GCS score falls to 9. Repeat CT scan shows no change in hematoma size, but marked enlargement of the third and lateral ventricles. Which of the following interventions is most likely to improve his clinical status? A. IV mannitol B. IV thiamine C. External ventricular drain D. Therapeutic hypothermia E. Lowering mean arterial pressure

19. A 61-year-old man with a history of coronary artery disease, dyslipidemia, and well- controlled hypertension presents with acute onset of left hemiparesis, hemisensory defi cit, and left-sided neglect. His initial BP is 260/120 mmHg, but it is reduced to 205/110 mmHg with a single dose of labetalol. Head CT scan is unremarkable, and he is treated with recombinant tissue-type plasminogen activator. He is then transferred to a stroke unit for careful monitoring. National Institute of Neurological Disorders and Stroke (NINDS) guidelines recommend treatment of elevated BP equal to or greater than which of the fol- lowing values (in mmHg)? A. Systolic 160, diastolic 90 B. Systolic 180, diastolic 105 C. Systolic 200, diastolic 110 D. Systolic 220, diastolic 120 E. Systolic 240, diastolic 130

20. Which of the following EEG patterns best correlates with full recovery after cardiac arrest? A. Dominant theta–delta activity without detectable normal alpha B. Dominant nonreactive alpha activity C. Burst suppression D. Low-voltage delta activity without theta or alpha activity E. Periodic lateralizing discharges

21. A 70-year-old woman presents to the hospital with acute onset of left hemiparesis and hemisensory loss. Her BP is 205/100 mmHg. On examination, she is drowsy, but can follow commands with effort. Her Glasgow Coma Scale (GCS) score is 13. A noncontrast brain CT scan shows a 15 cm3 right basal ganglia hemorrhage with no ventricular involve- ment. She is started on a nitroprusside infusion, reducing her BP to 165/90 mmHg. Her mental status deteriorates, with the GCS score falling to 8. Repeat CT scan shows no change in the hemorrhage size, no hydrocephalus, and no subarachnoid or intraventricu- lar blood. Which of the following is the most likely etiology of her altered mental status? A. Hypertensive encephalopathy B. Seizure C. Vasospasm D. Pressure-dependent reinfarction E. Nitroprusside-induced intracranial pressure (ICP) elevation 163

66485457-66485438 www.ketabpezeshki.com

ZZakaria_87574_PTR_CH12_10-06-13_159-174.inddakaria_87574_PTR_CH12_10-06-13_159-174.indd 163163 66/19/2013/19/2013 8:46:318:46:31 PMPM DISORDERS, DISEASES, SEIZURES, AND EPILEPSY

22. A 54-year-old woman develops a rapidly progressive paraparesis with a sensory level at T6. Which of the following features would be most suggestive of an etiology other than idiopathic transverse myelitis (TM)? A. Asymmetrical weakness B. Cerebrospinal fl uid (CSF) pleocytosis C. Elevated IgG index D. Enhancing spinal cord lesion on spinal imaging E. Progression to nadir within 2 hours

23. A 32-year-old man with a history of chronic thrombocytopenia presents to the ED with an abrupt onset of headache, vomiting, and left hemiparesis. A CT scan of the brain dem- onstrates a 45 cm3 hematoma within the right lenticular nucleus and internal capsule. He has been taking warfarin for the past year. A “spot sign” is seen on neuroimaging. Which of the following has the least supporting evidence in terms of the next best clinical step? A. Fresh-frozen plasma (FFP) B. Vitamin K C. Factor VII D. Platelet transfusion or 1-deamino-8-d-arginine vasopressin E. BP control

24. A 49-year-old man is involved in a motor vehicle accident and sustains a signifi cant closed traumatic brain injury (TBI). He presents to the ED with a Glasgow Coma Scale (GCS) score of 3 and remains unchanged for weeks until he is transferred to a long-term care facility. He has responsive pupils, makes conjugate roving eye movements, grimaces inconsistently to noxious stimuli, and withdraws to pain but does not make a localized response. Which of the following terms best describes this man’s state of consciousness? A. Catatonia B. Locked-in state C. Minimally conscious state D. Persistent vegetative state E. Coma

25. A 50-year-old man with vascular risk factors for hypertension, coronary artery disease, and prior myocardial infarction 1 year ago is brought to the ED within 30 minutes onset of acute right hemiparesis and dysarthria. His family reports no other medical history. BP is 175/90 mmHg. A brain CT scan shows effacement of the gray–white interface in the left insula. Electrocardiogram, blood glucose, and platelet count are normal. Coagulation studies are pending. His home regimen includes metoprolol, aspirin, and simvastatin. What is the recommended approach regarding treatment with IV recombinant tissue plasminogen activator (rt-PA)? A. Administer rt-PA as soon as possible B. Await the results of coagulation studies before administering tissue plasminogen activator C. Do not administer rt-PA in this patient since he has been taking aspirin D. Obtain written informed consent E. Reduce arterial BP to reduce the risk of intracerebral hemorrhage

164

66485457-66485438 www.ketabpezeshki.com

ZZakaria_87574_PTR_CH12_10-06-13_159-174.inddakaria_87574_PTR_CH12_10-06-13_159-174.indd 164164 66/19/2013/19/2013 8:46:318:46:31 PMPM CLINICAL SYNDROMES: Questions

26. An 80-year-old man is found collapsed in a parking lot. Emergency services arrive at the scene and fi nd no respirations or palpable pulse. Cardiopulmonary resuscitation is started with an initial rhythm of ventricular fi brillation. A pulse is restored after 15 min- utes, with a subsequent Glasgow Coma Scale (GCS) score of 3. On examination, pupils are 2 mm sluggishly reactive, but oculocephalic refl ex and corneal refl exes are absent. During his hospital stay, he develops myoclonic jerks of his face, tongue, and upper extremities but no improvement in GCS score or brainstem refl exes. An EEG is most likely to demon- strate which of the following patterns? A. Delta–theta coma B. Atypical spike-and-wave activity C. Alpha coma D. Burst suppression E. Electrocerebral silence

27. A 60-year-old woman experiences a cardiac arrest after a run of ventricular tachycardia. Emergency resuscitation requires 30 minutes of effort before stabilization. Emergency ser- vices report the absence of pupillary response in either eye after the code. On morning rounds, the intern cites a study in which a nonreactive pupillary response is almost uni- formly predictive of severe defi cit and poor neurological outcome. Which of the follow- ing is/are marker(s) of poor outcome after cardiac resuscitation? A. Initial absence of pupillary light response B. Extensor or no motor response after 3 days C. Myotonic status epilepticus D. Burst suppression on EEG E. Bilaterally absent cortical somatosensory evoked potential (SSEP) responses F. Elevated serum neuron-specifi c enolase G. All of the above

28. A 75-year-old woman involved in a motor vehicle accident while wearing a lap belt is brought into the ED and found to have a low-resting BP, bradycardia, and systemic signs of shock. Which of the following underlying etiologies is the likely cause? A. Vasodilation B. Heart failure C. Sudden epinephrine release D. Volume depletion

29. A 64-year-old man is brought to the ED by family after he was found confused at home. The onset of the event was not witnessed. Examination reveals left hemiparesis and dys- arthria. A CT head scan without contrast is ordered. Which of the following would be a contraindication for thrombolytic therapy? A. Hyperdense right middle cerebral artery sign B. Loss of gray–white differentiation in right insula C. Loss of gray–white differentiation in right temporal cortex D. Sulcal effacement in right hemisphere E. Delineated hypodensity in right middle cerebral artery territory

165

66485457-66485438 www.ketabpezeshki.com

ZZakaria_87574_PTR_CH12_10-06-13_159-174.inddakaria_87574_PTR_CH12_10-06-13_159-174.indd 165165 66/19/2013/19/2013 8:46:318:46:31 PMPM DISORDERS, DISEASES, SEIZURES, AND EPILEPSY

30. An 80-year-old man underwent a prolonged abdominal aortic aneurysm repair. After anesthesia wears off, he notices diffi culty moving his legs. Neurology service is consulted and documents a fl accid paraplegia with a sensory level at T10. Upon follow-up with the same neurologist 3 months later, his examination reveals lower extremity weakness, bilateral ankle clonus, absent knee jerks, and plantar toe extension. Which of the follow- ing examination fi ndings is also to be expected? A. Fasciculations B. Abnormal tibial sensory-evoked potentials C. Decreased amplitudes in gastrocnemius D. Fibrillations in sural nerve E. Bladder detrusor–sphincter dysfunction

166

66485457-66485438 www.ketabpezeshki.com

ZZakaria_87574_PTR_CH12_10-06-13_159-174.inddakaria_87574_PTR_CH12_10-06-13_159-174.indd 166166 66/19/2013/19/2013 8:46:318:46:31 PMPM 12

ANSWERS

1. The answer is D. Cerebral herniation refers to displacement of brain tissue into nearby com- partments from local intracranial pressure (ICP) gradients (see accompanying illustration). Clinical fi ndings of cerebellar/tonsillar herniation include episodic extensor posturing and cardiac dysrhythmias from downward displacement of cerebellar tonsils through the foramen magnum with compression of the medulla. Uncal herniation classically presents with ipsilateral pupillary dilation secondary to compression of cranial nerve (CN) III from herniation of the medial temporal lobe under the tentorium cerebelli, thereby displacing the midbrain. Central herniation can present with bilateral pupillary dilation and extensor posturing with obliteration of the suprasellar cistern on imaging as both medial temporal lobes herniate through the tentorial notch. Similarly, upward herniation of posterior fossa contents through the tentorial notch presents with bilateral pupillary dilation and extensor posturing, typically as a result of excessive cerebrospinal fl uid (CSF) ventricular drainage.

Herniation Syndromes. (1) Uncal herniation can result in third cranial nerve, posterior cerebral artery and midbrain compression; (2) central herniation can cause downward displacement of the entire brainstem with lateral gaze palsy; (3) subfalcine herniation can result in stran- gulation of the anterior cerebral artery under the falx; (4) extracranial herniation can occur through a traumatic skull defect or therapeutic craniectomy; (5) upward tentorial herniation can occur due to posterior fossa masses; (6) tonsillar herniation can result in brainstem com- pression, pupillary dilation, and cardio respiratory arrest. Used with permission from: Zakaria, A., Aisiku, I. Management of acute intracra- nial hypertension. In: Farey, D., Flaxman, A., Chiu, W., eds. Critical Care Emergency Medicine. New York, NY: McGraw-Hill.

167

66485457-66485438 www.ketabpezeshki.com

ZZakaria_87574_PTR_CH12_10-06-13_159-174.inddakaria_87574_PTR_CH12_10-06-13_159-174.indd 167167 66/19/2013/19/2013 8:46:318:46:31 PMPM DISORDERS, DISEASES, SEIZURES, AND EPILEPSY

2. The answer is E. ICP waveforms originate from transient increases in pressure from transmission of arterial pulses to the brain. P1 (percussion wave) is produced from local increases in ICP from arterial pulse pressure waves to the choroid plexus. P2 (elastance or tidal wave) results from a restriction of ventricular expansion by a closed rigid skull. P3 (dicrotic wave) is produced by closure of the aortic valve associated with the arterial dicrotic notch. Under normal circumstances, P1 > P2 > P3. Elevation of P2 > P1 is concern- ing for disturbed intracranial elastance and loss of nonlinear dynamics such that small increases in intracranial volume may dramatically increase ICP (1).

3. The answer is E. A unilateral hemispheric lesion rarely causes a disturbance in conscious- ness. Exceptions include a large dominant hemispheric stroke or a small lesion in the brainstem or thalamus, either of which can cause signifi cant alteration of consciousness. Otherwise, classically a disturbance in consciousness must involve a process that affects the reticular activating system (RAS) or a diffuse process that affects the cortex bilaterally, for example, metabolic encephalopathy, paraneoplastic disorders, head trauma, and pos- terior reversible encephalopathy syndrome.

4. The answer is B. ICH of any etiology is likely to be associated with headache and a focal neurologic defi cit. Oral anticoagulant–associated ICH is more likely to expand over a lon- ger period (up to 7 days), have a fl uid level and multiple discrete areas of hemorrhage compared to spontaneous intracerebral hemorrhage. Continued elevation of BP is consid- ered a risk factor for hematoma expansion (2).

5. The answer is F. An external ventricular drain is the gold standard for ICP monitor- ing. It consists of a fl uid-coupled transducer placed in the lateral ventricle, with the tip located in the foramen of Monro. Placement allows for continuous ICP monitor- ing and intermittent drainage of cerebrospinal fl uid (CSF) for ICP control. Potential complications include catheter tract hemorrhage, infection, or injury to critical tissue. The catheter should be calibrated or zeroed at the level of the foramen of Monro (exter- nal tragus of the ear). Contraindications include collapsed ventricles, coagulopathy, or infection (3).

6. The answer is E. Vasogenic edema is defi ned as excess fl uid within the interstitial space and can be readily identifi ed by the preservation of gray–white junction on neuroimag- ing. Common causes include malignant brain tumors, abscesses, meningitis, and contu- sions, with the mechanism believed to be disruption of blood–brain barrier perivascular tight junctions leading to movement of water from the vascular space to the interstitium. Dexamethasone will rapidly decrease vasogenic edema, but will have minimal or no effect on cytotoxic edema. Osmolar therapy with mannitol or hypertonic saline has never been shown to alter outcomes for vasogenic edema (4).

7. The answer is C. Decreased carbon dioxide tension is a potent vasoconstricting trigger

to the cerebral arteries. A decrease in CO2 tension by 10 mmHg can produce suffi cient reduction in cerebral blood volume (CBV) to signifi cantly reduce ICP. Hyperventilation reduces ICP by refl ex cerebral vasoconstriction secondary to cerebrospinal fl uid (CSF) alkalosis. Unfortunately, it may produce suffi cient decrease in CBV to induce ischemia.

168

66485457-66485438 www.ketabpezeshki.com

ZZakaria_87574_PTR_CH12_10-06-13_159-174.inddakaria_87574_PTR_CH12_10-06-13_159-174.indd 168168 66/19/2013/19/2013 8:46:328:46:32 PMPM CLINICAL SYNDROMES: Answers

A PaCO2 less than 28 mmHg may provoke or worsen cerebral ischemia. In patients with

ICP crisis, a target PaCO2 of 28 to 32 mmHg can be utilized with the effect of hyperven- tilation starting within 10 minutes and lasting as long as 4 to 8 hours. Hyperventilation should generally not be used prophylactically, and most patients should be maintained

at a target PaCO2 of 35 mmHg (5,6).

8. The answer is C. Prolonged hyperventilation (>4 hours) can lead to rebound intracra- nial hypertension due to cerebrospinal fl uid (CSF) buffering. Hypocarbia, once induced, should be slowly weaned over 6 to 24 hours to minimize the rebound hyperemia of ree- quilibration. Maintenance of deliberate respiratory alkalosis for a sustained time has been shown to worsen outcomes in head injury patients (7,8).

9. The answer is E. First described by Lundberg, plateau waves, also known as Lundberg A waves, are considered essentially pathognomonic of intracranial hypertension. A waves are 5- to 20-minutes-long, high-amplitude (50–100 mmHg) increases in ICP, and are indicative of decreased intracranial compliance. The danger of plateau waves is the abolishment of cerebral perfusion pressure; waves of suffi cient severity and duration to produce global cerebral ischemia must be reversed. B and C waves are of less clinical signifi cance. B waves last 1 to 2 minutes, are of 20 to 50 mmHg in ampli- tude, and do not necessarily represent any pathological disturbance. C waves last 4 to 5 minutes, are less than 20 mmHg in amplitude, and have no pathological conse- quence (9).

10. The answer is A. Acute hydrocephalus can be seen on neuroimaging, in particular on CT scan, with dilated cerebral ventricles, bowing of the third ventricle if under pressure, and a dilated fourth ventricle (communicating hydrocephalus) or nondilated fourth ven- tricle suggesting noncommunicating hydrocephalus. MRI fi ndings show increased T2 intensity in the periventricular area, revealing transependymal cerebrospinal fl uid (CSF) fl ow. Compression of the quadrigeminal plate can result in Parinaud’s syndrome (vertical gaze palsy). Hydrocephalus is also associated with abducens nerve defi cits, which can be a false localizing sign. Any process (brain tumor, hydrocephalus, pseudotumor cerebri, edema) that exerts downward pressure on the brainstem can stretch the nerve along the clivus, causing indirect damage to the sixth nerve.

11. The answer is E. Substantive dysfunction of one of three areas of the brain can be suf- fi cient to cause coma: the reticular activating system (RAS), bilateral frontal lobes, and bilateral temporal lobes (see CT scans on following page). Injury to the basal or ventral pons might cause a locked-in syndrome, but would spare the RAS. Bilateral injury to the parietal and occipital lobes is much less likely to cause severe impairment of conscious- ness compared to the temporal and frontal lobes.

169

66485457-66485438 www.ketabpezeshki.com

ZZakaria_87574_PTR_CH12_10-06-13_159-174.inddakaria_87574_PTR_CH12_10-06-13_159-174.indd 169169 66/19/2013/19/2013 8:46:328:46:32 PMPM DISORDERS, DISEASES, SEIZURES, AND EPILEPSY

Bilateral thalamic infarctions seen after embolism to the artery of Percheron can affect the reticular activating system (RAS) and result in a comatose state.

12. The answer is C. Although it has been documented that fever is associated with poorer outcomes in patients with cerebral infarction, spinal cord injury, subarachnoid hemor- rhage, and TBI, no studies have determined the utility of therapeutic hypothermia in these conditions. Several studies have shown a clear benefi t from therapeutic hypothermia (TH) (32°C–34°C) in the fi rst 24 hours after cardiac arrest. The neuroprotective effects of TH were most pronounced when treatment was started early, and conversely the benefi ts waned as treatment was delayed. The role of hypothermia in head injury is less clear. The recent completion of two large multicenter trials, NABISH I and II, appears to confi rm smaller studies that induced hypothermia after severe head injury does not improve long- term outcome (10).

13. The answer is A. A major challenge during the induction phase is the occurrence of shiv- ering, which produces heat, increases oxygen consumption, and prevents/slows cooling to target temperature. Combination of meperidine/buspirone, dexmedetomidine, and paralytics may control shivering. One of the most effective pharmacologic therapies to combat shivering is meperidine, which in combination with buspirone has been shown to lower the shivering threshold to 33.4°C—two degrees below normal and signifi cantly lower than either drug alone (11). It has specifi c antishivering properties resulting from κ-receptor activity. Meperidine should be used with caution, however, as it may depress the sensorium and complicate patient monitoring. Meperidine may also lower the sei- zure threshold, particularly in patients with renal dysfunction. Although the optimal rewarming rate is unknown, current consensus suggests a rewarming rate of 0.25°C to 0.5°C/hour; rapid rewarming should be avoided because of a higher incidence of electro- lyte abnormalities and cardiac arrhythmias.

14. The answer is E. Several potential complications of hypothermia must be guarded against if any therapeutic utility is to be realized. A rebound increase in ICP may occur with rewarming after hypothermia and can result in herniation. Risk of this complication may

170

66485457-66485438 www.ketabpezeshki.com

ZZakaria_87574_PTR_CH12_10-06-13_159-174.inddakaria_87574_PTR_CH12_10-06-13_159-174.indd 170170 66/19/2013/19/2013 8:46:328:46:32 PMPM CLINICAL SYNDROMES: Answers

be reduced by slower-controlled warming. The other options are risks of hypothermia itself and are more commonly seen at deeper levels of hypothermia.

15. The answer is B. Clinical and radiologic predictors of fatal brain edema as a com- plication of ischemic stroke include high NIH stroke scale score, early nausea and vomiting, 12-hour systolic BP greater than or equal to 180 mmHg, early hypodensity of greater than 50% of the middle cerebral artery territory on CT scan, diffusion lesion volume greater than 80 mL within 6 hours of stroke onset, involvement of additional vascular territories, elevated white blood cell count, and a history of hypertension or heart failure.

16. The answer is E. A study group of the Quality Standards Subcommittee of the American Academy of Neurology reviewed the literature and reported in 2006 that tests such as CT and MRI scans, EEG, and duration of cardiopulmonary resuscitation had insuffi cient evi- dence for predictive discrimination of outcome after cardiac arrest. Two tests were found to be predictive of outcome: a blood test for neuron-specifi c enolase and the median nerve somatosensory evoked potentials (SSEP) study (12).

17. The answer is B. There is evidence from several randomized clinical trials that induced hypothermia with a target temperature of 32°C to 34°C for 24 hours improves neurologic outcome in patients who are unconscious after successful resuscitation from ventricu- lar fi brillation–associated cardiac arrest. Profound hypothermia has been shown to cause coagulopathy, suppression of cortical function, hypotension, cardiac dysrhythmia, and immunosuppression. The risk of these adverse effects is lower in therapeutic temperature ranges. Autoimmunity and renal and hepatic dysfunction are not associated with hypo- thermia (13).

18. The answer is C. This patient’s clinical deterioration is likely caused by progressive hydrocephalus from obstruction of the . The next step in management would be the placement of an external ventricular drain. Osmotic therapy would not pro- duce a meaningful or lasting reduction in intracranial pressure (ICP). Unless the patient is febrile, lowering his body temperature is unlikely to improve clinical outcome. The patient’s mean arterial pressure is normal, and further reduction is unlikely to be benefi - cial (14).

19. The answer is B. NINDS guidelines for management of BP after treatment of acute ischemic stroke with recombinant tissue plasminogen activator (rt-PA) recommend monitoring BP every 15 minutes during and for 2 hours after treatment, then every 30 minutes for 6 hours, and then every hour for 18 hours. Strict parameters are rec- ommended for BP prior to administration of rt-PA. If systolic BP is 180 to 230 mmHg and/or diastolic BP is 105 to 120 mmHg on two readings 5 to 10 minutes apart, give labetalol 10 mg IV over 1 to 2 minutes. Monitor BP every 15 minutes during the anti- hypertensive therapy.

20. The answer is A. Dominant theta–delta activity without detectable normal alpha activity offers the possibility of full recovery. In one review (Scollo-Lavizzari and Bassetti, 1987), 27% of patients with this EEG pattern experienced a full neurological recovery after resus- citation from cardiac arrest (15).

171

66485457-66485438 www.ketabpezeshki.com

ZZakaria_87574_PTR_CH12_10-06-13_159-174.inddakaria_87574_PTR_CH12_10-06-13_159-174.indd 171171 66/19/2013/19/2013 8:46:338:46:33 PMPM DISORDERS, DISEASES, SEIZURES, AND EPILEPSY

21. The answer is E. A variety of antihypertensives can be safely used to lower the BP in patients with intracranial hemorrhage (ICH). Nitroprusside is a nitric oxide donor, a potent vasodilator, and an inhibitor of circulating platelets, thereby reducing both preload and afterload. It is not often the antihypertensive of choice because of its potent venodila- tory effects and risk of intracranial pressure (ICP) elevation. Alternative agents include β-blockers, nicardipine, and fast-acting angiotensin-converting enzyme inhibitors.

22. The answer is E. TM is a heterogeneous focal infl ammatory disorder of the spinal cord characterized by acute or subacute development of motor weakness, sensory impair- ment, and autonomic dysfunction. Infl ammation of the spinal cord (e.g., CSF pleocytosis, elevated IgG index, or an enhancing spinal cord lesion on MRI) is highly suggestive of TM. Most patients with idiopathic TM have some degree of weakness, either symmet- ric or asymmetric. Asymmetric cases are likely to be diagnosed with multiple sclerosis. Neurological function usually worsens progressively over the course of 4 to 21 days, with the vast majority of cases reaching their clinical nadir within 2 weeks. Progression to nadir within 4 hours of symptom onset is one of the exclusion criteria for idiopathic or postinfectious TM (16).

23. The answer is C. Contrast extravasations seen on initial CT imaging are associated with the presence of hematoma progression. In the setting of anticoagulation use, the risk of intracranial hemorrhage (ICH) increases by 5- to 10-fold and carries a poor prognosis. High-dose IV vitamin K can fully reverse warfarin-induced anticoagulation, but may take up to 12 to 24 hours to take effect. FFP is well accepted in the effort to reverse the coagul- opathy. Factor VII has been shown to reverse vitamin K coagulopathy more rapidly than FFP. However, it should be used in conjunction with FFP and vitamin K, as factor VII cor- rects only the warfarin-induced defi cit of factor VII while FFP and vitamin K correct the defi cits in all vitamin K–dependent coagulation factors (17,18).

24. The answer is D. Coma implies a profound disturbance in consciousness that affects both the reticular activating system (RAS) and the cerebral hemispheres. Patients in a coma have a disturbance in their sleep–wake cycle and exhibit no meaningful interaction with the environment. Minimally conscious state refers to impairment of consciousness but with discernible evidence of some level of awareness to self or envi- ronment. The hallmark of the vegetative state is that no evidence of interaction with the physical environment is present, but the patient has regained sleep–wake cycles mediated through the RAS. Behaviors commonly seen in the vegetative states include roving eye movements, purposeless smiles, and occasional verbalizations. The vegeta- tive state is deemed persistent when it exceeds 1 month in duration. There is potential for recovery from the persistent vegetative state, especially in cases of trauma. After 12 months posttraumatic injury and 3-month post-nontraumatic injury, the chance of recovery is exceedingly low (permanent vegetative state) (19).

25. The answer is A. The NINDS recommends administration of IV rt-PA at 60 minutes from arrival to hospital to patients who are eligible according to current American Heart Association criteria. The patient has not been taking anticoagulants or received heparin in the previous 48 hours, has no clinical or CT evidence of bleeding, and has no previous conditions that may predispose to bleeding. Therefore, rt-PA can be administered even

172

66485457-66485438 www.ketabpezeshki.com

ZZakaria_87574_PTR_CH12_10-06-13_159-174.inddakaria_87574_PTR_CH12_10-06-13_159-174.indd 172172 66/19/2013/19/2013 8:46:338:46:33 PMPM CLINICAL SYNDROMES: Answers

before results of the prothrombin time and activated partial thromboplastin times are available. Although the patient or family members should understand the potential risks and benefi ts of IV rt-PA treatment, informed consent is not required since rt-PA is consid- ered the standard of care.

26. The answer is D. Myoclonus is the most common type of convulsive activity after car- diac arrest and usually suggests a poor prognosis. It is not clear whether this represents true seizure activity or random fi ring of uninhibited cortical neurons. Burst suppression is the most common EEG pattern associated with generalized myoclonus after cardiac arrest.

27. The answer is G. The Multi-Society Task Force on PVS (persistent vegetative state) con- cluded that a patient diagnosed with a vegetative state 1 year after a traumatic brain injury (TBI) or 3 months after anoxic brain injury is very unlikely to improve. All of the options are indicators of poor outcome after cardiopulmonary resuscitation (19,20).

28. The answer is A. Lap belt injuries are associated with thoracolumbar flexion dis- traction injuries. Multiple-level injuries occur in as high as 20% of cases. The sympa- thetic nervous system exits the spinal cord from the thoracolumbar segment. Injury to the thoracolumbar segment may cause loss of supraspinal regulatory control and reduced sympathetic activity. The sympathetic nervous system is compromised, and the parasympathetic nervous system dominates, causing massive vasodilation or neurogenic shock. This risk of hypotension and bradycardia is greatest soon after injury (21).

29. The answer is E. The presence of a region of clearly delineated hypodensity with asso- ciated mass effect is indicative of established infarction and inconsistent with an acute ischemic insult within 3 hours as established by NINDS criteria.

30. The answer is E. The spinal cord is perfused by two posterior spinal arteries, which sup- ply the posterior one-third of the cord, and the single anterior spinal artery, which supplies the anterior two-thirds of the cord. The artery of Adamkiewicz typically arises between T8 and L4 and is a signifi cant anterior spinal artery feeder. The middle and lower thoracic cord and central gray matter of the cord are particularly vulnerable to watershed injuries. A sensory level at T10 and his abnormal refl exes suggest a lesion extending to L4 but sparing the lower segments. This can produce a spastic bladder with detrusor–sphincter dyssynergy.

References

1. Lee KR, Hoff JT. Intracranial pressure. In: Youmans JR, ed. Neurological Surgery. Vol 1. Philadelphia, PA: Saunders; 1996:491–518. 2. Kazui S, Minematsu K, Yamamoto H, Sawada T, Yamaguchi T. Predisposing factors to enlargement of spontaneous intracerebral hematoma. Stroke. 1997;28(12):2370–2375. 3. Lang EW, Chesnut RM. Intracranial pressure: monitoring and management. Neurosurg Clin North Am. 1994;5:573–605.

173

66485457-66485438 www.ketabpezeshki.com

ZZakaria_87574_PTR_CH12_10-06-13_159-174.inddakaria_87574_PTR_CH12_10-06-13_159-174.indd 173173 66/19/2013/19/2013 8:46:338:46:33 PMPM DISORDERS, DISEASES, SEIZURES, AND EPILEPSY

4. Nath F, Galbraith S. The effect of mannitol on cerebral white matter water content. J Neurosurg. 1986;65(1):41–43. 5. Stringer WA, Hasso AN, Thompson JR, Hinshaw DB, Jordan KG. Hyperventilation- induced cerebral ischemia in patients with acute brain lesions: demonstration by xenon- enhanced CT. AJNR Am J Neuroradiol. 1993;14(2):475–484. 6. Lang EW, Chesnut RM. Intracranial pressure. Monitoring and management. Neurosurg Clin N Am. 1994;5(4):573–605. 7. Muizelaar JP, Marmarou A, Ward JD, et al. Adverse effects of prolonged hyperven- tilation in patients with severe head injury: a randomized clinical trial. J Neurosurg. 1991;75(5):731–739. 8. Frank JL. Management of intracranial hypertension. Med Clin North Am. 1993;77:61–76. 9. Lundberg N. Continuous recording and control of ventricular fl uid pressure in neurosur- gical practice. Acta Psychiatr Scand Suppl. 1960;36(149):1–193. 10. Hypothermia After Cardiac Arrest Study Group. Mild therapeutic hypothermia to improve the neurological outcome after cardiac arrest. N Eng J Med. 2002;346:557–563. 11. Mokhtarani M, Mahgoub AN, Morioka N, et al. Buspirone and meperidine synergisti- cally reduce the shivering threshold. Anesth Analg. 2001;93(5):1233–1239. 12. Wijdicks EF, Hijdra A, Young GB, Bassetti CL, Wiebe S. Practice parameter: prediction of outcome in comatose survivors after cardiopulmonary resuscitation (an evidence-based review): report of the Quality Standards Subcommittee of the American Academy of Neurology. Neurology. 2006;67(2):203–210. 13. Rohrer MJ, Natale AM. Effect of hypothermia on the coagulation cascade. Crit Care Med. 1992;20(10):1402–1405. 14. Frank JI. Management of intracranial hypertension. Med Clin North Am. 1993;77(1): 61–76. 15. Scollo-Lavizzari G, Bassetti C. Prognostic value of EEG in post-anoxic coma after cardiac arrest. Eur Neurol. 1987;26(3):161–170. 16. Brinar VV, Habek M, Brinar M, Malojcic B, Boban M. The differential diagnosis of acute transverse myelitis. Clin Neurol Neurosurg. 2006;108(3):278–283. 17. Wintzen AR, de Jonge H, Loeliger EA, Bots GT. The risk of intracerebral hemorrhage dur- ing oral anticoagulant treatment: a population study. Ann Neurol. 1984;16(5):553–558. 18. Fredriksson K, Norrving B, Strömblad LG. Emergency reversal of anticoagulation after intracerebral hemorrhage. Stroke. 1992;23(7):972–977. 19. Multi-Society Task Force on PVS. Medical aspects of the persistent vegetative state. N Engl J Med. 1994;330:1572–1578. 20. Levy DE, Caronna JJ, Singer BH, Lapinski RH, Frydman H, Plum F. Predicting outcome from hypoxic-ischemic coma. JAMA. 1985;253(10):1420–1426. 21. Teasell RW, Arnold JM, Krassioukov A, Delaney GA. Cardiovascular consequences of loss of supraspinal control of the sympathetic nervous system after spinal cord injury. Arch Phys Med Rehabil. 2000;81(4):506–516.

174

66485457-66485438 www.ketabpezeshki.com

ZZakaria_87574_PTR_CH12_10-06-13_159-174.inddakaria_87574_PTR_CH12_10-06-13_159-174.indd 174174 66/19/2013/19/2013 8:46:338:46:33 PMPM 13 Perioperative Neurosurgical Care Yoshua Esquenazi and Nitin Tandon QUESTIONS

1. A 56-year-old woman with a medical history of diabetes presents to the ED after the acute onset of headache, nausea, and vomiting. On physical examination, she is somnolent, opens her eyes to commands, and is oriented only to herself. The accompanying CT scan of the head (shown at the top of the following page) is obtained, and an emergent exter- nal ventriculostomy is placed. The patient is then taken immediately to the endovascular suite, and a cerebral angiogram shows a small right posterior communicating aneurysm. The patient undergoes a successful balloon-assisted coil embolization and is transferred to the ICU. Which of the following measures are recommendations for the prevention of delayed cerebral ischemia (DCI) secondary to vasospasm? A. hypervolemic hemodynamic augmentation B. euvolemia C. hypothermia D. prophylactic balloon angioplasty on postbleed day 7 E. prophylactic intra-arterial vasodilator therapy on postbleed day 7

ANSWERS TO THIS SECTION CAN BE FOUND ON PAGE 179 175

66485457-66485438 www.ketabpezeshki.com

ZZakaria_87574_PTR_CH13_10-06-13_175-182.inddakaria_87574_PTR_CH13_10-06-13_175-182.indd 175175 66/19/2013/19/2013 4:45:264:45:26 PMPM DISORDERS, DISEASES, SEIZURES, AND EPILEPSY

2. A 72-year-old woman with a medical history of atrial fi brillation and congestive heart failure was admitted to the hospital for an elective three-level anterior cervical discectomy and fusion (ACDF) due to progressive spondylotic cervical myelopathy. She had stopped her warfarin 5 days before surgical intervention and had normal coagulation studies before the procedure. The intraoperative course was uncomplicated, and the patient was transferred from the operating room to the recovery area, and subsequently to the ICU for observation. Seven hours after the procedure, you receive a call from the nurse telling you that for the past 1 hour, the patient has been complaining of neck discomfort and has recently developed stridor. On physical examination, the patient has signifi cant swelling in the anterior aspect of the neck, has stridor, and is marginally hypoxic. What is the next step in the management of this patient? A. Intravenous dexamethasone and diphenhydramine B. Chest x-ray, intravenous furosemide, and non-rebreather oxygen face mask C. Electrocardiogram, CT scan of the chest, and intravenous heparin D. Emergent bedside surgical intervention E. Reassurance and high-dose nonsteroidal anti-infl ammatory agents

3. Which of the following is the most common complication from an anterior cervical discec- tomy and fusion (ACDF)? A. Postoperative hematoma B. Wound infection C. Recurrent laryngeal nerve palsy D. Esophageal perforation E. Dysphagia

176

66485457-66485438 www.ketabpezeshki.com

ZZakaria_87574_PTR_CH13_10-06-13_175-182.inddakaria_87574_PTR_CH13_10-06-13_175-182.indd 176176 66/19/2013/19/2013 4:45:264:45:26 PMPM PERIOPERATIVE NEUROSURGICAL CARE: Questions

4. A 55-year-old right-handed man with a medical history of atrial fi brillation is brought to the ED after the acute onset of headache, vomiting, and left hemiparesis. CT scan of the head shows evidence of a 30-cc3 intracerebral hemorrhage in the right basal ganglia, with extension into the ventricular system and mild hydrocephalus. On physical exami- nation, he is somnolent but arousable to pain. Further history reveals that 2 months ago, the patient was switched from warfarin to dabigatran etexilate (Pradaxa) for stroke pre- vention because of his atrial fi brillation. The following laboratory values were obtained: INR = 1.32; PT = 1.13; PTT = 90.2. Which of the following is the best next step to address this patient coagulopathy? A. Vitamin K 10 mg administered intravenously B. Fresh-frozen plasma—15 mL/kg administered stat C. Platelet transfusion—10 mL/kg D. rFVIIa 80 mcg/kg E. Emergent hemodialysis

5. All of the following are risk factors for the development of an infection related to external ventricular drainage (EVD) except: A. Presence of intraventricular hemorrhage B. Duration of the EVD placement beyond 4 days C. Routine daily cerebrospinal fl uid (CSF) sampling D. Manipulation of the catheter to relieve clogging or improve fl ow E. Steroid use

6. A 31-year-old right-handed man with a previous gunshot wound to the head 2 years ago and medically refractory seizures since then was admitted electively to the hospital as part of a Phase II epilepsy surgery evaluation for left-sided platinum–iridium subdural grid placement and video EEG monitoring. The intraoperative course was uncomplicated, and the patient was transferred from the operating room to the recovery area, and subse- quently to the ICU, where he was doing well. Seven hours after the procedure, the patient complains of worsening headaches, vomiting, and weakness in his right arm. On physical examination, the patient has a drift in his right upper extremity but is able to hold his arm up against gravity. Which of the following is the most appropriate next step in the manage- ment of this patient? A. Check antiepileptic levels B. Mannitol administration C. MRI of the brain without contrast D. CT scan of the head without contrast E. Thromboelastogram

177

66485457-66485438 www.ketabpezeshki.com

ZZakaria_87574_PTR_CH13_10-06-13_175-182.inddakaria_87574_PTR_CH13_10-06-13_175-182.indd 177177 66/19/2013/19/2013 4:45:264:45:26 PMPM DISORDERS, DISEASES, SEIZURES, AND EPILEPSY

7. A 68-year-old woman underwent an elective clipping of a left 9-mm unruptured posterior communicat- ing artery aneurysm. An intraoperative angiogram showed evidence of complete aneurysm obliteration. The intraoperative course was uncomplicated, and the patient was extubated in the operating room suc- cessfully. She was then transferred to the neurosur- gical ICU for observation. Ten hours after the pro- cedure, the patient started complaining of nausea, vomiting, and headaches. On physical examination, the patient was confused as to place and time, but no focal fi ndings were found. Baseline laboratory and coagulation studies were normal, and her blood pressure at the time of the examination was 160/90 mmHg. The accompanying CT scan of the head was obtained. What is the most likely explanation for her condition? A. Aneurysm rupture due to clip misplacement B. Hypertensive cerebellar hemorrhage C. Remote cerebellar hemorrhage (RCH) D. Normal postoperative CT scan E. Contrast extravasation from diagnostic angiography

8. A 72-year-old man with a greater than 90% symptomatic right carotid artery stenosis was admitted to the hospital for an elective carotid endarterectomy. Which of the following is the correct management of this patient in the immediate postoperative period? A. Head of bed 30 to 45 degrees, NPO, continue antiplatelet therapy, systolic blood pres- sure (SBP) 100 to 150 mmHg B. Head of bed 30 to 45 degrees, regular diet, stop antiplatelet therapy, SBP 160 to 180 mmHg C. Head of bed 30 to 45 degrees, activity as tolerated, intravenous heparin drip, SBP < 100 D. Head of bed 15 to 30 degrees, NPO, stop antiplatelet therapy, SBP < 100 E. Head of bed 30 to 45 degrees, regular diet, warfarin administration, SBP 100 to 150

9. A 69-year-old woman with a medical history of hypertension and diabetes was found to have a 9-mm basilar aneurysm as part of the workup for new onset of vertigo. The patient was admitted to the hospital electively and underwent a balloon-assisted coil emboliza- tion of her cerebral aneurysm. Which of the following would decrease the patient’s risk of developing the most common complication of this procedure? A. Keep SBP < 140 mmHg in the immediate postoperative period B. Intravenous heparin during the immediate postoperative course C. Oral antiplatelet agents in the perioperative course D. Coumadin administration in the perioperative course E. Keep SBP > 140 mmHg in the immediate postoperative period

178

66485457-66485438 www.ketabpezeshki.com

ZZakaria_87574_PTR_CH13_10-06-13_175-182.inddakaria_87574_PTR_CH13_10-06-13_175-182.indd 178178 66/19/2013/19/2013 4:45:264:45:26 PMPM 13

ANSWERS

1. The answer is B. Vasospasm of major cerebral arteries after aneurysmal subarachnoid hemorrhage is common; it occurs most frequently 7 to 10 days after aneurysm rup- ture and resolves spontaneously after 21 days. DCI, associated with arterial vasospasm, remains a major cause of death and disability in patients with this condition. There is class I level evidence that oral nimodipine should be administered to all patients with aneurysmal subarachnoid hemorrhage, and this agent has been shown to improve neurological outcomes, but does not prevent cerebral vasospasm. Maintenance of euv- olemia and normal circulating blood volume is recommended to prevent DCI, but when this condition is diagnosed, the initial treatment is the induction of hemodynamic aug- mentation to improve cerebral perfusion. No randomized trials have been performed, but the quick improvement of patients with this therapy and their worsening when it is stopped prematurely are convincing proof of effi cacy. Prophylactic hypervolemia or bal- loon angioplasty before the development of angiographic spasm is not recommended. Induction of hypertension is recommended for patients with DCI unless blood pres- sure is elevated at baseline or cardiac status precludes it. Cerebral angioplasty and/or selective intra-arterial vasodilator therapy is reasonable in patients with symptomatic cerebral vasospasm, particularly those who are not rapidly responding to hypertensive therapy (1).

2 and 3. The answers are D and E, respectively. ACDF represents one of the most commonly performed spinal procedures for the management of patients with cervical radiculopa- thy and/or myelopathy secondary to degenerative disk disease and/or spondylosis. The clinical outcome of this procedure is good or excellent in the majority of cases. However, on rare occasions, a complication may occur, which can be catastrophic. Early identifi ca- tion and prompt management of these potential complications are imperative. The most

179

66485457-66485438 www.ketabpezeshki.com

ZZakaria_87574_PTR_CH13_10-06-13_175-182.inddakaria_87574_PTR_CH13_10-06-13_175-182.indd 179179 66/19/2013/19/2013 4:45:264:45:26 PMPM DISORDERS, DISEASES, SEIZURES, AND EPILEPSY

commonly reported complications include postoperative dysphagia (9.5%), postopera- tive hematoma (5.6%), and symptomatic recurrent laryngeal nerve palsy (3.1%); other less common complications (<1%) include cerebrospinal fl uid (CSF) leak, esophageal perforation, Horner syndrome, and superfi cial wound infections. The patient described in this vignette has the signs and symptoms of an enlarging postoperative hematoma. Postoperative hematomas from ACDF have been reported to occur in up to 5% of patients; the management of this condition can be conservative, but when there is evidence of respiratory compromise or hematoma expansion, immediate emergent surgical reinter- vention should be done, including reopening the wound at bedside to decompress the upper airway, prior to efforts at endotracheal intubation (2).

4. The answer is E. New direct coagulation factor inhibitors, such as dabigatran, have dem- onstrated superior stroke and systemic embolism prevention without the burdensome monitoring and drug–drug interactions seen with agents such as warfarin. Although dab- igatran has been shown to have a lower incidence of major bleeding than warfarin, in the event of catastrophic hemorrhage, no effective reversal agent exists, and these events may prove to be fatal. Dabigatran etexilate (Pradaxa) is an oral anticoagulant that acts as a direct, competitive thrombin inhibitor. In 2010, the U.S. FDA approved the use of dabiga- tran for the prevention of stroke and systemic embolism in patients with atrial fi brillation. Patients who are taking dabigatran do not require routine monitoring with prothrombin times or international normalized ratios. There is currently no effective antidote to reverse the anticoagulant effect of dabigatran in the event of a hemorrhagic emergency. As a direct thrombin (factor II) inhibitor, dabigatran exerts its mechanism of action at the very end of the coagulation cascade, and therefore neither factor VIIa nor fresh-frozen plasma is effective as a treatment option. Because dabigatran is primarily excreted through the renal system, dialysis is an alternative for drug clearance and can remove approximately 35% to 60% of the drug in 2 to 3 hours. Dabigatran is different from warfarin, whose inhibitory effect on clotting factors II, VII, IX, and X can be reversed by fresh-frozen plasma, vitamin K, and factor VII (3).

5. The answer is E. Hydrocephalus is a common problem encountered in neurosurgi- cal patients. EVD is frequently used in the acute treatment of these patients. The major complication of this procedure is an EVD-related infection (meningitis or ventriculitis). Manipulation, or daily sampling from the EVD, is associated with increased infection risks. Infection may lead to removal and reinsertion of a new EVD, prolonged hospi- tal stay, treatment with antibiotics, and associated cost and morbidity. There is a well- established relationship between the duration of EVD and the occurrence of EVD-related infections that start at about 4 days after EVD insertion (4).

6. The answer is C. The occurrence of a delayed and progressive neurological defi cit sug- gests the presence of a postoperative hematoma, which can occur especially in patients with multiple prior cranial procedures. CT scanning is signifi cantly degraded by beam- hardening artifacts and is of little value in predicting the development of symptoms and possible postoperative complications after subdural grid implantation. If the electrodes are MRI compatible, an emergent MRI is the most useful adjunct to the diagnosis; clinical

180

66485457-66485438 www.ketabpezeshki.com

ZZakaria_87574_PTR_CH13_10-06-13_175-182.inddakaria_87574_PTR_CH13_10-06-13_175-182.indd 180180 66/19/2013/19/2013 4:45:264:45:26 PMPM PERIOPERATIVE NEUROSURGICAL CARE: Answers

judgment must guide management and determine the potential need for reexploration. This complication may occur even in the presence of normal coagulation parameters and should be anticipated and managed urgently (5).

7. The answer is C. Intracerebral hemorrhage remote from the site of surgery is an infrequent complication after neurosurgical procedures. Cerebellar hemorrhage after supratentorial craniotomy is the most commonly described pattern of hemorrhage, with an incidence of 0.3% to 0.6%, and it may cause signifi cant neurological morbidity and mortality. RCH most commonly follows supratentorial neurosurgical procedures—performed with the patients in the supine position—that involve opening of cerebrospinal fl uid (CSF) cis- terns or the ventricular system (such as unruptured aneurysm repair or temporal lobec- tomy). The exact etiology remains unclear, but perioperative hypertension, coagulopathy, preoperative aspirin use, head positioning during surgery, CSF overdrainage, and post- operative epidural drainage are recognized risk factors. Although RCH can cause death or major morbidity, most cases are asymptomatic or exhibit a benign course. Cerebellar “sag” as a result of CSF hypovolemia, causing transient occlusion of superior bridging veins within the posterior fossa and consequent hemorrhagic venous infarction, is the most likely pathophysiological cause. There is no evidence of subarachnoid hemorrhage, which may suggest aneurysm rupture; the location of the hemorrhage along the cerebel- lar folia corresponds to the territory drained by the superior cerebellar veins. Cerebellar hemorrhages related to hypertension are not restricted to the cerebellar folia. Contrast extravasation from the cerebral angiogram would not occur unless there is disruption of the blood–brain barrier or aneurysm rupture (6,7).

8. The answer is A. The head of the bed should be elevated 30 to 45 degrees postopera- tively to diminish edema and to facilitate deep breathing. Antiplatelet therapy is con- tinued without interruption, and the SBP should be kept below 150 mmHg and above 100 mmHg. This may require vasoactive intravenous medications in the fi rst 24 hours after carotid endarterectomy (CEA). The patient is also kept NPO until the fi rst postoperative morning since reexploration is occasionally necessary. Although the patient is NPO, a main- tenance intravenous infusion of 5% dextrose and one-half normal saline should be run at 1 mL/kg/hour. Hypertension is a common postoperative complication that occurs in approx- imately 20% of patients who have carotid endarterectomy. Patients who were hypertensive before the operation, especially if poorly controlled, are more likely to have severe postop- erative hypertension. The incidence of neurologic defi cit and death is signifi cantly higher in these hypertensive patients, necessitating close monitoring of postoperative SBP (8).

9. The answer is C. Although the overall complication rate for elective endovascular treat- ment of cerebral aneurysms is relatively low, thromboembolic complications remain com- mon and account for more than half of the complications resulting from this procedure. Antiplatelet drugs (aspirin/clopidogrel) administered before and after coil embolization of cerebral aneurysms have been reported to lower the symptomatic thromboembolic complication rate. Intraprocedural anticoagulation using heparin is considered a stan- dard protocol for preventing procedural thromboembolic risks. Oral antiplatelet prepa- ration before stent-assisted aneurysm coiling has also been considered a standard step based on the benefi ts observed during coronary intervention (9).

181

66485457-66485438 www.ketabpezeshki.com

ZZakaria_87574_PTR_CH13_10-06-13_175-182.inddakaria_87574_PTR_CH13_10-06-13_175-182.indd 181181 66/19/2013/19/2013 4:45:274:45:27 PMPM DISORDERS, DISEASES, SEIZURES, AND EPILEPSY

References

1. Connolly ES Jr, Rabinstein AA, Carhuapoma JR, et al. Guidelines for the management of aneurysmal subarachnoid hemorrhage: a guideline for healthcare professionals from the American Heart Association/American Stroke Association. Stroke. 2012;43(6):1711–1737. 2. Fountas KN, Kapsalaki EZ, Nikolakakos LG, et al. Anterior cervical discectomy and fusion associated complications. Spine. 2007;32(21):2310–2317. 3. Garber ST, Sivakumar W, Schmidt RH. Neurosurgical complications of direct thrombin inhibitors—catastrophic hemorrhage after mild traumatic brain injury in a patient receiv- ing dabigatran. J Neurosurg. 2012;116(5):1093–1096. 4. Hoefnagel D, Dammers R, Ter Laak-Poort MP, Avezaat CJ. Risk factors for infections related to external ventricular drainage. Acta Neurochir (Wien). 2008;150(3):209–214; dis- cussion 214. 5. Mocco J, Komotar RJ, Ladouceur AK, Zacharia BE, Goodman RR, McKhann GM 2nd. Radiographic characteristics fail to predict clinical course after subdural electrode place- ment. Neurosurgery. 2006;58(1):120–125; discussion 120. 6. Marquardt G, Setzer M, Schick U, Seifert V. Cerebellar hemorrhage after supratentorial craniotomy. Surg Neurol. 2002;57(4):241–251; discussion 251. 7. Friedman JA, Piepgras DG, Duke DA, et al. Remote cerebellar hemorrhage after supraten- torial surgery. Neurosurgery. 2001;49(6):1327–1340. 8. Rasmussen TE, Clouse WD, Tonnessen BH. Handbook of patient care in vascular dis- eases. Great Vessels and Carotid. 5th ed. Philadelphia, PA: Lippincott Williams & Wilkins; 2008:159–195. 9. Hwang G, Jung C, Park SQ, et al. Thromboembolic complications of elective coil embo- lization of unruptured aneurysms: the effect of oral antiplatelet preparation on peripro- cedural thromboembolic complication. Neurosurgery. 2010;67(3):743–748; discussion 748.

182

66485457-66485438 www.ketabpezeshki.com

ZZakaria_87574_PTR_CH13_10-06-13_175-182.inddakaria_87574_PTR_CH13_10-06-13_175-182.indd 182182 66/19/2013/19/2013 4:45:274:45:27 PMPM 14 Pharmacology and Practical Use of Medications in Neurocritical Care Teresa A. Allison and Sophie Samuel QUESTIONS

1. When should antimicrobial therapy be initiated in patients with suspected bacterial meningitis? A. As soon as possible after diagnosis B. Within 2 hours of diagnosis C. Within 4 hours of diagnosis D. Within 8 hours of diagnosis

2. Dexamethasone has been shown to be benefi cial in treating bacterial meningitis with which organism? A. Neisseria meningitidis B. Haemophilus infl uenzae C. Streptococcus pneumoniae D. Escherichia coli

ANSWERS TO THIS SECTION CAN BE FOUND ON PAGE 192 183

66485457-66485438 www.ketabpezeshki.com

ZZakaria_87574_PTR_CH14_10-06-13_183-202.inddakaria_87574_PTR_CH14_10-06-13_183-202.indd 183183 66/19/2013/19/2013 8:47:038:47:03 PMPM DISORDERS, DISEASES, SEIZURES, AND EPILEPSY

3. What vancomycin trough level concentrations should be achieved for the treatment of bacterial meningitis? A. 5 to 10 mcg/mL B. 10 to 15 mcg/mL C. 15 to 20 mcg/mL D. 20 to 25 mcg/mL

4. Why is rifampin recommended for combination therapy only? A. Resistance develops when used alone B. Rifampin does not penetrate the cerebrospinal fl uid (CSF) suffi ciently to be used alone C. The onset of action with rifampin is delayed D. Rifampin’s in vitro activity against common meningeal pathogens is poor

5. What is the initial antimicrobial of choice for Listeria monocytogenes? A. Ceftriaxone B. Ampicillin C. Levofl oxacin D. Trimethoprim–sulfamethoxazole

6. Warfarin is challenging to use in clinical practice for which of the following reasons? A. Narrow therapeutic window B. Variability in dosing response among individual patients C. Interactions with diet and medications D. Patient compliance E. All of the above

7. Which of the following coagulation factors does warfarin inhibit? A. II, VII, IX, and X B. II, VIII, IX, and X C. II, VIII, X, and XI D. II, VII, X, and XI

8. Warfarin has the ability to act as a procoagulant. A. True B. False

9. Which agent when administered concomitantly with warfarin will prolong the patient’s prothrombin time (PT) and International Normalized Ratio (INR) response? A. Barbiturates B. Amiodarone C. Rifampin D. Carbamazepine

184

66485457-66485438 www.ketabpezeshki.com

ZZakaria_87574_PTR_CH14_10-06-13_183-202.inddakaria_87574_PTR_CH14_10-06-13_183-202.indd 184184 66/19/2013/19/2013 8:47:048:47:04 PMPM PHARMACOLOGY AND PRACTICAL USE OF MEDICATIONS: Questions

10. When is it not necessary to begin heparin before or at the time of warfarin initiation? A. Patient has chronic stable atrial fi brillation poststroke B. Patient has a known protein C defi ciency C. Patient has an acute pulmonary embolism D. Patient has a known thrombophilic state

11. For serious and or life-threatening bleeding, how should vitamin K be administered? A. Orally B. Subcutaneously C. Intravenously by a bolus D. Intravenously by slow infusion

12. What is the dose of protamine for the reversal of intravenous heparin? A. 1 mg of protamine neutralizes 50 USP units of heparin B. 1 mg of protamine neutralizes 75 USP units of heparin C. 1 mg of protamine neutralizes 115 USP units of heparin D. 1 mg of protamine neutralizes 125 USP units of heparin

13. What is the mechanism of action of dabigatran (Pradaxa®)? A. Vitamin K antagonist B. Direct thrombin inhibitor C. Factor Xa inhibitor D. Factor IIa inhibitor

14. What is the only route of administration available for dabigatran? A. Intravenous B. Subcutaneous C. Oral (including nasogastric and nasojejunal tubes) D. Oral (excluding nasogastric and nasojejunal tubes)

15. What is the most effective method of reversing the anticoagulant effects of dabigatran? A. Dialysis B. Vitamin K C. Recombinant activated factor VII (rFVIIa) D. Fresh-frozen plasma

16. The major use(s) of benzodiazepines is/are to provide: A. Anxiolysis B. Amnesia C. Analgesia D. A and B E. A and C

17. Advantages of benzodiazepines for neurologically injured patients include: A. Limited effects on cerebral and systemic vascular tone B. Anticonvulsant effects C. Coupled decreases in cerebral metabolic demand for oxygen and cerebral blood fl ow D. All of the above

185

66485457-66485438 www.ketabpezeshki.com

ZZakaria_87574_PTR_CH14_10-06-13_183-202.inddakaria_87574_PTR_CH14_10-06-13_183-202.indd 185185 66/19/2013/19/2013 8:47:048:47:04 PMPM DISORDERS, DISEASES, SEIZURES, AND EPILEPSY

18. Which benzodiazepine is best for continuous-infusion short-term sedation but not long- term sedation? A. Alprazolam B. Midazolam C. Lorazepam D. Diazepam

19. Which effects of benzodiazepines does fl umazenil (Romazicon®) reverse? A. Sedation B. Respiratory C. Sedation and respiratory D. None of the above

20. Chest wall muscular rigidity occurs with large intravenous bolus doses for which agent? A. Morphine B. Fentanyl C. Hydromorphone D. Meperidine

21. Potential adverse reactions to the opioid antagonist naloxone (Narcan®) include: A. Pain B. Hypertension C. Tachyarrhythmias D. All of the above

22. Propofol is available as an emulsion in a phospholipid vehicle. How many kcal/mL from fat should be counted into the daily caloric provisions? A. 0.5 B. 1.1 C. 1.5 D. 1.7

23. Which adverse effect of propofol is most concerning and should be monitored for? A. Infections B. Propofol infusion syndrome C. Green urine D. Pancreatitis

24. Sodium chloride 23.4% 30 mL IV over 30 minutes can be administered in place of manni- tol for the management of elevated intracranial pressure. Which of the following can be used if the 23.4% concentration is not available? A. Sodium chloride 3% 235 mL IV over 30 minutes B. Sodium chloride 5% 140 mL IV over 30 minutes C. Sodium chloride 14.6% 48 mL IV over 30 minutes D. All of the above E. None of the above

186

66485457-66485438 www.ketabpezeshki.com

ZZakaria_87574_PTR_CH14_10-06-13_183-202.inddakaria_87574_PTR_CH14_10-06-13_183-202.indd 186186 66/19/2013/19/2013 8:47:048:47:04 PMPM PHARMACOLOGY AND PRACTICAL USE OF MEDICATIONS: Questions

25. BG is a 22-year-old man (Wt: 150 kg, Ht: 5’6”) who presented with headache, neck stiff- ness, photophobia, and vomiting. On examination, he was febrile with meningismus. Analysis of cerebrospinal fl uid showed white cells 640 × 106/L (99% lymphocytes), protein 1.6 g/L, and glucose 2.5 mmol/L. No concomitant serum sample was available to report. Polymerase chain reaction (PCR) detected HSV-2 DNA in the cerebrospinal fl uid, which is consistent with the diagnosis of herpes simplex virus meningitis. The team started him on acyclovir empirically. What is the appropriate dose for acyclovir? A. Acyclovir 1,500 mg IV every 8 hours = 10 mg/kg every 8 hours of total body weight B. Acyclovir 640 mg IV every 8 hours = 10 mg/kg every 8 hours of ideal body weight (IBW) C. Acyclovir 1,000 mg IV every 8 hours = 10 mg/kg every 8 hours of adjusted body weight D. Acyclovir 400 mg IV every 12 hours

26. BG was started on 1,500 mg of acyclovir every 8 hours. On day 3 of acyclovir treatment, the nurse informed you that BG’s urine output decreased to 20 mL/hour. His creatinine and blood urea nitrogen (BUN) increased to 2.5 mg/dL and 46 mg/dL, respectively. Intravenous fl uids were not administered while he was treated with acyclovir. What should you do next? A. Acyclovir does not affect renal function. Continue with current dose and start hydrat- ing BG B. Discontinue acyclovir and start treating with ganciclovir C. Correct the dose of acyclovir per renal function and ideal body weight (IBW), start IV fl uid at 100 mL/hour, and continue treatment D. None of the above

27. BG’s renal function improved, and he was discharged home to complete his remaining medication for a total of 2 weeks. Which statement is correct? A. Start BG on acyclovir 1,500 mg PO every 8 hours B. IV to PO conversion of acyclovir is a 1:1 ratio; therefore start on the same dose of acyclovir C. Discharge BG on IV acyclovir since no options are available D. Switch to valacyclovir 1 g PO every 8 hours

28. Which one of the antiviral medications causes myelosuppression? A. Acyclovir B. Ganciclovir C. Foscarnet D. Cidofovir

29. Which antiviral medication might require coadministration of probenecid because of the risk of nephrotoxicity? A. Acyclovir B. Valacyclovir C. Cidofovir D. Foscarnet

187

66485457-66485438 www.ketabpezeshki.com

ZZakaria_87574_PTR_CH14_10-06-13_183-202.inddakaria_87574_PTR_CH14_10-06-13_183-202.indd 187187 66/19/2013/19/2013 8:47:048:47:04 PMPM DISORDERS, DISEASES, SEIZURES, AND EPILEPSY

30. Which organism causes the most common nosocomial urinary tract infection (UTI)? A. Escherichia coli B. Pseudomonas aeruginosa C. Klebsiella pneumoniae D. Enterococcus

31. Which of the following is/are predisposing factor(s) for urinary tract infections (UTIs)? A. Female sex B. Neurologic dysfunction C. Urinary tract instrumentation D. All of the above

32. IA is an 80-year-old man who lives in a nursing home. He was transferred to the ICU because of altered mental status. His medical history includes cerebral vascular disease, hyperten- sion, and diabetes mellitus. On physical examination, he is confused and disoriented with the following vital signs: temperature 102.3°F, heart rate 110 beats/minute, respiratory rate 17 breaths/minute, and BP 120/65 mmHg. His laboratory values are within normal limits except for increased blood urea nitrogen (BUN) 26 mg/dL, serum creatinine 2.1 mg/dL, and WBC 14,000 (71 polymorphonuclear leukocytes, 8-band neutrophils, 10 lymphocytes, and 5 monocytes). His urinalysis shows turbidity, 2+ glucose, pH 7.0, protein 100 mg/dL, 50 to 100 WBC, + nitrites, 8 red blood cells, and many bacteria and + casts. His preliminary urine culture is positive for Gram-negative rods. Which is the best empiric therapy for IA? A. Fluconazole 400 mg IV daily for 5 days B. Amoxicillin 500 mg IV daily every 12 hours for 5 days C. Gentamicin 150 mg IV every 8 hours for 5 days D. Cefepime 500 mg IV every 12 hours for 5 days

33. IA’s urine culture is positive for Proteus mirabilis (>100,000 bacterial colonies/mL of urine) and pan susceptible except for trimethoprim–sulfamethoxazole. What is the best next step? A. Continue treatment with broad spectrum antibiotic B. Discontinue current therapy and start ceftriaxone 1 g IV daily C. Switch to gentamicin 150 mg IV every 8 hours D. Add gentamicin to the current regimen

34. AP is a 51-year-old man who has been hospitalized in the Neuro-ICU for intracranial hemorrhage for several weeks. His medical history is signifi cant for myocardial infarc- tion, hypertension, and congestive heart failure. His hospital stay has been complicated by aspiration pneumonia. Today, he has been spiking high temperatures. His BP is 74/45 mmHg, HR is 120 beats/minute, oxygen saturation is down to 88%, WBC count is 21,000, and temperature is 103.3°F. A chest x-ray shows infi ltrates in the right lower lobe. Which of the following therapies should be initiated next? A. Norepinephrine 0.2 mcg/kg/minute to keep mean arterial pressure (MAP) > 65 mmHg B. Phenylephrine 0.5 mcg/kg/minute to keep MAP > 65 mmHg C. Empiric intravenous antibiotics to start now D. Normal saline 1,000 mL bolus followed by 50 mL/hour E. C and D

188

66485457-66485438 www.ketabpezeshki.com

ZZakaria_87574_PTR_CH14_10-06-13_183-202.inddakaria_87574_PTR_CH14_10-06-13_183-202.indd 188188 66/19/2013/19/2013 8:47:048:47:04 PMPM PHARMACOLOGY AND PRACTICAL USE OF MEDICATIONS: Questions

35. After appropriate treatment, AP’s BP improved to 125/80 mmHg. His HR continued to rise to the 140s. The decision was made to change to a different vasopressor. He is cur- rently on norepinephrine 0.5 mcg/kg/minute to keep his MAP > 65 mmHg. Which vaso- pressor should you select? A. Phenylephrine B. Dopamine C. Epinephrine D. Continue with norepinephrine

36. Which of the following vasopressors does/do not affect the β-receptor? A. Norepinephrine B. Epinephrine C. Phenylephrine D. Vasopressin E. C and D 37. Two days later, AP became hemodynamically unstable. His urine output was diminished, and his serum creatinine rose to 3.2 mg/dL. He was drowsy and confused during morning rounds. He was intubated, and a pulmonary artery catheter was placed, which revealed a pulmonary capillary wedge pressure of 25 mmHg, cardiac index of 1.2 L/minute/m2, and systemic vascular resistance of 2,800 dynes/cm–5. What is the next appropriate drug for AP? A. Initiate milrinone 0.375 mcg/kg/minute B. Add vasopressin 0.04 units/minute C. Start epinephrine and discontinued previous vasopressors D. All are appropriate options

38. Which of the following is a noncatecholamine, phosphodiesterase inhibitor, and a posi- tive inotrope. The side effect of this drug is vasodilatation and possible arrhythmias. A. Dopamine B. Furosemide C. Sildenafi l D. Milrinone

39. Which neuromuscular blocker should be avoided in patients with neuromuscular disease (e.g., Guillain–Barré syndrome)? A. Rocuronium B. Vecuronium C. Succinylcholine D. Cisatracurium 40. Which of the following neuromuscular blockers is the drug of choice for continuous infu- sion in patients with severe renal or hepatic dysfunction who require paralysis? A. Pancuronium B. Rocuronium C. Succinylcholine D. Cisatracurium

189

66485457-66485438 www.ketabpezeshki.com

ZZakaria_87574_PTR_CH14_10-06-13_183-202.inddakaria_87574_PTR_CH14_10-06-13_183-202.indd 189189 66/19/2013/19/2013 8:47:048:47:04 PMPM DISORDERS, DISEASES, SEIZURES, AND EPILEPSY

41. Pancuronium’s most common side effect is: A. Development of tolerance B. Histamine release C. Increased seizure risk D. Tachycardia

42. Which drug can increase the risk of critical illness polyneuropathy when used in conjunc- tion with neuromuscular blockade? A. Haloperidol B. Corticosteroids C. Propofol D. Dexmedetomidine

43. JP is a 43-year-old woman admitted to the ICU for traumatic brain injury. Her medical history includes complex partial seizures for which she was prescribed carbamazepine. Which of the following is true about this drug? A. It works by blocking the T-type calcium receptors B. It is indicated for absence seizure C. It increases the clearance of some drugs and also possesses autoinduction properties D. None of the above

44. The advantage of fosphenytoin over phenytoin is: A. There are no drug interactions with fosphenytoin, but with phenytoin, there are drug interactions B. Higher risk of phlebitis is documented with fosphenytoin C. The infusion rate can be up to 150 mg of phenytoin equivalent per minute D. The infusion rate can be up to 50 mg of phenytoin equivalent per minute

45. Which of the following antiepileptic drugs can prolong the PR interval and can cause atrioventricular (AV) block? A. Lacosamide B. Lamotrigine C. Benzodiazepines D. Phenytoin

46. A woman was admitted to your unit for an acute-on-chronic subdural hematoma (SDH). She was started on valproic acid for better control of her seizure disorder from her initial SDH. This morning her WBC count was 16,000 and temperature is 102.6°F. Her urine cul- ture and urinalysis came back positive with an extended-spectrum β-lactamase (ESBL). She was started on meropenem. What would you like to do with her valproic acid? A. Nothing, continue with current dose B. Check the valproic acid level since meropenem can inhibit the clearance of valproic acid C. Check the valproic acid level since meropenem can increase the clearance of valproic acid D. Change meropenem to cefepime to avoid any possible drug interaction with valproic acid

190

66485457-66485438 www.ketabpezeshki.com

ZZakaria_87574_PTR_CH14_10-06-13_183-202.inddakaria_87574_PTR_CH14_10-06-13_183-202.indd 190190 66/19/2013/19/2013 8:47:048:47:04 PMPM PHARMACOLOGY AND PRACTICAL USE OF MEDICATIONS: Questions

47. The mechanism of action of this drug is by augmenting gamma-aminobutyric acid (GABA) receptors and mediating chloride infl ux? A. Carbamazepine B. Diazepam C. Lacosamide D. Phenytoin

48. Which of the following drugs causes the most prolongation of the QTc interval? A. Ziprasidone B. Quetiapine C. Olanzapine D. Haloperidol

49. Among the currently available antipsychotic agents, which drug has the lowest risk for extrapyramidal symptoms (EPS)? A. Haloperidol B. Fluphenazine C. Quetiapine D. Olanzapine

50. Which of the following antipsychotic drugs causes the most weight gain with important clinical implications, as this has been linked with development of impaired glucose toler- ance, hyperlipidemia, and increased mortality? A. Olanzapine B. Ziprasidone C. Quetiapine D. Lurasidone

51. Which of the following is not a common side effect of atypical antipsychotic drugs? A. Renal failure B. Anticholinergic effect C. QTc prolongation D. Hyperprolactinemia

191

66485457-66485438 www.ketabpezeshki.com

ZZakaria_87574_PTR_CH14_10-06-13_183-202.inddakaria_87574_PTR_CH14_10-06-13_183-202.indd 191191 66/19/2013/19/2013 8:47:048:47:04 PMPM 14

ANSWERS

1. The answer is A. There is no prospective clinical data that correlates the timing of anti- biotic administration to clinical outcome in patients with bacterial meningitis. Current literature only examines the duration of symptoms prior to antibiotic administration. However, most would agree the longer the duration of symptoms in patients with bac- terial meningitis, the greater the possibility of a poor outcome. Studies have shown that poor outcome is associated with greater amounts of antigen or a large number of microorganisms in cerebrospinal fl uid (CSF) samples obtained prior to initiation of anti- biotics, and that delayed CSF sterilization after 24 hours of antibiotic administration is a risk factor for subsequent neurological sequelae. The sensible approach is to administer antibiotic therapy as soon as possible after the diagnosis of meningitis is suspected or proven (1). Two recent retrospective studies support this thought: One study demon- strated a reduction in mortality with early administration of antibiotics, and the other showed a benefi t in neurological outcome and survival in patients who received antibi- otic therapy prior to the patient’s level of consciousness deteriorating to less than 10 on the Glasgow Coma Scale.

2. The answer is C. Experimental meningitis studies have demonstrated that outcome is correlated with the severity of the infl ammatory response in the subarachnoid space. Steroids have been evaluated as adjunct therapy secondary to their ability to reduce this infl ammation. Results from a randomized controlled trial of adjunctive steroid therapy in adults demonstrated that early treatment with dexamethasone improves outcomes. The benefi cial effect was most apparent in patients with pneumococcal meningitis. These results were also observed in a systemic review of steroids in adults with acute bacte- rial meningitis. Treatment with steroids signifi cantly reduced mortality (relative risk = 0.6; 95% CI 0.4–0.8, P = .002) and neurological sequelae (relative risk = 0.6, 95% CI 0.4–1,

192

66485457-66485438 www.ketabpezeshki.com

ZZakaria_87574_PTR_CH14_10-06-13_183-202.inddakaria_87574_PTR_CH14_10-06-13_183-202.indd 192192 66/19/2013/19/2013 8:47:048:47:04 PMPM PHARMACOLOGY AND PRACTICAL USE OF MEDICATIONS: Answers

P = .05), with a reduction in case fatality of 21% (P = .001) in patients with pneumococcal meningitis. The results were not statistically signifi cant in patients with meningococcal meningitis (2).

3. The answer is C. Based on the potential to improve cerebrospinal fl uid (CSF) penetration, optimize therapeutic serum vancomycin concentrations, and improve clinical outcomes, total serum trough vancomycin concentrations of 15 to 20 mcg/mL should be maintained (1). There is no literature to support the possibility that higher serum trough concentra- tions than this will improve patient outcomes.

4. The answer is A. Rifampin is an excellent agent for the treatment of meningitis secondary to good CSF infection penetration and in vitro activity against many meningeal patho- gens. However, when used alone, resistance rapidly develops; it must be used in combi- nation with other antibiotics (1).

5. The answer is B. Ampicillin is the most commonly recommended initial antibiotic for the treatment of meningitis with Listeria monocytogenes. Penicillin G may also be used as initial therapy. Addition of an aminoglycoside should be considered to either agent on the basis of the patient’s presentation. Owing to poor penetration of the aminoglycosides into the cerebrospinal fl uid (CSF) with traditional dosing regimens, extended dosing regimens of 7 mg/kg based on ideal or adjusted body weight should be administered daily. Alternative agents include trimethoprim–sulfamethoxazole and meropenem (1).

6. The answer is E. Warfarin is challenging to use in clinical practice for several reasons. Warfarin’s therapeutic range is narrow, and it is often diffi cult to maintain a therapeu- tic INR for multiple reasons. Patients exhibit considerable variability in dose–response because of genetic differences, concomitant disease states such as congestive heart failure, and daily factors such as the common cold and fever. In addition, the therapeutic INR is susceptible to interactions with medications and diets with variations of vitamin K. Patient compliance and missing doses will also alter the INR level and lead to inappropriate dos- ing changes (3).

7. The answer is A. The vitamin K–dependent coagulation factors are II, VII, IX, and X. These factors require γ-carboxylation for their procoagulant activity. Administration of warfarin diminishes the amount of vitamin K available for the carboxylation process, which results in the production of undercarboxylated proteins with reduced coagulant activity. The prothrombin time (PT) responds to depletion of factors II, VII, and X at a rate proportional to their respective half-lives. During the fi rst few days, the prolongation of PT is due to reduction of factor VII, which has a half-life of 6 hours. Subsequent effects are due to the depletion of factor X (half-life ~45 hours) and factor II (half-life ~60 hours) (3).

8. The answer is A. Warfarin inhibits carboxylation of the regulatory anticoagulant proteins C, S, and Z. It has the ability to be a procoagulant, especially during the initiation period of therapy (3).

9. The answer is B. The S-enantiomer of warfarin is 5 times more potent than the R-enantiomer. Drugs that inhibit the clearance of the S-enantiomer potentiate the anti- coagulant effect. Amiodarone is a potent inhibitor of the metabolic clearance of both the

193

66485457-66485438 www.ketabpezeshki.com

ZZakaria_87574_PTR_CH14_10-06-13_183-202.inddakaria_87574_PTR_CH14_10-06-13_183-202.indd 193193 66/19/2013/19/2013 8:47:048:47:04 PMPM DISORDERS, DISEASES, SEIZURES, AND EPILEPSY

S-enantiomer and the R-enantiomer, leading to extensive anticoagulation. Rifampin, car- bamazepine, and barbiturates each increase hepatic clearance of warfarin, which leads to an inhibition of warfarin’s anticoagulation effect (3).

10. The answer is A. Unfractionated heparin or low-molecular-weight heparin should be initiated concurrently with warfarin when a rapid anticoagulant effect is required. Early recurrent ischemic stroke occurs in 5% of patients with atrial fi brillation during the initial 2 weeks. Often warfarin will be initiated 2 to 3 days after a stroke to allow for full antico- agulation to occur by day 7 to day 10 and to avoid the period when hemorrhagic transfor- mation is most likely to occur (4).

11. The answer is D. The most concerning adverse effect of vitamin K infusions is anaphy- laxis. It is recommended that it be administered orally in patients who are not currently bleeding or at signifi cant risk for bleeding. Oral administration has been shown to more effectively and rapidly reduce the INR than subcutaneous vitamin K in patients with warfarin-associated coagulopathy. Absorption of subcutaneous vitamin K is often unpre- dictable and delayed. Anaphylactoid reactions have been reported with other routes of administration; however, they are more prevalent when administered IV. The exact mech- anism is unknown; however, many believe it to be the dispersants used in the various preparations. One such dispersant, polyethoxylated castor oil (Cremophor EL), which has been used in several agents, was removed from the market because of an alarmingly high rate of anaphylactoid reactions. Intravenous vitamin K should be administered only when a patient is actively bleeding or at a signifi cant risk for bleeding. There is no evi- dence that lower doses or lower infusion times will reduce the risk of anaphylactoid reac- tions. However, it is recommended to infuse it no faster than 1 mg/minute, while many clinicians will infuse 10 mg over 30 minutes (3).

12. The answer is C. The protamine dose is determined by the dose of heparin; 1 mg of protamine neutralizes 115 USP units of unfractionated heparin. In addition, the dose is based on the duration of time since the heparin infusion was discontinued secondary to rapid decreases in heparin serum concentrations. The dose of protamine to neutralize 100 units of heparin immediately after discontinuation is 1 to 1.5 mg. However, if 30 to 60 minutes has elapsed since discontinuation of the heparin the dose will be 0.5 to 0.75 mg for every 100 units of heparin administered. If the time is 2 hours or greater the protamine dose is 0.25 to 0.375 mg. The dosing is different for subcutaneous heparin and low-molecular-weight heparin. Following subcutaneous heparin administration, 1 to 1.5 mg protamine is administered for every 100 units of heparin. Infuse 20 to 25 mg slowly, followed by the remainder of the dose over 8 to 16 hours. The antifactor Xa activity of enoxaparin is neutralized by only 60% to 75%. One milligram of protamine should be administered for each milligram of enoxaparin received. If the partial throm- boplastin time (PTT) is prolonged 2 to 4 hours after the fi rst dose, consider giving an additional dose of protamine at 0.5 mg for each milligram of enoxaparin (one-half of the fi rst dose) (5).

13. The answer is B. Dabigatran (Pradaxa) and its acyl-glucuronide metabolites are direct thrombin inhibitors. Both free and clot-bound thrombin and thrombin-induced platelet aggregations are inhibited by the active moieties (6).

194

66485457-66485438 www.ketabpezeshki.com

ZZakaria_87574_PTR_CH14_10-06-13_183-202.inddakaria_87574_PTR_CH14_10-06-13_183-202.indd 194194 66/19/2013/19/2013 8:47:048:47:04 PMPM PHARMACOLOGY AND PRACTICAL USE OF MEDICATIONS: Answers

14. The answer is D. Dabigatran is a capsule, which must be swallowed whole and intact. It cannot be opened and fl ushed down a feeding tube. The oral bioavailability of dabigatran increases by 75% when the pellets are removed from the capsule. This may result in a signifi cant increase in the immediate anticoagulation effects of dabigatran (6).

15. The answer is A. There are no known antidotes available to reverse the effects of dabiga- tran. The drug should be discontinued immediately. The serum half-life of dabigatran is 12 to 14 hours, and given adequate renal function, the plasma concentration is approxi- mately 50% of the maximum concentration at this time. Because the protein binding is low, the most effective method of reversing dabigatran is dialysis; 50% to 60% of the drug is removed over 4 hours of hemodialysis (6,7).

16. The answer is D. Benzodiazepines act principally through potentiation of the central nervous system (CNS) inhibitory neurotransmitter, gamma-aminobutyric acid (GABA). The major use of benzodiazepines is to provide anxiolysis and anterograde amnesia. Additional benefi cial effects include sedation and anticonvulsant activity. These agents do not possess any direct analgesic properties (8).

17. The answer is D. Advantages of the benzodiazepines for neurologically ill patients include limited disturbance of cerebral and vascular tone, anticonvulsant activity, and a coupled decrease in cerebral metabolic oxygen demand with a decrease in cerebral blood fl ow (8).

18. The answer is B. Midazolam has a rapid onset and short duration of action with sin- gle doses, which makes it ideal to treat acutely agitated patients or for short procedures. However, it is recommended that midazolam use be limited to 48 to 72 hours for continu- ous infusions. Studies directly comparing wake-up times with midazolam and lorazepam show a shorter time to wake up with lorazepam, despite its longer half-life. Prolonged sedative effects may be caused by the accumulation of an active metabolite, α-hydroxy- midazolam, or its conjugated salt, especially in patients with renal dysfunction. In addi- tion, prolonged sedation has been observed in obese patients or those with low serum albumin levels (9).

19. The answer is A. Flumazenil reverses the sedative effects of benzodiazepines. It does not reverse the respiratory depression, hypoventilation, or cardiac depression effects of benzodiazepines. Note, fl umazenil does not reverse the central nervous system (CNS) effects of other agents that affect GABA-ergic neurons, such as barbiturates, ethanol, or general anesthetics (10).

20. The answer is B. Fentanyl has some potential advantages over morphine. It has less effect on blood pressure (BP) at sedative doses because it does not promote the release of his- tamine. Caution is still warranted with intravenous boluses of high doses. In addition, it tends to reduce heart rate, which is favorable in cardiovascular patients. Fentanyl and fentanyl-containing products are known to cause jaw, abdominal, and chest wall rigidity with administration. This effect is most commonly associated with either high doses or rapid administration. However, it has also been reported with lower doses. Chest wall rigidity is thought to be an infrequent but serious complication. Respiratory and cardio- vascular complications can develop from hypoxemia and hypercarbia, which can lead to signifi cant morbidity or mortality (8).

195

66485457-66485438 www.ketabpezeshki.com

ZZakaria_87574_PTR_CH14_10-06-13_183-202.inddakaria_87574_PTR_CH14_10-06-13_183-202.indd 195195 66/19/2013/19/2013 8:47:048:47:04 PMPM DISORDERS, DISEASES, SEIZURES, AND EPILEPSY

21. The answer is D. Naloxone’s effects are due to its action on narcotic reversal, not due to direct effect on opioid receptors. Therefore, adverse events occur secondary to reversal of narcotic analgesia and sedation, which can cause severe reactions. Adverse events that can occur include hyper- or hypotension, tachycardia, ventricular arrhythmias, restless- ness, seizures, vomiting, diarrhea, pulmonary edema, pain, and cardiac arrest. Many cli- nicians will administer 0.4 mg IV push. However, to avoid complications, the 0.4 mg/mL vial should be diluted with 9 mL of sodium chloride 0.9% and administered at 1 mL incre- ments until desired results are achieved (11).

22. The answer is B. Propofol is available as an emulsion in a phospholipid vehicle, which provides 1.1 kcal/mL from fat and should be counted as a caloric source. Long-term or high-dose infusions may result in hypertriglyceridemia (9).

23. The answer is B. A clinically relevant incidence of infections would be suspected because of the phospholipid vehicle. However, this has not been reported. The manufacturers sug- gest propofol infusions, and tubing should be changed at least every 12 hours. In addi- tion, a preservative has been added to the propofol to decrease the potential for bacterial overgrowth. Propofol, a phenolic derivative, may cause green urine due to a chemical reaction. The green urine is not related to length of therapy or dose. Elevations of pancre- atic enzymes have been reported during prolonged infusion of propofol. However, the most concerning adverse effect of propofol is propofol infusion syndrome. Infusions of high doses (>83 mcg/kg/minute) of propofol for more than 48 hours have been associ- ated with lactic acidosis, bradycardia, and an increased risk of cardiac arrest (9).

24. The answer is D. All of the options provided can be used alternatively to the 23.4% con- centration. Thirty milliliters of 23.4% sodium chloride contain 120 mEq of sodium chlo- ride. The volume of the different concentrations should be adjusted to provide 120 mEq sodium chloride. Three percent, 5%, and 14.6% sodium chloride contain 0.51, 0.86, and 2.5 mEq/mL of sodium chloride, respectively. This would provide a fi nal volume of sodium chloride 3% of 235 mL, sodium chloride 5% of 140 mL, and sodium chloride 14.6% of 48 mL (11).

25. The answer is B. Acyclovir should be dosed on the basis of the ideal body weight (IBW). BG’s actual body weight is 150 kg, and his height is 5’6”. His IBW is 64 kg. Dosing for a cerebrospinal fl uid (CSF) infection is 10 mg/kg/dose (IBW); therefore, his appropriate dose is 640 mg IV every 8 hours (adequate kidney function). Acute renal failure (ARF) has been reported when acyclovir is dosed using actual body weight in obese patients (12). Absorption, distribution, elimination, and metabolism of the pharmacokinetic parameters of a drug can vary in obese patients. Obese patients have increase levels of lipoproteins, which can bind to serum protein and decrease the protein binding of drugs. Since acy- clovir is poorly protein bound (9%–22%), it is unlikely that drug–drug or drug–disease interactions due to protein binding can happen. Volume of distribution of a drug can also be affected in obese patients. For example, vancomycin has a slightly larger volume of distribution, which requires dosing on the basis of the actual body weight. In compari- son, acyclovir has a steady-state volume of distribution, which corresponds to total body water. Therefore, dosing using actual body weight can result in a signifi cant overdose. The package insert recommends using IBW when dosing acyclovir (13).

196

66485457-66485438 www.ketabpezeshki.com

ZZakaria_87574_PTR_CH14_10-06-13_183-202.inddakaria_87574_PTR_CH14_10-06-13_183-202.indd 196196 66/19/2013/19/2013 8:47:048:47:04 PMPM PHARMACOLOGY AND PRACTICAL USE OF MEDICATIONS: Answers

26. The answer is C. The maximum solubility of acyclovir should not exceed 2.5 mg/mL at 37°C in water. If this number is exceeded, acyclovir can precipitate in the renal tubules. High concentrations are achieved in the tubular lumen of the kidneys, as the drug is cleared rapidly through glomerular fi ltration. The frequency of “acyclovir crystal” is approximately 12% to 48%. Rapid administration of bolus acyclovir can also contribute to precipitation of the drug. Acute renal failure (ARF) can happen within 24 to 48 hours of initiating therapy (14). BG did not receive any volume resuscitation, and his acyclovir was dosed on the basis of his actual body weight. This explains the reason for his renal failure. If no improvement is seen or renal failure worsens, then acyclovir should be discontinued. Most patients recover renal function after discontinuation of acyclovir and volume resus- citation. The plan is to continue treatment with appropriate dosing and hydration.

27. The answer is D. Oral acyclovir has poor bioavailability (15%–30%). Valacyclovir is a prodrug, an esterifi ed version of acyclovir that has a greater bioavailability (55%). It is rapidly and nearly completely converted to acyclovir by intestinal and hepatic metabo- lism. The oral dose for BG will be 1 g every 8 hours since his renal function has returned to normal (12).

28. The answer is B. Ganciclovir is the drug of choice for any cytomegalovirus (CMV) infec- tion. Ganciclovir is both a competitive inhibitor of vDNA polymerase and a chain termi- nator. Its triphosphate form is more stable than acyclovir triphosphate and undergoes slower catabolism. Compared with acyclovir, it achieves 10 times greater concentrations in infected cells. Since ganciclovir is also incorporated into human DNA, it affects blood cells resulting in neutropenia, granulocytopenia, and thrombocytopenia. Additional com- plications with this drug include seizures and retinal detachment (15).

29. The answer is C. Cidofovir is indicated for cytomegalovirus (CMV) retinitis in patients with AIDS. Cidofovir is a last-line agent secondary to nephrotoxicity. The drug must be administered with concomitant probenecid and saline hydration to reduce the potential for nephrotoxicity (16).

30. The answer is A. The highest incidence of nosocomial UTIs is caused by E. coli; this accounts for 31%. The incidence of P. aeruginosa and other Gram-negative bacilli is approx- imately 10%, followed by K. pneumoniae (9%), Staphylococcus aureus (6%), P. mirabilis (5%), and Enterococcus (2%). The incidence of fungal UTIs is about 14% (17).

31. The answer is D. Predisposing factors for UTI include age, female gender, diabetes melli- tus, pregnancy, immunosuppression, urinary tract instrumentation, urinary tract obstruc- tion, renal disease, renal transplant, and neurological dysfunction (17).

32. The answer is D. Nosocomial urinary tract infections (UTIs) are the source of up to 15% of all nosocomial bloodstream infections, and the associated mortality rate is approximately 15%. Prevention is crucial, and antibiotic treatment should be started in a timely fashion. Before initiating antibiotics, each hospital should refer to the “antibiogram” to determine current trends in antibiotic susceptibility. As described previously, the most common incidence of nosocomial UTIs is Gram-negative organisms such as E. coli, P. aeruginosa, Proteus, Enterobacter, Serratia, and Acinetobacter. Escherichia coli is still the predominant

197

66485457-66485438 www.ketabpezeshki.com

ZZakaria_87574_PTR_CH14_10-06-13_183-202.inddakaria_87574_PTR_CH14_10-06-13_183-202.indd 197197 66/19/2013/19/2013 8:47:048:47:04 PMPM DISORDERS, DISEASES, SEIZURES, AND EPILEPSY

urinary tract pathogen. However, increased infection rates due to other Gram-negative bacteria have been reported. IA should be empirically started on cefepime for broader coverage until susceptibility is available. Gentamicin is also a drug of choice and should be considered as an option because of the broader Gram-negative bacterial coverage, but the dosing listed for UTI is incorrect. The correct dosing is 1 mg/kg IV every 8 to 12 hours on the basis of kidney function (17).

33. The answer is B. There is no need to continue with broader spectrum antibiotics. Once susceptibility has been reported by microbiology, appropriate de-escalation should occur. As explained previously, gentamicin should not be initiated because of the broader cover- age (17).

34. The answer is E. Early goal-directed therapy should be fl uid resuscitation. This can be achieved with crystalloids or colloids, with most treatment algorithms requiring aggres- sive resuscitation during the fi rst 24 hours of management. There are no clinical outcome differences between crystalloids or colloids; crystalloids require more volume to achieve the same endpoints. Given the patient’s chest x-ray, elevated WBC count, and tempera- ture, intravenous antibiotics should be started empirically within the fi rst hour of recogni- tion of severe sepsis, preferably after appropriate cultures are obtained (18).

35. The answer is A. Phenylephrine is a pure α-adrenergic agonist with minimal cardiac activity. Dopamine stimulates both adrenergic and dopaminergic receptors. Lower doses are mainly dopaminergic stimulating and produce renal and mesenteric vaso- dilation. Medium doses are both dopaminergic and β-1 adrenergic stimulants and produce cardiac stimulation and renal vasodilation, whereas high doses stimulate α-adrenergic receptors. Epinephrine stimulates α, β-1, and β-2 receptors resulting in relaxation of smooth muscle of the bronchial tree and increased myocardial oxygen consumption (18).

36. The answer is E. Vasopressin binds to V1, V2, and V3 receptors. V1 receptors are found on vascular smooth muscle, which leads to vasoconstriction. V2 receptors located in the kidney collecting ducts lead to an increase in water permeability and resorption in the distal tubule and collecting ducts. V3 receptors in the pituitary gland lead to an increase in adrenocorticotropic hormone and cortisol production to relieve relative adrenal insuf- fi ciency (18).

37. The answer is A. AP has signs of decreased cardiac output, such as increased serum creatinine and decreased mental status. Milrinone is a positive inotropic agent that will increase the cardiac output to maintain perfusion to vital organs. Milrinone works by vasodilating the peripheral vessels to decrease systemic vascular resistance and “unload” the heart (19).

38. The answer is D. Milrinone has dual mechanisms of action and inhibits cyclic adenosine monophosphate (cAMP) breakdown in the heart to increase inotropic activity and car- diac output. In addition, it inhibits cAMP breakdown in vascular smooth muscle to cause peripheral vasodilation and reduce systemic vascular resistance. Hypotension can be one of the concerning side effects of this drug (19).

198

66485457-66485438 www.ketabpezeshki.com

ZZakaria_87574_PTR_CH14_10-06-13_183-202.inddakaria_87574_PTR_CH14_10-06-13_183-202.indd 198198 66/19/2013/19/2013 8:47:048:47:04 PMPM PHARMACOLOGY AND PRACTICAL USE OF MEDICATIONS: Answers

39. The answer is C. Succinylcholine, the only available depolarizing agent, is used for routine endotracheal intubation and to relax skeletal muscles during surgeries of short duration. It has a rapid onset of action (10–15 seconds) and short duration (10–15 minutes), which makes it the ideal agent for these indications. Depolarization leads to the loss of potassium ions from cells. The serum potassium level typically increases by 0.5 to 1 mEq/L with the administration of succinylcholine. Although it is rare, sudden rises to life-threatening levels of potassium have been reported. Succinylcholine should be used cautiously in patients with insufficient renal func- tion, neuromuscular diseases such as Guillain–Barré, myasthenia gravis, and pro- longed immobility (e.g., spinal cord injury), as it may trigger severe hyperkalemia. The use of a nondepolarizing agent (atracurium, cisatracurium, rocuronium, and vecuronium) in these patients is recommended despite the slower onset (1–4 min- utes) and longer duration of action (20–60 minutes) (20).

40. The answer is D. Many of the neuromuscular blockers can accumulate in patients with renal or hepatic failure. However, any agent can be used with appropriate monitoring. The preferred drug of choice is atracurium or cisatracurium because organ-independent Hofmann elimination and plasma esterases eliminate these agents. Both pancuronium and vecuronium accumulate in hepatic and renal dysfunction. Rocuronium accumulates in hepatic dysfunction. It is important to adjust the dose on the basis of hepatic and renal function, especially when choosing the initial dose. However, titration of neuromuscular blockers should be based on train-of-four monitoring (20).

41. The answer is D. Development of tolerance is commonly seen with atracurium and cisa- tracurium. Accumulation of laudanosine, the metabolite of atracurium and cisatracu- rium, predisposes the patient to a theoretical seizure risk. This risk is of greater concern with atracurium than with cisatracurium. Histamine release has also been associated with atracurium. For these reasons, the use of atracurium has been widely replaced with cisa- tracurium. Tachycardia is the most common side effect of pancuronium; this is signifi cant, especially in patients with cardiovascular disease (20).

42. The answer is B. Prolonged muscle weakness, including acute quadriplegic myopathy syndrome, critical illness polyneuropathy, and acute myopathy of critical care, have been reported after discontinuation of neuromuscular blocking agents. The risk factors for these syndromes are not very well understood, but are strongly related to use of neuro- muscular blocking agents with concurrent drug therapy (e.g., corticosteroids) and mul- tiorgan failure. Neuromuscular blocking agents with steroid-like structures (vecuronium and pancuronium) were initially reported to cause prolonged weakness. In more recent case reports, however, coadministration of the newer agents such as cisatracurium with steroids also contributed to prolonged muscle weakness (21).

43. The answer is C. Carbamazepine is a potent inducer of the hepatic cytochrome P450 3A4 (CYP3A4) system. The drug is also a substrate for this specifi c enzyme. Carbamazepine stimulates the metabolism of other CYP3A4 substrates but also autoinduces its own metabolism. This can increase clearance of the drug, requiring higher doses or more frequent dosing. The autoinduction effect usually lasts for approximately 1 month.

199

66485457-66485438 www.ketabpezeshki.com

ZZakaria_87574_PTR_CH14_10-06-13_183-202.inddakaria_87574_PTR_CH14_10-06-13_183-202.indd 199199 66/19/2013/19/2013 8:47:048:47:04 PMPM DISORDERS, DISEASES, SEIZURES, AND EPILEPSY

Carbamazepine works by inhibiting the sodium channel receptors. Indications for car- bamazepine include partial seizures with complex symptomatology, generalized t onic- clonic seizures, mixed seizure patterns, and trigeminal neuralgia (22).

44. The answer is C. Fosphenytoin, a phosphate ester prodrug of phenytoin, is highly water soluble. It can be administered either intramuscularly (IM) or intravenously (IV) with less risk of tissue damage and venous irritation than with parenteral administration of phenytoin. Fosphenytoin is rapidly absorbed and converted to phenytoin by phosphatase enzymes. Phenytoin is associated with infusion-related adverse reactions due to the sodium hydroxide, propylene glycol, and alcohol content of the IV formulation. Venous irritation and tissue damage are most likely to occur when large doses of undiluted phe- nytoin are administered. Complications such as hypotension and arrhythmias are related to rapid IV administration (>50 mg/minute). Caution should be applied to patients with hemodynamic instability or cardiovascular diseases. The rate of infusion when adminis- tering fosphenytoin is 150 mg phenytoin equivalent/minute. However, phenytoin infu- sions should not exceed a rate of 50 mg/minute. Drug interaction and metabolism effects are similar to those for phenytoin (22).

45. The answer is A. Lacosamide is indicated for the management of partial onset seizures in combination with other anticonvulsants. Lacosamide should be used cautiously in patients with known cardiac conduction abnormalities. (e.g., marked fi rst-degree AV block, second- or third-degree AV block, and sick sinus syndrome without a pacemaker) or severe cardiovascular disease (e.g., myocardial ischemia and heart failure). The manu- facturer recommends obtaining an electrocardiogram before initiating lacosamide and after titration to steady state in such patients (23).

46. The answer is C. Valproic acid is a substrate of the hepatic cytochrome P450 system, which can lead to the induction or inhibition of the metabolism of valproic acid. In addition, valp- roic acid can affect the metabolism of other agents by this enzyme system. Numerous cases have reported the drug interaction between meropenem and valproic acid. Coadministration of valproic acid and meropenem decreases the serum concentration of valproic acid to sub- therapeutic levels and is associated with occurrence of seizures (24).

47. The answer is B. Gamma-aminobutyric acid (GABA) is an inhibitory neurotransmitter. It promotes opening of the postsynaptic receptor, GABA type A (GABA-A). This opening leads to an increased conductance of chloride ions, which results in membrane hyperpo- larization and neuronal inhibition. The binding of benzodiazepines to the GABA-A recep- tor increases the affi nity of GABA and its receptor, increasing the frequency of GABA-A receptor fi ring and potentiating GABA-ergic neurotransmission (22).

48. The answer is A. All antipsychotic drugs have the potential to prolong the QTc interval to varying degrees. Ziprasidone prolongs the QTc interval longer than haloperidol, ris- peridone, olanzapine, or quetiapine. The exact point at which QTc prolongation becomes clinically dangerous is unclear. The criterion to defi ne QTc prolongation, which is based on established guidelines from the Committee for Proprietary Medicinal Products, is an increase by greater than 60 milliseconds above baseline—or greater than 450 milliseconds for males and greater than 470 milliseconds for women (25).

200

66485457-66485438 www.ketabpezeshki.com

ZZakaria_87574_PTR_CH14_10-06-13_183-202.inddakaria_87574_PTR_CH14_10-06-13_183-202.indd 200200 66/19/2013/19/2013 8:47:048:47:04 PMPM PHARMACOLOGY AND PRACTICAL USE OF MEDICATIONS: Answers

49. The answer is C. Acute dystonia, parkinsonism, and akathisia are EPS, which usually occur early after the initiation of treatment, whereas tardive dyskinesia, tardive dystonia, and tardive akathisia have a later onset, usually after years of treatment. In general, the typical agents such as haloperidol and fl uphenazine are more likely to cause EPS than atypical agents when used at the current therapeutic doses (26). Among the currently available atypical agents, quetiapine has the lowest risk for EPS.

50. The answer is A. Weight gain is documented as one of the most signifi cant side effects and concerns associated with the atypical antipsychotic drugs. The proposed mechanism of action for inducing weight gain can be contributed to the antagonistic effect on the

histamine H1 receptors and serotonin 5-HT2c. The weight gain is predominantly observed with clozapine and olanzapine; the incidence is lower with ziprasidone and aripiprazole and intermediate with risperidone and quetiapine (27).

51. The answer is A. Renal failure is not a concern with atypical antipsychotic agents. Anticholinergic effects are more signifi cant with the atypical agents. Clinically, patients present with dry mouth, constipation, and urinary retention. Hyperprolactinemia is induced because of the inhibitory effect on the dopamine receptor in the tuberoinfundibu- lar tract, which then elevates serum prolactin. To a lesser extent, risperidone and olanzap- ine increase prolactin levels; this is dose related. This side effect is more common with the typical antipsychotic agents because of their higher affi nity to dopamine receptors (28).

References

1. Tunkel AR, Hartman BJ, Kaplan SL, et al. Practice guidelines for the management of bac- terial meningitis. Clin Infect Dis. 2004;39:1267–1284. 2. van de Beek D, de Gans J, McIntyre P, et al. Steroids in adults with acute bacterial menin- gitis: a systematic review. Lancet Infect Dis. 2004;4:139–143. 3. Ansell J, Hirsh J, Hylek E, et al. Pharmacology and management of the vitamin K antago- nists: American College of Chest Physicians Evidence-based clinical practice guidelines (8th Edition). Chest. 2008;133:160S–198S. 4. Hart RG, Palacio S, Pearce LA. Atrial fi brillation, stroke, and acute antithrombotic ther- apy: analysis of randomized clinical trials. Stroke. 2002;33:2722–2727. 5. Lacy CF, Armstrong LL, Goldman MP, et al. Drug Information Handbook. 15th ed. Hudson, OH: Lexi-Comp, Inc.; 2007:1451–1452. 6. Pradaxa Package Insert. Ridgefi eld, CT: Boerhinger Ingelheim Pharmaceuticals; 2012 Dec. 7. Stangier J, Clemens A. Pharmacology, pharmacokinetics, and pharmacodynam- ics of dabigatran etexilate, an oral direct thrombin inhibitor. Clin Appl Thromb Hemost. 2009;15:9S–16S. 8. Mirski MA, Lewin JJ 3rd. Sedation and analgesia in acute neurologic disease. Curr Opin Crit Care. 2010;16:81–91. 9. Society of Critical Care Medicine and American Society of Health-System Pharmacists. Clinical practice guidelines for the sustained use of sedatives and analgesics in the criti- cally ill adult. Am J Health-Syst Pharm. 2002;59:150–78. 10. Lacy CF, Armstrong LL, Goldman MP, et al. Drug Information Handbook. 15th ed. Hudson, OH: Lexi-Comp, Inc.; 2007:718–720.

201

66485457-66485438 www.ketabpezeshki.com

ZZakaria_87574_PTR_CH14_10-06-13_183-202.inddakaria_87574_PTR_CH14_10-06-13_183-202.indd 201201 66/19/2013/19/2013 8:47:058:47:05 PMPM DISORDERS, DISEASES, SEIZURES, AND EPILEPSY

11. Lacy CF, Armstrong LL, Goldman MP, et al. Drug Information Handbook. 15th ed. Hudson, Ohio: Lexi-Comp, Inc.; 2007:1194–1195. 12. Hernandez JO, Norstrom J, Wysock G. Acyclovir-induced renal failure in an obese patient. Am J Health Syst Pharm. 2009;66(14):1288–1291. 13. Zovirax package insert. Research Triangle Park, NC: GlaxoSmithKlein; 2007 Nov. 14. Perazella MA. Crystal-induced acute renal failure. Am J Med. 1999;106(4):459–465. 15. Spector SA, Weingeist T, Pollard RB, et al. A randomized, controlled study of intrave- nous ganciclovir therapy for cytomegalovirus peripheral retinitis in patients with AIDS. AIDS Clinical Trials Group and Cytomegalovirus Cooperative Study Group. J Infect Dis. 1993;168(3):557–563. 16. Wolf DL, Rodríguez CA, Mucci M, Ingrosso A, Duncan BA, Nickens DJ. Pharmacokinetics and renal effects of cidofovir with a reduced dose of probenecid in HIV-infected patients with cytomegalovirus retinitis. J Clin Pharmacol. 2003;43(1):43–51. 17. Gupta K, Hooton TM, Naber KG, et al. International clinical practice guidelines for the treatment of acute uncomplicated cystitis and pyelonephritis in women: A 2010 update by the Infectious Diseases Society of America and the European Society for Microbiology and Infectious Diseases. Clin Infect Dis. 2011;52(5):e103–e120. 18. Dellinger RP, Levy MM, Carlet JM, et al. Surviving Sepsis Campaign: international guidelines for management of severe sepsis and septic shock: 2008. Crit Care Med. 2008:36:296–327. 19. Lindenfeld J, Albert NM, Boehmer JP, et al. HFSA 2010 Comprehensive heart failure prac- tice guideline. J Card Fail. 2010;16:e1–e194. 20. McManus MC. Neuromuscular blockers in surgery and intensive care, Part 2. Am J Health Syst Pharm. 2001;58:2381–2395. 21. Murray MJ, Cowen J, DeBlock H, et al. Clinical practice guidelines for sustained neuro- muscular blockade in the adult critically ill patient. Crit Care Med. 2002;30(1):142–156. 22. Johannessen SI, Ben-Menachem E. Management of focal-onset seizures: an update on drug treatment. Drugs. 2006;66:1701–1725. 23. Vimpat package insert. Symrna, GA:UCB Inc, 2011 April. 24. Coves-Orts FJ, Borrás-Blasco J, Navarro-Ruiz A, et al. Acute seizures due to a probable interaction between valproic acid and meropenem. Ann Pharmacother. 2005;39:533–537. 25. Glassman AH, Bigger JT Jr. Antipsychotic drugs: prolonged QTc interval, torsade de pointes, and sudden death. Am J Psychiatry. 2001;158:1774–1782. 26. Dolder CR, Jeste DV. Incidence of tardive dyskinesia with typical versus atypical antipsy- chotics in very high risk patients. Biol Psychiatry. 2003;53:1142–1145. 27. McIntyre RS, Mancini DA, Basile VS. Mechanisms of antipsychotic-induced weight gain. J Clin Psychiatry. 2001;62(Suppl 23):23–29. 28. Wirshing DA, Pierre JM, Erhart SM, Boyd JA. Understanding the new and evolving pro- fi le of adverse drug effects in schizophrenia. Psychiatr Clin North Am. 2003;26:165–190.

202

66485457-66485438 www.ketabpezeshki.com

ZZakaria_87574_PTR_CH14_10-06-13_183-202.inddakaria_87574_PTR_CH14_10-06-13_183-202.indd 202202 66/19/2013/19/2013 8:47:058:47:05 PMPM II General Critical Care: Pathology, Pathophysiology, and Therapy

66485457-66485438 www.ketabpezeshki.com

ZZakaria_87574_PTR_CH15_10-06-13_203-216.inddakaria_87574_PTR_CH15_10-06-13_203-216.indd 203203 66/19/2013/19/2013 4:45:484:45:48 PMPM 66485457-66485438 www.ketabpezeshki.com 66485457-66485438 www.ketabpezeshki.com

ZZakaria_87574_PTR_CH15_10-06-13_203-216.inddakaria_87574_PTR_CH15_10-06-13_203-216.indd 204204 66/19/2013/19/2013 4:45:494:45:49 PMPM 15 Cardiovascular Physiology David J. Powner QUESTIONS

1. The four determinants of cardiac output include: A. Diastolic time B. Heart rate C. Systolic time interval D. Arterial pulse–pressure variation E. All of the above F. None of the above

2. Complete the following equation: Pressure = ______× Resistance A. Afterload B. Volume C. Flow D. Heart rate E. Impedance

ANSWERS TO THIS SECTION CAN BE FOUND ON PAGE 209 205

66485457-66485438 www.ketabpezeshki.com

ZZakaria_87574_PTR_CH15_10-06-13_203-216.inddakaria_87574_PTR_CH15_10-06-13_203-216.indd 205205 66/19/2013/19/2013 4:45:494:45:49 PMPM GENERAL CRITICAL CARE: PATHOLOGY, PATHOPHYSIOLOGY, AND THERAPY

3. The components of the equation for oxygen content of blood (mL O2/100 mL blood) include:

A. Partial pressure of O2 in central venous blood B. Oxygen saturation in blood drawn slowly from a pulmonary artery catheter (PAC) C. Cardiac output

D. Partial pressure of CO2 in arterial blood E. Solubility coeffi cient of oxygen in blood

4. The partial pressure or percent hemoglobin saturation of oxygen in blood drawn slowly from a pulmonary artery catheter (PAC) is intended to evaluate: A. Oxygen consumption by lung tissue B. Relationship of oxygen delivered versus consumed by all body tissues C. Oxygen absorption across the alveolar–capillary membrane

D. Oxygen transport (mL O2/minute) E. None of the above

5. To compare cardiovascular parameters among patients and to normalize values, measure- ments may be “indexed” by dividing the measured value by: A. Body mass index B. Body weight C. Creatinine-height index D. Cardiac output E. None of the above

6. The ejection fraction measured by echocardiography primarily evaluates: A. Myocardial contractility B. Valvular incompetence or stenosis C. Cardiac output D. Venous preload E. Vascular resistance

7. The thermodilution method of cardiac output measurement by iced-saline injection uti- lizes all variables listed here except: A. Temperature of the pulmonary artery blood B. Distance between the injection port and the thermistor C. Transpulmonary gas temperature D. Volume of saline injected

8. The normal partial pressure of oxygen in a properly collected mixed venous blood sample is: A. 40 mmHg B. 65 to 70 mmHg C. 100 mmHg D. 65% to 70% E. None of the above

206

66485457-66485438 www.ketabpezeshki.com

ZZakaria_87574_PTR_CH15_10-06-13_203-216.inddakaria_87574_PTR_CH15_10-06-13_203-216.indd 206206 66/19/2013/19/2013 4:45:494:45:49 PMPM CARDIOVASCULAR PHYSIOLOGY: Questions

9. Anatomic mixing of venous blood from the superior and inferior venae cavae occurs in the: A. Right atrium B. Pulmonary artery C. Left atrium D. Pulmonary veins E. None of the above

10. The pulmonary artery “wedge” or occlusion pressure is intended to refl ect and direct therapy as a surrogate of: A. The ejection fraction B. Left ventricular end-diastolic volume (LVEDV) C. Right ventricular end-diastolic pressure D. Afterload E. None of the above

11. Which cardiac variable listed here increases in proportion to preload? A. Left ventricular (LV) systolic fi lling B. Mitral stenosis C. Left atrial dp/dt during atrial fi brillation D. Contractility E. Right atrial systole during atrial fi brillation

12. Although “shunt” has been discussed throughout sections about the lung, the actual evaluation is measured as the QS/QT or ratio of the blood “shunted” around the lung as a fraction (or percentage) of the total cardiac output fl ow, that is, shunt fl ow/total fl ow. Stated another way, it is the percentage of cardiac output that does not effectively par- ticipate in full oxygenation, acting as though it has never been to the lung. The complex

formula for this important ratio is QS/QT = (CcO2 − CaO2)/(CcO2 − CvO2) A pulmonary artery catheter (PAC) is required to measure what variable needed in this formula?

A. CcO2

B. CaO2 C. Cardiac output

D. CvO2 E. None because a PAC is not needed

13. The arterial pulse pressure variation is used by some cardiac output monitoring devices to suggest that the patient will benefi t from what intervention? A. Preload augmentation B. Afterload reduction C. Vasopressor (e.g., norepinephrine) administration D. Inotropic (e.g., dopamine) support E. Resumption of normal sinus rhythm

207

66485457-66485438 www.ketabpezeshki.com

ZZakaria_87574_PTR_CH15_10-06-13_203-216.inddakaria_87574_PTR_CH15_10-06-13_203-216.indd 207207 66/19/2013/19/2013 4:45:494:45:49 PMPM GENERAL CRITICAL CARE: PATHOLOGY, PATHOPHYSIOLOGY, AND THERAPY

14. Arterial pressure waveform analysis is used by several commercially available devices to measure cardiac output. These devices calculate the cardiac output after using arterial waveform analysis to defi ne: A. The dp/dt of left ventricular (LV) diastole B. LV ejection time (speed) C. Stroke volume (SV) D. First derivative of the rate of rise of the fi rst 0.04 seconds of the arterial pressure dur- ing systole E. The area under the curve of the diastolic relaxation waveform

15. The oxygen saturation in blood drawn from a thoracic central venous catheter has been suggested as a clinically satisfactory replacement for what hemodynamic parameter? A. Oxygen-carrying capacity of pulmonary blood B. Oxygen content of arterial blood

C. Arteriovenous O2 difference D. Oxygen transport into the systemic circulation E. None of the above

208

66485457-66485438 www.ketabpezeshki.com

ZZakaria_87574_PTR_CH15_10-06-13_203-216.inddakaria_87574_PTR_CH15_10-06-13_203-216.indd 208208 66/19/2013/19/2013 4:45:494:45:49 PMPM 15

ANSWERS

1. The answer is B. The four determinants of cardiac output are preload, afterload, heart rate, and contractility. Heart rate is a straightforward measurement, except when myocar- dial effi ciency is altered by various dysrhythmias. i. Afterload (or more properly, impedance) is usually evaluated by the equation for sys- temic vascular resistance index (SVRI): SVRI = 80 (MAP − CVP)/CI where MAP, mean arterial pressure; CVP, central venous pressure; CI, cardiac index (cardiac output/body surface area [BSA]) (Normal: 1,600–2,400 dyne·second·m2/cm5) ii. Contractility is usually measured in the ICU as the ejection fraction obtained by echocardiography. Other measures, such as the dp/dt (rate of rise of left ventricular (LV) pressure) during initial LV systole, may be more accurate, but are not available at bedside. Another estimate of contractility is LV stroke work index (LVSWI) as work done during systole. This parameter requires a pulmonary artery catheter (PAC) and is calculated as: LVSWI = 0.0136 (MAP – PCWP) × SVI where PCWP, pulmonary capillary wedge (occluded) pressure; SVI, stroke volume index (CI/heart rate) (Normal: 40–60 g·m/m2) iii. Preload is the volume of blood returning to the heart (usually per minute), best mea- sured as the left ventricular end-diastolic volume (LVEDV). This volume has tradi- tionally been represented by a pressure, variously available as the pulmonary artery occlusion (wedge) pressure, which corresponds to the left atrial pressure that rep- resents LV end-diastolic pressure (LVEDP). Another clinical estimate has been the pu lmonary artery diastolic pressure when any of the preceding pressures are unavail- able. As a function of physiology, for a pressure to represent a volume, especially as

209

66485457-66485438 www.ketabpezeshki.com

ZZakaria_87574_PTR_CH15_10-06-13_203-216.inddakaria_87574_PTR_CH15_10-06-13_203-216.indd 209209 66/19/2013/19/2013 4:45:494:45:49 PMPM GENERAL CRITICAL CARE: PATHOLOGY, PATHOPHYSIOLOGY, AND THERAPY

a trended variable overtime, their relationship, that is, ΔP representing a ΔV, must be constant. This relationship, ΔV/ΔP, defi nes LV compliance, which is not constant, particularly during sepsis, hypertension, or coronary artery–induced myocardial dysfunction. In addition, changes in thoracic pressure that affect venous return dur- ing mechanical ventilation may make interpretation of these pressures more diffi cult. Therefore, defi ning preload remains diffi cult because of the imprecise methodology used to measure it (1,2).

2. The answer is C. This is a fundamental equation in many aspects of physiology and is modifi ed for cardiovascular issues as: Blood pressure = Cardiac output × Systemic vascular resistance As a fl ow variable, the units of cardiac output are L/minute. These relationships highlight the interdependency of cardiac output and changes in vascular constriction to maintain blood pressure. In clinical therapy, of course, one must treat one of the determinants of cardiac output (Answer 1) to affect change in cardiac output.

3. The answer is E. The oxygen content of arterial blood in mL O2/100 mL blood is deter- mined by the equation:

CaO2 = (Hgb × 1.37 × SaO2) + (PaO2 × .003)

where Hgb, hemoglobin in g/dL; SaO2, the % Hgb saturation at the given FIO2; .003, the

solubility coeffi cient of oxygen in blood (Normal: 18 to 20 mL O2/dL).

Oxygen delivery (transport) (VDO2 or DO2) extends the O2 content in mL/100 mL blood to the oxygen carried forward to tissues by the cardiac output (L/minute):

DO2 = CaO2 × CO × 10 (Normal: 900–1,100 mL O2/L/minute) As oxygen is delivered to body tissues, some is consumed and some enters the venous circulation to return via the superior and inferior venae cavae to the right atrium. These caval streams of blood do not mix fully until both enter the right ventricle, and the resid- ual venous oxygen enters the pulmonary artery as the mixed venous oxygen. Sampling of the mixed venous blood by aspirating from a PAC allows calculation of the mixed venous

content of oxygen (CvO2), from measurement of the partial pressure of O2 (PvO2; nor- mal 40 mmHg), blood Hgb, and Hgb saturation (SvO2; normal 0.66–0.74) from a mixed venous blood gas:

CvO2 = (Hgb × 1.37 × SvO2) + (PvO2 × .003) (Normal: ~15 mL/dL)

The amount of oxygen consumed (VO2) by all tissues is the difference between what was delivered and what remained in the circulation within mixed venous blood. The

a-vDO2:

a-vDO2 = CaO2 − CvO2 (Normal: 3.6–5.0 mL/dL) When adjusted by the cardiac output to calculate consumption for the whole body:

VO2 = (CaO2 – CvO2) × CO × 10 (Normal: 200–300 mL/minute) All of the preceding values can be indexed by dividing the parameter by the patient’s BSA, providing the ability to compare patients of different body habitus. Clinically, oxygen

delivery index (DO2I) and oxygen consumption index (VO2I) may be compared: Normal 2 2 DO2I is 530 to 600 mL/minute/m ; VO2I is 110 to 160 mL/minute/m .

210

66485457-66485438 www.ketabpezeshki.com

ZZakaria_87574_PTR_CH15_10-06-13_203-216.inddakaria_87574_PTR_CH15_10-06-13_203-216.indd 210210 66/19/2013/19/2013 4:45:494:45:49 PMPM CARDIOVASCULAR PHYSIOLOGY: Answers

Therefore, it is possible and clinically important to compare oxygen supply to oxygen consumed by the entire body. Organ-specifi c demand/supply relationships would be important, but are unavailable unless the specifi c arteries to and veins from individual organs are cannulated. Because of the unique isolated anatomy of the brain, its supply and consumption have been explored using arterial oxygen delivery and venous return obtained from the jugular vein. Another variable derived from this information is the oxygen extraction ratio that pro- vides perspective on the fraction or percentage of oxygen delivered that is used by the body during normal or stressed metabolism. Oxygen extraction may be increased during hypoperfusion to compensate for reduced delivery. Extraction may appear to be reduced by anatomic arteriovenous connections such as in liver disease or may actually be reduced when mitochondrial oxygen uptake is inhibited in sepsis or cyanide poisoning.

O2 extraction = VO2/DO2 (Normal: 0.22–0.28) As cardiac output falls, the slower fl ow through tissue capillaries, and perhaps increased

O2 extraction within that tissue bed, removes maximal amounts of arterial oxygen, thereby reducing the amount of venous oxygen that returns to the circulation. This, of course,

decreases the CvO2 and its components, PvO2 and SvO2, assuming hemoglobin concen-

tration remains constant. Because the PvO2 is diminished by the O2 solubility factor, the

SvO2 has become the commonly used surrogate for the CvO2 in monitoring the mixed

venous:arterial O2 relationship. Continuous monitoring of SvO2 is available via special- ized PACs, allowing abnormalities to be trended. During clinical conditions (e.g., sepsis) that reduce oxygen uptake by mitochondria, more

O2 remains in the venous blood, and mixed venous content and SvO2 rise. Similarly, in

patients with therapeutic or pathophysiological arteriovenous shunts, venous O2 and

SvO2 rise (3).

4. The answer is B. See Answer 3 for a full explanation of the value of monitoring the SvO2. Mixed venous samples are processed via a blood gas analyzer as an arterial specimen. Care must be taken that blood is not drawn too quickly from the PAC, because oxygenated blood may be pulled backward through the capillary and cause an erroneous elevation in

the PvO2 from which the SvO2 is abstracted.

5. The answer is E. The “indexing” of many parameters allows a normal value for that parameter to be determined among patients with differing body confi gurations. Body surface area (BSA) is most easily available from a Dubois body surface chart but can be calculated from complex equations (2,3): BSA (m2) = [√ Height (cm) × Weight (kg)]/3,600, or BSA = 0.007184 × Weight0.425 × Height0.725

6. The answer is A. Contractility is one of the four determinants of cardiac output. LV stroke work (see Answer 3) has been used as an estimate of contractility and refl ects work done by the left ventricle to overcome outfl ow impedance. Contractility is diffi cult to quantify in the ICU setting because of its interdependence with preload, afterload, and heart rate. The ejection fraction (Normal: >55%–60%) obtained by echocardiography is most often used clinically.

211

66485457-66485438 www.ketabpezeshki.com

ZZakaria_87574_PTR_CH15_10-06-13_203-216.inddakaria_87574_PTR_CH15_10-06-13_203-216.indd 211211 66/19/2013/19/2013 4:45:494:45:49 PMPM GENERAL CRITICAL CARE: PATHOLOGY, PATHOPHYSIOLOGY, AND THERAPY

Contractility is abnormal in several neurological conditions associated with large amounts of catecholamine release from the brain. “Myocardial stunning,” evidenced by decreased contractility, is documented in subarachnoid hemorrhage and particularly during the evolution of brain death in some patients. This pattern appears similar to the Takotsubo cardiomyopathy documented in patients with pheochromocytoma and other syndromes associated with high catecholamine release (4).

7. The answer is C. The traditional thermodilution method of measuring cardiac output has evolved from Stewart’s original work in the 1890s to the traditional method of inject- ing iced or room temperature saline into the pulmonary artery. This injection of a known quantity of injectate at a known temperature into a fl owing bloodstream also of known temperature induces a temporary temperature change in the pulmonary artery blood as it passes a temperature sensor (thermistor) a known distance from the site of injection. The Stewart-Hamilton equation (2) to determine the cardiac output from this thermodilution method is: CO = [60 × Vi × Ci × Si × Kcal × Kcor × (Tb − Ti)]/[Cb × Sb × ∫ΔTb(t)dt] where Vi, injectate volume (mL); Ci and Cb, specifi c heats of injectate and blood (con- stants); Kcal, calibration constant; Si and Sb, specifi c gravity of injectate and blood ( constants); Kcor, temperature loss constant; Tb and Ti, baseline blood and injectate tem- peratures; integral term, area under thermal curve of temperature change versus time. Continuous cardiac output pulmonary artery catheters (PACs) use similar changes in pul- monary blood temperature, but instead generate a burst of heat (not cold) into the blood- stream, and the change in blood temperature is sensed along a thermistor fi lament within the catheter.

8. The answer is A. See Answer 3 for a more complete discussion. Abnormal PvO2, either below or above normal, will help evaluate the predominant abnormality in cardiovascu-

lar performance or oxygen debt (when O2 delivery does not meet need). It is helpful to

note the PvO2, although the SvO2 is used more extensively. Because blood gas analyzers

do not directly measure the SaO2 or SvO2, as does an oximeter or cooximeter, it is useful to ensure that the saturation (venous or arterial) presented by the blood gas analyzer is

consistent with the measured PaO2 or PvO2. The extrapolation from partial pressure to saturation is, of course, a refl ection of the oxyhemoglobin dissociation curve. The “30–60-90” guideline is a useful rule of thumb describing Hgb-association param-

eters: At a PaO2 or PvO2 of 30 mmHg, there is 60% Hgb saturation, and at PaO2 or PvO2 of 60 mmHg, Hgb is 90% saturated.

9. The answer is E. Streaming of blood from the superior and inferior venae cavae into the relatively small right atrium does not achieve full mixing. Mixing occurs in the right ven- tricle (5).

10. The answer is B. As discussed in Answer 1, the intention of the pulmonary artery (PA) wedge (or PA-occlusion pressure [PAoP]) is to evaluate cardiac preload, a volume, not pressure, measurement. Although a direct conversion of pressure to volume is not pos- sible, the relationship between the two could be useful as a relative trend in their values over time as therapy changes. However, to be useful even as a trended value, there must

212

66485457-66485438 www.ketabpezeshki.com

ZZakaria_87574_PTR_CH15_10-06-13_203-216.inddakaria_87574_PTR_CH15_10-06-13_203-216.indd 212212 66/19/2013/19/2013 4:45:494:45:49 PMPM CARDIOVASCULAR PHYSIOLOGY: Answers

be a linear (straight-line) relationship between the two variables of pressure and volume. The relationship between left ventricular end-diastolic volume (LVEDV) and LV end- diastolic pressure (LVEDP) refl ects LV compliance (ΔV/ΔP). Therefore, LV compliance must be constant (and normal) if LVEDP (or its surrogate, the PA wedge pressure) is to accurately refl ect LVEDV or its trend. LV compliance is not constant and may change over short periods of time. This concept also introduces the potential for abnormal diastolic relaxation (distension) of the left ventricle, as it alters LV compliance characteristics. Coronary artery disease, hypertension, diabetes, some forms of cardiomyopathy, and sepsis are examples of con- ditions that decrease LV diastolic relaxation and cause the LV myocardium to become more stiff. This change in diastolic compliance also may reduce venous return, as LVEDV, and/or alter the wedge reading. A smaller volume in the stiffer left ventricle may be represented by an elevated wedge pressure, a false measurement of LVEDV (3,6,7). The prevalence among ICU patients of those conditions that might alter LV compliance makes this an important clinical consideration in trying to interpret wedge measurements and their infl uence on treatment.

11. The answer is D. Starling, using Howell and Donaldson’s in vitro venous reservoir to simulate venous return, showed the heart’s ability to increase its output as venous pre- load increased, “up to the limit of its capacity” (8).

12. The answer is D. The CvO2 represents the mixed venous specimen returned to the heart

after tissues have removed oxygen from the arterial blood delivered to them (VDO2). The mixed venous specimen is obtained distal to mixing in the right ventricle from pulmonary

artery blood through the pulmonary artery (PA) catheter. The CcO2 represents the capil- lary oxygen content of an idealized “perfect” alveolus adjacent to a “perfect” capillary. This

value, of course, is not measurable, but utilizes the PAO2 in the standard content equation:

CcO2 = (Hgb × 1.37 × SPAO2) + (0.003 × PAO2)

where (as discussed in Chapter 17, Answer 18) the PA (short form) equation is PAO2 =

PIO2 − (1.25 × PaCO2), where PIO2 = (Pb − 47) × FIO2. The shunt equation is similar to the Bohr equation (Chapter 17, Answer 17) in that it co mpares lung failure in the numerator to lung potential in the denominator. In the

numerator, the CcO2 represents the ideal transfer of oxygen from the perfect alveolus to the perfect capillary, whereas the actual accomplishment of that transfer is represented by

the CaO2. The difference between them shows the failure of that perfect opportunity. The denominator highlights the optimal opportunity to add oxygen to the blood delivered to the lung from the mixed venous circulation. Therefore, the shunt equation relates fail- ure (numerator) to ideal opportunity (denominator) as a fraction or percentage. Normal shunt (QS/QT) is up to 0.08 or 8%. This equation utilizes oxygen as an indicator gas and informs the clinician as to what portion of the cardiac output perfused the lung but failed to maximally gain oxygen. This equation quantifi es the magnitude of the V/Q mismatch (shunt effect) as the primary cause of hypoxemia.

13. The answer is A. Variability in the arterial pulse pressure (systolic–diastolic pressures), stroke volume (SV), systolic pressure, and preload are caused by cyclic variation in intrathoracic pressure during mechanical ventilation. Several “minimally invasive”

213

66485457-66485438 www.ketabpezeshki.com

ZZakaria_87574_PTR_CH15_10-06-13_203-216.inddakaria_87574_PTR_CH15_10-06-13_203-216.indd 213213 66/19/2013/19/2013 4:45:494:45:49 PMPM GENERAL CRITICAL CARE: PATHOLOGY, PATHOPHYSIOLOGY, AND THERAPY

devices (e.g., FloTrac and LIDCO) derive cardiac output from arterial pressure waveform contour and power analysis, and include an analysis of the pulse pressure, SV, and sys- tolic pressure variabilities. When the calculated arterial pulse pressure variability reaches a particular magnitude (10%–13%), the manufacturer recommends rapid fl uid adminis- tration to improve cardiovascular instability (9,10).

14. The answer is C. Characteristics of the arterial pressure waveform are used to derive SV. Cardiac output is then calculated from CO = SV × heart rate. Heart rate is separately measured. The strength of myocardial systole depends on the preload (see Answer 11), and the systolic arterial pressure refl ects that contractility. Therefore, SV is proportional to the force of systolic contraction as assessed by the arterial waveform. This contractile force is also infl uenced by the impedance of the aorta and larger arterial vessels. The devices (e.g., FloTrac, LIDCO, and others) that utilize this technology and proprietary algorithms calculate CO from the equation: SV = (∫dp/dt)/Z where Z, aortic impedance; ∫dp/dt, integral of changing pressure over time during systole. Correlation with thermodilution methods for CO measurement is 0.88 to 0.91, but data are controversial among some patient groups wherein algorithms used in the devices may apply less well (9–12).

15. The answer is E. Substitution of the percentage oxygen saturation from central venous

blood (ScvO2) for the true mixed venous blood oxygen saturation (SvO2) from the pul- monary artery was initially proposed within the treatment protocol for septic patients in the emergency department (13). Review of this utilization among several patient groups with other diagnoses shows a variable correlation between the two measures (14–16).

Proponents of the substitution suggest that an ScvO2 above 70% indicates that ongoing

treatment is safe and likely meets tissue oxygen delivery needs. Utilization of the ScvO2 during titrated neurocritical care, however, remains poorly defi ned.

References

1. Oren-Grinberg A, Lerner AB, Talmor D. Echocardiography in the intensive care unit. In: Irwin RS, Rippe JM, eds. Intensive Care Medicine. 6th ed. Philadelphia, PA: Wolters Kluwer/ Lippincott Williams & Wilkins; 2008:289–302. 2. Kruse JA. Hemodynamic monitoring. In: Kruse JA, Fink MP, Carlson RW, eds. Saunders Manual of Critical Care. Philadelphia, PA: Saunders; 2003:774–777. 3. Cheatham ML, Block EFJ, Promes JT, et al. Shock: an overview. In: Irwin RS, Rippe JM, eds. Intensive Care Medicine. 6th ed. Philadelphia, PA: Wolters Kluwer/Lippincott Williams & Wilkins; 2008:1831–1842. 4. Nykamp D, Titak JA. Takotsubo cardiomyopathy, or broken-heart syndrome. Ann Pharmacother. 2010;44(3):590–593. 5. Barratt-Boyes BG, Wood EH. The oxygen saturation of blood in the venae cavae, right- heart chambers, and pulmonary vessels of healthy subjects. J Lab Clin Med. 1957;50(1): 93–106.

214

66485457-66485438 www.ketabpezeshki.com

ZZakaria_87574_PTR_CH15_10-06-13_203-216.inddakaria_87574_PTR_CH15_10-06-13_203-216.indd 214214 66/19/2013/19/2013 4:45:494:45:49 PMPM CARDIOVASCULAR PHYSIOLOGY: Answers

6. Ogunyankin KO. Assessment of left ventricular diastolic function: the power, possibilities, and pitfalls of echocardiographic imaging techniques. Can J Cardiol. 2011;27(3):311–318. 7. Mendoza DD, Codella NC, Wang Y, et al. Impact of diastolic dysfunction severity on global left ventricular volumetric fi lling—assessment by automated segmentation of rou- tine cine cardiovascular magnetic resonance. J Cardiovasc Magn Reson. 2010;12:46. 8. Starling EH. The Linacre lecture on the law of the heart given at Cambridge 1915. London: Longmans Green; 1918. 9. Powner DJ, Hergenroeder GW. Measurement of cardiac output during adult donor care. Prog Transplant. 2011;21(2):144–50; quiz 151. 10. Marik PE, Cavallazzi R, Vasu T, Hirani A. Dynamic changes in arterial waveform derived variables and fl uid responsiveness in mechanically ventilated patients: a systematic review of the literature. Crit Care Med. 2009;37(9):2642–2647. 11. de Waal EE, Wappler F, Buhre WF. Cardiac output monitoring. Curr Opin Anaesthesiol. 2009;22(1):71–77. 12. Cecconi M, Dawson D, Casaretti R, Grounds RM, Rhodes A. A prospective study of the accuracy and precision of continuous cardiac output monitoring devices as compared to intermittent thermodilution. Minerva Anestesiol. 2010;76(12):1010–1017. 13. Rivers E, Nguyen B, Havstad S, et al. Early goal-directed therapy in the treatment of severe sepsis and septic shock. N Engl J Med. 2001;345(19):1368–1377. 14. Powner DJ, Doshi PB. Central venous oxygen saturation monitoring: role in adult donor care? Prog Transplant. 2010;20(4):401–5; quiz 406. 15. Giraud R, Siegenthaler N, Gayet-Ageron A, Combescure C, Romand JA, Bendjelid K. ScvO(2) as a marker to defi ne fl uid responsiveness. J Trauma. 2011;70(4):802–807. 16. Ho KM, Harding R, Chamberlain J, Bulsara M. A comparison of central and mixed venous oxygen saturation in circulatory failure. J Cardiothorac Vasc Anesth. 2010;24(3):434–439.

215

66485457-66485438 www.ketabpezeshki.com

ZZakaria_87574_PTR_CH15_10-06-13_203-216.inddakaria_87574_PTR_CH15_10-06-13_203-216.indd 215215 66/19/2013/19/2013 4:45:494:45:49 PMPM 66485457-66485438 www.ketabpezeshki.com

ZZakaria_87574_PTR_CH15_10-06-13_203-216.inddakaria_87574_PTR_CH15_10-06-13_203-216.indd 216216 66/19/2013/19/2013 4:45:494:45:49 PMPM 16 Cardiovascular Diseases Jean Onwuchekwa Ekwenibe, Francisco Fuentes, Siddharth Mukerji, Husnu Evren Kaynak, Nicoleta Daraban, Charles Hebenstreit, and Ketan Koranne QUESTIONS

1. The FDA-approved dosage for dabigatran etexilate in patients with renal insuffi ciency is: A. 150 mg orally twice daily B. 110 mg orally twice daily C. 75 mg orally twice daily D. 150 mg orally once daily

2. A patient presents with right-sided weakness and aphasia. On subsequent workup, he is found to have a dissection of the ascending thoracic aorta, which is extending into the great arteries of the neck. What is the fi rst step in management of this patient? A. Immediate surgery for repair of the aortic dissection B. Chest x-ray (CXR) C. Medical management of BP and heart rate (HR) D. Cardiac enzymes

ANSWERS TO THIS SECTION CAN BE FOUND ON PAGE 226 217

66485457-66485438 www.ketabpezeshki.com

ZZakaria_87574_PTR_CH16_10-06-13_217-238.inddakaria_87574_PTR_CH16_10-06-13_217-238.indd 217217 66/19/2013/19/2013 8:47:288:47:28 PMPM GENERAL CRITICAL CARE: PATHOLOGY, PATHOPHYSIOLOGY, AND THERAPY

3. A 57-year-old man is admitted for progressive shortness of breath, chest pain, and palpita- tions. During his hospital course, he undergoes a diagnostic cardiac catheterization and his hemodynamic tracings are shown below. What is the most likely diagnosis for this patient? A. Valvular aortic stenosis B. Supravalvular aortic stenosis C. Hypertrophic obstructive cardiomyopathy (HOCM) D. Aortic coarctation

200 200 PVC 180 180 160 160 140 140 120 120 100 100 80 80 60 60 40 40 20 20 0 0

9:25:33 AM 9:25:34 AM 9:25:35 AM 9:25:36 AM 9:25:37 AM

4. A 48-year-old man with a history of hypertension and end-stage renal disease on perito- neal dialysis is hospitalized for acute onset ischemic stroke. Because onset of his symptoms occurred 10 hours prior to presentation in the ED, he is treated conservatively with oral aspirin. On the third day of admission, he develops acute onset shortness of breath, and a ventilation/perfusion (V/Q) scan confi rms a segmental pulmonary thromboembolus. He loses peripheral intravenous access, and attempts to reestablish it are unsuccessful. Peripheral blood draws are still possible. What method of anticoagulation is preferable to begin treatment for the pulmonary thromboembolism? A. Subcutaneous enoxaparin B. Subcutaneous unfractionated heparin C. Subcutaneous fondaparinux D. Oral warfarin

218

66485457-66485438 www.ketabpezeshki.com

ZZakaria_87574_PTR_CH16_10-06-13_217-238.inddakaria_87574_PTR_CH16_10-06-13_217-238.indd 218218 66/19/2013/19/2013 8:47:288:47:28 PMPM CARDIOVASCULAR DISEASES: Questions

5. A 58-year-old woman who suffers from a signifi cant history of alcohol and substance abuse is admitted to the ICU after she is discovered obtunded on the street by police. She is profoundly hypotensive and hypoxic. She is intubated for mechanical ven- tilation, and IVs are placed for fl uid resuscitation. Initial serum chemistries are sent to the laboratory and an ECG and chest x-ray (CXR) are completed. Her CXR is shown here. A Swan–Ganz catheter is placed to further assess her volume status, and reveals a pulmonary capillary wedge pressure (PCWP) of 10. An echocardiogram is also com- pleted, and the results are still pending. Which of the following diagnoses is the least likely to be the cause of the patient’s pulmonary edema? A. Aspiration pneumonia B. Septic shock C. Subarachnoid hemorrhage (SAH) D. Decompensated congestive HF

6. A 55-year-old man with chronic arterial hypertension for 16 years presented with sudden onset of severe crushing, substernal chest pain radiating to the back. Physical examination demonstrated a BP of 200/130 mmHg, with a heart rate of 84 beats/minute. Pulmonary and cardiac examination revealed no abnormalities. Electrocardiogram (ECG) showed no acute ST-T wave changes, and cardiac enzymes were negative. Computed tomography angiography (CTA) of the chest was performed and revealed the fi ndings shown here. Which of the following is the best approach for managing this patient’s BP? A. Nicardipine infusion to reduce the BP to a goal of 170/110 mmHg over 3 to 6 hours B. Nicardipine infusion to reduce the BP immediately below 105 mmHg C. IV labetalol to rapidly reduce the SBP below 105 mmHg D. IV labetalol to reduce the BP to a goal of 170/100 mmHg over 3 to 6 hours

7. The commonly occurring ECG changes noted in patients with an subarachnoid hemor- rhage (SAH) include all of the following except: A. ST-segment elevation B. ST-segment depression C. QTc interval prolongation D. PR segment prolongation

219

66485457-66485438 www.ketabpezeshki.com

ZZakaria_87574_PTR_CH16_10-06-13_217-238.inddakaria_87574_PTR_CH16_10-06-13_217-238.indd 219219 66/19/2013/19/2013 8:47:298:47:29 PMPM GENERAL CRITICAL CARE: PATHOLOGY, PATHOPHYSIOLOGY, AND THERAPY

8. A patient with a history of smoking and hypertension presents with an ischemic stroke. During the stroke workup, he is incidentally found to have a 5.7-cm ascending aortic aneurysm. What is the next step in management of his aortic disease? A. Surgical consultation for aneurysm repair B. Smoking cessation C. Lipid profi le optimization D. Blood pressure (BP) management

9. The following parameters are obtained after performing a cardiac catheterization on a 38-year-old woman who presents with shortness of breath: Ao saturation, 97%; PA satu- ration, 71%; hemoglobin, 14 g/dL; body surface area (BSA), 1.68 m2. What is the cardiac output of this patient using the Fick formula? A. 3.5 L/minute B. 4.2 L/minute C. 2 L/minute D. 3.9 L/minute E. Cannot be calculated with the given data

10. A 72-year-old man with a history of type II dia- betes mellitus and hypertension presents with hypoxia and right-sided weakness. Spiral CT scan of the chest reveals bilateral segmental pulmo- nary emboli, and MRI of the brain demonstrates a left middle cerebral artery occlusion. Echocardio- graphy reveals a large thrombus partially crossing a patent foramen ovale (PFO). Which of the follow- ing is not a risk factor traditionally associated with paradoxical embolization? A. Large size of PFO B. Presence of atrial septal aneurysm C. Prominent eustachian valve D. Mitral valve stenosis

11. All of the following are associated with neurogenic pulmonary edema (NPE) except: A. Presence of a CNS insult such as an subarachnoid hemorrhage (SAH), seizure, or cere- brovascular accident (CVA) B. Decreased pulmonary capillary permeability C. Normal left ventricular systolic function D. Increased sympathetic response after a central nervous system (CNS) event

220

66485457-66485438 www.ketabpezeshki.com

ZZakaria_87574_PTR_CH16_10-06-13_217-238.inddakaria_87574_PTR_CH16_10-06-13_217-238.indd 220220 66/19/2013/19/2013 8:47:298:47:29 PMPM CARDIOVASCULAR DISEASES: Questions

12. A 55-year-old woman presented to the ED complaining of worsening occipital headache and confusion. She was oriented to person but not to place or time. On arrival, her BP was 220/135 mmHg. On physical examination, she was confused. Papilledema was seen on fundoscopic examination. Laboratory studies demonstrated an elevated creatinine of 2.3 mg/dL. ECG revealed left ventricular hypertrophy by voltage criteria and nonspe- cifi c ST-T wave abnormalities in the lateral leads. CT scan of the head without contrast revealed diffuse bilateral white matter changes consistent with hypertensive encephal- opathy. Which of the following is the best next step in management? A. Reduction of the BP to 190/100 over 1 hour using nicardipine infusion B. Watchful observation over next 2 hours to determine whether the BP will spontane- ously decrease C. Rapid reduction in the BP to 160/100 using IV labetalol D. Gradual reduction of BP over 24 to 48 hours using oral captopril and long-acting nifedipine

13. Tall R wave on 12-lead ECG is noted in all the following conditions except: A. Duchenne muscular dystrophy B. Friedreich’s ataxia C. Limb girdle muscular dystrophy D. Facioscapulohumeral muscular dystrophy

14. What is the most important laboratory test for diagnosing acute pericarditis? A. 2D echocardiogram B. ECG C. Chest x-ray (CXR) D. Cardiac enzymes

15. Which of the following statements does not apply to catheter-based reperfusion therapy in acute myocardial infarction (MI) when performed by experienced operators? A. Primary stenting compared with angioplasty reduces mortality and recurrent infarction B. Primary angioplasty results in lower stroke rates than thrombolysis C. Stenting in patients with an acute MI decreases the need for subsequent target vessel revascularization D. Primary angioplasty results in higher coronary artery patency rates than thrombolysis E. Primary angioplasty results in lower mortality than thrombolysis

16. A 34-year-old woman with melanoma is admitted for mental status changes and is found to have multiple brain metastases. Restaging is performed and incidentally reveals multiple bilateral subsegmental pulmonary emboli. Physical examination is remark- able for left lower extremity pitting edema, and doppler ultrasound reveals a partially occlusive popliteal deep venous thrombosis. What is the best treatment for her venous thromboembolic disease? A. Inferior vena cava fi lter placement alone B. Inferior vena cava fi lter placement and thrombolytic therapy C. Dabigatran D. Systemic anticoagulation and systemic thrombolytic therapy

221

66485457-66485438 www.ketabpezeshki.com

ZZakaria_87574_PTR_CH16_10-06-13_217-238.inddakaria_87574_PTR_CH16_10-06-13_217-238.indd 221221 66/19/2013/19/2013 8:47:308:47:30 PMPM GENERAL CRITICAL CARE: PATHOLOGY, PATHOPHYSIOLOGY, AND THERAPY

17. Sleep-disordered breathing is common in patients with heart failure (HF). A. True B. False

18. A 72-year-old man with a history of hyperten- sion, diabetes mellitus, and hyperlipidemia pre- sented to the ED 10 hours after sudden onset of right arm and right leg weakness. His mental sta- tus was intact, and he was alert and oriented to time, place, and person. His BP on presentation was 210/130 mmHg with a mean arterial pres- sure (MAP) of 157 mmHg. Physical examination demonstrated 2/5 strength in the right upper and lower extremities and 5/5 strength on the left side. No papilledema was seen on fundoscopic exami- nation. Laboratory studies were normal. CT scan of head without contrast showed no evidence of acute hemorrhage. MRI of the brain without con- trast demonstrated the fi nding shown here. Which of the following is the best next step in managing this patient’s hypertension? A. Gradual reduction of MAP by 15% to 20% over 3 hours B. Rapid reduction of BP to less than 185/110 mmHg C. Gradual reduction of MAP by 15% to 20% over 24 hours D. Watchful observation over 2 to 3 hours to determine whether the BP spontaneously decreases

19. Idebenone is indicated in patients with Friedreich’s ataxia because: A. It decreases overall left ventricular mass B. It increases markers of oxidative damage, thus acting as a prognostic indicator for pro- gression of disease C. It has no direct effect on left ventricular function D. All of the above

20. What is the most sensitive physical fi nding that suggests cardiac tamponade? A. Systemic arterial hypotension B. Elevated jugular venous pressure C. Pulsus paradoxus D. Tachycardia

21. Which of the following cardiac biomarkers will provide information about prognosis and help in determining the patient’s possible infarct size? A. Creatinine kinase-MB fraction (CK-MB) B. Myoglobin C. Cardiac troponin T (cTnT) D. B-natriuretic peptide (BNP) E. Matrix metalloproteinase (MMP)

222

66485457-66485438 www.ketabpezeshki.com

ZZakaria_87574_PTR_CH16_10-06-13_217-238.inddakaria_87574_PTR_CH16_10-06-13_217-238.indd 222222 66/19/2013/19/2013 8:47:308:47:30 PMPM CARDIOVASCULAR DISEASES: Questions

22. A 49-year-old woman with multiple sclerosis is being treated for acute pulmonary throm- boembolism with IV heparin and warfarin. She has a previous history of prophylactic heparin use while inpatient. One day after initiation of therapy, her platelet count has fallen from 230,000 to 45,000. Physical examination reveals dusky areas on several digits as well as edema in the left arm and hand, which was not seen previously. What is the next immediate step in management? A. Discontinuation of heparin, administration of vitamin K and argatroban B. Discontinuation of heparin, administration of bivalirudin C. Discontinuation of heparin, administration of enoxaparin D. Discontinuation of heparin, continued administration of warfarin to goal INR 2.0 to 3.0

23. A healthy 40-year-old man with no medical history travels to east Africa to join a hiking expedition to trek to the top of Mount Kilimanjaro. The group ascends to 3,500 meters in 2 days. On the third day of the expedition, he experiences diffi culty breathing, head- ache, cough with pink frothy sputum, chest tightness, and congestion. One of the guides decides that it is no longer safe for him to continue the climb so together they begin their descent down the mountain. All of the following therapies are benefi cial in improving the patient’s symptoms immediately except: A. Nifedipine B. Lasix C. Supplemental oxygen D. Hyperbaric treatment E. Acetazolamide

24. A 60-year-old woman with a history of hypertension, diabetes mellitus, and hyperlipid- emia presented to the clinic with the complaint of pressure-like headache. Her BP was noted to be 180/120 mmHg. She did not have any altered mental status and denied chest pain or shortness of breath. Neurological examination revealed no motor, sensory, or cra- nial nerve defi cits. No papilledema was seen on fundoscopic examination. Which of the following is the most appropriate approach in managing this patient’s hypertension? A. Oral short-acting antihypertensives under observation B. Reduction of BP to 155/100 mmHg over 3 to 6 hours using nicardipine infusion C. Rapid reduction of SBP to less than 100 mmHg using IV labetalol D. Reduction of BP to 155/100 mmHg over 24 hours using nicardipine infusion

25. According to current guidelines, in patients with cardiovascular implantable electric devices (CIEDs) needing transcutaneous electrical nerve stimulation (TENS), the correct statement is: A. TENS units can now be safely used in all patients with devices implanted after 2009 B. Use of TENS units is not recommended because of possible electromagnetic interference C. TENS units can be safely used in all patients except in the area of the thoracic spine D. TENS units can be safely used in all patients if the frequency utilized is less than 30 Hz

223

66485457-66485438 www.ketabpezeshki.com

ZZakaria_87574_PTR_CH16_10-06-13_217-238.inddakaria_87574_PTR_CH16_10-06-13_217-238.indd 223223 66/19/2013/19/2013 8:47:308:47:30 PMPM GENERAL CRITICAL CARE: PATHOLOGY, PATHOPHYSIOLOGY, AND THERAPY

26. What is the most common form of pericardial infection? A. Viral infection B. Tuberculosis C. Bacterial infection D. Fungal infection

27. All the following situations are absolute contraindications to fi brinolytic therapy in acute myocardial infarction (MI) except: A. Suspected aortic dissection B. Any prior intracranial hemorrhage C. Known malignant intracranial neoplasm (primary or metastatic) D. Severe uncontrolled hypertension on presentation (SBP > 180 mmHg or diastolic blood pressure [DBP] > 110 mmHg) E. Signifi cant closed head or facial trauma within 3 months

28. Cardiogenic shock is defi ned by the presence of all of the following except: A. SBP < 80 mmHg for more than 30 minutes B. Decreased cardiac output resulting in decreased tissue perfusion C. Pulmonary arterial wedge pressure greater than 15 mmHg D. Cardiac index greater than 1.8 L/minute/m2

29. In patients being evaluated for cardiac resynchronization therapy, in addition to HF symptoms, the QRS duration on surface ECG should be at least: A. 100 milliseconds B. 110 milliseconds C. 115 milliseconds D. 120 milliseconds

30. What is the leading cause of cardiac tamponade in developed countries? A. Viral pericarditis B. Malignant pericardial effusion C. Radiation-induced pericardial disease D. Post-myocardial infarction (MI) pericarditis

31. A 58-year-old man is admitted to the coronary care unit after a diagnosis of inferior myo- cardial infarction (MI). The patient did not receive any thrombolytic or catheter-based reperfusion therapy because he was not in the window for reperfusion. The initial ECG showed 1-mm ST elevation with ST depressions and pathologic Q waves in II, III, and aVF. Twenty-four hours after admission, the patient develops mild dyspnea and the chest x-ray (CXR) shows pulmonary vascular redistribution. A faint late systolic murmur is heard at the apex. What is the probable cause of the murmur? A. Infarcted posterior papillary muscle B. Tricuspid regurgitation C. Ventricular septal defect D. Aortic stenosis E. Ruptured posterior papillary muscle

224

66485457-66485438 www.ketabpezeshki.com

ZZakaria_87574_PTR_CH16_10-06-13_217-238.inddakaria_87574_PTR_CH16_10-06-13_217-238.indd 224224 66/19/2013/19/2013 8:47:308:47:30 PMPM CARDIOVASCULAR DISEASES: Questions

32. The following have been documented as adverse effects in more than 1% of patients tak- ing amiodarone except: A. Peripheral neuropathy B. Photosensitivity C. Worsening of ejection fraction (EF) by less than 5% D. Halo vision

33. Ventricular tachycardia (VT) ablation should be considered in all of the following sce- narios except: A. A 23-year-old man with no evidence of structural heart disease and frequent episodes of syncope related to VT B. A 67-year-old woman with coronary artery disease (CAD) and depressed systolic function on optimal medical therapy with persistent VT C. A 56-year-old man on dofetilide therapy with recurrent episodes of VT D. A 47-year-old man with prior history of CAD, no evidence of systolic dysfunction, and stage I prostate cancer with frequent episodes of symptomatic VT

34. The gold standard for diagnosis of neurocardiogenic syncope is: A. Tilt-table testing B. Implantable loop recorder C. Electrophysiological study D. None of the above

225

66485457-66485438 www.ketabpezeshki.com

ZZakaria_87574_PTR_CH16_10-06-13_217-238.inddakaria_87574_PTR_CH16_10-06-13_217-238.indd 225225 66/19/2013/19/2013 8:47:308:47:30 PMPM 16

ANSWERS

1. The answer is C. Dabigatran etexilate is a prodrug that is rapidly converted to the active direct thrombin (factor IIa) inhibitor dabigatran. This conversion is independent of cyto- chrome P-450, making drug–drug interactions less likely. It is predominantly excreted via a renal pathway. Dabigatran was recently evaluated in a large, open-label, randomized trial (RE-LY) in which it was compared with warfarin in 18,113 patients with nonval- vular AF. There was no difference in mortality with dabigatran compared to warfarin. Dabigatran appeared to be noninferior to warfarin with respect to the primary effi cacy outcome of stroke or systemic embolism. A dose of 150 mg twice daily was approved for patients with a creatinine clearance more than 30 mL/minute, whereas in patients with severe renal insuffi ciency (creatinine clearance 15–30 mL/minute) the approved dose is 75 mg twice daily. The 110 mg twice-daily dose used in the RE-LY trial did not receive FDA approval (1).

2. The answer is C. Medical management of BP and heart rate is the fi rst step in manage- ment of aortic dissection. In the absence of contraindications, IV β blockade should be initiated and titrated to a target heart rate of 60 beats/minute or less. In patients with clear contraindications to β blockade, nondihydropyridine calcium channel–blocking agents should be used as an alternative for rate control. If the SBP remains greater than 120 mmHg after adequate heart rate control has been obtained, then angiotensin- converting enzyme inhibitors and/or other vasodilators should be administered intra- venously to further reduce BP while maintaining adequate end-organ perfusion. Beta- blockers should be used cautiously in the setting of acute aortic regurgitation because they will block the compensatory tachycardia. Urgent surgical consultation should be obtained for all patients diagnosed with thoracic aortic dissection regardless of the ana- tomic location (ascending vs. descending) as soon as the diagnosis is made or highly suspected (2).

226

66485457-66485438 www.ketabpezeshki.com

ZZakaria_87574_PTR_CH16_10-06-13_217-238.inddakaria_87574_PTR_CH16_10-06-13_217-238.indd 226226 66/19/2013/19/2013 8:47:318:47:31 PMPM CARDIOVASCULAR DISEASES: Answers

3. The answer is C. The following tracing demonstrates a premature ventricular complex (PVC), and during the beat following the PVC, there is an increase in the gradient between the left ventricle and the aorta, as well as a decrease in the aortic systolic pressure. This is called the Brockenbrough–Braunwald–Morrow sign and signifi es dynamic outfl ow tract obstruction. In aortic stenosis and fi xed obstruction, in the beat following the PVC, there is an increase in the aortic pressure, whereas in dynamic obstruction, there is a decrease in the aortic pressure and an increase in the left ventricular pressure. This is seen in HOCM and can be observed during physical examination as well (3).

4. The answer is B. Although subcutaneous enoxaparin can be dose adjusted for glomeru- lar fi ltration rates less than 30, its longer half-life compared with unfractionated hepa- rin presents an increased risk for bleeding complications in renal failure. Subcutaneous unfractionated heparin can be monitored by activated partial thromboplastin time (aPTT) in a similar fashion to intravenous administration. Subcutaneous fondaparinux is con- traindicated in renal failure. Oral warfarin cannot be used alone in initial management of pulmonary embolism (4).

5. The answer is D. The distinction between cardiogenic and noncardiogenic pulmonary edema can be diffi cult to determine; however, the diagnosis is important because treat- ment is guided by the underlying pathophysiology. Cardiogenic pulmonary edema occurs as a result of an increase in pulmonary capillary hydrostatic pressure, which causes fl uid extravasation into the interstitial space due to changes in oncotic pressure. Noncardiogenic pulmonary edema, in contrast, is a result of increased alveolar–capillary membrane per- meability seen in disease states such as acute respiratory distress syndrome. A Swan–Ganz catheter can be helpful in distinguishing between the two entities. The PCWP refl ects fi lling pressures on the left side of the heart and indirectly intravascular volume status. Since the PCWP is normal in this patient, decompensated congestive HF is the least likely etiology. The remaining choices are examples of noncardiogenic pulmonary edema and therefore could be present in the patient (5).

6. The answer is C. This patient presents with hypertensive emergency. The CTA of the chest on page 219 demonstrates a descending aortic dissection. The appropriate manage- ment goal for BP in aortic dissection is to rapidly reduce the SBP below 105 mmHg. Either intravenous labetalol or a combination of esmolol and nicardipine are the drugs of choice. Beta-blockers help to control the heart rate and reduce the shearing stress on the aorta, and therefore should be used. Nicardipine infusion alone is not the appropriate choice, as it increases the shearing stress on the aorta. Intravenous labetalol aiming at 15% to 25% reduction in BP would be appropriate in neurological emergencies, but in aortic dissec- tion, the BP should be reduced rapidly (6).

7. The answer is D. ECG changes associated with an SAH primarily refl ect repolarization abnormalities involving the ST segment, T wave, U wave, and QTc interval. Because of the combination of ST-segment elevation or depression and abnormal T-wave morphology, myocardial ischemia or infarction is often suspected in patients with SAH. Arrhythmias are a relatively common occurrence as well. Factors that may infl uence the development of arrhythmias in patients with SAH include cerebral vasospasm, hypoxia, electrolyte imbalance, and sudden increase in intracranial pressure triggering a sympathetic or vagal

227

66485457-66485438 www.ketabpezeshki.com

ZZakaria_87574_PTR_CH16_10-06-13_217-238.inddakaria_87574_PTR_CH16_10-06-13_217-238.indd 227227 66/19/2013/19/2013 8:47:318:47:31 PMPM GENERAL CRITICAL CARE: PATHOLOGY, PATHOPHYSIOLOGY, AND THERAPY

discharge due to compression of brain structures. Another well-recognized entity is the T-wave abnormality. The pattern of broad, slurred, inverted T waves associated with long QTc intervals is commonly termed cerebral, neurogenic, or giant T waves. The results have indicated that ECG abnormalities are not a signifi cant predictor of mortality. To date, patients’ outcomes have not been studied in a prospective investigation that includes a large sample size. Whether the observed ECG changes are transient or permanent is unclear (7).

8. The answer is A. Surgical consultation is warranted in all asymptomatic patients with an ascending aortic or aortic sinus of Valsalva diameter of 5.5 cm or greater. Patients with an aneurysm growth rate of more than 0.5 cm/year in the aorta that is less than 5.5 cm in diameter should also be considered for surgery. Stringent control of hypertension, lipid profi le optimization, smoking cessation, and other atherosclerosis risk-reduction measures should be instituted for patients with small aneurysms not requiring surgery, as well as for patients who are not considered surgical or stent graft candidates (2).

9. The answer is B. Cardiac output can be calculated using the Fick formula as demon- strated here:

[O2 consumption (mL/minute)]/[(AVO2 difference (mL O2/100 mL of blood) × 10)] The oxygen consumption can be taken as 125 mL/minute/m2 in women, 110 mL/minute/ m2 in the elderly, and 140 mL/minute/m2 in men. Since the patient’s BSA is 1.68 m2, the total oxygen consumption is 210 mL/minute. 2 2 O2 consumption: 125 mL/minute/m × 1.68 m = 210 mL/minute

The AVO2 difference is the amount of oxygen taken up by the tissues in muscle. It is the arterial oxygen content minus mixed venous oxygen content multiplied by 1.36 (this

number represents the fact that each gram of hemoglobin can carry 1.36 mL of O2) and by the hemoglobin concentration.

AVO 2 difference: (AO2 − VO2) × 1.36 × hemoglobin

AVO 2 difference: (0.97 − 0.71) × 1.36 × 14 = 4.95 mL O2/100 mL of blood Placing all of the numbers back into the original formula: Cardiac output (CO) = 210/(4.95 × 10) = 4.24 L/minute (8)

10. The answer is D. Risk factors thought to be associated with increased risk of paradoxical embolism include a large PFO, atrial septal aneurysm, prominent eustachian valve, and conditions that cause elevated right atrial pressure, such as pulmonary embolism or tri- cuspid regurgitation. Conditions causing increased left atrial pressure are more likely to decrease shunting across the PFO and decrease risk of paradoxical embolism (9).

11. The answer is B. Several CNS insults can be complicated by the development of acute pulmonary edema. This occurrence has been termed neurogenic pulmonary edema (NPE) and is not fully understood. NPE is a form of noncardiogenic pulmonary edema, and these patients are often found to have normal left ventricular systolic function. CNS insults associated with NPE include SAH, seizures, brain tumors, and CVAs. Two main theories about the mechanism of NPE are postulated in the literature: (a) the presence of increased permeability of the pulmonary capillaries causing exudative edema and (b) a

228

66485457-66485438 www.ketabpezeshki.com

ZZakaria_87574_PTR_CH16_10-06-13_217-238.inddakaria_87574_PTR_CH16_10-06-13_217-238.indd 228228 66/19/2013/19/2013 8:47:318:47:31 PMPM CARDIOVASCULAR DISEASES: Answers

sympathetic discharge after a CNS event that produces hydrostatic edema. Treatment for NPE is supportive (10,11).

12. The answer is A. Hypertensive emergencies are characterized by severe elevations in BP (>180/120 mmHg) complicated by evidence of impending or progressive end organ dysfunction. Hypertensive encephalopathy is one such example. The initial goal of ther- apy in hypertensive emergencies is to reduce the mean arterial BP by 15% to 25% over 1 to 2 hours. Rapid reduction in BP may precipitate renal, cerebral, or coronary ischemia. These patients must be managed in the ICU using IV antihypertensives for appropriate titration of BP, and therefore use of oral antihypertensives is inappropriate (12).

13. The answer is D. ECG abnormalities can be noted in up to 90% of patients with Becker and Duchenne muscular dystrophy. Tall R waves and an increased RS amplitude in V1 with deep narrow Q waves in the left precordial leads, are a characteristic ECG pat- tern of the posterolateral left ventricular involvement. Incomplete right bundle branch block may also be noted, suggesting right ventricular involvement in these patients. Friedreich’s ataxia is commonly associated with concentric hypertrophic cardiomyopa- thy and, at times, asymmetric septal hypertrophy. Up to 95% of these patients manifest ECG abnormalities. Surprisingly, ECG manifestations do not always include left ven- tricular hypertrophy, although echo demonstrates this. Widespread T-wave abnormali- ties are noted with tall R waves in all leads. Limb-girdle muscular dystrophy is a sarco- glycanopathy and is associated with cardiomyopathy. ECG changes are consistent with those seen in Becker and Duchenne muscular dystrophy. Cardiac involvement is rare in facioscapulohumeral muscular dystrophy. Specifi c cardiac monitoring or treatment has not yet been described (13).

14. The answer is B. ECG is the most important laboratory test for diagnosing acute peri- carditis. A diagnosis of acute pericarditis should be reserved for patients with an audible pericardial friction rub or chest pain with typical ECG fi ndings. The classic presentation is diffuse, concave upward ST-segment elevation and PR segment depression. The distinc- tion between acute pericarditis and transmural ischemia is usually not diffi cult because of more extensive lead involvement in acute pericarditis and the presence of reciprocal ST-segment depression in acute ischemia. An echocardiogram and CXR would be useful in identifying a pericardial effusion or other causes of chest pain, but have otherwise no role in diagnosing pericarditis. A signifi cant fraction of patients with pericarditis have elevated creatinine kinase-MB fraction or troponin I values, which suggests concomitant myocarditis (14).

15. The answer is A. Primary angioplasty in patients with an acute MI, when performed by experienced operators, has shown in large registries and randomized trials to result in higher patency rates (93%–98% vs. 54%) and lower 30-day mortality rates (5% vs. 7%) than thrombolytic therapy. An additional advantage of primary angioplasty over throm- bolysis is a signifi cant reduction in bleeding complications and strokes. The superior- ity of primary angioplasty to thrombolytic therapy is particularly evident in higher-risk patients such as diabetics and the elderly. The use of primary stenting versus primary angioplasty does not result in a mortality advantage, but correlates well with a reduced need for subsequent target vessel revascularization. A meta-analysis of trials comparing

229

66485457-66485438 www.ketabpezeshki.com

ZZakaria_87574_PTR_CH16_10-06-13_217-238.inddakaria_87574_PTR_CH16_10-06-13_217-238.indd 229229 66/19/2013/19/2013 8:47:318:47:31 PMPM GENERAL CRITICAL CARE: PATHOLOGY, PATHOPHYSIOLOGY, AND THERAPY

primary stenting with angioplasty found little difference in the rates of death (3.7% vs. 3.6%) or recurrent MI (2.1% vs. 2.9%) (15).

16. The answer is A. Certain high-risk intracranial carcinomas such as melanoma, renal cell cancer, thyroid cancer, and choriocarcinoma are strict contraindications to sys- temic anticoagulation and systemic thrombolysis. Inferior vena cava fi lter placement is indicated for treatment of pulmonary embolism when anticoagulation is not possible; although anticoagulation in some cases is controversial and should be started once a reversible contraindication has resolved (16).

17. The answer is A. Cheyne–Stokes breathing is a central sleep apnea that is common in patients with congestive HF and is seen in approximately 40% of patients. Obstructive sleep apnea is seen in 10% of patients. These patients have an increased risk of mortality and cardiac transplantation. Central sleep apneas cause neurohumoral activation (espe- cially norepinephrine), and this results in elevations in nocturnal BP and heart rate (17).

18. The answer is D. This patient presented with an acute ischemic stroke. MRI demon- strates a left subcortical ischemic infarct. In such patients, perfusion pressure distal to the obstructed vessel is low, and compensatory vasodilation occurs to maintain adequate blood fl ow. A high systemic pressure is required to maintain blood fl ow in these dilated vessels. Therefore, in patients with ischemic stroke, BP should be carefully observed for 1 to 2 hours to see whether it will spontaneously decrease. A persistent elevation in MAP above 130 mmHg or SBP above 220 mmHg should be carefully treated with an aim to lower the MAP by 15% to 20%. Rapid reduction in BP may compromise the cerebral per- fusion and worsen ischemia (18).

19. The answer is A. Idebenone is a free-radical scavenger from the quinone family that is also a synthetic analogue of Coenzyme Q10. It has been studied in an unblinded, noncon- trolled trial and reportedly decreases the left ventricular mass in patients with Friedreich’s ataxia signifi cantly. However, no characteristics were identifi ed that separated responders from nonresponders. It is currently being studied in patients with Duchenne muscular dystrophy and Alzheimer disease. In patients with low ejection fraction (EF), an improve- ment was noted after initiation of idebenone therapy. It decreases markers of oxidative damage, without a clear improvement in neurological outcome. Thus, it reduces the rate of cardiac function deterioration without actually halting the progression of ataxia (19).

20. The answer is C. Pulsus paradoxus is the most sensitive, although not specifi c, physi- cal fi nding that suggests cardiac tamponade. Pulsus paradoxus represents a decrease in the systolic arterial pressure of more than 10 mmHg with inspiration. Systemic arterial hypotension, elevated jugular venous pressure, and tachycardia are physical fi ndings associated with cardiac tamponade, but are neither sensitive nor specifi c (20).

21. The answer is C. Data support the use of cardiac biomarkers to estimate prognosis and infarct size. The GUSTO-III trial of over 12,000 patients showed that 16% of patients with an elevated cardiac TnT had died within 30 days compared with 6% of those without an enzyme leak. A recent meta-analysis of patients with non-ST-segment elevation myo- cardial infarction (STEMI) suggested that patients with elevated cardiac troponins had a

230

66485457-66485438 www.ketabpezeshki.com

ZZakaria_87574_PTR_CH16_10-06-13_217-238.inddakaria_87574_PTR_CH16_10-06-13_217-238.indd 230230 66/19/2013/19/2013 8:47:318:47:31 PMPM CARDIOVASCULAR DISEASES: Answers

greater than 3-fold increase of death compared with those with a normal value. Troponins are more specifi c to cardiac muscle, whereas CK-MB can rise in skeletal muscle damage as well. One-third of patients with acute myocardial infarction (MI) have an elevated cTn despite a normal CK-MB. Myoglobin, although fast in detecting damage, is not sensitive. BNP and MMP are newer markers correlating with muscle damage and plaque rupture, but will need further studies for determining their effects on prognosis (21).

22. The answer is A. Onset of heparin-induced thrombocytopenia (HIT) is typically within 24 hours of heparin administration when there is a history of previous heparin exposure, but is otherwise seen in 5 to 14 days when there is no history of previous heparin expo- sure. Current recommendations from the American College of Chest Physicians (ACCP) for HIT include discontinuation of heparin, avoidance of low-molecular-weight heparin, and immediate initiation of an alternative parenteral anticoagulant such as bivalirudin or argatroban. When warfarin has been coadministered with heparin, the ACCP recom- mends administration of vitamin K to prevent potential venous limb gangrene associated with protein C defi ciency (22).

23. The answer is B. The patient is suffering from high-altitude pulmonary edema (HAPE), which is a form of noncardiogenic pulmonary edema. HAPE can result from a rapid ascension in altitude without proper acclimatization. The mainstay of treatment is descent for anything other than mild symptoms. Oxygen is life saving and should be given at 4 L/minute for 4 to 6 hours. In addition to oxygen, nifedipine, acetazolamide, and portable hyperbaric chambers are all benefi cial therapies. Nifedipine is thought to work by dilat- ing the pulmonary vasculature, reducing hydrostatic pressures and subsequent edema in the lungs. Acetazolamide is shown to accelerate acclimatization and acts as a stimulant to induce breathing. Portable hyperbaric chambers are widely used among climbing expe- ditions and quickly fl ush carbon dioxide from the system. Diuretics are not advised, or should be used with substantial caution in patients with HAPE since many are already depleted intravascularly and can clinically deteriorate (23,24).

24. The answer is A. This patient has hypertensive urgency. Hypertensive urgency is severe elevation of BP (>180/120 mmHg) without any evidence of end organ damage. This patient’s only symptom is headache; there is no evidence of papilledema and no evidence of cardiac, neurological, or renal complications. In general, hypertensive urgency can be managed using oral antihypertensive agents in an observation setting or outpatient set- ting with close follow-up. Excessive and rapid reduction in BP should be avoided to pre- vent hypotension and compromising the cerebral perfusion (25,26).

25. The answer is B. TENS is the use of electric current produced by a device to stimulate the nerves for therapeutic purposes. It covers the complete range of transcutaneously applied currents used for nerve excitation, although more specifi cally the unit encompasses a stimulator that produces pulses to treat pain. It is usually connected to the skin using two or more electrodes. A typical battery-operated TENS unit is able to modulate pulse width, frequency, and intensity. Generally TENS is applied at high frequency (>50 Hz) with an intensity below motor contraction (sensory intensity) or at low frequency (<10 Hz) with an intensity that produces motor contraction. According to the Heart Rhythm Society (HRS)/American Society of Anesthesiologists (ASA) Expert Consensus Statement on the

231

66485457-66485438 www.ketabpezeshki.com

ZZakaria_87574_PTR_CH16_10-06-13_217-238.inddakaria_87574_PTR_CH16_10-06-13_217-238.indd 231231 66/19/2013/19/2013 8:47:318:47:31 PMPM GENERAL CRITICAL CARE: PATHOLOGY, PATHOPHYSIOLOGY, AND THERAPY

Perioperative Management of Patients with Implantable Defi brillators, Pacemakers and Arrhythmia Monitors: Facilities and Patient Management, the use of TENS units is not recommended in patients with CIEDs. TENS can interfere with pacemaker and implant- able cardioverter-defi brillator (ICD) function. Adverse responses include inhibition of pacing (or triggering noise reversion mode) and inappropriate ICD therapy due to mis- interpreted electrical noise. The transcutaneous impulses could also be misinterpreted as inappropriate supraventricular arrhythmia in atrial tachycardia devices. High-frequency stimulation (>30 Hz) should be maintained at all times. TENS units should be avoided in the thoracic spine, cervical spine, shoulder, upper lumbar, and chest areas because of the proximity of the ICD or pacemaker (PM) and lead system. These recommendations generally extend to spinal cord stimulators as well (27).

26. The answer is A. Viral pericarditis is the most common cause of pericardial infec- tion. Echoviruses and coxsackie viruses are most often involved. The most defi nitive way to diagnose viral pericarditis is by detection of DNA by polymeras chain reaction (PCR) or in situ hybridization in pericardial fl uid or tissue. This is rarely necessary, as it will not change prognosis or course of treatment. Tuberculous pericarditis remains a major problem in the developing world and in immunocompromised patients. Bacterial pericarditis is usually caused by direct extension from empyema and pneu- monia. Hematogenous spread during bacteremia and contiguous spread after thoracic surgery or trauma are also important. Fungal infections can rarely cause pericarditis. Histoplasmosis is the most common cause in endemic areas. Immunocompromised patients are at increased risk (28).

27. The answer is D. Severe uncontrolled hypertension on presentation (SBP > 180 mmHg or DBP > 110 mmHg) is a relative contraindication for fi brinolytic therapy; however, it could be an absolute contraindication in low-risk patients with MI. Lowering the BP to levels less than 180/110 mmHg will enable the physician to administer fi brinolytic ther- apy, but catheter-based reperfusion methods would be preferred if available. All the other choices are absolute contraindications to fi brinolytic therapy, and in addition to those mentioned previously, known structural vascular lesion (e.g., V malformation), ischemic stroke within 3 months except acute ischemic stroke within 3 hours, active bleeding, or bleeding diathesis are also contraindications to fi brinolytic therapy (29).

28. The answer is D. Cardiogenic shock results in a low cardiac index. The cardiac index is used to measure the cardiac output, or the amount of blood pumped out of the left ventri- cle each minute. In cardiogenic shock, the cardiac index (CI) is less than 1.8 L/minute/m2. The cardiac index is derived by dividing the patient’s CO by the patient’s body surface area (BSA). CI = CO/BSA CO is calculated by multiplying the patient’s stroke volume (SV) by the heart rate (HR). Therefore, we can insert these entities into the previous equation. CI = (SV × HR)/BSA Normal values for the cardiac index range from 2.6 to 4.2 L/minute/m2. The remaining answer choices all defi ne the presence of cardiac shock and provide clinical clues that the patient is unstable and needs additional hemodynamic support (30).

232

66485457-66485438 www.ketabpezeshki.com

ZZakaria_87574_PTR_CH16_10-06-13_217-238.inddakaria_87574_PTR_CH16_10-06-13_217-238.indd 232232 66/19/2013/19/2013 8:47:318:47:31 PMPM CARDIOVASCULAR DISEASES: Answers

29. The answer is D. Approximately 15% to 30% of all patients with HF and moderate to severe symptoms have inter- and intraventricular conduction delays with QRS duration greater than 120 milliseconds. This results in mechanical dyssynchrony of right and left ventricular contractions. Furthermore, prolonged conduction has been associated with adverse outcomes. In a large study conducted in Italy involving more than 5,500 patients, a left bundle branch block (LBBB) was associated with an increased 1-year mortality from any cause (hazard ratio, 1.70; 95% confi dence interval, 1.41–2.05). LBBB was also associated with an increased 1-year mortality rate from sudden death (hazard ratio, 1.58; 95% confi dence interval, 1.21–2.06). Multivariate analysis showed that this increased risk of death due to LBBB was still signifi cant even after adjusting for age, underlying car- diac disease, other indicators of HF severity, and prescription of angiotensin- converting enzyme inhibitors and β-blockers. A substudy analysis from the vesnarinone study (VEST) assessed the relationship between QRS duration and mortality. In this analysis, 3,654 resting, baseline ECGs of patients with NYHA Class II to IV HF were digitally scanned. Age, creatinine, LV ejection fraction (LVEF), heart rate, and QRS duration were found to be independent predictors of mortality (p < .0001). Patients with wider QRS durations (>200 milliseconds) had a 5 times greater risk of death than those with narrow QRS durations (<90 milliseconds). On the basis of this fi nding, the authors concluded that the resting ECG is a powerful, accessible, and inexpensive marker of prognosis in dilated cardiomyopathy (31).

30. The answer is B. Malignant pericardial disease is the most common cause of cardiac tamponade in developed countries. Lung carcinoma is the most common, accounting for about 40% of malignant effusions; breast carcinoma and lymphomas are responsible for about another 40%. In most cancer patients with effusions, it is important that metastatic involvement of the pericardium be confi rmed by identifi cation of malignant cells or tumor markers in pericardial fl uid (32,33).

31. The answer is A. The posterior papillary muscle is more susceptible to ischemia than the anterior papillary muscle because of the nature of vascularization. The pos- terior muscle has one blood supply (posterior descending branch of the right coro- nary artery), whereas the anterior papillary muscle has dual blood supply (diagonal branches of the left anterior descending artery and marginal branches of the circum- fl ex artery). The clinical picture in Question 31 does not fi t a ruptured papillary muscle scenario because the ruptured papillary muscle would cause a graver situation result- ing in pulmonary edema. Ventricular septal defect is also a mechanical complication of MI; however, the physical examination fi ndings are not compatible, and a sternal holosystolic murmur would be heard. Tricuspid regurgitation and aortic stenosis are not complications of MI (34).

32. The answer is C. Amiodarone is a complex pharmacological agent with multiple adverse effects on multiple organ systems. Owing to its prolonged half-life (~100 days), organ tox- icity is potentially more severe and more diffi cult to manage. The more common effects include a decrease in the diffusing capacity of the lung for carbon monoxide (DLCo), interstitial pneumonitis, thyroid abnormalities, and photophobia, which can be seen in 15% to 60% of patients. Neurological effects are typically dose dependent. These include

233

66485457-66485438 www.ketabpezeshki.com

ZZakaria_87574_PTR_CH16_10-06-13_217-238.inddakaria_87574_PTR_CH16_10-06-13_217-238.indd 233233 66/19/2013/19/2013 8:47:318:47:31 PMPM GENERAL CRITICAL CARE: PATHOLOGY, PATHOPHYSIOLOGY, AND THERAPY

ataxia, tremors, neuropathy, and paresthesias, with an incidence of 3% to 30%. Adjusting the dose results in resolution of symptoms. Skin effects include photosensitivity and dis- coloration, which may occur in 75% of patients. These can be managed with reassurance and sun block. Ocular effects such as halo vision and optic neuritis can be noted in up to 5% of cases, with incidence of photophobia being as high as 80%. Reassurance is rec- ommended, although the medication needs to be discontinued if optic neuritis occurs. Adverse cardiac reactions are uncommon. Although prolongation of QT is expected, tosades de pointes is noted in less than 1% of patients. Ventricular systolic function is not compromised (35).

33. The answer is C. Selection of catheter ablation for an individual patient should con- sider risks and benefi ts that are determined by patient characteristics, as well as the availability of appropriate facilities with technical expertise. In patients with struc- tural heart disease, episodes of sustained VT are a marker for increased mortality; with reduced quality of life in patients who have implanted defi brillators and structural heart disease. Antiarrhythmic medications can reduce the frequency of implantable cardioverter-defi brillator (ICD) therapies, but have disappointing effi cacy and side effects. In the past, ablation was often not considered until pharmacological options had been exhausted, often after the patient had suffered substantial morbidity from recurrent episodes of VT and ICD shocks. However, since the release of data of the SMASH-VT trial, the general approach is changing. Contraindications to this proce- dure include: i. Presence of a mobile ventricular thrombus (epicardial ablation may be considered) ii. Asymptomatic premature ventricular complex (PVC) and/or nonsustained VT that are not suspected of causing or contributing to ventricular dysfunction iii. VT due to transient, reversible causes, such as acute ischemia, hyperkalemia, or drug- induced torsade de pointes (TdP). In our patient, it is imperative to rule out drug- induced TdP before considering ablation (36).

34. The answer is D. Neurocardiogenic (vasovagal) syncope is the most common of a group of refl ex (neurally mediated) syncopes, characterized by a sudden failure of the autonomic nervous system to maintain blood pressure (BP) and sometimes heart rate at a level suffi cient to maintain cerebral perfusion and consciousness. A detailed his- tory and physical examination are central to the diagnosis. In the absence of another identifi able cause, a compatible history is often suffi cient to make the diagnosis of neu- rocardiogenic syncope. Further testing is warranted if the diagnosis remains uncer- tain. Tilt-table testing is used to aid in the diagnosis. Specifi city is about 90%, with uncertain sensitivity. Moreover, the reproducibility of the test (over time) is lower for an initially positive response, making this test not completely reliable. Implantable loop recorders are small recording devices that are placed in a subcutaneous pocket and can store about 45 minutes of retrospective ECG recording. A diagnostic yield of 25% to 40% has been reported with the use of the device during a period of 8 to 10 months. The diagnostic effi cacy of EPS to determine the cause of syncope is highly dependent on the degree of suspicion for the abnormality. Positive results occurred predominantly in patients with structural heart disease, as in heart failure (HF). There is no gold standard for the diagnosis (37).

234

66485457-66485438 www.ketabpezeshki.com

ZZakaria_87574_PTR_CH16_10-06-13_217-238.inddakaria_87574_PTR_CH16_10-06-13_217-238.indd 234234 66/19/2013/19/2013 8:47:318:47:31 PMPM CARDIOVASCULAR DISEASES: Answers

References

1. Wann LS, Curtis AB, Ellenbogen KA, et al. 2011 ACCF/AHA/HRS focused update on the management of patients with atrial fi brillation (update on dabigatran). A report of the American College of Cardiology Foundation/American Heart Association Task Force on Practice Guidelines. Heart Rhythm. 2011;8(3):e1–e8. 2. Hiratzka LF, Bakris GL, Beckman JA, et al. 2010 ACCF/AHA guidelines for the diagnosis and management of patients with thoracic aortic disease: executive summary. Circulation. 2010;121:1544–1579. 3. Libby P, Bonow RO, Mann DL, et al. Braunwald’s Heart Disease: A Textbook of Cardiovascular Medicine. 8th ed. Philadelphia, PA: Saunders Elsevier; 2008:1766–1767. 4. Kearon C, Kahn SR, Agnelli G, Goldhaber S, Raskob GE, Comerota AJ. Antithrombotic therapy for venous thromboembolic disease: American College of Chest Physicians evidence-based clinical practice guidelines (8th edition). Chest. 2008;133(6 Suppl): 454S–545S. 5. Ware LB, Matthay MA. Clinical practice. Acute pulmonary edema. N Engl J Med. 2005;353(26):2788–2796. 6. Cornfeld D, Israel G, Detroy E, Bokhari J, Mojibian H. Impact of adaptive statis- tical iterative reconstruction (ASIR) on radiation dose and image quality in aor- tic dissection studies: a qualitative and quantitative analysis. AJR Am J Roentgenol. 2011;196(3):W336–W340. 7. Libby P, Bonow RO, Mann DL, et al. Braunwald’s Heart Disease: A Textbook of Cardiovascular Medicine. 8th ed. Philadelphia, PA: Saunders Elsevier; 2008:2149–2150. 8. Willerson JT, Cohn JN, Wellens HJJ, et al. Cardiovascular Medicine. 3rd ed. New York, NY: Springer-Verlag; 2007:464. 9. Homma S, Sacco RL. Patent foramen ovale and stroke. Circulation. 2005;112(7): 1063–1072. 10. Bruder N, Rabinstein A. Cardiovascular and pulmonary complications of aneurysmal subarachnoid hemorrhage. Neurocrit Care. 2011;15(2):257–269. 11. Fein IA, Rackow EC. Neurogenic pulmonary edema. Chest. 1982;81(3):318–320. 12. Adams HP Jr, del Zoppo G, Alberts MJ, et al. Guidelines for the early management of adults with ischemic stroke: a guideline from the American Heart Association/American Stroke Association Stroke Council, Clinical Cardiology Council, Cardiovascular Radiology and Intervention Council, and the Atherosclerotic Peripheral Vascular Disease and Quality of Care Outcomes in Research Interdisciplinary Working Groups: The American Academy of Neurology affi rms the value of this guideline as an educa- tional tool for neurologists. Circulation. 2007;115(20):e478–e534. 13. Libby P, Bonow RO, Mann DL, et al. Braunwald’s Heart Disease: A Textbook of Cardiovascular Medicine. 8th ed. Philadelphia, PA: Saunders/Elsevier; 2008:2135–2142. 14. Lange RA, Hillis LD. Clinical practice. Acute pericarditis. N Engl J Med. 2004; 351(21):2195–2202. 15. Libby P, Bonow RO, Mann DL, et al. Braunwald’s Heart Disease: A Textbook of Cardiovascular Medicine. 8th ed. Philadelphia, PA: Saunders/Elsevier; 2008:1301–1303, 1308. 16. Alvarado G, Noor R, Bassett R, et al. Risk of intracranial hemorrhage with anticoagulation therapy in melanoma patients with brain metastases. Melanoma Res. 2012;22:310–315.

235

66485457-66485438 www.ketabpezeshki.com

ZZakaria_87574_PTR_CH16_10-06-13_217-238.inddakaria_87574_PTR_CH16_10-06-13_217-238.indd 235235 66/19/2013/19/2013 8:47:318:47:31 PMPM GENERAL CRITICAL CARE: PATHOLOGY, PATHOPHYSIOLOGY, AND THERAPY

17. Libby P, Bonow RO, Mann DL, et al. Braunwald’s Heart Disease: A Textbook of Cardiovascular Medicine. 8th ed. Philadelphia, PA: Saunders/Elsevier; 2008:1917–1918. 18. Lisk DR, Grotta JC, Lamki LM, et al. Should hypertension be treated after acute stroke? a randomized controlled trial using single photon emission computed tomography. Arch Neurol. 1993;50:855–862. 19. Libby P, Bonow RO, Mann DL, et al. Braunwald’s Heart Disease: A Textbook of Cardiovascular Medicine. 8th ed. Philadelphia, PA: Saunders/Elsevier; 2008:2146–2147. 20. Libby P, Bonow RO, Mann DL, et al. Braunwald’s Heart Disease: A Textbook of Cardiovascular Medicine. 8th ed. Philadelphia, PA: Saunders/Elsevier; 2008:1835–1838. 21. Murphy JG, Lloyd MA. Mayo Clinic Cardiology: Concise Textbook. 3rd ed. Boca Raton, FL: Informa Healthcare; 2006:776. 22. Kearon C, Kahn SR, Agnelli G, Goldhaber S, Raskob GE, Comerota AJ. Antithrombotic therapy for venous thromboembolic disease: American College of Chest Physicians Evidence-based clinical practice guidelines (8th edition). Chest. 2008;133(6 Suppl): 454S–545S. 23. Harris NS. Altitude illness: pulmonary syndromes treatment and management. http:// emedicine.medscape.com/article/773065-treatment 24. Hackett PH, Rennie IDB. High altitude pulmonary edema. JAMA. 2002;287(17): 2275–2278. 25. Lipsitz LA, Gagnon M, Vyas M, et al. Antihypertensive therapy increases cerebral blood fl ow and carotid distensibility in hypertensive elderly subjects. Hypertension. 2005;45(2):216–221. 26. Bertel O, Marx BE, Conen D. Effects of antihypertensive treatment on cerebral perfusion. Am J Med. 1987;82(3B):29–36. 27. Crossley GH, Poole JE, Rozner MA, et al. The Heart Rhythm Society (HRS)/American Society of Anesthesiologists (ASA) Expert Consensus Statement on the perioperative management of patients with implantable defi brillators, pacemakers and arrhythmia monitors: facilities and patient management this document was developed as a joint pro- ject with the American Society of Anesthesiologists (ASA), and in collaboration with the American Heart Association (AHA), and the Society of Thoracic Surgeons (STS). Heart Rhythm. 2011;8(7):1114–1154. 28. Libby P, Bonow RO, Mann DL, et al. Braunwald’s Heart Disease: A Textbook of Cardiovascular Medicine. 8th ed. Philadelphia, PA: Saunders/Elsevier; 2008: 1846–1848. 29. Willerson J, Cohn JN, Wellens HJJ, Holmes DR. Textbook of Cardiovascular Medicine. 3rd ed. London, United Kingdom: Springer Verlag; 2007: 966. 30. Libby P, Bonow RO, Mann DL, et al. Braunwald’s Heart Disease: A Textbook of Cardiovascular Medicine. 8th ed. Philadelphia, PA: Saunders/Elsevier; 2008: 1269. 31. Saxon LA, DeMarco T, Prystowsky EN, et al. Effects of long-term biventricular stimu- lation for rsynchronization on ehocardiographic measures of remodeling. Circulation. 2002;105:1304–1310. 32. Little WC, Freeman GL. Pericardial disease. Circulation. 2006;113(12):1622–1632. 33. Spodick DM. Acute cardiac tamponade. N Engl J Med. 2003;349:684. 34. Birnbaum Y, Chamoun AJ, Conti VR, Uretsky BF. Mitral regurgitation following acute myocardial infarction. Coron Artery Dis. 2002;13(6):337–344. 35. Goldschlager N, Epstein AE, Naccarelli GV, et al. A practical guide for clinicians who treat patients with amiodarone: 2007. Heart Rhythm. 2007;4(9):1250–1259.

236

66485457-66485438 www.ketabpezeshki.com

ZZakaria_87574_PTR_CH16_10-06-13_217-238.inddakaria_87574_PTR_CH16_10-06-13_217-238.indd 236236 66/19/2013/19/2013 8:47:318:47:31 PMPM CARDIOVASCULAR DISEASES: Answers

36. Aliot EM, Stevenson WG, Almendral-Garrote JM, et al. EHRA/HRS Expert Consensus on Catheter Ablation of Ventricular Arrhythmias: developed in a partnership with the European Heart Rhythm Association (EHRA), a registered branch of the European Society of Cardiology (ESC), and the Heart Rhythm Society (HRS); in collaboration with the American College of Cardiology (ACC) and the American Heart Association (AHA). Heart Rhythm. 2009;6(6):886–933. 37. Libby P, Bonow RO, Mann DL, et al. Braunwald’s Heart Disease: A Textbook of Cardiovascular Medicine. 8th ed. Philadelphia, PA: Saunders/Elsevier; 2008: 977–983.

237

66485457-66485438 www.ketabpezeshki.com

ZZakaria_87574_PTR_CH16_10-06-13_217-238.inddakaria_87574_PTR_CH16_10-06-13_217-238.indd 237237 66/19/2013/19/2013 8:47:318:47:31 PMPM 66485457-66485438 www.ketabpezeshki.com

ZZakaria_87574_PTR_CH16_10-06-13_217-238.inddakaria_87574_PTR_CH16_10-06-13_217-238.indd 238238 66/19/2013/19/2013 8:47:318:47:31 PMPM 17 Pulmonary Physiology and Fundamentals of Mechanical Ventilation David J. Powner QUESTIONS

1. The respiratory center sensitive to CO2 and regional acidosis is located in the medulla.

Chemo-receptors sensitive to hypoxemia (PaO2 < about 60 torr) and its effect on hydrogen ion sensitivity are located in the: A. Carotid and aortic bodies B. Pulmonary artery oxygen sensors at the fi rst division of the main pulmonary artery C. Oxygen sensory center(s) in the medial and lateral dorsal brainstem D. Left ventricular sensor(s) associated with pro-brain natriuretic peptide release sites E. Neurofi brillary bodies at bifurcation of femoral arteries from the aorta

ANSWERS TO THIS SECTION CAN BE FOUND ON PAGE 249 239

66485457-66485438 www.ketabpezeshki.com

ZZakaria_87574_PTR_CH17_10-06-13_239-266.inddakaria_87574_PTR_CH17_10-06-13_239-266.indd 239239 66/19/2013/19/2013 8:47:488:47:48 PMPM GENERAL CRITICAL CARE: PATHOLOGY, PATHOPHYSIOLOGY, AND THERAPY

2. As the spontaneously breathing patient initiates inspiration, the subambient pleural pres- sure is transmitted through the open airways, overcomes the inertia of atmospheric gas molecules and initiates the inspiratory gas fl ow into the patient. This fl ow is another mea- sure of parenchymal “distensibility” and is proportional to the amount of subambient pressure created by the muscles of inspiration. This measure, Δfl ow ÷ Δpressure, is: A. Pulmonary conductance B. Pulmonary resistance C. Pulmonary compliance D. Airway fl ow capacitance E. Flow-gated airway distension

3. At the end of inspiration gas fl ow into the lung stops, turbulence subsides and a tidal volume has been delivered. The tidal volume is also proportional to the transpulmonary inspiratory pressure created by the patient during spontaneous inhalation or by the ven- tilator during mechanical infl ation, that is, Δvolume ÷ Δpressure. This relationship is known as: A. Pulmonary conductance B. Pulmonary resistance C. Pulmonary compliance D. Functional residual capacity (FRC) E. Airway volume capacity

4. The tidal volume produced at the end of inspiration has distended airways and lung units. As the muscles of inspiration relax, a recoil force is created that initiates and maintains an outward fl ow of gas, which is proportional to the amount of distension created by the tidal inhalation. The relationship is known as: A. Pulmonary conductance B. Pulmonary resistance C. Pulmonary compliance D. Pulmonary elastance E. Expiratory fl ow force

5. Water vapor is added to inhaled gas through the process of evaporative insensible fl uid loss. The relative humidity of inhaled gas is increased to 100% at body temperature and atmospheric pressure. In which anatomic area is this important respiratory function pre- dominately accomplished? A. Distal airways beyond the terminal bronchiole B. Respiratory bronchioles C. Nose and trachea D. Segmental bronchi beyond division two airways E. All of the above

240

66485457-66485438 www.ketabpezeshki.com

ZZakaria_87574_PTR_CH17_10-06-13_239-266.inddakaria_87574_PTR_CH17_10-06-13_239-266.indd 240240 66/19/2013/19/2013 8:47:488:47:48 PMPM PULMONARY PHYSIOLOGY AND MECHANICAL VENTILATION: Questions

6. The so-called “bulk” fl ow of gas responding to changes in pleural pressure occurs through the terminal bronchiole (16th of the 23 normal airway divisions). Distal to the terminal bronchiole, gas-in-gas migration carries oxygen to the alveolo-capillary membrane (ACM)

and CO2 from it. Which statement describes this gas-in-gas diffusion correctly?

A. Oxygen diffuses more quickly than CO2 but dissolves more slowly in the ACM

B. Oxygen diffuses more slowly than CO2 and dissolves more slowly in the ACM

C. Oxygen diffuses more quickly than CO2 and dissolves more quickly in the ACM D. Both gases diffuse and dissolve equally

7. The primary determinant of the normal distribution of blood perfusion in the lung is: A. Transthoracic pressure B. Transpulmonary pressure gradient C. Cardiac output D. Gravity E. Pulmonary vascular resistance

8. John West, MD, has described three lung zones that characterize normal and potentially abnormal ventilation and perfusion relationships. West’s zone 1 is abnormal, may pro- duce increased dead space or erroneous interpretations of pulmonary artery occlusion pressure measurements, and is characterized by: A. Airway pressure > venous capillary pressure > arteriolar capillary pressure B. Venous capillary pressure > arteriolar capillary pressure > airway pressure C. Arteriolar capillary pressure > venous pressure > airway pressure D. Left atrial pressure > “wedge” pressure > bronchial airway pressure E. Airway pressure > arteriolar capillary pressure > venous capillary pressure

9. Dalton’s Law reminds us that the sum of the partial pressures of gases in the airway must always equal barometric pressure. The four gases normally in greatest quantity in normal airways are:

A. O2, CO2, N2, NO2

B. CO2 , water vapor, N2, CO

C. O2, N2, water vapor, CO2

D. H2, O2, CO2, N2

10. Because of Dalton’s Law, breathing 100% oxygen “washes out” nitrogen from normal lungs and may lead to microatelectasis throughout poorly ventilated lung units. Nitrogen washout normally occurs in about: A. An hour B. 8 minutes C. 15 to 20 minutes D. 24 hours

241

66485457-66485438 www.ketabpezeshki.com

ZZakaria_87574_PTR_CH17_10-06-13_239-266.inddakaria_87574_PTR_CH17_10-06-13_239-266.indd 241241 66/19/2013/19/2013 8:47:488:47:48 PMPM GENERAL CRITICAL CARE: PATHOLOGY, PATHOPHYSIOLOGY, AND THERAPY

11. As gas fl ows into and out of the lung, impedance forces in the form of mucus, bifurcations in the airways, bronchoconstriction, and so on are encountered that are proportional to the gas fl ow (ΔP ÷ Δfl ow). This measures: A. Pulmonary compliance B. Pulmonary elastance C. Pulmonary resistance D. Pulmonary conductance

12. The equation: Vg = (A) (ΔP) (solubility) (temp) ÷ (L) (viscos) (square root of mol wt of gas) (where A is the cross-sectional area of open alveoli, ΔP the pressure gradient for gas, L is the length of pathway, and viscos is the viscosity of pathway), describes what respiratory function? A. Transcapillary blood fl ow from pulmonary arteriole to pulmonary venule B. Gas fl ow along airways distal to the terminal bronchioles C. Oxygen delivery to peripheral tissue beds D. Oxygen content in the blood E. Gas diffusion across the alveolo-capillary membrane (ACM)

13. A pulmonary ventilation:perfusion (V/Q) mismatch described as a “shunt” or “shunt effect” describes a preponderance of lung units in which: A. Dead space (VD/VT) to tidal volume ratio is increased B. Airway ventilation is greater than capillary perfusion C. Capillary perfusion exceeds airway ventilation D. Carbon dioxide retention occurs E. Pulmonary edema occurs

14. The primary determinants of CO2 loss are indicated by the alveolar minute ventilation equation, which includes: A. Transpulmonary pressure gradient B. Inspiratory force (negative pressure generated by patient) C. Zones of V/Q shunting D. Functional residual capacity (FRC) E. Zones of dead space effect

15. During mechanical ventilation, the primary determinant(s) of oxygenation is/are:

A. Fraction of inspired oxygen (FIO2) B. Positive end-expiratory airway pressure (PEEP) C. Respiratory rate D. Mean airway pressure E. A and D F. A and B G. B and C H. B and D

242

66485457-66485438 www.ketabpezeshki.com

ZZakaria_87574_PTR_CH17_10-06-13_239-266.inddakaria_87574_PTR_CH17_10-06-13_239-266.indd 242242 66/19/2013/19/2013 8:47:488:47:48 PMPM PULMONARY PHYSIOLOGY AND MECHANICAL VENTILATION: Questions

16. Match the defi nition in the left column to the formula on the right:

A. Conductance 1. VT ÷ Pawp – PEEP B. Static compliance 2. Δfl ow ÷ ΔP at inhalation

C. Resistance 3. VT ÷ Pplat – PEEP D. Elastance 4. Δfl ow ÷ ΔP at exhalation E. Dynamic compliance 5. ΔP ÷ Δfl ow Δ where VT is the tidal volume; is the change; Pawp is the peak airway pressure; Pplat is the plateau airway pressure; P is the general airway pressure; and PEEP is the positive end-expiratory pressure.

17. Which pulmonary parameter does Enghoff’s modifi cation of the Bohr equation, PaCO2 −

PECO2 ÷ PaCO2, evaluate?

A. CO2 diffusion through the alveolo-capillary membrane (ACM)

B. CO2 production during resting metabolism C. VD/VT (dead space to tidal volume ratio)

D. Total CO2 excretion measured from indirect calorimetry E. None of the above

where PECO2 is the partial pressure of CO2 in the patient’s exhaled gas collected over a timed period.

18. If a normal 20-year-old subject, resting at sea level, is placed on 100% oxygen for about

15 minutes, you will expect his or her PaO2 to be about: A. 100 mmHg B. 650 mmHg C. 250 mmHg D. 800 mmHg E. 925 mmHg

19. A patient’s FEV1/FVC (forced exhaled volume in one second compared to the full forced vital capacity volume) ratio is less than 50% on pulmonary function testing (PFT) last week. He is intubated now and initial ventilator settings are ordered. Although all listed parameters are important, particular attention should be paid to which one, based on his PFT information? A. Peak airway pressure B. Trigger sensitivity C. Level of pressure support (PS) you order D. Inhalation time to exhalation time (I:E) ratio E. Peak:plateau airway pressure ratio

243

66485457-66485438 www.ketabpezeshki.com

ZZakaria_87574_PTR_CH17_10-06-13_239-266.inddakaria_87574_PTR_CH17_10-06-13_239-266.indd 243243 66/19/2013/19/2013 8:47:498:47:49 PMPM GENERAL CRITICAL CARE: PATHOLOGY, PATHOPHYSIOLOGY, AND THERAPY

20. During incentive spirometry, the patient is asked to duplicate which spirometric component? A. Functional residual capacity (FRC) B. Inspiratory reserve volume C. Inspiratory capacity D. Total lung volume E. Maximum ventilatory volume

21. The normal percentage of the tidal volume that occupies anatomic and physiological dead space is: A. 5% to 7% B. 10% to 15% C. 1% to 2% D. 20% to 40% E. Variable depending on minute ventilation

22. A patient has moderate to severe kyphoscoliosis and has required intubation for airway protection after his intracerebral hemorrhage. His Glasgow Coma Scale score is 10 T. What concern(s) do you anticipate during mechanical ventilation based on his body habitus? A. High probability of auto-positive end-expiratory airway pressure (PEEP) B. Increased spontaneous breathing rate C. Inhalation time to exhalation time (I:E) ratio indicating prolonged expiratory time D. Low plateau airway pressure during ventilator infl ation E. All of the above

23. During respiratory distress, the characteristic fi ndings of paradoxic abdominal motion during spontaneous breathing are: A. Tensing of the lateral oblique abdominal muscles with each inhalation B. Protrusion of the rectus abdominus muscle during inhalation C. Simultaneous expansion of the thoracic and abdominal cavities with inhalation D. Concurrent retraction of the abdomen and chest walls during exhalation E. Chest wall expansion and abdominal retraction during inhalation

24. Although controversial, the negative inspiratory force (NIF) measurement, also known as the maximal inspiratory pressure (MIP) measurement, has been used to assess the patient’s readiness for extubation. The NIF value commonly considered acceptable before initiating an extubation trial is:

A. −5 cmH2O

B. −15 cmH2O

C. −25 cmH2O

D. −35 cmH2O

244

66485457-66485438 www.ketabpezeshki.com

ZZakaria_87574_PTR_CH17_10-06-13_239-266.inddakaria_87574_PTR_CH17_10-06-13_239-266.indd 244244 66/19/2013/19/2013 8:47:498:47:49 PMPM PULMONARY PHYSIOLOGY AND MECHANICAL VENTILATION: Questions

25. The rapid, shallow breathing index, f/VT, as a weaning assessment tool, is measured during at least 1 minute of spontaneous breathing, supported only by oxygen. What cal- culated value is associated with a poor chance of weaning success? A. Greater than 130 B. Greater than 105 C. Greater than 150 D. Less than 12 E. None of the above 26. The American College of Chest Physicians, the American College of Critical Care Medicine, and the American Association for Respiratory Care issued weaning guidelines in 2001 that recommend which primary method of assessment and weaning technique be used? A. Synchronized intermittent mandatory ventilation (SIMV) with pressure support (PS) and gradual reduction in the SIMV B. Assist-control but progressively making the trigger more insensitive C. Utilization of progressive airway resistive devices attached to the endotracheal tube to incrementally strengthen inspiratory muscles D. Spontaneous breathing trials on a T-piece E. Pressure support mode alone with gradually decreasing PS levels 27. Cheyne–Stokes respirations are characterized by sequentially increasing tidal volumes followed by similarly regular decreasing tidal volumes separated by periods of apnea. Neurological localization of this pattern possibly includes the diencephalon, basal gan- glia, and: A. Cerebral hemispheres B. High spinal cord below medulla C. Ventro-lateral medulla D. Dorsal pons E. Carotid bodies 28. A patient with Guillain–Barré syndrome is being monitored every 2 hours to measure her vital capacity. During that test the patient must: A. Inhale fully from resting tidal exhalation to total lung volume B. Exhale as rapidly as possible from total lung capacity C. Inhale forcibly against a pressure manometer to test inspiratory strength D. Inhale repetitively to assess if early fatigue is present E. None of the above 29. A 47-year-old patient is known from prior pulmonary function testing (PFT) testing to have a uniform moderate reduction in total lung volume, functional residual capacity (FRC), and residual volumes, but a normal FEV1/FVC (forced exhaled volume in one second compared to the full forced vital capacity volume) ratio. He is receiving pressure-limited assist-control

ventilation at 25 cmH2O, 8 cmH2O positive end-expiratory airway pressure (PEEP) and a set frequency of 6. In evaluating his current interaction with the ventilator, we expect: A. Early onset of auto-PEEP, low trigger sensitivity B. Short-to-normal expiratory time and high tidal volume C. Low plateau airway pressure and low respiratory rate D. Increased respiratory rate and low-normal tidal volume E. None of the above

245

66485457-66485438 www.ketabpezeshki.com

ZZakaria_87574_PTR_CH17_10-06-13_239-266.inddakaria_87574_PTR_CH17_10-06-13_239-266.indd 245245 66/19/2013/19/2013 8:47:498:47:49 PMPM GENERAL CRITICAL CARE: PATHOLOGY, PATHOPHYSIOLOGY, AND THERAPY

30. A 43-year-old female patient, who is admitted with a thrombotic stroke, is known to have lupus erythematosus. Prior pulmonary function testing (PFT) showed a reduced carbon monoxide (CO) diffusing capacity relative to her effective total lung capacity. Anticipating her possible needs for respiratory support, we expect: A. Substantial need for bronchodilators B. High levels of inspiratory pressure support (IPAP) via continuous positive airway pressure (CPAP) or bilevel noninvasive positive pressure treatment C. High dependence upon incentive spirometry to ensure that airways remain open D. Poor tolerance of any stress that increases cardiac output E. Need for increased levels of expiratory airway support via CPAP or bilevel noninva- sive positive pressure treatment

31. During controlled volume-limited mechanical ventilation in an apneic patient, the most potentially dangerous parameter displayed by the ventilator and requiring vigilant bed- side attention by respiratory care and nursing staff is: A. Auto-positive end-expiratory airway pressure (PEEP) B. Inspiratory time C. Minute ventilation D. Peak airway pressure E. Negative inspiratory force (NIF)

32. During pressure-limited ventilation, the variable that requires vigilant bedside attention is: A. Inspiratory time B. Tidal volume C. Peak airway pressure D. Expiratory time E. Auto-positive end-expiratory airway pressure (PEEP)

33. The inhalation time to exhalation time (I:E) ratio refl ects the relationship between: A. Inspiratory airway pressure and expiratory pressure B. Inspiratory pause and expiratory pressure C. Inspiratory versus expiratory times D. Inspiratory fl ow and elasticity E. Inspiratory versus expiratory volumes

34. Auto-positive end-expiratory airway pressure (PEEP; intrinsic, inadvertent PEEP) occurs: A. When expiratory time is too long B. When fi xed PEEP is below critical closing pressure C. When elastic recoil is high, as in acute lung injury (ALI) D. When respiratory rate is too slow E. All of the above F. None of the above

246

66485457-66485438 www.ketabpezeshki.com

ZZakaria_87574_PTR_CH17_10-06-13_239-266.inddakaria_87574_PTR_CH17_10-06-13_239-266.indd 246246 66/19/2013/19/2013 8:47:498:47:49 PMPM PULMONARY PHYSIOLOGY AND MECHANICAL VENTILATION: Questions

35. During the assist-control mode of ventilation, the respiratory rate set on the ventilator: A. Adds that rate to the patient’s spontaneous breathing rate B. Is added to the patient’s spontaneous breathing rate if the patient’s rate is below 10 breaths/minute C. Is used to reduce work of breathing if the patient becomes tachypneic D. Operates only if the patient’s spontaneous breathing rate is below the ventilator set rate E. Assures the set minute ventilation

36. During airway pressure release ventilation (APRV), user-prescribed variables include: A. Tidal volume B. Respiratory frequency C. Inspiratory hold (pause or plateau) D. Expiratory fl ow rate E. All the above F. None of the above

37. The equation for men, 106 + [6(height in inches − 60)], is recommended for the calculation of what variable when initiating mechanical ventilation to treat acute lung injury (ALI)? A. Inspiratory fl ow rate B. Expiratory retardation to fl ow C. Tidal volume D. Minute ventilation

38. There may be several reasons why the patient “bucks the ventilator.” The observation that the patient attempts, but fails, to initiate a ventilator infl ation during the assist-control mode of ventilation is likely due to: A. Positive end-expiratory airway pressure (PEEP) is set too low B. Inspiratory fl ow rate is set too low C. Ventilator rate is set too low D. There is a ventilator malfunction E. Any of the above F. None of the above

39. Plateau airway pressure is: A. Considered a better refl ection of distal airway pressure than is peak airway pressure B. Used to calculate static lung/thoracic compliance C. Usually evaluated after an inspiratory pause within the infl ation phase of ventilation D. Recorded after inspiratory gas turbulence subsides E. All of the above F. None of the above

40. Poiseuille’s Law relates the resistance to gas fl ow through an endotracheal tube to: A. Material from which the tube is made, that is, tubing compliance B. Route of intubation (oral vs. nasal) C. Length of the tube D. Inspiratory fl ow rate of any pressure support (PS) applied E. Mucus accumulation inside the endotracheal tube

247

66485457-66485438 www.ketabpezeshki.com

ZZakaria_87574_PTR_CH17_10-06-13_239-266.inddakaria_87574_PTR_CH17_10-06-13_239-266.indd 247247 66/19/2013/19/2013 8:47:498:47:49 PMPM GENERAL CRITICAL CARE: PATHOLOGY, PATHOPHYSIOLOGY, AND THERAPY

41. Assuming normal lungs, the distribution of inspiratory gas delivered to an apneic patient during controlled mechanical ventilation: A. Favors gravitationally dependent lung units B. Matches ventilation with perfusion C. Prevents derecruitment D. Determines the functional residual capacity (FRC) E. Preferentially infl ates gravitationally nondependent lung units

42. During airway pressure release ventilation (APRV), spontaneous breathing by the patient: A. Should be suppressed by sedation or pharmacological paralysis B. Augments the Ti (inspiratory time) parameter ordered by the physician C. Must be coordinated with set positive end-expiratory airway pressure (PEEP) to pre- vent derecruitment D. May be permitted as tolerated by the patient E. None of the above

43. Pressure-limited assist/control ventilation: A. Is often used to limit plateau air way pressure B. Prevents auto-positive end-expiratory airway pressure (PEEP) C. May be associated with lower exhaled tidal volume when lung compliance falls D. May be associated with higher tidal volumes if pulmonary elastance is high E. A and C F. A, C, and D G. B and D H. None of the above

44. Positive end-expiratory airway pressure (PEEP) increases: A. Functional residual capacity (FRC) B. Inspiratory reserve volume C. Expiratory capacity D. Total lung volume E. All of the above F. None of the above

45. The amount of inspiratory airway pressure ordered during pressure support (PS), syn- chronized intermittent mandatory ventilation (SIMV) with PS, or continuous positive air- way pressure (CPAP) ventilation should be monitored by following the: A. Exhaled tidal volume from the patient B. Inhalation time to exhalation time (I:E) ratio displayed on the ventilator

C. Fraction of inspired oxygen (FIO2), PaO2 ratio on serial arterial blood gas measurements D. Peak airway pressure E. Level of auto-positive end-expiratory airway pressure (PEEP)

248

66485457-66485438 www.ketabpezeshki.com

ZZakaria_87574_PTR_CH17_10-06-13_239-266.inddakaria_87574_PTR_CH17_10-06-13_239-266.indd 248248 66/19/2013/19/2013 8:47:498:47:49 PMPM 17

ANSWERS

1. The answer is A. Peripheral chemo-receptors for hypoxemia are located in carotid bod- ies bilaterally at the bifurcation of the common carotid arteries and in aortic bodies in the arch of the aorta. These sensors respond primarily to H+ ion concentration due to respi- ratory or metabolic acidosis. Their sensitivity and neural discharges are increased with

hypoxemia and are proportional to a PaO2 less than about 60 torr. The response is made less sensitive by other factors, for example, alkalemia, hyperoxia, hypocarbia, anesthetic agents, and older age (1–3).

2. The answer is A. Pulmonary conductance is as defi ned in the question and is a dynamic measure of distensibility because fl ow, defi ned as gas volume moving per unit time, is considered. Conductance is, of course, reduced in some restrictive lung diseases or if other airway impedance (restriction) is present. Compliance is a static measure of distensibility at completion of inspiratory gas fl ow, to be discussed next.

3. The answer is C. As discussed in Answer 2, at the termination of the spontaneous or ven- tilator-delivered inspiratory gas fl ow, a tidal volume has been delivered and compliance, as another measure of distensibility, can be measured. In general clinical practice, both compliance and conductance evaluate combined lung + thoracic distensibility. To evaluate true lung values alone, a measure of intrapleural pressure must be used for the ΔP. The ΔP in the esophagus has been substituted, but placement of an esophageal manometer is cumbersome and rarely used clinically. Therefore, compliance and conductance con- siderations must be considered in the context of any primary thoracic as well as pulmo- nary pathology. During mechanical ventilation, compliance is usually evaluated during an apneic period (no spontaneous breathing) as either dynamic or static compliance of the combined lung and thorax. Tidal volume is assumed to be that set and delivered by the ventilator, and one of two changes in pressure are noted. Note that in both equations a

249

66485457-66485438 www.ketabpezeshki.com

ZZakaria_87574_PTR_CH17_10-06-13_239-266.inddakaria_87574_PTR_CH17_10-06-13_239-266.indd 249249 66/19/2013/19/2013 8:47:498:47:49 PMPM GENERAL CRITICAL CARE: PATHOLOGY, PATHOPHYSIOLOGY, AND THERAPY

change in pressure is required, so that positive end-expiratory airway pressure (PEEP), as the baseline end-expiratory pressure, must be considered in the equations for both static and dynamic compliance. Static compliance of the lung + thorax assumes that the turbu- lence of the inspiratory gas fl ow has subsided and is considered a more accurate measure (estimate) of the combined distensibility. Dynamic compliance is easier to estimate but must be recognized to include some component of gas turbulence. The two pressures used for measurement of static and dynamic compliance are peak and plateau airway pressures, respectively. Therefore, static compliance is correctly estimated as tidal volume ÷ plateau airway pressure – PEEP, and dynamic compliance is tidal volume ÷ peak airway pressure − PEEP. Plateau airway pressure is measured (and usually displayed on the ventilator) after insertion of a brief “inspiratory pause” or “inspiratory hold” into the infl ation cycle of the ventilator. This parameter delays the opening of the exhalation valve of the ventilator and holds the inspired tidal volume in the lung. This hold allows turbulence in the trachea and proximal airways to subside and some of the tidal volume to move passively into distal airways. The initial peak airway pressure noted at the end of inspiration will fall slightly to a lower pressure designated as the plateau airway pressure. Many modern ventilators will make these measurements internally and provide a display. If the patient’s spontaneous breathing rate is too fast to tolerate the inspiratory hold, the ventilator will so indicate and be unable to calculate static compliance (4).

4. The answer is D. At the end of exhalation the glottis normally opens, creating a pressure gradient between distal airways (responsive to the elastic recoil created by the parenchy- mal elastic and collagen fi bers within the lung) and atmospheric pressure. Gas fl ow (vol- ume per unit time) from the lung responds to that pressure gradient during the expiratory time, hopefully completing a normally passive exhalation. If elastic recoil forces (mea- sured as elastance) are weak, as in chronic obstructive pulmonary disease (COPD), gas fl ow is slow and full exhalation may not occur, leaving end-exhalation gas trapped within the lung, increasing functional residual volume (FRC), and possibly creating inadvertent (auto-, intrinsic) positive pressure in the airway. COPD patients are susceptible to these consequences, emphasizing the importance, especially during mechanical ventilation, of monitoring the inhalation time to exhalation time (I:E) ratio that results from ordered ventilator settings to assure an adequate expiratory time and avoiding auto-positive end-expiratory airway pressure (PEEP) (5). Pulmonary elastance is, therefore, extremely important in determining if exhalation can fully occur during the expiratory time. The expiratory time is a passive component of the I:E ratio as that time left over in the full ventilation cycle of inhalation/exhalation at the termination of the inspiratory time. The determinants of the inspiratory time are tidal volume, respiratory rate, and the ventila- tor’s inspiratory fl ow rate. Therefore, if tidal volume is increased, the rate is increased, or the inspiratory fl ow rate of the ventilator is slowed, the expiratory time is shortened. If elastance concurrently is decreased, as in COPD, the force driving a passive exhalation is reduced and a longer time for exhalation is needed to avoid air stacking, air trapping, and auto-PEEP from developing during mechanical ventilation. The interdependency of inspiratory time, expiratory time, and auto-PEEP incorporates elastance in their balance.

5. The answer is C. Source oxygen from a wall-mounted outlet or oxygen cylinder has a rel- ative humidity of less than 1%, creating a large humidity defi cit that must be corrected via evaporative losses from the upper airway or lung mucosa. The requirement for external

250

66485457-66485438 www.ketabpezeshki.com

ZZakaria_87574_PTR_CH17_10-06-13_239-266.inddakaria_87574_PTR_CH17_10-06-13_239-266.indd 250250 66/19/2013/19/2013 8:47:498:47:49 PMPM PULMONARY PHYSIOLOGY AND MECHANICAL VENTILATION: Answers

humidifi cation is controversial for nonintubated well-hydrated patients receiving supple- mental oxygen. Bypassing the upper airway by endotracheal intubation or after tracheo- stomy, however, may lead to airway mucosal injury if the delivered gas is not externally humidifi ed (6).

6. The answer is A. Graham’s Law relates the speed of gas-in-gas diffusion to the square root of the molecular weight of the gas such that the lower molecular weight of oxygen allows it to diffuse more quickly than other gases (water vapor, nitrogen, and carbon dioxide) in the air- way to the ACM. Henry’s Law, however, reminds that gases dissolve in the aqueous ACM in

proportion to their solubility coeffi cient, which is much higher for CO2 than O2 (7). The clinical relevance of this information is that oxygenation can be “diffusion limited”; that is, hypoxemia may result from an ACM diffusion limitation. Hypercarbia, however, does not represent failure

of CO2 diffusion from pulmonary blood, because of its high solubility. CO2 is considered to

be perfusion limited, that is, hypercarbia occurs because of inadequate perfusion of CO2-rich

blood through capillaries adjacent to open airways into which the CO2 may be delivered.

7. The answer is D. Under normal circumstances gravity determines the distribution of pulmonary artery blood fl ow. Therefore, the gravitationally dependent zones of the lung receive more perfusion than nondependent areas (8). In the supine patient, this, of course, is the posterior plane of the lung; standing directs blood fl ow to the anatomic lung bases, and so on. Because of blood engorgement in the gravitationally dependent lung zones, the gravitationally dependent small airways are slightly smaller than the gravitationally nondependent small airways at the beginning of inhalation. This smaller size reduces small airway surface tension forces and allows those airways to be expanded more eas- ily during normal spontaneous inhalation. During normal conditions, therefore, gas fl ow is preferentially directed along the path of least resistance, toward the gravitationally dependent airways adjacent to the normally distributed pulmonary blood fl ow. In this

Source: Printed with permission from West JB. Regional differences in gas exchange in the lung of erect man. J Appl Physiol. 1962;17:893–898.

251

66485457-66485438 www.ketabpezeshki.com

ZZakaria_87574_PTR_CH17_10-06-13_239-266.inddakaria_87574_PTR_CH17_10-06-13_239-266.indd 251251 66/19/2013/19/2013 8:47:498:47:49 PMPM GENERAL CRITICAL CARE: PATHOLOGY, PATHOPHYSIOLOGY, AND THERAPY

fashion during normal spontaneous breathing, ventilation and perfusion are mostly matched. Ventilation and perfusion are not perfectly matched, with an overall normal V/Q ratio of about 0.85. The accompanying illustration (from standing position) shows those relationships (8), where Vol % is percentage of tidal volume better distributed to

base; VA/Q is the V/Q ratio (note the lower ratio at base, indicating increased perfusion);

and VA is liters/minute, showing better volumes at base.

8. The answer is E. Although West’s zones (9) are not objectively measurable, the concepts and clinical value they provide are very useful. For example, increased dead space is the

reason for a higher-than-expected PaCO2 in the setting of an above-normal (>8 L/minute) minute ventilation (see Answer 14). Similarly, if a tachypneic, hyperpneic patient is not hyperventilating on blood gas assessment, increased dead space is the cause. Because dead space is defi ned as a predominance of lung units in which ventilation exceeds perfusion (perfusion less than ventilation), a zone 1 abnormality may be the cause. Other causes of

low perfusion that may lead to relatively increased PaCO2 include any cause of low cardiac output, pulmonary embolus, hypovolemia, and so on. Zone 1 physiology, however, should be considered, particularly if the patient is receiving mechanical ventilation and may be fur- ther suggested by high airway pressure, increased positive end-expiratory airway pressure (PEEP), signs of hypovolemia, or low systemic perfusion. The clinical response to inappro-

priate PaCO2 retention in such a situation is to reduce ventilator parameters that produce increased airway pressure and to consider whether fl uid administration would improve pulmonary perfusion. With regard to West’s other zones: zone 3 is the normal occurrence (shown as answer option C), and zone 2 is arteriolar > airway > venous pressures (9).

9. The answer is C (see explanation for Question 10).

10. The answer is B. The clinical relevance of Dalton’s Law is that as the fraction of inspired

oxygen (FIO2) is raised, nitrogen is rapidly (normal N2 washout is 2–5 minutes) replaced in airways by oxygen. Nitrogen (78% of room air) is a nondiffusible gas that normally provides an airway “splinting” effect. As nitrogen is replaced by oxygen, lung units that are poorly ventilated due to lung pathology will often close as the oxygen diffuses into the blood. This absorption leads to absorption atelectasis, increases the number of lung units with a poor ventilation:perfusion (V/Q) ratio, that is, shunt effect, and potentially worsens hypoxemia (7). Dalton’s Law, therefore, provides the impetus for maintaining

the patient’s FIO2 at the lowest level needed for adequate oxygenation.

11. The answer is C. Impedance to the fl ow of gas by factors listed in the question produce resistance proportional to the gas movement created during inhalation by the patient’s respiratory effort when spontaneously breathing or by the ventilator’s inspiratory gas fl ow. During exhalation, elastic recoil with or without the assistance of accessory mus- cles of exhalation generates the force necessary to overcome similar expiratory resistance factors. Other factors that create resistance include nonelastic tissue; viscous resistance needed to overcome forces inherent in moving the lung tissue, thorax, and so on; and airway resistance, including turbulent gas, mucus, size of the airway, and so on (4).

12. The answer is E. Important factors in this equation that describes diffusion of oxygen into the blood from the airway are: (A) describing the availability of open airways and the

252

66485457-66485438 www.ketabpezeshki.com

ZZakaria_87574_PTR_CH17_10-06-13_239-266.inddakaria_87574_PTR_CH17_10-06-13_239-266.indd 252252 66/19/2013/19/2013 8:47:498:47:49 PMPM PULMONARY PHYSIOLOGY AND MECHANICAL VENTILATION: Answers

negative effects of a long (L) or highly viscous (viscos) interstitial pathway for oxygen to travel. Fibrosis, edema, albumin, pus, and so on, in that route will reduce diffusing capac- ity and limit oxygen uptake, during the brief time (normally 0.75 seconds) a red blood cell remains in the pulmonary capillary bed. Although ventilation:perfusion mismatching is the most common cause for hypoxemia, a diffusion defect is also considered important, especially in such conditions as pulmonary edema, capillary leak, and so on, where the interstitial space is at risk. Opening (recruitment) and maintaining (preventing derecruit- ment) distal airways, as can be seen from this equation, are important not only in maintain- ing a ventilation:perfusion ratio (V/Q) balance but also in the process of diffusion (10).

13. The answer is B. A ventilation:perfusion mismatch described here is considered a com- mon if not the most common cause of hypoxemia and is a common pathological pathway for most etiologies of disordered lung function leading to hypoxemia. Other mechanisms that lead to hypoxemia include diffusion defects as discussed in Answer 12, alveolar hypoventilation that produces hypercarbia, and unusual situations such as high altitude

or experimental situations in which the partial pressure of inspired oxygen (PIO2) or frac-

tion of inspired oxygen (FIO2) is reduced (8,10). The clinical relevance of shunt effect as a cause of hypoxemia is that therapy should be directed toward opening airways (e.g., manual or ventilator recruitment maneuvers) (11) and maintaining them open by pre- venting derecruitment (usually ascribed to best positive end-expiratory airway pressure [PEEP]). Mean airway pressure, rather than peak or plateau pressures, is generally con- sidered the preferred method to achieve a critical opening pressure for lung units while minimizing the potential for barotrauma (12).

14. The answer is E. The alveolar minute ventilation equation is: VA = respiratory rate (tidal volume – dead space). This indicates that, to reduce hypercarbia, therapy should be directed toward increasing respiratory rate or tidal volume and reducing dead space (areas of the lung where perfusion is less than ventilation; see West’s zone 1 and other considerations in Answer 8) (4).

15. The answer is E. Mean airway pressure allows appropriate expansion of airways to over-

come a shunt effect, while minimizing risks of barotrauma (12). Increasing the FIO2 should, of course, be the clinician’s initial response to urgent treatment of potential or measured hypoxemia, but as discussed in Answers 9 and 10, denitrogenation of the airways occurs

rapidly and may limit the benefi cial effects of the increased FIO2.

16. The answers are A → 2, B → 3, C → 5, D → 4, and E → 1. These bedside tests of pulmo- nary function are fundamental to the intensivist’s understanding of the lung’s response to disease or injury and to the formulation of safe and effective plans for respiratory support and mechanical ventilation. As discussed previously, these are both dynamic and static measures of lung distensibility and are components of the work of breathing and the oxy- gen consumption by the lung to perform that work.

17. The answer is C. This equation should be considered a description of lung failure to exchange carbon dioxide. Consider it this way: ideally, the denominator of the equa-

tion should have been the mixed venous PvCO2 coming to the lung via the pulmonary

artery. This value was not available to Bohr, but is close to the PaCO2, which, instead, is

253

66485457-66485438 www.ketabpezeshki.com

ZZakaria_87574_PTR_CH17_10-06-13_239-266.inddakaria_87574_PTR_CH17_10-06-13_239-266.indd 253253 66/19/2013/19/2013 8:47:498:47:49 PMPM GENERAL CRITICAL CARE: PATHOLOGY, PATHOPHYSIOLOGY, AND THERAPY

used to represent optimal CO2 exchange. The PECO2 in the numerator measures actual

loss of CO2 from the pulmonary circulation into the airway. The numerator of the equa-

tion, therefore, represents the difference between what “could have been lost (PaCO2)

and what was lost (PECO2),” a difference that represents what was not removed from the body. Therefore, the entire ratio is what was not removed divided by the potential

to be lost, or failure divided by potential. The PECO2 in this equation is measured as

the partial pressure of CO2 in the patient’s exhaled gas collected over a timed period.

It is not the percentage exhaled CO2 or end-tidal CO2 evaluated by capnography (13). Exhaled gas is collected for a timed period in a large airtight bag (Douglas bag), and then

a sample is analyzed using a standard blood gas machine. PaCO2 is obtained from an

arterial blood gas. The importance of dead space in determining total CO2 removal from the body relates to the alveolar minute ventilation equation and pulmonary perfusion, as discussed in Answer 14 (14).

18. The answer is B. This question introduces the calculation of the PAO2 (partial pressure of

oxygen) in the alveolar gas as part of considering the P(A-a) O2 gradient between the alve-

olus and arterial blood. The PAO2 is also used in the formal shunt equation to calculate the theoretic oxygen content of the “perfect” pulmonary capillary (discussed elsewhere).

The formal equation to calculate the PAO2 is complex, which has led to several short hand

equations, more easily used clinically. The complete equation is: PAO2 = PIO2 – {PaCO2

[FIO2 + (1 − FIO2 ÷ 0.8)]}, where PIO2 = (PB − 47) × FIO2 and PB = barometric pressure. Note

that all FIO2 notations are fractions, not percentages; 0.8 is the patient’s respiratory quo- tient (assumed, not measured). By appropriate arithmetic rearrangements and assump- tions, simplifi ed versions are used clinically:

a. PAO2 = FIO2 (PB – 47) – PaCO2

b. PAO2 = PIO2 – (PaCO2 × 1.25)

In the example presented in the question, the PAO2 from (a) = 1.0 (760 − 47) – 40 = 673 mmHg, or from (b) = 1.0 (760 – 47) − (40 × 1.25) = 663 mmHg.

The P(A-a)O2 gradient (also called, A-aO2 gradient) may be thought to be a refl ection or estimate of the diffusion of oxygen into the blood. Under ideal normal circumstances, the

A-aO2 gradient is 5 to 10 mmHg when breathing room air and no higher than 65 mmHg when breathing 100% oxygen. A wider-than-normal gradient, refl ecting failure of all the oxygen available to appear in the arterial blood, may be due to causes other than a diffu- sion defect. A diffusion barrier (as discussed in Answer 12) may be present, but also a shunt effect from V/Q mismatching and/or increased cardiac output may contribute to the relatively low oxygen. See Answer 30 for how high cardiac output may cause hypoxemia. Therefore,

the cause of an increased A-aO2 gradient may not be, and usually is not, purely a failure of diffusion (15,16). Some consider a V/Q mismatch shunt effect to be the most common cause of hypoxemia. I offer a rule of thumb I fi nd most accurate in estimating the percent- age shunt present: a. Allow the patient to receive 100% oxygen for 15 minutes

b. Obtain an arterial PaO2

c. Subtract the patient’s PaO2 from 650 mmHg and divide the difference by 20 to esti- mate the patient’s % shunt (V/Q mismatch). Normal shunt is 4% to 10%.

254

66485457-66485438 www.ketabpezeshki.com

ZZakaria_87574_PTR_CH17_10-06-13_239-266.inddakaria_87574_PTR_CH17_10-06-13_239-266.indd 254254 66/19/2013/19/2013 8:47:508:47:50 PMPM PULMONARY PHYSIOLOGY AND MECHANICAL VENTILATION: Answers

19. The answer is D. The FEV1/FVC ratio is the measure of obstructive pulmonary changes least susceptible to measurement biases and best able to separate patients with normal or restrictive lungs from those with obstructive disease. A normal percentage is above 70% (17). Patients with obstructive lung disease may depend on a prolonged expiratory time to ensure emptying of their tidal volume and may have an abnormal I:E ratio dur- ing spontaneous breathing (i.e., expiratory component greater than the normal 1:2). A clinical observation of the patient’s I:E ratio prior to intubation might have been helpful in initiating ventilator settings. However, during mechanical ventilation, auto-positive end-expiratory airway pressure (PEEP) may occur if the expiratory time is too short (see Answer 34). Therefore, careful observation of the I:E ratio, calculated by the ventilator, and the occurrence of auto-PEEP will be important to monitor.

20. The answer is C. The purpose of incentive spirometry is to exercise lung infl ation to total lung volume from a resting tidal volume exhalation, not from forced exhalation into the expiratory reserve volume. Breath holding for 5 to 10 seconds at maximal inhalation effort is encouraged, to hold open a maximum number of airways. The spirometric vol- umes of tidal volume and inspiratory reserve volume, comprising the inspiratory capac- ity, are inhaled by the cooperative patient (18). The inspiratory capacity is about 75% of the patient’s vital capacity and varies by age, height, and between men and women. Conveniently, the brochure that accompanies the incentive spirometry device has a table of predicted values for inspiratory capacity. This expectation can then be compared to the actual performance of the patient to assess the effectiveness of this therapy in accomplish- ing a lung expansion goal (30,31).

21. The answer is D. See Answer 17 for the calculation of dead space to tidal volume ratio (VD/VT). This provides the normal range, identifying that 20% to 40% of the tidal vol-

ume does not participate in CO2 removal. Measurement of the VD/VT has also been used to predict mortality and diagnose other conditions that limit perfusion, for exam- ple, heart failure, or pulmonary embolism (19,20).

22. The answer is B. Kyphoscoliosis of this magnitude is likely to be associated with a restric- tive lung disorder. Anticipated concerns would be for increased peak and plateau airway pressure (risk of lung injury) during volume-controlled modes of ventilation or limited tidal volume during pressure-limited modes, unless a high pressure limit is ordered. A high work of breathing is expended by patients with lung or thoracic restriction when generating a suffi ciently negative inspiratory force (NIF) to create a normal tidal volume. Such patients often compensate by breathing shallowly but more rapidly. In addition, increased work of breathing may occur after intubation, depending on the size of the endotracheal tube (see Answer 40). An elevated level of pressure support (PS) may be helpful in such circumstances (21).

23. The answer is E. Normally, contraction of the diaphragm during spontaneous inhala- tion enlarges the thorax and displaces the abdominal contents downward and outward. During respiratory distress when accessory muscles of inspiration are used, the thorax is actively expanded, and, despite continued diaphragmatic contraction, the anterior abdominal wall is moved, paradoxically, inward due to the high subambient intrapleural pressure (22).

255

66485457-66485438 www.ketabpezeshki.com

ZZakaria_87574_PTR_CH17_10-06-13_239-266.inddakaria_87574_PTR_CH17_10-06-13_239-266.indd 255255 66/19/2013/19/2013 8:47:508:47:50 PMPM GENERAL CRITICAL CARE: PATHOLOGY, PATHOPHYSIOLOGY, AND THERAPY

24. The answer is C. Recent summary analyses by the Cochrane Registry and others have concluded that ventilator weaning protocols, agreed upon by the critical care team and implemented by nurses and/or respiratory care practitioners, are safe and are more effi - cient than traditional physician-driven methods (23). The specifi c components of indi- vidual protocols tested, however, remain varied and specifi c to individual institutions. Therefore, no fi nal evidence-based method for weaning has been universally established. The NIF (MIP) is commonly used because of its easy application at the bedside, but is recognized to be highly dependent on the patient’s cooperation and effort and the prac- titioner’s encouragement. A normal subject can generate a NIF far above the minimal

subambient pressure of −25 cmH2O, but that level is moderately successful in predicting successful extubation (24,25).

25. The answer is B. This test for weaning is very popular and identifi es the ratio of sponta- neous breathing rate and averaged tidal volume, in liters. It appears to be less effective in elderly patients and those who have required mechanical ventilation for more than 8 days. The value of greater than 105 is considered the ratio that does not support extuba- tion (24,26,27).

26. The answer is D. As noted, these authoritative groups advocate the use of spontaneous breathing trials as a primary method of weaning from mechanical ventilation. Tolerance of T-piece trials for 30 to 120 minutes “should prompt consideration for permanent ven- tilatory discontinuation.” Prerequisites for initiating spontaneous breathing trials include hemodynamic stability, evidence of some improvement in the condition leading to ventila- tion, adequate oxygenation, and the capability to initiate an inspiratory effort (24,32,33).

27. The answer is A. Respiratory dysrhythmias refl ect both primary neurological etiolo- gies (apneustic respirations, central neurogenic hyperventilation, cluster breathing, and Biot’s pattern) and systemic abnormalities (Cheyne–Stokes). Cheyne–Stokes respi- ratory pattern is seen in congestive heart failure and blood sugar changes as well as in stroke (34).

28. The answer is E. The vital capacity maneuver requires the patient to exhale maximally and then inhale to total lung volume (17). When using the vital capacity to determine when/if intubation should be done, some authors use a value less than 12 to 15 mL/kg ideal body weight as an indication for intubation for airway protection (28).

29. The answer is D. The PFT fi ndings describe a restrictive lung disease without an obstructive component. The selection of a pressure-limited mode may be appropriate, particularly if the cause of the restriction is an intrinsic lung disease wherein pulmonary, rather than thoracic, resistance is high. As discussed in Answer 22, a compensatory tachypnea is expected and,

depending on the degree of restriction, a pressure limit of 25 cmH2O may not be suffi cient to produce an adequate tidal volume. A complex formula for calculating the pressure sup- port (PS) needed to overcome some aspects of the work of breathing is presented in Answer 45. While this formula might approximate the initial pressure limit needed in this scenario, initial ventilator orders in most clinical situations will be empiric. Assessment of the tidal volume generated by an initial pressure limit setting will allow adjustment of the pres- sure limit to increase or decrease tidal volume, while still recalling protective ventilation

256

66485457-66485438 www.ketabpezeshki.com

ZZakaria_87574_PTR_CH17_10-06-13_239-266.inddakaria_87574_PTR_CH17_10-06-13_239-266.indd 256256 66/19/2013/19/2013 8:47:508:47:50 PMPM PULMONARY PHYSIOLOGY AND MECHANICAL VENTILATION: Answers

guidelines to maintain plateau airway pressure less than 30 cmH2O. Similarly, after the tidal volume produced by the pressure limit setting is known, the patient’s respiratory rate in the assist/control mode can be assessed. An ideal ventilator backup rate should support the patient’s spontaneous breathing to achieve the desired minute ventilation (usually 7–8 L/ minute) while minimizing work of breathing due to tachypnea and restrictive changes. The trigger setting should open the demand valve to initiate inspiratory gas fl ow with minimal

(1–2 cmH2O) effort from the patient. PEEP may limit the tidal volume delivered in pressure- limited modes if the FRC increases enough to shorten the time during which inspiratory gas fl ow subsides or the pressure limit is achieved. Arterial blood gas monitoring thereafter should guide appropriate ventilator adjustments (17,29).

30. The answer is D. Answer 12 discusses factors that affect diffusion of oxygen to and through the alveolo-capillary membrane (ACM). Carbon monoxide is used as the indicator gas for the diffusing capacity testing identifi ed in the question. Normal values are predicted from calcu- lations based on the patient’s age, height, and sex. Test values are presented as milliliters of bulk fl ow of gas across the membrane/minute/1 mmHg pressure gradient for the gas across the membrane, and compared to the predicted value. Low diffusing capacity may result from anemia, abnormal hemoglobin types, pulmonary embolism, and interstitial lung disease. Another important variable introduced by this question is termed “time limits to diffusion” that relates oxygen uptake to the time the red blood cell remains available in the pulmonary capillary. A red blood cell normally remains in the pulmonary capillary about 0.75 seconds. If the cardiac output increases and the red blood cell’s available time falls to as short as 0.25 seconds, adequate oxygen diffusion can still occur as long as the ACM is normal. However, if a diffusion defect is present and cardiac output or capillary blood fl ow is increased, such as in sepsis, oxygen diffusion may not have time to occur, resulting in hypoxemia (15,17). Because the defect measured by a low diffusing capacity is not related to bronchospasm or airway closure, providing higher levels of positive airway pressure or bronchodilators will not be effective. Support from higher inspired oxygen will be effective as the transmembrane gradient for oxygen will be enhanced, but that choice was not provided.

31. The answer is D. During types of volume-limited mechanical ventilation (i.e., controlled [CMV], assist control [ACMV], and synchronized intermittent mandatory ventilation [SIMV]), a set tidal volume is delivered to the patient during ventilator-initiated inhala- tion. The resulting peak airway pressure, in the clinical scenario presented, is the param- eter that varies with ordered ventilator settings (dependent variable) and potentially risks barotrauma. Guidelines from the Acute Respiratory Distress Syndrome (ARDS) Network and other authoritative groups recommend that peak airway pressure be maintained less

than 40 cmH2O and that plateau airway pressure should be less than 30 to 35 cmH2O to minimize the risk of ventilator-related lung injury (35) (see Answer 3 for how to mea- sure plateau airway pressure). Other forms of lung injury, such as biotrauma (concurrent parenchymal infl ammatory conditions), volutrauma (overdistension of more compliant lung units), and atelectotrauma (repetitive stress factors related to opening and closing of small airways/alveoli) may also lead to ventilator-related lung injury. Acutely increased peak airway pressure observed at the bedside may represent mucus plugging, pneu- mothorax, patient arousal, kinking of the endotracheal tube, and so on, whereas a fall in peak airway pressure may herald a circuit leak (36).

257

66485457-66485438 www.ketabpezeshki.com

ZZakaria_87574_PTR_CH17_10-06-13_239-266.inddakaria_87574_PTR_CH17_10-06-13_239-266.indd 257257 66/19/2013/19/2013 8:47:508:47:50 PMPM GENERAL CRITICAL CARE: PATHOLOGY, PATHOPHYSIOLOGY, AND THERAPY

32. The answer is B. During pressure-limited or pressure-controlled ventilation (e.g., pressure support [PS], pressure assist control, pressure limited synchronized intermittent manda- tory ventilation (SIMV), and airway pressure release ventilation [APRV]), the variable that responds to various events at the bedside is the tidal volume. For example, if a pneu- mothorax or mucus plug occurs, the peak pressure created or achieved by the ventilator occurs earlier in the inspiratory time and infl ation is shortened or aborted, reducing tidal volume. Similarly, a substantial circuit leak would allow decompression of the infl ation fl ow seeking to achieve the pressure limit, and less tidal volume would be delivered.

33. The answer is C. The cycle time of the ventilator is the time allotted for one complete inhalation and exhalation and is determined by 60 seconds ÷ the set respiratory rate. Therefore, for an apneic patient with a set ventilator rate of 10, the cycle time is 6 sec- onds; that is, each 6 seconds a new inhalation will be initiated by the ventilator. The cycle time is divided into the inspiratory time (Ti) and the expiratory time (Te). The I:E ratio is a ratio of Ti:Te. By convention, Ti is given the value of 1 and the Te is calculated and displayed by the ventilator as a function of the Ti at 1. The normal I:E ratio during spontaneous breathing is about 1:2, indicating that normally twice as much time is avail- able for exhalation than is usually needed for inspiration. In chronic obstructive pulmo- nary disease (COPD), of course, this ratio may be 1:>2 as more time is taken to exhale. During mechanical ventilation, the primary determinants of Ti are tidal volume and the inspiratory fl ow rate generated by the ventilator. Therefore, as tidal volume increases, the time to deliver that volume (i.e., Ti) is prolonged, and as the fl ow rate of the ventila- tor decreases, Ti increases. Te is a passive variable as it represents what is left over in the cycle time once the Ti has been expended. It is, however, extremely important relative to the lung’s intrinsic elasticity and any expiratory resistance encountered. If inadequate Te is present, air “trapping” or “stacking” may occur and lead to auto-, intrinsic, or inadver- tent positive end-expiratory airway pressure (PEEP). Te may be shortened, placing the patient at risk of auto-PEEP by: a. Increasing the respiratory rate, thus shortening the cycle time, without compensatory reduction in tidal volume or increasing inspiratory fl ow rate b. Increasing tidal volume (prolongs Ti) c. Slowing the inspiratory fl ow rate (prolongs Ti). Auto-PEEP develops only in susceptible patients with intrinsic diminished elasticity or increased resistance (e.g., COPD, bronchoconstriction). Other patients who have restric- tive components to their disease (e.g., acute respiratory distress syndrome [ARDS] or acute lung injury [ALI]) may have increased elastic recoil and will tolerate shortened Te without developing auto-PEEP. Auto-PEEP can be measured with the assistance of the respiratory care practitioner and may be displayed by most modern ventilators, if the patient’s spontaneous breathing rate is not excessive (37).

34. The answer is F. As discussed previously, auto-PEEP results if the Te is too short to allow full exhalation. The answers proposed are either irrelevant (B) or actually prolong Te (A, C, D). Auto-PEEP is additive to the PEEP set on the ventilator to create an increased total PEEP. Total PEEP increases the functional residual capacity (FRC), defi ned as the gas remaining in the lung at the end of a resting exhalation. During the next ventila- tor infl ation, the tidal volume delivered by the ventilator (in a volume-limited mode of

258

66485457-66485438 www.ketabpezeshki.com

ZZakaria_87574_PTR_CH17_10-06-13_239-266.inddakaria_87574_PTR_CH17_10-06-13_239-266.indd 258258 66/19/2013/19/2013 8:47:508:47:50 PMPM PULMONARY PHYSIOLOGY AND MECHANICAL VENTILATION: Answers

ventilation) is added to the increased FRC and produces a concomitant elevated peak airway pressure with potential consequences of lung injury, reduced cardiac preload, and so on. If a pressure-limited mode of ventilation is being used, the increased total PEEP allows the set pressure limit to be reached more quickly during ventilator infl ation, thus reducing the tidal volume delivered (38).

35. The answer is D. During synchronized intermittent mandatory ventilation (SIMV), the set ventilator rate cycles in addition to the patient’s spontaneous breathing rate. In addi- tion, in all modes of ventilation the ventilator’s apnea alarm is always set by the respira- tory care practitioner. This alarm usually monitors the number of seconds (adjustable, but commonly 20 seconds) during which no gas fl ow through the circuit is detected by the ventilator. If the alarm parameter(s) is/are met, the ventilator will sound an audible apnea alarm and initiate backup ventilation, as determined by the ventilator manufac- turer (commonly 8–10 mL/kg, 100% oxygen at 10–12 breaths/minute) (37).

36. The answer is F. APRV is a pressure-limited mode of mechanical ventilation usually initiated to treat hypoxemia after initial therapy has resulted in high peak airway pressures or high

fraction of inspired oxygen (FIO2). APRV is intended to raise mean airway pressure so as to recruit underexpanded airways (shunt effect) and prevent derecruitment, while minimizing barotrauma risk. The four variables manipulated during APRV are time high (T high), time low (T low), pressure high (P high), and pressure low (P low). T high controls the duration the continuous positive airway pressure (CPAP) is held between release periods. The dura- tion of the release periods is the T low. P high controls the continuous airway pressure held, and P low determines the positive airway pressure during the release period (39,40).

37. The answer is C. The acute respiratory distress syndrome (ARDS) Network guidelines (35) recommend a protective strategy for lung ventilation in ALI that includes a tidal volume of 4 to 8 mL/kg of ideal body weight. Current or usual body weights, therefore, do not apply to this calculation. Because most patients are not at ideal body weight, it is useful for the clinician to recall the formula provided in the question for this calculation for males. The formula differs for women as: 105 + [5(height in inches) − 60)] (37).

38. The answer is F. The behavior described most likely indicates that the trigger sensitiv- ity on the ventilator is set too insensitively for this patient. Under this circumstance, the patient is unable to generate a suffi ciently strong subambient pressure (suck) to open the ventilator’s internal demand valve to initiate inspiratory gas fl ow. Other causes include auto-PEEP that must be overcome before a suffi cient negative pressure is created (although most modern ventilators are PEEP compensated, which eliminates this problem), injury to the abdomen or ribs that limits production of the negative pressure, overly compliant ventilator circuit tubing that deforms when the patient attempts to create the subambient pressure, and a defective/unresponsive demand valve. An insensitive demand valve set- ting will increase the patient’s work of breathing and can be detected by careful attention to the patient’s behavior. The same complication may occur during synchronized inter- mittent mandatory ventilation (SIMV) with pressure support (PS), continuous positive airway pressure (CPAP), or bilevel as the primary mode, if the sensitivity is not properly adjusted. Conversely, if the trigger is set too sensitively, the ventilator may “auto-cycle” and potentially risk hyperventilation (37).

259

66485457-66485438 www.ketabpezeshki.com

ZZakaria_87574_PTR_CH17_10-06-13_239-266.inddakaria_87574_PTR_CH17_10-06-13_239-266.indd 259259 66/19/2013/19/2013 8:47:508:47:50 PMPM GENERAL CRITICAL CARE: PATHOLOGY, PATHOPHYSIOLOGY, AND THERAPY

39. The answer is E. Plateau airway pressure is measured (and usually displayed on the ventilator) after insertion of a brief inspiratory pause or inspiratory hold into the infl a- tion cycle of the ventilator. This pause delays the opening of the exhalation valve in the ventilator and holds the inspired tidal volume in the lung. The hold allows turbulence in the trachea and proximal airways to subside and some of the tidal volume to move pas- sively into distal airways. The initial peak airway pressure noted at the end of inspiration will fall slightly to a lower pressure designated as the plateau airway pressure. As noted, it is considered a more accurate measure of distal airway pressure and is used to measure static lung/thoracic compliance. Acute respiratory distress syndrome (ARDS) Network guidelines recommend that the plateau pressure be periodically measured and ventilator

parameters be adjusted to maintain it below 30 cmH2O (38).

Paw = airway pressure waveforms during lung infl ation by the mechanical ventilator. The two waveforms on the right have no inspiratory hold. Peak inhalation airway pressure at the end of inspiration falls quickly during exhalation. The waveform on the left demonstrates an inspiratory “pause” or “hold.” Peak airway pressure falls to the plateau airway pressure during the “hold” time as gas turbulence ceases. The plateau pressure is read at the lower inspiratory pressure before the exhalation valve opens at the end of the “hold” time and airway pressure falls during exhalation.

40. The answer is C. Poiseuille’s Law identifi es variables that affect laminar fl ow of gases or low viscosity fl uids through tubes, such as an artifi cial airway: ΔP = n8lV ÷ π r4 where n is the viscosity of the gas, l is the tube length, V is the gas fl ow, and r is the radius of the tube (41). These variables clinically affect the process of weaning and the work of breathing nec- essary to overcome resistance from endotracheal or tracheostomy tubes. As expected, the smaller the tube diameter, the higher its resistance and the work of breathing at any minute ventilation, needed to overcome the resistance during spontaneous breathing (42–44).

260

66485457-66485438 www.ketabpezeshki.com

ZZakaria_87574_PTR_CH17_10-06-13_239-266.inddakaria_87574_PTR_CH17_10-06-13_239-266.indd 260260 66/19/2013/19/2013 8:47:508:47:50 PMPM PULMONARY PHYSIOLOGY AND MECHANICAL VENTILATION: Answers

41. The answer is E. The distribution of ventilation is dramatically changed when the nor- mal mechanical vector of the thorax favoring thoracic expansion is lost and spontaneous breathing is absent, as described in this question. Although the gravitational distribution of blood fl ow remains the same, a new force must be considered, the weight of the thoracic musculature, skin, bones, and breasts. As shown in the accompanying illustration, this weight increases along the anterior-to-posterior plane in the supine patient. The effect of this progressive anterior-to-posterior distribution of tissue weight is to change the path of least resistance for the ventilator’s inspiratory gas fl ow. The anterior lung units in the supine patient have the least external weight and become the path of least resistance into which gas fl ow is preferentially directed. However, the distribution of perfusion remains gravitationally dependent toward the posterior plane in the supine patient. Therefore, a ventilation:perfusion (V/Q) mismatch is generated in the apneic patient because of the loss of the subatmospheric pleural pres- sure generated during spontaneous breathing (38). This altered distri- bution is worsened when pharma- cological or injury-related muscle paralysis also eliminates the nor- mal tissue vector forces that tend to expand the thorax.

42. The answer is D. Despite the unusual pressure and time components during APRV ventilation, many patients comfortably maintain spontaneous breathing throughout T high and T low changes in airway pressure with minimal or no sedation. The sub- ambient pleural pressure generated during the spontaneous breaths allows improved distribution of ventilation to gravitationally dependent lung areas receiving blood perfusion (38). The unique characteristics of APRV, without clear inspiratory and expi- ratory cycles, reinforces the physiological fact that gas exchange occurs constantly during all forms of ventilation and does not depend on the particular phase of the ventilator’s cycle.

43. The answer is E. The general goal of pressure-limited (also termed pressure-controlled) ventilation is to assure that a set peak or plateau airway pressure is not exceeded, placing the patient at risk for lung injury or potentially generating a West’s zone 1 (see Answer 8 for discussion of West’s zones of the lung). The recommendation from the acute respiratory distress syndrome (ARDS) Network and other authorities is

that the plateau pressure should be below 30 to 35 cmH2O. As discussed in Answer 32, the dependent variable in any pressure-limited mode is tidal volume. It is sensi- tive to any change in the patient’s condition or response to the ventilator that might alter the conductance or resistance of the patient’s lung or thorax. Common pressure- limited modes include pressure-limited (PL) mandatory ventilation, PL-assist con- trol, PL-synchronized intermittent mandatory ventilation (SIMV), pressure support (PS), airway pressure release ventilation (APRV), continuous positive airway pressure (CPAP), bilevel positive airway pressure (BiPAP), and others (37).

261

66485457-66485438 www.ketabpezeshki.com

ZZakaria_87574_PTR_CH17_10-06-13_239-266.inddakaria_87574_PTR_CH17_10-06-13_239-266.indd 261261 66/19/2013/19/2013 8:47:518:47:51 PMPM GENERAL CRITICAL CARE: PATHOLOGY, PATHOPHYSIOLOGY, AND THERAPY

44. The answer is A. As an expiratory variable, PEEP halts expiratory gas fl ow at an above- ambient pressure, thereby retaining gas within the lung that would, without PEEP, escape to the atmosphere. The FRC is the gas remaining in the lung at the end of exhala- tion and is therefore increased by the gas detained because of the PEEP. Best or optimal PEEP, in concept, has historically been defi ned as the lowest PEEP that provides the benefi t(s) sought. Those benefi ts have been variously defi ned by authors as: the lowest

(or some predetermined) fraction of inspired oxygen (FIO2), least shunt, best oxygen

delivery, best lung/thoracic compliance and, most recently, as 2 cmH2O above the lower infl ection point on a static pressure–volume curve (37,45,46). All benefi ts seem related to stabilization of airways that are functionally or anatomically open by preventing their derecruitment (closure) during exhalation.

45. The answer is A. The intention of the inspiratory support provided by the venti- lation modes noted in the question is to reduce the work of breathing for the patient. The desired PS to overcome the imposed work of breathing can be calculated by the equation (37):

PSV = [(PIP − Pplat) × VI spont] ÷ Vmach where PIP is the peak airway pressure, Pplat is the plateau airway pressure, Vmach is

the ventilator fl ow during a machine infl ation in liters/second, and VI spont is the patient’s peak inspiratory fl ow in liters/second during a maximum inspiratory effort. Some elements of the preceding equation are diffi cult to measure, so usually a clini- cal examination to evaluate the overall patient’s ease of breathing plus monitoring the patient’s spontaneous respiratory rate and exhaled tidal volume provide ongoing assessment.

References

1. Beachey W. Regulation of breathing. In: Wilkins RL, Stoller JK, Kacmarek RM. Eds. Egan’s Fundamentals of Respiratory Care. 9th ed. St Louis: Mosby Elsevier; 2009:305–315. 2. Fagerlund MJ, Kahlin J, Ebberyd A, et al. The human carotid body: expression of oxygen sensing and signaling genes of relevance for anesthesia. Anesthesiology. 2010;113:1270–1279. 3. Garcia-Rio F, Villamor A, Gomez-Mendieta A, et al. The progressive effects of ageing on chemosensitivity in healthy subjects. Respir Med. 2007;101:2192–2198. 4. Ruppl GL. Ventilation. In: Wilkins RL, Stoller JK, Kacmarek RM, eds. Egan’s Fundamentals of Respiratory Care. 9th ed. St Louis: Mosby Elsevier; 2009:215–236. 5. Op’t Holt TB. Physiology of ventilatory support. In: Wilkins RL, Stoller JK, Kacmarek RM, eds. Egan’s Fundamentals of Respiratory Care. 9th ed. St Louis: Mosby Elsevier; 2009:1001–1042. 6. Fink J. Humidity and bland aerosol therapy. In: Wilkins RL, Stoller JK, Kacmarek RM, eds. Egan’s Fundamentals of Respiratory Care. 9th ed. St Louis: Mosby Elsevier; 2009: 775–799. 7. Wilkins RL. Physical principles of respiratory care. In: Wilkins RL, Stoller JK, Kacmarek RM, eds. Egan’s Fundamentals of Respiratory Care. 9th ed. St Louis: Mosby Elsevier; 2009:93–117.

262

66485457-66485438 www.ketabpezeshki.com

ZZakaria_87574_PTR_CH17_10-06-13_239-266.inddakaria_87574_PTR_CH17_10-06-13_239-266.indd 262262 66/19/2013/19/2013 8:47:518:47:51 PMPM PULMONARY PHYSIOLOGY AND MECHANICAL VENTILATION: Answers

8. West JB. Respiratory Physiology: The Essentials. 6th ed. Philadelphia: Lippincott Williams & Wilkins; 2000:45–61. 9. West, JB. Regional differences in the lung. Chest. 1978;74:426–437. 10. Wilkins RL. Gas exchange and transport. In: Wilkins RL, Stoller JK, Kacmarek RM, eds. Egan’s Fundamentals of Respiratory Care. 9th ed. St Louis: Mosby Elsevier; 2009: 237–260. 11. Xi XM, Jiang L, Zhu B, RM Group. Clinic effi cacy and safety of recruitment maneuver in patients with acute respiratory distress syndrome using low tidal volume ventilation: a multicenter randomized controlled clinical trial. Chin Med J. 2010;123:3100–105. 12. Marini JJ, Ravenscraft SA. Mean airway pressure: physiologic determinants and clini- cal importance—part I: physiologic determinants and measurements. Crit Care Med. 1992:20:1461–1421. 13. Wilkins RL. Analysis and monitoring of gas exchange. In: Wilkins RL, Stoller JK, Kacmarek RM, eds. Egan’s Fundamentals of Respiratory Care. 9th ed. St Louis: Mosby Elsevier; 2009: 365–97. 14. Siddiki H, Kojicic M, Li G, et al. Bedside quantifi cation of dead-space fraction using rou- tine clinical data in patients with acute lung injury: secondary analysis of two prospective trials. Crit Care. 2010;14:R141. 15. Wilkins RL. Gas exchange and transport. In: Wilkins RL, Stoller JK, Kacmarek RM, eds. Egan’s Fundamentals of Respiratory Care. 9th ed. St Louis: Mosby Elsevier; 2009: 237–260. 16. Moammar MQ, Azam HM, Blamoun AI, et al. Alveolar-arterial oxygen gradient, pneu- monia severity index and outcomes in patients hospitalized with community acquired pneumonia. Clin Exp Pharmacol Physiol. 2008;35:1032–1037. 17. Douce FH. Pulmonary function testing. In: Wilkins RL, Stoller JK, Kacmarek RM, eds. Egan’s Fundamentals of Respiratory Care. 9th ed. St Louis: Mosby Elsevier; 2009: 399–430. 18. Wilkins RL. Lung expansion therapy. In: Wilkins RL, Stoller JK, Kacmarek RM, eds. Egan’s Fundamentals of Respiratory Care, 9th ed. St Louis: Mosby Elsevier; 2009:903–919. 19. Raurich JM, Vilar M, Colomar A, et al. Prognostic value of the pulmonary dead-space fraction during the early and intermediate phases of acute respiratory distress syndrome. Respir Care. 2010;55:282–287. 20. Yoon YH, Lee SW, Jung DM, et al. The additional use of end-tidal alveolar dead space fraction following D-dimer test to improve diagnostic accuracy for pulmonary embolism in the emergency department. Emerg Med J. 2010;27:663–667. 21. Menadue C, Alison JA, Piter AJ, et al. High- and low-level pressure support during walk- ing in people with severe kyphoscoliosis. Eur Respir J. 2010;36:370–378. 22. Wilkins RL. Bedside assessment of the patient. In: Wilkins RL, Stoller JK, Kacmarek RM, eds. Egan’s Fundamentals of Respiratory Care. 9th ed. St Louis: Mosby Elsevier; 2009:319–342. 23. Blackwood B, Alderdice F, Burns K, et al. Use of weaning protocols for reducing duration of mechanical ventilation in critically ill adult patients: Cochran systematic review and meta-analysis. BMJ. 2011:342:c7237. doi:10.1136/bmj.c7237. 24. Shelledy, DC. Discontinuing ventilatory support. In: Wilkins RL, Stoller JK, Kacmarek RM, eds. Egan’s Fundamentals of Respiratory Care. 9th ed. St Louis: Mosby Elsevier; 2009:1153–1184.

263

66485457-66485438 www.ketabpezeshki.com

ZZakaria_87574_PTR_CH17_10-06-13_239-266.inddakaria_87574_PTR_CH17_10-06-13_239-266.indd 263263 66/19/2013/19/2013 8:47:518:47:51 PMPM GENERAL CRITICAL CARE: PATHOLOGY, PATHOPHYSIOLOGY, AND THERAPY

25. Monaco F, Drummond GB, Ramsay P, et al. Do simple ventilation and gas exchange mea- surements predict early successful weaning from respiratory support in unselected gen- eral intensive care patients? Br J Anaesth. 2010; 104:326–333. 26. Danaga AR, Gut AL, Antnes LC, et al. Evaluation of the diagnostic performance and cut- off value for the rapid shallow breathing index in predicting extubation. J Bras Pneumol. 2009;35:541–547. 27. Paltel KN, Ganatra KD, Bates JH, et al. Variation in the rapid shallow breathing index associated with common measurement techniques and conditions. Respir Care. 2009;54:1462–1466. 28. Schilz R. Neuromuscular and other diseases of the chest wall. In: Wilkins RL, Stoller JK, Kacmarek RM, eds. Egan’s Fundamentals of Respiratory Care. 9th ed. St Louis: Mosby Elsevier; 2009:609–623. 29. Shelledy, DC. Initiating and adjusting ventilatory support. In: Wilkins RL, Stoller JK, Kacmarek RM, eds. Egan’s Fundamentals of Respiratory Care. 9th ed. St Louis: Mosby Elsevier; 2009:1045–1090. 30. Guimaraes MM, ElDib R, Smith AF, et al. Incentive spirometry for prevention of post- operative pulmonary complications in upper abdominal surgery. Cochrane Database Syst Rev. 2009 Jul 8(3):CD006058. 31. Kulkarni SR, Fletcher E. McConnell AK, et al. Pre-operative inspiratory muscle train- ing preserves postoperative inspiratory muscle strength following major abdominal s urgery—a randomized pilot study. Ann R Coll Surg Engl. 2010;92:700–707. 32. Cabello B, Thille AW, Roche-Campo F, et al. Physiological comparison of three spon- taneous breathing trials in diffi cult-to-wean patients. Intensive Care Med. 2010;36: 1171–1179. 33. Robertson TE, Mann HJ, Hyzy R, et al. Multicenter implementation of a consensus- developed, evidence-based, spontaneous breathing trial protocol. Crit Care Med. 2008;36:2753–2762. 34. Wedewardt J, Bitter T, Prinz C, et al. Cheyne–Stokes respiration in heart failure: cycle length is dependent on left ventricular ejection fraction. Sleep Med. 2010;11:137–142. 35. Imai Y, Slutsky AS. High-frequency oscillatory ventilation and ventilator-induced lung injury. Crit Care Med. 2005; 33(Suppl): S129–S134. 36. The Acute Respiratory Distress Syndrome Network. Ventilation with lower tidal volumes as compared with traditional tidal volumes for acute lung injury and the acute respi- ratory distress syndrome. The Acute Respiratory Distress Syndrome Network. N Engl J Med. 2000;342:1301–1308. 37. Shelledy DC. Initiating and adjusting ventilatory support. In: Wilkins RL, Stoller JK, Kacmarek RM, eds. Egan’s Fundamentals of Respiratory Care. 9th ed. St Louis: Mosby Elsevier; 2009:1045–1090. 38. Op’t Holt, TB. Physiology of ventilatory support. In: Wilkins RL, Stoller JK, Kacmarek RM, eds. Egan’s Fundamentals of Respiratory Care. 9th ed. St Louis: Mosby Elsevier; 2009:1001–1043. 39. Powner DJ, Graham R. Airway pressure release ventilation during adult donor care. Prog Transplant. 2010;20:269–273. 40. Maxwell RA, Green JM, Waldrop J, et al. A randomized prospective trial of airway pres- sure release ventilation and low tidal volume ventilation in adult trauma patients with acute respiratory failure. J Trauma. 2010;69:501–510.

264

66485457-66485438 www.ketabpezeshki.com

ZZakaria_87574_PTR_CH17_10-06-13_239-266.inddakaria_87574_PTR_CH17_10-06-13_239-266.indd 264264 66/19/2013/19/2013 8:47:518:47:51 PMPM PULMONARY PHYSIOLOGY AND MECHANICAL VENTILATION: Answers

41. Ruppel G. Ventilation. In: Wilkins RL, Stoller JK, Kacmarek RM, eds. Egan’s Fundamentals of Respiratory Care. 9th ed. St Louis: Mosby Elsevier; 2009:215–260. 42. Flevari AG, Maniatis N, Dremiotis TE, et al. Rohrer’s constant, K2, as a factor of determin- ing inspiratory resistance of common adult endotracheal tubes. Anaesth Intensive Care. 2011;39:410–417. 43. Mehta S, Heffer MJ, Maham N, et al. Impact of endotracheal tube size on preextubation respiratory variables. J Crit Care. 2010;25:483–488. 44. Shapiro M, Wilson RK, Casar G, et al. Work of breathing through different sized endotra- cheal tubes. Crit Care Med. 1986;14:1028–1031. 45. Mercat A, Richard JM, Vielle B, et al. Positive end-expiratory pressure setting in adults with acute lung injury and acute respiratory distress syndrome. JAMA. 2008;299: 646–655. 46. Ward NS, Lin DY, Nelson DL, et al. Successful determination of lower infl ection point and maximal compliance in a population of patients with acute respiratory distress syn- drome. Crit Care Med. 2002;30:963–968.

265

66485457-66485438 www.ketabpezeshki.com

ZZakaria_87574_PTR_CH17_10-06-13_239-266.inddakaria_87574_PTR_CH17_10-06-13_239-266.indd 265265 66/19/2013/19/2013 8:47:518:47:51 PMPM 66485457-66485438 www.ketabpezeshki.com

ZZakaria_87574_PTR_CH17_10-06-13_239-266.inddakaria_87574_PTR_CH17_10-06-13_239-266.indd 266266 66/19/2013/19/2013 8:47:518:47:51 PMPM 18 Respiratory Diseases Imoigele P. Aisiku QUESTIONS

1. An adequate sputum specimen is identifi ed by: A. Greater than 10 epithelial cells, few polymorphonuclear (PMN) white blood cells, and many bacterial fl ora B. Greater than 10 epithelial cells, greater than 25 PMNs, and a predominant bacterial fl ora C. Less than 10 epithelial cells, few PMNs, and many bacterial fl ora D. Less than 10 epithelial cells, greater than 25 PMNs, and a predominant bacterial fl ora

2. Which of the following statements regarding the TB skin test is most accurate? A. It is positive in 100% of the patients with TB meningitis B. A positive reaction indicates the presence of an infection with Mycobacterium tuberculo- sis but not necessarily the presence of active disease C. It is always positive in patients infected with M. tuberculosis D. Criteria for interpreting the test as positive are the same for all patients regardless of background or concomitant illness

3. All of the following are appropriate in the acute treatment of aspiration syndromes, except: A. Administration of steroids and prophylactic antibiotics B. Administration of supplemental oxygen C. Bronchoscopy to remove large particles D. Use of continuous positive airway pressure (CPAP)

ANSWERS TO THIS SECTION CAN BE FOUND ON PAGE 273 267

66485457-66485438 www.ketabpezeshki.com

ZZakaria_87574_PTR_CH18_10-06-13_267-284.inddakaria_87574_PTR_CH18_10-06-13_267-284.indd 267267 66/19/2013/19/2013 8:48:048:48:04 PMPM GENERAL CRITICAL CARE: PATHOLOGY, PATHOPHYSIOLOGY, AND THERAPY

4. All of the following statements regarding the use of β-adrenergic agonists in the treat- ment of acute asthma are correct, except: A. The primary effect is in the large central airways B. They promote bronchodilation by increasing cyclic adenosine monophosphate (cAMP) C. Their onset of action is less than 5 minutes D. Agents with beta-2 selectivity are preferred

5. All of the following are causes of noncardiogenic pulmonary edema (NCPE), except: A. Fat embolism B. Fluid overload C. Drug overdose D. Multiple trauma

6. All of the following statements regarding drowning are accurate, except: A. Saltwater drowning is more common than freshwater drowning B. Males are more commonly affected than females regardless of age C. Children less than 5 years of age and teenagers are most commonly affected D. The fi nal common pathway in both “wet” and “dry” drowning is profound hypoxia

7. All of the following statements regarding noncardiogenic pulmonary edema are true, except: A. The heart size is small or normal B. Pulmonary artery wedge pressure (PAWP) is elevated C. Chest x-ray shows bilateral pulmonary infi ltrates D. Lung compliance is reduced

8. Acute respiratory distress syndrome (ARDS) most commonly occurs in association with: A. Aspiration B. Trauma C. Sepsis D. Massive blood transfusion

9. Which of the following is not part of rapid sequence intubation? A. Preoxygenation for 5 minutes B. Pretreatment with atropine C. Sedation D. Paralysis

10. What is the maximum safe dose of lidocaine for topical anesthesia of the airway? A. 3 mg/kg B. 5 mg/kg C. 1 mg/kg D. There is no toxic dose for topical lidocaine

268

66485457-66485438 www.ketabpezeshki.com

ZZakaria_87574_PTR_CH18_10-06-13_267-284.inddakaria_87574_PTR_CH18_10-06-13_267-284.indd 268268 66/19/2013/19/2013 8:48:048:48:04 PMPM RESPIRATORY DISEASES: Questions

11. What is the most likely predisposing factor in the development of aspiration pneumonia? A. Presence of food or particulate matter in aspirate B. Volume of gastric contents aspirated C. pH of the gastric contents D. Preexisting lung disease 12. All of the following interfere with the bellows function of the chest, except: A. Third-degree chest burn B. Flail chest C. Pneumonia D. Pneumothorax 13. What are the key factors in determining total work of breathing? A. Compliance and resistance B. Dead space and pulmonary shunting C. Tidal volume and respiratory rate D. Respiratory drive and airspace disease 14. Which of the following best describes West’s zone 3 lung physiology? A. Pa > Pv > PA B. PA > Pa > Pv C. Pa > PA > Pv D. Ppl > Pa > Pv 15. What is the proposed physiologic benefi t of Heliox in postextubation stridor? A. Reduces airway edema

B. Provides a low-density high fraction of inspired oxygen (FiO2) gas mixture C. Lowers the Reynolds number of the airway (airway resistance) D. Decreases patient anxiety 16. Match the following: A. Inspiratory reserve volume (IRV) 1. The volume of air breathed in and out without conscious effort B. Total lung capacity (TLC) 2. The additional volume of air that can be inhaled with maximum effort after a normal inspiration C. Vital capacity (VC) 3. The additional volume of air that can be forcibly exhaled after normal exhalation D. Expiratory reserve volume (ERV) 4. The total volume of air that can be exhaled after a maximum inhalation: TV + IRV + ERV E. Minute ventilation (MV) 5. The volume of air remaining in the lungs after maximum exhalation (the lungs can never be completely emptied) F. Tidal volume (TV) 6. VC + RV G. Residual volume (RV) 7. The volume of air breathed in 1 minute: (TV) (breaths/minute)

269

66485457-66485438 www.ketabpezeshki.com

ZZakaria_87574_PTR_CH18_10-06-13_267-284.inddakaria_87574_PTR_CH18_10-06-13_267-284.indd 269269 66/19/2013/19/2013 8:48:048:48:04 PMPM GENERAL CRITICAL CARE: PATHOLOGY, PATHOPHYSIOLOGY, AND THERAPY

17. All of the following are indications for a chest tube in parapneumonic effusions, except: A. Pus on gross visualization B. Pleural fl uid glucose less than 40 g/dL C. pH less than 7 D. Elevated lactate dehydrogenase (LDH)

18. Which of the following neuromuscular disorders is not acute in onset? A. Myasthenia gravis B. Guillain–Barré C. Organophosphate poisoning D. Amyotrophic lateral sclerosis (ALS)

19. What are the key parameters in extubating patients with neuromuscular disorders? A. Rapd Shallow Breathing Index (RSBI) less than 105 B. Passes a spontaneous breathing trial (SBT) C. Vital capacity/expiratory and inspiratory forces D. Minimal secretions and strong cough

20. All of the following are acute treatments for primary pulmonary hypertension (PPH), except: A. Calcium channel blockers B. Angiotensin-converting-enzyme (ACE) inhibitors C. Prostacyclin D. Nitric oxide

21. Advantages of tracheostomy include all of the following, except: A. Decreased airway resistance B. Decreased work of breathing C. Improved secretion clearance D. Lower incidence of nosocomial pneumonia E. Decrease in incidence of tracheal stenosis

22. Positive pressure ventilation (PPV) has all of the following effects, except: A. Increased preload B. Reduced afterload C. Ventilator-induced lung injury (VILI) D. Reduced respiratory work of breathing

23. Noninvasive positive pressure ventilation (NPPV) has all of the following advantages, except: A. Prevents speech B. Requires less sedation C. Less ventilator-associated pneumonia D. Decreased work of breathing (WOB)

270

66485457-66485438 www.ketabpezeshki.com

ZZakaria_87574_PTR_CH18_10-06-13_267-284.inddakaria_87574_PTR_CH18_10-06-13_267-284.indd 270270 66/19/2013/19/2013 8:48:048:48:04 PMPM RESPIRATORY DISEASES: Questions

24. Which is the best ventilator mode? A. Bilevel B. Synchronized intermittent mandatory ventilation (SIMV) C. Assist control (AC) D. Pressure support ventilation (PSV) E. None F. All the above

25. A 23-year-old man experienced a grand mal seizure and had a large witnessed aspiration of gastric material. He is now on assist-control ventilation. His ventilator settings are as

follows: Fraction of inspired oxygen (FiO2) 1.0; positive end-expiratory airway pressure (PEEP) 5; tidal volume (VT) 6 mL/kg; ABG 7.2/57/48; Pplat 29; adequate expiratory time. What, if anything, should be done? A. Increase PEEP B. Reduce VT and increase rate C. Increase PEEP, reduce VT, and increase rate D. Increase PEEP and reduce VT E. Increase VT

26. A 22-year-old, 85-kg male with asthma is admitted with dyspnea. Despite bronchodila- tors and steroids, he has ongoing respiratory distress and is intubated and placed on assist control (AC) ventilation with a set rate of 10, tidal volume of 600, and a positive end-expiratory airway pressure (PEEP) of 0. Fifteen minutes after intubation, he is noted to be struggling to breathe with a respiratory rate of 40 and a systolic blood pressure of 60. What should be done immediately? A. Decrease tidal volume to 500 mL B. Switch patient to pressure support C. Temporarily disconnect patient from ventilator D. Increase PEEP to 10 E. Decrease respiratory rate to 8

27. A 50-year-old male nonsmoker, house painter by occupation, presents to the ED with fever, chills, cough, expectoration, and pleuritic chest pain for 3 days. On admission, MAP is 48. WBC count is 24, lactic acid is 2, creatinine is 1.3, and right lower lobe (RLL) infi l- trate on chest x-ray (CXR). He is admitted and receives 2 L of normal saline, ceftriaxone,

and levofl oxacin. In the ICU, HR is 124, BP is 98/60, and O2 saturation is 93% on non- rebreather mask (NRB). In addition to continuing fl uids and antibiotics, which of the fol- lowing is the best therapeutic approach? A. Methylprednisone 125 mg q6 B. No corticosteroids C. Hydrocortisone 50 mg IV q6 D. Hydrocortisone 100 mg IV q6 with fl udrocortisone 0.1 mg orally

271

66485457-66485438 www.ketabpezeshki.com

ZZakaria_87574_PTR_CH18_10-06-13_267-284.inddakaria_87574_PTR_CH18_10-06-13_267-284.indd 271271 66/19/2013/19/2013 8:48:048:48:04 PMPM GENERAL CRITICAL CARE: PATHOLOGY, PATHOPHYSIOLOGY, AND THERAPY

28. A 50-year-old male is in the ICU after undergoing colostomy, sigmoid resection, and Hartman’s pouch procedures for perforated diverticulitis. His vital signs are: BP 110/

60 mmHg, HR 100, and RR 18. On the ventilator his tidal volume is 700 mL, FiO2 90%, and

PEEP 15 cmH2O. His laboratory fi ndings are: Hgb 13, WBC 16, ABG: pH 7.4, PCO2 31, PO2

60, SaO2 91%, VBG: pH 7.42, PCO2 34, PO2 35, SaO2 64%. What is the AV DO2? A. 1% to 5% B. 6% to 10% C. 11% to 15% D. Greater than 15%

29. A decrease in the pulmonary capillary wedge pressure is seen with all of the following agents, except: A. Isoproterenol B. Dopamine C. Dobutamine D. Milrinone

30. A 45-year-old male with a medical his- tory of alcohol abuse presents to the hospital with persistent abdominal pain, fever, nausea, and vomiting. His initial vitals are temperature 101°F, HR 110, RR 32, and BP 100/72. He has midepigas- tric tenderness on exam. While NPO, the patient aspirates and his chest x-ray is shown in the accompanying image. Labs

are: ABG 7.37/35/82 on 100% FiO2, WBC 18K. What is the diagnosis? A. Pulmonary edema B. Aspiration pneumonia C. Acute respiratory distress syndrome (ARDS) D. Acute lung injury (ALI) E. Pancreatitis

272

66485457-66485438 www.ketabpezeshki.com

ZZakaria_87574_PTR_CH18_10-06-13_267-284.inddakaria_87574_PTR_CH18_10-06-13_267-284.indd 272272 66/19/2013/19/2013 8:48:058:48:05 PMPM 18

ANSWERS

1. The answer is C. Routine sputum samples are not recommended in the intensive care setting for diagnosing pneumonia. Bronchoscopic or blind (nonbronchoscopic) broncho- alveolar lavage (BAL) is recommended. In the nonintubated patient, a routine sputum sample may be helpful but requires an adequate sample that has minimum saliva. An adequate sample is characterized by few epithelial cells and signifi cant presence of poly- morphonuclears (PMNs). In a BAL sample, 104 organisms are considered diagnostic and 103 in a protected sputum brush (PSB). Initial Gram stain should help with initial antibi- otic selection.

2. The answer is B. The prognosis in patients with pulmonary TB and acute respiratory failure requiring mechanical ventilation is believed to be poor. The hospital mortality has been reported to be approximately 60% for patients with respiratory failure due to pulmonary TB. By contrast, the mortality for patients with respiratory failure due to severe pneumonia is only about 25%. The most common reasons for ICU admissions in patients with TB are the development of acute respiratory distress syndrome (ARDS) and severe organ failure, such as renal failure. Tuberculin testing is helpful in ranking tuberculosis among the differential diagnoses of conditions with symptoms, signs, and radiological changes that would be compatible with pulmonary tuberculosis but where sputum is negative on direct smear or culture. A strongly positive tuberculin test in such a patient, who has not previously had bacille Calmette-Guérin (BCG) vaccination or tuberculosis, increases the probability that tuberculosis is the diagnosis. In those who have previously received BCG vaccination, γ interferon tests will differentiate between that and M. tuberculosis infection as a cause of the strongly positive tuberculin test. Chest radiography, when available and reliable, is an important investigation, especially when clinical suspicion of tuberculosis exists but the sputum is negative. Fluffy upper

273

66485457-66485438 www.ketabpezeshki.com

ZZakaria_87574_PTR_CH18_10-06-13_267-284.inddakaria_87574_PTR_CH18_10-06-13_267-284.indd 273273 66/19/2013/19/2013 8:48:058:48:05 PMPM GENERAL CRITICAL CARE: PATHOLOGY, PATHOPHYSIOLOGY, AND THERAPY

zone shadowing, frequently bilateral and often associated with cavitation, is classic, as is miliary shadowing. New, soft shadowing among old, fi brotic changes often indicates relapse of previous disease. Paratracheal, mediastinal, and hilar lymphadenopathy are not unusual in African and Indian patients with tuberculosis. In patients infected with HIV, the radiological appearances are often less specifi c, symptoms and signs may not be classical, and sputum may be negative on direct smear. Standard chemotherapy, as recommended by the British Thoracic Society, International Union Against Tuberculosis and Lung Disease, World Health Organization, and National Institute for Health and Care Excellence (NICE), consists of 6 months of rifampicin and isoniazid (usually given as combination tablets), initially supplemented by 2 months of pyrazinamide and etham- butol. Pyridoxine is indicated only in malnourished patients or those with conditions predisposing to peripheral neuropathy.

3. The answer is A. Aspiration is a process in which matter present in the oropharynx is carried through inhalation into the lower tracheobronchial tree. Usually the matter is cleared from the lungs; however, if the person is compromised in some fashion or inhales large amounts of matter, pneumonia can result. Aspiration includes several aspiration syndromes: i. Chemical pneumonitis: Aspiration of toxic agents such as gastric acid; instantaneous injury with marked hypoxemia. Treatment requires positive-pressure ventilatory sup- port. The condition is known as Mendelson’s syndrome. ii. Refl ex airway closure: Aspiration of inert fl uids (water, saline, and nasogastric feed- ings) may produce laryngospasm and pulmonary edema with resultant hypox-

emia. Treatment includes intermittent positive-pressure breathing with 100% O2 and isoproterenol. iii. Mechanical obstruction: Aspiration of inert fl uid or particulate matter (partially digested food, hot dogs, and peanuts) may produce simple mechanical obstruction. Coughing, wheezing, and dyspnea occur with atelectasis seen on x-ray of the chest. Treatment requires tracheobronchial suction and removal of the particulate matter by fi beroptic bronchoscopy. iv. Aspiration pneumonia: Aspiration of bacteria from the oropharynx. Patients have a cough, fever, purulent sputum, and radiographic evidence of infi ltrate. Treatment requires antibiotics.

4. The answer is A. Beta-adrenergic agents have cardiac and pulmonary effects. In particu- lar, beta-2 agonists function on the beta-2 receptors on bronchial smooth muscle to achieve bronchodilation without the tachycardia associated with activation of beta-1 receptors on cardiac muscle. The beta-2 receptor is a G protein–coupled transmitter that mediates its activity through cAMP. The onset of short-acting agents is typically less than 5 minutes. Long-acting beta-agonists may take up to 20 minutes for onset of action and therefore should be avoided as sole acute management agents. Acute management can be a com- bination therapy of long- and short-acting agents. Steroids should be considered manda- tory therapy in established asthma and chronic obstructive pulmonary disease (COPD) patients, especially in the ICU setting.

5. The answer is B. Pulmonary edema is defi ned as excess fl uid shift into the alveolar spaces from various etiologies. The two most common forms of pulmonary edema are those initiated by an imbalance of Starling forces (hydrostatic pulmonary edema) and

274

66485457-66485438 www.ketabpezeshki.com

ZZakaria_87574_PTR_CH18_10-06-13_267-284.inddakaria_87574_PTR_CH18_10-06-13_267-284.indd 274274 66/19/2013/19/2013 8:48:058:48:05 PMPM RESPIRATORY DISEASES: Answers

those initiated by disruption of alveolar-capillary membrane (NCPE). The Starling rela- tionship that predicts the net fl ow of liquid across a membrane can be expressed in the following equation: Net fi ltration = (Lp × S) × (Δ hydrostatic pressure − Δ oncotic pressure) Hydrostatic pulmonary edema is usually cardiogenic. The usual causes of systolic and dia- stolic left ventricular dysfunction (coronary artery disease, myocarditis, cardiomyopathy, hypertension, congenital heart diseases, and so on) are responsible for the development of acute pulmonary edema. Usual triggering factors are acute ischemia, myocardial infarction, rhythm or conduction abnormalities, and high blood pressure. Many condi- tions are associated with pulmonary edema that appears to be due to diffuse damage and increased permeability of the alveolar-capillary membrane. These conditions include infections (bacterial, viral, and parasitic), septicemia, trauma, and disseminated intravas- cular coagulation. In addition, shock lung in association with nonthoracic trauma, acute hemorrhagic pancreatitis, inhalation of toxic gases (smoke, ozone, cadmium, phosgene, chlorine, and nitrogen dioxide), circulating foreign substances (snake venom, alloxan, and alphanaphthyl thiourea), endogenous vasoactive substances (histamine, kinins), burns, aspiration of gastric contents, inhalation of foreign bodies, acute radiation pneumonitis, and drowning have also been implicated in the development of pulmonary edema.

6. The answer is A. Drowning usually occurs silently and rapidly. Drowning may be further classifi ed as cold-water or warm-water injury. Warm-water drowning occurs at water tem- peratures of 20°C or higher, and cold-water drowning occurs at water temperatures of less than 20°C. Although ice-cold water has been reported to be protective, especially in young children, prolonged immersions can nullify the effect of temperature on survivability. Additional classifi cation may include the type of water in which the submersion occurred, such as freshwater and saltwater, or natural bodies of water versus man-made. Although initial treatment of submersion victims is not affected by the type of water, serum electrolyte derangements may be related to the salinity of the water (particularly if large amounts of water are ingested), while long-term infectious complications are primarily related to whether the victim was submerged in a natural or a man-made body of water. Immediate threats include effects on the central nervous and cardiovascular systems. The most critical actions in the immediate management of drowning victims include prompt correction of hypoxemia and acidosis. The degree of central nervous system (CNS) injury depends on the severity and duration of hypoxia. Posthypoxic cerebral hypoperfusion may occur. Long-term effects of cerebral hypoxia, including vegetative survival, are the most devastating. Drowning is the sixth leading cause of accidental death for people of all ages and the second leading cause of death for children aged 1 to 14 years, after motor vehicle collision. This averages out to about 10 deaths per day. Approximately one-quarter of these deaths occur in children 14 years of age or younger. A bimodal distribution of deaths is observed, with an initial peak in the tod- dler age group and a second peak in adolescent to young adult males.

7. The answer is B (see Answer 5).

8. The answer is C. ARDS is believed to occur when a pulmonary or extrapulmonary insult causes the release of infl ammatory mediators, promoting neutrophil accumula- tion in the microcirculation of the lung. Neutrophils damage the vascular endothelium

275

66485457-66485438 www.ketabpezeshki.com

ZZakaria_87574_PTR_CH18_10-06-13_267-284.inddakaria_87574_PTR_CH18_10-06-13_267-284.indd 275275 66/19/2013/19/2013 8:48:058:48:05 PMPM GENERAL CRITICAL CARE: PATHOLOGY, PATHOPHYSIOLOGY, AND THERAPY

and alveolar epithelium, leading to pulmonary edema, hyaline membrane formation, decreased lung compliance, and diffi cult air exchange. Most cases of ARDS are asso- ciated with pneumonia or sepsis. It is estimated that 7.1% of all patients admitted to an ICU and 16.1% of all patients on mechanical ventilation develop acute lung injury (ALI) or ARDS. In-hospital mortality related to these conditions is between 34% and 55%, and most deaths are due to multiorgan failure. In 1994, the American–European Consensus Conference on ARDS defi ned both ALI and ARDS. These defi nitions have been widely adopted by clinicians and researchers. ALI requires all four of the follow- ing features in patients who have a risk factor for ARDS and no history of chronic lung disease: i. Acute onset ii. Bilateral infi ltrates (radiographically similar to pulmonary edema) iii. No evidence of elevated left atrial pressure (the pulmonary capillary wedge pressure is less than or equal to 18 mmHg if measured)

iv. A ratio of arterial oxygen tension to fraction of inspired oxygen (PaO2/FiO2) of

201 mmHg to 300 mmHg. ARDS is the same, except the PaO2/FiO2 is less than or equal to 200 mmHg.

Thus, only the PaO2/FiO2 distinguishes ALI from ARDS (see x-ray). Treatment of ARDS is supportive and includes mechanical ventila- tion, prophylaxis for stress ulcers and venous thromboembolism, nutritional support, and treatment of the underlying injury. Low tidal volume, high positive end-expiratory pres- sure, and conservative fl uid therapy may improve outcomes. A spontaneous breathing trial is indicated as the patient improves and the underlying illness resolves. Patients who survive ARDS are at risk of diminished func- tional capacity, cognitive impairment, and decreased quality of life; ongoing care by a primary care physician is benefi cial for these patients.

9. The answer is B. Rapid sequence intubation (RSI) is the preferred method of endotra- cheal intubation because it results in rapid unconsciousness (induction) and neuro- muscular blockade (paralysis). This is important in patients who have not fasted and are at much greater risk for vomiting and aspiration. To this end, the goal of RSI is to intubate the trachea without having to use bag-valve-mask (BVM) ventilation, which is often necessary when attempting to achieve intubating conditions with sedative agents alone (e.g., midazolam, diazepam). Instead of titrating to effect, RSI involves admin- istration of weight-based doses of an induction agent (e.g., etomidate) immediately followed by a paralytic agent (e.g., succinylcholine, rocuronium) to render the patient unconscious and paralyzed within 1 minute. This method has been proven safe and effective over the past two decades, and it is considered the standard of care. When administered by experienced, well-trained intensivists, the use of neuromuscular

276

66485457-66485438 www.ketabpezeshki.com

ZZakaria_87574_PTR_CH18_10-06-13_267-284.inddakaria_87574_PTR_CH18_10-06-13_267-284.indd 276276 66/19/2013/19/2013 8:48:058:48:05 PMPM RESPIRATORY DISEASES: Answers

blocking agents in patients undergoing emergent tracheal intubation is associated with a signifi cant decrease in procedure-related complications. Certain clinical sce- narios may call for pretreatment medications prior to induction/paralysis to optimize physiologic parameters for intubation, such as blunting the sympathetic response to laryngoscopy, preventing upward or downward spikes in blood pressure, avoiding increased intracranial pressure, and facilitating bronchodilation. These conditions include suspected high intracranial pressure (e.g., intracranial hemorrhage or trauma), severe asthma or chronic obstructive pulmonary disease (COPD), hypovolemic shock, and aortic emergencies.

10. The answer is B. Topical anesthetics are partially absorbed into the bloodstream and in high doses may lead to cardiovascular (arrhythmias) and neurologic (seizures) toxicity. Lidocaine is one of the safer topical anesthetics but can still lead to these complications if improperly applied. To prevent the risk of these toxic side effects, it is important to limit the volume of topically applied lidocaine to doses well within the safety range. Lidocaine is the most commonly used local anesthetic for a wide range of procedures. It is typically used in 1% solution; a 0.5% solution can be used when larger volumes are desired, and 2% solution, common in dentistry, is also often available in emergency departments. The maximum dose of lidocaine 1% is 4.5 mg/kg. Lidocaine’s duration of action is typically up to 1 hour. Lidocaine is available with and without epinephrine (1:100,000). Epinephrine causes local vasoconstriction, which limits local spread of the drug, therefore allowing a larger dose of drug to be infi ltrated without causing systemic toxicity. The maximum dose of lidocaine 1% with epinephrine is 7 mg/kg (0.7 mL/kg, maximum dose of 50 mL in a 70-lb adult).

11. The answer is D (see Answer 3). Aspiration toxicity is usually determined by the volume and pH of gastric contents and the presence and type of particulate matter.

12. The answer is C. The chest bellows component of the respiratory system includes the bony thoracic cage that contains the lungs; the diaphragms, which are the major mus- cles of breathing; and pleural membranes, thin tissues that line both the outside of the lungs and the inside of the thoracic cage. The thoracic or chest cage consists of the ribs that protect the lungs from injury, the muscles and connective tissues that tie the ribs together, and all the nerves that lead into these muscles. The chest bellows is syn- onymous with the respiratory pump, which is the component of the respiratory system excluding the lungs. Some diseases that impact respiration affect the pump or central nervous system (CNS) without any direct lung disease.

13. The answer is A. The total work of breathing is the sum of the elastic and resistive work. Resistive work (resistance) decreases with increasing lung volumes and widening air- ways, and elastic work (compliance) increases at both high and low lung volumes. In obstructive lung disease, resistance increases with no change in compliance. And in restrictive lung disease, compliance decreases with no change in resistance. In the ICU setting, most patients have compliance detriments and increased work of breathing as a result. Mechanical ventilation should be employed in ratio to the degree of decreased work of breathing required to maintain adequate gas exchange, while treating the under- lying lung disorder.

277

66485457-66485438 www.ketabpezeshki.com

ZZakaria_87574_PTR_CH18_10-06-13_267-284.inddakaria_87574_PTR_CH18_10-06-13_267-284.indd 277277 66/19/2013/19/2013 8:48:058:48:05 PMPM GENERAL CRITICAL CARE: PATHOLOGY, PATHOPHYSIOLOGY, AND THERAPY

14. The answer is A. West’s zones within the lung are three vertically split zones (in the upright subject), which explain how alveoli, arterial, and venous pressures differ in each zone and thus affect perfusion and ventilation throughout the lung. In zone 1, the alveolar pressure may exceed the arterial and venous pressure and thus little perfusion will occur as the vessels collapse; this then leads to dead space. In zone 2, the arterial pressure will exceed that of the alveoli but not the venous pressure. In zone 3, both the arterial and the venous pressures exceed the alveolar pressure.

15. The answer is C. After endotracheal extubation, upper and total airway resistances may frequently be increased, resulting in high inspiratory effort to breathe. A few patients, ranging from 5% to 16%, develop postextubation airway obstruction and frank respira- tory distress. In addition, a substantial number of patients develop inspiratory distress after extubation, leading to reintubation. In these patients an increase in upper airway and total inspiratory resistance may also contribute to respiratory distress. Helium–oxygen

(HeO2; Heliox) mixture has a low density and a high kinematic viscosity, allowing for a reduction in airway resistance. Some studies have shown that it could have benefi cial effects in the treatment of upper airway obstruction. Previous studies have demonstrated that breathing a helium–oxygen mixture reduces dyspnea and improves gas exchange in nonintubated patients with severe exacerbation of chronic obstructive lung disease or asthma. Recently, helium–oxygen breathing was shown to improve pH, partial pres-

sure of carbon dioxide (PaCO2), and inspiratory effort in patients with acute exacerba- tion of chronic obstructive pulmonary disease (COPD), and these benefi cial effects were enhanced by combining noninvasive pressure support ventilation with helium–oxygen. The use of helium–oxygen in the postextubation period where the upper and total airway resistances may be increased could also decrease the patient’s effort to breathe in this situ-

ation. Note that helium–oxygen combinations provide a lower fraction of inspired (FiO2), hence, hypoxic patients may not tolerate Heliox as well.

16. The answers are: A → 2; B → 6; C → 4; D → 3; E → 7; F → 1; G → 5 • IRV: The additional volume of air that can be inhaled with maximum effort after a nor- mal inspiration • TLC: = VC + RV • VC: The total volume of air that can be exhaled after a maximum inhalation: VC = TV + IRV + ERV • ERV: The additional volume of air that can be forcibly exhaled after a normal exhalation • MV: The volume of air breathed in 1 minute: (TV) (breaths/minute) • TV: The volume of air breathed in and out without conscious effort • RV: The volume of air remaining in the lungs after maximum exhalation (the lungs can never be completely emptied)

Measurements of lung volumes are important to confi rm or clarify the nature of lung disorders. The fl ow volume loop may indicate an obstructive, restrictive, or obstruc- tive/restrictive pattern, but a further test of lung volume is often necessary for clarifi ca- tion. In obstructive lung disease, airway obstruction causes an increase in resistance. During normal breathing, the pressure volume relationship is no different from that in a

278

66485457-66485438 www.ketabpezeshki.com

ZZakaria_87574_PTR_CH18_10-06-13_267-284.inddakaria_87574_PTR_CH18_10-06-13_267-284.indd 278278 66/19/2013/19/2013 8:48:058:48:05 PMPM RESPIRATORY DISEASES: Answers

normal lung. However, when breathing rapidly, greater pressure is needed to overcome the resistance to fl ow, and the volume of each breath gets smaller. Increase in the effort to breathe can cause an overdistention of the lungs. The fl ow volume loop may show lower than normal forced expiratory volume in 1 second (FEV1) and forced expiratory fl ow between 25% and 75% (FEF25–75), but it is not until a lung volume has been determined that an increase in total lung capacity (TLC), functional residual capacity (FRC), and residual volume (RV) can be confi rmed. Common obstructive diseases include asthma, bronchitis, and emphysema. In restrictive lung disease, the compliance of the lung is reduced, which increases the stiffness of the lung and limits expansion. In these cases, a greater pressure than normal is required to allow the same increase in volume. The fl ow volume loop may show lower than normal forced vital capacity (FVC), but the FEV1 and FEF25–75 may be only mildly affected. The lung volume measurement will clearly show a reduction in TLC, FRC, and RV. Common causes of decreased lung compliance are pulmonary fi brosis, pneumonia, and pulmonary edema. Patients whose respiratory muscles are unable to perform normally because of a neuromuscular disease or paralysis can show a restrictive pattern.

17. The answer is B. Pleural fl uid effusion can be classifi ed as either transudative or exu- dative. An effusion is deemed exudative if pleural fl uid protein/serum protein ratio is greater than 0.5, or pleural fl uid LDH/serum LDH is greater than 0.6, or pleural fl uid LDH is greater than two-thirds of the upper normal limit for serum. Other commonly used tests include total fl uid protein of more than 3.0 g/dL and a specifi c gravity greater than 1.016. If none of these criteria are met, then the effusion is a transudate and the underlying medical condition is treated. Drainage is needed only if there is respiratory dysfunction. If the effusion is exudative, additional studies such as cytology, amylase, complete blood count, bacterial cultures, and pH are obtained. Surgery is more commonly indicated. The presence of bacteria indicates empyema.

18. The answer is D. In about 10% of cases, ALS is caused by a genetic defect. In the remaining cases, the cause is unknown. In ALS, motor neurons (anterior horn cells) waste away or die, and can no longer send messages to muscles. This eventually leads to muscle weaken- ing, twitching, and an inability to move the arms, legs, and body. When the muscles in the chest area are involved, it becomes hard or impossible to breathe on one’s own. Compared to organophosphate poisoning, myasthenia, or Guillain–Barré, ALS is more indolent and chronic in its development. Acute respiratory failure is the most feared complication in patients with neuromuscular disease. The decision to intubate these patients should be made earlier rather than later, to avoid emergency intubation or cardiorespiratory arrest. Regular assessment for clinical signs of respiratory-muscle fatigue and objective monitor- ing of vital capacity (VC), inspiratory, and expiratory forces are essential to determine the appropriate timing of intubation and mechanical ventilation. The morbidity and mortal- ity of patients who require mechanical ventilation are not insubstantial. Once they are intubated, attention to prevention of complications and prompt treatment of complica- tions, such as ventilator-associated pneumonia, may improve patient outcomes.

19. The answer C. In patients with respiratory failure due to neuromuscular disease, the deci- sion to extubate can be a challenging one, and no studies have specifi cally addressed wean- ing and extubation in this patient population. In general, patients should have adequate

279

66485457-66485438 www.ketabpezeshki.com

ZZakaria_87574_PTR_CH18_10-06-13_267-284.inddakaria_87574_PTR_CH18_10-06-13_267-284.indd 279279 66/19/2013/19/2013 8:48:058:48:05 PMPM GENERAL CRITICAL CARE: PATHOLOGY, PATHOPHYSIOLOGY, AND THERAPY

cough, should have few secretions, and should tolerate a low level of pressure support for a prolonged period without signs of fatigue. Objective measures used for extubation typi- cally mirror the objective measures known to predict intubation. Factors associated with progression to respiratory failure include a vital capacity (VC) less than 20 mL/kg, nega-

tive inspiratory force (NIF) less than −30 cmH2O, or a reduction of 30% in VC.

20. The answer is B. Calcium channel blockers may alleviate pulmonary vasoconstriction and prolong life in about 20% of patients with primary pulmonary hypertension (PPH). Unfortunately, there is no way to predict which patients will respond to orally administered vasodilators, and these drugs frequently have signifi cant adverse effects. Consequently, it is helpful to evaluate pulmonary vaso-reactivity during catheterization, before a long-term therapy is selected. The most suitable drugs for testing acute response are potent, short- acting, and titratable. In patients who show evidence of an acute hemodynamic response, long-term treatment with calcium channel blockers, administered orally in high dosages, can produce a sustained hemodynamic response and increase survival. Epoprostenol (Flolan), or prostacyclin, is the single most important advance in the treatment of primary pulmonary hypertension. This potent, short-acting vasodilator and inhibitor of platelet aggregation is produced by vascular endothelium. In one study, continuous intravenous infusion of epo- prostenol improved exercise capacity, quality of life, hemodynamics, and long-term survival in patients with class III or IV function. Although the delivery system for continuous infusion is complex, most patients are able to learn how to prepare and infuse the drug. Inhaled nitro- gen oxide (NO) has been used in the treatment of persistent pulmonary hypertension of the newborn, primary pulmonary hypertension in adults, acute respiratory distress syndrome (ARDS), and postoperative graft dysfunction after lung and heart transplantation. Clinical and hemodynamic benefi cial effects of long-term NO inhalational therapy in patients with severe primary pulmonary hypertension have been reported in the literature.

21. The answer is D. Reduction in nosocomial pneumonia may be a potential benefi t, which may be more of an indirect benefi t if able to liberate from the ventilator sooner. Timing is probably the most signifi cant decision; if prolonged intubation is anticipated, then early tracheostomy is recommended. The dogma of 7 to 14 days is becoming more controversial, and recent data suggest no difference in outcomes between early and late tracheostomy. Tracheal stenosis is a potential concern increased with endotrachial tubes (ET) but may not be signifi cant. Recent advances in low-pressure cuffs and ET tubes have decreased overall incidence of stenosis with ET tubes. Decreased airway resistance and work of breathing occur due to a shorter tube, occasionally of a larger diameter.

22. The answer is A. Decreased preload: • Positive alveolar pressure → lung volume → compression of the heart by the infl ated lungs → the intramural pressure of the heart cavities rises (e.g., ↑ RAP) → venous return decreases → preload is reduced → stroke volume decreases → cardiac out- put and blood pressure may drop. This can be minimized with IV fl uids, which help restore adequate venous return and preload. • Patients who are very sensitive to changes in preload (e.g., presence of hypovolemia, tamponade, pulmonary embolism, severe air trapping) are particularly prone to hypotension when PPV is initiated.

280

66485457-66485438 www.ketabpezeshki.com

ZZakaria_87574_PTR_CH18_10-06-13_267-284.inddakaria_87574_PTR_CH18_10-06-13_267-284.indd 280280 66/19/2013/19/2013 8:48:058:48:05 PMPM RESPIRATORY DISEASES: Answers

Reduced afterload: • Lung expansion increases extramural pressure (which helps pump blood out of the thorax) and thereby reduces left ventricular (LV) afterload. • In conditions when the cardiac performance is mainly determined by changes in after- load rather than in preload (e.g., hypervolemic patient with systolic heart failure), PPV may be associated with an improved stroke volume. PPV is very helpful in patients with cardiogenic pulmonary edema, as it helps to reduce preload (lung congestion) and afterload. As a result, stroke volume tends to increase. Potential detrimental effects associated with PPV: • Heart and circulation i. Reduced venous return and afterload ii. Hypotension and reduced cardiac output • Lungs i. Barotrauma ii. Ventilator-induced lung injury iii. Air trapping • Gas exchange i. May increase dead space (compression of capillaries) ii. Shunt (e.g., unilateral lung disease—the increase in vascular resistance in the normal lung associated with PPV tends to redirect blood fl ow in the abnormal lung)

23. The answer is D. Noninvasive PPV (NPPV) has become a standard therapy for the treat- ment of acute respiratory failure in selected populations and is increasingly being used in the critical and acute care settings. This increased utilization has been driven in large part by the desire to avoid complications of invasive mechanical ventilation, such as ven- tilator-induced lung injury (VILI), ventilator-associated pneumonia and other nosocomial infections, patient discomfort, and increased sedation needed with invasive mechanical ventilation. In addition, NPPV has been shown to reduce the rates of endotracheal intuba- tion, mortality, and hospital length of stay in specifi c conditions, particularly hypercarbic respiratory failure due to exacerbations of chronic obstructive pulmonary disease (COPD) and hypoxic respiratory failure in immunocompromised hosts. In addition, NPPV is ben- efi cial in many patients with cardiogenic pulmonary edema in the absence of acute coro- nary ischemia. Patient selection and understanding of contraindications is a key.

24. The answer is E. Understanding principles of mechanical ventilation, respiratory mechanics, and the underlying disease process is best practice. No ventilator mode has been proven to be superior to another in any trial including acute respiratory distress syndrome (ARDS).

25. The answer is C. The patient has aspirated and has profound hypoxemia with concomi- tant hypercarbia and acidosis. His Pplat is approaching 30 and he is at risk for worsening or developing ventilator-induced lung injury (VILI). Increasing PEEP will improve oxy- genation and recruit atelectatic lung units. Decreasing tidal volume will decrease plateau but may worsen acidemia. The combination of increasing rate to maximize minute venti- lation and protect from VILI gives the patient the best chance.

281

66485457-66485438 www.ketabpezeshki.com

ZZakaria_87574_PTR_CH18_10-06-13_267-284.inddakaria_87574_PTR_CH18_10-06-13_267-284.indd 281281 66/19/2013/19/2013 8:48:068:48:06 PMPM GENERAL CRITICAL CARE: PATHOLOGY, PATHOPHYSIOLOGY, AND THERAPY

26. The answer is C. The goal of therapy in obstructive lung disease and mechanical ventila- tion is to rest the respiratory muscles and prevent dynamic hyperinfl ation or auto-PEEP. Differential diagnosis should include drugs and tension pneumothorax. The patient has most likely developed auto-PEEP and is stacking breaths due to increased work of breath- ing (WOB); disconnecting from ventilator is best.

27. The answer is B. The patient has community-acquired pneumonia and meets criteria for sepsis. He was initially hypotensive but responded to fl uids. The role of low-dose steroids in sepsis is controversial. CORTICUS study did not show mortality improvement with steroids, although there were fewer sick patients than in the study by Annane.

28. The answer is A. AV DO 2 (arterio-venous difference in O2 content) is calculated by sub- tracting the mixed venous blood oxygen content from the arterial blood oxygen content and is most accurate using a pulmonary artery (PA) catheter.

AV DO2 = CaO2 – CvO2,

where CaO2 = [(Hgb × 1.34 × SaO2) + (PaO2 × 0.003)] and CvO2 = [(Hgb × 1.34 × SvO2) +

(PvO2 × 0.003). The content 1.34 is the number of milliliters of oxygen that can bind to a gram of hemo-

globin. The amount of unbound oxygen dissolved in the blood is represented by (PO2 ×

0.003). The normal AV DO2 is approximately 5% volume. A hypermetabolic state is pres-

ent if the AV DO2 is less than 5% volume. Any cause of a low cardiac output will elevate the oxygen content difference.

29. The answer is B. Epinephrine and dopamine, depending on the dosage, either leave the pulmonary capillary wedge pressure unchanged or increase it, based on the amount of vasoconstriction. Dobutamine and isoproterenol hydrochloride increase cardiac output, decrease systemic vascular resistance, and decrease the pulmonary capillary wedge pres- sure. Milrinone is believed to cause vasodilation by increasing intracellular cyclic ade- nosine monophosphate (cAMP) levels in smooth muscle. The action of milrinone is not related to sympathetic stimulation.

30. The answer is C. ARDS is associated with a generalized systemic infl ammatory response syndrome (SIRS) and is defi ned as an acute onset of noncardiogenic pulmonary edema resulting from an increase in lung permeability leading to intractable hypoxemia and bilateral diffuse infi ltrates on chest x-ray. Aspiration pneumonia and pancreatitis are

well-established causes for ARDS. Hypoxemia is defi ned as PaO2/FiO2 ratio less than 200 mmHg regardless of the level of positive end-expiratory airway pressure (PEEP). A wedge pressure of less than 18 mmHg defi nes noncardiogenic edema. Ventilation is very diffi cult due to poor compliance that results from the protein leak into the lungs. The clinicians should be familiar with the ventilator they are using and the mode of ventila- tion used to facilitate ventilation. Application of PEEP usually improves oxygenation, but increases the likelihood of barotrauma; therefore, a ventilating plateau pressure of

no greater than 35 cmH2O should be tolerated. Tidal volumes should be decreased down

to 5 to 6 mL/kg as necessary. An acceptable SaO2 is greater than 90%. To maintain lower plateau pressures, permissive hypercapnia is tolerated except in acute head or neurologi- cally injured patients. Other possible considerations include sedation, paralysis, prone

282

66485457-66485438 www.ketabpezeshki.com

ZZakaria_87574_PTR_CH18_10-06-13_267-284.inddakaria_87574_PTR_CH18_10-06-13_267-284.indd 282282 66/19/2013/19/2013 8:48:068:48:06 PMPM RESPIRATORY DISEASES: Answers

positioning, steroids (controversial with data showing mixed reports and one report sug- gesting increased mortality in some populations), and inhaled nitric oxide.

References

1. Tintinalli JE, Stapczynski JS, John MO, et al. Tintinalli’s Emergency Medicine: A Compre- hensive Study Guide. 7th ed. New York, NY: McGraw-Hill; 2011. 2. Pinsky MR, Mancebo J, Brouchard L, et al. Applied Physiology in Intensive Care Medicine. 2nd ed. New York, NY: Springer; 2009. 3. Papadakos PJ, Lachmann B. Mechanical Ventilation: Clinical Applications and Pathophysiology. 1st ed. Philadelphia, PA: Saunders/Elsevier; 2007. 4. Mason RJ, Broaddus VC, Martin T, et al. Murray & Nadel’s Textbook of Respiratory Medicine. 5th ed. Philadelphia, PA: Saunders/Elsevier; 2010 5. Irwin RS, Rippe JM. Irwin & Rippe’s Intensive Care Medicine. 7th ed. Philadelphia, PA: Lippincott Williams & Wilkins; 2011. 6. Bach JR. Noninvasive Mechanical Ventilation, 1st ed. Philadelphia, PA: Hanley & Belfus; 2002.

283

66485457-66485438 www.ketabpezeshki.com

ZZakaria_87574_PTR_CH18_10-06-13_267-284.inddakaria_87574_PTR_CH18_10-06-13_267-284.indd 283283 66/19/2013/19/2013 8:48:068:48:06 PMPM 66485457-66485438 www.ketabpezeshki.com

ZZakaria_87574_PTR_CH18_10-06-13_267-284.inddakaria_87574_PTR_CH18_10-06-13_267-284.indd 284284 66/19/2013/19/2013 8:48:068:48:06 PMPM 19 Renal Diseases Ala Abudayyeh and Maen Abdelrahim QUESTIONS

1. A 27-year-old-man came to the ED after a gunshot wound to the chest. Postoperatively, he is in stable condition. BP is 100/80 mmHg, HR 120, and T 98.5°F. He is intubated and sedated on ventilatory support with the following settings: AC at the rate of 12, TV

600, and O2 saturation 98% on FiO2 of 60%. Laboratory fi ndings are as follows: serum Na 150 mEq/L, serum K 3.5 mEq/L, serum chloride 100 mEq/L, serum bicarbonate 30 mEq/L, BUN 120 mg/dL, and creatinine 1.0 mg/dL. He has been on tube feeds and is tolerating them well. He is noted to be having 3 L of urine output Q8 hours with urine Osm > 300 mosmol/kg. The most likely cause for polyuria and hypernatremia in this patient is: A. Polyuria is a result of increasing IV fl uids B. Hypernatremia is a result of central diabetes insipidus C. Increased osmotic load with high protein intake in tube feeds D. Postobstructive diuresis

ANSWERS TO THIS SECTION CAN BE FOUND ON PAGE 292 285

66485457-66485438 www.ketabpezeshki.com

ZZakaria_87574_PTR_CH19_10-06-13_285-298.inddakaria_87574_PTR_CH19_10-06-13_285-298.indd 285285 66/19/2013/19/2013 4:46:364:46:36 PMPM GENERAL CRITICAL CARE: PATHOLOGY, PATHOPHYSIOLOGY, AND THERAPY

2. A 23-year-old African American male who has a history of obsessive–compulsive disor- der (OCD) and heavy tobacco use presents to the ED with pain and fatigue. He has had 2 days of nausea with one episode of vomiting. He has been taking paxil prescribed for his OCD symptoms. In the ED, his laboratory fi ndings are as follows: serum Na 114 mEq/L, serum K 3.5 mEq/L, serum chloride 90 mEq/L, serum bicarbonate 23 mEq/L, BUN 10 mg/dL, creatinine 0.5 mg/dL, urine Osm 350 mosmol/kg, serum Osm 200 mosmol/kg, urine Na 70 mEq/L, and urine K 35 mEq/L. BP is 130/80 mmHg, orthostatics are negative, and HR 80/minute. The patient is alert and shows no indication of confusion. What is the cause of his hyponatremia? A. Increased free water intake B. Syndrome of inappropriate antidiuretic hormone (SIADH) secretion due to use of selective serotonin reuptake inhibitors (SSRIs) C. Poor oral intake D. SIADH due to lung cancer

3. A 60-year-old white female admitted with increasing abdominal distention was found to have a small bowel obstruction (SBO) on x-ray. A nasogastric tube (NGT) was placed to suc- tion and normal saline (NS) was infused while awaiting surgical consult. Her blood work was as follows: serum Na 140 mEq/L, serum K 3.4 mEq/L, serum chloride 85 mEq/L, serum bicarbonate 35 mEq/L, BUN 40 mg/dL, and creatinine 1.4 mg/dL. Urinary chlo- ride is 8. How would you correct her alkalosis? A. Increase the IV fl uid rate and correct her hypokalemia

B. Discontinue NS and start NaHCO3 infusion C. Clamp NGT D. Start infusing 0.1% of HCl via central line E. Initiate patient on hemodialysis with low bicarbonate dialysate solution

4. A 70-year-old white male with a history of diabetes, hypertension, and chronic obstruc- tive pulmonary disease (COPD) presented to the ED with a COPD exacerbation. He was in severe respiratory distress and therefore intubated. His arterial blood gas (ABG) was as

follows: pH 7.35, pCO2 55 mmHg, and pO2 40 mmHg. His respiratory rate (RR) on the ven- tilator was increased and he was later noted to be unresponsive. Oxygen saturation was greater than 90%. His serum electrolytes were: serum Na 130 mEq/L, serum K 4.6 mEq/L, serum chloride 90 mEq/L, serum bicarbonate 40 mEq/L, BUN 30 mg/dL, and creatinine 1.5 mg/dL. What is the cause of his neurological decline? A. An acute stroke B. Acute correction of his acidosis with intubation and increased RR on ventilator C. Hyponatremia D. Pneumothorax

286

66485457-66485438 www.ketabpezeshki.com

ZZakaria_87574_PTR_CH19_10-06-13_285-298.inddakaria_87574_PTR_CH19_10-06-13_285-298.indd 286286 66/19/2013/19/2013 4:46:364:46:36 PMPM RENAL DISEASES: Questions

5. A 30-year-old male presents to the ED with altered mental status and combative behavior. He is found to have the following labs: serum Na 135 mEq/L, serum K 4.0 mEq/L, serum chloride 90 mEq/L, serum bicarbonate 28 mEq/L, BUN 28 mg/dL, creatinine 1.5 mg/ dL, and CK 300,000. Urine drug screen is positive for cocaine and LSD. He has multiple bruises and a fractured femur. How would you most effectively treat this patient’s acute renal failure? A. Start normal saline (NS) infusion 1 L to 2 L/hour to maintain urine output of 200 mL to 300 mL/hour

B. Start D5W with 150 mEq of NaHCO3 infusion C. Hemodialysis D. Mannitol infusion E. Lasix infusion

6. A 40-year-old male with bipolar disorder presents to the ED with ataxia, tremors, and con- fusion. His lithium level was noted to be 3.0. Serum Na 140 mEq/L, serum K 3.5 mEq/L, serum chloride 100 mEq/L, serum bicarbonate 28 mEq/L, BUN 30 mg/dL, and creatinine 1.5 mg/dL. What is the next step in management? A. Normal saline (NS) infusion B. Mannitol infusion C. Lasix infusion D. Administration of activated charcoal E. Hemodialysis

7. A 70-year-old African American male presented with urinary retention and pain for the last week. A Foley catheter was placed in the ED and greater than 1 L of clear urine was drained and continued to fl ow. Labs are as follows: serum Na 140 mEq/L, serum K 5.0 mEq/L, serum chloride 100 mEq/L, serum bicarbonate 20 mEq/L, BUN greater than 100 mg/dL, and creatinine 10 mg/dL. What is the best step in managing this patient? A. Keep the Foley catheter and start benign prostatic hypertrophy (BPH) treatment with follow-up with urology in 1 week B. Infuse 1 L to 2 L of normal saline (NS) in the emergency room and discharge home with follow-up in 1 week with indwelling Foley catheter C. Instruct patient on intermittent self-catheterization and place on BPH medications with follow-up D. Admit patient to the hospital and continue to follow urine output closely. Start 0.45% NaCl infusion and adjust fl uids as needed with frequent lab checks to ensure electro- lyte stability

287

66485457-66485438 www.ketabpezeshki.com

ZZakaria_87574_PTR_CH19_10-06-13_285-298.inddakaria_87574_PTR_CH19_10-06-13_285-298.indd 287287 66/19/2013/19/2013 4:46:364:46:36 PMPM GENERAL CRITICAL CARE: PATHOLOGY, PATHOPHYSIOLOGY, AND THERAPY

8. A 40-year-old white female with a history of uncontrolled type 2 diabetes (insulin depen- dent) and hypertension presented to the ED with a lower extremity ulcer and erythema. She was admitted with a diagnosis of cellulitis, and was started on nafcillin with WBC count decreasing from 35 to 15 in 2 days. On day 6 of treatment, she was noted to have a rash and fever of 102°F. Her creatinine continued to rise from 1.0 to 10 in 5 days. What is the cause of her acute kidney injury, and how would you treat it? A. Acute tubular necrosis (ATN) secondary to infection and sepsis. Attain cultures and add broader antibiotic coverage. B. Acute interstitial nephritis (AIN) secondary to nafcillin. Would stop nafcillin now and start steroids 1 mg/kg immediately if creatinine does not improve. C. Obstruction and urinary retention secondary to her diabetes. Would place Foley now. D. Immune (idiopathic) thrombocytopenic purpura (ITP) due to nafcillin. Would stop nafcillin and start prednisone 1 mg/kg. 9. A 70-year-old white male with a history of neuro-endocrine tumor of unknown origin presents with hypokalemia and metabolic alkalosis. Serum K 2.4 mEq/L, serum bicar- bonate 30 mEq/L. Blood pressure is elevated at 180/90 mmHg. How would you manage this patient? A. Replete potassium and continue to follow up his labs B. Add aldactone or amiloride and titrate as tolerated to keep potassium within a normal range C. Start acetazolamide to prevent proximal bicarbonate reabsorption D. Start hydrochloric acid infusion 10. A 60-year-old with type 2 diabetes, hypertension, hyperlipidemia, and stage 3 chronic kidney disease, with a baseline creatinine of 2.0 mg/dL, presents with chest pain. She is found to have elevated troponins and is diagnosed with a non-ST-elevation myocar- dial infarction (NSTEMI). Cardiology will take the patient for cardiac catheterization but needs recommendations for renal protection: A. Would not recommend cardiac catheterization and follow-up for now B. Would proceed with cardiac catheterization with infusion of normal saline (NS) or sodium bicarbonate at 1 mL/kg for 6 to 8 hours prior to the procedure and continue for 6 to 8 hours thereafter C. Mucomyst only D. Hemodialysis to be performed prophylactically after contrast administration 11. A 28-year-old female with a history of heavy alcohol use was found obtunded on the side of the road with a bottle of a sweet-smelling unknown substance. Her labs are as follows: serum Na 135 mEq/L, serum K 3.5 mEq/L, serum chloride 100 mEq/L, serum bicarbonate 10 mEq/L, BUN 20 mg/dL, creatinine 1.5 mg/dL, and glucose 120 mg/ dL, serum Osm 330 mosmol/kg, ABG 7.23/25/98, tachypenia with RR of 30/minute, BP 80/40 mmHg, T 97°F, and HR 110/min. What would be your next step? A. Perform emergent hemodialysis to help in clearing probable ethylene glycol B. Start ethanol (EtOH) infusion to counteract ethylene glycol breaking down into its toxic metabolites C. Administer fomepizole to inhibit alcohol dehydrogenase D. Administer activated charcoal E. Start sodium bicarbonate infusion

288

66485457-66485438 www.ketabpezeshki.com

ZZakaria_87574_PTR_CH19_10-06-13_285-298.inddakaria_87574_PTR_CH19_10-06-13_285-298.indd 288288 66/19/2013/19/2013 4:46:364:46:36 PMPM RENAL DISEASES: Questions

12. An 18-year-old who suffered a severe head trauma while riding his motorcycle is admitted to the neuroscience ICU, intubated, and found to have a subarachnoid hemorrhage with cerebral edema and intracranial pressures measured in the 30s. Over the next few days he has an acute drop in his serum sodium to 112 mEq/L. Other laboratory fi ndings are as fol- lows: serum K 4.0 mEq/L, serum chloride 74 mEq/L, serum bicarbonate 25 mEq/L, BUN 20 mg/dL, creatinine 0.9 mg/dL, serum Osm 282 mosmol/kg, urine Osm 368 mosmol/kg, urine Na 112 mEq/L, and urine K 26 mEq/L. His systolic blood pressures have decreased to the 90s with decreased skin turgor and he has increased urine output of 9 L/24 hours. What is the etiology of patient’s polyuria and hyponatremia? A. Central diabetes insipidus B. Physiologic water diuresis in response to hyponatremia C. Physiological salt diuresis D. Syndrome of inappropriate antidiuretic hormone secretion (SIADH) E. Cerebral salt wasting

13. A 60-year-old male with a history of type 2 diabetes, hypertension, and bipolar disorder, on lithium for years, is admitted to the hospital with a diagnosis of small bowel obstruction (SBO). He undergoes nasogastric tube (NGT) placement and is started on normal saline (NS) at 75 mL/hour. He is later noted to be more lethargic and his labs are as follows: serum Na 160 mEq/L, serum K 3.5 mEq/L, serum chloride 124 mEq/L, serum bicarbonate 23 mEq/L, BUN 32 mg/dL, creatinine 1.5 mg/dL, glucose 125 mg/dL, Urine Osm is 250mosmol/kg and serum Osm 331 mosmol/kg. His urine output is 2 L every 8 hours. What is the cause of his hypernatremia? A. Central diabetes insipidus B. Osmotic diuresis C. Increased insensible water losses D. Nephrogenic diabetes insipidus E. Hypernatremia secondary to NS

14. A 60-year-old patient with type 2 diabetes, hypertension, and stage 3 chronic kidney disease with a glomerular fi ltration rate (GFR) of 35 mL/min per 1.73 m2 presents with uncontrolled hypertension. There is a concern for renal artery stenosis, and magnetic resonance angiog- raphy (MRA) with gadolinium is planned for further evaluation. Would you proceed with MRA? A. Yes, accepted GFR for gadolinium infusion is greater than 20 mL/min per 1.73 m2 B. Yes, accepted GFR for gadolinium infusion is greater than 30 mL/min per 1.73 m2 C. MRA with gadolinium is contraindicated in this patient. D. Start IV fl uids and mucomyst prior to administration of gadolinium

289

66485457-66485438 www.ketabpezeshki.com

ZZakaria_87574_PTR_CH19_10-06-13_285-298.inddakaria_87574_PTR_CH19_10-06-13_285-298.indd 289289 66/19/2013/19/2013 4:46:364:46:36 PMPM GENERAL CRITICAL CARE: PATHOLOGY, PATHOPHYSIOLOGY, AND THERAPY

15. A 50-year-old male with type 2 diabetes mellitus, hypertension, and congestive heart fail- ure (ejection fraction [EF] < 20%) presents with pulmonary edema and acute distress.

BP 80/40 mmHg, HR 120/minute, O2 saturation 90% on 100% non-rebreather mask,

and RR greater than 25/minute. ABG: pH 7.57, pCO2 25 mmHg, pO2 60 mmHg, serum bicarbonate 22 mEq/L, serum Na 140 mEq/L, serum K 2.9 mEq/L, and serum chloride 100 mEq/L. What is the acid base disturbance? A. Acute metabolic acidosis B. Acute respiratory alkalosis C. Acute respiratory alkalosis with metabolic alkalosis D. Acute respiratory alkalosis, metabolic acidosis, and metabolic alkalosis E. Acute respiratory alkalosis and metabolic acidosis

16. A 35-year-old male with a history of alcohol abuse presented to the emergency depart- ment after being found confused on the street. Serum Na was 133 mEq/L, serum K 3.2 mEq/L, serum chloride 86 mEq/L, serum bicarbonate 14 mEq/L, BUN 12 mg/dL, crea-

tinine 1.0 mg/dL, albumin 2.0 g/dL, ABG: pH 7.38, pCO2 20 mmHg, and pO2 90 mmHg. What is the patient’s acid–base disorder? A. Metabolic acidosis, metabolic alkalosis, and respiratory alkalosis B. Metabolic acidosis C. Hyperchloremic metabolic acidosis D. Metabolic acidosis and metabolic alkalosis E. Metabolic acidosis and respiratory acidosis

17. A 70-year-old male with a ruptured abdominal aortic aneurysm was taken to the operat- ing room and required multiple units of packed red blood cells (PRBC). Postoperatively, he was in stable condition and his labs were as follows: serum Na 135 mEq/L, serum K 5.0 mEq/L, serum chloride 110 mEq/dL, serum bicarbonate 35 mEq/L, BUN

30 mg/dL, and creatinine 1.5 mg/dL. ABG: pH 7.47, pCO2 50 mmHg, pO2 90 mmHg. What is the cause of the patient’s metabolic alkalosis? A. Hyperventilation B. Nasogastric tube (NGT) suctioning C. Total parenteral nutrition (TPN) D. Massive PRBC transfusion E. Ischemia

18. A 25-year-old patient with a history of anorexia nervosa presents to the ED with fatigue and systolic blood pressures in the 80s. Her labs are as follows: serum Na 135 mEq/L, serum K 2.8 mEq/L, serum chloride 80 mEq/L, serum bicarbonate 35 mEq/L, BUN 30 mg/dL, creatinine 1.0 mg/dL, urine Na 30 mEq/L, urine K 35 mEq/L, and urine chloride 30 mEq/L. What is the cause of this patient’s metabolic alkalosis? A. Patient has been taking high quantities of baking soda as a diet regimen B. Patient has been using diuretics such as lasix C. Patient has been inducing nausea and vomiting D. Patient has been using excessive amount of laxatives and has profuse diarrhea E. Patient has hyperaldosteronism

290

66485457-66485438 www.ketabpezeshki.com

ZZakaria_87574_PTR_CH19_10-06-13_285-298.inddakaria_87574_PTR_CH19_10-06-13_285-298.indd 290290 66/19/2013/19/2013 4:46:364:46:36 PMPM RENAL DISEASES: Questions

19. A 30-year-old African American female is admitted for sickle cell crisis. She is noted to have the following labs: serum Na 135 mEq/L, serum K 5.0 mEq/L, serum bicarbonate 20 mEq/L, serum chloride 110 mEq/L, BUN 15 mg/dL, creatinine 1.0 mg/dL, and urine pH greater than 5.5. Patient is started on normal saline (NS) infusion and pain medica- tions. What is the acid–base abnormality and the cause? A. Metabolic acidosis secondary to increased lactic acid with vaso-occlusive disease B. Alkalosis due to excess mineralo-corticoid release C. Hyperkalemia leading to acidosis D. Non–anion gap (AG) metabolic acidosis secondary to a type 1 renal tubular acidosis

20. A 55-year-old white female with diabetes, hypertension , and coronary artery disease was admitted with a ST elevation myocardial infarction (STEMI). She underwent emergent cardiac catheterization and stenting of the left anterior descending artery and circumfl ex arteries, followed by an intra-aortic balloon pump placement (IABP). On day 2 of admis- sion, her BUN (mg/dL) and creatinine (mg/dL) increased from 10/1.0 to 30/1.5 with a decrease in urine output. Upon examination, her legs were mottled and ulcerated. What is the cause of her acute kidney injury (AKI)? A. Cholesterol emboli causing tubular obstruction and acute tubular necrosis (ATN) B. Contrast-induced nephropathy C. Cardio-renal syndrome leading to prerenal azotemia D. Urinary obstruction

21. A 60-year-old white male with chronic obstructive pulmonary disease (COPD), hyper- tension, type 2 diabetes, stage 3 chronic kidney disease, and morbid obesity was admit- ted for left lower-leg swelling. Ultrasound indicated acute deep vein thrombosis and he was started on a therapeutic dose of IV heparin. Two days later, he was noted to have the following labs: serum Na 135 mEq/L, serum K 6.0 mEq/L, serum bicarbonate 23 mEq/L, serum chloride 110 mEq/L, BUN 30 mg/dL, and creatinine 2.0 mg/dL. What is the cause of the patient’s hyperkalemia? A. Insulin defi ciency B. Renal insuffi ciency C. Heparin infusion D. Distal renal tubular acidosis

291

66485457-66485438 www.ketabpezeshki.com

ZZakaria_87574_PTR_CH19_10-06-13_285-298.inddakaria_87574_PTR_CH19_10-06-13_285-298.indd 291291 66/19/2013/19/2013 4:46:364:46:36 PMPM 19

ANSWERS

1. The answer is C. This is a typical cause of polyuria and hypernatremia in the ICU setting. In osmotic diuresis, nonelectrolyte solutes such as glucose, mannitol, or urea increase the output of urine. The concentration of sodium plus potassium is well below the plasma concentration. In this case, the likely high protein load in the tube feeds will need to be reduced, and free water needs to be repleted to meet the patient’s current losses. In central or nephrogenic diabetes insipidus the urine is dilute. Hypernatremia in the ICU setting has been associated with higher mortality and prolonged hospitalization (1).

2. The answer is B. When evaluating hyponatremia, it is essential to assess the volume status of the patient. BP and orthostatics are normal; therefore, he has hyponatremia with euvolemic volume status. The patient has a state of increased antidiuretic hormone (ADH) secondary to nausea and vomiting. This is evident in the concentrated urine and high urine electrolytes, suggesting that the patient has been retaining free water. Other causes are pain and use of SSRIs. All of these factors cause an increase in ADH secretion, and therefore free water retention. In order to correct this, the patient should be placed on volume restriction, stop SSRIs, suppress nausea and vomiting, and control his pain. Because his vitals are stable and there is no indication of volume depletion, IV fl uids should be held for now. If fl uids were to be given in this setting, they would worsen the hyponatremia as a result of retention of more free water. Once his increased ADH stimulus is suppressed, he will start urinating more free water and will start correcting quickly. This is where careful monitoring with frequent serum sodium and urine electro- lytes should be ordered (2).

3. The answer is A. The patient has overall volume depletion as a result of the NGT suction. This increases aldosterone levels, which subsequently increases H/ATPase transporter in the intercalated cells of the collecting tubule of the nephron. This mediates further bicarbonate

292

66485457-66485438 www.ketabpezeshki.com

ZZakaria_87574_PTR_CH19_10-06-13_285-298.inddakaria_87574_PTR_CH19_10-06-13_285-298.indd 292292 66/19/2013/19/2013 4:46:364:46:36 PMPM RENAL DISEASES: Answers

reabsorption. In the presence of nasogastric tube (NGT) suctioning there is also loss of HCl. Chloride depletion alone can lead to worsening alkalosis (3). This is mediated by a reduc- tion in bicarbonate excretion by increasing distal reabsorption in the intercalated type A cells and reducing distal secretion in type B intercalated cells. The low tubular concentra- tion of chloride will result in decreased function of the chloride/bicarbonate transporter and will worsen the alkalosis. Hypokalemia can lead to worsening alkalosis due to distal hydrogen secretion mediated by H-K-ATPase exchange pumps in the luminal membrane that actively reabsorb potassium as well as secrete hydrogen. This will lead to further bicar- bonate reabsorption. Therefore, to treat this condition, the NS infusion should be increased and potassium repleted, while the patient’s SBO resolves or until surgical intervention (4).

4. The answer is B. The patient is a chronic retainer of carbon dioxide, and acutely correct- ing his hypercapnia would lead to metabolic alkalosis. Acutely raising the intracerebral pH can lead to severe brain damage and death. In addition, posthypercapnic alkalemia can decrease cardiac output, decrease cerebral blood fl ow, and lead to seizures (5). Hyponatremia is not severe enough to lead to neurological changes. There is no sugges- tion of an acute stroke or pneumothorax at this point.

5. The answer is A. Volume expansion is the key to treating severe rhabdomyolysis. NS is

proven to be most effective. Another choice would be NaHCO3 solution to prevent heme– protein precipitation and acute kidney injury (AKI), although there is not enough data to suggest that it is superior to NS. The goal would be to keep urinary pH above 6.5 if using bicarbonate fl uids. There is no current indication for hemodialysis. Mannitol and lasix may help in increasing urinary fl ow and prevent further precipitation of heme-proteins, but there are no data available to establish improved outcomes (6).

6. The answer is E. The patient has severe lithium toxicity. Per guidelines, hemodialysis should be performed to remove lithium if the level is greater than 4 regardless of the presence of neurological changes. In this case, the patient would need to undergo hemo- dialysis because he has both acute kidney injury (AKI) and a Li level greater than 2.5 with neurological changes. Hemodialysis should be performed for a longer period of time (4–6 hours) to ensure a lesser rebound effect after completion. Follow-up Li levels should be obtained. IV fl uids are indicated but will not be effective enough to help the kidney clear such a toxic level. The other modalities have not shown effectiveness in the literature (7).

7. The answer is D. The patient likely has BPH and would benefi t from keeping the Foley catheter in place. Urology should be consulted and BPH medications started. However, it would not be appropriate to discharge a patient with this level of obstruction and acute kidney injury (AKI). The patient will develop postobstructive diuresis due to the intrinsic damage to the renal concentrating ability and therefore lose free water with consequent hypernatremia. A 0.45% NaCl infusion should be started with close electrolyte monitor- ing until blood urea nitrogen (BUN) and creatinine improve and polyuria resolves. Down- regulation of aquaporin 2 (AQP-2) channels due to bilateral ureteral obstruction has been shown to be the cause of slow recovery in concentration capacity in rats (8).

8. The answer is B. The patient developed acute kidney injury (AKI), rash, and fever acutely after a few days of administration of nafcillin. This is AIN. The sharp rise in her serum

293

66485457-66485438 www.ketabpezeshki.com

ZZakaria_87574_PTR_CH19_10-06-13_285-298.inddakaria_87574_PTR_CH19_10-06-13_285-298.indd 293293 66/19/2013/19/2013 4:46:364:46:36 PMPM GENERAL CRITICAL CARE: PATHOLOGY, PATHOPHYSIOLOGY, AND THERAPY

creatinine is a typical presentation. Urine eosinophils can be checked, although these are not always present. Peripheral eosinophilia is more common. The appropriate manage- ment is to stop the offending agent and start steroids at 1 mg/kg if creatinine does not improve. ATN secondary to the infectious process is possible, but the patient has a rash and has been responding to nafcillin with decreasing WBC counts. No features of ITP or obstruction fi t the clinical scenario (9).

9. The answer is B. The patient’s electrolyte abnormality and alkalosis are consistent with hyperaldosteronism. In the setting of malignancy, the tumor is likely secreting an aldosterone-like hormone. Aldosterone acts on the epithelial sodium channel (ENaC) located in the principal cells of the collecting tubule. The increased sodium reabsorp- tion helps in creating a gradient and increases potassium secretion into the lumen via renal outer-medullary potassium channels, promoting hypokalemia. As a result of increased sodium reabsorption, the increased lumen electronegativity causes type A intercalated cells in the collecting tubules to increase hydrogen ion secretion and therefore increase bicarbonate reabsorption, leading to alkalosis. Acetazolamide and hydrochloric acid infusions will correct the alkalosis, but this is not the major underly- ing cause for the hypokalemia. Amiloride would directly block the ENaC channel cor- recting hypokalemia and alkalosis. Aldactone is a direct antagonist to aldosterone and would also ameliorate its effects (10).

10. The answer is B. Data have shown that volume expansion with NS or NaHCO3 has helped prevent contrast-induced AKI. A recent study indicates that NS is superior in prevention of contrast-induced nephropathy (11). Mucomyst can be started, although there are no data for defi nite protection (12). There are no data supporting prophylactic hemodialysis for preventing acute kidney injury (AKI) in stages 2 to 4 chronic kidney disease.

11. The answer is A. Perform hemodialysis emergently because the patient has likely ingested ethylene glycol and has severe acidosis with end-organ damage. The other therapies have proven effective in less severe toxicity. Activated charcoal would be ineffective because ethylene glycol would have been absorbed immediately after ingestion (13).

12. The answer is E. This patient has an acute decline with polyuria, volume depletion, and hyponatremia. This is likely secondary to cerebral salt wasting. Increased intracranial pressures resulting in increased brain natriuretic peptide (BNP) are believed to cause salt and water diuresis. Urine osmolality is elevated as in SIADH. Correction requires nor- mal saline (NS) infusion, which would replete the volume status and also suppress the high antidiuretic hormone (ADH) state, correcting the hyponatremia. SIADH patients are euvolemic and infusion of NS would further worsen the hyponatremia because there is greater retention of the free water component in saline. Hyponatremia in SIADH is cor- rected by volume restriction (14).

13. The answer is D. The patient has nephrogenic diabetes insipidus secondary to chronic exposure to lithium. Lithium enters the epithelial sodium channel (ENaC) channel in the principal cells of the tubule and interferes with the signaling pathway, inhibiting aquaporin-2 water channels, resulting in dilute urine, and if not treated, hypernatremia. Nephrogenic diabetes insipidus is treated with thiazide diuretics and/or amiloride. These

294

66485457-66485438 www.ketabpezeshki.com

ZZakaria_87574_PTR_CH19_10-06-13_285-298.inddakaria_87574_PTR_CH19_10-06-13_285-298.indd 294294 66/19/2013/19/2013 4:46:364:46:36 PMPM RENAL DISEASES: Answers

medications’ effect is presumably mediated by a hypovolemia-induced increase in proxi- mal sodium and water reabsorption, thereby diminishing water delivery to the antidi- uretic hormone (ADH)-sensitive sites in the collecting tubules and reducing the urine out- put. Persistent polyuria can be treated with nonsteroidal anti-infl ammatory drugs, which inhibit prostaglandin production and increase the kidneys’ concentrating ability (15).

14. The answer is B. There is an increased risk for nephrogenic systemic fi brosis with gado- linium infusion in patients with GFR of 30 mL/min per 1.73m2 and less, especially in the presence of diabetes. There is no possible treatment once it occurs. If the patient has end-stage renal disease or stage 5 chronic kidney disease, and gadolinium is necessary, the study can be completed provided that the patient undergoes three sessions of hemo- dialysis (HD), each greater than 4 hours, to clear the gadolinium. There are no data yet to confi rm this strategy (16).

15. The answer is D. To approach this question, one would fi rst calculate the anion

gap (AG): Na – (Cl + HCO3), which is 18 (normal AG 12mEq/L +/-4). Therefore, there is

an AG metabolic acidosis. Then calculate the change in AG relative to the change in HCO3

(delta ratio). The change in HCO3 is 25 − 22 = 3 and AG change is 18 − 12 = 6, with a delta ratio of 2:1 suggests an underlying metabolic alkalosis. Looking at the arterial blood gas

(ABG), the primary disturbance is respiratory alkalosis. A decrease of 10 in PaCO2 causes

a decrease of 2 in HCO3, which is consistent in acute respiratory alkalosis (17).

16. The answer is A. The patient has an anion gap (AG) of 33 corrected for hypo-albumin- emia (normal AG 12mEq/L+/- 4). For every decrease of 1 g/dL of albumin, there is a decrease in AG by 2.5 (Corrected AG = AG + [2.5 × (4 – albumin)]. This is an AG metabolic acidosis. The change in AG is 2 times greater than the change in bicarbonate (change in AG is 21 and change in bicarbonate is 11), suggesting a metabolic alkalosis. Upon review-

ing the arterial blood gas (ABG), the expected pCO2 should be 28; however, it is 20 [using

Winter’s formula PaCO2 = (1.5 × HCO3) + 8], explaining the respiratory alkalosis compo- nent (17).

17. The answer is D. The patient has received multiple units of PRBCs, which contain cit- rate. The liver metabolizes citrate to produce 23 mEq of bicarbonate for each unit that is transfused. In the setting of acute kidney injury, there is decreased clearance of bicarbon- ate, compounding alkalosis. The hypercapnia that occurs as a compensatory measure can further impair the renal correction of the metabolic alkalosis (17).

18. The answer is B. The patient has contraction alkalosis, which is evident from the low chloride and elevated bicarbonate. This can be due to diarrhea or vomiting in the set- ting of a volume-depleted patient. However, the patient’s urinary chloride is elevated instead of low (<20 is expected in a volume-depleted patient). This is an indicator of likely diuretic abuse. Hyperaldosteronism would be considered in a volume-expanded patient because there is increased sodium reabsorption (17).

19. The answer is D. Sickle cell disease–induced hypoxia in the renal medulla results in impaired secretion of potassium and hydrogen ions in the collecting tubules and the development of a nongap acidosis. This patient has a nongap acidosis with hyperkalemia

295

66485457-66485438 www.ketabpezeshki.com

ZZakaria_87574_PTR_CH19_10-06-13_285-298.inddakaria_87574_PTR_CH19_10-06-13_285-298.indd 295295 66/19/2013/19/2013 4:46:364:46:36 PMPM GENERAL CRITICAL CARE: PATHOLOGY, PATHOPHYSIOLOGY, AND THERAPY

and elevated urine pH. The presence of hyperkalemia is due to reduced distal sodium reabsorption. Treatment includes replacing bicarbonate at 1 to 2 mEq/kg to offset acid buildup (18).

20. The answer is A. The patient’s skin fi ndings are consistent with livedo reticularis second- ary to cholesterol emboli. She has had vascular manipulation with both cardiac catheter- ization and an IABP placement. Her acute kidney injury (AKI) is likely due to choles- terol emboli causing ischemic damage to the renal tubules. An infl ammatory process with interleukin-5 (IL-5) release by activated T-cells has been described in up to 88% of cases, with some reports of increased survival and renal function improvement with corticoster- oids (19).

21. The answer is C. Heparin causes suppression of aldosterone synthesis by reducing the number and affi nity of angiotensin-II receptors in the zona glomerulosa of the adrenal gland. Aldosterone causes increased sodium reabsorption in the principal cells of the col- lecting tubule (ENaC) and also promotes potassium excretion into the tubular lumen. This uncommon side effect is more common in elderly, renally insuffi cient, and diabetic patients who are unable to compensate the aldosterone reduction with renin increase. The other possibilities are unlikely (20).

References

1. Polderman KH, Schreuder WO, Strack van Schijndel RJ, et al. Hypernatremia in the inten- sive care unit: an indicator of quality of care? Crit Care Med. 1999 Jun;27(6):1105–1108. 2. Adrogué HJ, Madias NE. Hyponatremia (Review). N Engl J Med. 2000 May 25;342(21):1581–1589. 3. Galla JH, Gifford JD, Luke RG, et al. Adaptations to chloride-depletion alkalosis. Am J Physiol. 1991;261(4-Pt2):R771. 4. Wingo CS, Smolka AJ. Function and structure of H-K-ATPase in the kidney. Am J Physiol. 1995;269(1-Pt2):F1. 5. Zimmerman JL. Acid-base disorders. ACCP Pulmonary Med Brd Rev. 2009; 25: 309–318. 6. Shapiro ML, Baldea A, Luchette FA et al. Rhabdomyolysis in the intensive care unit. J Intensive Care Med. 2011 Mar 24. 7. Bailey AR, Sathianathan VJ, Chiew AL, et al. Comparison of intermittent haemodialysis, prolonged intermittent renal replacement therapy and continuous renal replacement hae- mofi ltration for lithium toxicity: a case report. Crit Care Resusc. 2011 Jun;13(2):120–122. 8. Frøkiaer J, Marples D, Knepper MA, et al. Bilateral ureteral obstruction downregulates expression of vasopressin-sensitive AQP-2 water channel in rat kidney. Am J Physiol. 1996 Apr;270(4 Pt 2):F657-F668. 9. Perazella MA, Markowitz GS. Drug-induced acute interstitial nephritis. Nat Rev Nephrol. 2010 Aug;6(8):461–470. Epub 2010 Jun 1. 10. Kishimoto S, Hirota K, Segawa H et al. Ectopic ACTH syndrome revealed as severe hypokalemia and persistent hypertension during the perioperative period: a case report. J Anesth. 25(1):104–107. 11. Fishbane S. N-acetylcysteine in the prevention of contrast-induced nephropathy. Clin J Am Soc Nephrol. 2008 Jan;3(1):281–287.

296

66485457-66485438 www.ketabpezeshki.com

ZZakaria_87574_PTR_CH19_10-06-13_285-298.inddakaria_87574_PTR_CH19_10-06-13_285-298.indd 296296 66/19/2013/19/2013 4:46:374:46:37 PMPM RENAL DISEASES: Answers

12. Klima T, Christ A, Marana I, Kalbermatter S et al. Sodium chloride vs. sodium bicar- bonate for the prevention of contrast medium-induced nephropathy: a randomized con- trolled trial. Eur Heart J. 2012 Jan 19. 13. Barceloux DG, Krenzelok EP, Olson K, et al. American Academy of Clinical Toxicology Practice Guidelines on the Treatment of Ethylene Glycol Poisoning, Ad Hoc Committee AU. J Toxicol Clin Toxicol. 1999;37(5):537. 14. Yee AH, Burns JD, Wijdicks EF et al. Cerebral salt wasting: pathophysiology, diagnosis, and treatment. Neurosurg Clin N Am. 2010 Apr;21(2):339–352. 15. Christensen BM, Zuber AM, Loffi ng J et al. alphaENaC-mediated lithium absorption pro- motes nephrogenic diabetes insipidus. J Am Soc Nephrol. 2011 Feb;22(2):253–61. Epub 2010 Nov 4. 16. Hellman RN. Gadolinium-induced nephrogenic systemic fi brosis. Semin Nephrol. 2011 May;31(3):310–316. 17. Ratnam S, Kaehny W, Shapiro J, et al. Pathogenesis and management of metabolic acido- sis and alkalosis. In: Schrier RW, ed. Renal and Electrolyte Disorders. 7th ed. Philadelphia, PA: Lippincott Williams & Wilkins; 2010:86 –119. 18. Reddy P. Clinical approach to renal tubular acidosis in adult patients. Int J Clin Pract. 2011 Mar;65(3):350–60 19. Faria B, Vidinha J, Pêgo C, et al. Atheroembolic renal disease with rapid progression and fatal outcome. Clin Exp Nephrol. 2011 Feb;15(1):159–63. Epub 2010 Nov 11. 20. Bengalorka G, Sarala N, Venkatrathnamma PN, et al. Effect of heparin and low-molecular weight heparin on serum potassium and sodium levels. J Pharmacol Pharmacother. 2011 Oct–Dec; 2(4): 266–269.

297

66485457-66485438 www.ketabpezeshki.com

ZZakaria_87574_PTR_CH19_10-06-13_285-298.inddakaria_87574_PTR_CH19_10-06-13_285-298.indd 297297 66/19/2013/19/2013 4:46:374:46:37 PMPM 66485457-66485438 www.ketabpezeshki.com

ZZakaria_87574_PTR_CH19_10-06-13_285-298.inddakaria_87574_PTR_CH19_10-06-13_285-298.indd 298298 66/19/2013/19/2013 4:46:374:46:37 PMPM 20 Electrolyte and Endocrine Disorders David J. Powner QUESTIONS

1. What effect will this arterial blood gas have on the total serum concentration of calcium?

pH 7.58, pCO2 22 mmHg, pO2 114 mmHg, HCO3 17 mEq/L, and BE + 1 A. Lower total calcium, raise ionized Ca2+ B. Raise both total and ionized calcium levels C. Decrease total calcium D. No effect E. Raise total calcium and lower ionized Ca2+

2. A 32-year-old, 34½-week-gestation, eclamptic patient has been admitted to the neurocriti- cal care unit with seizures. She is receiving a magnesium infusion. In addition to serum magnesium (Mg) concentrations, what other fi nding suggests hypermagnesemia? A. Hypertension B. Hyperactivity C. Hyporefl exia D. Tachypnea E. Ventricular dysrhythmias

ANSWERS TO THIS SECTION CAN BE FOUND ON PAGE 306 299

66485457-66485438 www.ketabpezeshki.com

ZZakaria_87574_PTR_CH20_10-06-13_299-318.inddakaria_87574_PTR_CH20_10-06-13_299-318.indd 299299 66/19/2013/19/2013 8:48:268:48:26 PMPM GENERAL CRITICAL CARE: PATHOLOGY, PATHOPHYSIOLOGY, AND THERAPY

3. An 18-year-old patient is a direct admission following a self-infl icted gunshot wound to the head. Initial treatment at the referring ED included 75 g of mannitol, furosemide 20 mg, 30 mL 23.4% NaCl (hypertonic saline) intravenously, and infusion of D5% in 0.9% saline (D5NS). Laboratory data at admission include alcohol non-detected, Na 153 mEq/L,

K 3.2 mEq/L, Cl 114 mEq/L, HCO3 27 mEq/L, glucose 288 mg/dL, BUN 14 mg/dL, osmo- lality 343 mosm/kg. What is his osmolar gap? A. 0 mosm/kg B. 21 mosm/kg C. 28 mosm/kg D. 8 mosm/kg E. None of the above

4. A 34-year-old patient, with a history of bipolar disorder with signifi cant manic periods, is admitted with progressive myoclonus, polydipsia, confusion, fever, leukocytosis, polyuria, and one seizure. She has taken lithium for some years. Her initial evaluation showed a normal head CT scan and lumbar puncture. Laboratory data show Na as 162 mEq/L, increased blood urea nitrogen (BUN)/creatinine ratio, hypermagnesemia, hypercalcemia, and white cell count of 22,000/µL. Other laboratory data are pending. Initial treatment may include all of the interventions below, except: A. Seizure suppression with benzodiazepines B. 0.9% (normal) saline administration C. IV aqueous vasopressin boluses or infusion D. Hemodialysis E. Polystyrene sulfonate (kayexalate)

5. What effect will the following arterial blood gas have on serum potassium? pH 7.20, pCO2

60 mmHg, pO2 85 mmHg, HCO3 23 mEq/L, and BE −4 A. Decrease in ionized K, increase in bound K B. Increase in serum K C. Movement of serum K into intracellular space D. No change in intracellular K E. Decrease in total serum K

6. The osmotic demyelination, central pontine, and extrapontine myelinolysis syndromes represent a continuum of potentially severe neurological outcomes associated with which condition(s)? A. Chronic alcohol abuse B. Hyponatremia or hypernatremia C. Liver transplantation treated with tacrolimus D. A and B E. A and C F. B and C G. All of the above H. None of the above

300

66485457-66485438 www.ketabpezeshki.com

ZZakaria_87574_PTR_CH20_10-06-13_299-318.inddakaria_87574_PTR_CH20_10-06-13_299-318.indd 300300 66/19/2013/19/2013 8:48:268:48:26 PMPM ELECTROLYTE AND ENDOCRINE DISORDERS: Questions

7. A 32-year-old patient with an aneurysmal subarachnoid hemorrhage (SAH) has been returned urgently from the operating room after developing malignant hyperthermia after induction and initiation of isofl urane inhalational anesthesia. Which consequence(s) might you anticipate during her stabilization? A. Increased carbon dioxide production B. Metabolic alkalosis C. Extensor muscle fl accidity D. Hypokalemia E. All the above

8. In addition to the major consequences of hypothermia, coagulopathy, and acidosis after rapid and massive transfusion (>20 units packed RBCs) following poly-trauma in adults, specifi c electrolyte changes that should be anticipated may include: A. Hypokalemia B. Hyperionized calcemia C. Hypermagnesemia D. All of the above E. None of the above

9. A 66-year-old patient with anuric chronic renal failure has been directly admitted to the neuro-intensive care unit from an outside “urgent center.” He suffered a spontaneous intracerebral hemorrhage 20 minutes after initiating his regular hemodialysis treatment at a freestanding dialysis center. He also sustained an asystolic cardiac arrest at the dialy- sis center that required 25 minutes of resuscitative effort before return of spontaneous cir- culation. Signifi cant metabolic acidosis was documented. Treatment at the urgent center included single doses of 50% dextrose, 15 units of insulin, and one ampule of bicarbonate. You have initiated an emergent renal consultation and hemodialysis for his severe meta- bolic acidosis, uremia, and serum potassium of 6.9 mEq/L. Additional therapy should include: A. Sodium polystyrene (kayexalate) enterally or per rectum B. Fludrocortisone (Florinef®), a mineralocorticoid C. IV magnesium D. Additional insulin plus nebulized beta-adrenergic drug E. None of the above

10. A 32-year-old patient has been admitted with Sheehan’s syndrome. Her brain CT scan shows hemorrhagic extension beyond the sella. She has had one seizure prior to admis- sion and currently has a Glasgow Coma Scale (GCS) score of E2-V3-M5. Her mean arte- rial pressure is 56 mmHg, urine output is 500 to 600 mL/hour, serum Na is 153 mEq/L, hemoglobin is 12 g/dL, and free T4 is low. Urgent therapy should include: A. Packed red blood cells (RBCs) for her hypotension B. IV vasopressin C. Hydrocortisone D. Gonadotropin replacement to support lactation E. Levothyroxine (T4) to treat hypothyroidism

301

66485457-66485438 www.ketabpezeshki.com

ZZakaria_87574_PTR_CH20_10-06-13_299-318.inddakaria_87574_PTR_CH20_10-06-13_299-318.indd 301301 66/19/2013/19/2013 8:48:268:48:26 PMPM GENERAL CRITICAL CARE: PATHOLOGY, PATHOPHYSIOLOGY, AND THERAPY

11. Within the nonthyroidal illness syndrome (formerly, euthyroid sick syndrome or low-T3 syndrome) some commonly used medications in the ICU, including glucocorticoids, dop- amine, and dobutamine, alter interpretation of thyroid hormone serum concentrations. What change do these drugs cause? A. Decreased reverse-T3 (rT3) B. Increased thyrotropin-releasing hormone (TRH) C. Improved metabolism of rT3 D. Reduced production of thyroid-stimulating hormone (TSH) E. Increased tetraiodothyronine (T4)

12. In 2008, the American College of Critical Care Medicine issued recommendations for the diagnosis of corticosteroid insuffi ciency among critically ill patients. The diagnostic cri- teria include: A. Increase in serum cortisol to above 32 mcg/dL after low-dose adrenocorticotropin hormone (ACTH) stimulation B. Resting hypoglycemia (serum glucose < 60 mg/dL) C. Hyponatremia with serum sodium less than 130 mEq/L D. All of the above E. None of the above

13. Suggestive data implicate IV etomidate with which endocrine consequence? A. Hypoglycemia secondary to insulin release B. Hyperglycemia due to insulin suppression C. Hypercalcemia (transient) D. Increased glucagon release E. Adrenal insuffi ciency

14. Cushing’s syndrome (hypercortisolism) is most often caused by a tumor of the anterior pituitary and may rapidly progress to several serious consequences, including: A. Infection by opportunistic organisms B. Hypokalemia C. Hypertension D. All of the above E. None of the above

15. Glycemic control for patients in the ICU has evolved over the past several years with particular concern about the incidence of neuroglycopenia after subarachnoid hemor- rhage (SAH), stroke, and traumatic brain injury (TBI). A consensus statement in 2009 from major endocrine associations suggests which blood glucose level be maintained for criti- cally ill patients? A. 80 to 110 mg/dL B. 110 to 140 mg/dL C. 180 to 200 mg/dL D. Less than 40 mg/dL E. None of the above

302

66485457-66485438 www.ketabpezeshki.com

ZZakaria_87574_PTR_CH20_10-06-13_299-318.inddakaria_87574_PTR_CH20_10-06-13_299-318.indd 302302 66/19/2013/19/2013 8:48:268:48:26 PMPM ELECTROLYTE AND ENDOCRINE DISORDERS: Questions

16. Several hormone defi ciencies are well documented after traumatic brain injury (TBI) and may persist into the rehabilitative phase of recovery. All of the following hormones have been implicated, except: A. Gonadotropins B. Vasopressin C. Insulin D. Cortisol E. Growth hormone

17. Polyuria following brain death should be treated with: A. Aqueous vasopressin by IV infusion titrated to urine output B. Normal saline administration to replace urine output C. Intranasal vasopressin but at an increased frequency of each 8 hours D. Hypertonic saline to reduce edema of the posterior pituitary E. None of the above

18. A 72-year-old patient has recovered from evacuation of a subdural hematoma 36 hours ago and was extubated without complications. However, he has new onset of diarrhea, vomiting, confusion, temperature elevation, and atrial fi brillation with a ventricular rate of 135 bpm. His head CT scan shows only minimal post-op changes, and stat laboratory testing shows normal electrolytes, complete blood count, and arterial blood gases. Other laboratory tests and cultures are pending. Initial urgent treatment should include: A. Amiodarone B. Propranolol C. Levothyroxine D. Saturated solution of potassium iodide E. Reserpine

19. A 36-year-old patient is admitted after polytrauma with a Glasgow Coma Scale (GCS) score of E1-T-M5. Friends in the emergency department know that she has lupus ery- thematosus, but can report no other medical information. Her fractured femur has been repaired and she is admitted to the neurocritical care unit for continued observation of concussion (head CT is negative). Her clinical course 48 hours after admission has evolved with hypotension despite generous normal saline infusions and vasopressor therapy, moderate temperature, appropriate tachycardia, and no evidence of blood loss into or from her leg. Other data include normal hemoglobin, moderate eosinophilia, no elec- trocardiographic changes or elevated cardiac enzymes, and mild hyponatremia. Urgent treatment should include: A. 100 mcg (0.1 mg) levothyroxine (T4), intravenously B. 25 mcg (0.025 mg) liothyronine (T3) orally C. Dexamethasone 4 to 6 mg intravenously D. 3% saline infusion calculated to replace ½ Na defi cit E. Broad spectrum Gram-negative antibiotic

303

66485457-66485438 www.ketabpezeshki.com

ZZakaria_87574_PTR_CH20_10-06-13_299-318.inddakaria_87574_PTR_CH20_10-06-13_299-318.indd 303303 66/19/2013/19/2013 8:48:268:48:26 PMPM GENERAL CRITICAL CARE: PATHOLOGY, PATHOPHYSIOLOGY, AND THERAPY

20. A patient with a grade II subarachnoid hemorrhage (SAH) is now day 4 post-hemor- rhage. She is more disoriented this morning and her serum sodium (Na) is 124 mEq/L (mmol/L). She has moderate orthostatic hypotension and tachycardia at 112 bpm. Her urine Na is elevated at 130 mmol (mEq)/L. Transcranial doppler assessment is consid- ered normal and unchanged from her initial measurements at admission. Initial therapy might include any of the following except: A. Mineralocorticoid administration B. Fluid restriction to reduce free water intake C. Vasopressin receptor antagonist (conivaptan) D. Hypertonic saline E. 0.9% saline infusion

21. A 78-year-old patient has arrived after a fall at his nursing home. Information at admis- sion indicates the patient has been confused today with odiferous urine (urinalysis pend- ing). Admission CT head shows a “rim” subdural that does not require evacuation at this time. After 1.5 L of 0.9% saline in the ED, ICU admission labs show glucose of 880 mgm/

dL, Na 136 mEq/L, K 3.6 meq/dL, Cl 101 mEq/L, HCO3 21 mEq/L, ketones positive at 1:4 dilution, BUN 18 mg/dL, and serum osmolarity 332 mosm/kg. IV insulin therapy is initiated in 0.9% saline and hydration is continued using: A. 0.45% saline (1/2 normal saline) B. Hypertonic (23.4%) saline boluses to correct serum Na C. 2.0% saline to normalize serum Na

D. 0.45% saline with 50 mEq NaHCO3 to correct low serum HCO3 E. 0.45% saline with 5% dextrose to avoid “overshoot” hypoglycemia

22. A 60-year-old patient with metastatic lung cancer is admitted with a new lesion in his brain and is awaiting urgent surgery. His serum calcium, however, is 12.6 mg/dL (nor- mal 8.2–10.2 mg/dL), and his surgery has been delayed pending its correction. Primary treatment is: A. Calcitonin B. Corticosteroid C. Plicamycin (mithramycin) D. Phosphate to inhibit osteoclasts E. None of the above

23. An elderly female patient has been admitted to the neurology service from a long-term care facility with the diagnosis of dementia and new onset seizures. Physical examina- tion reveals moderate orthostatic hypotension, pitting edema, sinus bradycardia, obesity, hypothermia (93.6°F or 36.4°C), and depressed affect. Laboratory data confi rm moder- ate hyponatremia (125 mEq/L), low–normal glucose, low T4, and markedly elevated thyroid-stimulating hormone (TSH). In addition to thyroid hormone, initial therapy should include: A. Rapid active rewarming to improve circulation B. Low-dose benzodiazepine to assure patient comfort C. A positive chronotropic agent to increase heart rate D. Diuretic therapy to reduce pitting edema E. None of the above

304

66485457-66485438 www.ketabpezeshki.com

ZZakaria_87574_PTR_CH20_10-06-13_299-318.inddakaria_87574_PTR_CH20_10-06-13_299-318.indd 304304 66/19/2013/19/2013 8:48:268:48:26 PMPM ELECTROLYTE AND ENDOCRINE DISORDERS: Questions

24. The hyperreninemic hypoaldosteronism syndrome of critical illness has been docu- mented in several clinical groups, including aneurysmal subarachnoid hemorrhage (SAH) patients. The syndrome is characterized by a plasma aldosterone-to-plasma renin activity ratio of less than 2. The clinical manifestation of this syndrome among SAH patients is: A. Hypotension B. Hyponatremia C. Increased incidence of vasospasm D. Hypertension causing increased secondary aneurysmal rupture E. None of the above

305

66485457-66485438 www.ketabpezeshki.com

ZZakaria_87574_PTR_CH20_10-06-13_299-318.inddakaria_87574_PTR_CH20_10-06-13_299-318.indd 305305 66/19/2013/19/2013 8:48:268:48:26 PMPM 20

ANSWERS

1. The answer is D. Serum calcium is in two forms, bound and unbound (ionized). Ionized calcium (Ca2+) is the physiologically active form. Ca2+ is normally measured directly but can also be estimated by the albumin-adjusted calcium: Adjusted Ca = total serum Ca (mg/dL) – 0.8 [4 − serum albumin (g/dL)] Binding of calcium is mostly with albumin (a small percentage with lactate, sulfate, and citrate) and is affected by the blood pH. Increased binding to albumin (hence decrease in Ca2+) occurs in alkalemia, the reverse in acidemia. The total calcium during pH changes, however, remains the same. Total serum calcium is altered directly by changes in serum albumin concentration, but ionized is not (1). a. Ionized hypocalcemia: Signs and symptoms of signifi cantly lowered Ca2+ are related to neuronal hyperexcitability (e.g., Chvostek and Trousseau signs), seizures, tetany, and so on. The most common cause is following parathyroid surgery but causes may also include pancreatitis and large volume blood transfusions during which citrate bind- ing with calcium occurs. Although present in a large number of hospitalized patients, treatment of hypoionized calcemia and protocols to do so are controversial and may even be harmful (as seen in animals), suggesting that this laboratory abnormality is physiologically advantageous. b. Ionized hypercalcemia: As expected, signs and symptoms are of weakness and fatigue, usually caused by hyperparathyroidism, cancer [production of parathyroid-related protein (PTHrP)], or renal failure (2).

2. The answer is C. Magnesium is the divalent ion in highest intracellular concentration. Extracellular Mg is equally distributed through intravascular and interstitial compart- ments, with about 30% of the intravascular Mg bound to albumin and 70% ionized, as its physiologically active form. Blood levels of Mg are, therefore, lower in hypoalbuminemia.

306

66485457-66485438 www.ketabpezeshki.com

ZZakaria_87574_PTR_CH20_10-06-13_299-318.inddakaria_87574_PTR_CH20_10-06-13_299-318.indd 306306 66/19/2013/19/2013 8:48:278:48:27 PMPM ELECTROLYTE AND ENDOCRINE DISORDERS: Answers

Ionized Mg (Mg2+) can be measured, but is rarely available. Hypermagnesemia (serum concentration > 6.0 mEq/L) in the ICU is most commonly due to renal failure. It pro- duces neuronal depression manifested centrally by confusion to coma and peripherally by hyporefl exia (especially Achille’s tendon refl ex) to paralysis. In this case of eclampsia symptoms, hypermagnesemia would be treated by reducing the infusion and/or stop- ping it for a short time. Active treatment includes IV administration of calcium gluconate as a neuronal antagonist of Mg plus fl uid (and perhaps diuretic) infusion to increase renal excretion. In other conditions, hemodialysis is advocated for symptomatic hypermag- nesemia (1,3).

3. The answer is E. The osmolar (osmolal) gap is the difference between the measured osmolality and its calculated value. Several equations have been used to calculate serum osmolality (4), but the most common is: Serum osmolality (mosm/kg) = 2 Na + (glucose ÷ 18) + (BUN ÷ 2.8) + (ethanol ÷ 4.6) Calculated osmolality is 327 mosm/kg, measured is 343 mos/kg, and the gap is 16. Normal subjects demonstrate a gap of −8 to 11 mosm/kg (4). The osmolar gap has traditionally been used to diagnose toxic alcohol (ethylene glycol, methanol, and isopropyl alcohol) ingestion, radiographic contrast exposure, or acetone intake. Other osmotically active substances, such as mannitol in this question, will also produce an increased osmolar gap. Periodic measurement of serum osmolality and serum electrolytes when mannitol is given allows assessment of potential hyperosmolar renal injury and/or related changes in sodium, potassium, and other electrolytes (5). Mannitol-associated renal injury is more likely during concurrent subacute renal failure and dehydration and is due to afferent renal arteriolar vasoconstriction affecting renal blood fl ow and osmotic renal glomerular and tubular injury. Its occurrence appears increased when mannitol dosing is greater than 200 g/24 hours, serum mannitol concentration is greater than 1,000 mg/dL, or the peak osmolar gap is greater than 55 mosm/kg. Serum mannitol testing is rarely available but can be estimated by: Estimated serum mannitol = osmolar gap × (182 ÷ 10) Mannitol is considered a renal artery vasodilator at lower doses but a vasoconstrictor above 1,000 mg/dL, based on animal data. Prolonged mannitol use may also present as a hypertonic hyponatremia, as discussed by Tsai and Shu (6), and produces pseudo- hyponatremia due to a serum dilutional effect, as in hyperglycemia (for each 100 mg/dL of serum mannitol concentration, the Na will be lowered by 1.6 mEq/L). Hemodialysis is very effective in reversing this form of renal failure if more conservative measures to reverse hypovolemia are not effective (4–8).

4. The answer is C. Chronic or acute lithium toxicity may cause a range of symptoms, depending on its severity. Lithium causes nephrogenic diabetes insipidus via interac- tion with aquaporin 2 (AQP2) via mechanisms still under investigation, leading to interaction with vasopressin and impairing the concentrating capability of the renal collecting ducts. Neurological signs and symptoms are prominent at all levels of toxicity. In addition, cardiovascular, renal, and gastrointestinal symptoms produce signifi cant morbidity, including hypotension due to dehydration, ECG changes, dys- rhythmias, emesis, impaired renal concentrating capability, nephrogenic diabetes

307

66485457-66485438 www.ketabpezeshki.com

ZZakaria_87574_PTR_CH20_10-06-13_299-318.inddakaria_87574_PTR_CH20_10-06-13_299-318.indd 307307 66/19/2013/19/2013 8:48:278:48:27 PMPM GENERAL CRITICAL CARE: PATHOLOGY, PATHOPHYSIOLOGY, AND THERAPY

insipidus, multiple electrolyte abnormalities, and chronic renal failure. Patients may be dehydrated, and volume expansion is appropriate with normal saline to increase uri- nary lithium excretion even though hypernatremia may be present. Kayexalate binds the drug and increases elimination, and hemodialysis is effective in severe intoxica- tion. Administration of vasopressin, however, is ineffective as the kidney is not able to respond (9–11).

5. The answer is B. Potassium moves from the intracellular space into the serum to compen- sate for increased intracellular hydrogen ion derived from diffusible acid into cells. This translocation occurs when acidemia is due to mineral acids (e.g., respiratory, uremic, and ammonium chloride), but not in response to organic acids (e.g., diabetic, lactic, salicylate, and ethylene glycol). The amount of change is an approximately 0.6 mEq/L increase in K for each 0.1 pH unit decrease. The converse movement into the cell occurs in the same magnitude in alkalemia. Depending on many other variables, this change may be clini- cally important (12,13).

6. The answer is G. Recent evaluations of these related syndromes have identifi ed more diverse clinical and MRI evidence of demyelination and loss of oligodendrocytes than the traditional central syndrome of pontine myelinolysis. Pontine demyelination presents with brainstem signs (e.g., dysphagia, dysarthria, quadriparesis, and locked-in syndrome) traditionally associated with overly rapid correction of hyponatremia among alcoholic patients. Signifi cant among the more recent, diverse, and less severe manifesta- tions are seizures, neuropsychiatric changes, basal ganglion and white matter involve- ment, dysautonomia, cognitive impairment, and tremor. Predisposing factors include malnutrition, alcoholism, electrolyte abnormalities (hyponatremia and hypokalemia), other chronic diseases, and liver transplantation. The differential diagnosis includes central nervous system (CNS) infection, Wernicke’s encephalopathy, toxicity from other immunosuppressives, and posterior reversible encephalopathy syndrome. Although the exact pathophysiological mechanisms of demyelination remain unclear, the syndrome often has a biphasic course. Initial signs coinciding with hyponatremia, when present, often improve with correction of acute electrolyte abnormalities, but are followed by pro- gressive neurological changes after approximately 1 week. It is recommended that cor- rection of chronic hyponatremia be targeted at 8 to 12 mEq/L per 24 hours. Otherwise, treatment is supportive (14–16).

7. The answer is A. The incidence of anesthesia-related malignant hyperthermia is low (~0.5–2 cases/100,000 anesthesia procedures) and may occur in patients with prior unaf- fected operations. Susceptible patients respond to a variety of triggers, including volatile anesthetics and succinylcholine, that release calcium from skeletal muscle, especially the masseter muscles, due to an abnormal ryanodine receptor. Concurrent morbidity may

include increased oxygen consumption/CO2 production, increased lactate production/ possible acidosis, rhabdomyolysis causing renal failure, hypotension, and hypermetabo- lism with fever and increased skeletal muscle contraction. The overall incidence of some complicating morbidity was 35% and was proportional to the rise in the patient’s tem- perature and to any delay in initiating dantrolene dosing. Incremental dantrolene therapy in an amount that produces the desired effect is the only treatment and stops the release

308

66485457-66485438 www.ketabpezeshki.com

ZZakaria_87574_PTR_CH20_10-06-13_299-318.inddakaria_87574_PTR_CH20_10-06-13_299-318.indd 308308 66/19/2013/19/2013 8:48:278:48:27 PMPM ELECTROLYTE AND ENDOCRINE DISORDERS: Answers

of calcium from muscle. Additional supportive treatment of other consequences may be required (17,18).

8. The answer is E. The reverse of each answer option is true. Each occurs due to one or more factors: a. Age of the stored packed red cells b. Lysis of RBCs in the stored blood c. Hypothermia Hypothermia potentially results from exposure, rapid infusion of room temperature or cold IV fl uids/blood products, or impaired shivering. Coagulopathy potentially results from consumption or dilution of coagulation factors and platelets during resuscitation with fl uids devoid of factors and platelets, hyperfi brinolysis, reduced function of coagu- lation factors V and VIII in stored blood, preexisting anticoagulant or antiplatelet drugs, trauma-induced protein C activation, and impaired clotting factor function or production during acidosis. Hyperkalemia potentially results from high K concentration (7 mEq/L or above) in stored blood and transcellular relocation during acidemia. Hyperkalemia dur- ing massive transfusion may produce cardiac arrest. Hypoionized calcemia and hypo- magnesemia potentially result from infusion of Ca and Mg-poor fl uids, binding by citrate in packed RBCs, and decreased liver metabolism of citrate during hypothermia or due to pre-existing liver disease. Hypoionized calcemia may produce a prolonged QT inter- val, dysrhythmias, decreased left ventricular contractility, decreased systemic vascular resistance, and increased mortality. Hypomagnesemia may also prolong the QT interval and cause dysrhythmias. Acidosis potentially results from hypoperfusion from primary injuries, pH (<7.0) of stored packed RBCs, and hyperchloremia from excessive infusion of normal saline. Bacterial infection, transfusion-related acute lung injury, systemic infl am- matory response syndrome, multiple organ failure, and mortality are also increased after massive transfusion (19–22).

9. The answer is D. The kidney and associated renin/angiotensin/aldosterone hormones are highly effi cient in assuring potassium homeostasis. This patient example, however, introduces both renal failure and translocation of potassium during acidemia/acidosis as causes of his signifi cant hyperkalemia. Other less common causes of hyperkalemia include tissue injury, massive transfusion, rhabdomyolysis, pseudohypoaldosteronism, and hypoaldosteronism. In addition, many medications may cause an increase in serum K, including amiloride, spironolactone, cyclosporine, trimethoprim, nonsteroidal anti- infl ammatory drugs (NSAIDs), ACE inhibitors, angiotensin-II receptor antagonists, beta-blockers, Ca-channel blockers, digoxin, succinylcholine, mannitol, and heparin. Manifestations of acute severe hyperkalemia include characteristically peaked T-waves on EKG; see the fi gure at the top of the next page. A variety of dysrhythmias, such as sinus arrest, intraventricular conduction delay, and ventricular tachycardia/fi brillation can occur at very high serum potassium concentrations. Emergency treatment includes stopping any source of potassium and translocation of serum K into the intracellular compartment with IV dextrose, insulin, and bicarbonate, although bicarbonate use is controversial. Beta-adrenergic agonist therapy is highly effective in stimulating the Na/ K-ATPase transcellular exchange pump and lowers extracellular K. Salbutamol, albuterol,

309

66485457-66485438 www.ketabpezeshki.com

ZZakaria_87574_PTR_CH20_10-06-13_299-318.inddakaria_87574_PTR_CH20_10-06-13_299-318.indd 309309 66/19/2013/19/2013 8:48:278:48:27 PMPM GENERAL CRITICAL CARE: PATHOLOGY, PATHOPHYSIOLOGY, AND THERAPY

Source: Courtesy of Francisco Fuentes, MD, Professor, Department of Cardiology, University of Texas, Houston.

levalbuterol, and terbutaline via nebulizer inhalation are effective and may be more so when administered concurrently with insulin. Their effect is more rapid than kayexalate and “loop” diuretics (e.g., furosemide, not applicable in this case). Fludrocortisone is not effective. In the example here, renal replacement therapy is, of course, critical. IV calcium chloride may be given to counter or prevent dysrhythmias, but does not effect serum concentrations of K (23–25).

10. The answer is C. First described in 1913 and later expanded by Sheehan in 1937, this rare occurrence of postpartum pituitary hemorrhage is thought to be related to the normal pre- partum enlargement of the anterior portion of the pituitary during pregnancy. Increased intrasellar pressure, particularly if the woman’s sella is anatomically small, perhaps leads to ischemia within the gland and secondary vessel rupture. Ischemia in the pituitary also often follows a signifi cant hemorrhage during childbirth, possibly leading to transient hypoperfusion that may contribute to the ischemic injury. Because of the separate blood supply to anterior and posterior portions of the pituitary, it is unusual, but well docu- mented, to demonstrate diabetes insipidus in this syndrome. Hemorrhage in the anterior portion of the gland is more common because its blood supply is through a portal ves- sel from the hypothalamus that fl ows with lower pressure, making it more susceptible to ischemic injury. The primary manifestation of the hemorrhage/ischemia in this form of pituitary apoplexy is loss of anterior pituitary hormone production. These hormones include gonadotropins, growth hormone, thyroid-stimulating hormone (TSH), and adre- nocorticotropin hormone (ACTH). The most dangerous immediate morbidity is related to an abrupt adrenal failure, usually producing hyponatremia (loss of mineralocorticoid),

310

66485457-66485438 www.ketabpezeshki.com

ZZakaria_87574_PTR_CH20_10-06-13_299-318.inddakaria_87574_PTR_CH20_10-06-13_299-318.indd 310310 66/19/2013/19/2013 8:48:278:48:27 PMPM ELECTROLYTE AND ENDOCRINE DISORDERS: Answers

hypotension, and associated relative hypovolemia. Hemodynamic consequences of acute adrenal insuffi ciency are typically high cardiac output and low systemic vascular resistance that require immediate volume replacement and corticosteroid administration at stress dosing. Of note, thyroid hormone should not be given until after corticosteroid admin- istration, as thyroid therapy will accelerate the metabolism of endogenous corticosteroid and potentially lower blood pressure further. Growth hormone and gonadotropins are not usually replaced during initial treatment (26).

11. The answer is C. These medications and the presence of cytokines reduce the response to hypothalamic TRH and decrease production of TSH. In addition, they, along with amio- darone and propranolol, reduce peripheral conversion of thyroxine (T4) to triiodothyro- nine (T3). Previously considered to be a benign observation in critically ill patients, the hormonal changes in the nonthyroidal illness syndrome do increase mortality and con- tribute to a worse outcome, particularly in patients with head trauma, prior thyroid dis- ease, or concomitant autoimmune pathology. Differentiation of the syndrome from true hypothyroidism depends on the sensitivity of the laboratory test available. True hypothy- roidism is indicated by a moderately reduced TSH and low T4. Treatment of the nonthy- roidal illness syndrome with either T4 or T3 remains controversial. Therapy is indicated if hypothyroidism is documented (27–29).

12. The answer is E. The diagnosis of hypoadrenalism, particularly in the ICU, remains under active discussion. The American College of Critical Care Medicine created the term criti- cal illness-related corticosteroid insuffi ciency to identify dysfunction of the hypothalamic- pituitary-adrenal (HPA) axis during critical illness, particularly during sepsis and acute respiratory distress. Clinical signs that suggest adrenal insuffi ciency might be present are extensive and include hypotension resistant to fl uid resuscitation and vasopressors; hyperdynamic circulation (high cardiac output, low systemic vascular resistance); emesis; diarrhea; weakness; eosinophilia; metabolic acidosis; hyponatremia with hyperkalemia; hypoglycemia; decreased mental status; fever; and/or conditions likely associated with increased cytokine release. Testing for adrenal insuffi ciency remains controversial. Because diurnal variation in cortisol blood levels is lost during critical illness, a random cortisol is used. A blood level of less than 10 mcg/dL or an increase in serum cortisol less than 9 mcg/ dL after administration of a 250 mcg cosyntropin stimulation test is recommended by the College as a diagnostic criterion. Variables that affect both parameters include: a. The high degree of binding of cortisol to serum albumin and cortisol-binding globulin and the reduction in total cortisol when those transport proteins are low b. Limited availability of testing of free cortisol as the hormone’s biologically active form c. Continued discussion of high-dose (250 mcg) versus low-dose (1 mcg) stimulation testing d. The role of cytokines in the defi ciency syndrome (30,31).

13. The answer is E. Etomidate has several advantages as a sedative: It has few adverse cardiovascular effects, does not increase intracranial pressure, maintains cerebral perfu- sion pressure, has rapid onset (5–15 seconds), and has relatively short duration of action (5–14 minutes). However, it blocks conversion of 11-deoxycortisol into cortisol. Studies have shown reduced cortisol blood concentration and poorer response to cosyntropin

311

66485457-66485438 www.ketabpezeshki.com

ZZakaria_87574_PTR_CH20_10-06-13_299-318.inddakaria_87574_PTR_CH20_10-06-13_299-318.indd 311311 66/19/2013/19/2013 8:48:288:48:28 PMPM GENERAL CRITICAL CARE: PATHOLOGY, PATHOPHYSIOLOGY, AND THERAPY

stimulation for up to 24 hours (or possibly longer) after a single dose. Patients with sep- sis appear to be particularly affected. Some data show associations with increased vaso- pressor requirements, increased mortality, and longer hospital, ICU, and ventilator days. Treatment with corticosteroid for some poorly defi ned period of time after etomidate dos- ing is controversial (32).

14. The answer is D. Considerable attention is paid to cortisol defi ciency in patients in the ICU (see Answer 12), but cortisol excess may cause the morbidity listed in the question. Although 70% of hypercortisolism is from a pituitary tumor (as Cushing’s disease), 15% is from an adrenal tumor and 15% from another tumor source (e.g., carcinoid in the small intestine, carcinomas of lung, pancreas, or thymus). Rapidly evolving Cushing’s syndrome may lack traditional physical fi ndings of obesity, weakness, and so on, but may manifest metabolic changes, neurological signs, and severe hypertension. Medical suppression of cortisol production while diagnosing the tumor source can be attempted using ketoconazole, metyrapone, or etomidate (see Answer 13). In extreme cases, adrena- lectomy may be required to curb symptoms if the source of hormone production cannot be identifi ed (33,34).

15. The answer is E. The evolution of evidence-based data regarding glucose control in the ICU has liberalized the earlier goal for serum glucose. The importance of hypoglycemic episodes has been emphasized, and benefi t from the prior goal of 80 to 110 mg/dL has not been sustained by comparative studies. A negative mortality effect exists with persistent hyperglycemia so that current recommendations are 140 to 180 mg/dL by some authors and less than 150 mg/dL by Surviving Sepsis Campaign authors (35–37).

16. The answer is C. Signifi cant disruption of the hypothalamic-pituitary-adrenal (HPA) axis after TBI is well documented with historical documentation of anatomic injury within a series of autopsy examinations (38). Consequently, all pituitary-based hormones are decreased. Multiple studies have shown varying speeds of recovery for hormone produc- tion, some of which (especially gonadotropin and growth hormones) may persist into rehabilitation. The incidence of acute hypopituitarism after TBI depends on testing meth- ods and the hormones evaluated, and ranges from 25% to 50%. In addition, manifesta- tions may be acute, as with hypoadrenalism and secondary hypothyroidism, or remain undiagnosed for years as with growth hormone defi ciency and hypogonadism (39,40).

17. The answer is E. As with any symptom, the differential diagnosis for polyuria should be considered, and its cause should not be automatically assumed to be diabetes insipidus. Other causes may be physiological diuresis to mobilize fl uid previously infused, residual effect from diuretics given, or osmotic diuresis due to hyperglycemia or mannitol. These other causes may cause slight hypernatremia, whereas diabetes insipidus will result in much higher serum sodium. Therefore, vasopressin therapy is best reserved until hyper- natremia becomes the focus of therapy. Aqueous vasopressin as an IV infusion or repeated small boluses are preferred. In the presence of hypernatremia, hypotonic saline (0.25% or 0.45%) is indicated and is often titrated each hour to equal urine output until the serum Na improves. Fluid administration as dextrose in water may complicate management of hyperglycemia, and its potential to produce an osmotic diuresis that may produce more free water loss than naturesis can potentially worsen hypernatremia (41).

312

66485457-66485438 www.ketabpezeshki.com

ZZakaria_87574_PTR_CH20_10-06-13_299-318.inddakaria_87574_PTR_CH20_10-06-13_299-318.indd 312312 66/19/2013/19/2013 8:48:288:48:28 PMPM ELECTROLYTE AND ENDOCRINE DISORDERS: Answers

18. The answer is B. This scenario challenges you to maintain an index of suspicion for the unusual diagnosis, in this case, thyrotoxicosis. More common among older patients under stress that increases catecholamine release, such as surgery, thyrotoxicosis may present with multi-organ symptoms, including new onset atrial fi brillation, fever, and gastrointestinal changes of diarrhea and emesis. Urgent control of the patient’s heart rate is important, and although amiodarone is popular for that purpose, it is relatively contraindicated here. The iodine component of the amiodarone molecule might enhance thyroid hormone synthesis and make toxicity worse. Potassium iodide is similarly contraindicated until hormone synthesis is suppressed. Reserpine is of historical interest, but the possibility of hypotension and worse diarrhea limits is use. Propranolol has theo- retical advantage above other beta-blockers because it also decreases T4 conversion to T3 and provides heart rate control. Cortisol is also recommended to offset the possibility of concomitant hypoadrenalism and because it, too, reduces peripheral T4 to T3 (27,42).

19. The answer is C. This patient has also been stressed with her trauma and subsequent sur- gery. Although not stated, it might be assumed that she was taking corticosteroid for her lupus. Without continued medication plus the infl uence of her stress factors, acute adrenal insuffi ciency is likely. Suggestive signs and symptoms include those listed in the question, but most importantly hypotension unresponsive to fl uid replacement and vasopressors, hyponatremia, and hypoglycemia. Therapy should be titrated to the severity of the clinical presentation. When diagnostic confi rmation is sought, dexamethasone may be given to allow physiological stabilization while testing proceeds with the recommended high dose of cosyntropin (see Answer 12). Hydrocortisone can then be given while the results of test- ing are awaited. Mineralocorticoid (fl udrocortisone) may be used when hyponatremia is signifi cant, but is not usually given as initial therapy. If immediate cortisol administration is selected, 100 mg hydrocortisone intravenously each 8 hours is given for at least 24 hours and then the dose is often reduced to 50 mg at the same frequency (31,43).

20. The answer is B. The association of hyponatremia with several neurological conditions is, of course, well recognized and may represent the secretion of inappropriate antidiuretic hormone (SIADH) syndrome and/or cerebral salt-wasting syndrome (CSWS). However, other potential causes of hyponatremia must be considered (e.g., hypoadrenalism, diuretic excess, heart failure, and hypothyroidism). Complex theories about the cause of CSWS include dysfunction of the renin angiotensin or sympathetic nervous systems in the kidney; intrinsic central nervous system (CNS) production of a substance that might infl uence rennin–angiotensin function; and increased production of C-type natriuretic peptide or adrenomedullin in the brain. The physiological distinction emphasizes that the resulting CSWS produces hypovolemic hyponatremia, whereas the SIADH syndrome is associated with euvolemic hyponatremia. Measuring intravascular volume status in the ICU to separate the two syndromes is diffi cult, while other laboratory measures of serum/urine osmolality or sodium concentration show considerable overlap between the two syndromes. Hyponatremia may cause neurological symptoms indistinguish- able from those of the primary process, such that correcting both a volume defi cit and the intravascular sodium concentration are necessary. Likewise, because hypovolemia is associated with an increased incidence of vasospasm, fl uid restriction, even if SIADH is considered the likely diagnosis, is not clinically implemented. Therefore, the other

313

66485457-66485438 www.ketabpezeshki.com

ZZakaria_87574_PTR_CH20_10-06-13_299-318.inddakaria_87574_PTR_CH20_10-06-13_299-318.indd 313313 66/19/2013/19/2013 8:48:288:48:28 PMPM GENERAL CRITICAL CARE: PATHOLOGY, PATHOPHYSIOLOGY, AND THERAPY

treatments listed in the question may be used as needed. The vasopressin receptor antag- onist, conivaptan, is approved by the U.S. Food and Drug Administration for euvolemic (e.g., SIADH) and hypervolemic hyponatremia (CHF) but is not recommended for the hypovolemic hyponatremia presumed in CSWS (44,45).

21. The answer is A. The hyperosmolar hyperglycemic state (formerly hyperglycemic hyper- osmolar nonketotic state [coma]) occurs most commonly in elderly patients and is asso- ciated with a precipitating stress such as infection, myocardial infarction, and so on. The levels of ketosis are lower (smaller dilution positivity) than in diabetic ketoacidosis (DKA) because of more endogenous insulin and a lower production of counter regulatory hormones (e.g., glucagon, catecholamines). Hyperglycemia, however, is usually above 600 mg/dL, leading to an osmotic diuresis, hypovolemia, and hyperosmolality (usually >320 mosm/kg). Treatment is similar to DKA management with primary attention to hypovolemia and reducing serum glucose with insulin. Because less acidemia/acidosis is

present, titration of insulin is primarily against serum glucose, not the HCO3. Recall that hyperglycemia produces a dilutional effect on the serum sodium and that a corrected Na should be calculated when planning replacement fl uids: Corrected Na = measured Na + {0.016 [glucose (g/dL) – 100]} In this scenario, corrected Na = 136 + (0.016 × 780) = 148 mEq/L. Therefore, assuming that the prior 1.5 L of normal saline provided adequate volume support, the replacement fl uid should be 0.45% saline. Otherwise, 0.9% may be continued to achieve euvolemia. Insulin is prescribed as with DKA treatment (initial insulin bolus of 0.1 U/kg followed by an infusion of 0.1 U/hour titrated to decrease serum glucose by 50–70 mg/dL/hour). Dextrose is added to the infusing fl uid when serum glucose reaches 250 mg/dL and as the insulin infusion is then reduced. Potassium, phosphorous, and other electrolytes poten- tially lost during intracellular relocation or diuresis should be monitored and replaced. Rhabdomyolysis may also be present, perhaps related to hypophosphatemia, and requires close monitoring of renal function and creatine phosphokinase. Concurrent identifi cation and treatment of any precipitating stress factors must, of course, occur.

22. The answer is E. Breast cancer is the most common cause of malignancy-related hyper- calcemia and is usually due to direct bone metastases. Lung cancer, however, causes pro- duction of parathyroid-related protein (PTHrP), a hormone that causes release of calcium from bone without direct bone metastases. Calcium is highly bound to serum albumin, but the unbound ionized Ca is the metabolically active component. Ionized Ca should also be quantifi ed and the total Ca corrected for serum albumin: Corrected Ca = measured Ca (mg/dL) + {0.8 × [4.0 – Alb (mg/dL)]} The primary treatment for all causes of hypercalcemia is saline-induced diuresis to increase renal calcium loss and to treat concomitant hypovolemia. When euvolemia is assured, a loop diuretic (e.g., furosemide) may be added to increase clearance. During vigorous rehy- dration and increasing urine output, monitoring/treatment of secondary hypokalemia or hypomagnesemia is essential. Calcitonin is given early after diuresis is initiated but tac- hyphylaxis quickly develops. Bisphosphonates (e.g., pamidronate) may be given early to decrease bone reabsorption, but do not take effect for several days. Corticosteroids decrease gastrointestinal absorption and may infl uence some tumors’ effect (46–48).

314

66485457-66485438 www.ketabpezeshki.com

ZZakaria_87574_PTR_CH20_10-06-13_299-318.inddakaria_87574_PTR_CH20_10-06-13_299-318.indd 314314 66/19/2013/19/2013 8:48:288:48:28 PMPM ELECTROLYTE AND ENDOCRINE DISORDERS: Answers

23. The answer is E. This patient demonstrates the usual fi ndings of established hypothyroid- ism with perhaps an accelerated decline related to her new seizures, also occasionally seen in this condition, or some other precipitating stress factor. Due to the multi-organ effects of defi cient thyroid hormone, treatment must be cautious but urgent. Active rewarming is relatively contraindicated because of its potential effect on oxygen needs and possible vasodilation-related worsening of her blood pressure. Sedation is relatively contraindi- cated because of its possible effect on neurologic status and blood pressure. Some patients with advanced hypothyroidism may also demonstrate adrenal insuffi ciency. Serum corti- sol measurement during the initial evaluation may be helpful in determining the duration of steroid treatment, but most guidelines recommend concurrent therapy with “stress– dose” hydrocortisone when thyroid hormone is started. Some authors also recommend empiric antibiotics until culture data are available (27).

24. The answer is B. Increased renin relative to aldosterone activity is documented in sepsis, liver disease, SAH, and other critical illnesses. It is associated with persistent hypotension and, in some studies with death and multi-organ failure. Etiological possibilities are not well defi ned, but include: a. Decreased renal perfusion that might increase renin–angiotensin release that should increase Na reabsorption b. Increase in natriuretic hormone from the brain or heart that increases renal Na loss and water reabsorption c. Reduced synthesis of aldosterone in the adrenal gland due to increased production of cortisol d. Damage to the zona glomerulosa of the adrenal gland wherein mineralocorticoids are produced e. Inhibition of aldosterone synthesis by dopamine, oxygen-free radicals, or cytokines. The predominant change in electrolyte concentration is hyponatremia. No specifi c therapy is proposed for the overall syndrome, as hyponatremia is treated as its most important symptom (49–51).

References

1. Baran DT, Aronin N. In: Irwin RS, Rippe JM, eds. Intensive Care Medicine. 6th ed. Philadelphia: Wolters Kluwer/Lippincott Williams & Wilkins 2009: 1287–1293. 2. Baird GS. Ionized calcium. Clin Chim Acta. 2011; 412:696–701. 3. Apostol, A, Apostol R, Ali E, et al. Cerebral spinal fl uid and serum ionized magnesium and calcium levels in preeclamptic women during administration of magnesium sulfate. Fertil Steril. 2010; 94:276–282. 4. Whittington JE, La’ulu SL, Hunsaker JJ, et al. The osmolal gap: what has changed? Clin Chem. 2010; 56:1353–1355. 5. Seo VV, Oh H. Alterations in serum osmolality, sodium and potassium levels after repeated mannitol administration. J Neurosci Nurs. 2010; 42:201–207. 6. Tsai SF, Shu KH. Mannitol-induced acute renal failure. Clin Nephrol. 2010;74:70–73. 7. Dorman HR, Sondheimer JH, Cadnapaphornchai, P. Mannitol-induced acute renal fail- ure. Medicine. 1990; 69:153–159.

315

66485457-66485438 www.ketabpezeshki.com

ZZakaria_87574_PTR_CH20_10-06-13_299-318.inddakaria_87574_PTR_CH20_10-06-13_299-318.indd 315315 66/19/2013/19/2013 8:48:288:48:28 PMPM GENERAL CRITICAL CARE: PATHOLOGY, PATHOPHYSIOLOGY, AND THERAPY

8. Garcia-Morales EJ, Cariappa R, Parvin CA, et al. Osmole gap in neurologic-neurosurgical intensive care unit: its normal value, calculation and relationship with mannitol serum concentrations. Crit Care Med. 2004; 32:986–991. 9. Trepiccione F, Christensen BM. Lithium-induced nephrogenic diabetes insipidus: new clinical and experimental fi ndings. J Nephrol. 2010; 16 (3 Suppl): S43–S48. 10. Olsen KR. Lithium poisoning. In: Irwin RS, Rippe JM, eds. Intensive Care Medicine. 6th ed. Philadelphia: Wolters Kluwer/Lippincott Williams & Wilkins, 2009:1639–1643. 11. Noda Y, Sohara E, Ohta E, et al. Aquaporins in kidney pathophysiology. Nat Rev Nephrol. 2010; 6:168–178. 12. Black RM, Noroian GO. Disorders of plasma sodium and plasma potassium. In: Irwin RS, Rippe JM, eds. Intensive Care Medicine. 6th ed. Philadelphia: Wolters Kluwer/Lippincott Williams & Wilkins; 2009:898–925. 13. Perez GO, Oster JR, Vaamonde CA. Serum potassium concentration in academic states. Nephron. 1981; 27: 233–243. 14. Odier,C, Nguyen DK, Panisset M. Central pontine and extrapontine myelinolysis: from epileptic and other manifestations to cognitive prognosis. J Neurol. 2010;257:1176–1180. 15. Al-Sarraf AJ, Haque M, Pudek M, et al. Central pontine myelinolysis after orthotopic liver transplant: a rare complication. Exp Clin Transplant. 2010;8:321–324. 16. Norenberg MD. Central pontine myelinolysis: historical and mechanistic considerations. Metab Brain Dis. 2010; 25:97–106. 17. Metterlein T, Zink W, Dranke E, et al. Cardiopulmonary bypass in malignant hyper- thermia susceptible patients: a systematic review of published cases. J Thorac Cardiovasc Surg. 2011; 141:1488–1495. 18. Larach MG, Gronert GA, Allen GC, et al. Clinical presentation, treatment, and compli- cations of malignant hyperthermia in North America from 1987–2006. Anesth Analg. 2010;110:498–507. 19. Sihler, KC, Napolitano LM. Complications of massive transfusion. Chest. 2010; 137:209–220. 20. Ho KM, Leonard AD. Concentration-dependent effect of hypocalcaemia on mortality of patients with critical bleeding requiring massive transfusion: a cohort study. Anaesth Intensive Care. 2011; 39:46–54. 21. Smith HM, Farrow SJ, Ackerman JD, et al. Cardiac arrests associated with hyperkalemia during red blood cell transfusion: a case series. Anesth Analg. 2008;106:1062–1069. 22. Ho KM, Leonard A. Risk factors and outcome associated with hypomagnesemia in mas- sive transfusion. Transfusion. 2011; 51:270–276. 23. Lehnhardt A, Kemper MJ. Pathogenesis, diagnosis and management of hyperkalemia. Pediatr Nephrol. 2011; 26:377–384. 24. El-Sherif N, Turitto G. Electrolyte disorder and arrhythmogenesis. Cardiol J. 2011; 18:233–245. 25. Elliott MJ, Ronksley PE, Clase CM, et al. Management of patients with acute hyper- kalemia. CMAJ. 2010; 182:1631–1635. 26. Tessnow AH, Wilson JD. The changing face of Sheehan’s syndrome. Am J Med Sci. 2010; 340:402–406. 27. Sims CA. How do I diagnose and manage acute endocrine emergencies in the intensive care unit? In: Deutschman CS, Neligan PJ, eds. Evidence-Based Practice of Critical Care. Philadelphia, PA: Saunders/Elsevier; 2010:525–541.

316

66485457-66485438 www.ketabpezeshki.com

ZZakaria_87574_PTR_CH20_10-06-13_299-318.inddakaria_87574_PTR_CH20_10-06-13_299-318.indd 316316 66/19/2013/19/2013 8:48:288:48:28 PMPM ELECTROLYTE AND ENDOCRINE DISORDERS: Answers

28. Warner MH, Beckett GJ. Mechanisms behind the non-thyroidal illness syndrome: an update. J Endocrinol. 2010: 205:1–13. 29. Meyer S, Schuetz P, Wieland M, et al. Low triiodothyronine syndrome: a prognostic marker for outcome in sepsis? Endocrine. 2011; 39:167–174. 30. Marik PE, Pastores SM, Annane D, et al. Recommendations for the diagnosis and man- agement of corticosteroid insuffi ciency in critically ill adult patients: Consensus state- ments from an international task force by the American College of Critical Care Medicine. Crit Care Med. 2008; 36:1937–1949. 31. Eid L, Weiss YG. Can acute adrenal insuffi ciency be diagnosed in the intensive care unit? If so, how should it be managed? In: Deutschman CS, Neligan PJ, eds. Evidence-Based Practice of Critical Care. Philadelphia: Saunders/Elsevier; 2010:509–520. 32. Edwin SB, Walker PL. Controversies surrounding the use of etomidate for rapid sequence intubation in patients with suspected sepsis. Ann Pharmacother. 2010; 44: 1307–1313. 33. Lutgers, HL, Vergragt J, Dong P, et al. Severe hypercortisolism: a medical emergency requiring urgent intervention. Crit Care Med. 2010; 38:1598–1601. 34. Sharma ST, Nieman LK. Cushing’s syndrome: all variants, detection, and treatment. Endocrinol Clin North Am. 2011; 40:379–391. 35. Moghissi ES, Korykowski MT, DiNardo M, et al. American Association of Clinical Endocrinologists and American Diabetes Association consensus statement on inpatient glycemic control. Diabetes Care. 2009; 32:1119–1131. 36. Marik PE, Preiser J. Toward understanding tight glycemic control in the ICU. Chest. 2010; 137:544–551. 37. Preiser J. Should blood glucose be tightly controlled in the intensive care unit? In: Deutschman CS, Neligan PJ, eds. Evidence-Based Practice of Critical Care. Philadelphia: Saunders/Elsevier ; 2010:505–508. 38. Powner DJ, Boccalandro C, Alp MS, Vollmer DG. Endocrine failure after traumatic brain injury in adults. Neurocrit Care. 2006; 5:61–70. 39. Mesquita J, Varela A, Medina JL. Trauma and the endocrine system. Endocrinol Nutr. 2010; 57:492–499. 40. Park KD, Kim DY, Lee JK, et al. Anterior pituitary dysfunction in moderate-to-severe chronic traumatic brain injury patients and the infl uence on functional outcome. Brain Inj. 2010; 24:1330–1335. 41. Powner DJ. How does one care for the heart-beating, brain dead, adult organ donor patient? In: Deutschman CS, Neligan PJ, eds. Evidence-Based Practice of Critical Care. Philadelphia: Saunders/Elsevier; 2010:637–642. 42. Bogazzi F, Bartalena L, Martino E. Approach to the patient with amiodarone-induced thy- rotoxicosis. J Clin Endocrinol Metab. 2010; 95:2529–2535. 43. Longcope C, Aronin N. Hypoadrenal crisis. In: Irwin RS, Rippe JM, eds. Intensive Care Medicine. 6th ed. Philadelphia: Wolters Kluwer/Lippincott Williams & Wilkins; 2008: 1280–1286. 44. Yee AH, Burns JD, Wijdicks EFM. Cerebral salt wasting: pathophysiology, diagnosis and treatment. Neurosurg Clin N Am. 2010: 21:339–352. 45. Wendell LC, Levine JM. How should aneurysmal SAH be managed? In: Deutschman CS, Neligan PJ, eds. Evidence-Based Practice of Critical Care. Philadelphia: Saunders/Elsevier; 2010:414–421.

317

66485457-66485438 www.ketabpezeshki.com

ZZakaria_87574_PTR_CH20_10-06-13_299-318.inddakaria_87574_PTR_CH20_10-06-13_299-318.indd 317317 66/19/2013/19/2013 8:48:288:48:28 PMPM GENERAL CRITICAL CARE: PATHOLOGY, PATHOPHYSIOLOGY, AND THERAPY

46. Pelosof, LC, Gerber DE. Paraneoplastic syndromes: an approach to diagnosis and treat- ment. Mayo Clin Proc. 2010; 85:838–854. 47. Lumachi F, Brunello A, Roma A, et al. Cancer-induced hypercalcemia. Anticancer Res. 2009; 29:1551–1555. 48. Baird GS. Ionized calcium. Clin Chim Acta. 2011; 412:696–701. 49. Annane D. Is there a mineralocorticoid defi ciency in critically ill patients? How can it be diagnosed? Should it be treated? In: Deutschman CS, Neligan PJ, eds. Evidence-Based Practice of Critical Care. Saunders/Elsevier, Philadelphia. 2010: 521–524. 50. Audibert G, Steinmann G, deTalace N, et al. Endocrine response after severe SAH related to sodium and blood volume regulation. Anesth Analg. 2009; 108:1922–1928. 51. du Cheyron D, Bouchet B, Cauquelin B, et al. Hyperreninemic hypoaldosteronism syn- drome, plasma concentrations of interleukin-6 and outcome in critically ill patients with liver cirrhosis. Intensive Care Med. 2008; 34:116–124.

318

66485457-66485438 www.ketabpezeshki.com

ZZakaria_87574_PTR_CH20_10-06-13_299-318.inddakaria_87574_PTR_CH20_10-06-13_299-318.indd 318318 66/19/2013/19/2013 8:48:298:48:29 PMPM 21 Infectious Disease Safdar A. Ansari QUESTIONS

1. A 35-year-old man presents to an ED in southern Virginia in July, with fever, headaches, confusion, myalgias, and malaise for 5 days. He has been anuric for the past 24 hours. He had been hiking with his friends through the countryside last week. His BP is 85/45, pulse is 133, and RR is 32. His skin has petechial lesions over his trunk and extremities with one suspicious necrotic lesion on his thigh. WBC count is 11,000 (20% bands) and platelets are 33,000, with transaminitis (3× normal). He receives all measures of early goal-directed therapy (EGDT) for sepsis. Besides broad-spectrum antibiotic coverage with vancomycin and ceftriaxone, what other antibiotic should you consider? A. Linezolid B. Doxycycline C. Amoxicillin D. Bacitracin E. Isoniazid

ANSWERS TO THIS SECTION CAN BE FOUND ON PAGE 327 319

66485457-66485438 www.ketabpezeshki.com

ZZakaria_87574_PTR_CH21_10-06-13_319-332.inddakaria_87574_PTR_CH21_10-06-13_319-332.indd 319319 66/19/2013/19/2013 4:46:584:46:58 PMPM GENERAL CRITICAL CARE: PATHOLOGY, PATHOPHYSIOLOGY, AND THERAPY

2. A 42-year-old woman with rheumatoid arthritis, on oral prednisone therapy, is brought to the ED with 2 days of fever, headache, severe myalgias, dry cough, dyspnea, and coryza. She is a schoolteacher, and several kids in her class have been sick with the fl u. Her lungs have scattered crackles in both lung fi elds, and labs reveal a WBC count of 13,000 with 69% lymphocytes. Chest x-ray (CXR) shows bilateral increased interstitial markings. Besides routine supportive and diagnostic measures, what test would you send next? A. Urine Legionella antigen B. Immunofl uorescence antibody staining for infl uenza from blood C. Rapid infl uenza diagnostic test from a urine specimen D. Rapid infl uenza diagnostic test from a throat swab E. Viral cell cultures for infl uenza virus from a throat swab

3. If the rapid infl uenza test for the patient in Question 2 is positive, which treatment would you institute next? A. Acyclovir 800 mg orally 5 times daily B. Ritonavir 600 mg orally 2 times daily C. Clarithromycin 500 mg orally 3 times daily D. Oseltamivir 75 mg 2 times daily E. Amantadine 100 mg orally 2 times daily

4. A 59-year-old prison inmate is being evaluated for an abnormal screening chest x-ray (CXR) done during incarceration. He denies any cough, fevers, night sweats, weight loss, or sick contacts. He does endorse receiving a “tuberculosis injection” when he was a young man in Guatemala, which ulcerated and was painful. If his CXR fi ndings are consistent with a remote TB infection, which would be the most effi cient, sensitive, and specifi c test for latent TB in this patient? A. Induced sputum for TB culture B. Bronchoalveolar lavage for acid-fast stain C. Tuberculin skin test for induration D. Quantiferon-TB Gold test E. Induced sputum for acid-fast stain

5. A patient is brought to the ICU, with a temperature of 35.2°C, heart rate of 101, and RR of 24. To fulfi ll systemic infl ammatory response syndrome (SIRS) criteria, what must he also have? A. Obvious source of infection B. Positive blood cultures C. WBC count greater than 10,000 or less than 5,000 (cells per cubic millimeter) D. WBC count greater than 12,000 or less than 4,000 (cells per cubic millimeter) E. WBC count greater than 12,000 (cells per cubic millimeter) with greater than 25% imma- ture (band) forms

320

66485457-66485438 www.ketabpezeshki.com

ZZakaria_87574_PTR_CH21_10-06-13_319-332.inddakaria_87574_PTR_CH21_10-06-13_319-332.indd 320320 66/19/2013/19/2013 4:46:584:46:58 PMPM INFECTIOUS DISEASE: Questions

6. A researcher wants to create an animal model of septic shock in the lab and then later in healthy human volunteers. He is considering the following mediators for this purpose. Which would be his best bet to induce high cardiac index/low systemic vascular resis- tance index/depressed ejection fraction in healthy animals and humans? A. Tumor necrosis factor B. Prostacyclin C. Teichoic acid antigen D. Bacterial endotoxin lipopolysaccharide (LPS) E. Toxic shock syndrome toxin

7. Prior to the administration of effective antimicrobial therapy in septic shock, the most important factor that determines survival is: A. Degree of depression of myocardial ejection fraction B. Duration of hypotension C. Degree of acute renal failure and azotemia D. Degree of lactic acidosis E. Time it takes for blood cultures to turn positive

8. Delaying the administration of appropriate antimicrobial therapy after the onset of hypotension, defi ned as mean arterial pressure (MAP) < 65, in a patient with septic shock results in increasing mortality trends in which hour? A. First hour of hypotension B. Second hour of hypotension C. Third hour of hypotension D. Fourth hour of hypotension E. Fifth hour of hypotension

9. A 75-year-old man is evaluated on the rehabilitation fl oor. He has chronic kidney dis- ease and recently had an intracerebral hemorrhage that left him hemiplegic. He has now developed a cough and confusion, and his neurological weakness is more pronounced. His temperature is 102.1°F, RR is 28, and oxygen saturation is 91% on room air. Rhonchi are auscultated on lung exam, and mild pitting edema is noted in the legs. Labs reveal leukopenia and a blood urea nitrogen of 58 mg/dL. Chest x-ray (CXR) reveals a retrocar- diac opacity. He is diagnosed with pneumonia. In addition to supplemental oxygen and antipyretics, what is the most appropriate management for this patient? A. Administer an oral macrolide agent and continue care on the rehabilitation fl oor B. Obtain blood cultures, administer IV piperacillin–tazobactam and vancomycin, and transfer to ICU C. Obtain blood cultures, administer IV ceftriaxone and a respiratory quinolone, and transfer to ICU D. Obtain blood cultures, administer IV vancomycin within 4 hours, and transfer to ICU E. Obtain blood cultures and await results prior to initiating antimicrobial therapy

321

66485457-66485438 www.ketabpezeshki.com

ZZakaria_87574_PTR_CH21_10-06-13_319-332.inddakaria_87574_PTR_CH21_10-06-13_319-332.indd 321321 66/19/2013/19/2013 4:46:584:46:58 PMPM GENERAL CRITICAL CARE: PATHOLOGY, PATHOPHYSIOLOGY, AND THERAPY

10. During transfer to the ICU, the patient in Question 9 becomes less responsive and hypotensive, which resolves with administration of 1 L of crystalloid saline solution as a bolus. A serum lactate level at this time is 3 times the upper limit of normal from the reference lab. Which of the following interventions has been shown to improve mortality in this setting? A. IV corticosteroids B. Early goal-directed therapy (EGDT) C. Mechanical ventilation at tidal volumes of 10 mL/kg and a RR of 6 to 8/minute D. Procalcitonin E. Recombinant activated protein C

11. As the patient in the previous question is stabilized in the ICU, his central venous pres- sure (CVP) and mean arterial pressure (MAP) after central line insertion, arterial line insertion, ventilation, and volume resuscitation are found to be 9 mmHg and 69 mmHg, respectively. Mixed venous oxygen saturation is found to be 66%. His hematocrit is 24%. What would be the next appropriate step in his management? A. Administration of IV norepinephrine B. Administration of IV dobutamine C. Administration of 100% fractional inspired oxygen D. Increase the RR on the ventilator by 4 E. Transfuse packed red blood cells to target hematocrit of 30%

12. A 69-year-old alcoholic man was admitted to the ICU. He was found passed out in an alley, covered in vomit and beer. He was transferred to a hospital, where he was som- nolent and looked uncomfortable. His temperature was 103.5°F, heart rate was 109, and oxygen saturation was 89%. He had a severe cough productive of thick sputum that was dark red and jellylike. Based on this information, what is the most likely pathogen? A. Aspergillus B. Mycoplasma pneumonia C. Mycobacterium tuberculosis D. Klebsiella pneumonia E. Infl uenza B

13. An ICU patient requires a central venous catheter (CVC) for central venous pressure (CVP)–guided resuscitation in the setting of sepsis. After insertion of the CVC, which of the following measures has been shown to minimize the risk of its infection? A. Exchanging the CVC over a guidewire every 3 days B. Exchanging the CVC over a guidewire every 7 days C. Exchanging the CVC with a fresh site every 7 days D. Prophylactic daily vancomycin IV through CVC E. None of the above

322

66485457-66485438 www.ketabpezeshki.com

ZZakaria_87574_PTR_CH21_10-06-13_319-332.inddakaria_87574_PTR_CH21_10-06-13_319-332.indd 322322 66/19/2013/19/2013 4:46:584:46:58 PMPM INFECTIOUS DISEASE: Questions

14. A patient is admitted to the ICU after exposure to poison ivy with a facial rash and edema. He subsequently develops airway obstruction due to tongue swelling and is intubated via the nasal route. Which of the following infectious complications is he at highest risk for? A. Urinary tract infection B. Meningitis C. Orbital cellulitis D. Maxillary sinusitis E. Otitis media

15. Which of the following interventions will signifi cantly reduce the chances of the patient in Question 14 developing the infectious complication? A. Prophylactic linezolid IV B. Inhaled tobramycin through the endotracheal tube C. Clindamycin gel applied to the nasal cavity D. Early transfer of endotracheal tube to the oral route E. Warmed and humidifi ed air through the endotracheal tube

16. A 62-year-old homeless man is admitted to the ICU with signs of impending respiratory failure. He has a 6-month history of productive cough, progressive weight loss, night sweats, and dyspnea. He progresses to respiratory failure and during intubation the bed- side nurse is exposed to his sputum, which later grows acid-fast bacilli. What is the next appropriate plan of action for the nurse? A. Offer reassurance that the chances of transmission are extremely low and she needs no further follow-up or care B. Order a purifi ed protein derivative (PPD) skin test for TB C. Order a PPD skin test for TB and order preventive treatment if skin induration is greater than 5 mm D. Start preventive therapy with isoniazid, rifampin, and ethambutol E. Start preventive therapy with daily isoniazid for 9 months

17. A postal worker is accidentally exposed to a suspicious white powder while handling a package at the post offi ce. He does not report the incident, and a few days later he develops progressive fever, rigors, chills, dry cough, dyspnea, and malaise. His condition worsens to the point of near collapse at work and he is rushed to the hospital. Initial investigation reveals an elevated WBC count (17,000 cells per mm3) and bilateral infi ltrates and small effusions on chest x-ray (CXR). He progresses to respiratory failure, requiring intubation and mechanical ventilation. Blood cultures grow Gram-positive rods on day 3. What is the most likely diagnosis? A. Cat scratch disease B. Gastrointestinal anthrax C. Legionnaires’ disease D. Inhalational anthrax E. Cutaneous anthrax

323

66485457-66485438 www.ketabpezeshki.com

ZZakaria_87574_PTR_CH21_10-06-13_319-332.inddakaria_87574_PTR_CH21_10-06-13_319-332.indd 323323 66/19/2013/19/2013 4:46:584:46:58 PMPM GENERAL CRITICAL CARE: PATHOLOGY, PATHOPHYSIOLOGY, AND THERAPY

18. Which of the following treatment regimens would be the most appropriate for the patient in Question 17? A. IV ciprofl oxacin + IV clindamycin B. Oral doxycycline C. PO ampicillin D. IV fl uconazole E. IV ceftriaxone

19. A 56-year-old man is admitted to the ICU from a nursing home with a large intracerebral hemorrhage. Despite receiving acute treatment for his hemorrhage, he progresses to an unresponsive state and is intubated and mechanically ventilated for hypercarbic respira- tory failure and airway protection. Which of the following interventions is most likely to reduce his chances of infectious complications of intubation? A. Exchanging the ventilator circuit every 3 days B. Use of oral antiseptic chlorhexidine swabs C. Exchanging the ventilator circuit every 7 days D. Use of proton pump inhibitors for gastric acid control E. Switching the orotracheal tube to a nasotracheal tube when able

20. Despite adequate management, on the sixth day of intubation, the patient in Question 19 develops a temperature of 39.3°C, blood leukocyte count of 16,700/mm, and a new infi ltrate on chest radiography. Ventilator-associated pneumonia is suspected. What is the most appropriate diagnostic maneuver at this stage? A. Tracheal aspiration of secretions through the endotracheal tube B. Bronchoscopy with bronchoalveolar lavage C. Blood cultures and tracheal aspiration of secretions through the endotracheal tube D. Bronchoscopy with a protected specimen brush sample E. Blood culture alone

21. A decision is made to start empiric antibiotic therapy for the patient in Question 20, for a clinical diagnosis of ventilator-associated pneumonia. Which of the following antibiotic regimens would be the most appropriate? A. IV vancomycin and IV piperacillin/tazobactam B. IV ciprofl oxacin C. Oral azithromycin D. IV vancomycin and IV ceftriaxone E. IV ampicillin/sulbactam

22. For the patient in Question 21, what is the most appropriate duration of antibiotic therapy if a sensitive pathogen is confi rmed with sputum cultures that is not a nonlactose fer- menting Gram-negative rod, and no other complications or infections arise? A. 15 days B. 4 days C. 8 days D. 18 days E. 21 days

324

66485457-66485438 www.ketabpezeshki.com

ZZakaria_87574_PTR_CH21_10-06-13_319-332.inddakaria_87574_PTR_CH21_10-06-13_319-332.indd 324324 66/19/2013/19/2013 4:46:584:46:58 PMPM INFECTIOUS DISEASE: Questions

23. A 38-year-old man is admitted to the ICU with an unknown illness. He reports a 2-week history of progressive fever, chills, anorexia, malaise, myalgias, and night sweats, with signifi cant worsening in the past 24 hours, prompting his arrival at the hospital. At the time of admission, his temperature is 39.9°C, BP is 94/58, heart rate is 105, and serum WBC count is 18,000/mm3. Cardiac auscultation reveals a new harsh systolic murmur. Blood cultures reveal growth of Gram-positive cocci in all four culture bottles within 12 hours. What is the appropriate initial antimicrobial regimen for this patient? A. IV ampicillin and sulbactam B. IV vancomycin and gentamycin C. IV vancomycin and cefepime D. IV vancomycin, ceftriaxone, and metronidazole E. IV vancomycin and azithromycin

24. The patient in Question 23 receives appropriate antibiotic therapy in the ICU. His hypotension progressively worsens despite adequate fl uid resuscitation, and transtho- racic echocardiogram reveals mitral vegetations and mitral regurgitation. After antibiotic selection and fl uid resuscitation, what is the next appropriate step in management? A. Start IV vasopressor agents B. Start IV vasopressor agents and insert intra-aortic balloon pump C. Start IV vasopressor agents and call cardiothoracic surgery for early valvular surgery D. Insert intra-aortic balloon pump E. Perform transesophageal echocardiogram

25. A 23-year-old woman presents to the hospital with petechial lesions and is diagnosed with idiopathic thrombocytopenic purpura. She is admitted to the ICU for hemorrhagic pleural effusions, and has had a central venous catheter (CVC) in place for 6 days when she develops purulence and erythema at the site of insertion. On the same day she is noted to have fever spikes up to 39.5°C, platelet counts of 46,000/mm3, and she becomes delirious. Initial Gram stain of her blood reveals Gram-positive cocci in clusters. Culture identifi cation and sensitivities will not be available for another 24 hours at least. What is the appropriate antibiotic therapy at this time? A. IV linezolid B. IV penicillin C. IV oxacillin D. IV vancomycin and zosyn E. IV vancomycin

325

66485457-66485438 www.ketabpezeshki.com

ZZakaria_87574_PTR_CH21_10-06-13_319-332.inddakaria_87574_PTR_CH21_10-06-13_319-332.indd 325325 66/19/2013/19/2013 4:46:594:46:59 PMPM GENERAL CRITICAL CARE: PATHOLOGY, PATHOPHYSIOLOGY, AND THERAPY

26. A penicillin-allergic patient is admitted to the Neuro-ICU for refractory status epilepticus requiring maximal antiepileptic drug therapy and propofol with mechanical ventilation. During the course of his ICU stay, the neighboring patient is diagnosed with Pseudomonas aeruginosa decubitus ulcers and osteomyelitis. On his fi fth day of mechanical ventilation he develops purulent green secretions through his endotracheal tube, high-grade fevers, and a relapse of his seizures. Initial Gram stain from his blood confi rms Gram-negative rods. What is the most appropriate antibiotic therapy at this time? A. IV doripenem B. IV meropenem C. IV piperacillin/tazobactam D. IV vancomycin E. IV cefotaxime

326

66485457-66485438 www.ketabpezeshki.com

ZZakaria_87574_PTR_CH21_10-06-13_319-332.inddakaria_87574_PTR_CH21_10-06-13_319-332.indd 326326 66/19/2013/19/2013 4:46:594:46:59 PMPM 21

ANSWERS

1. The answer is B. Doxycycline would be an appropriate additional agent for coverage of suspected rickettsial and tick-borne infectious diseases. The patient’s condition with systemic infl ammatory response syndrome (SIRS), petechial rash, necrotic skin lesion, thrombocytopenia, and transaminitis are all consistent with rickettsial disease.

2. The answer is D. Rapid infl uenza diagnostic test from a throat swab would be the most appropriate and time-effi cient way to confi rm the diagnosis in this case. This immuno- suppressed patient appears to have been exposed to and developed the fl u and therefore would require a diagnostic rapid infl uenza screen to confi rm prior to treatment. A rapid infl uenza screen and immunofl uorescence studies should be checked on nasal secretions and not urine, and viral cell cultures take at least 3 to 5 days for confi rmation. There is no historical exposure to Legionella, and the diagnostic suspicion is low.

3. The answer is D. Although both oseltamivir and amantadine have antiviral activ- ity against infl uenza, amantadine is no longer recommended therapy due to the high incidence of emerging resistance. Oseltamivir 75 mg twice daily is the correct choice. A macrolide antibiotic, an anti-herpes antiviral, or an antiretroviral agent would not be indicated.

4. The answer is D. Quantiferon-TB Gold test is the most sensitive and specifi c screening test for latent TB. Tuberculin skin test is less sensitive and specifi c and may yield false positive results with a childhood Bacille–Calmette–Guerin (BCG) vaccination, and the other options listed are not appropriate screening tests for latent TB.

5. The answer is D. WBC count greater than 12,000 or less than 4,000 (cells per cubic milli- meter). There is no requirement to identify or confi rm a source of infection (choices A and B) and the cutoff for immature (band) forms is 10% and not 25%.

327

66485457-66485438 www.ketabpezeshki.com

ZZakaria_87574_PTR_CH21_10-06-13_319-332.inddakaria_87574_PTR_CH21_10-06-13_319-332.indd 327327 66/19/2013/19/2013 4:46:594:46:59 PMPM GENERAL CRITICAL CARE: PATHOLOGY, PATHOPHYSIOLOGY, AND THERAPY

6. The answer is D. Bacterial endotoxin LPS has been shown to most reliably induce septic shock in healthy human and animal volunteers in research studies. The other agents listed are not as potent as endotoxin.

7. The answer is B. The duration of hypotension prior to the administration of effective antimicrobial therapy is the number one factor that determines survival in septic shock. Mortality risk starts increasing in the second hour of hypotension prior to antibiotics and continues to increase exponentially. Though the other options are also implicated in sepsis pathophysiology, they have not been shown to infl uence survival as much.

8. The answer is B. The second hour of hypotension sees increasing mortality trends. It has been shown that mortality is increased without adequate antimicrobial therapy and circu- latory support after the fi rst golden hour of hypotension with septic shock (1).

9. The answer is B. Obtain blood cultures, administer IV piperacillin–tazobactam and van- comycin, and transfer to the ICU. This is pneumonia acquired during an inpatient set- ting and therefore requires prompt diagnostic blood cultures and appropriate antibiotics. Community-acquired pneumonia would best be treated by IV ceftriaxone and a respira- tory quinolone, but this patient has been in the hospital for his recent intracerebral hemor- rhage and would require broader coverage, such as vancomycin and piperacillin–tazobac- tam. His triage to the ICU is justifi ed by his age, recent brain hemorrhage, confusion, and uremia, which place him at high risk for further complications and decompensation. Various scores (such as the CURB-65) or prognostic scoring systems (Pneumonia Severity Index) can be used to appropriately triage patients with pneumonia.

10. The answer is B. This patient has severe sepsis, which is defi ned by an infection, systemic infl ammatory response syndrome (SIRS), and organ dysfunction or tissue hypoperfusion (elevated lactate). EGDT consisting of aggressive IV fl uids, vasopressors, packed red blood cells, and inotropic agents has been shown to reduce mortality by 16.5% compared to stan- dard therapy (2). The other interventions listed have not had such a profound effect on mortality. Mechanical ventilation at 6 to 8 mL/kg ideal body weight (IBW) has been shown to be better than higher tidal volumes, such as 10 mL/kg IBW, to avoid volutrauma.

11. The answer is E. The patient has met all the goals of early goal-directed therapy (EGDT) except for mixed venous oxygen saturation, which is below the target of 70%. With a low hematocrit, the appropriate next step is to transfuse packed red blood cells to a target hematocrit of 30%, and if this does not correct the mixed venous oxygen, to move to ino- tropic agents. If this patient’s hematocrit had been above 30%, then inotropes would have been the next step in EGDT (2).

12. The answer is D. Klebsiella pneumonia is the most likely etiology in the setting of patients who are at the extremes of age, immunocompromised, and alcoholic. Currant red jelly sputum is also consistent with this pathogen. The other pathogens listed would not be associated with this color of sputum and are less likely in this patient’s demographic and clinical picture.

13. The answer is E. None of the measures listed (scheduled catheter exchanges, catheter replacements, or prophylactic antibiotics) have been shown to reduce the incidence of CVC-related infections.

328

66485457-66485438 www.ketabpezeshki.com

ZZakaria_87574_PTR_CH21_10-06-13_319-332.inddakaria_87574_PTR_CH21_10-06-13_319-332.indd 328328 66/19/2013/19/2013 4:46:594:46:59 PMPM INFECTIOUS DISEASE: Answers

14. The answer is D. Maxillary sinusitis is highly associated with nasal intubation due to trauma, edema, and obstruction of drainage from the sinus ostia. The incidence of sinus- itis can be from 2% to 25%.

15. The answer is D. Early transfer of an endotracheal tube to the oral route will signifi cantly reduce the chances of sinusitis without the unnecessary exposure to antibiotics by any route (local, inhaled, or intravenous).

16. The answer is E. Start preventive therapy with daily isoniazid for 9 months. After expo- sure to Mycobacterium tuberculosis, PPD testing may be initially negative and should only be checked after 12 weeks. If exposure to the bacterium is likely, preventive ther- apy with single-agent therapy (isoniazid daily for 9 months) is indicated. If the PPD is negative at 12 weeks, the preventive therapy can be discontinued. If the bacilli from the index patient are shown to be multi-drug-resistant, a more aggressive treatment plan can be pursued (3).

17. The answer is D. The patient has a pulmonary infectious etiology with Gram-positive bacteria growing from the blood after exposure to a possible biohazard agent, which is consistent with inhalational anthrax. He does not manifest or report any signs of cuta- neous or gastrointestinal disease. Both Legionella pneumophila and Bartonella henselae (cat scratch disease) are Gram-negative organisms that rarely grow out in blood cultures.

18. The answer is A. Ciprofl oxacin and doxycycline are both appropriate antimicrobial agents for Bacillus anthracis, but in this patient with severe disease (respiratory failure) oral treatment would be inadequate. IV ciprofl oxacin coupled with a second agent, IV clindamycin, would be the most appropriate treatment for this severe case of inhalational anthrax. Ampicillin would be inadequate as a single agent, and ceftriaxone would be an inappropriate choice for Bacillus anthracis.

19. The answer is B. Use of oral antiseptic chlorhexidine swabs is recommended. Large pro- spective studies have shown that oral antiseptics (like chlorhexidene) reduce the chances of ventilator-associated pneumonia. Such a benefi t has not been demonstrated with sched- uled ventilator circuit changes. Both proton pump inhibition and nasotracheal intubation have been associated with increased chances for pneumonia and sinusitis (4).

20. The answer is C. According to the 2005 American Thoracic Society and Infectious Disease Society of America Guidelines, the most appropriate diagnostic steps are blood and spu- tum cultures with tracheal aspiration of secretions. Bronchoscopic collection of secretions for initial diagnosis has not been shown to increase diagnostic yield and exposes the patient to the risk of intervention. Blood cultures alone would be inadequate.

21. The answer is A. The most appropriate regimen is IV vancomycin and IV piperacillin/ tazobactam. This patient has developed pneumonia more than 5 days after his hospital- ization and intubation and is classifi ed as late-onset pneumonia, which has a very high incidence of multi-drug-resistant organisms requiring broad-spectrum antibiotic cover- age. Other listed regimens would not adequately cover nosocomial organisms that are multi-drug-resistant. These regimens can be used once a pathogen has been identifi ed and antibiotic sensitivities have been obtained.

329

66485457-66485438 www.ketabpezeshki.com

ZZakaria_87574_PTR_CH21_10-06-13_319-332.inddakaria_87574_PTR_CH21_10-06-13_319-332.indd 329329 66/19/2013/19/2013 4:46:594:46:59 PMPM GENERAL CRITICAL CARE: PATHOLOGY, PATHOPHYSIOLOGY, AND THERAPY

22. The answer is C. According to a randomized prospective trial, 8 days of antibiotics were just as good as longer durations for non-multi-drug-resistant bacterial pathogens causing pneumonia with no other clinical complications (5).

23. The answer is B. Current treatment guidelines for infective endocarditis with a Gram- positive organism include either IV vancomycin or linezolid with double coverage with an aminoglycoside, in this case gentamycin. Other choices would be subtherapeutic or inappropriate (6).

24. The answer is C. Start IV vasopressor agents and call cardiothoracic surgery for early valvular surgery. According to current literature, early valvular surgery in the setting of infective endocarditis with shock and cardiac failure has led to improved outcomes (8). Cardiac support with vasopressor agents alone would be inadequate, and intra-aortic balloon pump insertion would delay defi nitive therapy. Transesophageal echocardio- gram would not be therapeutic and would likely not add more useful information to the picture. Transthoracic echocardiogram in this case provides confi rmatory and suffi cient information.

25. The answer is E. IV vancomycin is the most appropriate treatment at this time when the Gram-positive cocci have not been speciated and antibiotic susceptibilities are not known. IV linezolid would not be appropriate in a thrombocytopenic patient as it has a high inci- dence of inducing thrombocytopenia with therapy. IV penicillin and oxacillin would be inadequate treatment, and IV Zosyn would be unnecessary in the absence of any Gram- negative organisms on Gram stain.

26. The answer is B. IV meropenem would be the most appropriate therapy of the choices listed for this patient. IV doripenem, though active against pseudomonas, has a suspected epileptogenic role. IV piperacillin would not be suitable in a patient allergic to penicil- lins, and both vancomycin and cefotaxime would be ineffective against pseudomonal infections.

References

1. Kumar A, Roberts D, Wood KE, et al. Duration of hypotension before initiation of effec- tive antimicrobial therapy is the critical determinant of survival in human septic shock. Crit Care Med. 2006 Jun; 34(6):1589–1596. 2. Dellinger RP, Carlet JM, Masur H, et al. Surviving Sepsis Campaign Management Guidelines Committee. Surviving Sepsis Campaign guidelines for management of severe sepsis and septic shock. Crit Care Med. 2004;32:858–873. 3. National Tuberculosis Controllers Association; Centers for Disease Control and Prevention (CDC). Guidelines for the investigation of contacts of persons with infectious tuberculo- sis. Recommendations from the National Tuberculosis Controllers Association and CDC. MMWR Recomm Rep. 2005 Dec 16;54(RR-15):1–47. 4. American Thoracic Society; Infectious Diseases Society of America. Guidelines for the management of adults with hospital-acquired, ventilator-associated, and healthcare- associated pneumonia. Am J Respir Crit Care Med. 2005 Feb 15;171(4):388–416.

330

66485457-66485438 www.ketabpezeshki.com

ZZakaria_87574_PTR_CH21_10-06-13_319-332.inddakaria_87574_PTR_CH21_10-06-13_319-332.indd 330330 66/19/2013/19/2013 4:46:594:46:59 PMPM INFECTIOUS DISEASE: Answers

5. Chastre J, Wolff M, Fagon JY et al. PneumA Trial Group. Comparison of 8 vs 15 days of antibiotic therapy for ventilator-associated pneumonia in adults: a randomized trial. JAMA. 2003 Nov 19;290(19):2588–2598. 6. Thuny F, Grisoli D, Collart F, et al. Management of infective endocarditis: challenges and perspectives. Lancet. 2012 Mar 10; 379(9819): 965–975. Review.

331

66485457-66485438 www.ketabpezeshki.com

ZZakaria_87574_PTR_CH21_10-06-13_319-332.inddakaria_87574_PTR_CH21_10-06-13_319-332.indd 331331 66/19/2013/19/2013 4:46:594:46:59 PMPM 66485457-66485438 www.ketabpezeshki.com

ZZakaria_87574_PTR_CH21_10-06-13_319-332.inddakaria_87574_PTR_CH21_10-06-13_319-332.indd 332332 66/19/2013/19/2013 4:46:594:46:59 PMPM 22 Acute Hematologic Disorders Jitesh Kar and Hanh T. Truong QUESTIONS

1. Which one of the following tumors is associated with the highest risk for venous throm- boembolism (VTE)? A. Malignant glioma B. Meningioma C. Low-grade astrocytoma D. Pituitary adenoma

2. The gold standard test for diagnosis of pulmonary embolism is: A. D-dimer B. Doppler ultrasound of bilateral lower extremities C. Ventilation/perfusion (V/Q) scan D. CT pulmonary angiography E. Conventional pulmonary angiography

ANSWERS TO THIS SECTION CAN BE FOUND ON PAGE 339 333

66485457-66485438 www.ketabpezeshki.com

ZZakaria_87574_PTR_CH22_10-06-13_333-348.inddakaria_87574_PTR_CH22_10-06-13_333-348.indd 333333 66/19/2013/19/2013 4:47:094:47:09 PMPM GENERAL CRITICAL CARE: PATHOLOGY, PATHOPHYSIOLOGY, AND THERAPY

3. A previously healthy, 35-year-old African American female is admitted to the hospital with a rash over her legs and ankles bilaterally. She fi rst noticed the rash around 3 weeks ago, when it started in her feet and gradually spread up to her knees. She has no other medical history and denies taking any prescription medication. There is no history of trauma or bleeding disorders in her family. Physical examination revealed fi ne petechiae and purpura around her ankles and lower legs. The rest of the physical examination is unremarkable. HR is 88, BP 132/65, temperature 98.4°F, RR 15, oxygen saturation 98% on right atrium (RA). Laboratory studies showed the following values: Hemoglobin 14.4 g/dL Mean corpuscular volume (MCV) 90 fL Platelet count 16,000/mm3 Leukocyte count 8,000/mm3 Segmented neutrophils 60% Bands 3% Eosinophil 6% Lymphocytes 24% Monocytes 6% Prothrombin time (PT) 14 seconds Partial thromboplastin time (PTT) 30 seconds Plasma fi brinogen 300 mg/dL D-dimer 0.22 mcg/mL Na 142 mmol/L K 3.8 mmol/L Cl 109 mmol/L Blood urea nitrogen (BUN) 12 mg/dL Cr 1.0 mg/dL Peripheral blood smear is normal. The admitting resident is concerned about the lab results and orders 6 units of random donor platelet transfusion. The following day, the platelet count drops even further to 10,000/µL and the resident consults the intensive care unit team. Which of the following is the most likely cause of the drop in the platelet count? A. Disseminated intravascular coagulation (DIC) B. Thrombotic thrombocytopenic purpura (TTP) C. Antiplatelet antibodies D. Septicemia E. Drug-induced thrombocytopenia 4. A patient with acute myelogenous leukemia, on chemotherapy, had epistaxis that did not stop with pressure. You ordered a stat complete blood count (CBC), which showed hemo- globin of 8 g/dL and platelet count of 10,000/µL. You decided to transfuse the patient with 6 units of pooled platelets and to repeat the platelet count 1 hour after the transfusion, at which time it was found to be 15,000/µL. Which one of the following is the most likely explanation for these fi ndings? A. This is a normal response B. You have measured platelet counts too early C. The patient has antibodies against platelets D. You have not given enough platelets E. The patient has disseminated intravascular coagulation (DIC)

334

66485457-66485438 www.ketabpezeshki.com

ZZakaria_87574_PTR_CH22_10-06-13_333-348.inddakaria_87574_PTR_CH22_10-06-13_333-348.indd 334334 66/19/2013/19/2013 4:47:094:47:09 PMPM ACUTE HEMATOLOGIC DISORDERS: Questions

5. A 48-year-old white female with a medical history of hypertension, polycystic kidney dis- ease, erosive gastritis, and meningioma was hospitalized for elective meningioma resec- tion. On postoperative day 1, she had two episodes of coffee-ground-colored vomiting. She has had intermittent episodes of passing black stools over the past week. She had an endoscopy a week before she was admitted, which showed erosive gastritis and mild refl ux esophagitis for which she is on omeprazole. Her serum creatinine was elevated to 2.9 mg/dL, and coagulation studies were: Activated partial thromboplastin time (PTT; aPTT) 34 seconds (25–40 seconds) Prothrombin time (PT) 13 seconds (11–15 seconds) Bleeding time 14 minutes (N 2–7 minutes) Platelet count 250,000/mm3 Which of the following is the best next step in the management of this patient? A. Observation and supportive treatment B. IV desmopressin C. Platelets transfusion D. Cryoprecipitate infusion E. Immediate surgery Questions 6–8 are related to the following vignette: A 62-year-old woman with a medical history of hypertension and non-compliance to medications presented with left-sided weakness and headache. She was diagnosed with a right basal ganglia and internal capsule hemorrhage without any evidence of intraven- tricular hemorrhage. On hospital day 3, she developed sudden onset of shortness of breath and chest pain, and pulse oximetry showed oxygen saturation of 93% on 4 L of oxygen. Ventilation:perfusion (V/Q) scan of the chest revealed a high probability of pulmonary embolism. Her baseline labs revealed the following: Hemoglobin 13.4 g/dL Hematocrit 39% Platelet count 310,000/mm3 Leukocyte count 7,900/mm3 PT 14 seconds International Normalized Ratio (INR) 1.06 You start the patient on anticoagulation with unfractionated heparin infusion and warfa- rin. Her symptoms gradually resolve over the next 5 days. On day 6, she complains of pain and pallor in her left arm. Physical examination reveals a pale and tender distal left arm with diminished pulses. The patient’s labs at this time reveal: Hemoglobin 13.0 g/dL Hematocrit 38% Platelet count 48,000/mm3 Leukocyte count 8,400/mm3 PT 19 seconds INR 1.78 PTT 60 seconds

335

66485457-66485438 www.ketabpezeshki.com

ZZakaria_87574_PTR_CH22_10-06-13_333-348.inddakaria_87574_PTR_CH22_10-06-13_333-348.indd 335335 66/19/2013/19/2013 4:47:094:47:09 PMPM GENERAL CRITICAL CARE: PATHOLOGY, PATHOPHYSIOLOGY, AND THERAPY

6. Which one of the following is the most likely cause of the patient’s condition? A. Subtherapeutic anticoagulation B. Heparin-induced thrombocytopenia (HIT) C. Warfarin-induced skin necrosis D. Heparin-induced skin necrosis E. Warfarin-induced thrombocytopenia

7. Which of the following is the most appropriate next step in the management of this patient? A. Discontinue warfarin B. Discontinue unfractionated heparin C. Discontinue warfarin and unfractionated heparin D. Discontinue warfarin and unfractionated heparin; initiate low molecular weight hepa- rin (LMWH) E. Discontinue warfarin and unfractionated heparin; initiate argatroban

8. Which of the following would have been the most useful strategy in preventing the patient’s condition? A. Monitor prothrombin time (PT) frequently B. Monitor platelet count frequently C. Substitute low molecular weight heparin (LMWH) for unfractionated heparin D. Initiate warfarin after 3 to 5 days of heparin E. Using lower doses of heparin early in the course of treatment

9. A 78-year-old white female with medical history of atrial fi brillation on warfarin presented to the ED with altered mental status and lethargy. Her initial vitals and blood glucose were within normal limits. EKG showed atrial fi brillation at 90 beats/minute. A CT scan of the head revealed a 5 cm × 3.8 cm left-sided intracerebral hemorrhage without intraven- tricular extension. Her initial laboratory results showed an INR of 4.2 and platelet count of 320,000/mm3. Which of the following is the best next step in the management of this patient? A. Hold Coumadin B. Hold Coumadin and control blood pressure (BP) C. Hold Coumadin, give vitamin K 10 mg D. Hold Coumadin and transfuse platelets E. Hold Coumadin, give vitamin K 10 mg, and transfuse fresh-frozen plasma (FFP)

336

66485457-66485438 www.ketabpezeshki.com

ZZakaria_87574_PTR_CH22_10-06-13_333-348.inddakaria_87574_PTR_CH22_10-06-13_333-348.indd 336336 66/19/2013/19/2013 4:47:094:47:09 PMPM ACUTE HEMATOLOGIC DISORDERS: Questions

10. A 62-year-old male with a medical history of hypertension and noncompliance to medi- cation presented to the ED with double vision and diffi culty in maintaining his balance for 2 days. In the ED, his BP was 220/128 mmHg. A CT scan of the head showed a large cerebellar hemorrhage with effacement of the fourth ventricle. Neurosurgery was con- sulted and the patient underwent a suboccipital decompression. His hospital course was complicated by fever and elevated white blood cell count with positive urine culture for which he was started on antibiotics empirically. On hospital day 5, the patient became tachycardic, hypotensive, and febrile. His laboratory results were as follows: Hemoglobin 11.2 g/dL Hematocrit 34% Platelet count 43,000/cm Leukocyte count 15,000/cm with predominant neutrophils Activated partial thromboplastin time (aPTT) 72 seconds (25–40 seconds) Prothrombin time (PT) 26 seconds (11–15 seconds) The patient was on subcutaneous heparin for deep vein thrombosis prophylaxis, which was discontinued and a heparin-induced thrombocytopenia (HIT) panel was sent. Additional labs showed: D-dimer 6 mg/dL (normal value is less than 0.5 mg/L) Fibrinogen 0.6 g/L (normal range is 1.5–2.77 g/L); peripheral blood smear showed few schistocytes.

What is the diagnosis? A. Disseminated intravascular coagulation (DIC) B. Thrombotic thrombocytopenic purpura (TTP) C. Antiplatelet antibodies D. Septicemia E. Drug-induced thrombocytopenia

11. A 19-year-old otherwise healthy male was involved in a motor vehicle collision. He was emergently taken to the OR for evacuation of a large subdural hematoma and was trans- fused 5 units of packed red blood cells (RBCs) during the case. Postoperatively, the patient continued to have oozing from the incision site. His laboratory studies postoperatively showed a platelet count of 280,000/mm3, prothrombin time (PT) of 11.8 seconds, and acti- vated partial thromboplastin time (aPTT) of 48 seconds. Which one of the following is the most likely etiology of his bleeding? A. Hypothermia B. Von Willebrand disease C. Disseminated intravascular coagulation D. Sepsis E. Citrate toxicity from packed RBC transfusion

337

66485457-66485438 www.ketabpezeshki.com

ZZakaria_87574_PTR_CH22_10-06-13_333-348.inddakaria_87574_PTR_CH22_10-06-13_333-348.indd 337337 66/19/2013/19/2013 4:47:094:47:09 PMPM GENERAL CRITICAL CARE: PATHOLOGY, PATHOPHYSIOLOGY, AND THERAPY

12. A 52-year-old male with a history of congestive heart failure, diabetes, renal insuffi ciency, and hypertension developed intracranial hemorrhage after receiving thrombolytic ther- apy for a cerebrovascular accident (CVA). What is the most appropriate treatment? A. Protamine sulfate B. Fresh-frozen plasma (FFP) C. Cryoprecipitate D. Vitamin K E. Dialysis

13. You were called to evaluate a patient on a heparin infusion, being bridged to Coumadin, who is complaining of a severe headache. The patient was receiving 1,200 U/hour of heparin, and the most current partial thromboplastin time (PTT) and International Normalized Ratio (INR) were 68.7 and 2.04, respectively. Heparin infusion was discontin- ued and a stat CT head scan was obtained that showed a 2cc3 intracerebral hemorrhage in the left basal ganglia without any evidence of intraventricular hemorrhage (IVH). Repeat PTT was 110. Which of the following is the best next step in management? A. Control BP, monitor closely, do stat complete blood count (CBC), and if required a blood transfusion B. Give platelets and fresh-frozen plasma (FFP) immediately and control BP C. Give protamine sulfate 30 mg to reverse the heparin D. Give protamine sulfate 10 mg to reverse the heparin E. Close observation 14. An 82-year-old right-handed female with a medical history of nonvalvular atrial fi brilla- tion and previous stroke around 2 years ago without any residual effects was found to be lethargic and less responsive at home by family members. She was brought to the ED and upon arrival her BP was 188/94 mmHg and heart rate was 122 bpm; she was minimally responsive, plegic on the right, and fl exing on the left. The patient was recently started on dabigatran for atrial fi brillation and took her last dose approximately 4 hours ago. There was no history of fall or head trauma. A stat CT of the head revealed an intracerebral hem- orrhage in the left caudate area along with intraventricular hemorrhage in the third and lateral ventricles with mild hydrocephalous. Laboratory data in the ED were: Hemoglobin 9.4 g/dL Hematocrit 29.0 Platelets 224 INR 1.9 PTT 94 Prothrombin time (PT) 36 What is the best next step in management? A. Activated charcoal, BP control, and repeat labs as these abnormal values can be due to lab errors B. Blood pressure control, fresh-frozen plasma (FFP), and platelet transfusion C. Airway protection, BP control, and stabilize the patient D. Airway protection, BP control, protamine sulfate, FFP, and platelet transfusion E. Airway protection, BP control, emergent dialysis, and prothrombin complex concen- trate (PCC) transfusion

338

66485457-66485438 www.ketabpezeshki.com

ZZakaria_87574_PTR_CH22_10-06-13_333-348.inddakaria_87574_PTR_CH22_10-06-13_333-348.indd 338338 66/19/2013/19/2013 4:47:094:47:09 PMPM 22

ANSWERS

1. The answer is A. Malignant glioma is most commonly associated with VTE, with a 2-year cumulative incidence of ~7.5%. Fifty-fi ve percent of cases occur within 2 months of sur- gery. Occurrence of VTE is associated with a 30% increase in mortality within 2 years. Risk factors for VTE in patients with malignant glioma are as follows: a. Older age (>45 years) b. Male sex c. One or more comorbid conditions d. Histologic subtype: glioblastoma multiforme e. Major neurosurgical procedure

2. The answer is E. The D-dimer test has a high sensitivity of approximately 95% but also high false positive rates and may guide the decision regarding duration of therapy. Doppler ultrasound of lower extremities is highly accurate with a positive-predictive value of 97% with the fi nding of noncompressible common femoral or popliteal veins and a negative- predictive value of 98% in an asymptomatic patient with full compressibility of both sites. CT pulmonary angiography (spiral CT) is the fi rst-line imaging for suspected pulmonary embolism (PE) workup (1–6).

3. The answer is C. The patient has isolated thrombocytopenic purpura with no cause iden- tifi ed based on history, physical examination, complete blood count (CBC), differential count, and blood smear. Based on this, a presumptive diagnosis of idiopathic thrombo- cytopenic purpura (ITP) can be made. ITP occurs as a result of platelet destruction by specifi c autoantibodies. Platelet transfusions are rarely required to maintain the platelet count. Circulating platelets are rapidly removed by the autoantibodies, making platelet transfusions futile. Emergent management of symptomatic and life-threatening throm- bocytopenia (i.e., massive intracranial hemorrhage, gastrointestinal hemorrhage, or

339

66485457-66485438 www.ketabpezeshki.com

ZZakaria_87574_PTR_CH22_10-06-13_333-348.inddakaria_87574_PTR_CH22_10-06-13_333-348.indd 339339 66/19/2013/19/2013 4:47:094:47:09 PMPM GENERAL CRITICAL CARE: PATHOLOGY, PATHOPHYSIOLOGY, AND THERAPY

emergency surgery) includes a combination of high-dose IV corticosterioids plus intra- venous immunoglobulin and platelet transfusion. Additional options are antifi brinolytics and emergency splenectomy (7–10).

4. The answer is C. The patient has thrombocytopenia with a platelet count of 10,000. Platelets play a critical role in the normal hemostatic mechanism; hence their defi - ciency can cause life-threatening hemorrhage. Platelet transfusions are commonly used to prevent or treat acute hemorrhage in patients with thrombocytopenia due to any cause. Posttransfusion platelet counts should be measured 10 to 60 minutes after the completion of transfusion. For an adult with body surface area of 2.0 m2, 1 U of platelet transfusion should immediately raise the platelet count by 5,000/µL. Since this patient received 6 U of platelets, his expected count should be at least 30,000 more than his baseline. If a smaller than expected rise in platelet count is noticed, consider refracto- riness to platelet transfusions. Refractoriness to platelet transfusion is defi ned as an absolute platelet count increment of less than or equal to 2,000/µL per unit of platelet transfusion given to an average-sized adult. Alloimmunization is one of the common and treatable causes, which results from the production of antibodies to human leuko- cyte antigens (HLA) class-1 on the transfused platelets. A platelet count with a normal increment at 1 hour after transfusion, but with a rapid return to baseline within 24 hours, indicates reduced platelet survival in the circulation. This is commonly seen in DIC, sepsis, or active bleeding (11–16).

5. The answer is B. A normal platelet count with an elevated bleeding time is suggestive of impaired platelet function. Platelet dysfunction is the major contributor of increased bleed- ing tendency in patients with renal failure. Platelet transfusion would be of no benefi t, because transfused platelets will be exposed to the same uremic environment. Indications for correction of platelet dysfunction are active bleeding and whether the patient is about to undergo a surgical procedure. Correction of platelet dysfunction can be done in several ways, including: a. Desmopressin (0.3 mcg/kg IV): Simplest and least toxic, desmopressin increases the release of factor 8 and von Willebrand factor (vWF) from the endothelium, which increases platelet adhesiveness. Improvement in platelet function is rapid, with onset of action within 1 hour, and effects lasting for 4 to 8 hours. b. Dialysis: May require considerable time to institute. c. Estrogen treatment: Improves platelet dysfunction, but the effect takes several days. d. Cryoprecipitate infusion: Limited to patients who did not respond to desmopressin and blood transfusion (11–16).

6. The answer is B. After receiving unfractionated heparin, the patient developed arterial thrombosis of the left arm in conjunction with heparin-induced thrombocytopenia (HIT), a well-known complication of heparin therapy. Two forms of HIT have been recognized, depending on the onset, clinical course, and severity: a. Type 1 HIT is seen within 1 to 4 days of initiation of heparin therapy, and it is associated with a lesser degree of decline in platelet numbers. The platelet count normalizes in a few days despite heparin continuation, and there are no clinical consequences. Type 1 HIT occurs in 10% to 20% of patients receiving unfractionated heparin.

340

66485457-66485438 www.ketabpezeshki.com

ZZakaria_87574_PTR_CH22_10-06-13_333-348.inddakaria_87574_PTR_CH22_10-06-13_333-348.indd 340340 66/19/2013/19/2013 4:47:094:47:09 PMPM ACUTE HEMATOLOGIC DISORDERS: Answers

b. Type 2 HIT is more serious and is associated with an immune-mediated disorder char- acterized by the formation of antibodies against heparin-platelet factor 4 complex. These antibodies bind to the platelet surface, causing platelet activation and aggrega- tion, leading to thrombocytopenia and platelet-rich clots. It typically develops 5 to 10 days after the initiation of heparin therapy. However, in patients with prior expo- sure to heparin, it may occur earlier. HIT may also occur after heparin is discontinued. Platelet counts can drop in the range of 30,000 to 60,000/µL. Spontaneous bleeding is unusual. Immune-mediated HIT is associated with venous and arterial thrombosis. The major manifestations of venous thrombosis are deep venous thrombosis, pulmo- nary embolism, venous limb gangrene, and cerebral venous sinus thrombosis. Arterial thrombosis can lead to strokes, myocardial infarction, and limb and organ ischemia. Warfarin-induced skin necrosis: a. Occurs within the fi rst few days of taking high doses of warfarin b. Cause: High dose of warfarin causes rapid reduction in protein C level, which is an anticoagulant, resulting in a transient hypercoaguable state c. Lesion location: Extremities, trunk, and breasts d. Not associated with thrombocytopenia Heparin-induced skin necrosis: a. Complication of unfractionated heparin use b. Lesion location: Areas rich in fat—abdomen, distal extremities c. Lesion: Area of erythema, which quickly progresses to purpura, hemorrhage, and necrosis d. Most of these patients do not develop thrombocytopenia (17–29).

7. The answer is E. The fi rst and most important intervention in a patient with suspected or doc- umented heparin-induced thrombocytopenia (HIT) is the immediate cessation of exposure to all heparin products. The patient with HIT still remains at risk for thrombosis, even after the discontinuation of heparin. All such patients need to be anticoagulated with direct thrombin inhibitors like argatroban or lepirudin. These should be used for prophylaxis and treatment of patients with HIT with or without thrombosis. The choice of agent depends on the co-exist- ing medical conditions. Lepirudin should be used with caution in patients with renal failure. Argatroban should be used with caution in patients with hepatic dysfunction (17–29).

8. The answer is C. The best way to prevent heparin-induced thrombocytopenia (HIT) is to use LMWH instead of unfractionated heparin whenever possible. LMWH is associated with a lower incidence of HIT compared to unfractionated heparin. Another option is to limit heparin use to less than 5 days to prevent an antibody response as well as the develop- ment of HIT. Development of HIT is independent of the dose used and has been reported to occur even with heparin fl ushes and the use of heparin-coated catheters (17–29).

9. The answer is E. This patient developed a spontaneous intracerebral hemorrhage second- ary to coagulopathy. Common causes of elevated INR (INR > 6): a. Diarrhea b. Progression of heart failure c. Fever d. Impaired liver function e. Vitamin K defi ciency

341

66485457-66485438 www.ketabpezeshki.com

ZZakaria_87574_PTR_CH22_10-06-13_333-348.inddakaria_87574_PTR_CH22_10-06-13_333-348.indd 341341 66/19/2013/19/2013 4:47:094:47:09 PMPM GENERAL CRITICAL CARE: PATHOLOGY, PATHOPHYSIOLOGY, AND THERAPY

Treatment of Elevated INR

INR Bleeding Recommendations

Therapeutic to 5 No Reduce warfarin dose or omit one dose and resume at lower dose when INR is therapeutic 5–9 No Omit one or two doses and resume at lower dose or omit one dose and give 1 to 2.5 mg of vitamin K if patient is at high risk for hemorrhage* >9 No Hold warfarin and give vitamin K 2.5 to 5 mg and resume at lower dose once INR is therapeutic Any Yes Hold warfarin, give vitamin K 10 mg and supplement with fresh-frozen plasma (FFP), prothrombin complex concentrate, or recombinant Factor 7a

*Patients at high risk for hemorrhage include patients with a history of hemorrhage and patients with a history of stroke, renal insuffi ciency, anemia, and hypertension

Treatment Options

Treatment Options Time to Anticoagulation Comments Reversal

Stop warfarin 5–14 days • None Vitamin K 6–24 hours • Factor 9, 10 replacement takes >24 hours • Risk of anaphylaxis with IV injection FFP 1–12 hours depending on • Large volume (around 2–4 L) may be thawing time, volume, and needed to normalize INR, depending on rate of infusion degree of coagulopathy Prothrombin 15 minutes after • Limited availability complex completion of infusion • Cost concentrate • Variable factor VII content based on manufacturer • Potentially prothrombotic Factor VIIa 15 minutes after bolus • Short half-life concentrate infusion • Cost • Uncertain safety • Replaces only factor VII, not other vitamin K–dependent factors

This table is based on 2008 American College of Chest Physician (ACCP) Guidelines. Pharmacology and management of the vitamin K antagonists: American College of Chest Physicians Evidence-Based Clinical Practice Guidelines (8th ed.). Ansell, Chest. 2008;133:160S.

Fresh-frozen plasma: Patients with life-threatening intracranial hemorrhage from elevated INR may be treated with vitamin K and FFP. Fresh-frozen plasma is prepared using apheresis from a single unit of whole blood or plasma, within 8 hours of collection (fresh) and stored at −18 to –30°C (frozen).

342

66485457-66485438 www.ketabpezeshki.com

ZZakaria_87574_PTR_CH22_10-06-13_333-348.inddakaria_87574_PTR_CH22_10-06-13_333-348.indd 342342 66/19/2013/19/2013 4:47:094:47:09 PMPM ACUTE HEMATOLOGIC DISORDERS: Answers

Dose: 10 to 15 mL/kg (which corresponds to 3–5 U of FFP) Volume: • One FFP unit is equivalent to 250 mL of volume • 3 to 5 U of FFP corresponds to around 0.75 to 1.25 L of colloid volume Cautions (because of large volume): • Patients with congestive heart failure • Elderly patients • Children Alternative: • Reduce the rate of infusion to around 1 mL/kg/hour • Consider cryoprecipitate Prothrombin complex concentrate: Contains coagulation factors VII, IX, X, prothrombin, and proteins C, S, and Z in a concentrated form, and, unlike FFP, can be given without waiting for compatibility testing and thawing. The potency of prothrombin complex concentrate (PCC) is expressed as factor IX content in IU, and varies between preparations. Dose: 50 IU/kg Volume: 50 to 150 mL depending on the preparation Caution: Potential to induce thrombosis and disseminated intravascular coagulation (DIC) Cryoprecipitate: Cryoprecipitate is the precipitate that remains after FFP undergoes a pro- cess of centrifugation. Dose: 1 U of cryoprecipitate/10 kg body weight Volume: 250 mL of FFP is equivalent to 10 to 20 mL of cryoprecipitate (30–43).

10. The answer is A. This patient has thrombocytopenia with an elevated PT, partial throm- boplastin time (PTT), fi brin degradation products (D-dimer), and reduced fi brinogen lev- els in the setting of sepsis (urinary tract infection [UTI] with tachycardia and low BP) suggesting the diagnosis of DIC. DIC is a complication of systemic disease causing acti- vation of clotting systems within blood vessels. This results in consumption of coagula- tion factors, which leads to thrombocytopenia and elevated PT and PTT. Eventually these small blood clots can cause multisystem organ failure and bleeding from multiple sites. Common causes of DIC include: A. Malignancy: lung, breast, and prostate B. Sepsis, mainly Gram-negative sepsis C. Massive tissue injury: trauma, burns Treatment: Treat the underlying cause! Patients can be treated symptomatically based on severity. If the patient has severe bleeding, platelets or cryoprecipitate can be transfused, but the underlying problem must be addressed at the same time (44, 45).

11. The answer is B. Von Willebrand disease (vWD) is the most common inherited coagulation disorder. It is divided into three major categories. Type I, a partial quantitative defi ciency, is the most common, and is due to decreased release or production of von Willebrand fac- tor (vWF); Type II is due to a qualitative defect of vWF; and Type III is a total defi ciency of vWF. Bleeding is usually noted only with trauma or surgery. Impaired platelet adhe- sion at the site of injury results in prolonged bleeding. Treatment includes desmopressin (DDAVP) if the patient has Type I von Willebrand disease, but factor VIII concentrate and

343

66485457-66485438 www.ketabpezeshki.com

ZZakaria_87574_PTR_CH22_10-06-13_333-348.inddakaria_87574_PTR_CH22_10-06-13_333-348.indd 343343 66/19/2013/19/2013 4:47:104:47:10 PMPM GENERAL CRITICAL CARE: PATHOLOGY, PATHOPHYSIOLOGY, AND THERAPY

cryoprecipitate are required for the other types. Citrate toxicity is unlikely because only 5 units of packed RBCs were transfused. Postoperative hypothermia can result in platelet dysfunction, but is unlikely to cause isolated prolonged thromboplastin time. Disseminated intravascular coagulation (DIC) will cause both the PT and the aPTT to be prolonged along with thrombocytopenia.

12. The answer is C. Prepare for administration of 8 to 10 units of cryoprecipitate, which contains factor VIII, von Willebrand factor (vWF), and fi brinogen. One unit contains ~250 mg of fi brinogen, and 6 units (1 pool) contains 2,100 mg. Each unit of FFP contains about 500 mg of fi brinogen; however, each unit of FFP is about 200 mL to 250 mL of volume compared to 15 mL/unit of cryoprecipitate. Platelet transfusion should also be considered. Protamine sulfate is the antidote for heparin toxicity.

13. The answer is C. The management of heparin overdose is as follows: A. For over-anticoagulation with intravenous unfractioned heparin (in the case of major bleeding): a. For IV unfractionated heparin (half-life is about 60–90 minutes), only heparin given in the past several hours needs reversal b. If heparin infusion has been stopped for only a few minutes, dose protamine 1 mg for every 100 units of heparin infused c. If heparin infusion was stopped 30 minutes prior, dose protamine 0.5 mg for every 100 units of heparin infused d. If heparin infusion was stopped >2 hours ago, dose protamine 0.25 to 0.375 mg for every 100 units of heparin infused B. For subcutaneous unfractionated heparin, prolonged protamine infusion may be needed: a. 1 to 1.5 mg of protamine for every 100 units of heparin administered b. Give 25 to 50 mg slowly intravenously followed by the remaining dose over 8 to 16 hours (this is the expected absorption time for subcutaneous heparin) C. For low molecular weight heparin (LMWH), the dose of protamine sulfate is 1 mg for every 100 units of LMWH dosed within the past 8 hours (46).

14. The answer is E. Dabigatran (Pradaxa) is a newer anticoagulant that works as a direct thrombin inhibitor. Peak plasma concentration is reached 1 to 2 hours after oral inges- tion. Anticoagulant effects decrease by 50% at 12 hours after administration. However, in patients with renal insuffi ciency, the half-life can be substantially longer. Currently, there is no reversal agent or antidote for dabigatran. The anticoagulant effect is through direct inhibition of clotting factor, and not because of clotting factor depletion, making replacement of clotting factors ineffective. As this medication is primarily excreted renally, the most effective way to remove this medication from the body is acute hemodialysis. Hemodialysis may be effective in removing approximately 60% of the drug (47).

References

1. Stein PD, Henry JW, Gottschalk A. Reassessment of pulmonary angiography for the diagnosis of pulmonary embolism: relation of interpreter agreement to the order of the involved pulmonary arterial branch. Radiology. 1999 Mar;210:689.

344

66485457-66485438 www.ketabpezeshki.com

ZZakaria_87574_PTR_CH22_10-06-13_333-348.inddakaria_87574_PTR_CH22_10-06-13_333-348.indd 344344 66/19/2013/19/2013 4:47:104:47:10 PMPM ACUTE HEMATOLOGIC DISORDERS: Answers

2. Hofmann LV, Lee DS, Gupta A, et al. Safety and hemodynamic effects of pulmonary angiography in patients with pulmonary hypertension: 10-year single-center experience. Am J Roentgenol. 2004 Sep;183:779. 3. Hudson ER, Smith TP, McDermott VG, et al. Pulmonary angiography performed with iopamidol: complications in 1,434 patients. Radiology. 1996 Jan; 198(1):61. 4. Stein PD, Athanasoulis C, Alavi A, et al. Complications and validity of pulmonary angiog- raphy in acute pulmonary embolism. Circulation. 1992 Feb;85(2):462. 5. Kuiper JW, Geleijns J, Matheijssen NA, et al. Radiation exposure of multi-row detector spiral computed tomography of the pulmonary arteries: comparison with digital subtrac- tion pulmonary angiography. Eur Radiol. 2003 Jul;13(7):1496. 6. Resten A, Mausoleo F, Valero M, et al. Comparison of doses for pulmonary embolism detection with helical CT and pulmonary angiography. Eur Radiol. 2003 Jul;13(7):1515 7. Michel M, Lee K, Piette JC, et al. Platelet autoantibodies and lupus-associated thrombo- cytopenia. Br J Haematol. 2002 Nov;119(2):354. 8. Kuwana M, Kaburaki J, Okazaki Y, et al. Two types of autoantibody-mediated thrombo- cytopenia in patients with systemic lupus erythematosus. Rheumatology (Oxford). 2006 Jul;45(7):851–854. 9. Nugent D, McMillan R, Nichol JL, et al. Pathogenesis of chronic immune thrombocyto- penia: increased platelet destruction and/or decreased platelet production. Br J Haematol. 2009 Sep;146(6):585–596. 10. Toltl LJ, Arnold DM. Pathophysiology and management of chronic immune thrombocy- topenia: focusing on what matters. Br J Haematol. 2011 Jan;152(1):52–60. 11. DiMichele DM, Hathaway WE. Use of DDAVP in inherited and acquired platelet dys- function. Am J Hematol. 1990 Jan;33(1):39. 12. Cattaneo M. Desmopressin in the treatment of patients with defects of platelet function. Haematologica. 2002 Nov;87(11):1122–1124. 13. Kosch A, Kehrel B, Nowak-Göttl U, et al. Thrombocytic alpha-delta-storage-pool-disease: shortening of bleeding time after infusion of 1-desamino-8-d-arginine vasopressin. Klin Padiatr. 1999 Jul-Aug; 211(4):198–200. 14. Piot B, Sigaud-Fiks M, Huet P, et al. Management of dental extractions in patients with bleed- ing disorders. Oral Surg Oral Med Oral Pathol Oral Radiol Endod. 2002 Mar;93(3):247–250. 15. Burroughs AK, Matthews K, Qadiri M, et al. Desmopressin and bleeding time in patients with cirrhosis. Br Med J (Clin Res Ed). 1985 Nov; 291(6506):1377–1381. 16. Mannucci PM, Vicente V, Vianello L, et al. Controlled trial of desmopressin in liver cir- rhosis and other conditions associated with a prolonged bleeding time. Blood. 1986 Apr; 67(4):1148–1153. 17. Linkins LA, Dans AL, Moores LK, et al. Treatment and prevention of heparin-induced thrombocytopenia: Antithrombotic Therapy and Prevention of Thrombosis (9th edition): American College of Chest Physicians Evidence-Based Clinical Practice Guidelines. Chest. 2012 Feb; 141(2 Suppl):e495S–e530S. 18. Warkentin TE, Greinacher A, Koster A, et al. Treatment and prevention of heparin-in- duced thrombocytopenia: American College of Chest Physicians Evidence-Based Clinical Practice Guidelines (8th edition). Chest. 2008 Jun;133 (6 Suppl):340S–380S. 19. Chong BH, Castaldi PA. Platelet proaggregating effect of heparin: possible mecha- nism for non-immune heparin-associated thrombocytopenia. Aust N Z J Med. 1986 Oct;16(5):715–716.

345

66485457-66485438 www.ketabpezeshki.com

ZZakaria_87574_PTR_CH22_10-06-13_333-348.inddakaria_87574_PTR_CH22_10-06-13_333-348.indd 345345 66/19/2013/19/2013 4:47:104:47:10 PMPM GENERAL CRITICAL CARE: PATHOLOGY, PATHOPHYSIOLOGY, AND THERAPY

20. Greinacher A. Antigen generation in heparin-associated thrombocytopenia: the non-im- munologic type and the immunologic type are closely linked in their pathogenesis. Semin Thromb Hemost. 1995; 21(1):106–116. 21. Stanton PE Jr, Evans JR, Lefemine AA, et al. White clot syndrome. South Med J. 1988 May; 81(5):616–620. 22. Greinacher A, Warkentin TE. Recognition, treatment, and prevention of heparin-induced thrombocytopenia: review and update. Thromb Res 2006; 118(2):165–176. 23. Keeling D, Davidson S, Watson H. Haemostasis and Thrombosis Task Force of the British Committee for Standards in Haematology. The management of heparin-induced throm- bocytopenia. Br J Haematol. 2006 May;133(3):259–269. 24. Napolitano LM, Warkentin TE, Almahameed A, Nasraway SA. Heparin-induced throm- bocytopenia in the critical care setting: diagnosis and management. Crit Care Med. 2006 Dec; 34(12):2898–2911. 25. Kelton JG, Warkentin TE. Heparin-induced thrombocytopenia: a historical perspective. Blood. 2008 Oct 1;112(7):2607–2616. 26. Shantsila E, Lip GY, Chong BH. Heparin-induced thrombocytopenia. A contemporary clinical approach to diagnosis and management. Chest. 2009 Jun; 135(6):1651–1664. 27. Martel N, Lee J, Wells PS. Risk for heparin-induced thrombocytopenia with unfraction- ated and low-molecular-weight heparin thromboprophylaxis: a meta-analysis. Blood. 2005 Oct; 106(8):2710–2715. 28. Warkentin TE, Levine MN, Hirsh J, et al. Heparin-induced thrombocytopenia in patients treated with low-molecular-weight heparin or unfractionated heparin. N Engl J Med. 1995 May 18; 332(20):1330–1335. 29. Harbrecht U, Bastians B, Kredteck A, et al. Heparin-induced thrombocytopenia in neuro- logic disease treated with unfractionated heparin. Neurology. 2004 Feb 24; 62(4):657–9. 30. Ansell J, Hirsh J, Hylek E, et al. Pharmacology and management of the vitamin K antago- nists: American College of Chest Physicians Evidence-Based Clinical Practice Guidelines (8th Edition). Chest. 2008 Jun; 133(6 Suppl):160S–198S. 31. Watson HG, Baglin T, Laidlaw SL, et al. A comparison of the effi cacy and rate of response to oral and intravenous Vitamin K in reversal of over-anticoagulation with warfarin. Br J Haematol. 2001 Oct; 115(1):145–149. 32. Dentali F, Ageno W, Crowther M. Treatment of Coumadin-associated coagulopathy: a systematic review and proposed treatment algorithms. J Thromb Haemost. 2006 Sep; 4(9):1853–1863. 33. Diug B, Evans S, Lowthian J, et al. The unrecognized psychosocial factors contributing to bleeding risk in warfarin therapy. Stroke. 2011; 42:2866. 34. Penning-van Beest FJ, van Meegen E, Rosendaal FR, et al. Characteristics of anticoagu- lant therapy and comorbidity related to overanticoagulation. Thromb Haemost. 2001 Aug; 86(2):569–574. 35. Visser LE, Bleumink GS, Trienekens PH, et al. The risk of overanticoagulation in patients with heart failure on Coumadin anticoagulants. Br J Haematol. 2004 Oct;127(1):85–89. 36. Cushman M, Booth SL, Possidente CJ, et al. The association of vitamin K status with Warfarin sensitivity at the onset of treatment. Br J Haematol. 2001 Mar;112(3):572–577. 37. Cavallaro R, Iovino P, Castiglione F, et al. Prevalence and clinical associations of pro- longed prothrombin time in adult untreated coeliac disease. Eur J Gastroenterol Hepatol. 2004 Feb; 16(2):219–223.

346

66485457-66485438 www.ketabpezeshki.com

ZZakaria_87574_PTR_CH22_10-06-13_333-348.inddakaria_87574_PTR_CH22_10-06-13_333-348.indd 346346 66/19/2013/19/2013 4:47:104:47:10 PMPM ACUTE HEMATOLOGIC DISORDERS: Answers

38. Garcia D, Crowther MA, Ageno W. Practical management of coagulopathy associated with warfarin. BMJ. 2010 Apr 19;340:c1813 39. Banet GA, Waterman AD, Milligan PE, et al. Warfarin dose reduction vs watchful waiting for mild elevations in the international normalized ratio. Chest. 2003 Feb; 123(2):499–503. 40. Garcia DA, Regan S, Crowther M, Hylek EM. The risk of hemorrhage among patients with Warfarin-associated coagulopathy. J Am Coll Cardiol. 2006 Feb 21;47(4):804–808. 41. Crowther MA, Ageno W, Garcia D, et al. Oral vitamin K versus placebo to correct exces- sive anticoagulation in patients receiving warfarin: a randomized trial. Ann Intern Med. 2009 Mar 3;150(5):293–300. 42. Veeger NJ, Piersma-Wichers M, Meijer K, Hillege HL. Minor bleeds alert for subse- quent major bleeding in patients using vitamin K antagonists. Br J Haematol. 2011 May;153(4):508–514. 43. Kucher N, Connolly S, Beckman JA, et al. International normalized ratio increase before warfarin-associated hemorrhage: brief and subtle. Arch Intern Med. 2004 Oct 25;164(19):2176–2179. 44. Carr JM, McKinney M, McDonagh J. Diagnosis of disseminated intravascular coagula- tion. Role of D-dimer. Am J Clin Pathol. 1989 Mar;91(3):280–287. 45. Spero JA, Lewis JH, Hasiba U. Disseminated intravascular coagulation. Findings in 346 patients. Thromb Haemost. 1980 Feb 29;43(1):28–33 46. Garcia DA, Baglin TP, Weitz JI, et al. Parenteral anticoagulants: Antithrombotic Therapy and Prevention of Thrombosis (9th edition): American College of Chest Physicians Evidence-Based Clinical Practice Guidelines. Chest. 2012 Feb;141(2 Suppl):e24S–e43S 47. van Ryn J, Stangier J, Haertter S, et al. Dabigatran etexilate—a novel, reversible, oral direct thrombin inhibitor: interpretation of coagulation assays and reversal of anticoagu- lant activity. Thromb Haemost. 2010 Jun;103(6):1116–1127.

347

66485457-66485438 www.ketabpezeshki.com

ZZakaria_87574_PTR_CH22_10-06-13_333-348.inddakaria_87574_PTR_CH22_10-06-13_333-348.indd 347347 66/19/2013/19/2013 4:47:104:47:10 PMPM 66485457-66485438 www.ketabpezeshki.com

ZZakaria_87574_PTR_CH22_10-06-13_333-348.inddakaria_87574_PTR_CH22_10-06-13_333-348.indd 348348 66/19/2013/19/2013 4:47:104:47:10 PMPM 23 Acute Gastrointestinal and Genitourinary Disorders Luis J. Garcia and Asad Latif QUESTIONS

1. A 65-year-old woman with a known history of duodenal ulcer is admitted to the ED with a major upper gastrointestinal (UGI) hemorrhage. Her home treatment regimen consists of diet modifi cation and acid-suppressive therapy. After appropriate resuscitation, which of the following is the most appropriate next step? A. Begin bismuth, tetracycline, and metronidazole B. Start a pantoprazole infusion C. Endoscopy with coagulation of the bleeding vessel D. Immediate laparotomy and oversewing of the bleeding vessel

ANSWERS TO THIS SECTION CAN BE FOUND ON PAGE 355 349

66485457-66485438 www.ketabpezeshki.com

ZZakaria_87574_PTR_CH23_10-06-13_349-360.inddakaria_87574_PTR_CH23_10-06-13_349-360.indd 349349 66/19/2013/19/2013 8:48:458:48:45 PMPM GENERAL CRITICAL CARE: PATHOLOGY, PATHOPHYSIOLOGY, AND THERAPY

2. A 34-year-old man is brought to the ED by his friends early in the morning, complaining of hematemesis. He admits that the night before he had been drinking heavily and had vomited multiple times. On physical examination, his clothes are stained with bloody emesis, he is complaining of abdominal pain, and other than tachycardia his vital signs are normal. The most appropriate response regarding his primary complaint is: A. It is a complication of gastroesophageal (GE) refl ux B. It involves esophageal rupture near the GE junction C. Profuse hemorrhage is the most common manifestation D. Bleeding can generally be managed medically E. Vagotomy is indicated for patients requiring surgery

3. A 54-year-old female with a 20-year history of substantial alcohol abuse presents to the ED with fevers and hypotension. She has a history of ascites and had a large-volume paracen- tesis several months ago. She is diagnosed with spontaneous bacterial peritonitis (SBP), admitted to the ICU, and treated with appropriate antibiotics. During her hospitaliza- tion, she also undergoes upper endoscopy and is found to have several large esophageal varices. Which of the following decreases the risk of an initial upper gastrointestinal (UGI) hemorrhage from varices in this patient? A. Transjugular intrahepatic porto-systemic shunt B. Sclerotherapy C. Splenorenal shunt D. Beta blockade

4. A 55-year-old male construction worker fell off a 10-foot scaffolding 2 days ago. At the time he refused medical care, but has been having signifi cant back and chest wall pain since the fall. He has been medicating himself with acetaminophen. He presents to the ED this morning with an acute onset of fever, right-sided abdominal aching, and jaundice. His family states that he has also been more confused during the past 24 hours. He admits to taking more than 5 g of Tylenol during that time. His examination is signifi cant for right upper quadrant (RUQ) tenderness. His labs reveal elevated transaminases, an INR of 2.5, and a venous pH of 7.38. All of the following are components of the King’s College criteria, except: A. INR B. Serum creatinine C. Arterial pH D. Serum transaminases E. Hepatic encephalopathy

5. For the patient in Question 4, which of the following would be an appropriate next step in his treatment? A. Gastric lavage B. List for liver transplantation C. Administration of acetylcysteine D. Fluid resuscitation and stabilization E. Monitor for 2 hours and follow up in clinic

350

66485457-66485438 www.ketabpezeshki.com

ZZakaria_87574_PTR_CH23_10-06-13_349-360.inddakaria_87574_PTR_CH23_10-06-13_349-360.indd 350350 66/19/2013/19/2013 8:48:458:48:45 PMPM ACUTE GASTROINTESTINAL AND GENITOURINARY DISORDERS: Questions

6. An 88-year-old paraplegic female patient presents to the ED with new onset of abdominal pain and distention. She was at the nursing home in her usual state when these symptoms began. On examination she is afebrile and her abdomen is distended but not tender. She has no obvious previous surgical scars, and upon questioning states that she has been pass- ing scant watery stools. Abdominal x-ray shows dilated loops of large and small bowel. Her diagnosis could be associated with all of the following, except: A. Malignant infi ltration of the celiac plexus B. Neuroleptic medications C. Opiate usage D. Excess parasympathetic tone E. Scleroderma

7. Which of the following statements is true regarding small bowel obstruction (SBO)? A. Gas within the small bowel is distinguished from gas within the colon by luminal lines perpendicular to the bowel wall. The small bowel lines partially cross the lumen, whereas the colonic lines completely cross the lumen. B. Ileus may be diffi cult to distinguish from SBO, since both conditions can produce gas- eous distention of the bowel with air fl uid levels C. The “string of pearls” sign refers to a series of radiolucent images in the small bowel representing the gallstones of gallstone ileus D. A gasless abdomen seen on plain fi lm rules out a SBO

8. A 66-year-old male with a known history of sigmoid diverticulosis presents to the ED with an acute onset of fever, chills, and severe left lower quadrant pain. On examination, the patient has a temperature of 39°C, HR 120, and a BP of 90/40. He demonstrates peritoneal signs on abdominal examination. A CT scan is performed, which demonstrates evidence of pneumo-peritoneum and mesenteric stranding around the sigmoid colon. Which of the following is the correct treatment for this patient? A. ICU admission, fl uid resuscitation, IV antibiotics within 30 minutes, and laparotomy B. ICU admission, fl uid resuscitation, IV antibiotics within 30 minutes, and percutaneous drainage C. Admission to the surgical fl oor, fl uid resuscitation, and IV antibiotics on call to the OR for laparotomy D. Admission to the surgical fl oor and wait for OR availability E. Directly to OR for emergent laparotomy

9. A 37-year-old investment banker presents to the ED with an acute onset of abdominal pain. On history, he admits to a high-stress work environment, frequently taking antacids. He denies taking any nonsteroidal anti-infl ammatory drugs (NSAIDs). On examination, the patient does not appear toxic, but has signifi cant epigastric tenderness without obvious peri- tonitis. A plain abdominal radiograph demonstrates free air. A follow-up CT scan confi rms this fi nding along with a signifi cant amount of fat stranding surrounding the fi rst portion of the duodenum. Which of the following is not associated with the patient’s condition? A. Excessive NSAID use B. Acid hypersecretory state C. Helicobacter pylori infection D. Need for operative intervention

351

66485457-66485438 www.ketabpezeshki.com

ZZakaria_87574_PTR_CH23_10-06-13_349-360.inddakaria_87574_PTR_CH23_10-06-13_349-360.indd 351351 66/19/2013/19/2013 8:48:458:48:45 PMPM GENERAL CRITICAL CARE: PATHOLOGY, PATHOPHYSIOLOGY, AND THERAPY

10. Which of the following statements accurately characterizes acute occlusion of the supe- rior mesenteric artery (SMA)? A. The right and left colon are generally spared as a result of sparing of the middle colic artery B. Emboli most commonly arise from atheromatous plaques within the aorta C. Acute occlusion of the SMA usually results in complete foregut infarction D. Sudden complete occlusion is most often caused by embolism rather than by thrombosis

11. Ms Jones is a 76-year-old female who was incidentally found to have a 7-cm infrarenal aortic aneurysm. She elected to have open repair of her aneurysm and underwent the surgery without any complications. Regarding her intraoperative course, her vascular surgeon was able to place the aortic cross clamp below the renal arteries, and did not reimplant any vessels. On her fi rst postoperative day, the patient complains of severe abdominal pain and has an episode of bloody diarrhea. Which of the following is true regarding her probable condition? A. The most common symptoms of ischemic colitis are lower abdominal pain and bleeding B. Occlusion of major mesenteric vessels is responsible for producing ischemia in most cases C. Nonoperative management is not justifi ed, because, in a signifi cant percentage of such cases, perforation and peritonitis eventually develop D. The cecum and rectum are the most commonly involved areas E. A diagnosis is best made with an abdominal x-ray

12. An 80-year-old woman is seen in the ED with abdominal pain and obstipation. On exami- nation, she is afebrile and mildly tachycardic. Her abdomen is distended and tympanic, but there are no peritoneal signs. Abdominal x-ray shows a signifi cantly distended loop of bowel pointing to the right upper quadrant (RUQ). The fi rst step in her management should be: A. Administration of laxatives and cleansing enemas B. Barium enema C. Rigid sigmoidoscopy D. Sigmoid resection

13. A 54-year-old male presents to the ED with a 1-day history of cramping abdominal pain, multiple episodes of emesis, and inability to pass fl atus or have a bowel movement in 48 hours. The patient is otherwise healthy, but has a history of a previous open appendec- tomy at the age of 23. On examination he is afebrile with mild tachycardia. His abdominal exam is tender, distended, but without evidence of peritonitis. Which of the following is the best examination for his diagnosis? A. Abdominal radiographs B. Upper gastrointestinal (UGI) series with small bowel follow-through C. CT scan without contrast D. MRI E. Barium enema

352

66485457-66485438 www.ketabpezeshki.com

ZZakaria_87574_PTR_CH23_10-06-13_349-360.inddakaria_87574_PTR_CH23_10-06-13_349-360.indd 352352 66/19/2013/19/2013 8:48:468:48:46 PMPM ACUTE GASTROINTESTINAL AND GENITOURINARY DISORDERS: Questions

14. A 66-year-old male presents to the ED with a 3-hour history of abdominal pain, nausea, and vomiting. The patient states that the onset of pain was sudden and is radiating to the back. On examination, he is tachycardic and has tenderness to palpation in the epigas- trium. Hyperamylasemia is diagnostic of acute pancreatitis when associated with which of the following laboratory fi ndings? A. Hyperlipasemia B. Increased urinary amylase levels C. Hypocalcemia D. None of the above

15. Routine initial management of a patient with alcoholic pancreatitis and fi ve Ranson’s cri- teria should include which of the following measures? A. IV resuscitation, admission to the fl oor, and clear liquid diet B. Admission to an ICU, aggressive fl uid resuscitation, and gastric decompression C. Peritoneal lavage D. Octreotide E. Admission to the hospital for scheduled surgical exploration

16. An alcoholic patient has acute pancreatitis with fi ve of Ranson’s criteria. He gradually improves over the course of a couple of weeks but then develops tachycardia with a heart rate (HR) of 120 and a temperature of 39°C, along with abdominal distention. A CT scan is obtained that demonstrates a large amount of air within the pancreas. After initial sta- bilization in the ICU, which of the following is the most appropriate next step? A. Percutaneous catheter drainage B. Peritoneal lavage C. Laparoscopy D. Operative drainage

17. A 65-year-old man is unable to void after an abdominoperineal resection. Postvoid resid- uals have been 600 to 800 mL. The treatment of choice in this patient is: A. Chronic Foley catheterization B. Transurethral prostatectomy C. Clean intermittent catheterization D. Transurethral sphincterotomy E. Alpha-blockers alone

18. A 74-year-old male with a signifi cant history of tobacco and alcohol abuse presents to the ED with the chief complaint of “blood in my urine.” He denies fevers, abdominal pain, or nausea. His examination is essentially unremarkable. The evaluation of gross or micro- scopic hematuria involves all the following except: A. Upper tract imaging with intravenous pyelogram (IVP), ultrasound imaging, or CT scanning B. Cystoscopic studies C. Serum tumor markers D. Pelvic examination E. Repeat urinalysis

353

66485457-66485438 www.ketabpezeshki.com

ZZakaria_87574_PTR_CH23_10-06-13_349-360.inddakaria_87574_PTR_CH23_10-06-13_349-360.indd 353353 66/19/2013/19/2013 8:48:468:48:46 PMPM GENERAL CRITICAL CARE: PATHOLOGY, PATHOPHYSIOLOGY, AND THERAPY

19. A 27-year-old, otherwise healthy male is brought to the ED as a trauma activation. He is initially hemodynamically unstable and has a positive focused assessment with sonog- raphy in trauma (FAST) exam for blood in the abdomen. He is taken to the OR, where a splenic avulsion is diagnosed and a splenectomy is performed. The anesthesiologist informs the surgeon that there is gross blood in the Foley bag. Which of the following is most likely true about the patient’s condition? A. Rupture is usually extra peritoneal when associated with pelvic trauma B. A single-view retrograde cystogram in the ED demonstrates most signifi cant bladder injuries C. Primary closure is generally indicated for extra peritoneal ruptures D. Intraoperative injury usually requires repair with a suprapubic cystostomy

354

66485457-66485438 www.ketabpezeshki.com

ZZakaria_87574_PTR_CH23_10-06-13_349-360.inddakaria_87574_PTR_CH23_10-06-13_349-360.indd 354354 66/19/2013/19/2013 8:48:468:48:46 PMPM 23

ANSWERS

1. The answer is C. In the presence of an acute hemorrhage, none of the listed drug regimens provide immediate control of the bleeding. Answer A would be a treatment option for patients with H. pylori infection. While H. pylori infection is commonly found in patients with duodenal ulcers, treatment does not address the acute hemorrhage. Pantoprazole infusions may be of benefi t in patients with bleeding gastric ulcers; however, there is no convincing evidence that they are benefi cial in patients with duodenal ulcer hemorrhage. Therapy in this patient, after appropriate resuscitation, is endoscopic visualization and control of the bleeding vessel. If endoscopy fails to control the bleeding, or if the bleeding recurs, then immediate laparotomy with oversewing of the bleeding vessel is often neces- sary. After control of the bleeding, the patient should be tested for H. pylori and continued on antacid therapy.

2. The answer is D. Mallory–Weiss syndrome refers to a tear of the mucosa and submucosa near the GE junction that occurs as a result of retching. The tear is usually on the gastric side of the GE junction and on the lesser curvature. In only about 10% of cases is there profuse hemorrhage. Bleeding often stops spontaneously and can often be managed non- operatively. If not, an attempt at endoscopic clipping can be performed. The defi nitive treatment is laparotomy with simple oversewing of the bleeding mucosal tear.

3. The answer is D. Esophageal varices are the most common cause of massive bleeding in a patient with cirrhosis and typically result from the shunting of blood from the mesenteric circulation through the coronary vein to the esophageal submucosal venous plexus. When the pressure in the varices rises above 12 mmHg, spontaneous rupture will occur in 30% of patients. In addition to variceal hypertension, mucosal ulceration can also predispose to rupture. Beta blockade is the most effective way to prevent the fi rst bleeding episode. Sclerotherapy, transjugular intrahepatic portosystemic shunt, and surgical shunts have

355

66485457-66485438 www.ketabpezeshki.com

ZZakaria_87574_PTR_CH23_10-06-13_349-360.inddakaria_87574_PTR_CH23_10-06-13_349-360.indd 355355 66/19/2013/19/2013 8:48:468:48:46 PMPM GENERAL CRITICAL CARE: PATHOLOGY, PATHOPHYSIOLOGY, AND THERAPY

not been associated with a reduction in fi rst bleeding events in patients with alcoholic cir- rhosis. Prophylaxis is of utmost importance because variceal bleeding in cirrhotic patients carries a 1-year mortality rate of 70%. Child’s Class C patients with variceal bleeding have a 70% mortality rate at 6 weeks (1).

4. The answer is D. The King’s College criteria were fi rst developed in 1989, based on the observations of 588 patients with acute liver failure who presented to the King’s College Hospital between 1973 and 1985. These patients were retrospectively studied to deter- mine if there were certain clinical factors that predicted a worse prognosis. These criteria were further subdivided for acetaminophen and nonacetaminophen causes of acute liver failure. For acute liver failure as a result of acetaminophen toxicity, the criteria include an arterial pH < 7.3, INR > 6.5, serum creatinine > 300 µmol/L, and Grade III/IV encephal- opathy. The specifi city and sensitivity of these criteria are 90% and 69%, respectively. As a result, the American Society for the Study of Liver Disease has recommended that the criteria be used as an early parameter in ascertaining the need for liver transplantation in these patients.

5. The answer is C. Gastric lavage and activated charcoal have proven benefi t only if the ingestion was recent, more specifi cally within 30 minutes. Liver transplantation is an option in certain patients with acute liver failure; however, our patient does not meet any of the King’s College criteria; as a result his prognosis is fairly good and liver transplanta- tion would not be an option in him at this time. Acetylcysteine works by replenishing the body’s stores of glutathione, which then reacts with the toxic metabolites and allows for excretion. It is important to start the acetylcysteine within 8 hours of ingestion, because after 8 hours, irreversible liver damage has started to occur. The recommended adult dos- age regimen for the IV formulation is a loading dose of 150 mg/kg in 200 mL of 5% dextrose given over 15 to 30 minutes. This is followed by a maintenance dose at 50 mg/ kg in 500 mL of 5% dextrose given IV over 4 hours and then 100 mg/kg in 1,000 mL of 5% dextrose given IV over 16 hours. Adjustments are required for children and patients at risk for fl uid overload. Fluid overload can potentially cause hepatic congestion and worsen failure (2).

6. The answer is D. Ogilvie fi rst described a profound colonic ileus in the absence of bowel pathology. His patients had suffered from malignant infi ltration of the celiac plexus. Ogilvie’s syndrome most often appears as a complication of other clinical conditions. It is typically characterized by massive colonic distention in the absence of a mechanical obstruction. Neuroleptic medications, opiate usage, and severe metabolic disease have produced a similar clinical picture. The exact cause of colonic ileus has not been identi- fi ed; a presumed mechanism is excessive sympathetic tone, resulting in a loss of normal colonic peristalsis. It appears that the typical at-risk patient is elderly and bedridden. Associated medical conditions include hypothyroidism, diabetes, chronic renal failure, scleroderma, multiple sclerosis, and electrolyte disturbances. It may also be seen in the setting of severe trauma. Ogilvie’s syndrome is of concern due to the risk of colonic perforation, the cecum being the area of the colon most at risk, especially when the cecal diameter exceeds 12 cm. Patients typically present with a distended, tender abdomen. Unlike mechanical bowel obstructions, the pain is usually not colicky. Abdominal x-rays and CT scans can be used as adjuncts and typically demonstrate a diffusely dilated colon

356

66485457-66485438 www.ketabpezeshki.com

ZZakaria_87574_PTR_CH23_10-06-13_349-360.inddakaria_87574_PTR_CH23_10-06-13_349-360.indd 356356 66/19/2013/19/2013 8:48:468:48:46 PMPM ACUTE GASTROINTESTINAL AND GENITOURINARY DISORDERS: Answers

without evidence of a mechanical cause. Therapy aims at correcting electrolyte abnormal- ities and ruling out a mechanical cause of obstruction. Treatment of underlying medical disorders is mandatory. Colonic decompression can be accomplished with neostigmine, endoscopic decompression, or surgical decompression. Of note, patients receiving neo- stigmine should be kept in a monitored setting secondary to the risk of bradycardia. Typically, surgical therapy is reserved for refractory distention and/or complications such as colonic perforation (3).

7. The answer is B. Plain radiographs of the abdomen are useful in evaluating patients with suspected SBO. Gas-fi lled loops of small bowel are typically seen in the central portion of the abdomen. The presence of both normal and dilated small bowel loops is typical of SBO. The small bowel is recognized radiographically by the vavulae con- niventes that are visible as lines that completely cross the intestinal lumen. Colonic gas is typically located peripherally and the colon is identifi ed by lines that partially cross the colonic lumen (plicae semilunaris). Air–fl uid levels can be seen in both ileus and small bowel obstruction, and typically are seen only on upright or decubitus fi lms. A gasless abdomen can be seen in SBO and typically results from proximal decompres- sion by emesis or by completely fl uid-fi lled loops of small bowel. It is important to dis- tinguish between partial and complete SBO. In partial obstruction, gas is usually seen in decompressed loops of small bowel distal to the point of obstruction. In contrast, a complete obstruction presents with obstipation and a paucity of gas in the distal decom- pressed small bowel.

8. The answer is A. The patient in this scenario likely has a free perforation from acute diverticulitis. He is not a candidate for percutaneous therapy because he has peri- tonitis and a drainable collection is unlikely to form this early, nor was evident on the CT scan. This patient will require operative exploration in an expedited fashion. However, the fi rst priority is to start antibiotics (covering Gram-negative organisms and anaerobes) and resuscitate the patient with in vitro fertilisation (IVF). Since the patient is exhibiting signs of shock given his tachycardia and hypotension, the best location for this patient is in the ICU.

9. The answer is A. By history and by the CT scan results, this patient likely has a type II peptic ulcer. Type I ulcers are found in the lesser curve of the stomach, type II are duode- nal, type III are prepyloric, type IV are located on the lesser curve close to the GE junction, and type V are located anywhere in the stomach and are associated with NSAID use. Types II and III are classically associated with acid hypersecretion, while any of the types with the exception of type V are associated with H. pylori infection. Treatment priorities in this patient include IV antibiotics and operative closure of the perforation. Testing for H. pylori should also be done, and the patient should be treated if the test is positive.

10. The answer is D. Arterial emboli are the most common cause of acute occlusion of the SMA. These emboli most frequently arise from the heart, either from a mural throm- bus after myocardial infarction or from the atria in patients with atrial fi brillation. Paradoxical embolism through a patent foramen ovale may also be a cause of arte- rial occlusion. The initial abdominal pain is severe and often out of proportion to the physical fi ndings. The physical fi ndings of peritonitis imply transmural ischemia and

357

66485457-66485438 www.ketabpezeshki.com

ZZakaria_87574_PTR_CH23_10-06-13_349-360.inddakaria_87574_PTR_CH23_10-06-13_349-360.indd 357357 66/19/2013/19/2013 8:48:468:48:46 PMPM GENERAL CRITICAL CARE: PATHOLOGY, PATHOPHYSIOLOGY, AND THERAPY

thus represent a late stage in the evolution of the process. Acute occlusion of the SMA does not result in complete foregut infarction because the proximal jejunum is typically spared in the case of embolic disease.

11. The answer is A. Ischemic colitis should be considered in the differential diagnosis of any patient with lower abdominal pain and bright red blood per rectum, especially after an aortic aneurysm repair where the inferior mesenteric artery is usually sacrifi ced. It is most common in elderly patients, but can be found in patients of any age who also have polyarteritis nodosa, lupus, rheumatoid arthritis, scleroderma, or polycythemia vera. The severity of clinical presentation varies with the extent and duration of vascular occlu- sion, the adequacy of collateral circulation, and the extent of septic complications. Mild or moderate ischemia is compensated for by collateral circulation, and the mucosal slough- ing usually heals in 2 to 3 days. Transmural ischemia may lead to future stricture devel- opment and ultimately may progress to full-thickness gangrene leading to perforation and peritonitis. In ischemic colitis, the vascular insult appears to be confi ned to small arterioles, sparing the major colonic vessels. Although the ischemia can affect any part of the colon, it is more common in the splenic fl exure and distal sigmoid colon. These areas have been traditionally referred to as “watershed” areas that rely on the marginal artery of Drummond for perfusion. Diagnosis is typically made via endoscopy, which usually reveals pale/edematous mucosa that may be covered by an exudative membrane. Barium enema is also often used for diagnosis, demonstrating classic thumb printing of the bowel wall. If perforation is suspected, these diagnostic modalities are contraindicated, and urgent laparotomy is the diagnostic and therapeutic modality of choice. Management is usually nonoperative, as the ischemia typically improves over several days with appro- priate hydration and medical comorbidity optimization. Surgical therapy remains the mainstay of therapy for peritonitis and stricture development.

12. The answer is C. This patient has sigmoid volvulus. Although not an answer choice, the fi rst and foremost management principle is to ensure that the patient is nontoxic; that is, there is no peritonitis or shock. Maintenance of end-organ perfusion should be ensured with urine output monitoring and fl uid resuscitation as needed. For sigmoid volvulus without perforation, there is no need for the routine administration of antibiotics. Passing a sigmoidoscope past the point of obstruction allows immediate decompression of the dilated and gas-fi lled colonic segment. The likelihood of recurrence after sigmoidoscopic decompression is very high, and an elective sigmoid resection should be planned in the medically fi t patient. Laxatives, cleansing enemas, and barium enemas are dangerous and can lead to colonic perforation. There is no role for decompressive transverse colostomy.

13. The answer is C. Given the answer choices, CT scanning would be the best imaging modality. CT scanning can distinguish between partial and complete bowel obstructions and give additional information regarding the etiology of the obstruction. In the clinical scenario presented in the question, the most likely etiology is adhesive disease, although other causes are certainly possible. Addition of oral contrast, although helpful in identify- ing the transition point in cases of low-grade obstructions, is not mandatory. Many radiol- ogy protocols in tertiary care centers have abandoned the routine use of oral contrast for the diagnosis of bowel obstruction. It is important to appreciate that high-grade bowel obstructions can be associated with large fl uid losses, both from vomiting and secondary

358

66485457-66485438 www.ketabpezeshki.com

ZZakaria_87574_PTR_CH23_10-06-13_349-360.inddakaria_87574_PTR_CH23_10-06-13_349-360.indd 358358 66/19/2013/19/2013 8:48:468:48:46 PMPM ACUTE GASTROINTESTINAL AND GENITOURINARY DISORDERS: Answers

to sequestration within the bowel wall and lumen. This may lead to profound hypov- olemia with compromised end-organ perfusion. It is of utmost importance to ensure ade- quate volume status and electrolyte repletion. Elderly patients with multiple comorbidi- ties may require ICU monitoring. Routine use of antibiotics is not recommended in cases of bowel obstruction without intestinal perforation.

14. The answer is D. The diagnosis of acute pancreatitis is based on the clinical presentation and supported by biochemical data and morphological fi ndings on CT scan. There is no biochemical fi nding that is pathognomonic for pancreatitis. None of the other answer choices are specifi c or sensitive for the diagnosis of pancreatitis; they can occur in patients with other abdominal and extra-abdominal disorders. These other conditions include acute cholecystitis, intestinal infarction, and perforated peptic ulcer. In addition, severe pancreatitis can occur even without marked elevations in these serum enzymes.

15. The answer is B. The diagnosis of pancreatitis is usually a presumptive one. Patients presenting with an acute onset of abdominal pain need a thorough workup to exclude other causes of abdominal pain for which immediate laparotomy may be indicated. In a patient with acute pancreatitis, the initial management is nonoperative and focuses on fl uid resuscitation and prevention of complications. These patients are usually best man- aged in an intensive care setting. Gastric decompression is often indicated because of the associated paralytic ileus and delayed gastric emptying. Controlled studies have consis- tently demonstrated no benefi t from empiric antibiotics in patients with pancreatitis. If there is evidence of pancreatic necrosis with superimposed infection, then antibiotics are defi nitely recommended. Planned surgical exploration in patients with acute pancreati- tis is associated with a signifi cantly higher rate of complications. Operation is therefore delayed until complications such as hemorrhage or infection arise (4).

16. The answer is D. Pancreatic infection complicating acute pancreatitis should be suspected in any patient who fails to improve with supportive medical therapy or who initially improves but then deteriorates clinically. Pancreatic infection occasionally occurs early in the course of the disease, but typically presents later as in the clinical scenario presented in the question. CT scanning is the best method for pancreatic imaging and should be used frequently in patients with severe pancreatitis. When pancreatic infection is present, operative drainage and debridement are the gold standard for treatment. While some spe- cialized centers have had success with laparoscopic or endoscopic drainage techniques, these modalities should be reserved for practioners with signifi cant experience with these techniques.

17. The answer is C. Bladder dysfunction has been reported in up to 50% of patients after abdominoperineal resection and/or other major pelvic surgery. The type of voiding dys- function that occurs depends on the specifi c nerve involved and the degree of injury. The majority of patients are best treated by clean intermittent catheterization. Most patients (80%) will have resolution of their symptoms in 3 to 6 months. The use of a chronic cath- eter is of benefi t in some patients; however, chronic catheterization carries a higher risk of infection. The use of alpha-blockers alone or transurethral resection of the prostate (TURP) is likely to be unsuccessful. Transurethral sphincterotomy does not treat the underlying problem and may actually lead to incontinence (4).

359

66485457-66485438 www.ketabpezeshki.com

ZZakaria_87574_PTR_CH23_10-06-13_349-360.inddakaria_87574_PTR_CH23_10-06-13_349-360.indd 359359 66/19/2013/19/2013 8:48:468:48:46 PMPM GENERAL CRITICAL CARE: PATHOLOGY, PATHOPHYSIOLOGY, AND THERAPY

18. The answer is C. All patients with gross hematuria and all patients with microscopic hematuria on two separate urinalyses (UA) require some sort of evaluation. Patients should undergo upper-tract imaging, cystoscopic studies, bladder washings, and a pelvic examination. It is important to evaluate the UA for signs of infection and proteinuria. The differential diagnosis consists of nephrolithiasis, renal or bladder cancer, urinary tract infection, bleeding disorders, trauma, benign or essential hematuria, or prostatic disor- ders (in men).

19. The answer is A. Bladder injury may occur from blunt or penetrating trauma or during pelvic surgery. When associated with pelvic fractures, the injury is usually to the extra- peritoneal bladder, caused by the shearing force of the pelvic fracture. Isolated extraperi- toneal bladder rupture is treated with 7 to 10 days of Foley drainage. Blunt injury without pelvic fracture is associated with intraperitoneal bladder rupture, particularly if the blad- der is full and the injury occurs at the dome of the distended bladder. Bladder trauma should be suspected in any patient with lower abdominal trauma if there is hematuria or the patient is unable to void. Single-view cystographic studies will likely miss a signifi - cant injury. The treatment for an intraperitoneal bladder injury typically requires opera- tive repair with a two-layer closure. Intraoperative injury typically does not require a suprapubic cystostomy, but does require suture repair with several days of bladder drain- age with a Foley catheter. It is mandatory to be vigilant that the Foley catheter does not become obstructed, as this could lead to bladder distention and breakdown of the repair.

References

1. Brunicardi, FC, et al. Schwartz’s Principles of Surgery: Self-Assessment and Board Review. New York, NY: McGraw-Hill; 2007. 2. Howland MA. Antidotes in depth, acetaminophen. In: Goldfrank LR, Flomenbaum NE, Lewin NA, et al. eds. Goldfrank’s Toxicologic Emergencies. 7th ed. New York, NY: McGraw- Hill; 2002, 502–506. 3. Schermer, CR, et al. Ogilvie’s syndrome in the surgical patient: a new therapeutic modal- ity. J Gastrointest Surg. 1999;2:173. 4. Bines, et al. Rush University Medical Center: Review of Surgery. 4th ed. Philadelphia: Saunders; 2007.

360

66485457-66485438 www.ketabpezeshki.com

ZZakaria_87574_PTR_CH23_10-06-13_349-360.inddakaria_87574_PTR_CH23_10-06-13_349-360.indd 360360 66/19/2013/19/2013 8:48:468:48:46 PMPM 24 Diagnosis of Brain Death David J. Powner QUESTIONS

1. Criteria to be used in the diagnosis of brain death are specifi ed or required by: A. Harvard criteria B. The President’s Commission for the Study of Ethical Problems in Medicine and Biomedical and Behavioral Research C. Policy or procedures established by individual hospitals in accord with local current medical practice D. The American Academy of Neurology (AAN) guidelines from 1995 and revised in 2010 E. Federal mandates established by the Health and Human Services Administration’s Health Resources and Services Administration (HRSA) in 2009

2. The Uniform Determination of Death Act (UDDA) proposed by the President’s Commission and endorsed by the American Medical Association (AMA), American Bar Association (ABA), and National Conference of Commissioners on Uniform State Laws is accepted by most states as the basis for brain death statues. It stipulates: A. Certifi cation of death must be made by the attending physician B. Criteria for brain-stem death with coma and whole brain criteria are equivalent C. Cessation of cardiac/respiratory or brain function must be irreversible D. Confi rmatory examinations must be used if any question exists regarding the cause of the patient’s coma

ANSWERS TO THIS SECTION CAN BE FOUND ON PAGE 366 361

66485457-66485438 www.ketabpezeshki.com

ZZakaria_87574_PTR_CH24_10-06-13_361-370.inddakaria_87574_PTR_CH24_10-06-13_361-370.indd 361361 66/19/2013/19/2013 4:47:434:47:43 PMPM GENERAL CRITICAL CARE: PATHOLOGY, PATHOPHYSIOLOGY, AND THERAPY

3. During the evolution of brain death, a “catecholamine storm” is described due to release of catecholamines from the brain, a large sympathetic nervous system discharge, and fi nally a syndrome of cerebrospinal disconnection, similar to patients with spinal cord injury. Cardiovascular manifestations include: A. Increased initial systemic vascular resistance producing severe hypertension in some patients B. Catecholamine-induced injury to myocardial contractile elements producing decreased systolic contractility C. Irritative ventricular dysrhythmias D. Subsequent hypotension in some patients due to vasodilation and increased vascular capitance. E. All of the above

4. False positive examinations erroneously diagnosing brain death have been associated with which of the following conditions? A. Hypothermia B. Drug ingestion (i.e., midazolam, valproic acid, barbiturates, lidocaine, and baclofen) C. Guillain–Barré syndrome D. Brainstem encephalitis E. All of the above

5. Brain death criteria may include an ancillary atropine test. Atropine in the specifi ed dose is administered intravenously. The response that supports the diagnosis of brain death is: A. Constriction of the pupils B. Heart rate acceleration at least 10 bpm above the patient’s baseline heart rate C. Absent bowel sounds and decreased urine output D. Heart rate acceleration less than 5 bpm above patient’s baseline heart rate

6. Donation after cardiac death (DCD) organ donation does which of the following? A. Anticipates that cardiac arrest will occur within 3 hours B. May provide kidney, liver, and lungs for transplantation C. Requires that asystole or pulselessness be sustained for more than 10 minutes before cardiac death is declared D. Excludes members of a recipient’s transplantation team from determining the time of death E. B and D F. B, C, and D

362

66485457-66485438 www.ketabpezeshki.com

ZZakaria_87574_PTR_CH24_10-06-13_361-370.inddakaria_87574_PTR_CH24_10-06-13_361-370.indd 362362 66/19/2013/19/2013 4:47:434:47:43 PMPM DIAGNOSIS OF BRAIN DEATH: Questions

7. Guidelines or criteria regarding apnea testing during brain death examination commonly require:

A. Arterial PaCO2 must reach or exceed a specifi ed level or percentage above baseline

B. Hypoxemia below a specifi ed PaO2 must be achieved to assure that no central nervous system (CNS) hypoxia-sensing centers are active C. Testing must be discontinued if hypotension or hypoxemia cannot be controlled D. A specifi ed degree or change in blood pH (acidemia) must be achieved to confi rm medullary respiratory center inactivation E. A and C F. A, C, and D

8. The American Academy of Neurology (AAN) in 2010 updated its 1995 publication for determining brain death in adults. In the 2010 update, which three essential parameters were stated to be necessary for this diagnosis? A. Sustained blood pressure (mean arterial pressure > 65 mmHg) during testing B. Coma C. Heart rate > 65 bpm, although normal sinus rhythm is not necessary D. Apnea E. Permission by patient’s legal representative for brain death assessment F. Absent brainstem refl exes G. Body temperature above 96°F H. Isoelectric electroencephalogram

9. If a confi rmatory test for certifi cation of brain death is used, which two tests evaluate ana- tomic blood fl ow and have been used when confounding drugs and so on are present? A. Bispectral index B. Somatosensory evoked potentials C. Technetium-labeled (Tc99m-hexamethylpropyleneamine oxime [HMPAO]) radionu- clide sintigraphy D. Four-vessel angiography E. Electroencephalography

10. If a confounding drug is known or suspected to be present, how many half-lives should pass before a blood concentration will show a minimal or absent level? A. 1 B. 2 C. 3 D. 5 E. 10

363

66485457-66485438 www.ketabpezeshki.com

ZZakaria_87574_PTR_CH24_10-06-13_361-370.inddakaria_87574_PTR_CH24_10-06-13_361-370.indd 363363 66/19/2013/19/2013 4:47:434:47:43 PMPM GENERAL CRITICAL CARE: PATHOLOGY, PATHOPHYSIOLOGY, AND THERAPY

11. The train-of-four measurement by a peripheral nerve stimulator is used to determine the absence of which category(ies) of drugs? A. Benzodiazepines B. Barbiturates C. Nondepolarizing neuromuscular blockers D. Opiates E. All of the above F. None of the above

12. When evaluating the oculovestibular refl ex, to ensure maximal stimulation of the vestibu- lar semicircular canals, the external auditory canal should be clear of cerumen and the patient’s head should be: A. In 30° of Trendelenburg positioning B. Rotated 20° to 30° toward the contralateral side away from the side receiving irrigation C. Flat D. Elevated 30°

13. In brain death, the patient’s pupils should be: A. Bilaterally fi xed to direct light, dilated, and equal B. Bilaterally fi xed to direct light and dilated, but anisocoria can be present C. Bilaterally fi xed to direct light, dilated and equal, but constriction to consensual light stimulation may be present D. Bilaterally fi xed to direct and consensual light stimulation, but unequal in size and midposition may be present

14. After diagnosing brain death, a certifying note or document is entered into the patient’s medical record by the diagnosing physician. Thereafter: A. The patient’s attending physician meets with the patient’s family and requests organ donation B. The certifying MD meets with the family in the presence of the organ procurement organization (OPO) coordinator and requests organ donation C. The certifying MD and the OPO representative meet with the family to inform them of the diagnosis of brain death and the patient’s death D. The OPO representative meets the family, informs them of the patient’s brain death, and requests organ donation

15. The time of death to be entered into the medical record and recorded on the death certifi - cate for a heart-beating brain-dead organ donor is: A. At the time of transfer to the OR for organ procurement B. At completion and review testing as required by the certifying MD C. When cardiac arrest occurs in the operating room after solid organ removal D. After consultation between the certifying MD and the attending MD E. After informing the family and their fi nal visitation

364

66485457-66485438 www.ketabpezeshki.com

ZZakaria_87574_PTR_CH24_10-06-13_361-370.inddakaria_87574_PTR_CH24_10-06-13_361-370.indd 364364 66/19/2013/19/2013 4:47:444:47:44 PMPM DIAGNOSIS OF BRAIN DEATH: Questions

16. Despite the profoundly depressant effect of pentobarbital infusion on neuronal function, brain death may be diagnosed using which one of the following examinations? A. Physical examination after cessation of the infusion for 6 hours B. Technetium-hexamethylpropyleneamine oxime (HMPAO) bedside radionuclide testing C. Electroencephalography after cessation of the infusion for 6 hours D. Somatosensory-evoked potential absence of P14 E. None of the above: brain death cannot be diagnosed during pentobarbital infusion

365

66485457-66485438 www.ketabpezeshki.com

ZZakaria_87574_PTR_CH24_10-06-13_361-370.inddakaria_87574_PTR_CH24_10-06-13_361-370.indd 365365 66/19/2013/19/2013 4:47:444:47:44 PMPM 24

ANSWERS

1. The answer is C. The Harvard Criteria were published in 1968 (1) and, although fre- quently referenced, were not widely adopted because they did not include an allowance for continuation of spinal refl exes, recognized to be common in patients with confi rmation of brain death by four-vessel angiography. President Reagan’s President’s Commission, an esteemed group of knowledgeable physicians, ethicists, theologians, and others, attempted to defi ne and guide thinking about persistent coma states, not necessarily brain death. However, their publications (2) were widely interpreted as guidance for the area of brain death. A most important conclusion recommended the Uniform Determination of Death Act (UDDA): “An individual who has sustained either (1) irreversible cessation of circulatory and respiratory functions or (2) irreversible cessation of all functions of the entire brain, including the brainstem, is dead. A determination of death must be made with accepted medical standards.” This statement was endorsed by the American Medical Association (AMA), American Bar Association (ABA), and the National Conference of Commissioners on Uniform State Laws, and was adopted as the foundation of most state statutes. As noted, it leaves the policies and practices for the determination of death to “accepted medical standards,” which, in practice, have been interpreted as individual hospital policies and procedures. Many hospitals adopted guidelines from the AAN pub- lished in 1995 (3), but considerable diversity has been documented among these policies throughout the United States (4) and internationally (5). Each practitioner, therefore, must be familiar with, and follow, the individual hospital policy that applies to brain death determination and certifi cation. AAN guidelines of 1995 and 2010 (3,6) are not binding upon physician practice, and HRSA has not issued mandates.

2. The answer is C. See UDDA recommendation in Answer 1. Individual hospital policies may stipulate specifi cs within the brain death examination, who may perform it or certify fi ndings, if specialists are to be involved, the frequency of repeat exams, if confi rmatory

366

66485457-66485438 www.ketabpezeshki.com

ZZakaria_87574_PTR_CH24_10-06-13_361-370.inddakaria_87574_PTR_CH24_10-06-13_361-370.indd 366366 66/19/2013/19/2013 4:47:444:47:44 PMPM DIAGNOSIS OF BRAIN DEATH: Answers

tests are required or under what circumstances they are recommended, and other require- ments or variations.

3. The answer is E. Primate experiments in which brain death is suddenly induced (7) and human observations document the series of cardiovascular sequelae to the rapid evolution of brain death. When the brain herniation/destruction occurs less abruptly, the cardiovascular consequences are diminished. During the evolution of brain death, about 40% of patients initially demonstrate profound hypertension followed within minutes to a few hours by severe hypotension. Although many published recom- mendations about treatment of the brain-dead patient and donor exist (8), none is evidence-based. Donor care has not been systematically investigated to identify opti- mal treatment parameters or methods, either by prospective utilization of alternative physiological endpoints, medications, and so on or by retrospective analysis of, for example, graft or host survival that correspond to a detailed compilation of donor characteristics (9).

4. The answer is E. Each of the choices given is well documented. New 2010 American Academy of Neurology (AAN) guidelines (6) recommend a core body temperature above 36°C before testing. Documenting the absence of all drugs and perhaps the syn- ergistic effect of alcohol ingestion is diffi cult by history alone, and not all drugs are tested in screening laboratory batteries. When a drug is known to be present, AAN guidelines recommend that 5 half-lives of the drug pass or blood concentrations, if available, of the drug be below its therapeutic level. If alcohol ingestion is known or suspected, a blood alcohol level <0.08% is recommended.

5. The answer is D. When required, the dose of atropine specifi ed by the hospital policy is given intravenously. A smaller test dose may be prudent to prevent excessive tachycardia if the vagus nerve nuclei have not been obliterated. Loss of vagal response to atropine eliminates the expected tachycardia (10). Atropine dilates pupils that remain responsive, an accelerated heart rate would negate the diagnosis of brain death because of a respon- sive vagal effect, and although decreased bowel sounds and urine output are cholinergic responses, they would not be practical in brain death testing.

6. The answer is E. DCD organ donation anticipates a rapid progress to cardiac arrest after life-sustaining measures are decreased (11). This rapidity intends to minimize “warm ischemic time” of diminishing organ perfusion during a prolonged period of decreasing blood pressure, oxygenation, and so on. While no regulations specify the duration of tol- erable warm ischemic time, the customary time permitted is about 1 hour. Usually if car- diac arrest has not occurred during that time, organ removal is canceled and the patient is moved from the operating room to another patient unit. Either asystole or an absent pulse waveform during intravascular monitoring or to palpation initiates a waiting time before pronouncement of cardiac death and organ procurement. National recommendations and common practice (no regulations, only individual hospital policy) use either 3 or 5 min- utes of sustained asystole or pulselessness before organs may be removed. Individual policies address the response if recovery of cardiac function occurs after initial pulseless- ness or asystole, but before pronouncement of cardiac death. Kidneys and the liver are commonly removed for donation, and with increasing frequency one or both lungs are

367

66485457-66485438 www.ketabpezeshki.com

ZZakaria_87574_PTR_CH24_10-06-13_361-370.inddakaria_87574_PTR_CH24_10-06-13_361-370.indd 367367 66/19/2013/19/2013 4:47:444:47:44 PMPM GENERAL CRITICAL CARE: PATHOLOGY, PATHOPHYSIOLOGY, AND THERAPY

being recovered. Most policies exclude members of the recipient transplant team from any function until the patient has been declared deceased by the physician in attendance with that responsibility (12,13).

7. The answer is E. Testing for an absent respiratory response to a maximal physiological stimulus (apnea test) is considered an essential element in evaluating the brain stem (6). Current American Academy of Neurology (AAN) recommendations (6) include a

specifi ed time off the ventilator and an end-point PaCO2 (>60 mmHg) or 20 mmHg above the patient’s baseline. Testing should not be initiated if the patient is hypocarbic,

as the PaCO2 rises only 4 to 6 mmHg/minute after brain death. Therefore, time off the

ventilator will be prolonged if the PaCO2 is low when testing is started. Apneic oxygen-

ation using a cannula inserted into the endotracheal tubes and an O2 fl ow of about 6

L/minute generally sustain the PaO2. Pulse oximetry monitoring, however, is essential throughout the test. Expansion rupture of the lung has occurred if the endotracheal tube is obstructed by the cannula (too far in or too small endotracheal tube) and oxygen cannot exit the tube. Apnea testing should be discontinued immediately if respiratory efforts are detected or if hypoxemia or cardiovascular instability occurs. Hypoxemia stimulates peripheral carotid and aortic (not CNS) chemoreceptors. AAN guidelines do not specify a hypoxemic threshold for chronic obstructive pulmonary disease patients who may rely on a hypoxemic ventilatory stimulus. Although H ion concentration is the important local medullary stimulus for respiration, systemic acidosis is not a speci- fi ed criterion.

8. The answers are B, D, and F. New AAN guidelines (6) state these as specifi c criteria for brain death diagnosis. Other choices in this question are important qualifying precau- tions before testing is initiated to assure the validity of the examination results. Family permission is not required in any published criteria set.

9. The answers are C and D. Hospital policy stipulates which confi rmatory tests for brain death are acceptable in that facility. These tests evaluate either anatomic blood fl ow to the brain or some measure of neuronal function. If any confounding factors might be present that could compromise neuronal depolarization/transmission, a test of blood fl ow should be utilized. The gold standard for blood fl ow evaluation is the four-vessel angiogram. Cerebral blood fl ow using Tc99 HMPAO single-photon emission computed tomography (SPECT) scanning to show absence of indicator fl ow and uptake is widely available as a bedside assessment tool (14). This form of technetium allows visualization of uptake or metabolism of brain tissue in the posterior fossa as well as in the anterior circulation. Other choices in this question only evaluate aspects of neuronal function and may be compromised by drugs.

10. The answer is D. The general pharmacological principle applies that the time equivalent to 4 biopharmaceutical half-lives is required to assure full metabolism/removal of a drug from the circulation. American Academy of Neurology (AAN) guidelines (6) recommend that this be increased to 5 half-lives and, if available, blood concentrations of the drug be below its therapeutic range. If alcohol ingestion is suspected, the blood alcohol level should be below the legal limit for driving.

368

66485457-66485438 www.ketabpezeshki.com

ZZakaria_87574_PTR_CH24_10-06-13_361-370.inddakaria_87574_PTR_CH24_10-06-13_361-370.indd 368368 66/19/2013/19/2013 4:47:444:47:44 PMPM DIAGNOSIS OF BRAIN DEATH: Answers

11. The answer is C. This assessment tool only evaluates neuromuscular blockade from either depolarizing or nondepolarizing agents. A response that shows an absent “fade” during train-of-four stimulation or a maximal response to a single “twitch” stimulus, rather than use of tetanic stimulation, provides the best measure when residual neuromuscular block- ade, which may interfere with testing, is not present (15).

12. The answer is D. This technical detail is stipulated within the 2010 American Academy of Neurology (AAN) guidelines (6).

13. The answer is D. These are the examination fi ndings in brain death, as recommended in the 2010 American Academy of Neurology (AAN) guidelines (6).

14. The answer is C. Federal and Medicare regulations disseminated in 1986 and 1988 exclu- sively assign the duty of obtaining consent for organ donation from the patient’s legal rep- resentative to the regional OPO. Some OPOs extend this responsibility/privilege to others who have received training as “extended requestors.” All others, including physicians, nurses, chaplains, social service professionals, and so on, are prohibited from request- ing or initiating discussions about organ donation with family members. Inclusion of the OPO coordinator in family meetings, especially to inform about the process and results of brain death evaluation, is an important component of the donation process. Some data support separating the announcement of the patient’s death to the family from the request for donation. It is helpful to the OPO coordinator, however, to be present when the MD informs the family of the patient’s brain death, although the coordinator’s request for donation may be delayed.

15. The answer is B. Brain death is a medical diagnosis confi rmed in accord with the hos- pital’s policy and procedures by the responsible physician after reviewing the results of those tests deemed necessary. Other choices in this question are independent of or seque- lae to the physician’s diagnosis.

16. The answer is B. As previously discussed, when drugs, either recreational or therapeu- tic, are present that might alter neuronal function, only a confi rmatory test of anatomi- cal blood fl ow can confi rm the absence of nutrient blood fl ow, confi rming brain death. The biopharmaceutical half-life of pentobarbital is 15 to 50 hours (16), so withhold- ing the infusion for only 6 hours would not ensure its complete metabolism. Physical examination and somatosensory testing would not be valid due to the persistent pen- tobarbitol effect.

References

1. A Defi nition of Irreversible Coma. Report of the Ad Hoc Committee of the Harvard Medical School to Examine the Defi nition of Brain Death. JAMA. 1968;205:337–340. 2. Guidelines for the Determination of Death. Report of the Medical Consultants on the Diagnosis of Death to the President’s Commission for the Study of Ethical Problems in Medicine and Biochemical and Behavioral Research. JAMA. 1981;246: 2184–186.

369

66485457-66485438 www.ketabpezeshki.com

ZZakaria_87574_PTR_CH24_10-06-13_361-370.inddakaria_87574_PTR_CH24_10-06-13_361-370.indd 369369 66/19/2013/19/2013 4:47:444:47:44 PMPM GENERAL CRITICAL CARE: PATHOLOGY, PATHOPHYSIOLOGY, AND THERAPY

3. The Quality Standards Subcommittee of the American Academy of Neurology. Practice parameters for determining brain death in adults (summary statement). Neurology. 1995;45:1012–1014. 4. Powner DJ, Hernandez M, Rives TE, et al. Variability among hospital policies for deter- mining brain death in adults. Crit Care Med. 2004;32:1284–1288. 5. Wijdicks EFM. Brain death worldwide: accepted fact but no global consensus in diagnos- tic criteria. Neurology. 2002;58:20–25. 6. Wijdicks, EFM, Varelas PN, Gronseth GS, et al. Evidence-based guideline update: deter- mining brain death in adults. Report of the Quality Standards Subcommittee of the American Academy of Neurology. Neurology. 2110;74:1911–1918. 7. Cooper DKC, Novitzky D, Wicomb WN, et al. Pathophysiology of brain death in the experimental animal: extracranial aspects. Transplant Proc. 1988;20(Suppl 7):25–28. 8. Frontera JA, Kalb T. How I manage the adult potential organ donor: donation after neu- rological death. Neurocrit Care. 2010;12:103–110. 9. Powner DJ. How does one care for the heart-beating, brain dead adult organ donor? In Deutschman CS, Neligan PJ, eds. Evidenced-Based Practice of Critical Care. Philadelphia: Saunders/Elsevier; 2010:637–642. 10. Huttemann E, Schelenz C, Sakka SG, et al. Atropine test and circulatory arrest in the fossa posterior assessed by transcranial Doppler. Intensive Care Med. 2000;26:422–425. 11. Yee AH, Rabinstein AA, Wijdicks EFM, et al. Factors infl uencing time to death after with- drawal of life support in neurocritical patients. Neurology. 2010;74:1380–1385. 12. Frontera JA. How I manage the adult potential organ donor: donation after cardiac death (Part 2). Neurocrit Care. 2010;12:111–116. 13. Bernat JL, D’Alessandro FK, Port TP, et al. Report of a national conference on donation after cardiac death. Am J Transplant. 2006;6:281–291. 14. Busl KM, Greer DM. Pitfalls in the diagnosis of brain death. Neurocrit Care. 2009;11:276–287. 15. Murphy GS, Brull SJ. Residual neuromuscular block: lesson unlearned, part I: defi nitions, incidence, and adverse physiologic effects of residual neuromuscular block. Anesth Analg. 2010;111:120–128. 16. Molina DK, McCutechon JR, Rulon JJ. Head injuries, pentobarbital, and the determina- tion of death. Am J Forensic Med Pathol. 2009;30:75–77.

370

66485457-66485438 www.ketabpezeshki.com

ZZakaria_87574_PTR_CH24_10-06-13_361-370.inddakaria_87574_PTR_CH24_10-06-13_361-370.indd 370370 66/19/2013/19/2013 4:47:444:47:44 PMPM 25 General Trauma and Burns Sasha D. Adams and Amy R. Alger QUESTIONS

1. For evaluation of the cervical spine after trauma: A. Cervical collar can be removed from an awake patient with known femur fracture who does not complain of neck pain B. Cervical spine should be evaluated by three-view cervical spine series supplemented by swimmer’s views as needed C. Flexion/extension radiography should not be performed on an obtunded patient D. All penetrating cranial injuries must be stabilized with a cervical collar

2. Angiographic evaluation of the neck for vascular injury is warranted in all of the following situations, except: A. Blunt trauma with neurologic defi cit and no closed head injury B. C3 vertebral body fracture through the foramen transversarium C. Seat belt abrasion on the anterior neck D. Hyperextension or hyperfl exion mechanism of injury E. Orbital wall fracture

ANSWERS TO THIS SECTION CAN BE FOUND ON PAGE 377 371

66485457-66485438 www.ketabpezeshki.com

ZZakaria_87574_PTR_CH25_10-06-13_371-382.inddakaria_87574_PTR_CH25_10-06-13_371-382.indd 371371 66/19/2013/19/2013 4:47:544:47:54 PMPM GENERAL CRITICAL CARE: PATHOLOGY, PATHOPHYSIOLOGY, AND THERAPY

3. Which of the following is not a risk factor for coagulopathy following multisystem trauma? A. Resuscitation with 6 L crystalloid, 10 units of packed RBCs and 10 units of fresh-frozen plasma (FFP) B. Prolonged exposure and hypothermia C. Traumatic brain injury (TBI) D. Metabolic alkalosis E. 75-year-old patient with coronary artery disease

4. A healthy 30-year-old multisystem trauma patient is in the ICU for resuscitation. He has a known pelvic fracture, traumatic subarachnoid hemorrhage, and multiple extremity frac- tures. Hct = 17, pulmonary artery catheter shows cardiac index of 1.8, systemic vascular resistance = 3,000, and wedge pressure is 5. Urine output is 10 mL/hour. The most appro- priate initial treatment would be: A. Start levophed drip, goal mean arterial pressure (MAP) > 60 mmHg B. Give 3% hypertonic saline bolus C. Renal dose dopamine D. Lasix 40 mg IV × 1 E. 2 L lactated Ringer’s bolus

5. After emergent craniotomy for a large epidural hemorrhage, a 21-year-old ejected unre- strained passenger of a motor vehicle collision (MVC) is brought to the ICU for the remain- der of his workup. On examination of an open humeral fracture, which of the following signs does not require immediate vascular operative intervention? A. Pulsatile bleeding B. Expanding hematoma C. Proximity of wound or injury to artery D. Absent or diminished distal pulse E. Cold blue hand

6. A 21-year-old man is admitted to the ICU after being involved in a motorcycle collision (MCC) in which he was thrown 20 feet. He is intubated with a closed head injury, and although he initially responded to 2 L crystalloid resuscitation, is now hypotensive in the ICU. His pelvic x-ray revealed a fractured pelvis with anterolateral compression, and CT scan did not fi nd any intra-abdominal injury or contrast extravasation near fracture sites. The next step in management should be: A. Angioembolization of active hemorrhage B. Placement of pelvic binder C. Give 2 L of crystalloid fl uid D. Start epinephrine drip E. Exploratory laparotomy

372

66485457-66485438 www.ketabpezeshki.com

ZZakaria_87574_PTR_CH25_10-06-13_371-382.inddakaria_87574_PTR_CH25_10-06-13_371-382.indd 372372 66/19/2013/19/2013 4:47:544:47:54 PMPM GENERAL TRAUMA AND BURNS: Questions

7. An 18-year-old woman is involved in a motorcycle collision and is admitted with bilateral femoral fractures and closed head injury. She is intubated and taken to the ICU for moni- toring. On hospital day 2 she is taken to the OR for bilateral femur fi xation with placement of intramedullary rods. She receives 1 unit of packed RBCs in the OR. Postoperatively, she is increasingly hypoxic, is refractory to conventional ventilation, has bilateral pulmonary infi ltrates on chest x-ray (CXR), and worsening altered mental status. All the following are possible causes of hypoxia, except: A. Fat embolism syndrome (FES) B. Aspiration C. Transfusion-related acute lung injury D. Pulmonary contusion E. Bronchial injury

8. If the diagnosis for the patient is Question 7 is fat embolism syndrome (FES): A. Without ventilator support, FES can be fatal B. There are strict diagnostic criteria for FES C. She should be started immediately on steroids and heparin D. Respiratory symptoms are typically of short duration E. Delayed femur fi xation would have decreased her risk of symptoms

9. A 54-year-old man is in the ICU for serial neurological exams following a low-speed motor vehicle collision (MVC) with traumatic subarachnoid hemorrhage and a right tibial-pla- teau fracture. He has been stable over the past 10 hours, but is complaining of increasing pain in his leg. He states he cannot move his toes, and he describes a tingling sensation near his fi rst toe. On physical exam, his right leg is swollen and tense, with palpable dor- salis pedis/posterior tibial (DP/PT) pulses. The next step in management should be: A. Elevate the leg B. Check an ankle brachial index C. Call the surgeon D. Elastic compression bandage E. Electromyography (EMG)

10. A 67-year-old homeless man jumps off a bridge and lands on his feet. He has altered mental status due to alcohol intoxication. What injury pattern is typically seen with this mechanism? A. Calcaneal fracture B. Rib fractures C. Lumbar spine fracture D. A and C E. All of the above

373

66485457-66485438 www.ketabpezeshki.com

ZZakaria_87574_PTR_CH25_10-06-13_371-382.inddakaria_87574_PTR_CH25_10-06-13_371-382.indd 373373 66/19/2013/19/2013 4:47:544:47:54 PMPM GENERAL CRITICAL CARE: PATHOLOGY, PATHOPHYSIOLOGY, AND THERAPY

11. A 19-year-old man was unrestrained and ejected following a high-speed motor vehicle collision (MVC). His injuries include multiple rib fractures on the right and a hemopneu- mothorax for which a right-sided chest tube is placed. Of the following choices, which indicates a need for immediate surgical intervention? A. Persistent pneumothorax on chest radiograph B. Initial 900 mL of sanguineous output following tube placement C. 200 mL/hour of sanguineous drainage for 4 hours

D. SaO2 of 86% and pulmonary contusion on chest CT scan

12. A 25-year-old woman involved in a high-speed motorcycle accident is admitted to the ICU with a grade IV liver laceration, multiple right-sided rib fractures and associated pulmonary contusion, left femur fracture, and facial trauma. She is intubated and on mechanical ventilation. Over the fi rst 24 hours, her transfusion and fl uid requirements included 6 units of PRBC, 6 units of fresh-frozen platelets (FFP), a platelet 6-pack, and 6 L of crystalloid, after which she stabilized. On ICU day 3, she becomes hypotensive to 96/

54 mmHg, plateau airway pressures are 34 mmHg, and pCO2 is 55 mmHg. Urine output has decreased over the past 6 hours to 10 mL/hour. Blood draw reveals a creatinine of 2.8 g/dL, up from 1.0 on admission, and hemoglobin is stable from ICU day 2 at 9.8 mg/ dL. Central venous pressure (CVP) is 18. Bladder pressure is measured at 32 mmHg. The next intervention is: A. Angiography for liver embolization B. Operative abdominal decompression C. Continuous venovenous hemofi ltration (CVVH) D. PRBC and FFP transfusion E. Bronchoscopy

13. A 45-year-old man was thrown from a horse. He presented to the ED hypotensive at 90/50, but responded to a fl uid bolus and has remained normotensive since. Imaging revealed a small subdural hematoma (SDH) for which he is being observed, right 10–11 rib fractures, and a grade III liver laceration. A. Nonoperative management of liver lacerations results in increased length of hospital stay B. If liver lacerations are managed nonoperatively, the patient should remain on bed rest for 1 week C. Nonoperative management is appropriate for grades I and II liver laceration, but grades III and IV require either operation or angioembolization D. Nonoperative management can be used for select cases of penetrating trauma with isolated liver injury E. The CT scan should be repeated prior to discharge to ensure healing of the liver laceration

374

66485457-66485438 www.ketabpezeshki.com

ZZakaria_87574_PTR_CH25_10-06-13_371-382.inddakaria_87574_PTR_CH25_10-06-13_371-382.indd 374374 66/19/2013/19/2013 4:47:544:47:54 PMPM GENERAL TRAUMA AND BURNS: Questions

14. A 38-year-old man is admitted to the ICU after being involved in a motor vehicle collision (MVC) as an unrestrained driver. Among other injuries, he has signifi cant bruising to his chest from the steering wheel, and there is a sternal fracture and several left-sided rib fractures on his CXR. You are concerned about blunt cardiac injury (BCI). A. Normal electrocardiogram (EKG), enzymes, and blood pressure rule out BCI B. Angiography should be performed immediately to look for injury C. Cardiac enzymes, specifi cally CK-MB (creatine kinase MB isoenzyme) should be checked three times at 8-hour intervals, and if elevated, further workup for cardiac injury is warranted D. His sternum should be operatively stabilized E. An echocardiogram should be done on all patients with blunt chest trauma

15. A 37-year-old woman was brought to the ICU after assault, with injuries including blunt head trauma and stab wounds to the arm and anterior midabdomen. The only studies obtained were a CT scan of the head and plain fi lms of the chest and abdomen. On local wound exploration, her stab wounds did not appear to violate the fascia, and she was followed with serial exams. On ICU day 2 she is hypotensive, oliguric, and on exam has increased abdominal distension. What is your next step in management? A. Obtain upright kidney–ureter–bladder (KUB) x-ray B. Start dopamine infusion, MAP > 60 mmHg C. Hold tube feeds for 1 hour, then restart D. Give lasix 20 mg IV × 1 E. 3% hypertonic saline bolus

16. A patient with third-degree burns on 5% of his lower extremities is being treated with topical agents. Concerns include: A. Mafenide acetate is painful when applied B. Silver nitrate has good penetrance of eschar C. Pseudomonas is best treated with silver sulfadiazine D. Silver sulfadiazine is associated with signifi cant electrolyte abnormalities E. Metabolic acidosis is seen following silver nitrate treatment

17. A 26-year-old with electrical burns is brought to the ED after grabbing a high-voltage power line with both hands at work. He fell out of a bucket approximately 25 feet to the ground and was unconscious. He has third-degree burns over a 3% body surface area (BSA) and weighs 75 kg. He has been intubated and taken to the OR emergently for bilat- eral upper extremity fasciotomies and muscle debridement. A. IV fl uids should be titrated for urine output of at least 100 mL/hour B. IV fl uid resuscitation is per the Parkland formula C. Tetanus toxoid is not necessary since he has no open wounds D. Operative intervention should be delayed until fully resuscitated.

375

66485457-66485438 www.ketabpezeshki.com

ZZakaria_87574_PTR_CH25_10-06-13_371-382.inddakaria_87574_PTR_CH25_10-06-13_371-382.indd 375375 66/19/2013/19/2013 4:47:554:47:55 PMPM GENERAL CRITICAL CARE: PATHOLOGY, PATHOPHYSIOLOGY, AND THERAPY

18. In addition to the treatment in Question 17, the patient should be closely monitored and examined for: A. Cardiac arrhythmia B. Myoglobinemia C. Traumatic injuries to spine and extremities D. Compartment syndrome of both forearms E. All of the above

19. A 76-year-old woman is found in her burning home by fi refi ghters after apparently falling asleep with a heater on in her bedroom. She has fi rst-degree burns to both arms and chest and 3% second-degree burns to her right arm. She is breathing spontaneously but has a raspy voice, and has soot around her nose with singed nasal hair.

A. If her oxygen saturation (SaO2) is >97%, she does not require intubation

B. She should be placed on a non-rebreather face mask with 100% O2 C. Burns should be treated topically, and then she should be prescribed pain medicine and discharged to home under her son’s care D. Resuscitate her with IV hydration at 2 mL LR × body weight (kg) × % total body sur- face area (BSA) burn

20. A 42-year-old man was trapped in a house fi re and rescued by fi refi ghters from his basement meth lab. He is brought to the ICU with 80% third-degree burns, sparing only both feet and lower legs. He is started on Parkland formula resuscitation with adequate urine output. Four hours into resuscitation, the respiratory therapist (RT) reports that he is becoming diffi cult to ventilate and has elevated airway pressures. Your next step should be: A. Increase positive end-expiratory pressure (PEEP) B. Perform immediate chest escharotomies C. Switch to oscillator ventilation D. Give nebulizer treatments

376

66485457-66485438 www.ketabpezeshki.com

ZZakaria_87574_PTR_CH25_10-06-13_371-382.inddakaria_87574_PTR_CH25_10-06-13_371-382.indd 376376 66/19/2013/19/2013 4:47:554:47:55 PMPM 25

ANSWERS

1. The answer is C. CT scan evaluation of the cervical spine following trauma has sup- planted plain radiography as the primary screening modality following trauma. CT scan is more accurate, is both time- and cost-effective, and does not require additional fi lms that may delay fi nal diagnosis. Clinical clearance remains the standard in awake, alert trauma patients who have no neck pain or tenderness with full range of motion; how- ever, clinical exam is unreliable in those with neurologic defi cit or distracting injury such as a long bone fracture, and collars should be maintained. Flexion extension fi lms in an obtunded patient are minimally useful for identifi cation of ligamentous injury and may be harmful. Immobilization in a cervical collar after penetrating trauma to the brain is not necessary unless the trajectory suggests direct injury to the cervical spine. Many of these patients will require emergent airway management, and spine immobilization may complicate airway access (1).

2. The answer is E. Blunt injury to the carotid or vertebral vessels (blunt cerebrovascular injury [BCVI]) was historically diagnosed after the development of neurological symp- toms, with signifi cant neurologic morbidity. With increased screening of asymptomatic patients, BCVI has been diagnosed in 1% of all blunt trauma patients, allowing for treat- ment and prevention of related morbidity and mortality (2).

3. The answer is D. Hemostasis is the primary goal in a bleeding trauma patient, and following surgical and endovascular therapies, detection and aggressive correction of systemic coagulopathy remain critical. The coagulopathy in trauma is multifactorial in origin, not uncommon, and an independent predictor of mortality. Massive hemorrhagic loss includes coagulation factors and platelets, which are then further diluted with crys- talloid resuscitation. FFP transfusion replenishes most coagulation factors; however,

377

66485457-66485438 www.ketabpezeshki.com

ZZakaria_87574_PTR_CH25_10-06-13_371-382.inddakaria_87574_PTR_CH25_10-06-13_371-382.indd 377377 66/19/2013/19/2013 4:47:554:47:55 PMPM GENERAL CRITICAL CARE: PATHOLOGY, PATHOPHYSIOLOGY, AND THERAPY

platelet transfusion is necessary to correct thrombocytopenia. In a massive hemorrhage setting, 1 unit platelet transfusion is given with every 6 units of packed RBCs and FFP. Hypothermia and acidosis both impede the function of coagulation factors by interfering with necessary enzyme reactions. Traumatic brain injury can cause thromboplastin expo- sure, which results in a consumptive disseminated intravascular coagulopathy. Elderly patients with comorbidities are more likely to be on antiplatelet therapy or anticoagulant agents prior to the trauma (3).

4. The answer is E. This patient is in hypovolemic shock, as evidenced by the low central venous pressure (CVP), low cardiac index, and high systemic vascular resistance (SVR). The severity of shock correlates with the amount of blood loss, and cardiac output starts to decline after 20% to 40% of blood volume is lost. When more than 40% blood volume is lost, the resultant shock is imminently life-threatening. SVR is elevated due to the body’s vasoconstriction response to hypovolemia. The most important interventions are replace- ment of volume and control of hemorrhage (4–6).

5. The answer is C. Attention to physical exam fi ndings is essential to the diagnosis of peripheral vascular injury. Patients with hard signs of peripheral vascular injury need immediate operative intervention. Hard signs include absent or diminished pulses, pulsa- tile bleeding, palpable thrill or bruit, expanding or pulsatile hematoma, or signs of distal ischemia, pain, pallor, paresthesia, paralysis, and coolness. Soft signs of injury warrant further investigation with arteriography. Soft signs include a wound in proximity to the path of the artery, a small nonpulsatile hematoma, neurologic defi cit in the extremity, and prehospital arterial bleeding (7).

6. The answer is B. Pelvic fractures are estimated to occur in about 9% of all blunt injury patients, and 9% of these are severe with substantial deformation and displacement. Hemorrhage can be signifi cant, and pelvic fractures are the third leading cause of death in motor vehicle collisions (MVCs). Concerning physical exam fi ndings include pain with pelvic movement, perineal and/or scrotal bruising, vaginal bleeding, and blood at the urethral meatus. Pelvic fractures represent a hidden source of hemorrhage into the retro- peritoneum. Finding of a “blush” extravasation of contrast on CT scan represents active hemorrhage and may need to be embolized with angiography. The fastest way to gain control of pelvic hemorrhage is through external compression with a pelvic binder. After already giving 2 L of crystalloid, the next resuscitative fl uid should be blood (8,9).

7. The answer is E. FES is estimated to occur in 19% of patients with major trauma, although this may be underreported. There is no universal defi nition for FES, so diagnosis requires a high degree of suspicion and is a diagnosis of exclusion. The classic triad of symptoms is respiratory insuffi ciency, altered mental status, and upper extremity and thoracic pete- chiae, although all three are seldom seen together. Major signs present within 24 hours in 65% of patients. Transfusion-related acute lung injury (TRALI) symptoms include dysp- nea, hypotension, pulmonary edema, and fever. Symptoms must be of acute onset with bilateral infi ltrates that usually occur within 6 hours of transfusion. Pulmonary contu- sion is a direct bruise of the lung parenchyma, followed by alveolar hemorrhage and edema. In moderate-to-severe contusions, ventilator management is critical to minimize barotrauma. Bronchial injury is unlikely without a pneumothorax (10–13).

378

66485457-66485438 www.ketabpezeshki.com

ZZakaria_87574_PTR_CH25_10-06-13_371-382.inddakaria_87574_PTR_CH25_10-06-13_371-382.indd 378378 66/19/2013/19/2013 4:47:554:47:55 PMPM GENERAL TRAUMA AND BURNS: Answers

8. The answer is A. There are no specifi c treatments for FES. Previous treatments with hepa- rin, dextran, and corticosteroids have not been shown to reduce morbidity or mortality. Treatment is largely supportive with ventilator strategies similar to acute respiratory dis- tress syndrome treatment, maximizing recruitment and gas exchange while minimizing ventilator-associated lung injury. With supportive treatment, mortality is usually less than 10%, with complete resolution of pulmonary and neurologic abnormalities. If the emboli pass through pulmonary vasculature, they can cause brain and kidney injury. Early long- bone fi xation has resulted in a nearly 5-fold decrease in resultant FES (10–13).

9. The answer is C. This patient has compartment syndrome of his extremity. Compartment syndrome results from increased content within a closed musculofascial compartment. This can be due to edematous structures within the compartment (e.g., crushed muscle) or hemorrhage into or around these structures. Delay in treatment can result in severe ischemia and tissue loss. Typical symptoms are pain out of proportion to exam fi ndings, numbness or tingling of the extremity, pain with passive dorsifl exion, and sensory defi cit. Arterial pressure is signifi cantly higher than the elevated compartment pressure, so distal pulses are present unless there is concomitant arterial injury. Normal pressure is up to 8 mmHg. A pressure >30 mmHg is consistent with impaired tissue perfusion (14).

10. The answer is D. The so-called lover’s triad is what can occur after jumping from an upper fl oor window. The typical triad consists of calcaneous fractures, lumbar vertebral spine compression fracture, and forearm fractures. Any mechanism involving an axial load warrants a search for these injuries (15).

11. The answer is C. Acute massive hemothorax following trauma is an indication for an early thoracotomy to control intrathoracic hemorrhage. Indications include initial chest tube output exceeding 1,500 mL or a continued hourly output of greater than 200 mL for 4 consecutive hours. Delay in intervention is associated with a higher mortality due to the volume of blood loss. The source of bleeding can be lung parenchyma, heart, or any of the great vessels. Residual pneumothorax on chest radiograph may indicate a need for an additional chest tube. Pulmonary contusion on imaging associated with desaturation may indicate a need for optimization with nonconventional mechanical ventilation (16).

12. The answer is B. This patient has abdominal compartment syndrome (ACS). Intra- abdominal hypertension is defi ned by intra-abdominal pressure (IAP) >12 mmHg, and ACS is defi ned as IAP >20 mmHg associated with new onset single or multiple system organ failure. Treatment of ACS is decompressive laparotomy. Angiography would be appropriate if there were ongoing transfusion requirements; however, her hemoglobin is stable and further transfusion is not necessary. Continuous venovenous hemofi ltration would improve creatinine; however, the renal failure is due to decreased renal perfusion, and once the abdominal pressure is relieved, the kidneys will again be well perfused and will return to normal function. Increased plateau airway pressures are secondary to elevated abdominal pressures. There is no role for bronchoscopy (8,17).

13. The answer is D. Nonoperative management of blunt adult hepatic and splenic injuries is the treatment modality of choice in hemodynamically stable patients, irrespective of the grade of injury. This is associated with a low overall morbidity and mortality and does

379

66485457-66485438 www.ketabpezeshki.com

ZZakaria_87574_PTR_CH25_10-06-13_371-382.inddakaria_87574_PTR_CH25_10-06-13_371-382.indd 379379 66/19/2013/19/2013 4:47:554:47:55 PMPM GENERAL CRITICAL CARE: PATHOLOGY, PATHOPHYSIOLOGY, AND THERAPY

not result in increased lengths of stay, transfusion requirement, or bleeding complications when compared to operative management. Neither grade of injury nor degree of hemo- peritoneum on CT scan predicts the outcome of nonoperative management. The hemody- namic status is the most reliable criterion. The presence of a contrast blush on the vascular phase of the CT scan may portend failure of nonoperative management. In this situation, angiography and embolization may be used to successfully control ongoing hemorrhage in the hemodynamically stable patient. There is no evidence to support the practice of keeping a clinically stable patient on bed rest. There is no evidence that serial abdominal CT scans are needed to follow up the injury in a clinically stable patient without other reasons for a scan. Selective cases of penetrating injury without peritoneal signs and CT evaluation of trajectory indicating no additional injuries can be managed nonoperatively with either endovascular or percutaneous interventions to facilitate management (18,19).

14. The answer is A. An estimated 20% of trauma patients who die in the prehospital setting are thought to have sustained signifi cant cardiac injury. Of those who arrive to the hos- pital alive, BCI is suspected in those with a high-energy mechanism and is often seen in conjunction with multiple injuries. In a conscious patient, complaints of chest pain should raise suspicion. In an obtunded patient, worrisome signs include refractory hypotension, muffl ed heart sounds, and jugular venous distension. Associated injury patterns include chest bruising, multiple rib fractures, and sternul fracture. A trauma FAST (focused assess- ment with sonography for trauma) exam can evaluate the pericardium for fl uid. An EKG should be obtained. A normal EKG and normal troponin I level are associated with a nega- tive workup, and no further tests are indicated (8,20).

15. The answer is A. This patient is septic from a missed bowel injury. An upright KUB should demonstrate free air underneath the diaphragm. The only appropriate interven- tion at this time is exploratory laparotomy and resection of the bowel injury. This patient had previously been managed nonoperatively, which is an acceptable management plan in select stab wounds without fascial penetration on local exploration and no peritoneal signs on exam. One large study reported a 2.9% rate of missed injuries that required sub- sequent surgery. Some delayed diagnoses do not add signifi cant morbidity, but bowel injuries can occasionally lead to major septic complications (21).

16. The answer is A. Mafenide acetate, silver sulfadiazine, and silver nitrate are the three most common topical antimicrobials used in burn wounds treatment. Mafenide acetate (Sulfamylon) has good eschar penetration, but is quite painful. It is highly effective against Gram-negative organisms, including most Pseudomonas species. It is an inhibitor of car- bonic anhydrase and, therefore, can cause signifi cant metabolic acidosis. Silver sulfadi- azine (Silvadene) has limited eschar penetration but is painless. It occasionally induces neutropenia via bone marrow suppression. Silver nitrate also does not have good eschar penetrance and is therefore painless. Due to leaching of electrolytes across the eschar, it can cause signifi cant electrolyte abnormalities (22).

17. The answer is A. High-voltage burns have a very different treatment regimen than do thermal burns. Resuscitation requirements are often twice those of the Parkland formula recommendation for thermal burns, and renal failure from myoglobinemia is a signifi cant danger. Prevention of acute renal failure requires prompt aggressive fl uid resuscitation

380

66485457-66485438 www.ketabpezeshki.com

ZZakaria_87574_PTR_CH25_10-06-13_371-382.inddakaria_87574_PTR_CH25_10-06-13_371-382.indd 380380 66/19/2013/19/2013 4:47:554:47:55 PMPM GENERAL TRAUMA AND BURNS: Answers

and maintenance of polyuria greater than or equal to 1.5 to 2 mL/kg/hour. Some authors also advocate urine alkalinization to a goal pH greater than 6.5; however, this recommen- dation is not consistent. All patients with burns should receive tetanus toxoid, plus teta- nus immunoglobulin if the last booster is unknown or greater than 10 years old (23–25).

18. The answer is E. In addition to elevated myoglobin from rhabdomyolysis, patients with electrical burns have signifi cant risk for cardiac arrhythmias, and therefore require con- tinuous cardiac monitoring. Due to muscle damage and resultant edema with aggres- sive resuscitation, this patient will most likely develop compartment syndrome of both forearms requiring operative fasciotomies. This patient fell 25 feet and therefore is at a signifi cant risk for extremity and spine fractures (25,26).

19. The answer is B. Inhalation injury is one of the most critical types of thermal injury. Inhalation injury occurs with prolonged smoke exposure in an enclosed space and not usually with fl ash burns such as experienced by this patient. Head and neck cutaneous burns can cause swelling that endangers the airway; however, the patient has minimal facial burns. It is important to follow the carboxyhemoglobin level because both partial pressure of oxygen and oxygen saturations may be normal despite oxygen displacement by carbon monoxide. The correct Parkland formula is 4 ml fl uid x body weight (kg) x % total BSA burn (27).

20. The answer is B. In circumferential torso burns, escharotomies may be necessary to relieve chest wall restriction and improve ventilation. This procedure can be performed bedside with IV sedation and electrocautery (28).

References

1. Como JJ, Diaz JJ, Dunham CM, et al. Practice management guidelines for identifi ca- tion of cervical spine injuries following trauma: update from the eastern associa- tion for the surgery of trauma practice management guidelines committee. J Trauma. 2009;67(3):651–659. 2. Bromberg WJ, Collier BC, Diebel LN, et al. Blunt cerebrovascular injury practice man- agement guidelines: the Eastern Association for the Surgery of Trauma. J Trauma. 2010;68(2):471–477. 3. Rhee P, Inaba K. Coagulopathy in the trauma patient. In: Cameron JL, ed. Current Surgical Therapy. 9th ed. Philadelphia, PA: Mosby/Elsevier; 2008:940–944. 4. Moore EE, Mattox KL, Feliciano DV. Management of shock. In: Trauma Manual. 4th ed. New York, NY: McGraw-Hill; 2003:71–83. 5. Moreno PL. The ICU Book. 2nd ed. Philadelphia, PA: Lippincott Williams & Wilkins, 1998:154–165. 6. Cinat ME, Hoyt DB. Hemorrhagic shock. In: Gabrielli A, Layon AJ, Yu M, eds. Civetta, Taylor & Kirby’s Critical Care. 4th ed. Philadelphia, PA: Wolters Kluwer; 2009:893–923. 7. Zarzaur BL, Croce MA. The management of vascular trauma. In: Cameron JL, eds. Current Surgical Therapy. 9th ed. Philadelphia, PA: Mosby/Elsevier; 2008:1028–1032. 8. Winfi eld RD, Lottenberg L. Secondary and tertiary triage of the trauma patient. In: Gabrielli A, Layon AJ, Yu M, eds. Civetta, Taylor & Kirby’s Critical Care. 4th ed. Philadelphia, PA: Wolters Kluwer; 2009:1109–1128.

381

66485457-66485438 www.ketabpezeshki.com

ZZakaria_87574_PTR_CH25_10-06-13_371-382.inddakaria_87574_PTR_CH25_10-06-13_371-382.indd 381381 66/19/2013/19/2013 4:47:554:47:55 PMPM GENERAL CRITICAL CARE: PATHOLOGY, PATHOPHYSIOLOGY, AND THERAPY

9. Cryer HG. Pelvic fractures. In: Cameron JL, ed. Current Surgical Therapy. 9th ed. Philadelphia, PA: Mosby/Elsevier; 2008:1037–1042. 10. Stern D, Pollack A, Scalea TM. Orthopedic critical care. In: Gabrielli A, Layon AJ, Yu M, eds. Civetta, Taylor & Kirby’s Critical Care. 4th ed. Philadelphia, PA: Wolters Kluwer; 2009:1279–1289. 11. Sungur M, Uzun K. Other embolic syndromes (air, fat, amniotic fl uid). In: Gabrielli A, Layon AJ, Yu M, eds. Civetta, Taylor & Kirby’s Critical Care. 4th ed. Philadelphia, PA: Wolters Kluwer; 2009:2159–2170. 12. Moore EE, Mattox KL, Feliciano DV. Injury to the lung and pleura. In: Trauma Manual. 4th ed. New York, NY: McGraw-Hill; 2003:188–194. 13. Moore EE, Mattox KL, Feliciano DV. Injury to the esophagus, trachea and bronchus. In: Trauma Manual. 4th ed. New York, NY: McGraw-Hill; 2003:178–187. 14. Feliciano DV. The management of extremity compartment syndrome. In: Cameron JL, ed. Current Surgical Therapy. 9th ed. Philadelphia, PA: Mosby/Elsevier; 2008:1032–1037. 15. Scaletta TA, Schaider JJ. Emergent Management of Trauma. 2nd ed. New York, NY: McGraw- Hill; 2001:336. 16. Gruen RL, Jurkovich GJ. Hemothorax. In: Cameron JL, ed. Current Surgical Therapy. 9th ed. Philadelphia, PA: Mosby Elsevier; 2008:705–708. 17. Hojman J, Rabinovici R. Abdominal compartment syndrome. In: Cameron JL, ed. Current Surgical Therapy. 9th ed. Philadelphia: Mosby/Elsevier; 2008:970–975. 18. Demetriades D, Hadjizacharia P, Constantinou C, et al. Selective nonoperative manage- ment of penetrating abdominal solid organ injuries. Ann Surg. 2006;244(4):620–628. 19. Como JJ, Bokhari F, Chiu WC, et al. Practice management guidelines for selective nonop- erative management of penetrating abdominal trauma. J Trauma. 2010;68(3):721–733. 20. Haut ER. Blunt cardiac injury. In: Cameron JL, ed. Current Surgical Therapy. 9th ed. Philadelphia, PA: Mosby/Elsevier; 2008:1063–1066. 21. Moore EE, Mattox KL, Feliciano DV. Indications for celiotomy. In: Trauma Manual. 4th ed. New York, NY: McGraw-Hill; 2003:214–230. 22. Richards WT, Mozingo DW. Burn injury: thermal and electrical. In: Gabrielli A, Layon AJ, Yu M, eds. Civetta, Taylor & Kirby’s Critical Care, 4th ed. Philadelphia, PA: Wolters Kluwer; 2009:1313–1324. 23. Mlcak RP, Buffalo MC. Pre-hospital management, transportation, and emergency care. In: Herndon DN, ed. Total Burn Care. 3rd ed. Philadelphia, PA: Saunders; 2007:81–92. 24. Bagshaw SM, Bellomo R. Acute renal failure. In: Gabrielli A, Layon AJ, Yu M, eds. Civetta, Taylor & Kirby’s Critical Care. 4th ed. Philadelphia, PA: Wolters Kluwer; 2009:2381–2392. 25. Spence RJ. Electrical and lightning injuries. In: Cameron JL, ed. Current Surgical Therapy. 9th ed. Philadelphia, PA: Mosby/Elsevier; 2008:1079–1083. 26. Purdue GF, Arnoldo BD, Hunt JL. Electrical injuries. In: Herndon DN, ed. Total Burn Care. 3rd ed. Philadelphia, PA: Saunders; 2007:513–520. 27. Nugent N, Herndon DN. Diagnosis and treatment of inhalation injury. In: Herndon DN, ed. Total Burn Care. 3rd ed. Philadelphia, PA: Saunders, 2007:262–272. 28. Gamelli RL, Silver GM. Burn wound management. In: Cameron JL, ed. Current Surgical Therapy. 9th ed. Philadelphia, PA: Mosby/Elsevier; 2008:1066–1071.

382

66485457-66485438 www.ketabpezeshki.com

ZZakaria_87574_PTR_CH25_10-06-13_371-382.inddakaria_87574_PTR_CH25_10-06-13_371-382.indd 382382 66/19/2013/19/2013 4:47:554:47:55 PMPM 26 Ethical and Legal Aspects of Critical Care Medicine Nasiya Ahmed QUESTIONS

1. Which one of the following is not one of the four ethical principles used to balance decision making in a diffi cult situation? A. Benefi cence (do good) B. Justice (fairness) C. Respect (for moral and religious values) D. Autonomy (the right to self-determination) E. Nonmalefi cence (do no harm)

2. A 61-year-old woman with a past medical history of hypertension, hyperlipidemia, dia- betes, and chronic kidney disease presented to the ED with a subarachnoid hemorrhage and acute renal failure that has worsened during the course of her hospitalization. While she has been cooperative with procedures up until this point, she is adamantly refusing dialysis. What should you do next? A. Sign a two-physician consent for emergent dialysis B. Ask the family to consent for dialysis C. Assess her decision-making capacity D. Consult hospice services E. Continue your current plan of care and call an ethics consult

ANSWERS TO THIS SECTION CAN BE FOUND ON PAGE 387 383

66485457-66485438 www.ketabpezeshki.com

ZZakaria_87574_PTR_CH26_10-06-13_383-390.inddakaria_87574_PTR_CH26_10-06-13_383-390.indd 383383 66/19/2013/19/2013 4:48:084:48:08 PMPM GENERAL CRITICAL CARE: PATHOLOGY, PATHOPHYSIOLOGY, AND THERAPY

3. A 75-year-old nursing home resident with a past medical history of hypertension and hyperlipidemia has been admitted with a stroke that has resulted in left upper extrem- ity weakness and diffi culty swallowing. On physical exam, he is extremely cachectic. However, the patient is alert, oriented, and able to make his own decisions. On morning rounds, you discuss placing a feeding tube to help him eat and he adamantly refuses. Which ethical principle is he using? A. Benefi cence B. Justice C. Respect D. Autonomy E. Nonmalefi cence

4. Which of the following statements about informed consent is/are incorrect? A. Double framing allows patients to see the risk–benefi t analysis from both perspectives B. Informed consent is not needed in emergency circumstances C. During the process of informed consent, the physician should disclose to the patient the diagnosis and the risks and benefi ts of the various treatment options D. Informed consent is required for all human subject research E. The key to informed consent is communicating and establishing a relationship with the patient

5. Which of the following statements about decision makers is/are correct? A. A durable power of attorney is appointed by the patient and/or close family members to make decisions for him or her in the event that the patient is unable to B. Spouse, parents, children, and siblings, in this order, can become surrogate decision makers C. In many cultures, the family retains decision-making authority for a sick patient who has intact decision-making capacity, and physicians should respect this tradition D. Advance directives or living wills allow physicians to make decisions for the patient without permission from the durable power of attorney E. Patients for whom a surrogate decision maker cannot be found will require a court- appointed guardian to make decisions on his or her behalf

6. Which situation best describes substituted judgment? A. The medical power of attorney is unable to make a decision and appoints someone to do it on his or her behalf B. A person does not have an advance directive, so the family is asked to make decisions based on his religious, personal, or moral values C. A patient does not have a surrogate decision maker and cannot have a court-appointed guardian due to illegal immigration status, so the physician is required to make deci- sions for the patient D. A patient has replaced his written advance directives with a recently created oral advance directive heard by only a few family members, who must carry out his wishes

384

66485457-66485438 www.ketabpezeshki.com

ZZakaria_87574_PTR_CH26_10-06-13_383-390.inddakaria_87574_PTR_CH26_10-06-13_383-390.indd 384384 66/19/2013/19/2013 4:48:084:48:08 PMPM ETHICAL AND LEGAL ASPECTS OF CRITICAL CARE MEDICINE: Questions

7. Which statement about futility is incorrect? A. Futile interventions often increase the patient’s pain and discomfort in the fi nal days of life B. While patients have autonomy and the right to decide their plan of care, it is the physi- cian’s responsibility to not offer futile treatment options C. Deeming a treatment futile should be based on solid research and evidence-based medicine D. Occasionally, futile interventions may be continued to help the family come to terms with the situation E. A treatment that confers a physiological benefi t on a patient is not futile

8. Which of the following statements about terminal extubation is incorrect? A. In terminal extubation, the positive end expiratory pressure and the respiratory rate are weaned to normal, prior to removing the tube and placing the patient on oxygen B. After a patient is extubated he or she may live anywhere from minutes to days C. Terminal extubation is an acceptable form of euthanasia D. Low-dose paralytics can be used to prevent respiratory distress and agitation during extubation process E. Symptom control with opiates and benzodiazepines should be initiated prior to extu- bation unless the patient is comatose

9. Mrs. Smith is a 79-year-old white woman who has been in the Neurocritical Care Unit for 2 weeks after a hemorrhagic stroke. After a lengthy discussion with the family, they agree to terminal extubation. The nurse starts morphine at a rate of 1 mg/hour and extubates the patient. The family complains that the patient is having diffi culty breathing. Which of the following medications will not help improve her breathing? A. Glycopyrolate 0.25 mg subcutaneous injection B. Scopolamine patch C. Ativan 0.5 mg IV D. Morphine 2 mg IV E. Lasix 40 mg IV

10. Palliative care: A. Is the total care of patients, controlling pain and other symptoms, while minimizing emotional, social, and spiritual issues in a hospice setting B. Maintains quality of life, but may hasten death C. Is available to patients with AIDS or endstage cardiac, renal, pulmonary, and neuro- logic disease, regardless of how long they have to live D. Is mostly done in the ambulatory setting, usually when there are no additional treat- ment options available for the patient

11. Which of the following is not a part of the signs and symptoms of an actively dying patient? A. Fever B. Delirium C. Respiratory distress D. Skin changes E. Nausea and vomiting

385

66485457-66485438 www.ketabpezeshki.com

ZZakaria_87574_PTR_CH26_10-06-13_383-390.inddakaria_87574_PTR_CH26_10-06-13_383-390.indd 385385 66/19/2013/19/2013 4:48:084:48:08 PMPM GENERAL CRITICAL CARE: PATHOLOGY, PATHOPHYSIOLOGY, AND THERAPY

12. Which one of the following is not one of the six steps used in communicating bad news? A. Preparation B. Apologizing prior to telling bad news C. Determining what the patient would like to know D. Responding to the patient’s feelings E. Discussing treatment options

13. Sources of confl ict in a family meeting include which of the following? A. Inadequate or incorrect information B. Emotions, especially guilt and anger C. Family dynamics D. Physician mistrust E. All of the above

14. The United States spends the most per capita on health care of any developed country. If this trend continues, then Medicare will no longer be able to provide care for its members. Methods to prevent this include: A. Rationing B. Health care tax C. Cost containment D. Increasing Medicaid funding

15. Rationing care violates which one of the four ethical principles? A. Benefi cence (do good) B. Justice (fairness) C. Autonomy (the right to self-determination) D. Nonmalefi cence (do no harm)

386

66485457-66485438 www.ketabpezeshki.com

ZZakaria_87574_PTR_CH26_10-06-13_383-390.inddakaria_87574_PTR_CH26_10-06-13_383-390.indd 386386 66/19/2013/19/2013 4:48:084:48:08 PMPM 26

ANSWERS

1. The answer is C. The four principles, adapted from the Belmont report, do not include respect. These four principles help highlight diffi cult aspects of a case. Determining the role of each of the four principles from the physician’s perspective as well as from the patient’s perspective helps highlight diffi cult aspects of a case. Oftentimes, there is a con- fl ict not only between these principles but also within one of the principles.

2. The answer is C. Decisional capacity refers to the patient’s ability to make a decision. In order to have decision-making capacity, a patient must be able to describe the medical situation, describe the risks and benefi ts of various treatment options, and arrive at a logi- cal and consistent decision. Depending on patients’ cognitive status, they may not have decisional capacity to make all their decisions; for example, the patient may be able to consent to an arterial line being placed, but not fully understand the concept and the need for dialysis to be able to consent to it. The family cannot consent to dialysis if the patient has decision-making capacity. There is no indication that the dialysis is emergent, requir- ing a two-physician consent or withholding informed consent. There is no indication that the need for dialysis is permanent; therefore, the patient would be unlikely to meet hos- pice criteria for end-stage renal disease. Although you should continue your plan of care while the dialysis issue is sorted out, the only reason for an ethics consult is if other treat- ments are futile without dialysis.

3. The answer is D. This is based on the principle of autonomy. If the patient has decision- making capacity, providers have a duty to respect his or her decision.

4. The answer is D. Informed consent is required for all procedures, and to perform a proce- dure without informed consent is considered battery, regardless of the outcome. The goal of informed consent is to educate the patient (inform the patient) so that he or she can make

387

66485457-66485438 www.ketabpezeshki.com

ZZakaria_87574_PTR_CH26_10-06-13_383-390.inddakaria_87574_PTR_CH26_10-06-13_383-390.indd 387387 66/19/2013/19/2013 4:48:084:48:08 PMPM GENERAL CRITICAL CARE: PATHOLOGY, PATHOPHYSIOLOGY, AND THERAPY

a voluntary and intelligent decision. The physician should disclose the patient’s medical condition, the risks and benefi ts of the treatment option, as well as the risks and benefi ts of other treatment/nontreatment options. The risks and benefi ts should be based on sound medical research. The framing of information can signifi cantly affect decision making, and framing bias should be minimized using double framing; for example, the physician explains that there is a 70% chance that surgery will benefi t the patient, which means there is a 30% chance that there will be no benefi t from surgery. Not all human subject research requires informed consent.

5. The answer is E. Advance directives and living wills allow the patient to carry out his/ her treatment wishes in the event that he or she is not able to do so; in essence, advance directives allow patients to preserve their autonomy. These documents can be simple, stating that the patient does not want to have CPR or intubation, to extremely complex situations such as when artifi cial nutrition and dialysis may be permitted; laws dif- fer from state to state on what can be stated in an advance directive. Some laws also allow for oral advance directives with witnesses present; an oral advance directive can replace a previously written advance directive. A durable or medical power of attorney is a person who is legally appointed by the patient to make medical decisions for him or her in the event that he or she is unable to do so. If the patient has both a medical power of attorney and an advance directive, the medical power of attorney should fol- low the advance directive when making decisions for the patient, but this often does not happen, as the medical power of attorney does have the right to make the fi nal decision. If there is no legal medical power of attorney, a surrogate decision maker is chosen from the family; the hierarchy for selecting a surrogate is spouse, children, par- ents, siblings, and religious clergy (some states allow for other relatives prior to using religious clergy). If there is more than one child or sibling, it is best to reach a consensus or majority agreement. Not all states recognize same sex partners or common law part- ners as spouses. If there is no family available, then a court-appointed guardian must be chosen as a surrogate decision maker. Oftentimes, guardianship takes time and a two-physician consent or religious clergy is used for more emergent procedures. Finally, there are many cultures in which the family prefers to make decisions for the patient; it is the physician’s duty to ask the patient how much he or she would like to be involved in the decision-making process.

6. The answer is B. Often patients do not have advance directives, especially if they are young. The family should then make a decision in the best interest of the patient, based on his or her religious, moral, and personal values. This is called substituted judgment.

7. The answer is E. The physician’s goal is to care for sick people. While patients have auton- omy in deciding their plan of treatment, a physician should only offer the patient options that can improve the patient’s clinical state or quality of life and not futile interventions. A procedure that may improve a patient physiologically but does not provide a noticeable benefi t to the patient is futile. Futility should be based on sound research and standard of care practices. Futile interventions often increase the patient’s pain and discomfort in the fi nal days of life. Oftentimes family will request procedures that are futile; in such situa- tions, the physician should communicate openly with the family and discuss why certain treatment options were withheld because of futility. Occasionally, futile interventions may

388

66485457-66485438 www.ketabpezeshki.com

ZZakaria_87574_PTR_CH26_10-06-13_383-390.inddakaria_87574_PTR_CH26_10-06-13_383-390.indd 388388 66/19/2013/19/2013 4:48:084:48:08 PMPM ETHICAL AND LEGAL ASPECTS OF CRITICAL CARE MEDICINE: Answers

be continued to help the family come to terms with the situation; for example, patients in a comatose or vegetative state may be kept on a ventilator for a few days in hopes of improvement.

8. The answer is D. The physician and the family must jointly agree that the ventilator has become a futile intervention for the patient and that ventilator withdrawal or terminal extubation is a better option; this agreement should be documented in the chart. It is the physician’s job to explain in detail the extubation procedure as well as the types of treat- ment that will be initiated for symptom management. The physician should encourage the family to choose a time for all to be present and make arrangements for religious and cultural rituals. Furthermore, the physician must explain to the family that the patient may not pass away immediately. The time between extubation and death is a diffi cult time for the family and they often doubt their decision; the physician should be supportive during this period. Two methods for terminal extubation have been described. The fi rst, immedi- ate extubation, involves removing the endotracheal tube and replacing it with humidifi ed oxygen, while the second, terminal weaning, involves tapering the respiratory rate, oxygen saturation, and positive end expiratory pressure over time and removing the endotracheal tube if the patient survives the weaning process; the choice is determined by the patient’s clinical presentation and the health care team’s preferences. All life-sustaining treatment, including nutrition, hydration, paralytics, and blood pressure support, should be stopped prior to extubation. Symptom control with morphine and midazolam or propofol should be initiated prior to extubation, even if the patient is comatose.

9. The answer is E. Both scopolamine and glycopyrolate will help dry out secretions. Morphine will decrease the perceived work of breathing, allowing the patient to breathe at a normal rate and pattern. Benzodiazepines will help minimize anxiety that may cause an increased respiratory rate. Lasix will benefi t only if there is pulmonary edema, which is not mentioned in this case. Depending on the patient’s cognitive and functional status, nonpharmacologic means of treating shortness of breath include elevating the head of the bed and using a fan.

10. The answer is C. The defi nition of palliative care is the total care of patients, control- ling pain and other symptoms, while minimizing emotional, social, and spiritual issues, whether at home or in a clinic, hospital, or hospice. Palliative care maintains quality of life while neither hastening nor postponing death. It is available to patients with AIDS and end-stage cardiac, renal, pulmonary, and neurologic disease, regardless of how long they have to live. Most hospitals have palliative care consult services; there are also palliative care clinics and home visit physicians. In the hospital palliative care, physicians should be consulted early on; they will help determine goals of care and provide pain and symptom management recommendations that will improve the patient’s intervention and treatment experience. While palliative care physicians can follow patients in hospice and eventually death, not all patients needing palliative care are hospice patients.

11. The answer is E. Fever often occurs with pneumonia or other infections at the end of life; it also occurs with liver infi ltration due to malignancy. Terminal delirium is the agitation that may occur as death approaches; it can be due to pain, constipation, urinary reten- tion, side effects of medication, or simply anxiety associated with the impending death.

389

66485457-66485438 www.ketabpezeshki.com

ZZakaria_87574_PTR_CH26_10-06-13_383-390.inddakaria_87574_PTR_CH26_10-06-13_383-390.indd 389389 66/19/2013/19/2013 4:48:084:48:08 PMPM GENERAL CRITICAL CARE: PATHOLOGY, PATHOPHYSIOLOGY, AND THERAPY

Respiratory distress, Cheyne–Stokes breathing, or gurgling, (often called the death rattle) also occurs as death nears. Breathing becomes apneic and the body is no longer able to clear secretions in a normal manner. As death approaches, a patient’s skin will become cold and take on a bluish, cyanotic tinge.

12. The answer is B. The SPIKES protocol is a systematic approach to breaking bad news to the family. S stands for settings or, where the conversation will take place; make sure it is a quiet room with no distractions, that all family members involved are present, and that you, the physician, are calm, well versed in the details of the case, and ready to listen. P stands for perception; always ask fi rst what the patient and his or her family know about the medical situation. I stands for invitation; ask the patient how much information he or she would like to know and how much he or she would like to participate in making treatment decisions. K stands for knowledge; always preface your bad news with a warn- ing, such as “Unfortunately, I have some bad news.” Do not use technical language, and give the information in repetitive small bits. E stands for empathy; listen and validate the patient’s emotions. S stands for summary; summarize the main points that were dis- cussed and the plan of care.

13. The answer is E. Before leading a family meeting, a physician should decide the goals of the meeting. The fi rst step to leading a family meeting is to determine if everyone present understands the current medical situation; the explanatory model can be used for this. It is important for the physician to give information in small pieces, acknowledge the fam- ily members’ emotions as they speak, and summarize the diagnosis and treatment plan at the end of the meeting. Gaps in information occur when the family doesn’t understand the patient’s situation or they are confused by conversations with multiple physicians and advice from well-meaning friends. Emotions, especially grief and fear due to the impending loss, guilt from having neglected their loved one, and hope, often contribute to the confl ict. Different family members, with differing value systems, complicate the situation. Last, if the family does not trust the physician, they are apt to disagree with the physician’s treatment plan. All of these factors play a role in the medical plan of care; these emotions are often grouped under denial, but when you can tease out the different factors, it helps to improve the situation as a whole.

14. The answer is C. Cost containment and the Patient Protection and Affordable Care Act signed by President Obama propose to decrease or reallot Medicare reimbursements as well as streamline insurance processes to save money and offer better health care to more people.

15. The answer is B. Rationing care, where two patients with the same medical diagnosis receive different levels of interventions, violates the principle of justice. If, however, the intervention is futile in one patient but not futile in another patient, then withholding a futile intervention is not rationing care.

Reference

1. Watson M, Lucas C, Hoy A, et al. Oxford Handbook of Palliative Care. New York, NY: Oxford University Press; 2005.

390

66485457-66485438 www.ketabpezeshki.com

ZZakaria_87574_PTR_CH26_10-06-13_383-390.inddakaria_87574_PTR_CH26_10-06-13_383-390.indd 390390 66/19/2013/19/2013 4:48:084:48:08 PMPM 27 Principles of Research in Critical Care Suur Biliciler and Justin Kwan QUESTIONS

1. A study is conducted to determine the dose-limiting toxicity of a drug for treatment of acute hemorrhagic stroke. What type of study is this? A. Preclinical testing B. Phase I trial C. Phase II trial D. Phase III trial

2. What is the main benefi t of the randomization process in a clinical trial? A. It always results in equal numbers of patients in each treatment arm B. It allows the clinical investigator to assign the patients to treatment arms based on cur- rent practice guidelines C. It always assigns the same number of men and women to each treatment arm D. It reduces the chance of bias in a study

3. A noninferiority trial A. Provides evidence that two treatments are completely identical B. Is an example of a placebo-controlled study C. Requires a smaller sample size than a superiority trial D. Determines whether the experimental therapy is equal or noninferior to the standard therapy

ANSWERS TO THIS SECTION CAN BE FOUND ON PAGE 394 391

66485457-66485438 www.ketabpezeshki.com

ZZakaria_87574_PTR_CH27_10-06-13_391-396.inddakaria_87574_PTR_CH27_10-06-13_391-396.indd 391391 66/19/2013/19/2013 4:48:194:48:19 PMPM GENERAL CRITICAL CARE: PATHOLOGY, PATHOPHYSIOLOGY, AND THERAPY

4. Which of the following National Institutes of Health (NIH)–funded grants do not have training as a signifi cant component of the grant? A. K99/R00 B. K08 C. R01 D. K23

5. The Health Insurance Portability and Accountability Act: A. Allows researchers to enroll patients in studies without their knowledge B. Allows researchers to share identifi able data from patients without their consent C. Is governed by the Privacy Rule to protect an individual’s medical information D. Provides funding for all National Institutes of Health (NIH)-sponsored research

6. What is an expedited institutional review board (IRB) review? A. The review of a protocol with minimal risk, as defi ned by U.S. Department of Health and Human Services, by a single reviewer B. The same as routine IRB review, except it is performed in less than an hour C. A review of research protocols performed in life-threatening conditions D. A review of research protocols that have a high probability of death

7. A study of the benefi ts of hypothermia in stroke compared the outcomes of 17 patients admitted with acute stroke and treated with hypothermic therapy with 56 age-, gender-, stroke severity–, and body temperature–matched patients from the stroke database is an example of a: A. Cohort study B. Randomized controlled study C. Cross-sectional study D. Case–control study

Table Reference for Questions 8 and 9

Disease No Disease

Positive a b Negative c d

8. In the 2 × 2 table, positive and negative test results are compared with the gold standard determining the presence and absence of disease. What does a/(a + c) represent? A. Positive predictive value B. Negative predictive value C. Sensitivity D. Specifi city

392

66485457-66485438 www.ketabpezeshki.com

ZZakaria_87574_PTR_CH27_10-06-13_391-396.inddakaria_87574_PTR_CH27_10-06-13_391-396.indd 392392 66/19/2013/19/2013 4:48:194:48:19 PMPM PRINCIPLES OF RESEARCH IN CRITICAL CARE: Questions

9. In the 2 × 2 table on the preceding page, what does d/(c + d) represent? A. Positive predictive value B. Negative predictive value C. Sensitivity D. Specifi city

10. In the stenting versus endarterectomy for treatment of carotid artery stenosis study, the absolute difference in the percentage of patients who had a stroke of any type in the two interventions during the peri-procedure period was 1.8 with a 95% confi dence interval of 0.4 to 3.2. What is the most appropriate interpretation of this data? A. The average difference in the percentage of strokes between the two treatments is 1.8 in 95 out of 100 patients if both interventions are used in secondary stroke prevention B. In 95% of patients treated with stenting, there is a 1.8% increase in strokes C. There is 95% confi dence that the actual difference in the percentage of strokes in the two treatments is 1.8 and the standard deviation is between 0.4 and 3.2 D. The absolute difference in the percentage of strokes in the two treatments in this study is 1.8. If the same study using the same methods was carried out in many independent samples of patients, the difference in the percentage of stroke between the two treat- ments would be between 0.4 and 3.2 in 95% of the samples, but would be outside this range 5% of the time

11. What is alpha or type 1 error? A. Failure to reject the null hypothesis when the null hypothesis is false B. Failure to reject the null hypothesis when the null hypothesis is true C. Rejection of the null hypothesis when the null hypothesis is false D. Rejection of the null hypothesis when the null hypothesis is true

12. What is the consequence of setting a small p value in a study? A. The probability of type 2 error will decrease B. The probability of type 1 error will increase C. The probability of false positive will decrease D. The probability of false negative will decrease

13. Two doses of a novel agent were used and compared to placebo in a randomized con- trolled trial for the treatment of acute intracranial hemorrhage with the p value for sta- tistical signifi cance set at .05. Both doses of the novel agent showed lower mortality at 3 months compared with placebo. The p value for the lower dose was .02 and the p value for the higher dose was .01. How can these results be interpreted? A. The higher dose is twice as effective as the lower dose B. The high dose is more effective in preventing mortality C. The results are not statistically signifi cant because the p value for this study was set at .05 D. Both drugs are effective treatments

393

66485457-66485438 www.ketabpezeshki.com

ZZakaria_87574_PTR_CH27_10-06-13_391-396.inddakaria_87574_PTR_CH27_10-06-13_391-396.indd 393393 66/19/2013/19/2013 4:48:194:48:19 PMPM 27

ANSWERS

1. The answer is B. The goal of a phase I clinical trial is to determine the highest dose of a drug that can be administered with an acceptable level of toxicity. The goal of phase II and phase III trials is to establish the effi cacy of a drug. Preclinical testing is the evaluation of candidate drugs in relevant animal species to establish the safety and biological activity of the drug prior to use of the agent in human studies (1).

2. The answer is D. The goals of the randomization process in a clinical trial are to assign patients to treatment arms while reducing the likelihood of introducing bias in to the study (2).

3. The answer is D. The goal of a noninferiority trial is to determine whether a novel treat- ment is no worse than the standard treatment. It is often used to establish new treatments that have fewer side effects, are easier to administer, are less invasive, or cost less. The noninferiority margin or difference in effi cacy between the treatment arms is established in advance so that the new treatment is preferred if the difference in effi cacy is less than the prespecifi ed noninferiority margin. Noninferiority studies require larger sample sizes than superiority studies due to the statistical design. An infi nite sample size is required to establish two treatments as truly identical, and this type of study is not feasible (3).

4. The answer is C. The K99/R00 is the NIH Pathway to Independence Award. The K99/ R00 award provides 5 years of support in two separate phases. The fi rst phase is a 2-year mentor phase for investigators who have not had more than 5 years of postdoctoral research training experience at the time of initial application or resubmissions. In the second phase, the investigator is an independent scientist at an extramural sponsor- ing institution in which the grant recipient has a tenure-track full-time faculty position. The K08 is a Mentored Clinical Scientist Research Career Development Award. The K23

394

66485457-66485438 www.ketabpezeshki.com

ZZakaria_87574_PTR_CH27_10-06-13_391-396.inddakaria_87574_PTR_CH27_10-06-13_391-396.indd 394394 66/19/2013/19/2013 4:48:194:48:19 PMPM PRINCIPLES OF RESEARCH IN CRITICAL CARE: Answers

is a Mentored Patient-Oriented Research Career Development Award. The difference between the K08 and K23 is the type of research the individual is conducting. The R01 is the NIH Research Project Grant Program to support a specifi c project performed by an independent investigator (4).

5. The answer is C. The Health Insurance Portability and Accountability Act requires all researchers to obtain permission from patients before using identifi able medical data for research. The U.S. Department of Health and Human Services uses the Privacy Rule to set the standard for the use and disclosure of patient medical information (5).

6. The answer is A. An IRB-expedited review is a review of a study protocol that can be performed by a single reviewer without the entire IRB committee, for studies that have minimal risk to the participants as defi ned by the U.S. Department of Health and Human Services (6).

7. The answer is D. A case–control study is a study comparing a group of subjects treated with a new intervention with a group of subjects who are not treated with the new inter- vention. The allocation of subjects to the treatment arms is not random. A cohort study is a study of a group of subjects followed over time. A randomized controlled study is one in which an intervention group is compared to a control group in which the subject assignment is determined by a formal randomization procedure. A cross-sectional study is a study in which the clinical measures of interest are all obtained at the same time or within a short period of time (7).

8. The answer is C. Sensitivity is the proportion of subjects with the disease who test posi- tive for the disease. Specifi city is the proportion of subjects without the disease who test negative for the disease. Positive predictive value is the proportion of subjects who test positive and actually have the disease. Negative predictive value is the proportion of subjects who test negative and do not have the disease (8).

9. The answer is B. Negative predictive value is the proportion of subjects who test negative and do not have the disease (8).

10. The answer is D. The confi dence interval gives a range of values that refl ects the precision of a sample estimate. A 95% confi dence interval of an estimate provides the range of val- ues such that if the experiment is repeated in multiple independent samples, the estimate will fall in the confi dence interval in 95% of the samples (9,10).

11. The answer is D. Type 1 error is when an investigator rejects a null hypothesis when the null hypothesis is in fact true in the population (11).

12. The answer is C. The p value represents the probability that the fi nding will occur by chance if the null hypothesis is actually true. A lower p value will result in a low probabil- ity of making a type 1 error but increase the probability of making a type 2 error (11).

13. The answer is D. The p value represents the probability that the fi nding will occur by chance if the null hypothesis is actually true. The p value has no information regarding the treatment effect (12).

395

66485457-66485438 www.ketabpezeshki.com

ZZakaria_87574_PTR_CH27_10-06-13_391-396.inddakaria_87574_PTR_CH27_10-06-13_391-396.indd 395395 66/19/2013/19/2013 4:48:194:48:19 PMPM GENERAL CRITICAL CARE: PATHOLOGY, PATHOPHYSIOLOGY, AND THERAPY

References

1. Kelly WK, Halabi S, Schilsky R, et al. Oncology Clinical Trials: Successful Design, Conduct and Analysis. New York, NY: Demos; 2010:21–28, 35–42. 2. Kelly WK, Halabi S, Schilsky R, et al. Oncology Clinical Trials: Successful Design, Conduct and Analysis. New York, NY: Demos; 2010:73–82. 3. Kelly WK, Halabi S, Schilsky R, et al. Oncology Clinical Trials: Successful Design, Conduct and Analysis. New York: Demos; 2010:101–108. 4. Offi ce of Extramural Research, NIH. www.grants.nih.gov. 5. US Department of Health and Human Services, www.hhs.gov. 6. Hulley SB, Cummings SR, Browner WS, et al. Designing Clinical Research: An Epidemiologic Approach, Chap 14. 2nd ed. Philadelphia, PA: Lippincott Williams & Wilkins; 2001: 225–239. 7. Kammersgaard LP, Rasmussen BH, Jørgensen HS, et al. Feasibility and safety of inducing modest hypothermia in awake patients with acute stroke through surface cooling: a case– control study: the Copenhagen Stroke Study. Stroke. 2000 Sep;31(9):2251–2256. 8. Hulley SB, Cummings SR, Browner WS, et al. Designing Clinical Research: An Epidemiologic Approach, Chap 12. 2nd ed. Philadelphia, PA: Lippincott Williams & Wilkins; 2001: 183–205. 9. Brott TG, Hobson RW 2nd, Howard G, et al. CREST Investigators. Stenting versus endar- terectomy for treatment of carotid-artery stenosis. N Engl J Med. 2010 Jul 1;363(1):11–23. Epub 2010 May 26. 10. Motulsky H. Intuitive Biostatistics: A Nonmathematical Guide to Statistical Thinking. 2nd ed. New York, NY: Oxford University Press; 2010:87–95. 11. Motulsky H. Intuitive Biostatistics: A Nonmathematical Guide to Statistical Thinking. 2nd Ed. New York, NY: Oxford University Press; 2010:122–129. 12. Hulley SB, Cummings SR, Browner WS, et al. Designing Clinical Research: An Epidemiologic Approach, Chap 5. 2nd ed. Philadelphia, PA: Lippincott Williams & Wilkins; 2001;51–63.

396

66485457-66485438 www.ketabpezeshki.com

ZZakaria_87574_PTR_CH27_10-06-13_391-396.inddakaria_87574_PTR_CH27_10-06-13_391-396.indd 396396 66/19/2013/19/2013 4:48:194:48:19 PMPM 28 Procedural Skills and Monitoring George W. Williams QUESTIONS

1. A patient with severe peripheral vascular disease has a forehead pulse oximeter placed secondary to poor signal quality in his extremities. This oximeter operates primarily via which of the following principles? A. Absorption spectrophotometry B. Photoelectric spectrophotometry C. Mixed spectrophotometry D. Refl ectance spectrophotometry

2. A 65-year-old male, status post motor vehicle collision with bifrontal cerebral contusions, presents with a tense abdomen on examination. Which of the following could be used to assess gut perfusion? A. Femoral central line B. Foley catheter C. Ventilator driving pressure D. Arterial waveform

ANSWERS TO THIS SECTION CAN BE FOUND ON PAGE 404 397

66485457-66485438 www.ketabpezeshki.com

ZZakaria_87574_PTR_CH28_10-06-13_397-414.inddakaria_87574_PTR_CH28_10-06-13_397-414.indd 397397 66/19/2013/19/2013 8:49:048:49:04 PMPM GENERAL CRITICAL CARE: PATHOLOGY, PATHOPHYSIOLOGY, AND THERAPY

3. A bedside RN calls you to address signifi cant variability in a patient’s blood pressure. On assessment, you observe a clinical technician holding the patient’s arm still in order to facilitate acquisition of a blood pressure reading. Which of the following would explain the variable results in this case? A. False arterial pulsations from the arm-restraining clinical technician B. False Korotkoff sounds from the arm-restraining clinical technician C. Random oscillations from the arm-restraining clinical technician D. Agitation associated with signifi cant variability in blood pressure

4. During an application of the Advanced Cardiac Life Support protocol on an unstable patient, the medical student suggests that the pulse oximeter is inaccurate because the patient is hypotensive. Which is the lowest blood pressure at which a pulse oximeter is reliably accurate? A. 50 mmHg B. 40 mmHg C. 30 mmHg D. 30 mmHg, as long as there is no arterial line in the measured extremity

5. While placing a right subclavian central venous catheter, a lead can be removed while still maintaining the most sensitivity for arrhythmias during Seldinger wire placement. Which of the vector-based leads would meet this criterion? A. Right upper lead B. Left upper lead C. Left lower lead D. All of the above leads must be present

6. In order to maximize detection of left ventricular ischemia, what is the ideal position of the precordial lead monitored in the ICU? A. Midclavicular line, fi fth intercostal space B. Anterior axillary line, fi fth intercostal space C. Along left sternum, fourth intercostal space D. Along right sternum, fourth intercostal space

7. Hemodialysis catheters are generally 2-fold larger bore than comparable triple-lumen catheters. The primary rationale for this is to: A. Minimize the risk of venous stenosis following removal of the catheter B. Ensure a catheter fl ow rate of at least 100 mL/min C. Increase blood fl ow by 16-fold D. Prevent luminal thrombus

398

66485457-66485438 www.ketabpezeshki.com

ZZakaria_87574_PTR_CH28_10-06-13_397-414.inddakaria_87574_PTR_CH28_10-06-13_397-414.indd 398398 66/19/2013/19/2013 8:49:058:49:05 PMPM PROCEDURAL SKILLS AND MONITORING: Questions

8. A pulmonary arterial catheter (PAC) is placed under sterile conditions in a nonemer- gent setting. The following waveforms are noted on the monitor as the PAC is advanced. Which of the following is the most appropriate next step? A. Administer beta blockade B. Withdraw PAC C. Defl ate and infl ate balloon D. Administer fl uid bolus

Source: Image created by George Williams, MD.

9. Trendelenburg positioning is recommended while placing an internal jugular or subcla- vian central line primarily due to which of the following reasons? A. Ease of practitioner access to landmarks/anatomical structures B. To facilitate engorgement of target veins C. Reduction of ectopy/arrhythmia risks because of increased myocardial preload D. To reduce the complications of air embolism

10. You are caring for a patient with a medical history signifi cant for pulmonary hypertension and decide to place a pulmonary arterial catheter (PAC) to measure pulmonary systolic/ diastolic pressures and pulmonary arterial wedge pressure (PAWP). The accompanying chest x-ray (CXR) demonstrates the PAC placement (upright) with different directions that the catheter could potentially point as labeled. Which of the following statements is true? A. The pressure waveform on line A would refl ect left atrial pressure most accurately B. The pressure waveform on line B would refl ect left atrial pressure most accurately C. The pressure waveform on line C would refl ect left atrial pressure most accurately D. There are no signifi cant differences among the lines shown E. The PAC waveform cannot be accu- rate because of the right lower lobe pneumonia

399

66485457-66485438 www.ketabpezeshki.com

ZZakaria_87574_PTR_CH28_10-06-13_397-414.inddakaria_87574_PTR_CH28_10-06-13_397-414.indd 399399 66/19/2013/19/2013 8:49:058:49:05 PMPM GENERAL CRITICAL CARE: PATHOLOGY, PATHOPHYSIOLOGY, AND THERAPY

11. A patient with a history of systolic heart failure is placed on mechanical ventilation. Which of the following is an effect of adequate mechanical ventilation on the cardiovas- cular system? A. Increased preload B. Decreased afterload C. Decreased dead space D. Tachycardia

12. A standard thermometer probe placed in a patient’s bladder functions by which of the following mechanisms? A. Conductance B. Magnetic induction C. Compliance D. Photon absorption

13. A patient with a spinal abscess is admitted to the ICU in septic shock and requires pres- sors. Which of the following agents would be most effi cacious for increasing cardiac out- put while increasing mean arterial pressure (MAP)? A. Dopamine B. Phenylephrine C. Epinephrine D. Norepinephrine

14. Which of the following factors does not affect systemic vascular resistance (SVR)? A. Mean arterial pressure (MAP) B. Left ventricular end systolic volume C. Cardiac output D. Central venous pressure

15. Which of the following techniques may be used to prevent interference to an EEG recording? A. Use lighter electrodes B. Apply rigid electrode leads C. Use electrodes in which the metallic surface is as close to the skin as possible D. Minimize the volume of conducting jelly

16. You are called to the bedside of a patient who has an intracranial pressure (ICP) reading of 23 mmHg. The patient’s blood pressure is 99/78. A CT scan from 2 hours ago demon- strated diffuse cerebral edema. The accompanying waveform is shown on the monitor. What is the most appropriate next step in management? A. Administer mannitol 0.75 g/kg B. Elevate the head of the bed to 60° C. Repeat CT scan in 4 hours D. Increase sedation with fentanyl infusion

400

66485457-66485438 www.ketabpezeshki.com

ZZakaria_87574_PTR_CH28_10-06-13_397-414.inddakaria_87574_PTR_CH28_10-06-13_397-414.indd 400400 66/19/2013/19/2013 8:49:058:49:05 PMPM PROCEDURAL SKILLS AND MONITORING: Questions

17. A patient (status post left basal ganglia hemorrhagic stroke 2 days ago) has been on nasal cannula since admission, but now appears to be in respiratory distress. The patient has an

arterial blood gas that reveals: pH 7.39, PaCO2 41, PaO2 54, base excess −2. Which of the following would be the most appropriate choice for management of this patient? A. Amantadine B. Nasal tracheal suctioning C. Continuous positive airway pressure (CPAP) D. Bilevel positive airway pressure (BiPAP)

18. A 63-year-old patient with a Fisher grade 3 subarachnoid hemorrhage has an external ventricular drain (EVD) in place that is reporting an intracranial pressure (ICP) of 54 mmHg. Her blood pressure is 109/61. You look at the EVD and realize that the pressure transducer is mechanically fi xed 20 cm below the level of the circle of Willis. Which of the following most accurately refl ects the most appropriate interpretation of this EVD reading? A. The ICP is actually much higher B. The ICP is actually much lower and no intervention is necessary C. The ICP is actually much lower, but the patient’s cerebral perfusion pressure (CPP) remains <60 mmHg D. No accurate conclusion can be drawn since the transducer is not level

19. You are notifi ed of a patient having severe hypotension and diaphoresis secondary to hemorrhagic shock requiring intubation. The patient has an oxygen saturation of 93% on 4 L/minute nasal cannula, blood pressure of 74/30, and body mass index of 26 kg/m2. Which of the following would be the most appropriate order in which to proceed? A. Preoxygenate for 1 minute, perform a rapid sequence intubation B. Preoxygenate for 3 minutes, perform a normal sequence intubation C. Preoxygenate for 5 minutes, perform a normal sequence intubation D. Preoxygenate for 7 minutes, insert a nasogastric tube, perform a rapid sequence intubation

20. During an emergent intubation, you have performed direct laryngoscopy two times, with each attempt resulting in an esophageal intubation. Long-acting muscle relaxation has already been administered, and you are able to bag-mask the patient, although it is dif- fi cult. You were able to improve visualization of the glottis on the second attempt with towel rolls and cricoid pressure. Which of the following best describes the most appropri- ate next step? A. Perform a fi beroptic intubation B. Use a bougie C. Place a laryngeal mask airway (LMA) D. Call for help

401

66485457-66485438 www.ketabpezeshki.com

ZZakaria_87574_PTR_CH28_10-06-13_397-414.inddakaria_87574_PTR_CH28_10-06-13_397-414.indd 401401 66/19/2013/19/2013 8:49:058:49:05 PMPM GENERAL CRITICAL CARE: PATHOLOGY, PATHOPHYSIOLOGY, AND THERAPY

21. You are performing a conscious sedation with benzodiazepines on a patient having an external ventricular drain (EVD) placed. The patient’s oxygen saturation remains in the mid-90s but the patient is not responding to commands. You measure nasal cannula end-

tidal carbon dioxide (ETCO2) to assess the patient’s status; the ETCO2 is 16 mmHg and a respiratory rate of 12 breaths/minute. What does this result signify? A. The patient is likely appropriately sedated B. The patient should receive fl umazenil C. The patient is likely having a pulmonary embolism D. The patient should receive further sedation

22. A 34-year-old male (status post blunt head trauma 6 hours ago) has a poor neurologic examination. A jugular venous bulb catheter is placed. Which of the following best describes the most accurate technique for acquiring jugular venous bulb oxygen satura-

tion (SjvO2)? A. Right internal jugular B. Left internal jugular C. Use of a fi ber-optic catheter for continuous recording D. None of the above

23. You are caring for a trauma patient who has signifi cant bilateral contusions apparent on a recent head CT scan. The overnight team placed a jugular venous bulb oxygen saturation monitor and the value is elevated. Which of the following is the least likely cause of this elevation? A. Focal cerebral ischemia with surrounding hyperemia B. Decreased oxygen delivery C. Extracerebral contamination D. Blood draw rate of 0.2 mL/second

24. A 55-year-old male with a history of hypertension is now post-bleed day 5 following a Fisher grade 3 subarachnoid hemorrhage. Transcranial dopplers (TCDs) are ordered to determine the likelihood of vasospasm. Which of the following is not a factor or applica- tion that increases the clinical utility of TCDs? A. Lindegaard index B. High frequencies (10–12 Hz) C. Acoustic windows D. Brain death pattern

402

66485457-66485438 www.ketabpezeshki.com

ZZakaria_87574_PTR_CH28_10-06-13_397-414.inddakaria_87574_PTR_CH28_10-06-13_397-414.indd 402402 66/19/2013/19/2013 8:49:058:49:05 PMPM PROCEDURAL SKILLS AND MONITORING: Questions

25. A patient who has been admitted for 11 days, undergoing intermittent endovascular therapy for vasospasm following a moderate-grade subarachnoid hemorrhage, has been cleared by neurosurgery to have the external ventricular drain (EVD) weaned with the goal of removal. The EVD is currently at 10 cm and open. Which of the following is the safest and most expeditious method of weaning this EVD (with the EVD being reopened in the event of intracranial hypertension or hydrocephalus)? A. Clamp now, check a CT scan tomorrow B. Raise by 5 cm now and every day until 25 cm, then clamp, then perform a CT scan 24 hours later C. Raise to 20 cm now, clamp tomorrow, perform a CT scan 24 hours later only if patient has symptoms D. Raise to 20 cm now, clamp tomorrow, then perform a CT scan 24 hours later

26. In preventing meningitis in patients with external ventricular drains (EVDs), which of the following is most accurate? A. Catheter tunneling reduces long-term infection rates B. There is no correlation between catheter irrigation and infection rates C. Changing catheters every 5 days reduces infection rates D. Protocol development for insertion reduces infection rates

27. A patient returns from a procedure in the OR under general anesthesia and you attempt to acquire a postprocedure examination. Which of the following factors does not potenti- ate the drugs commonly given during anesthesia? A. Hypomagnesaemia B. Hypothermia C. Chronic renal insuffi ciency D. Gentamycin E. Hypernatremia

28. Which of the following factors most affects the respiratory quotient?

A. Fraction of inspired oxygen (FiO2)

B. Partial pressure of carbon dioxide in arterial blood (PaCO2)

C. Partial pressure of H2O D. Consumption of oxygen E. Nutritional status

403

66485457-66485438 www.ketabpezeshki.com

ZZakaria_87574_PTR_CH28_10-06-13_397-414.inddakaria_87574_PTR_CH28_10-06-13_397-414.indd 403403 66/19/2013/19/2013 8:49:058:49:05 PMPM 28

ANSWERS

1. The answer is D. This question is assessing knowledge of the mechanism by which pulse oximetry operates; this mechanism is spectrophotometry. Spectrophotometry allows transmission of light through a medium to determine its molecular composition (using the Lambert–Beer law). The most common type of pulse oximeter that is applied to a patient’s fi nger operates based on absorption spectrophotometry (light transmitted through a medium), whereas forehead pulse oximeters utilize refl ectance spectrophotom- etry. Answers B and C are distracters and have no clinical relevance (1).

2. The answer is B. Abdominal compartment syndrome (ACS) is a surgical emergency that should always be considered in a patient presenting with neurological trauma, especially when the patient is unable to express clinical distress secondary to neurological injury. Furthermore, ACS can serve to increase intracranial pressure (ICP) and impede venous return from the brain. Measuring intra-abdominal pressure (IAP) via the inferior vena cava is well described, but is invasive and not preferred; nonetheless, a femoral central line would not facilitate measurement of this value. Instilling as little as 20 mL of saline in an empty bladder with a Foley catheter in place can allow the intensivist to measure IAP accurately and is the most common means of measuring this variable. Ventilator driv- ing pressure can be expected to increase with ACS, though it is not a reliable means to measure IAP or determine the severity of ACS. While ACS impedes venous return to the heart, there is not a pathognomonic change appreciable on an arterial line (2).

3. The answer is C. Modern automated blood pressure cuffs do not utilize Korotkoff sounds, but oscillometry. Auscultatory blood pressure measurements have many downsides, including limited usefulness in the setting of shock or limited fl ow secondary to pressor administration, and lack of automation. With this system, the point of maximum oscilla- tion refl ects the mean arterial pressure (MAP). See the accompanying fi gure. If there is an

404

66485457-66485438 www.ketabpezeshki.com

ZZakaria_87574_PTR_CH28_10-06-13_397-414.inddakaria_87574_PTR_CH28_10-06-13_397-414.indd 404404 66/19/2013/19/2013 8:49:058:49:05 PMPM PROCEDURAL SKILLS AND MONITORING: Answers

aberrant oscillation (from pressing on the cuff) during the defl ation cycle, the software that calculates the MAP will potentially make errors in the calculation of the MAP, as well as systolic and diastolic blood pressures (see accompanying image). While agitation may result in increased blood pressure, it is not frequently associated with a high degree of blood pressure variability without pharmacologic intervention from sedative drugs (1).

Cuff In flation Pressure

MAP Oscillation measured by cuff

@2012 George Williams, MD

4. The answer is C. Pulse oximeters have been demonstrated to be accurate to a pressure of

30 mmHg (normal accuracy is within 3% of the actual SaO2). This accuracy is maintained even when taken distally to a cannulated radial artery (3).

5. The answer is B. The optimal lead for monitoring signal propagation through the cardiac conduction system is lead II, which best follows the natural course of this signal. Lead II is generated from the vector between the right upper and left lower leads, and the left upper lead is therefore not required to generate it. The accompanying illustration shows a schematic outlining of Einthoven’s triangle, demonstrating the relative vector direction in association with cardiac conduction system anatomy (4).

Right Upper Lead Left Upper Lead

Lead I

SA Node

*Lead II AV Node Lead III

Left Lower Lead © 2012 George Williams, MD

405

66485457-66485438 www.ketabpezeshki.com

ZZakaria_87574_PTR_CH28_10-06-13_397-414.inddakaria_87574_PTR_CH28_10-06-13_397-414.indd 405405 66/19/2013/19/2013 8:49:058:49:05 PMPM GENERAL CRITICAL CARE: PATHOLOGY, PATHOPHYSIOLOGY, AND THERAPY

6. The answer is B. All of the answers relate to positioning of the precordial leads; the leads listed include V1, V2, V4, and V5. V5 is the most sensitive lead for ventricular ischemia of the leads listed. The following list outlines the position of each precordial lead. Lead V1: along right sternum, fourth intercostal space Lead V2: left sternum, fourth intercostal space Lead V3: a line midway between V2 and V4 Lead V4: midclavicular line, fi fth intercostal space Lead V5: anterior axillary line, fi fth intercostal space Lead V6: midaxillary line, fi fth intercostal space It is important to note that each of these leads refl ects different components of the myocar- dium. Lead V5 demonstrates inferior, posterior, and a signifi cant portion of the anterior wall’s vascular territory. Many monitors default to a V5 label on the precordial lead for this purpose, though there are several that generically label the precordial lead as V1 (4).

7. The answer is C. The large-bore catheters used for hemodialysis should be able to sup- port blood fl ow between 200 and 300 mL/minute; the dialysate fl uid (opposite side of membrane) fl ows at 500 to 800 mL/minute. The primary reason for the 2-fold increase in size of the catheter, when compared to a triple lumen catheter, is the 16-fold increase in fl ow that is achieved (recall the Hagan–Poiseuille equation). • Resistance = (8 × µ × L) ÷ (π × r4), where L is the length of tubing, µ is the fl uid viscosity, and r is the tubing radius. Venous stenosis is correlated with the insertion site: subclavian insertion has the highest incidence of stenosis, reducing the eventual option of placing chronic access distal to the vein. The femoral vein has a higher rate of thrombosis when compared to the internal jugular vein, but patient comfort or cooperation frequently limits the option of placing internal jugular access; catheter maintenance (periodic fl ushing, etc.) is more important than catheter bore size in determining rates of thrombosis. It is important to note that increasing length will reduce fl ow through the catheter as well (5).

8. The answer is B. The waveform shown demonstrates a PAC being fl oated that is remain- ing in the right ventricle. The graph appears tachycardic from a resolution standpoint, but does not actually indicate an elevated heart rate. Defl ating and reinfl ating the balloon cannot be expected to reliably aid in advancing the catheter. Administering a fl uid bolus would help increase preload, but would not likely result in prompt movement of the cath- eter forward in the cardiac circulation; additionally, a bolus would take a relatively signifi - cant amount of time and could precipitate arrhythmias. The most appropriate action is to withdraw the PAC and reattempt insertion (6).

9. The answer is D. Normal Trendelenburg positioning is achieved at 10° to 30° tilt; however, factors such as moving unnecessary equipment, tilting the head, and shoulder rolls actu- ally improve the visualization of anatomical landmarks. Engorgement of the target veins is a clear benefi t of the Trendelenburg position; however, it is not the primary reason for this positioning from a patient safety perspective. The central venous line placement is highly risky for sudden cardiac death because of potential arrhythmias caused by irritation of the cardiac conduction system from the guide wire used to place the line; this is particularly important in patients with left bundle branch block in that the wire could precipitate a right

406

66485457-66485438 www.ketabpezeshki.com

ZZakaria_87574_PTR_CH28_10-06-13_397-414.inddakaria_87574_PTR_CH28_10-06-13_397-414.indd 406406 66/19/2013/19/2013 8:49:068:49:06 PMPM PROCEDURAL SKILLS AND MONITORING: Answers

bundle branch block, causing a complete heart block. Nonetheless, increased preload from Trendelenburg positioning has not been demonstrated to reduce the risk of ectopy. Air embo- lism may easily result from central line placement; a volume of 100 mL may be entrained through a 14-G catheter in 1 second. With increased venous pressures, air embolism becomes less likely. In the Trendelenburg position, if air embolism does occur, the likelihood of the air migrating to the brain is reduced when compared to 0° or greater elevation (6).

10. The answer is C. This question is actually asking about the distribution of the patient’s West zones. These theoretical zones are distributed based on gravity and determine the pressure relationships among the pulmonary arteries, pulmonary veins, and alveoli. In West zone 1, alveolar pressure exceeds pulmonary artery and pulmonary venous pressure. In West zone 2, pulmonary arterial pressure exceeds alveolar pressure, which exceeds pul- monary venous pressure. A PAC positioned in either West zone 1 or 2 will be susceptible to alveolar pressure, and measurements can be expected to refl ect alveolar or airway pressure rather than left atrial pressure. In West zone 3, pulmonary arterial pressure exceeds pul- monary venous pressure, which exceeds alveolar pressure; the tip of the PAC must lie in zone 3 for PAWP to accurately refl ect left atrial pressure. The CXR demonstrates a patient in the upright position; line C would most closely refl ect West zone 3 (because it is pointing downward) and is therefore the most accurate waveform. A small pneumothorax or lobar pneumonia should not signifi cantly derange PAC pressure waveforms (7).

11. The answer is B. Positive pressure ventilation has many effects on the cardiovascular sys- tem, with the commonly cited effect being reductions in preload secondary to increased intrathoracic pressures. It is important to remember that elevations in intrathoracic pres- sure, in effect, squeeze the ventricle and result in reductions in afterload. These combined effects result in the sometimes-observed fl ash pulmonary edema from ventilator discon- nection (secondary to the sudden increase in left ventricular afterload and right ventricular preload). Dead space generally increases with positive pressure ventilation because of the increase in the proportion of West zone 1 associated with alveolar distention (alveolar pres- sure > pulmonary arterial pressure > pulmonary venous pressure). Adequate ventilation should not be associated with tachycardia, which can be caused by feelings of dyspnea, anxiety, or a stress response from inadequate oxygen delivery to the myocardium (8).

12. The answer is A. Thermometer fi laments are metal and are appreciable on x-ray; this allows one to determine if a Foley catheter in place has a thermometer function available. The resistance of the metal increases with decreasing temperature, and vice versa in a pre- dictable fashion; this allows calculation of temperature. This technology and the materials used to make them are inexpensive. There is no magnetic or compliance component to commonly used temperature technology. Photon absorption, specifi cally in the infrared bandwidth, is commonly used by bedside nurses in the auricular cavity. However, these devices are not typically indwelling (9).

13. The answer is D. Dopamine is widely used as an inotrope in patients presenting in septic shock, but its inotropic effects (e.g., increase in cardiac output) do not necessarily result in increased MAP. There is heterogeneity with the hemodynamic response to dopamine depending on the underlying condition of the patient, including variable increases in sys- temic vascular resistance (SVR). Most of the increased blood pressure generally appreciated

407

66485457-66485438 www.ketabpezeshki.com

ZZakaria_87574_PTR_CH28_10-06-13_397-414.inddakaria_87574_PTR_CH28_10-06-13_397-414.indd 407407 66/19/2013/19/2013 8:49:068:49:06 PMPM GENERAL CRITICAL CARE: PATHOLOGY, PATHOPHYSIOLOGY, AND THERAPY

from dopamine results from increased cardiac stroke volume. Norepinephrine has been demonstrated to increase cardiac output to the same extent as dopamine while yield- ing an increase in MAPs; this is secondary to strong α-1 and β-1 adrenergic properties. Epinephrine is a potent inotrope (β-1 properties) but does not necessarily augment blood pressure to the same degree as norepinephrine in a consistent fashion across all dosing ranges. Phenylephrine is purely a sympathomimetic α-1 acting agent and therefore does not augment cardiac output (10).

14. The answer is B. SVR is a calculated number, and therefore is not directly measured. The formula for SVR is: SVR = [(MAP − CVP)/CO] × 80 As demonstrated, there is no factor included for ventricular volume.

15. The answer is A. To minimize interference during EEG recording, the electrodes are designed to fi xate to the scalp and provide minimal motion at both the skin–electrolyte and electrolyte–metal junctions. This is achieved by maximizing the amount of gel used; the gel creates an electrode–gel bridge in which maximal conduction occurs. As such, the electrodes are not ideally closer to the skin, but have more consistent contact. To maximize mechanical stability of the electrode and the skin in its vicinity, the physical load on the electrode should be kept as small as possible, and lightweight and fl exible electrode leads are applied (11).

16. The answer is A. The waveform shown here on the right demonstrates urgent intracranial

hypertension. This is demonstrated by p wave reversal. Normally, the P1 wave is the highest

pressure point (refl ecting cardiac pulsation), followed by P2, which is cerebral arterial recoil,

followed by P3, which refl ects cerebral venous engorgement. A P2 wave exceeding the P1 wave is refl ective of decreased cerebral compli- B P ance and urgent intracranial hyperten- 2 A P1 sion. Considering this, aggressive man- P2 agement must be pursed, which in this P P P 3 case would involve administration of 3 1 mannitol. The patient’s blood pressure is low, but upon calculating the mean arterial pressure (MAP, which is 85), a cerebral perfusion pressure (CPP) of 65 is confi rmed. Considering that all of the other measures would be inadequate to treat this urgent ICP elevation, manni- tol should be given now, and a reduced A: Normal ICP Waveform; blood pressure from the diuresis can be B: Urgent Intracranial Hypertension Waveform. Images created by George Williams, MD. treated later if appropriate (see accompa- nying waveforms).

17. The answer is C. The patient’s arterial blood gas demonstrates adequate ventilation but inadequate oxygenation. The treatment of choice to maximize alveolar recruitment is CPAP, which in turn will improve oxygenation. While BiPAP could be used secondary to the positive end-expiratory pressure (PEEP) component, it is not indicated since the

patient is maintaining a normal PaCO2 and has no respiratory acidosis. Amantadine may

408

66485457-66485438 www.ketabpezeshki.com

ZZakaria_87574_PTR_CH28_10-06-13_397-414.inddakaria_87574_PTR_CH28_10-06-13_397-414.indd 408408 66/19/2013/19/2013 8:49:068:49:06 PMPM PROCEDURAL SKILLS AND MONITORING: Answers

improve the mental status of a patient following a basal ganglia stroke over hours to days, but would not be an appropriate intervention for a patient in respiratory distress.

18. The answer is C. Every 20 cm change in height of a transducer from the level plane of what one is attempting to measure results in a change of 15 mmHg in the pressure measured. If the transducer is above the circle of Willis, then the pressure would appear lower than it actually is secondary to the effect of gravity in our atmosphere. Similarly, if the transducer is below the circle of Willis, then the pressure would appear higher than it actually is, again, secondary to the effect of gravity in our atmosphere. This allows us to accurately correct for the error in the monitor and determine that the ICP is actually 39 mmHg. Since this patient’s mean arterial pressure (MAP) is 77 mmHg, the CPP is actually 38 mmHg (7).

19. The answer is C. Preoxygenation is one of the single most effi cacious techniques that can be used to prevent desaturation following induction for intubation. Preoxygenation for

5 minutes with 100% FiO2 has been demonstrated to achieve a sustained saturation of >90% for approximately 6 minutes. Nasogastric tube insertion would not be indicated in this case and may actually be harmful given the unstable condition of this patient. No indications for a rapid sequence intubation are provided in this case. These indications include trauma, recent oral intake, bowel perforation, ileus, or pregnancy (12).

20. The answer is D. Whenever a patient is diffi cult to intubate, one should call for help following two or more failed attempts at direct laryngoscopy, regardless of potential improvement in visualization. Attempting to use adjunct techniques is maybe appropri- ate, but only after calling for help so that further resources are available. In this case, awakening the patient or returning to spontaneous ventilation is not a feasible option. Please review the American Society of Anesthesiologists diffi cult airway algorithms (13), (see the two following algorithms).

Awake Intubation

Airway Approached by Surgical/Percut- Noninvasive aneous Airway Intubation

Succeed Fail

Consider Cancel Surgical/Percut- Feasibility of Diffi cult Airway Algorithm for Awake Intubation aneous Airway Other Options Intubation [Image created by George Williams, MD; derived from Reference (13)].

409

66485457-66485438 www.ketabpezeshki.com

ZZakaria_87574_PTR_CH28_10-06-13_397-414.inddakaria_87574_PTR_CH28_10-06-13_397-414.indd 409409 66/19/2013/19/2013 8:49:068:49:06 PMPM GENERAL CRITICAL CARE: PATHOLOGY, PATHOPHYSIOLOGY, AND THERAPY

INTUBATION ATTEMPTS AFTER INDUCTION FOR INTUBATION

Initial Intubation Attempts Initial Intubation Attempts SUCCESSFUL FROM THIS POINT ONWARD UNSUCCESSFUL CONSIDER:

1. Calling for Help 2. Returning to Spontaneous Ventilation 3. Awakening the Patient

FACE MASK VENTILATION FACE MASK VENTILATION ADEQUATE CONSIDER NOT ADEQUATE / ATTEMPT LMA NON-EMERGENCY PATHWAY EMERGENCY PATHWAY Ventilation Adequate Ventilation Not Adequate Intubation Unsuccessful Intubation Unsuccessful

LMA Adequate LMA Not Adequate Alternative Approaches to Intubation Call for Help

IF BOTH FACE MASK Emergency Noninvasive Airway Ventilation AND LMA VENTILATION (ie, Jet Ventilation) FAIL BECOME INADEQUATE Successful After Intubation Multiple Successful FAIL Attempts Ventilation

Surgical/ Percutaneous Airway Surgical/Percutaneous Consider Feasibility of Cancel Intubation Airway Other Options

Diffi cult Airway Algorithm for Intubation After Induction [Image created by George Williams, MD; derived from Reference (13)].

21. The answer is A. ETCO2 is a highly useful tool, more commonly applied to patients who are intubated; however, it can be used in patients who are receiving oxygen via nasal can- nula or face mask. The capnograph is frequently an infrared absorption device, readily available for bedside use. There is normally a gradient of 3 to 4 mmHg between partial

pressure of carbon dioxide in arterial blood (PaCO2) and ETCO2, unless an open system such as a nasal cannula, face mask, and so on is being used. As such, it is normal to have

a usually low value for ETCO2 when used in this fashion. The normal respiratory rate indicates that the patient is not overly sedated (refl ected by a respiratory rate of <10) or anxious (tachypnea), and therefore no further intervention without a clinical cause would

be warranted. If the patient was intubated and suddenly had a drop in ETCO2, a pulmo- nary embolism workup would be appropriate (14).

22. The answer is D. SjvO2 is useful in determining outcomes in neurologically ill patients in whom there is a question about mismatch in oxygen supply and demand. Stocchetti et al. (15) demonstrated a ±5% variance in values between the right and left sides, although no one side was determined to be more accurate than the other in patients with bilateral, predominantly unilateral, cortical, or deeply located lesions. The authors used fi ber-optic catheters for con-

tinuous SjvO2 saturation data, although this is not a prerequisite for accurate measurement.

23. The answer is B. Jugular venous bulb saturation (SjvO2) is a refl ection of cerebral oxygen

supply and demand. While a low SjvO2 may be clearly indicative of a global mismatch of

410

66485457-66485438 www.ketabpezeshki.com

ZZakaria_87574_PTR_CH28_10-06-13_397-414.inddakaria_87574_PTR_CH28_10-06-13_397-414.indd 410410 66/19/2013/19/2013 8:49:078:49:07 PMPM PROCEDURAL SKILLS AND MONITORING: Answers

the supply demand ratio, an elevated SjvO2 is more diffi cult to isolate in terms of causative derangements. Potential causes include: a. Focal cerebral ischemia, if the reduced oxygen saturation from the ischemic brain is offset by a high oxygen saturation from surrounding hyperemic brain b. Oxygen-limited cytochrome turnover c. Restrictive oxygen diffusion d. Extracerebral contamination of blood in the jugular bulb when cerebral blood fl ood is low e. Rapid drawing of the blood sample through the catheter at rates >2 mL/minute (or 30 µL/second). Knowledge of the patient’s cerebral blood fl ow and/or cerebral metabolic rate is needed

to further delineate a cause of elevated SjvO2. Cormio et al. (16) demonstrated that higher

SjvO2 is associated with poorer outcomes when compared to severely head-injured patients

who had a normal SjvO2.

24. The answer is B. The Lindegaard index was developed to determine the likelihood of vasospasm with greater accuracy. It is defi ned as the ratio between the mean fl ow veloc- ity in the middle cerebral artery and the mean fl ow velocity in the internal carotid artery; the middle cerebral artery was chosen because it is the only artery in which threshold velocities have been clearly defi ned above which vasospasm is suspected. There is no fi rm consensus for other vessels. Lower frequencies (1–2 Hz) are used to maximize bone penetration. Four acoustic windows (transtemporal, transorbital, suboccipital, and retro- mandibular) are used, corresponding to the points at which sound can most easily pen- etrate the skull to allow assessment of the vessels. A description of a brain death pattern found on TCDs has been published; however, it is not universally accepted as defi ning brain death (17).

25. The answer is A. EVD weaning is a source of signifi cant debate and variation in practice. Choices A, B, and D are employed by different surgeons; however, only A has been dem- onstrated by a randomized control trial to be superior to choice B in terms of length of stay. Klopfenstein et al. (18) demonstrated that rapid weaning results in shorter lengths of stay and no increase in shunt insertion rates.

26. The answer is D. Catheter tunneling has been shown to reduce infection rates at the time of insertion; however, it still resulted in long-term infection rates of 17% (compared to 0%–22% for traditional EVDs). A correlation between irrigations and infection rates has been demonstrated and as such has led to the development of highly effective protocols [Dasic et al. (19)] that restrict EVD manipulation and reduce infection rates. Prophylactic catheter changes every 5 days have not demonstrated reduced infection rates compared to allowing a catheter to remain.

27. The answer is A. Hypermagnesemia may potentiate neuromuscular blockade by inhibit- ing acetylcholine release from the motor terminal as well as reducing end plate sensitivity to acetylcholine. Hypothermia reduces the metabolism of anesthetic drugs and therefore should be considered a sedative in its own right. Renal insuffi ciency impairs clearance of most muscle relaxants. Gentamycin, hypernatremia, hyperlithiumemia, hypokalemia, and hypocalcemia potentiate the effect of neuromuscular blocking agents (20).

411

66485457-66485438 www.ketabpezeshki.com

ZZakaria_87574_PTR_CH28_10-06-13_397-414.inddakaria_87574_PTR_CH28_10-06-13_397-414.indd 411411 66/19/2013/19/2013 8:49:078:49:07 PMPM GENERAL CRITICAL CARE: PATHOLOGY, PATHOPHYSIOLOGY, AND THERAPY

28. The answer is D. The respiratory quotient is defi ned as the production of carbon dioxide

divided by consumption of oxygen. FiO2, partial pressure of water, and PaCO2 are all components of the alveolar gas equation, along with the respiratory quotient. Type and status of nutrition are key factors in determining the respiratory quotient, but the con- sumption of oxygen must be measured and therefore most affects the respiratory quotient of all of the variables listed.

References

1. Marino PL, Sutin KM, et al. Oximetry and capnography, Chap 20. In: ICU Book. 3rd ed. Philadelphia: Lippincott Williams & Wilkins; 2007: 385–402. 2. Vincent JL, Abraham E, Moore FA, et al. Abdominal compartment syndrome, Chap 201. In: Textbook of Critical Care. 6th ed. Philadelphia: Elsevier; 2011;1469–1474. 3. Marino PL, Sutin KM, et al. Arterial blood pressure, Chap 8. In: ICU Book. 3rd ed. Philadelphia: Lippincott Williams & Wilkins; 2007: 151–162. 4. Goldberger AL, et al. ECG leads, Chap 3. In: Clinical Electrocardiography: A Simplifi ed Approach. 7th ed. St. Louis: Mosby/Elsevier; 2006: 21–32. 5. Marino PL, Sutin KM, et al. Oliguria and acute renal failure, Chap 31. In: ICU Book. 3rd ed. Philadelphia: Lippincott Williams & Wilkins; 2007: 579–594. 6. Rosen M, Latto P, Ng WS, et al. Practical aspects of technique, Chap 3. In: Handbook of Percutaneous Central Venous Catheterisation. 2nd ed. Philadelphia: W.B. Saunders Company; 1992: 45–52. 7. Miller RD, Eriksson LI, Fleisher LA, et al. Cardiovascular monitoring—physiologic con- siderations for pulmonary artery catheter monitoring: prediction of left ventricular fi ll- ing pressure, Chap 40. In: Miller’s Anesthesia. 7th ed. Philadelphia: Churchill-Livingstone; 2009: 1267–1328. 8. Vincent JL, Abraham E, Moore FA, et al. Mechanical ventilation—adverse effects of positive-pressure ventilation, Chap 48. In: Textbook of Critical Care. 6th ed. Philadelphia: Saunders/Elsevier; 2011: 328–334. 9. Dorsch JA, Dorsch SE, et al. Temperature monitoring, Chap 29. In: Understanding Anesthesia Equipment. 5th ed. Philadelphia: Lippincott Williams & Wilkins; 2008: 858–870. 10. Vincent JL, Abraham E, Moore FA, et al. Inotropic therapy—hemodynamic effects of ino- tropic agents in critically ill patients, effects on cardiac output, Chap 93. In: Textbook of Critical Care. 6th ed. Philadelphia: Saunders/Elsevier; 2011: 689–695. 11. Niedermeyer E, da Silva FL, et al. Technological basis of EEG recording, Chap 6. In: Electroencephalography: Basic Principles, Clinical Applications, and Related Fields. 5th ed. Philadelphia: Lippincott Williams & Wilkins; 2005: 128. 12. Barash PG, Cullen BF, et al. Airway management, Chap 29; anesthetic management, Sect VI. In: Clinical Anesthesia. 6th ed. Philadelphia: Lippincott Williams & Wilkins; 2009: 751–792. 13. American Society of Anesthesiologists Task Force on Management of the Diffi cult Airway. Practice guidelines for management of the diffi cult airway: an updated report by the American Society of Anesthesiologists Task Force on Management of the Diffi cult Airway. Anesthesiology. 2003 May;98(5):1269–1277/

412

66485457-66485438 www.ketabpezeshki.com

ZZakaria_87574_PTR_CH28_10-06-13_397-414.inddakaria_87574_PTR_CH28_10-06-13_397-414.indd 412412 66/19/2013/19/2013 8:49:078:49:07 PMPM PROCEDURAL SKILLS AND MONITORING: Answers

14. Marino PL, Sutin KM, et al. Cardiac arrest, Chap 15. In: ICU Book. 3rd ed. Philadelphia: Lippincott Williams & Wilkins; 2007: 277–296. 15. Stocchetti N, Paparella A, Bridelli F, et al. Cerebral venous oxygen saturation studied with bilateral samples in the internal jugular veins. Neurosurgery. 1994 Jan;34(1):38–43. 16. Cormio M, Valadka AB, Robertson CS et al. Elevated jugular venous oxygen saturation after severe head injury. J Neurosurg. 1999 Jan;90(1):9–15. 17. Rasulo FA, De Peri E, Lavinio A et al. Transcranial Doppler ultrasonography in intensive care. Eur J Anaesthesiol Suppl. 2008;42:167–173. 18. Klopfenstein JD, Kim LJ, Feiz-Erfan I, et al. Comparison of rapid and gradual weaning from external ventricular drainage in patients with aneurysmal subarachnoid hemor- rhage: a prospective randomized trial. J Neurosurg. 2004 Feb;100(2):225–229. 19. Dasic D, Hanna SJ, Bojanic S, et al. External ventricular drain infection: the effect of a strict protocol on infection rates and a review of the literature. Br J Neurosurg. 2006 Oct;20(5):296–300. 20. Lobato EB, Gravenstein N, Kirby RR, et al. Sodium, potassium and magnesium, Chap 32. In: Complications in Anesthesiology. 3rd ed. Philadelphia: Lippincott, Williams & Wilkins; 2008: 461–473.

413

66485457-66485438 www.ketabpezeshki.com

ZZakaria_87574_PTR_CH28_10-06-13_397-414.inddakaria_87574_PTR_CH28_10-06-13_397-414.indd 413413 66/19/2013/19/2013 8:49:078:49:07 PMPM 66485457-66485438 www.ketabpezeshki.com

ZZakaria_87574_PTR_CH28_10-06-13_397-414.inddakaria_87574_PTR_CH28_10-06-13_397-414.indd 414414 66/19/2013/19/2013 8:49:078:49:07 PMPM 29 Clinical Cases Asma Zakaria and Bülent Yapicilar QUESTIONS

1. An 18-year-old male admitted to the hospital with a gunshot wound to the head and intractable intracranial hypertension is being de-escalated from hyperosmolar therapy with hypertonic saline and slowly rewarmed at 0.25°C/hour to target temperature of 37°C. His current temperature is 35.5°C. The nurse informs you that his serum potassium is 2.0 mEq/L, despite having received approximately 120 mEq of KCl in replacement that day, and the urine output has been consistently greater than 200 mL/hour. Additional

pertinent labs include Na 153 mEq/L, Mg 1.5 mEq/L, and HCO3 20 mEq/L. You notice a change on the telemetry monitor and order a stat EKG. (See image on the following page). What is the likely cause of this abnormality? A. Acute myocardial infarction B. Acute pulmonary embolism C. Hypokalemia D. Acidosis E. Hypothermia

ANSWERS TO THIS SECTION CAN BE FOUND ON PAGE 423 415

66485457-66485438 www.ketabpezeshki.com

ZZakaria_87574_PTR_CH29_10-06-13_415-432.inddakaria_87574_PTR_CH29_10-06-13_415-432.indd 415415 66/19/2013/19/2013 8:43:478:43:47 PMPM GENERAL CRITICAL CARE: PATHOLOGY, PATHOPHYSIOLOGY, AND THERAPY

2. A 44-year-old morbidly obese woman with hypertension and diabetes is found slumped over in a bathroom stall. At the scene, her BP is documented at 180/90 with a pulse of 78; her blood sugar is 180 mmol/L. CT scan of the head is shown to the right. What is the likely cause of this patient’s fi ndings? A. Hypertensive intracerebral hemorrhage B. Embolic stroke with hemorrhagic conversion C. Venous sinus thrombosis D. Aneurysmal subarachnoid hemorrhage (SAH) E. Hemorrhagic encephalitis

3. A 29-year-old C5–C6 quadriplegic patient is being prepared for transfer to a rehabilitation facility when you are paged for with the following fi nding. What is an appropriate long- term therapy for this condition? A. Permanent pacemaker B. PRN IV atropine C. PO albuterol D. PO theophylline E. All of the above

416

66485457-66485438 www.ketabpezeshki.com

ZZakaria_87574_PTR_CH29_10-06-13_415-432.inddakaria_87574_PTR_CH29_10-06-13_415-432.indd 416416 66/19/2013/19/2013 8:43:478:43:47 PMPM CLINICAL CASES: Questions

4. A 67-year-old woman was transferred from an outside facility with a diagnosis of viral meningitis and coma. She had presented to the outside hospital with complaints of nau- sea, vomiting, ataxia, and progressive lethargy. The patient had a history of ovarian cancer in remission for 3 years and was otherwise healthy. CT scan and MRI brain were unre- markable. Lumbar puncture revealed a WBC count of 15 (40% N, 20% L, and 40% M), RBC count 250, glucose 40, and pro- tein 100. All bacterial and viral cultures and polymerase chain reactions were negative, and she was empirically treated with acyclovir. Upon your evaluation, the patient was comatose with sluggish pupillary response, dysconjugate gaze, absent oculocephalic refl exes, extensor posturing bilaterally, and brisk refl exes. She was breathing over the ventilator and was hemodynamically stable. Lumbar puncture was repeated and showed WBC 18 (100% M), RBC 50, glucose 28, and pro- tein 150. Serum glucose was 150 mg/dL. EEG revealed generalized delta and theta activity without rhythmicity. Brain MRI with contrast is shown on the right. What is the most likely cause of the patient’s condition? A. Miller–Fisher syndrome B. Bickerstaff brainstem encephalitis C. Meningeal carcinomatosis D. Pontine infarction E. Nonconvulsive status epilepticus (NCSE)

417

66485457-66485438 www.ketabpezeshki.com

ZZakaria_87574_PTR_CH29_10-06-13_415-432.inddakaria_87574_PTR_CH29_10-06-13_415-432.indd 417417 66/19/2013/19/2013 8:43:478:43:47 PMPM GENERAL CRITICAL CARE: PATHOLOGY, PATHOPHYSIOLOGY, AND THERAPY

5. A 60-year-old patient with diabetes and hypertension is transferred to your ICU from another facility, where he was being treated for urosepsis. The patient has been on stable, low-dose vasopressors for 4 days, is on antibiotics, and appears adequately volume resus- citated. On reviewing the chart, you realize that the patient has not received nutrition since his admission to the hospital 5 days ago because he was on vasopressors. You would: A. Insert a feeding tube and advance to full dose tube feeds despite vasopressors B. Order total parenteral nutrition (TPN) C. Order peripheral parenteral nutrition (PPN) D. Start a dextrose solution—it should provide enough calories! E. Start trophic feeds through the gut at 10 to 20 mL/hour, while he is on vasopressors and supplement with TPN or PPN

6. A 40-year-old woman is admitted to the hospital after witnessed cardiac arrest for 20 min- utes. Bystanders performed CPR until EMS arrived. Hypothermia protocol was initiated after admission to the hospital, and the patient was noted to have frequent myoclonic jerks during rewarming. Brain stem refl exes were intact and the patient had fl exor posturing to painful stimulation. MRI brain revealed scattered diffusion-weighted imaging changes in the cortex but no damage to the deep structures. EEG was performed and is shown in the image below. What is the next step in management? A. Have a family discussion regarding irreversible brain damage and comfort care B. Load with an antiepileptic agent until myoclonic jerks stop since they are diffi cult for the family to watch; then proceed with comfort care C. Continue EEG monitoring and titrate antiepileptic agents until the seizures stop or burst suppression is achieved D. Load with antiepileptic drugs until clinical myoclonus stops and then wait for the patient to wake up for 2 more days E. Recool the patient and rewarm more slowly next time

418

66485457-66485438 www.ketabpezeshki.com

ZZakaria_87574_PTR_CH29_10-06-13_415-432.inddakaria_87574_PTR_CH29_10-06-13_415-432.indd 418418 66/19/2013/19/2013 8:43:488:43:48 PMPM CLINICAL CASES: Questions

7. A 60-year-old woman with a history of hypertension presented with a cerebellar intra- parenchymal hemorrhage (IPH), fourth ventricle intraventricular hemorrhage (IVH), and hydrocephalus with partial brain stem dysfunction and somnolence on examination. She was emergently taken for surgical decompression and did well. A conventional angiogram was subsequently performed and is shown in the following images. Your next steps in management include all of the following, except: A. Wean external ventricular drain (EVD) and tightly control blood pressure (BP) B. Use intrathecal tissue plasminogen activator (tPA) to allow quick dissolution of the IVH to facilitate EVD wean C. Take a detailed family history D. Send genetic testing E. Continue outpatient, staged management of this condition

8. A 35-year-old, 2 days postpartum woman had sudden onset of the worst headache of her life. Her BP upon arrival to the ED was 180/90, and she was somnolent but had a nonfocal neurologic examination. Opening pressure was normal on lumbar puncture, and cere- brospinal fl uid (CSF) was clear with normal chem- istries and cell count. Imaging studies are shown in the fi gures at the right and at the top of the following page. What is the most likely diagnosis? A. Eclampsia/preeclampsia B. Posterior reversible encephalopathy syndrome (PRES) C. Venous sinus thrombosis D. Reversible cerebral vasoconstriction syndrome (RCVS) E. Sheehan’s syndrome

419

66485457-66485438 www.ketabpezeshki.com

ZZakaria_87574_PTR_CH29_10-06-13_415-432.inddakaria_87574_PTR_CH29_10-06-13_415-432.indd 419419 66/19/2013/19/2013 8:43:488:43:48 PMPM GENERAL CRITICAL CARE: PATHOLOGY, PATHOPHYSIOLOGY, AND THERAPY

9. An 18-year-old G1P0 woman presented with a severe headache behind her right ear, fol- lowed by a left-sided tingling sensation and a complex partial seizure. She is admitted to your ICU for close monitoring and treatment while she continues to have frequent complex partial seizures. On examination, she is awake, but somnolent with left-sided hemiparesis. Imaging studies are shown here. What is the most likely diagnosis? A. Eclampsia/preeclampsia B. Posterior reversible encephalopathy syndrome (PRES) C. Cerebral venous sinus thrombosis (CVST) D. Reversible cerebral vasoconstriction syndrome (RCVS) E. Sheehan’s syndrome

420

66485457-66485438 www.ketabpezeshki.com

ZZakaria_87574_PTR_CH29_10-06-13_415-432.inddakaria_87574_PTR_CH29_10-06-13_415-432.indd 420420 66/19/2013/19/2013 8:43:498:43:49 PMPM CLINICAL CASES: Questions

10. A 65-year-old alcoholic with traumatic brain injury, bifrontal contusions, and right-sided epidural hematoma with midline shift and uncal herniation is admitted to your ICU. After initial decompression, intracranial pressure (ICP) remained within normal limits. Four days postoperatively, the patient is now off all sedation and is still poorly responsive. He has intact brain stem refl exes, withdraws on the right, and fl exes on the left. Imaging studies and EEG are shown in the fi gures at the right and below. What is the cause of this man’s poor mental status? A. Bifrontal injury B. Diffuse axonal injury C. Persistent effects of midline shift after initial injury D. Nonconvulsive status epilepticus (NCSE) E. All of the above

421

66485457-66485438 www.ketabpezeshki.com

ZZakaria_87574_PTR_CH29_10-06-13_415-432.inddakaria_87574_PTR_CH29_10-06-13_415-432.indd 421421 66/19/2013/19/2013 8:43:498:43:49 PMPM GENERAL CRITICAL CARE: PATHOLOGY, PATHOPHYSIOLOGY, AND THERAPY

11. An 80-year-old woman is admitted to your ICU with mild traumatic brain injury after a fall in her dialysis center. The dialysis had been discontinued due to hypotension and frequent runs of nonsustained ventricular tachycardia. Upon admission, the patient was initially hemodynamically stable but developed acute hypotension requiring pressors, abdominal distension, and diffuse peri- toneal signs the next day. Kidney-ureter- bladder (KUB) x-ray was performed and is shown in the image on the right. What is the next step in management? A. Right upper quadrant ultrasound B. Stat gastroenterology medicine con- sult for endoscopic retrograde cho- langiopancreatography (ERCP) C. Nasogastric tube (NGT) to low inter- mittent wall suction to decompress the gaseous distension D. Rectal tube to decompress the gas- eous distention E. IV antibiotics and stat general surgery consult

12. A man is admitted to your ICU after facial assault with a penetrating object through his orbit. You are called to the bedside on postop day 1 after removal of the object because the patient is in severe pain. Upon your examination, the eye is swollen, injected, and pulsat- ing. The pupil is minimally reactive and extraocular movements are diffi cult to assess because of pain. He has light perception on visual acuity, which is unchanged from the time of presentation. What is the most likely diagnosis? A. Orbital cellulitis B. Orbital hematoma C. Cavernous sinus thrombosis D. Carotid-cavernous fi stula (CCF) E. Orbital compartment syndrome

422

66485457-66485438 www.ketabpezeshki.com

ZZakaria_87574_PTR_CH29_10-06-13_415-432.inddakaria_87574_PTR_CH29_10-06-13_415-432.indd 422422 66/19/2013/19/2013 8:43:518:43:51 PMPM 29

ANSWERS

1. The answer is C. These EKG changes are typically seen with severe refractory hypokalemia. The earliest EKG changes associated with hypokalemia are a decrease in T-wave amplitude followed by a ST segment depression and T-wave inversions. Subsequently, the PR interval may be prolonged with an increase in amplitude of the P wave. U waves (a positive defl ection after the T wave) may be seen and in severe hypokalemia may fuse with the T wave to form giant U waves as seen in the EKG. A pseudo-prolonged QT interval may be seen, which is actually a QU interval in the absence of a T wave (1,2). The most common cause of hypokalemia is renal losses after diuretic use (more commonly thiazide than loop or osmotic diuretics), especially if two diuretics acting on different parts of the tubular system are used. Patients who develop secondary hypoaldosteronism from liver disease, congestive heart failure, or nephrotic syndrome are also at risk (3). Antibiotics such as penicillins and aminoglycosides can promote potassium loss. In the neurocritical care setting, patients receiving mannitol or hypertonic saline infusions (4) have been noted to develop hypokalemia because of the diuretic effect and as a consequence of the high sodium load reaching the collect- ing ducts. For this reason some centers use potassium-sparing diuretics in conjunction with mannitol to avoid symptomatic hypokalemia in the setting of ICP management (5). Hypokalemia can be refractory if concomitant hypomagnesaemia is not corrected. The exact etiology of this is not known, but it may be multifactorial. Low intracellular magnesium levels may increase potassium wasting from the collecting tubules, espe- cially in the setting of additional factors such as an increase in distal sodium delivery (6). EKG changes in hypomagnesemia include an increased PR and QT interval, wid- ened QRS complex, and fl attened T wave. The fi ndings can be identical to those seen in hypokalemia; however, in this case, the patient’s potassium level is much lower than his magnesium level and, given the refractory nature of his disease, hypokalemia is the

423

66485457-66485438 www.ketabpezeshki.com

ZZakaria_87574_PTR_CH29_10-06-13_415-432.inddakaria_87574_PTR_CH29_10-06-13_415-432.indd 423423 66/19/2013/19/2013 8:43:518:43:51 PMPM GENERAL CRITICAL CARE: PATHOLOGY, PATHOPHYSIOLOGY, AND THERAPY

more likely cause. Hypothermia at 35°C (95°F) is associated with sinus tachycardia. As the core body temperature drops further (<90°F or 32°C), sinus bradycardia with pro- longation of PR interval, QRS widening, and QT interval are seen. Osborn waves (an upward defl ection after the QRS complex) are seen below 86°F or 30°C (7).

2. The answer is D. This axial CT scan of the head shows an intraparenchymal hemorrhage (IPH) with a sylvian subarachnoid hemorrhage (SAH) and a small subdural hematoma (SDH), with the outline of an aneurysm visible lateral to the IPH. Approximately 20% of patients with aneurysmal rupture have associated IPH, occurring more commonly in patients with anterior communicating artery, distal anterior cerebral artery, and distal middle cerebral artery aneurysms. These patients present with callosal, interhemispheric, and sylvian/temporal hematomas (8). In addition, patients may have concurrent intra- ventricular hemorrhage (IVH) and SDH. Rarely, patients may present with IPH, IVH (9), or SDH (10) without any evidence of SAH. In these cases, a history of thunderclap headache, absence of risk factors for IPH or IVH, and an absence of a history of trauma should raise the suspicion of a saccular aneurysm. Likely causes of the absence of SAH are the location of the dome of the aneurysm close to the ventricular system or brain parenchyma and delayed presentation after the hemorrhage, with radiographic resolu- tion of subarachnoid blood. Another theory is that the IPH/IVH may represent a rebleed after the subarachnoid space has been scarred down by a sentinel hemorrhage.

3. The answer is E. Patients with acute spinal cord injury suffer a myriad of complications of which pulmonary and cardiac are the most common. Among the cardiac complications, shock and hypotension occur early and are usually the reason for ICU admission in the fi rst week after injury. This is followed by sinus bradycardia, dysrhythmia, and cardiac arrest occurring more frequently in the fi rst 14 days after injury (11). Acute spinal cord injury above T6 can disrupt the descending sympathetic pathways to the intermediolateral cell column in the T1-L2 spinal cord (12). This results in loss of supraspinal sympathetic control and unopposed parasympathetic activity in the respiratory and cardiac systems in quadriplegic patients, making them prone to prolonged episodes of bradycardia, pauses, and intermittent heart blocks, especially when suctioned or turned. Atropine should be readily available for patients with bradycardia, and 0.5 to 1 mg should be administered when symptomatic or prophylactically before suctioning. PO albuterol and theophylline can be used to increase resting heart rate, although there are no randomized trials to support this. Some patients with signifi cant heart block or recurrent cardiac arrests may require a permanent pacemaker.

4. The answer is C. The history of ovarian cancer and the diffuse leptomeningeal enhance- ment are suggestive of meningeal carcinomatosis. Meningeal carcinomatosis is seen in 1% to 5% of patients with solid tumors (commonly breast, non-small-cell lung cancer, and melanoma), 5% to 15% of patients with leukemia/lymphoma (leukemic/lymphoma- tous meningitis), and 1% to 2% of patients with primary brain tumors (13). Neurologic symptoms usually localize to several regions of the neuraxis, with those involving the spinal cord and cauda equina being most common, followed by cranial neuropathies and hemispheric dysfunction. Patients often present with signs of hydrocephalus and elevated intracranial pressures (ICP) (14). Diagnosis is made by having a high index of suspicion, meningeal enhancement (often most prominent in the basal meninges, dorsal spinal column, and cauda equina), cerebrospinal fl uid (CSF) monocytosis, and isolating

424

66485457-66485438 www.ketabpezeshki.com

ZZakaria_87574_PTR_CH29_10-06-13_415-432.inddakaria_87574_PTR_CH29_10-06-13_415-432.indd 424424 66/19/2013/19/2013 8:43:518:43:51 PMPM CLINICAL CASES: Answers

tumor cells in CSF on cytology. Although the latter is the gold standard, tumor cells are isolated in only 50% of samples on the fi rst lumbar puncture (15), with the yield increas- ing to 80% on the second puncture. There is no added benefi t of subsequent lumbar punctures (16).Treatment is mostly palliative, with median survival of 2 to 3 months. Whole brain radiation therapy and intrathecal chemotherapy are the mainstays of treat- ment and may prevent further neurologic deterioration and improvement in quality of life. Supportive treatments such as antiepileptic medications, pain medications, and occasionally corticosteroids in patients with concomitant parenchymal disease should be prescribed. Miller–Fisher syndrome is a demyelinating cranial and peripheral neuropa- thy, which is a variant of Guillain–Barré syndrome and is defi ned by a triad of arefl exia, ophthalmoplegia, and ataxia. Patients classically do not have signs of encephalitis or encephalopathy. Anti-GQ1b antibodies are present in 90% of patients. Bickerstaff brain stem encephalitis presents with ataxia, ophthalmoplegia, hyperrefl exia, and alteration of consciousness. Hyperintense lesions are seen in the midbrain, pons, and medulla on MRI. A signifi cant number of patients have evidence of concurrent axonal Guillain– Barré syndrome, suggesting that this may be another variant of the disease. The pro- gression of symptoms and MRI fi ndings preclude pontine infarction, and the EEG does not support a diagnosis of NCSE.

5. The answer is A. Initiation of enteral nutrition (EN) in critically ill patients is not always clear-cut. It is preferable to use EN in critically ill patients as it reduces infectious com- plications, promotes enterocyte health while maintaining a strong mucosal barrier, and has a lesser stress response than parenteral nutrition (PN). However, there is concern that initiating EN in a patient with hemodynamic compromise and possible splanchnic vasoconstriction may promote nonocclusive mucosal ischemia (NOMI) due to increased oxygen demand. Turza et al. (17) recommend a four-stage approach to initiating EN in patients requiring vasopressors: a. Evaluate the patient’s medical and nutritional history. Patients with multiple vascular risk factors may be predisposed to NOMI, while those with poor nutritional response or high metabolic demand will benefi t from early nutrition. b. Evaluate the current physiologic state. Low-dose, nonescalating vasopressors in patients who are volume resuscitated and able to maintain a mean arterial pressure greater than 60 should not deter the initiation of EN. Alternately, patients with drop- ping urine output, worsening lactate levels, and base defi cit and those requiring fre- quent transfusions may not be able to tolerate EN. c. Establish gastrointestinal (GI) access and pick appropriate tube feeds. The use of for- mulas with lower osmolarity (<700 mOsm) and fi ber content and simpler sugars will reduce metabolic demands, improve transit time, and reduce dysmotility, thereby alleviating factors which may result in the bacterial overgrowth that worsens NOMI. d. Post-initiation monitoring includes serial abdominal examinations and gastric resid- ual checks. Laboratory monitoring of lactate, white blood cell counts, and hemoglobin levels or radiologic testing may be performed if there is concern for ileus.

At times, a combination of trophic EN and supplemental PN is adopted as an intuitive compromise. This has been shown to increase the risk of nosocomial infections in a retro- spective review of trauma patients (18).

425

66485457-66485438 www.ketabpezeshki.com

ZZakaria_87574_PTR_CH29_10-06-13_415-432.inddakaria_87574_PTR_CH29_10-06-13_415-432.indd 425425 66/19/2013/19/2013 8:43:518:43:51 PMPM GENERAL CRITICAL CARE: PATHOLOGY, PATHOPHYSIOLOGY, AND THERAPY

6. The answer is C. The EEG is consistent with postanoxic status epilepticus (PSE). Although this diagnosis usually portends a poor prognosis, the administration of therapeutic hypo- thermia (TH) to post–cardiac arrest patients has made the prediction of outcomes based on American Academy of Neurology (AAN) criteria more diffi cult. In a prospective study by Rossetti et al. (19), three clinical variables were demonstrated to have higher false- positive rates compared to the AAN guidelines: incomplete brain stem refl exes, myoclonus, and absent motor responses to pain. Early lack of reactivity on continuous EEG, prolonged periods of discontinuity, epileptiform discharges or seizures, and absent cortical responses on somatosensory evoked potentials (SSEP) were strongly associated with mortality (20). PSE is independently associated with a poor outcome after anoxic injury. However, in the presence of brain stem refl exes, SSEP responses and EEG reactivity can have a favorable outcome if the condition is treated as status epilepticus (21).

7. The answer is B. The patient has multiple arteriovenous malformations (AVMs) warrant- ing a detailed family history and genetic workup. Intrathecal tPA is contraindicated in this setting. Intracranial vascular malformations include developmental venous anoma- lies (DVA), capillary telangiectasias, AVM, and cavernous malformations, each of which has different natural histories and treatment options. • Capillary telangiectasias are usually angiographically occult lesions, detected inciden- tally on contrast-enhanced MRI or CT scans, and are rarely symptomatic. They are benign, thin-walled capillaries surrounded by normal brain parenchyma, do not need treatment or follow-up, and account for 4% to 12% of all vascular malformations (22). De novo development has been reported (24). • DVA also known as venous angiomas are congenitally enlarged, thickened, hyalinized venous vessels draining normal brain tissue. They are the most common intracranial vascular malformation with a prevalence of 2.5% (22), discovered mostly incidentally and often associated with cavernous malformations. A causative link between the two has been suggested, with DVAs being a precursor to cavernous malformations (24). DVAs can be detected on MRI, CT scan, and angiogram as a single dilated vein or caput medusa. They usually have a benign clinical course with low morbidity and mortality and do not warrant treatment. • AVMs are a collection of abnormal blood vessels, comprising arteries, veins, and an intervening collection of abnormal vessels called the nidus (23). They are congenital lesions with an incidence of 1 per 100,000 (23), often presenting as an intracerebral hemorrhage in the third to fourth decade (22). The bleeding risk varies according to size, location, draining pattern, and so on. Magnetic resonance angiography (MRA) and computed tomography angiography (CTA) can both visualize AVMs; however, conventional angiography remains the gold standard for diagnoses, formulation and implementation of a treatment plan which may include a combination of surgery, embolization, and radiation therapy. • Cavernous malformations are acquired or congenital lesions formed by endothelium- lined vascular spaces without intervening brain parenchyma. They can be seen on CT and MRI as “popcorn” shaped dystrophic calcifi cations or hemosiderin deposits from prior hemorrhages. They are angiographically occult and most commonly pres- ent with seizures. Treatment is not always warranted, but surgical resection is often necessary if associated with recurrent hemorrhages or intractable epilepsy.

426

66485457-66485438 www.ketabpezeshki.com

ZZakaria_87574_PTR_CH29_10-06-13_415-432.inddakaria_87574_PTR_CH29_10-06-13_415-432.indd 426426 66/19/2013/19/2013 8:43:518:43:51 PMPM CLINICAL CASES: Answers

• Dural arteriovenous fi stulas are malformations located in the meninges fed by dural arteries and drained by dural or leptomeningeal venous channels. The most common location is the transverse sinus. They can develop after trauma, venous thrombosis, or spontaneously, and account for 10% to 15% of all cranial malformations (23). Clinical presentation includes headache, neurologic defi cits, bruits, and hemorrhage, with a cortical drainage pattern consistent with more aggressive behavior. Six-vessel cerebral angiogram remains the gold standard in diagnosing a fi stula. Treatment includes endo- vascular or surgical interventions, with surgery being the more defi nitive approach.

8. The answer is D. Reversible cerebral vasosconstriction syndrome unifi es a group of disorders with similar clinical and radiographic characteristics, such as: Call–Fleming syndrome, benign angiopathy of the central nervous system (CNS), migrainous angiitis, drug-induced cerebral angiopathy (selective serotonin reuptake inhibitors [SSRIs], mari- juana), and postpartum angiopathy (hormonal changes, serotonergic surge). Typically, patients present with a thunderclap headache with or without focal neurologic signs, normal CSF analysis without evidence of CNS infl ammation, exclusion of other causes of sudden severe headache (aneurysm or vascular abnormalities), and presence of segmen- tal cerebral arterial vasoconstriction on catheter, CT, or magnetic resonance (MR) angiog- raphy, which resolves within 12 weeks (25–26). Cortical (non-aneurysmal) subarachnoid hemorrhage (SAH) is the most common radiologic fi nding (27). The gold standard for diagnosis is detecting the presence and subsequent resolution of segmental vasoconstric- tion on conventional angiography. In most cases the vascular changes resolve without treatment. Calcium channel blockers (nimodipine, verapamil), glucocorticoids, and IV magnesium have been tried with limited success. Although the course is usually benign, severe vasospasm has been reported resulting in transient ischemic attacks, seizures, and ischemic and hemorrhagic infarctions (28). Intra-arterial milrinone and verapamil as well as angioplasty have been attempted with some success in severe or refractory cases. Preeclampsia is defi ned as pregnancy-induced hypertension with proteinuria, but there is no information about urine studies in this case and the condition is not associ- ated with the imaging fi ndings of cerebral vasoconstriction. Eclampsia, a life-threatening condition occurring in pregnancy or early puerperium, is the occurrence of a tonic–clonic seizure in the setting of preeclampsia. Treatment includes IV magnesium to a goal of 4 to 7 mEq/L (4–6 g IV loading dose, then 1–2 g/hour), blood pressure (BP) control with IV hydralazine or labetalol, and emergent termination of pregnancy. If the patient continues to have seizures, the dose of magnesium may be increased (with close observation for respiratory failure or heart block). IV anticonvulsants and mechanical ventilation can be initiated in refractory cases. PRES is characterized by headache, confusion, seizures, and visual loss with acute subcortical and cortical edema on MRI. Although classically seen in the parieto-occipital lobes, brainstem, and cerebellum, the edema can extend as far anteriorly as the temporal and frontal lobes (29). PRES can be seen in the setting of malig- nant hypertension, eclampsia, hypercalcemia, and due to drugs such as tacrolimus and cyclosporine. Sheehan’s syndrome, or postpartum pituitary necrosis, is a complication of postpartum hypovolemic shock resulting in panhypopituitarism. The most common presenting symptom is agalactorrhea.

9. The answer is C. Cerebral venous sinus thrombosis is a rare disorder affecting 3 to 4 adults per million annually (30). With the increasing use of oral contraceptives (OCP) in the past

427

66485457-66485438 www.ketabpezeshki.com

ZZakaria_87574_PTR_CH29_10-06-13_415-432.inddakaria_87574_PTR_CH29_10-06-13_415-432.indd 427427 66/19/2013/19/2013 8:43:518:43:51 PMPM GENERAL CRITICAL CARE: PATHOLOGY, PATHOPHYSIOLOGY, AND THERAPY

few decades, the disease has become more prevalent in adult women of child-bearing age with an almost 6-fold increase in the risk of CVST among OCP users (31). This is followed by patients with inherited thrombophilia, hypercoagulability associated with pregnancy and puerperium, and head and neck infections. Presentation is varied and dependent on the location and extent of venous involvement. Headache is a ubiquitous presenting com- plaint, accompanied by seizures in 47% and paresis in 43% of patients (32). The majority of patients have an indolent course with symptoms developing over days to months. Rarely, they may resemble an arterial infarction but with a waxing and waning course. Focal edema and infarctions are often seen when cortical veins are involved. Larger infarctions and hem- orrhages are associated with worsening mental status and coma. Abnormal signal change in the venous sinus on MRI with concomitant loss of fl ow on magnetic resonance venography (MRV) is diagnostic. Treatment includes systemic anticoagulation with weight-based low molecular weight heparin or unfractionated heparin with transition to vitamin K antago- nists to a goal INR of 2 to 3 for 3 to 6 months. Patients with a history of deep vein thrombo- sis (DVT) or recurrent CVST will need indefi nite treatment. Intraparenchymal hemorrhage is not a contraindication to anticoagulation in this population. Local administration of endo- vascular thrombolysis has been reported, but there is insuffi cient effi cacy or safety data available to justify its utility in patients who are not refractory to systemic anticoagulation. Decompressive hemicraniectomy can be performed in the setting of malignant cerebral edema with reasonable outcomes (33). Over 80% of the patients have favorable recovery. Mortality of 7% to 13% is seen within the fi rst month, usually due to cerebral edema in the acute phase or due to underlying cause on subsequent follow-up (30).

10. The answer is E. The patient has suffered a severe head injury with multiple contusions, cerebral edema, and herniation. Individually, each of these conditions can result in a persistent comatose state. In addition, the patient is having nonconvulsive focal seizures, which may be contributing to the encephalopathy. The growing recognition of noncon- vulsive seizures in the critically ill population and the need for treatment has been a topic of debate in recent years. Are these seizures a cause of the encephalopathy or simply a manifestation of the dying brain (34)? Similarly, should they be aggressively treated with the hope of resolution of coma or are they a hallmark of irreversible brain injury and a poor prognostic sign? Although case reports supporting both arguments exist, these questions are yet to be answered in a randomized controlled trial. Additionally, many reports of NCSE lump together patients who are delirious with patients who are deeply comatose in the setting of nonconvulsive seizures, making it all the more diffi cult to establish prognosis (35). Until the availability of further evidence, the best way to approach these situations is to look at the entire clinical picture. Aggressive treatment, with its risks, may be warranted if the clinical picture looks worse than can be explained by the level of injury. Similarly, in the setting of a devastating injury, administration of further sedatives to treat focal NCSE may not be worthwhile.

11. The answer is E. Pneumobilia, defined as air within the hepatic portion of the bil- iary tree, suggests an abnormal communication between the intestine and biliary tree or the presence of gas-forming bacteria within the biliary tree. It can be dif- ferentiated and is managed differently, from air within the portal venous system by its appearance on abdominal CT scan (see image on following page). The flow of

428

66485457-66485438 www.ketabpezeshki.com

ZZakaria_87574_PTR_CH29_10-06-13_415-432.inddakaria_87574_PTR_CH29_10-06-13_415-432.indd 428428 66/19/2013/19/2013 8:43:518:43:51 PMPM CLINICAL CASES: Answers

bile toward the hilum of the liver directs the air more centrally in pneumobilia as opposed to air in the portal system, which follows an exten- sive branching pattern approaching the capsule. Causes of pneumobilia include surgical or spon- taneous (gallstones, peptic ulcer disease) bil- iary-enteric anastomoses, incompetent sphinc- ter of Oddi, and, less commonly, infections such as emphysematous cholecystitis, acute cholan- gitis, and liver abscesses. Portal venous gas is associated with mesenteric ischemia in 50% of the patients and can be seen postoperatively or secondary to gastric ulcers, diverticulitis, small bowel obstruction, or septicemia. Fifteen per- cent of cases are idiopathic (36). Our patient had mesenteric ischemia likely secondary to embo- lism and hypoperfusion.

12. The answer is D. The patient has a CCF, which is an abnormal communication between the arterial and venous blood within the cavernous sinus and is characterized by pain, chemosis, pulsatile proptosis, ocular bruit, and progressive vision loss. The most com- mon form of CCF is a direct communication between the internal carotid artery (ICA) and cavernous sinus (type A) usually as a result of trauma (young males) or aneurysm rupture (older women) (37). Traumatic CCFs are the most common type, accounting for 75% of all CCFs, and occur in 0.2% of all head trauma and 4% of basilar skull fractures. Conventional angiogram is the gold standard for diagnosis. Endovascular transvenous embolization of the fi stula while maintaining patency of the ICA is the mainstay of treat- ment, with greater than 80% cure rates at 1 year. Symptoms of chemosis and proptosis usually resolve within hours to days of intervention, while cranial nerve palsies may per- sist for a few weeks. Visual loss may or may not be reversed depending on the degree of blindness at presentation and the underlying cause (37). Cavernous sinus thrombosis is a close differential diagnosis and usually presents with ptosis, chemosis, proptosis, cranial nerve (CN) palsies, vision loss, and a dilated, sluggishly reactive pupil. The most com- mon etiology is infectious with direct spread from the nose, sinuses, or teeth. Diagnosis is made by clinical fi ndings, MRI, and magnetic resonance venography (MRV), and treat- ment includes IV antibiotics and close monitoring for complications such as meningi- tis, vision loss, sepsis, or septic emboli. Orbital cellulitis is a bacterial infection of the tissues surrounding the eye, including eyelids, eyebrows, and cheeks, resulting in swell- ing of the eyelids, pain with eye movements, fever, and decreased vision if not treated promptly. Orbital hematomas can be preseptal or postseptal. Preseptal hemorrhages are usually posttraumatic and benign, resulting in extensive ecchymoses of the eyelids. Postseptal hemorrhages may occur due to trauma, surgical intervention, arteriovenous malformations, or bleeding diathesis, among other causes, and can lead to orbital com- partment syndrome with increase in intraocular pressures and vision loss from orbital nerve compression.

429

66485457-66485438 www.ketabpezeshki.com

ZZakaria_87574_PTR_CH29_10-06-13_415-432.inddakaria_87574_PTR_CH29_10-06-13_415-432.indd 429429 66/19/2013/19/2013 8:43:518:43:51 PMPM GENERAL CRITICAL CARE: PATHOLOGY, PATHOPHYSIOLOGY, AND THERAPY

References

1. Diercks DB, Shumaik GM, Harrigan RA, et al. Electrocardiographic manifestations: elec- trolyte abnormalities. J Emerg Med. 2004;27(2):153–160. 2. El-Sherif N, Turitto G. Electrolyte disorders and arrhythmogenesis. Cardiol J. 2011; 18(3):233–245. 3. Weiner ID, Wingo CS. Hypokalemia: consequences, causes, and correction. J Am Soc Nephrol. 1997;8(7):1179–1188. 4. Huang SJ, Chang L, Han YY, Lee YC, Tu YK. Effi cacy and safety of hypertonic saline solu- tions in the treatment of severe head injury. Surg Neurol. 2006;65(6):539–546. 5. Bilotta F, Giovannini F, Aghilone F, et al. Potassium sparing diuretics as adjunct to man- nitol therapy in neurocritical care patients with cerebral edema: effects on potassium homeostasis and cardiac arrhythmias. Neurocrit Care. 2012;16(2):280–285. 6. Huang CL, Kuo E. Mechanism of hypokalemia in magnesium defi ciency. J Am Soc Nephrol. 2007;18(10):2649–2652. 7. Alhaddad IA, Khalil M, Brown EJ Jr. Osborn waves of hypothermia. Circulation. 2000;101(25):E233–E244. 8. Tokuda Y, Inagawa T, Katoh Y, et al. Intracerebral hematoma in patients with ruptured cerebral aneurysms. Surg Neurol. 1995;43(3):272–277. 9. Thai QA, Raza SM, Pradilla G, et al. Aneurysmal rupture without subarachnoid hemor- rhage: case series and literature review. Neurosurgery. 2005;57(2):225–229. 10. Marbacher S, Fandino J, Lukes A. Acute subdural hematoma from ruptured cerebral aneurysm. Acta Neurochir (Wien). 2010;152(3):501–507. 11. Casha S, Christie S. A systematic review of intensive cardiopulmonary management after spinal cord injury. J Neurotrauma. 2011;28(8):1479–1495. 12. Hagen EM, Faerestrand S, Hoff JM, et al. Cardiovascular and urological dysfunction in spinal cord injury. Acta Neurol Scand Suppl. 2011;191:71–78. 13. Chamberlain MC. Carcinomatous meningitis. Arch Neurol. 1997;54(1):16–17. 14. Chamberlain MC. Neoplastic meningitis. Oncologist. 2008;13(9):967–977. 15. Miller E, Dy I, Herzog T. Leptomeningeal carcinomatosis from ovarian cancer. Med Oncol. 2012;29(3):2010–2015. 16. Wasserstrom WR, Glass JP, Posner JB. Diagnosis and treatment of leptomeningeal metas- tases from solid tumors: experience with 90 patients. Cancer. 1982;49(4):759–772. 17. Turza, KC, Krenitsky, J, Sawyer, RG. Enteral feeding and vasoactive agents: suggested guidelines for clinicians. Pract Gastroenterol. 2009;33(9):11–22. 18. Sena MJ, Utter GH, Cuschieri J, et al. Early supplemental parenteral nutrition is associ- ated with increased infectious complications in critically ill trauma patients. J Am Coll Surg. 2008;207(4):459–467. 19. Rossetti AO, Oddo M, Logroscino G, et al. Prognostication after cardiac arrest and hypo- thermia: a prospective study. Ann Neurol. 2010;67(3):301–307. 20. Rossetti AO, Urbano LA, Delodder F, et al. Prognostic value of continuous EEG mon- itoring during therapeutic hypothermia after cardiac arrest. Crit Care. 2010;14(5): R173. 21. Rossetti AO, Oddo M, Liaudet L, et al. Predictors of awakening from postanoxic status epilepticus after therapeutic hypothermia. Neurology. 2009;72(8):744–749.

430

66485457-66485438 www.ketabpezeshki.com

ZZakaria_87574_PTR_CH29_10-06-13_415-432.inddakaria_87574_PTR_CH29_10-06-13_415-432.indd 430430 66/19/2013/19/2013 8:43:528:43:52 PMPM CLINICAL CASES: Answers

22. Chalouhi N, Dumont AS, Randazzo C, et al. Management of incidentally discovered intracranial vascular abnormalities. Neurosurg Focus. 2011;31(6):E1. 23. Byrne JV. Cerebrovascular malformations. Eur Radiol. 2005;15(3):448–452. 24. Abla A, Wait SD, Uschold T, et al. Developmental venous anomaly, cavernous malfor- mation, and capillary telangiectasia: spectrum of a single disease. Acta Neurochir (Wien). 2008;150(5):487–489. 25. Garcin B, Clouston J, Saines N. Reversible cerebral vasoconstriction syndrome. J Clin Neurosci. 2009;16(1):147–150. 26. Velez A, McKinney JS. Reversible cerebral vasoconstriction syndrome: a review of recent research. Curr Neurol Neurosci Rep. 2013;13(1):319. 27. Ducros A, Boukobza M, Porcher R, et al. The clinical and radiological spectrum of reversible cerebral vasoconstriction syndrome. A prospective series of 67 patients. Brain. 2007;130(Pt 12):3091–3101. 28. Albano B, Del Sette M, Roccatagliata L, et al. Cortical subarachnoid hemorrhage asso- ciated with reversible cerebral vasoconstriction syndrome after elective triplet cesarean delivery. Neurol Sci. 2011;32(3):497–501. 29. Bartynski WS, Boardman JF. Distinct imaging patterns and lesion distribution in poste- rior reversible encephalopathy syndrome. Am J Neuroradiol. 2007;28(7):1320–1327. 30. Martinelli I, Passamonti SM, Rossi E, et al. Cerebral sinus-venous thrombosis. Intern Emerg Med. 2012;7(Suppl 3):S221–S225. 31. Dentali F, Crowther M, Ageno W. Thrombophilic abnormalities, oral contraceptives, and risk of cerebral vein thrombosis: a meta-analysis. Blood. 2006;107(7):2766–2773. 32. Dangal G, Thapa LB. Cerebral venous sinus thrombosis presenting in pregnancy and puerperium. BMJ Case Rep. 2009;2009. doi:10.1136/ bcr.06. 2009.2045. 33. Ferro JM, Crassard I, Coutinho JM, et al. Second International Study on Cerebral Vein and Dural Sinus Thrombosis (ISCVT 2) Investigators. Decompressive surgery in cere- brovenous thrombosis: a multicenter registry and a systematic review of individual patient data. Stroke. 2011;42(10):2825–2831. 34. Bauer G, Trinka E. Nonconvulsive status epilepticus and coma. Epilepsia. 2010; 51(2):177–190. 35. Kaplan PW. Assessing the outcomes in patients with nonconvulsive status epilepticus: nonconvulsive status epilepticus is underdiagnosed, potentially overtreated, and con- founded by comorbidity. J Clin Neurophysiol. 1999;16(4):341–352. 36. Sherman SC, Tran H. Pneumobilia: benign or life-threatening. J Emerg Med. 2006; 30(2):147–153. 37. Ellis JA, Goldstein H, Connolly ES Jr, et al. Carotid-cavernous fi stulas. Neurosurg Focus. 2012;32(5):E9.

431

66485457-66485438 www.ketabpezeshki.com

ZZakaria_87574_PTR_CH29_10-06-13_415-432.inddakaria_87574_PTR_CH29_10-06-13_415-432.indd 431431 66/19/2013/19/2013 8:43:528:43:52 PMPM 66485457-66485438 www.ketabpezeshki.com

ZZakaria_87574_PTR_CH29_10-06-13_415-432.inddakaria_87574_PTR_CH29_10-06-13_415-432.indd 432432 66/19/2013/19/2013 8:43:528:43:52 PMPM Index

Italic page numbers indicate the subject appearing in Answer text.

ABCD2 system, 16 diabetic ketoacidosis, 127 abdominal compartment syndrome and hypothermia, 309 (ACS), 404 metabolic, 290, 295, 301, 309–310 at extremity, 373, 379 AComm see anterior communication artery operative abdominal decompression for, aneurysm 374, 379 ACS see abdominal compartment syndrome abscess ACTIVE-A see Atrial Fibrillation Clopidogrel cerebral, 89–90 Trial with Irbesartan for Prevention of spinal epidural, 82, 88–89 Vascular Events trial absent brainstem refl exes, diagnosis of, for acute disseminated encephalomyelitis brain death, 363, 368 (ADEM) ACDF see anterior cervical discectomy and diagnosis of, 113, 118 fusion measles and, 112, 117 ACE see angiotensin-converting-enzyme presentation of, 112, 117 inhibitors symptoms of, 112, 117 acetaminophen acute interstitial nephritis (AIN) for controlling fever after left-middle secondary to nafcillin, 288, 293–294 cerebral artery infarction, 15 acute liver failure (ALF) overdose and liver failure, 96, 100 liver transplantation for, 144, 151 acetazolamide, for high-altitude pulmonary neomycin and, 144, 152 edema, 231 prior to placement of ICP monitoring, acetylcholine esterase inhibitor 145, 152 overdose and myasthenia gravis, 74, 76 acute lung injury (ALI) acetylcysteine, for gastrointestinal disorders, mechanical ventilation for, 247, 259 350, 356 transfusion-related, and hypoxia, 373, 378 acid–base disturbance, 289, 290, 295 acute massive hemothorax, following acidosis trauma, 374, 379

433

66485457-66485438 www.ketabpezeshki.com

ZZakaria_87574_PTR_Index_10-06-13_433-460.inddakaria_87574_PTR_Index_10-06-13_433-460.indd 433433 66/19/2013/19/2013 4:42:244:42:24 PMPM 66485457-66485438 www.ketabpezeshki.com INDEX

acute partial transverse myelitis (APTM), 117 gastric decompression for, 353, 359 acute renal failure (ARF) operative drainage for, 353, 359 acyclovir for, 187, 196, 197 see also pancreatitis valacyclovir for, 187, 197 aldactone, for hyperaldosteronism, 294 acute respiratory distress syndrome (ARDS), ALF see acute liver failure 272, 282–283 ALI see acute lung injury aspiration pneumonia and, 282 alloimmunization, for thrombocytopenia, 340 generalized systemic infl ammatory alpha coma, 55, 65–66, 165, 172 response syndrome and, 282 ALS see amyotrophic lateral sclerosis pancreatitis and, 282 alveolar minute ventilation equation, 242, 253 sepsis and, 268, 275–276 Alzheimer’s dementia, 83 acute right hemiparesis, lisinopril for, 4, 11 amantadine, 408–409 acute tubular necrosis (ATN), 291 amiloride, for hyperaldosteronism, 294 acyclovir, for acute renal failure, amiodarone 187, 196, 197 adverse effects of, 225, 233–234 Addison’s disease, 127 and thyrotoxicosis, 127 ADEM see acute disseminated plus warfarin, 184, 193–194 encephalomyelitis amphotericin B adequate sputum specimen, identifi cation for cryptococcal CNS infection in HIV of, 267, 273 patients, 81, 87 adjuvant chemotherapy, for grade IV plus fl uconazole, 81, 87 glioblastoma, 132, 135–136 plus fl ucytosine, 81, 87 adrenal insuffi ciency and hyponatremia, ampicillin, for Listeria monocytogenes- 123, 128 induced bacterial meningitis, 183, 192 β-adrenergic agonists, for asthma, 268, 274 amyotrophic lateral sclerosis (ALS), 270, 279 adults, metastatic brain tumors in, 132, 136 cognitive impairment of, 71, 75 afterload, 209 lithium carbonate for, 71, 75 decreased afterload ventilation, effects on riluzole and, 71, 75 cardiovascular system, 281, 400, 407 anesthesia-related malignant hyperthermia, aggressive fl uid repletion, for 301, 308–309 rhabdomyolysis, 74, 75 aneurysm AIN see acute interstitial nephritis anterior circulation, 22, 28 airway pressure release ventilation (APRV) anterior communication artery, 29 patient’s spontaneous breathing during, cerebral, 178, 181 248, 261 posterior communicating artery, 24, 28 user-prescribed variables of, 247, 259 rebleeding of, 21, 27 albumin-adjusted calcium, 306 repair, surgical consultation for, 220, 228 albuterol, for C5–C6 quadriplegic patient, aneurysmal subarachnoid 416, 424 hemorrhage, 416, 424 alcohol abuse angiotensin-converting-enzyme (ACE) and central pontine, 300, 308 inhibitors, 270 and extrapontine myelinolysis syndromes, for BP management in stroke patients, 14 300, 308 plus thiazide diuretics, 15 and osmotic demyelination, 300, 308 angiotensin receptor blockers alcoholic pancreatitis for BP management in stroke patients, 15 fl uid resuscitation for, 353, 359 plus thiazide diuretics, 15

434

66485457-66485438 www.ketabpezeshki.com

ZZakaria_87574_PTR_Index_10-06-13_433-460.inddakaria_87574_PTR_Index_10-06-13_433-460.indd 434434 66/19/2013/19/2013 4:42:244:42:24 PMPM INDEX

Anopheles, 86 arterial pulse pressure variation, 207, 213–214 anterior cervical discectomy and fusion arterial waveform analysis, 208, 214 (ACDF), 176, 179–181 arteriovenous malformations (AVMs), 426 anterior circulation aneurysms, 22, 28 arteritis anterior communication artery (AComm) giant cell, 115 aneurysm, 29 temporal, 110 anticonvulsants see also arteritis for Alzheimer’s dementia, 83 artesunate, for cerebral malaria, 80, 86–87 for ischemic stroke, 8, 15–16 ascending paralysis, 112, 117 antimicrobial therapy aseptic meningitis, penicillins for, 109 for bacterial meningitis, 183, 192 aspiration, 274 in septic shock, hypotension duration and hypoxia, 373, 378 prior to, 321, 328 pneumonia, 269, 274, 277, 282 anti-muscle-specifi c-kinase (anti-MUSK) syndromes, 267, 274 antibodies and myasthenia aspirin gravis, 74, 76 for acute right hemiparesis, 11 antiplatelet antibodies for carotid artery dissection, 7, 14 for cerebral aneurysm, 178, 181 for cerebral aneurysm, 181 for idiopathic thrombocytopenic purpura, for cryptogenic stroke, 16 334, 339–340 for stroke, 15 antiretroviral therapy, for immune assist-control mode of ventilation, 247, 259 reconstitution infl ammatory asthma, β-adrenergic agonists for, 268, 274 syndrome, 88 ativan, for breathing diffi culties, 385 antiseptic chlorhexidine swabs, 324, 329 ATN see acute tubular necrosis aortic dissection atorvastatin, for acute right hemiparesis, 11 BP and heart rate, management of, atrial fi brillation 217, 226 ischemic stroke and, 8, 15 aphasia, 3 risk associated with, 98, 102 apnea testing, for brain death, 363, 368 Atrial Fibrillation Clopidogrel Trial with APRV see airway pressure release Irbesartan for Prevention of Vascular ventilation Events (ACTIVE-A) trial, 15 APTM see acute partial transverse myelitis atrioventricular block, lacosamide ARDS see acute respiratory distress and, 190, 200 syndrome atropine ARF see acute renal failure for C5–C6 quadriplegic patient, 416, 424 arrhythmias test, for brain death, 362, 367 cardiac arrhythmia, high-voltage burns atypical antipsychotic drugs, side effect of, and, 376, 381 191, 201 risk associated with, 98, 102 autonomy, 384, 387 arterial blood, oxygen content of, 206, auto-PEEP, 246, 258–259 210–211 autosomal dominant pattern, associated partial pressure saturation of, from PAC, with malignant hyperthermia 206, 211 abnormalities, 96, 101

arterial line transducer position, in AV DO2 (arterio-venous difference in O2 monitoring cerebral perfusion content), 272, 282 pressure, 22, 29 AVMs see arteriovenous malformations

435

66485457-66485438 www.ketabpezeshki.com

ZZakaria_87574_PTR_Index_10-06-13_433-460.inddakaria_87574_PTR_Index_10-06-13_433-460.indd 435435 66/19/2013/19/2013 4:42:254:42:25 PMPM INDEX

awake intubation, 401 beta-blockers airway algorithm for, 409–410 for BP management in stroke patients, 15 azathioprine, for transverse myelitis, 115 plus thiazide diuretics, 15 bevacizumab, for grade IV glioblastoma, 136 Bacille Calmette-Guérin (BCG) BG-12, for lymphocyte reduction, 114 vaccination, 273 Bickerstaff brain stem encephalitis, 425 Bacillus anthracis infections, ciprofl oxacin bifrontal injury, 421, 428 plus clindamycin for, 324, 329 bilateral thalamic lesions injury, and coma, baclofen 161, 168 and brain death, 362 bi-level positive airway pressure (BiPAP), 408 toxicity of, 66 BiPAP see bi-level positive airway pressure bacterial endotoxin lipopolysaccharide, bitemporal hemianospia, 28 321, 328 bladder detrusor–sphincter dysfunction, bacterial meningitis, 81, 87–88 166, 173 antimicrobial therapy for, 183, 192 bladder dysfunction, transurethral ceftriaxone for, 121 sphincterotomy for, 359 meropenem for, 194 bleeding rifampin for, 184, 193 beta blockade for, 350, 355–356 Streptococcus pneumonia-induced, infratentorial, 5, 12 dexamethasone for, 183, 192–193 polyethoxylated castor oil for, 194 trimethoprim–sulfamethoxazole for, 194 rebleeding of aneurysm, 21, 27 vancomycin for, 121, 184, 193 in trauma patients, hemostasis for, 377 warfarin for, 184, 193 vessel coagulation, endoscopy with, for bag-valve-mask (BVM) ventilation, 276 gastrointestinal disorders, 349, 355 barbiturate, 194 vitamin K for, 185, 194 and brain death, 362 blood cultures, 324, 329 -induced coma, 40, 47 blood perfusion in lung, normal distribution and brainstem abnormalities, 145, 153 of, 241, 251–252 overdosing, 99, 104 blood pressure (BP) Bartonella henselae, 329 labetalol for, 219, 227 basilar skull fracture, prophylactic management of antibiotics for, 36, 42 for acute ischemic stroke, 163, Battle’s sign, 42 171, 222, 230 BCG see Bacille Calmette-Guérin for aortic dissection, 217, 226 vaccination for hypertensive encephalopathy, BCI see blunt cardiac injury 143, 150 Behcet’s disease, HLA-B5 and, 109 in stroke patients, 7, 14–15 benzodiazepines measurement variation, by random for anxiety problems, 389 oscillations from arm-restraining mechanism of action, 191, 200 clinical technician, 398, 404–405 for neurologically injured patients, 185, 195 see also hypertension; hypotension sedative effects of, reversed by blunt cardiac injury (BCI), 375, 380 fl umazenil, 186, 195 body surface area (BSA), 206, 211 for seizure, 87 Bohr equation, 213 use of, 185, 195 Enghoff’s modifi cation of, 243, 253 beta blockade, for bleeding, 350, 355–356 BP see blood pressure

436

66485457-66485438 www.ketabpezeshki.com

ZZakaria_87574_PTR_Index_10-06-13_433-460.inddakaria_87574_PTR_Index_10-06-13_433-460.indd 436436 66/19/2013/19/2013 4:42:254:42:25 PMPM INDEX

brain death, 52, 64–65 carbamazepine, 190, 194, 199–200 absent brainstem refl exes, diagnosis of, for partial seizures, 4 363, 368 carcinomatous meningitis (CM) apnea testing for, 363, 368 CSF examination for, 134, 138 atropine test for, 362, 367 diplopia and, 134, 138 brainstem encephalitis and, 362 cardiac arrest, 161, 170–171 cardiovascular manifestations associated EEG pattern of completely with, 362, 367 recovered, 163, 171 coma, diagnosis of, 363, 368 neuron-specifi c enolase level and, 162, 171 defi ned, 64 cardiac arrhythmia, high-voltage burns and, diagnosis of, 361–369 376, 381 drug ingestion and, 362 cardiac index, 232 electroencephalogram for, 53–54, 65 cardiac output Guillain–Barré syndrome and, 362 determinants of, 205, 209–210 half-lives, 363, 368 measurement, thermodilution method of, hypothermia and, 362 206, 212 into document, entering, 364, 369 cardiac resynchronization therapy, 224, 233 time of death, 364, 369 cardiac tamponade polyurea following, 303, 312 malignant pericardial effusion and, 224, 233 pupils position during, 364, 368 pulsus paradoxus and, 222, 230 technetium-HMPAO bedside radionuclide cardiogenic shock, 224, 232 testing for, 365, 369 cardiomyopathy brainstem abnormalities, drug overdosing hypertrophic obstructive, 218, 227 and, 145, 153 Takotsubo, 27 brainstem encephalitis and brain cardiovascular abnormalities and Guillain– death, 362 Barré syndrome, 74, 77–78 Brockenbrough–Braunwald–Morrow cardiovascular physiology, 205–214 sign, 227 carotid artery dissection, aspirin for, 7, 14 bromocriptine, for Parkinson’s disease, 101 carotid-cavernous fi stula (CCF), 422, 429 bronchial injury, 373, 378 carotid endarterectomy BS see burst-suppression for right carotid artery stenosis, 178, 181 BSA see body surface area for transient ischemia attack, 3, 11 “bucks-the-ventilator,” 247, 259 carotid stenting, for myocardial infarction, 11 burr hole evacuation, 36, 42 case–control study, 392, 395 burst-suppression (BS), 67 catecholamine storm, 362 BVM see bag-valve-mask ventilation catheter tunneling, for meningitis with extra-ventricular drain, 403, 411 C7-T1 jumped facets, for cerebrovascular cat scratch disease, 329 injury, 37, 43 cavernous malformations, 426 calcium channel blockers cavernous sinus thrombosis, 429 for primary pulmonary CCF see carotid-cavernous fi stula hypertension, 270, 280 CEEG see continuous electroencephalogram for reversible cerebral vasosconstriction monitoring syndrome, 427 cefepime Call–Fleming syndrome, 16 toxicity of, 66 capillary telangiectasias, 426 for urinary tract infections, 188, 197–198

437

66485457-66485438 www.ketabpezeshki.com

ZZakaria_87574_PTR_Index_10-06-13_433-460.inddakaria_87574_PTR_Index_10-06-13_433-460.indd 437437 66/19/2013/19/2013 4:42:254:42:25 PMPM INDEX

ceftriaxone subarachnoid blood more than 1 mm for bacterial meningitis, 121 thickness, risk of, 21, 26 for urinary tract infections, 188, 198 cerebral venous sinus thrombosis (CVST), central herniation, 167 420, 427–428 central pontine cerebral venous thrombosis alcohol abuse and, 300, 308 heparin drip for, 13 hypernatremia and, 300, 308 noncontrast head computed tomography hyponatremia and, 300, 308 for, 6, 13 liver transplantation treated with cerebrospinal fl uid (CSF) tacrolimus and, 300, 308 examination myelinolysis, 113, 118 for carcinomatous meningitis, 134, 138 central venous catheter (CVC), for relapsing remitting multiple insertion of, 322 sclerosis, 116 cerebellar herniation, 159, 167 otorrhea, for posttraumatic cerebral abscesses, 89–90 CSF fi stulae, 37, 43 cerebral angiography rhinorrhea, for posttraumatic for carotid-cavernous fi stula, 6, 13 CSF fi stulae, 37, 43 for infratentorial bleeds, 12 cerebrovascular injury for peri-mesencephalic subarachnoid C7-T1 jumped facets for, 37, 43 hemorrhage, 23, 30–31 subluxation with spinal cord transaction for vasospasm, 21, 25–26 for, 37, 43 cerebral anoxia cervical spine after trauma, evaluation of, spontaneous respirations, control of, 371, 377 146, 153 chemical pneumonitis, 274 cerebral arteries, decreased carbon dioxide chest bellows component, of respiratory tension in, 160, 168–169 system, 269, 277 cerebral herniation, ammonia levels chest escharotomies, for circumferential associated with, 141, 149 torso burns, 376, 381 cerebral ischemia, delayed, 25–26 chest wall muscular rigidity, fentanyl and, cerebral perfusion pressure (CPP), 12 186, 195 transducer position in monitoring, Cheyne–Stokes breathing, 230, 245, 256 22, 29 chronic alcoholism and osmotic cerebral salt wasting (CSW), 22, 29 demyelination syndrome, 124, 129 for hyponatremia, 289, 294 chronic infl ammatory demyelinating for polyuria, 289, 294 polyradiculoneuropathy (CIDP), 76 syndrome chronic obstructive pulmonary disease distinguished from SIADH, 123, 128 (COPD), 250 positive sodium balance with chronic stable atrial fi brillation poststroke, hypertonic saline, restoration heparin for, 185, 194 of, 124, 128 Chvostek sign, 306 cerebral vasculature, 80, 85 cidofovir, for CMV retinitis, 187, 197 cerebral vasospasm, 24–25 CIDP see chronic infl ammatory cerebral angiography for, 21, 25–26 demyelinating hyporefl exia and, 21, 26 polyradiculoneuropathy in middle cerebral artery, transcranial ciprofl oxacin doppler ultrasound for, 23, 31 for Bacillus anthracis infections, 324, 329

438

66485457-66485438 www.ketabpezeshki.com

ZZakaria_87574_PTR_Index_10-06-13_433-460.inddakaria_87574_PTR_Index_10-06-13_433-460.indd 438438 66/19/2013/19/2013 4:42:254:42:25 PMPM INDEX

plus clindamycin, 324, 329 for stroke, 15 circumferential torso burns, chest for subarachnoid hemorrhage, 19, 24 escharotomies for, 376, 381 without contrast, for bowel obstruction, cisatracurium, for sever renal dysfunction, 352, 358–359 189, 199 computed tomography angiography (CTA) clindamycin for cerebral vasospasm, 21, 26 for Bacillus anthracis infections, 324, 329 computer tomographic perfusion (CTP) plus ciprofl oxacin, 324, 329 for cerebral vasospasm, 21, 26 clopidogrel confi dence interval, 393, 395 for cerebral aneurysm, 181 conivaptan, for hyponatremia, 124, 129, for stroke, 15 311–312 CLOSURE trial, 16 CONSCIOUS studies, 1 and 2, 25 CM see carcinomatous meningitis consciousness, vegetative state of, 164, 172 CMV retinitis, cidofovir for, 187, 197 continuous electroencephalogram (CEEG) CNS infections, 79–91 monitoring coagulopathy, 162, 171 in ICU, rationale for, 62, 68 following multisystem trauma, risk factor for subarachnoid hemorrhage, 23, 30 for, 372, 377–378 continuous positive airway pressure (CPAP) and hypothermia, 309 for respiratory distress, 401, 408–409 intracerebral hemorrhage secondary to, continuous veno-venous hemofi ltration 336, 341–342 (CVVH) Cochrane Injuries Group, 45 for renal failure, 145, 152 Columbia University Comprehensive contractility, 209, 211–212 Epilepsy Center convulsive status epilepticus, Status Epilepticus Adult Treatment electroencephalogram for, 57, 66 Protocol, 64 COPD see chronic obstructive pulmonary coma disease alpha, 55, 65–66, 165, 172 corticosteroids barbiturate-induced, 40, 47 for acute spinal cord injury, 37, 43–44 bilateral thalamic lesions injury and critical illness polyneuropathy, 190, 199 and, 161, 169 insuffi ciency, diagnostic criteria for, 302, 311 diagnosis of, for brain death, 363, 368 for metastatic epidural spinal cord myxedema, 125, 127, 129 compression, 133, 137–138 spindle, 54, 65 for multiple sclerosis, 111, 116 communicating bad news, 386, 390 for ophthalmoplegia, 132, 136 community-acquired pneumonia, 271, 282 for transverse myelitis, 115 piperacillin–tazobactam for, 321, 328, 329 CORTICUS study, 282 vancomycin for, 321, 328, 329 COSSACS trial, 14 compartment syndrome of both forearms, cost containment, 386, 390 high-voltage burns and, 376, 381 CPAP see continuous positive airway computed tomography (CT) scan pressure of brain before lumbar puncture, 83, 91 CPP see cerebral perfusion pressure for cerebral venous thrombosis, 6, 13 craniotomy for cervical spine following trauma, 371, 377 for acute epidural hematoma, 35, 41 for hemicraniectomy, 3, 10 osteoplastic, 42 for partial seizures, 4, 10–11 CREST trial, 11

439

66485457-66485438 www.ketabpezeshki.com

ZZakaria_87574_PTR_Index_10-06-13_433-460.inddakaria_87574_PTR_Index_10-06-13_433-460.indd 439439 66/19/2013/19/2013 4:42:254:42:25 PMPM INDEX

Creutzfeldt–Jakob disease, dantrolene, for malignant hyperthermia, 98, electroencephalogram for, 57, 66, 67 101, 102 critical care, research principles in, 391–395 DC see decompressive hemicraniectomy critical illness neuropathy, and DCD see donation after cardiac death succinylcholine-induced DCI see delayed cerebral ischemia hyperkalemia, 72, 75–76 decision-making capacity, assessment of, critical illness polyneuropathy, 383, 387 corticosteroids and, 190, 199 decompensated congestive heart failure, critical illness-related corticosteroid 219, 227 insuffi ciency, 311 decompressive hemicraniectomy (DC) cross-sectional study, 395 complications after, 39, 46 cryoprecipitate infusion, for platelet for subdural hematoma, 42 dysfunction, 338, 340, 342 for traumatic brain injury, 38, 45 cryptococcal meningitis, intracranial decreased afterload ventilation, effects on pressures in, treatment for, 81, 87 cardiovascular system, 400, 407 cryptogenic stroke, 16 delayed cerebral ischemia (DCI), 25–26 CSF see cerebrospinal fl uid secondary to vasospasm, prevention of, CSF HSV PCR study, for herpes simplex 175, 179 encephalitis, 79, 85 demeclocycline, for hyponatremia, 124, CSW see cerebral salt wasting 128–129 CT see computed tomography scan dementia CTA see computed tomography angiography Alzheimer’s, 83 cTnT see troponin T frontotemporal, 75 CTP see computer tomographic perfusion demyelinating diseases, 107–118 Cushing’s syndrome, 127 Denver criteria, for cerebrovascular injury, 43 hypertension and, 302, 312 desmopressin hypokalemia and, 302, 312 for acute liver failure, 145, 152 infection by opportunistic organisms and, for diabetes insipidus, 121, 126 302, 312 for platelet dysfunction, 335, 340 CVC see central venous catheter developmental venous anomalies (DVA), 426 CVST see cerebral venous sinus thrombosis dexamethasone CVVH see continuous veno-venous for seizure, 131, 135 hemofi ltration for Streptococcus pneumonia-induced cyclosporine, for posterior reversible bacterial meningitis, 183, 192–193 encephalopathy syndrome, 115 for systemic lupus erythematosus, 303, 313 cyproheptadine, for neuroleptic malignant for vasogenic edema, 168 syndrome, 96, 100–101 DI see diabetes insipidus diabetes insipidus (DI), 121, 126 dabigatran (Pradaxa), 338, 344 desmopressin for, 121, 126 anticoagulant effects of, reversing, 185, 195 diabetic ketoacidosis, 127 mechanism of action, 185, 194 dialysis, for platelet dysfunction, 340 route of administration of, 185, 195 diazepam, for status epilepticus, 64 dabigatran etexilate, for renal insuffi ciency, DIC see disseminated intravascular 217, 226 coagulation DAI see diffuse axonal injury diffuse axonal injury (DAI), 37, 43, 421, 428 Dalton’s Law, 241, 252 MRI diffusion-weighted imaging for, 38, 44

440

66485457-66485438 www.ketabpezeshki.com

ZZakaria_87574_PTR_Index_10-06-13_433-460.inddakaria_87574_PTR_Index_10-06-13_433-460.indd 440440 66/19/2013/19/2013 4:42:254:42:25 PMPM INDEX

diffuse encephalopathy, EEG patterns ECI see electrocerebral inactivity associated with, 55, 66 EC-IC bypass. See right-middle cerebral diplopia and carcinomatous artery, surgical bypass of, for meningitis, 134, 138 Moyamoya disease disseminated intravascular coagulation eclampsia (DIC), 146, 153, 337, 343 magnetic resonance imaging for, 143, 150 distal bypass, use in spinal cord infarction preeclampsia, 142, 149 surgery, 17 EDH see epidural hematoma diuretics, for metabolic alkalosis, 290, 295 EEG see electroencephalogram DNR see do-not-resuscitate orders EGDT see early goal-directed therapy dobutamine, for elevating BP in cerebral Einthoven’s triangle, 405 vasospasm, 29–30 ejection fraction, 206, 211–212 dominant hemispheric infarct and electrocardiogram (ECG) consciousness disturbance, 160, 168 for acute pericarditis, 221, 229 donation after cardiac death (DCD) for Duchenne muscular dystrophy, 221, 229 organ donation, 362, 367–368 for Friedreich’s ataxia, 221, 229 do-not-resuscitate (DNR) orders, 5, 12–13 for limb girdle muscular dystrophy, 221, 229 dopamine electrocerebral inactivity (ECI), 54, 65, 66 effect on pulmonary capillary wedge electroencephalogram (EEG) pressure, 272, 282 associated with diffuse encephalopathy, for elevating BP in cerebral vasospasm, 29 55, 66 for patients presenting in septic shock, for brain death, 53–54, 65 400, 407–408 requirements for, 53, 65 doxycycline burst-suppression, 67 for Rickettsial disease, 327 continuous see continuous for tick-borne infectious diseases, 327 electroencephalogram drowning, 268, 275 for convulsive status epilepticus, 57, 66 drug overdosing for Creutzfeldt–Jakob disease, 57, 66 acetaminophen, 96, 100 for hypothermia, 60, 67 acetylcholine esterase inhibitors, 72, 76 of left carotid endarterectomy, 61, 67–68 barbiturate, 99, 104 for lithium toxicity, 57, 66 and brainstem abnormalities, 145, 153 monitoring, for left hemispheric cerebral and consciousness disturbance, 160, 168 infarction, 51, 63 heparin, 338, 344 for nonconvulsive focal right frontal- opiate, 98, 101 temporal seizure, 59, 67 tricyclic antidepressant, 97, 101, 145, 153 for postanoxic status epilepticus, 418, 426 valproic acid, 145, 153 recording, minimizing interference Duchenne muscular dystrophy, ECG for, during, 400, 408 221, 229 for right frontal subdural DVA see developmental venous anomalies hemorrhage, 62, 68 dysphagia, 176, 179–180 triphasic waves, 58, 66 dyspnea, 271, 282 electrolyte disorders, 299–315 embolic stroke, infective endocarditis and, early goal-directed therapy (EGDT), 322, 328 80, 85–86 ECASS III trial, 15 EN see enteral nutrition ECG see electrocardiogram enalapril, for hypertension, 27, 143, 151

441

66485457-66485438 www.ketabpezeshki.com

ZZakaria_87574_PTR_Index_10-06-13_433-460.inddakaria_87574_PTR_Index_10-06-13_433-460.indd 441441 66/19/2013/19/2013 4:42:254:42:25 PMPM INDEX

encephalopathies, 141–155 cost containment, 386, 390 diffuse, 55, 66 decision-making capacity, 383, 387 Hashimoto, 66 family meeting, sources of confl ict hepatic, 144–146, 151–153 in, 386, 390 hypertensive, 143, 146, 150, futile interventions, 385, 388–389 153–155, 221, 229 guardian (as surrogate decision maker), metabolic, 160, 168 384, 388 posterior reversible, 109, 115, 142, 150, informed consent, 384, 387–388 153–154 rationing care, 386, 390 progressive multifocal respect (for moral and religious values), leukoencephalopathy, 88, 110, 116 383, 387 uremic, 143, 146, 151, 153 substituted judgment, 384, 388 Wernicke’s, 147, 154 etomidate, for adrenal insuffi ciency, 302, endocrine disorders, 299–315 311–312 endoscopy with bleeding vessel coagulation, euthermia, 8 for gastrointestinal disorders, 349, 355 euthyroid sick syndrome see nonthyroidal endovascular stenting, for Moyamoya illness syndrome disease, 12 euvolemia, for delayed cerebral ischemia,

end-tidal carbon dioxide (ETCO2), 402, 410 175, 179 Enghoff’s modifi cation, of Bohr equation, euvolemic hyponatremia, 128 243, 253 evaporative insensible fl uid loss, 240, enteral nutrition (EN) 250–251 initiation of, 418, 425 EVD see external ventricular drainage see also parenteral nutrition excessive cholinergic activity, 99, 103 ependymoma, 137 expiatory reserve volume (ERV), 269, 278 ephalosporin, for Alzheimer’s dementia, 83 expiratory fl ow force, 240, 250, 270, 280 epidural hematoma (EDH) external ventricular drainage (EVD), 168 acute, craniotomy for, 35, 41 placement, for basal ganglia hemorrhage, epinephrine, for patients presenting in septic 163, 171 shock, 408 steroid-induced complications associated EPS see extrapyramidal symptoms with, 177, 180 ERV see expiatory reserve volume extracranial herniation, 167 Escherichia coli, 86 extrapontine myelinolysis syndromes and nosocomial urinary tract infection, alcohol abuse and, 300, 308 188, 197 hypernatremia and, 300, 308 esmolol, for hypertyension, 27 hyponatremia and, 300, 308 esophageal varices, 350, 355–356 liver transplantation treated with estrogen treatment, for platelet tacrolimus and, 300, 308 dysfunction, 340 reading, for Fisher Grade 3 subarachnoid

ETCO2 see end-tidal carbon dioxide hemorrhage, 401, 409 ethambutol, for tuberculous meningitis, 91 weaning, 403, 411 ethanol withdrawal, 100 extrapyramidal symptoms (EPS) ethical and legal aspects of, critical care quetiapine for, 191, 201 medicine, 383–390 autonomy, 384, 387 facioscapulohumeral muscular dystrophy, communicating bad news, 386, 390 221, 229

442

66485457-66485438 www.ketabpezeshki.com

ZZakaria_87574_PTR_Index_10-06-13_433-460.inddakaria_87574_PTR_Index_10-06-13_433-460.indd 442442 66/19/2013/19/2013 4:42:254:42:25 PMPM INDEX

family meeting, sources of confl ict in, 386, 390 for acute liver failure, 145, 152 FAST trail, 13 for intracerebral hemorrhage, 342–343 fat embolism syndrome (FES) Friedreich’s ataxia diagnosis of, 373, 379 ECG for, 221, 229 and hypoxia, 373, 378 idebenone for, 222, 230 fentanyl and chest wall muscular rigidity, frontotemporal dementia (FTD), 75 186, 195 FTD see frontotemporal dementia FES see fat embolism syndrome FTY720 (fi ngolimod), for lymphocyte FEV1/FVC ratio, 243, 255 reduction, 114 FFP see fresh-frozen plasma functional residual capacity (FRC), fi brinolytic therapy, for myocardial 248, 262 infarction, contraindications fundamental equation (cardiovascular to, 224, 232 physiology), 210 Fick formula, 220, 228 futile interventions, 385, 388–389 Fisher Grade 3 subarachnoid hemorrhage, f/VT see rapid, shallow breathing index extra-ventricular drain reading for, 401, 409 gadolinium infusion, for stage-3 kidney fl avivirus infection and meningitis, 82, 88 disease, 289 fl uconazole ganciclovir and myelosuppression, 187, 197 plus amphotericin B, 81, 87 gastrointestinal disorders, 349–360 for cryptococcal CNS infection in HIV endoscopy with bleeding vessel patients, 81, 87 coagulation for, 349, 355 plus fl ucytosine, 81, 87 GBS see Guillain–Barré syndrome fl ucytosine GCA see giant cell arteritis plus amphotericin B, 81, 87 GCS see Glasgow Coma Scale for cryptococcal CNS infection in HIV generalized periodic epileptiform discharges patients, 81, 87 (GPEDs), 66 plus fl uconazole, 81, 87 generalized systemic infl ammatory response fl udrocortisones, for hyponatremia, 29 syndrome, 282 fl uid overload, 268 genitourinary disorders, 349–360 fl uid resuscitation, for high-voltage burns, gentamycin 375, 380–381 for infectious endocartitis, 325, 330 fl umazenil for urinary tract infections, 198 benzodiazepine sedative effects reversed giant cell arteritis (GCA), 115 by, 186, 195 Giemsa statins, 80, 86 for cholinergic poisoning, 103–104 Glasgow Coma Scale (GCS), 5, 13, 36 focal neurologic defi cit, 160, 168 glucocorticoids Foley catheter, for bifrontal cerebral for myxedema coma, 129 contusions, 397 for reversible cerebral vasosconstriction fosphenytoin syndrome, 427 advantages of, 190, 200 glycemic control, for patients in ICU, for status epilepticus, 64 302, 312 four-vessel angiography, for brain death, glycopyrolate, for breathing 363, 368 diffi culties, 385, 389 FRC see functional residual capacity GPEDs see generalized periodic epileptiform fresh-frozen plasma (FFP) discharges

443

66485457-66485438 www.ketabpezeshki.com

ZZakaria_87574_PTR_Index_10-06-13_433-460.inddakaria_87574_PTR_Index_10-06-13_433-460.indd 443443 66/19/2013/19/2013 4:42:254:42:25 PMPM INDEX

grade IV glioblastoma, adjuvant decompressive chemotherapy for, 132, 135–136 for subdural hematoma, 42 Graham’s Law, 251 for traumatic brain injury, 38, 45 Grave’s disease, 127 hemodialysis guardian (as surrogate decision maker), for addressing coagulopathy, 177, 180 384, 388 catheters, 398, 406 Guillain–Barré syndrome (GBS), 74, 76–77, for clearing ethylene removal, 288, 294 113, 117, 425 for lithium toxicity, 287, 293 and brain death, 362 for renal insuffi ciency, 142, 149 cardiovascular abnormalities and, 74, 77–78 hemorrhage(s) intravenous immunoglobulin for, 74, 77 intracerebral see intracerebral hemorrhage lumbar puncture for, 74, 77 intraventricular, 13 mechanical ventilating testing for, 245, 256 left putamen, 6 succinylcholine for, 189 remote cerebellar, 178, 181 subarachnoid see subarachnoid Haemophilus infl uenzae, 86 hemorrhage Hagan–Poiseuille equation, 406 hemorrhagic stroke, infective endocarditis haloperidol, for hepatic encephalopathy, and, 80, 85–86 145, 152 hemostasis, for bleeding in trauma HAMLET–DESTINY–DECIMAL pooled patients, 377 analysis, of hemicraniectomy, 10 Henry’s Law, 251 HAPE see high-altitude pulmonary edema heparin Harvard Criteria, for diagnosis of brain for chronic stable atrial fi brillation death, 366 poststroke, 185, 194 Hashimoto encephalopathy, 66 drip, for cerebral venous thrombosis, 13 HE see hepatic encephalopathy and hyperkalemia, 291, 295 Health Insurance Portability and -induced skin necrosis, 341 Accountability Act, 392, 395 low-molecular-weight see low-molecular- heart failure and valve replacement surgery weight heparin delay, 83, 89 overdose, protamine sulfate for, 338, 344 heart rate, 205, 209 protamine dose to neutralize, 185, 194 management, for aortic dissection, 217, 226 unfractionated see unfractionated heparin heat exhaustion, 98, 102–103 heparin-induced thrombocytopenia (HIT), Heliox in postextubation stridor, physiologic 223, 231, 336, 340–341 benefi t of, 269, 278 low-molecular weight heparin for, 341 HELLP syndrome, 153 type-1, 340 hemangioblastoma, 137 type-2, 341 hematologic disorders, acute, 333–344 unfractionated heparin, hematoma, surgical evacuation of discontinuation of, 336 for infratentorial bleeds, 5, 12 hepatic encephalopathy (HE) for intracerebral hemorrhage, 5, 12 grades of, 144, 151 hemianopsia haloperidol for, 145, 152 bitemporal, 28 physical examination fi ndings for, 145, 152 homonymous, 28 prognosis postcardiac arrest without hemicraniectomy, 3, 10 hypothermia, 146, 153 for cerebral venous thrombosis, 13 hepatic failure, acute, 144, 151

444

66485457-66485438 www.ketabpezeshki.com

ZZakaria_87574_PTR_Index_10-06-13_433-460.inddakaria_87574_PTR_Index_10-06-13_433-460.indd 444444 66/19/2013/19/2013 4:42:254:42:25 PMPM INDEX

herniation hypercarbia, 251 central, 167 hyperglycemic hyperosmolar nonketotic cerebellar, 159, 167 state see hyperosmolar hyperglycemic extracranial, 167 state subfalcine, 167 hyperkalemia, heparin infusion and, 291, 295 tonsillar, 167 hypermagnesemia, 299, 306–307, 403, 411 uncal, 167 hypernatremia, 285, 292 upward tentorial, 167 and central pontine, 300, 308 herpes simplex encephalitis (HSE) cerebral salt wasting for, 289, 294 CSF HSV PCR study for, 85 and extrapontine myelinolysis syndromes, high-altitude pulmonary edema (HAPE), 231 300, 308 high-protein diet, for hyponatremia, 124, 128 nephrogenic diabetes insipidus and, 289, high-voltage burns 294–295 and cardiac arrhythmia, 376, 381 and osmotic demyelination, 300, 308 and compartment syndrome of both hyperosmolar, nonketotic dehydration forearms, 376, 381 syndrome (HONK), 123, 127 fl uid resuscitation for, 375, 380–381 hyperosmolar hyperglycemic state, 304, 314 and myoglobinemia, 376, 381 hyperreninemic hypoaldosteronism and traumatic injuries to spine and syndrome extremities, 376, 381 in SAH patients, clinical manifestation of, HIT see heparin-induced thrombocytopenia 305, 315 HLA-B1 and Behcet’s disease, 109 hypertension HLA-B5 and Behcet’s disease, 109 and brain death, 362, 367 HOCM see hypertrophic obstructive and Cushing’s syndrome, 302, 312 cardiomyopathy intra-abdominal, 379 homonymous hemianopsia, 28 primary pulmonary, 270, 280 Horner’s syndrome, 14, 28 rebound intracranial, 161, 169 HONK see hyperosmolar, nonketotic see also blood pressure dehydration syndrome hypertensive encephalopathy, 221, 229 H. pylori infection and gastrointestinal blood pressure in, management guidelines disorders, 355, 357 of, 143, 150 HSE see herpes simplex encephalitis defi ned, 154 hydralazine, for hypertension during diagnosis of, 146, 153–154 pregnancy, 143, 151 differential diagnosis of, 147, 154–155 hydrocephalus, 20, 180 hypertensive urgency, 231 with elevated ICP signs, hyperthermia, anesthesia-related malignant, communicating, 160 301, 308–309 hydrochlorothiazide, for acute right hyperthyroidism, 127 hemiparesis, 11 Lugol’s solution for, 123, 127 hydrocortisone, for Sheenan’s syndrome, propranolol for, 123, 127 301, 310–311 propylthiouracil for, 123, 127 hydrostatic pulmonary edema, 275 hypertonic saline, for intracranial pressure, hyperamylasemia, 353, 359 37, 43 hypercalcemia hypertrophic obstructive cardiomyopathy ionized, 306 (HOCM), 218, 227 and metastatic lung cancer, 304, 314 hypocalcemia, ionized, 306

445

66485457-66485438 www.ketabpezeshki.com

ZZakaria_87574_PTR_Index_10-06-13_433-460.inddakaria_87574_PTR_Index_10-06-13_433-460.indd 445445 66/19/2013/19/2013 4:42:254:42:25 PMPM INDEX

hypoglycemia, 127 pulmonary contusion and, 373, 378 hypokalemia, 415–416, 423–424 transfusion-related acute lung injury and, and Cushing’s syndrome, 302, 312 373, 378 hyponatremia and central pontine, 300, 308 IAP see intra-abdominal pressures etiologies of, 124, 128 ibuprofen, for bacterial meningitis, 81 euvolemic, 128 ICH see intracerebral hemorrhageICP see and extrapontine myelinolysis syndromes, intracranial pressure 300, 308 idebenone, for Friedreich’s ataxia, 222, 230 fl uid restriction to reduce free water idiopathic thrombocytopenic purpura (ITP) intake, 304, 313–314 antiplatelet antibodies for, 334, 339–340 and osmotic demyelination, 300, 308 IE see infective endocarditis secondary to diuretic use, 113 I:E ratio, 246, 258 and subarachnoid hemorrhage, 22, 29, immune reconstitution infl ammatory 123, 128 syndrome (IRIS), 88 tolvaptan for, 124, 129 IMSCTs see intramedullary spinal cord hypopituitarism, radiation therapy tumors and, 122, 127 incentive spirometry, 244, 255 hyporefl exia, 299, 306–307 infectious diseases, 319–330 and cerebral vasospasm, 21, 26 infective endocarditis (IE) hypotension embolic stroke and, 80, 85–86 and brain death, 362, 367 hemorrhagic stroke and, 80, 85–86 duration of, prior to antimicrobial therapy meningitis and, 80, 85–86 in septic shock, 321, 328 inferior vena cava, 404 second hour of, 321, 328 blood streaming from, 207, 212 see also blood pressure fi lter placement, for venous hypothermia, 96, 101, 147, 154 thromboembolism, 221, 230 acidosis and, 309 infl ammatory diseases, 107–118 after cardiac arrest, 142, 150 infl ammatory myopathy and and brain death, 362 succinylcholine-induced cardiac arrest and, 161, 170–171 hyperkalemia, 74, 75–76 coagulopathy and, 309 informed consent, 384, 387–388 electroencephalogram for, 60, 67 infracted posterior papillary muscle, increased ICP and, 162, 170–171 murmur caused by, 233 induced, for perihematoma edema, 6, 13 infratentorial bleeds shivering and, 162, 170 cerebral angiography for, 12 hypothyroidism, 122, 127, 304, 315 surgical hematoma evacuation for, 5, 12 and hyponatremia, 123, 128 inhalational anthrax, 323, 329 lithium effects on, 127 inhalation injury, 376, 381 hypovolemia, 124 INR see international normalized ratio hypoxemia, 251, 282 inspiratory airway pressure, 248, 262 peripheral chemo-receptors for, 239, 249 inspiratory capacity, 244, 255 hypoxia inspiratory forces, 270, 280 aspiration and hypoxia and, 373, 378 inspiratory gas distribution to apneic brain structures sensitivity to, 147, 154 patients, 248, 261 fat embolism syndrome and, 373, 378 inspiratory process, 277

446

66485457-66485438 www.ketabpezeshki.com

ZZakaria_87574_PTR_Index_10-06-13_433-460.inddakaria_87574_PTR_Index_10-06-13_433-460.indd 446446 66/19/2013/19/2013 4:42:254:42:25 PMPM INDEX

inspiratory reserve volume (IRV), 269, 278 IRIS see immune reconstitution institutional review board (IRB) infl ammatory syndrome expedited review, 392, 395 IRV see inspiratory reserve volume insulin plus nebulized beta-adrenergic drug, ISAT see International Subarachnoid for metabolic acidosis, 301, 309–310 Aneurysm Trial international normalized ratio (INR) Ischemic colitis, symptoms of, 352, 358 elevated, treatment of, 341–342 ischemic stroke International Subarachnoid Aneurysm Trial BP management for, 222, 230 (ISAT), 22, 28 fatal brain edema, risk factor for, 162, 171 intra-abdominal hypertension, 379 isocitrate dehydrogenase mutation, 135–136 intra-abdominal pressures (IAP), 404 isoniazid intracerebral hemorrhage (ICH) for respiratory failure, 323, 329 anticoagulant-associated, 160, 168 for tuberculous meningitis, 91 with intraventricular involvement of the isotonic IV fl uids, for rhabdomylosis, 101 third and fourth ventricle, 160 ITP see idiopathic thrombocytopenic phenylephrine for, 188, 198 purpura secondary to coagulopathy, 336, 341–342 IV diclofenac, for controlling fever after spontaneous hypertensive, 160, 168 left-middle cerebral artery surgical hematoma evacuation for, 5, 12 infarction, 15 intracranial hypertension, rebound, 161, 169 IVIg see intravenous immunoglobulin intracranial pressure (ICP), 39, 46

elevation, nitroprusside-induced, jugular venous bulb saturation (SjvO2), 402, 163, 172 410–411 increased, and hypothermia, 162, 170–171 justice (fairness), 386, 390 mannitol effect on, 36, 41–42, 400, 408 juvenile absence epilepsy monitor, 160, 168 phenytoin for, 52, 64 position in monitoring cerebral perfusion pressure, 22, 29 K08, 395 monitoring, indications for, 39, 46 K23, 394–395 sodium chloride for, 186, 196 K99/R00, 394–395 waveforms, 159, 168 King’s College Criteria, 350, 356 intramedullary spinal cord tumors Klebsiella pneumonia, 322, 328 (IMSCTs), 133, 136–137 kyphoscoliosis, 244, 255 intraventricular hemorrhage, intraventricular tPA for, 13 labetalol intravenous immunoglobulin (IVIg) for blood pressure, 219, 227 for bacterial meningitis, 81, 88 for hypertension, 27 for Guillain–Barré syndrome, 74, 77 during pregnancy, 143, 151 for myasthenia gravis, 73, 76 lacosamide and atrioventricular block, intraventricular tissue plasminogen 190, 200 activator, for intraventricular lactated Ringer’s bolus, applications of, hemorrhage, 13 during hypovolemic shock, 372, 378 ionized hypercalcemia, 306 Lambert–Beer law, 404 ionized hypocalcemia, 306 Lambert–Eaton myasthenic syndrome, 139 ipecac-induced myopathy, 72, 75 lap belt injuries, vasodilation for, 173 IRB see institutional review board laquinimod, for lymphocyte reduction, 114

447

66485457-66485438 www.ketabpezeshki.com

ZZakaria_87574_PTR_Index_10-06-13_433-460.inddakaria_87574_PTR_Index_10-06-13_433-460.indd 447447 66/19/2013/19/2013 4:42:254:42:25 PMPM INDEX

lasix, 223, 231, 385, 389 treated with tacrolimus for metabolic alkalosis, 290, 295 and central pontine, 300, 308 LeForte II fracture, 43 and extrapontine myelinolysis LeForte III fracture, 43 syndromes, 300, 308 left carotid endarterectomy, and osmotic demyelination, 300, 308 electroencephalogram of, 61, 67–68 LMWH see low-molecular weight heparin left hemispheric cerebral infarction, lorazepam, 101 acute, 51, 63 and malignant hyperthermia, 102 left-middle cerebral artery infarction, 8, 15 for status epilepticus, 64 left temporal lobe, periodic lateralized loss of refl uxes, 76 epileptiform discharges in, 56, 66 “lover’s triad,” 373, 379 left upper lead, 398, 405 low-molecular weight heparin (LMWH) Legionella pneumophila, 329 for chronic stable atrial fi brillation lenticulostriates, hypertrophy of, 9, 16 poststroke, 185, 194 leukocytosis, 147, 154 for thrombocytopenia, 341 levetiracetam see also heparin and intracerebral hemorrhage, 12 low-T3 syndrome see nonthyroidal illness for seizure, 131, 135 syndrome lidocaine Lugol’s solution, for hyperthyroidism, and brain death, 362 123, 127 for topical anesthesia of the airway, lumbar puncture 268, 277 brain CT scan before, 83, 91 limb girdle muscular dystrophy, ECG for, for Guillain–Barré syndrome, 74, 77 221, 229 postural headache following, 79, 84 limbic encephalitis, 139 Lundberg A wave, 161, 169 Lindegaard index, 402, 411 LV stroke work index (LVSWI), 209 linezolid, 319 LVSWI see LV stroke work index lisinopril, for acute right hemiparesis, 11 lymphocyte traffi cking, reduction Listeria monocytogenes, 80, 86 of, 107, 114 -induced bacterial meningitis, ampicillin for, 183, 192 mafenide acetate (Sulfamylon), for burns, West Nile virus infection and, 88 375, 380 lithium magnesium, for reversible cerebral effects on hypothyroidism, 127 vasosconstriction syndrome, 427 toxicity, 100 magnesium sulfate, for preeclampsia, 145, electroencephalogram for, 57, 66 153, 155 hemodialysis for, 287, 293 magnetic resonance imaging (MRI) vasopressin for, 300, 307–308 of brain without contrast, 177, 180–181 lithium carbonate, for amyotrophic lateral diffusion, for cerebral vasospasm, 21, 26 sclerosis, 71, 75 diffusion-weighted imaging, for diffuse liver cirrhosis and osmotic demyelination axonal injuries, 38, 44 syndrome, 124, 129 for eclampsia, 143, 150 liver failure, acetaminophen overdose and, for posterior reversible encephalopathy 96, 100 syndrome, 153–154 liver transplantation, 350, 356 magnetic resonance venography, for venous for acute liver failure, 144, 151 sinus thrombosis, 24

448

66485457-66485438 www.ketabpezeshki.com

ZZakaria_87574_PTR_Index_10-06-13_433-460.inddakaria_87574_PTR_Index_10-06-13_433-460.indd 448448 66/19/2013/19/2013 4:42:254:42:25 PMPM INDEX

malaria, 80, 86–87 infective endocarditis and, 80, 85–86 malignant glioma and venous posttraumatic CSF fi stulae and, 43 thromboembolism, 333, 339 tuberculous, 83, 90–91 malignant hyperthermia (MH), 100 West Nile virus infection and, 88 autosomal dominant, ryanodine receptor meropenem patterns associated with molecular for Listeria monocytogenes-induced abnormalities in, 96, 101 bacterial meningitis, 194 malignant pericardial effusion and cardiac for pseudomonal infections, 326, 330 tamponade, 224, 233 MESCC see metastatic epidural spinal cord Mallory–Weiss syndrome, 350, 355 compression malnutrition and osmotic demyelination metabolic acidosis syndrome, 124, 129 diuretics for, 290, 295 mannitol insulin plus nebulized beta-adrenergic for intracranial pressure, 36, 37, 41–42, drug for, 301, 309–310 43, 47, 400, 408 massive PRBC transfusion and, 290, 295 serum testing, 307 metabolic disorders, 66 Marburg’s variant, of demyelinating disease, metabolic encephalopathy and 110, 116 consciousness disturbance, 160, 168 massive PRBC transfusion and metabolic metastatic brain tumors, in adults, 132, 136 alkalosis, 290, 295 metastatic epidural spinal cord compression maxillary sinusitis, 323, 329 (MESCC) maximal inspiratory pressure (MIP) see symptoms of, 133, 137 negative inspiratory force treatment for, 133, 137–138 McDonald diagnostic criteria, for multiple metastatic lung cancer, hypercalcemia and, sclerosis, 111, 116 304, 314 measles and acute disseminated methyldopa, for hypertension during encephalomyelitis, 112, 117 pregnancy, 143, 151 mechanical obstruction, 274 methylene blue, for mechanical ventilation, 239–262 methemoglobinemia, 104 assist-control mode of, 247, 259 methylprednisolone pressure-limited, 246, 258 for acute spinal cord injury, 43 volume-limited, 246, 257 for transverse myelitis, 115 Medicare, 386, 390 metronidazole, for Alzheimer’s mefl oquine, 80 dementia, 83 MELAS see mitochondrial encephalopathy MG see myasthenia gravis with lactic acidosis and stroke-like MGMT see O-6-methylguanine episodes methyltransferase Memphis criteria, for cerebrovascular MH see malignant hyperthermia injury, 43 MI see myocardial infarction meningeal carcinomatosis, 417, 424–425 midazolam meningioma, 133, 136–137 and brain death, 362 meningitis for short-term sedation, 186, 195 aseptic, 109 for status epilepticus, 64 bacterial. See bacterial meningitis midline shift after initial injury, persistent cryptococcal, 81, 87 effects of, 421, 428 fl avivirus infection and, 82, 88 Miller–Fisher syndrome, 425

449

66485457-66485438 www.ketabpezeshki.com

ZZakaria_87574_PTR_Index_10-06-13_433-460.inddakaria_87574_PTR_Index_10-06-13_433-460.indd 449449 66/19/2013/19/2013 4:42:264:42:26 PMPM INDEX

milrinone myelosuppression, ganciclovir and, 187, 197 mechanism of action, 189, 198 myocardial infarction (MI) for regulating cardiac output, 189, 198 carotid stenting for, 11 for reversible cerebral vasosconstriction catheter-based reperfusion therapy for, syndrome, 427 221, 229–230 minute volume (MV), 269, 278 contraindications to fi brinolytic therapy miosis, 97, 101 for, 224, 232 MIP see maximal inspiratory pressure myoglobinemia, high-voltage burns and, missed bowel injury, septic from, 375, 380 376, 381 mitochondrial encephalopathy with lactic myonecrosis, 17 acidosis and stroke-like episodes myopathy (MELAS), 143, 150–151 cardiomyopathy, 27, 218, 227 diagnosis of, 148, 155 infl ammatory, 72, 75–76 mitral valve stenosis, 220, 228 ipecac-induced, 72, 75 modifi ed Rankin Score, 10 myxedema coma, 125, 127, 129 morphine, for breathing diffi culties, 385 myxopapillary, 137 Moyamoya disease, 16 right-middle cerebral artery, surgical NaCl infusion, for acute renal failure, 287, 293

bypass of, 4, 11–12 NaHCO3, for contrast-induced acute kidney MRI see magnetic resonance imaging injury, 288, 294 MS see multiple sclerosis naloxone (Narcan) multiple sclerosis (MS), 107 adverse effects of, 186, 196 corticosteroids for, 111, 116 for opiate overdose complications, 98, 101 McDonald diagnostic criteria for, 111, 116 naltrexone, for cholinergic poisoning, 104 rituximab for, 111, 116 NASCET see North American Symptomatic tumefactive, 111, 117 Carotid Endarterectomy Trial murmur, caused by infracted posterior NASCIS (National Acute Spinal Cord Injury papillary muscle, 233 Studies) trial, 43–44 muscular dystrophy nasogastric tube (NGT) suction Duchenne, 221, 229 overall volume depletion due to, 286, facioscapulohumeral, 221, 229 292–293 limb girdle, 221, 229 natalizumab (Tysabri) and succinylcholine-induced -associated progressive multifocal hyperkalemia, 72, 75–76 leukoencephalopathy, 116 MV see minute volume for relapsing remitting multiple myasthenia gravis (MG), 72 sclerosis, 110 acetylcholine esterase inhibitor National Conference of Commissioners on overdosing, impact of, 72, 76 Uniform State Laws, 361 anti-MUSK antibodies and, 72, 76 National Institutes of Health (NIH) high-dose initiation therapy for, 73, 76 Stroke Scale, 7, 8, 15 intravenous immunoglobulin for, 73, 76 nausea and vomiting, 385, 389–390 mechanical ventilation, indications NCPE see noncardiogenic pulmonary edema for, 73, 76 NCSE see nonconvulsive status epilepticus plasma exchange for, 73, 76 negative inspiratory force (NIF), 244, 256 myelogenous leukemia antibodies against negative predictive value, 393, 395 platelets, 334, 340 Neisseria meningitidis, 86

450

66485457-66485438 www.ketabpezeshki.com

ZZakaria_87574_PTR_Index_10-06-13_433-460.inddakaria_87574_PTR_Index_10-06-13_433-460.indd 450450 66/19/2013/19/2013 4:42:264:42:26 PMPM INDEX

neomycin and acute liver failure, 144, 152 NMO see neuromyelitis óptica nephrogenic diabetes insipidus and NMS see neuroleptic malignant syndrome hyponatremia, 289, 294–295 NOMI see nonocclusive mucosal ischemia neurocardiogenic (vasovagal) syncope, noncardiogenic pulmonary edema (NCPE), diagnosis of, 234 268, 274–275 neurocysticercosis, 81, 87 nonconvulsive focal right frontal-temporal neuroendocrine disorders, 121–129 seizure, electroencephalogram diabetes insipidus, 121, 126 for, 59, 67 neurogenic pulmonary edema (NPE), 22, nonconvulsive status epilepticus (NCSE), 28–29, 220, 228–229 421, 428 neuroleptic malignant syndrome (NMS), 100 in ICU setting, 62, 68 neuromuscular disease, succinylcholine for, nondepolarizing neuromuscular blockers, 189, 199 364, 369 neuromyelitis óptica (NMO), 114–115 noninferiority trial, 391, 394 neuron-specifi c enolase level, and cardiac noninvasive positive pressure ventilation arrest, 162, 171 (NIPPV), 270, 281 neuro-oncology, 131–139 nonocclusive mucosal ischemia (NOMI), 425 neurosarcoidosis, 113, 118 nonsteroidal anti-infl ammatory drugs neurotrauma, 35–47 (NSAIDs) NGT see nasogastric tube suction for gastrointestinal/genitourinary nicardipine disorders, 351, 357 for blood pressure, 221, 229 nonthyroidal illness syndrome, 302, 311 for controlling hypertyension in SAH, 21, 27 norepinephrine for hypertension during pregnancy, 143, 151 for elevating BP in cerebral vasospasm, 29 NIF see negative inspiratory force for patients presenting in septic shock, 408 nifedipine, for high-altitude pulmonary normal saline (NS) infusion edema, 231 for acute renal failure, 287, 293 NIH see National Institutes of Health for contrast-induced acute kidney injury, nimodipine 288, 294 for delayed cerebral ischemia, 175, 179 North American Symptomatic Carotid for reversible cerebral vasosconstriction Endarterectomy Trial (NASCET), 11 syndrome, 427 nosocomial urinary tract infection, for subarachnoid hemorrhage, 23, 30 Escherichia coli and, 188, 197 NIPPV see noninvasive positive pressure NPE see neurogenic pulmonary edema ventilation NS see normal saline infusion nitric oxide, for primary pulmonary NSAIDs see nonsteroidal anti-infl ammatory hypertension, 270, 280 drugs nitroglycerin, for blood pressure, 150 nitroprusside O-6-methylguanine methyltransferase for blood pressure, 150 (MGMT), 135 for hypertyension, 27 obsessive–compulsive disorder (OCD), 286 -induced ICP elevation, 163, 172 obstructive hydrocephalus, ICP monitoring NMDA see N-methyl-d-aspartate receptor for, 160 encephalitis OCD see obsessive–compulsive disorder N-methyl-d-aspartate (NMDA) receptor oculovestibular refl ex, evaluation of, 364, 368 encephalitis, 108, 114 ODS see osmotic demyelination syndrome

451

66485457-66485438 www.ketabpezeshki.com

ZZakaria_87574_PTR_Index_10-06-13_433-460.inddakaria_87574_PTR_Index_10-06-13_433-460.indd 451451 66/19/2013/19/2013 4:42:264:42:26 PMPM INDEX

Ogilvie’s syndrome, 351, 356–357 pancreatitis, 282 olanzapine and weight gain, 191, 201 alcoholic ON see optic neuritis fl uid resuscitation for, 353, 359 open depressed skull fracture, criteria for gastric decompression for, 353, 359 repairing, 36, 42 operative drainage for, 353, 359 operative abdominal decompression, for diagnosis of, 353, 359 compartment syndrome, 374, 379 pancuronium, side effects of, 190, 199

ophthalmoplegia, 13 PAO 2 (partial pressure of oxygen), 243, 254 opsoclonus-myoclonus and paraneoplastic paradoxic abdominal motion, during syndromes, 134, 139 spontaneous breathing, 244, 255 optic neuritis (ON), 114 paradoxical embolization, 220, 228 orbital cellulitis, 429 paraneoplastic syndromes (PNS) orbital wall fracture, 371, 377 opsoclonus-myoclonus and, 134, 139 oseltamivir, 320, 327 paraparesis, etiology of, 164, 172 osmolar (osmolal) gap, 307 parenteral nutrition (PN), 425 osmolar therapy, for vasogenic edema, 168 see also enteral nutrition osmotherapy Parinaud’s syndrome, 161, 169 for cerebral venous thrombosis, 13 partial seizures, complex, 4 for hemicraniectomy, 10 partial thromboplastin time (PTT) osmotic demyelination elevation and traumatic cerebral alcohol abuse and, 300, 308 contusions, 44 hypernatremia and, 300, 308 patent foramen ovale (PFO) closure, 9, 16 hyponatremia and, 300, 308 Patient Protection Affordable Care Act, 390 liver transplantation treated with PComm see posterior communicating artery tacrolimus and, 300, 308 aneurysm osmotic demyelination syndrome (ODS) PCP see phencyclidine intoxication comorbidities predispose to, 124, 129 PEEP see positive end-expiratory airway risk factors of, 125, 129 pressure osteoplastic craniotomy, for subdural pelvic fractures, binder placement hematoma, 42 for, 372, 378 oxygen pelvic trauma, extra-peritoneal rupture and, content, in blood, 242, 252–253 354, 360 content, of arterial blood, 206, 210–211 penicillins, for aseptic meningitis, 109 partial pressure saturation of, from pentobarbital, for intracranial pressure, 47 PAC, 206, 211 peptic ulcer consumption effects, on respiratory type I, 357 quotient, 403, 412 type II, 357 for high-altitude pulmonary edema, 231 type III, 357 normal partial pressure of, 206, 212 type IV, 357 oxygenation during mechanical ventilation, pericarditis, 224, 232 determinants of, 242, 253 ECG for, 221, 229 peri-mesencephalic subarachnoid hemorrhage, PAC see pulmonary arterial catheter cerebral angiography for, 30–31 pacemaker, for C5–C6 quadriplegic patient, periodic lateralized epileptiform discharges 416, 424 (PLEDs) palliative care, 385 in left temporal lobe, 56, 66

452

66485457-66485438 www.ketabpezeshki.com

ZZakaria_87574_PTR_Index_10-06-13_433-460.inddakaria_87574_PTR_Index_10-06-13_433-460.indd 452452 66/19/2013/19/2013 4:42:264:42:26 PMPM INDEX

perioperative neurosurgical care, 175–181 pleural fl uid effusion, 270, 279 peripheral vascular disease, 397 PML see progressive multifocal peripheral vascular injury, attention to leukoencephalopathy physical examination fi ndings for, PN see parenteral nutrition 372, 378 pneumobilia, 422, 428–429 PFO see patent foramen ovale closure pneumonia PFT see pulmonary function testing aspiration, 269, 274, 277, 282 phase I trial, 391, 394 community-acquired, 271, 282, 321, 328, 329 phencyclidine (PCP) intoxication, 99, 103 ventilator-associated, 324, 329 phenobarbital pneumonitis, chemical, 274 for seizure, 87 Pneumonia Severity Index, 328 for status epilepticus, 64 pneumothorax, 293 phentolamine, for adrenergic-induced PNS see paraneoplastic syndromes hypertension, 28 Poiseuille’s Law, 247, 260 phenylephrine, 189, 198 poliomyelitis, West Nile virus effect on β-receptor, 189, 198 infection and, 88 for elevating BP in cerebral vasospasm, polyethoxylated castor oil (Cremophor EL), 22, 29–30 for bleeding, 194 for intracranial hemorrhage, 188, 198 polyneuropathy, 17 for patients presenting in septic shock, 408 polyuria, 285, 292 phenytoin, 190, 200 cerebral salt wasting for, 289, 294 for juvenile absence epilepsy, 52, 64 following brain death, 303, 312 for left hemispheric cerebral infarction, portable hyperbaric chambers, for high- 51, 63 altitude pulmonary edema, 231 for seizure, 87 positive end-expiratory airway pressure for traumatic brain injury, 38, 45 (PEEP) pheochromocytoma, 122, 126 auto-, 246, 258–259 physostigmine, for cholinergic poisoning, 104 functional residual capacity increased by, piperacillin–tazobactam, for community- 248, 262 acquired pneumonia, 321, 328, 329 positive IgG for aquaporin 4 antibody pituitary apoplexy, 133, 136 testing, 115 plasma exchange, for myasthenia positive predictive value, 395 gravis, 73, 76 positive pressure ventilation (PPV), 270, plasma metanephrines, for von 280–281 Hippel–Lindau disease, 122, 126 decreased preload, 280 Plasmodium, 86 detrimental effects of, 281 Plasmodium falciparum, 86 noninvasive, 270, 281 Plasmodium malariae, 86 reduced afterload, 281 Plasmodium ovale, 86 postanoxic coma, 66 Plasmodium vivax, 86 posterior communicating artery (PComm) Plateau airway pressure, 247, 260 aneurysm, 24, 28 Plateau wave, 161, 169 posterior fossa subdural hematoma, 35, 41 platelet dysfunction, desmopressin posterior reversible encephalopathy for, 335, 340 syndrome (PRES) PLEDs see periodic lateralized epileptiform cyclosporine, 115 discharges magnetic resonance imaging for, 153–154

453

66485457-66485438 www.ketabpezeshki.com

ZZakaria_87574_PTR_Index_10-06-13_433-460.inddakaria_87574_PTR_Index_10-06-13_433-460.indd 453453 66/19/2013/19/2013 4:42:264:42:26 PMPM INDEX

posterior reversible encephalopathy progressive systemic sclerosis, 115 syndrome (PRES) (cont.) prophylactic antibiotics, for basilar skull risk factors of, 142, 150 fracture, 36, 42 tacrolimus for, 109, 115 prophylactic anticonvulsants, for traumatic posttraumatic CSF fi stulae brain injury, 38, 45 CSF otorrhea for, 37, 43 prophylactic hypothermia, in traumatic CSF rhinorrhea for, 37, 43 brain injury, 38, 44–45 posttraumatic seizures prophylactic seizure therapy and development, risk factors for, 39, 45 intracerebral hemorrhage, 12 in TBI patients, 45 propofol postural headache, following lumbar adverse effects of, 186, 196 puncture, 79, 84 as emulsion, 186, 196 potassium translocation from intracellular for status epilepticus, 52, 64 space into serum, 300, 308 propranolol PPH see primary pulmonary hypertension for hyperthyroidism, 123, 127 PPV see positive pressure ventilation for subdural hematoma, 303, 313 pralidoxime, for cholinergic propylthiouracil, for hyperthyroidism, poisoning, 99, 103 123, 127 predictive value prostacyclin, for primary pulmonary negative, 393, 395 hypertension, 270, 280 positive, 395 protamine dose, to neutralize prednisone, for myasthenia gravis, 73, 76 heparin, 185, 194 preeclampsia protamine sulfate, for heparin headache and, 142, 149 overdose, 338, 344 magnesium sulfate for, 145, 152 prothrombin complex concentrate, for see also eclampsia intracerebral hemorrhage, 343 preload, 209–210 pseudomonal infections, meropenem for, premature ventricular complex (PVC), 227 326, 330 preoxygenation, 401, 409 Pseudomonas aeruginosa, 326 PRES see posterior reversible pseudovasculitis, 16 encephalopathy syndrome ptosis, 24 pressure-limited assist/control ventilation, PTT see partial thromboplastin time 248, 261 pulmonary angiography, for pulmonary pressure-limited mechanical ventilation, embolism, 333, 339 246, 258 pulmonary arterial catheter (PAC), 399, 406 primary pulmonary hypertension (PPH) pulmonary artery wedge pressure, 207, calcium channel blockers for, 270, 280 212–213 nitric oxide for, 270, 280 pulmonary compliance, 240, 249–250 prostacyclin for, 270, 280 dynamic, 243, 253 PROACT II trial, 10 static, 243, 253 progressive multifocal leukoencephalopathy pulmonary conductance, 240, 243, 249, 253 (PML), 88 pulmonary contusion and hypoxia, 373, 378 natalizumab-associated, 116 pulmonary edema pathology of, 110, 116 defi ned, 274 progressive spondylotic cervical hydrostatic, 275 myelopathy, surgery for, 176, 179–180 noncardiogenic, 268, 274–275

454

66485457-66485438 www.ketabpezeshki.com

ZZakaria_87574_PTR_Index_10-06-13_433-460.inddakaria_87574_PTR_Index_10-06-13_433-460.indd 454454 66/19/2013/19/2013 4:42:264:42:26 PMPM INDEX

pulmonary elastance, 243, 250, 253 red lens test, 14 pulmonary embolism, pulmonary refl ectance spectrophotometry, 397 angiography for, 333, 339 refl ex airway closure, 274 pulmonary function testing (PFT), 245, regional wall motion abnormalities 256–257 (RWMA), 27 pulmonary vascular resistance, 241–243, relapsing remitting multiple sclerosis 251–252 (RRMS), 112 pulse oximeter, accuracy of, at lowest blood CSF examination for, 110, 116 pressure, 398, 405 natalizumab for, 110 pulsus paradoxus and cardiac tamponade, remote cerebellar hemorrhage (RCH), 222, 230 178, 181 pupils position, during brain death, 364, 368 renal diseases, 285–296 p value, 393, 395 renal insuffi ciency and osmotic PVC see premature ventricular complex demyelination syndrome, 124, 129 pyrazinamide, for tuberculous meningitis, 91 residual volume (RV), 269, 278, 279 respect (for moral and religious values), quadrantanopsia, 28 383, 387 Quantiferon-TB Gold test, 320, 327 respiratory diseases, 267–283 quetiapine, for extrapyramidal symptoms, reticular activating system (RAS), 168 191, 201 reversible cerebral vasoconstriction quinine, for cerebral malaria, 80, 86–87 syndrome (RCVS), 16, 419, 427 rhabdomyolysis, aggressive fl uid repletion R01, 392, 395 for, 72, 75 radiation therapy Rickettsial disease, doxycycline for, 327 and hypopituitarism, 122, 127 rifampicin, for tuberculous meningitis, 91 for metastatic epidural spinal cord rifampin, for bacterial meningitis, compression, 133, 138 184, 193, 194 randomization process, 391, 394 right carotid artery stenosis, carotid randomized control study, 395 endarterectomy for, 178, 181 random oscillations from arm-restraining right facial sensory loss, 4 clinical technician, BP measurement right frontal subdural hemorrhage, variation by, 398 electroencephalogram for, 62, 68 rapid, shallow breathing index (f/VT), right-middle cerebral artery 245, 256 surgical bypass of, for Moyamoya disease, rapid diagnostic tests (RDTs), 86 4, 11–12 rapid infl uenza diagnostic test, from throat territory, delineated hypodensity in, 165, 173 swab, 320, 327 right upper extremity incoordination, 4 rapid sequence intubation (RSI), 268, 276–277 rituximab RAS see reticular activating system for multiple sclerosis, 111, 116 rationing care, 386, 390 for transverse myelitis, 115 RCH see remote cerebellar hemorrhage rocuronium and malignant hyperthermia, 102 RCVS see reversible cerebral RRMS see relapsing remitting multiple vasoconstriction syndrome sclerosis RDTs see rapid diagnostic tests RSI see rapid sequence intubation recombinant tissue plasminogen activator rt-PA see recombinant tissue plasminogen (rt-PA), 164, 172–173 activator

455

66485457-66485438 www.ketabpezeshki.com

ZZakaria_87574_PTR_Index_10-06-13_433-460.inddakaria_87574_PTR_Index_10-06-13_433-460.indd 455455 66/19/2013/19/2013 4:42:264:42:26 PMPM INDEX

RV see residual volume SjvO2 see jugular venous bulb saturation RWMA see regional wall motion SLE see systemic lupus erythematous abnormalities sleep-disordered breathing, heart failure and, 222, 230 SAH see subarachnoid hemorrhage SMA see superior mesenteric artery SAMMPRIS see Stenting and Aggressive small bowel obstruction (SBO), 351, 357 Medical Management for Preventing small brainstem lesion and consciousness Recurrent Stroke in Intracranial disturbance, 160, 168 Stenosis trial sodium bicarbonate, for TCA SBO see small bowel obstruction poisoning, 97, 101 SCAST trial, 14 sodium chloride, for intracranial scopolamine patch, for breathing diffi culties, pressure, 186, 196 385, 389 somatosensory evoked potentials (SSEPs), 17 SDH see subdural hematoma median nerve, 59, 67 second hour of hypotension, 321, 328 convulsive, 57, 66 selective serotonin reuptake inhibitors nonconvulsive, 62, 68, 421, 428 (SSRI) propofol for, 52, 64 and SIADH syndrome, 286 specifi city, 395 sensitivity, 392, 395 spectrophotometry, 404 sepsis, CVP-guided resuscitation for, 322, 328 SPIKES protocol, 390 serotonin syndrome, 95, 100 spinal cord infarction, 9, 16–17 serum calcium, 299, 306 spinal epidural abscess, Staphylococcus aureus serum mannitol testing, 307 infection and, 82, 88–89 serum osmolality, 300, 307 spinal tap, 19 serum transaminases, 350, 356 spindle coma, 54, 65 serum tumor markers, 353, 360 spontaneous breathing trials, use of, 245, 256 Sheenan’s syndrome, hydrocortisone for, SSEPs see somatosensory evoked potentials 301, 310–311 SSRI see selective serotonin reuptake shivering and hypothermia, 162, 170 inhibitors short QT interval, 21, 26–27 Staphylococcus aureus infection and spinal shunt equation, 213 epidural abscess, 82, 88–89 SIADH see syndrome of inappropriate β-stimulating catecholamines, for antidiuretic hormone adrenergic-induced hypertension, 28 sickle cell disease Stenting and Aggressive Medical -induced hypoxia, 290, 295–296 Management for Preventing transcranial doppler monitoring for, 5, 12 Recurrent Stroke in Intracranial sigmoid diverticulosis, 351, 357 Stenosis (SAMMPRIS) trial, 11 sigmoid volvulus, sigmoidoscopy for, stereotactic aspiration, for Alzheimer’s 352, 358 dementia, 83 silver sulfadiazine (Silvadene), for burns, 380 steroid-induced complications, associated sinusitis with external ventricular drainage, endotracheal tube early transfer to oral 177, 180 route and, 323, 329 STICH see Surgical Trial in Intracerebral maxillary, 323, 329 Haemorrhage trial SIRS see systemic infl ammatory response STOP trial, 12 syndrome Streptococcus agalactiae, 86

456

66485457-66485438 www.ketabpezeshki.com

ZZakaria_87574_PTR_Index_10-06-13_433-460.inddakaria_87574_PTR_Index_10-06-13_433-460.indd 456456 66/19/2013/19/2013 4:42:264:42:26 PMPM INDEX

Streptococcus pneumonia-induced bacterial suboccipital craniectomy, for infratentorial meningitis, dexamethasone for, 183, bleeds, 12 192–193 substituted judgment, 384, 388 streptomycin, for tuberculous meningitis, 91 succinylcholine “string of pearls” sign, 351 -induced hyperkalemia stroke critical illness neuropathy and, 72, cryptogenic, 16 75–76 embolic, 80, 85–86 infl ammatory myopathy and, 72, 75–76 hemorrhagic, 80, 85–86 muscular dystrophy and, 72, 75–76 ischemic, 162, 171, 222, 230 and malignant hyperthermia, 102 patients, blood pressure management in, for neuromuscular disease, 189, 199 7, 14–15 sudden severe throbbing headache and and subarachnoid hemorrhage, 20 eclampsia, 141, 149 thrombotic, 246, 257 superior mesenteric artery (SMA) stunned neurogenic myocardium, 26–27 occlusion of, 352, 357–358 subarachnoid hemorrhage (SAH), 19–31 superior venae cavae, blood streaming from, with altered mental status, ICP 207, 212 monitoring for, 160 surface cooling, 8, 15 aneurysmal, 416, 424 surgery cerebral angiography for, 23, 30–31 for ophthalmoplegia, 132, 136 clinical change after clinical drain for progressive spondylotic cervical placement, 20, 25 myelopathy, 176, 179–180 continuous electroencephalography Surgical Trial in Intracerebral Haemorrhage monitoring for, 23, 30 (STICH) trial, 12 CT scan for, 19, 24 SVR see systemic vascular resistance ECG changes associated with, 219, SVRI see systemic vascular resistance index 227–228 syndrome of inappropriate antidiuretic Fisher Grade 3, 401, 409 hormone (SIADH), 22, 29, 74, 77 and hyperreninemic hypoaldosteronism distinguished from cerebral salt wasting syndrome, 305, 315 syndrome, 123, 128 hyponatremia and, 123, 128 and hyponatremia, 123, 128 initial clinical grade of, 23, 30 SSRI inhibitors and, 286, 292 nimodipine for, 23, 30 syndrome of the trephined, 40, 46–47 peri-mesencephalic, 30–31 systemic infl ammatory response syndrome stroke and, 20 (SIRS), 320, 327 subdural hematoma (SDH) systemic lupus erythematous (SLE) acute, 36, 42 CNS manifestations of, 113, 118 in older patients, surgery for, 36, 42 dexamethasone for, 303, 313 posterior fossa, 35, 41 systemic vascular resistance (SVR), propranolol for, 303, 313 378, 400, 408 right frontal, 62, 68 systemic vascular resistance index valproic acid for, 190, 200 (SVRI), 209 subependymoma, 137 subfalcine herniation, 167 tacrolimus subluxation with spinal cord transaction, for liver transplantation treated with cerebrovascular injury, 37, 43 and central pontine, 300, 308

457

66485457-66485438 www.ketabpezeshki.com

ZZakaria_87574_PTR_Index_10-06-13_433-460.inddakaria_87574_PTR_Index_10-06-13_433-460.indd 457457 66/19/2013/19/2013 4:42:264:42:26 PMPM INDEX

tacrolimus (cont.) thyroid hormone therapy, for myxedema and extrapontine myelinolysis coma, 129 syndromes, 300, 308 thyroid storm, 123, 127 and osmotic demyelination, 300, 308 thyrotoxicosis, 313 for posterior reversible encephalopathy amiodarone and, 127 syndrome, 109, 115 type 1, 127 Takotsubo cardiomyopathy, 27 type 2, 127 TB see tuberculosis skin test TIA see transient ischemia attack TBI see traumatic brain injury tick-borne infectious diseases, doxycycline TCA see tricyclic antidepressant for, 327 TCD see transcranial doppler ultrasound tidal volume (TV), 269, 278 technetium-labeled (Tc99m-HMPAO) tissue plasminogen activator (t-PA), 419 radionuclide sintigraphy, for brain intraventricular, for intraventricular death, 363, 368, 369 hemorrhage, 13 temozolomide, for grade IV glioblastoma, 135 for left hemispheric cerebral TENS see transcutaneous electrical nerve infarction, 63 stimulation TLC see total lung capacity terifl unomide, for lymphocyte reduction, 114 TM see transverse myelitis terminal delirium, 389 tolvaptan, for hyponatremia, 124, 129 terminal extubation, 385, 389 tonsillar herniation, 167 Terson’s syndrome, 24 total lung capacity (TLC), 269, 278, 279 theophylline, for C5–C6 quadriplegic total work of breathing, 269, 277 patient, 416, 424 toxic–metabolic disorders, 95–104 thermometer probe, placement of, 400, 407 t-PA see tissue plasminogen activator thiamine intake and Wernicke’s tracheal aspiration of secretions, 324, 329 encephalopathy, 147, 154 tracheostomy, advantages of, 270, 280 thiazide diuretics transcranial doppler (TCD) ultrasound plus angiotensin converting enzyme for cerebral vasospasm in middle cerebral inhibitors, 15 artery, 23, 31 plus angiotensin receptor blockers, 15 monitoring for sickle cell disease, 5, 12 plus beta-blockers, 15 transcutaneous electrical nerve stimulation for BP management in stroke patients, (TENS), 223, 231–232 14–15 transfuse packed red blood cells to target third nerve palsy with pupillary hematocrit, 322, 328 sparing, 22, 28 transfusion-related acute lung injury and thoracic central venous catheter hypoxia, 373, 378 oxygen saturation in blood, 208, 214 transient ischemia attack (TIA) thoracic corpectomy, for metastatic epidural carotid endarterectomy for, 3, 11 spinal cord compression, 133, 138 crescendo, 11 thrombocytopenia transurethral sphincterotomy, for bladder antibodies against platelets, 334, 340 dysfunction, 353, 359 thrombolysis, for stroke, 8, 15 transverse myelitis (TM), 114, 164, 172 thrombolytic therapy, contraindications to, acute partial (incomplete), 117 165, 173 trauma thrombotic stroke, respiratory support for, acute massive hemothorax following, 246, 257 374, 379

458

66485457-66485438 www.ketabpezeshki.com

ZZakaria_87574_PTR_Index_10-06-13_433-460.inddakaria_87574_PTR_Index_10-06-13_433-460.indd 458458 66/19/2013/19/2013 4:42:274:42:27 PMPM INDEX

with isolated live injury, nonoperative uremic twitch syndrome, 153 management for, 374, 379–380 urinary tract infections (UTIs) traumatic brain injury (TBI) cefepime for, 188, 197 decompressive hemicraniectomy for, 38, 45 ceftriaxone for, 188, 198 hormone defi ciencies after, 303, 312 Escherichia coli-induced nosocomial, ICP monitoring for, 160 188, 197 prophylactic anticonvulsants for, 38, 45 gentamycin for, 198 prophylactic hypothermia in, 38, 44–45 predisposing factors for, 188, 197 traumatic cerebral contusions, partial U.S. Department of Health and Human thromboplastin time elevation and, 44 Services, 395 traumatic injuries to spine and extremities, UTIs see urinary tract infections high-voltage burns and, 376, 381 Trendelenburg positioning, 399, 406–407 vaccines, for bacterial meningitis, 81, 88 tricuspid valve vegetation, 82, 89 valacyclovir, for acute renal failure, 187, 197 tricyclic antidepressant (TCA) valproate, for status epilepticus, 64 overdose and brainstem abnormalities, valproic acid 145, 153 and brain death, 362 poisoning, sodium bicarbonate for, 97, 101 overdose and brainstem abnormalities, trimethoprim–sulfamethoxazole 145, 153 for bacterial meningitis, 81 for subdural hematoma, 190, 200 for Listeria monocytogenes-induced valve replacement surgery delay, heart bacterial meningitis, 194 failure and, 83, 89 troponin T (cTnT), 222, 230–231 vancomycin Trousseau sign, 306 for Alzheimer’s dementia, 83 tuberculin skin test, 327 for bacterial meningitis, 121, 184, 193 tuberculosis (TB) skin test, 267, 273–274 for community-acquired pneumonia, 321, tuberculous meningitis, 83, 90–91 328, 329 tubular obstruction, 291, 295–296 for infectious endocartitis, 325, 330 tumefactive multiple sclerosis, 111, 117 for thrombocytopenia, 325, 330 TV see tidal volume vascular injury, angiography evaluation of type I error, 393, 395 neck for, 371, 377 type II error, 395 vascular malformations, 19–31 vasoconstriction UDDA see Uniform Determination of for Call–Fleming syndrome, 16 Death Act complete resolution of, 9 uncal herniation, 167 for reversible cerebral vasoconstriction unfractionated heparin syndrome, 16 for chronic stable atrial fi brillation vasodilation, for lap belt injuries, 173 poststroke, 185, 194 vasogenic edema, 160, 168 subcutaneous, for pulmonary vasopressin thromboembolism, 218, 227 effect on β-receptor, 189, 198 Uniform Determination of Death Act for elevating BP in cerebral vasospasm, 30 (UDDA), 361, 366 for lithium toxicity, 300, 307–308 upward tentorial herniation, 167 vasopressors, for mitral vegetation/ uremic encephalopathy, 143, 151 regurgitation, 325, 330 signs and symptoms of, 146, 153 VD/VT (dead space), 244, 255

459

66485457-66485438 www.ketabpezeshki.com

ZZakaria_87574_PTR_Index_10-06-13_433-460.inddakaria_87574_PTR_Index_10-06-13_433-460.indd 459459 66/19/2013/19/2013 4:42:274:42:27 PMPM INDEX

vegetative state of consciousness, 164, 172 von Hippel–Lindau disease, 122, 126 venous angiomas see developmental venous plasma metanephrines for, 122, 126 anomalies von Willebrand disease (vWD), 337, venous sinus thrombosis, magnetic 343–344 resonance venography for, 24 type I, 343 venous stenosis, 406 type II, 343 venous thromboembolism (VTE) type III, 343 inferior vena cava fi lter placement for, VT see ventricular tachycardia ablation 221, 230 VTE see venous thromboembolism malignant glioma and, 333, 339 vWD see von Willebrand disease ventilation:perfusion mismatch, 242, 253 ventilator-associated pneumonia Wallenberg syndrome, 14 antibiotic therapy for, duration of, 324, 329 warfarin diagnosis of, 324, 329 plus amiodarone, 184, 193–194 ventilator mode, 271, 281 anticoagulant effect of, 184, 193 ventricular dysrhythmias and brain death, for bacterial meningitis, 184, 193 362, 367 -induced defi cit of factor VII, 164, 172 ventricular fi brillation, risk associated with, -induced skin necrosis, 341 98, 102 as procoagulant, 184, 193 ventricular ischemia for stroke, 8, 15 precordial leads, positioning of, 398, 406 Warfarin–Aspirin Symptomatic Intracranial ventricular tachycardia (VT) ablation, Disease (WASID) trial, 11 225, 234 WASID see Warfarin–Aspirin Symptomatic ventriculostomy Intracranial Disease trial infections, 39, 46 Weber’s syndrome, 28 for infratentorial bleeds, 12 weight gain, olanzapine and, 191, 201 placement, complications associated Wernicke’s encephalopathy, thiamine intake with, 40, 47 and, 147, 154 verapamil, for reversible cerebral West Nile virus (WNV) infection, 80, 85 vasosconstriction syndrome, 427 and meningitis, 88 vertebral artery dissection, 7, 14 and poliomyelitis, 88 viral infection, 224, 232 West zone 1 abnormalities, 241, 252 vision loss refractory to IV steroids, 108 West zone 3 lung physiology, 269, 278 vital capacity forces, 269, 270, 278, 279–280 West zones, distribution of, 399, 407 vital capacity maneuver, 245, 256 WNV see West Nile virus infection vitamin K for acute liver failure, 145, 152 xanthochromia, 20 for bleeding, 185, 194 volume-limited mechanical ventilation, ziprasidone and QTc interval prolongation, 246, 257 191, 200

460

66485457-66485438 www.ketabpezeshki.com

ZZakaria_87574_PTR_Index_10-06-13_433-460.inddakaria_87574_PTR_Index_10-06-13_433-460.indd 460460 66/19/2013/19/2013 4:42:274:42:27 PMPM